introduction au management - ofppt maroc

48
Initiation au management © CRCF J. Sornet Page : 1 / 48 Centre de ressources comptabilité finance Lycée Emmanuel Mounier 6 avenue Marcelin Berthelot 38029 Grenoble CEDEX 2 INITIATION AU MANAGEMENT Jacques Sornet © CRCF Avertissement Ce support introduit très brièvement le management, ses défis contemporains, et il établit un lien avec d’autres parties du programme de management ou d’autres unités d’enseignement (notamment avec le contrôle de gestion). Il dépasse parfois l’introduction au management du référentiel de l’UE7 du DCG et apporte certaines ouvertures hors programme, il constitue donc une base adaptable par l’enseignant. Cette « initiation » n’aborde pas les aspects humains du management, fondamentaux, car ils font par ailleurs l’objet de développements spécifiques dans le programme de l’épreuve. Enfin, l’entreprise est souvent prise en exemple pour la commodité, ou pour éviter toute confusion entre l’organisation en tant qu’entité ou en tant que structure. Il est donc utile de rappeler que le programme de management du DCG concerne très généralement toutes les formes d’organisation. SOMMAIRE INTRODUCTION AU MANAGEMENT LE MANAGEMENT EN PRATIQUE ORGANISATION ET PROCESSUS DEFIS ET TENDANCES DU MANAGEMENT

Upload: others

Post on 02-Mar-2022

18 views

Category:

Documents


0 download

TRANSCRIPT

Initiation au management copy CRCF ndash J Sornet Page 1 48

Centre de ressources comptabiliteacute finance

Lyceacutee Emmanuel Mounier

6 avenue Marcelin Berthelot ndash 38029 Grenoble CEDEX 2

INITIATION AU MANAGEMENT

Jacques Sornet copy CRCF

Avertissement

Ce support introduit tregraves briegravevement le management ses deacutefis contemporains et il eacutetablit un

lien avec drsquoautres parties du programme de management ou drsquoautres uniteacutes

drsquoenseignement (notamment avec le controcircle de gestion) Il deacutepasse parfois lrsquointroduction

au management du reacutefeacuterentiel de lrsquoUE7 du DCG et apporte certaines ouvertures hors

programme il constitue donc une base adaptable par lrsquoenseignant

Cette laquo initiation raquo nrsquoaborde pas les aspects humains du management fondamentaux car ils

font par ailleurs lrsquoobjet de deacuteveloppements speacutecifiques dans le programme de lrsquoeacutepreuve

Enfin lrsquoentreprise est souvent prise en exemple pour la commoditeacute ou pour eacuteviter toute

confusion entre lrsquoorganisation en tant qursquoentiteacute ou en tant que structure Il est donc utile de

rappeler que le programme de management du DCG concerne tregraves geacuteneacuteralement toutes

les formes drsquoorganisation

SOMMAIRE

INTRODUCTION AU MANAGEMENT

LE MANAGEMENT EN PRATIQUE

ORGANISATION ET PROCESSUS

DEFIS ET TENDANCES DU MANAGEMENT

Initiation au management copy CRCF ndash J Sornet Page 2 48

INTRODUCTION AU MANAGEMENT

Le management concerne tout groupe organiseacute en fonction drsquoun but Nous prendrons

toutefois le plus souvent lrsquoentreprise comme reacutefeacuterence courante sachant que les theacuteories et

les techniques preacutesenteacutees sont transposables agrave drsquoautres situations

1 ndash Les contextes du management

11 ndash Lrsquoorganisation

Une organisation est un ensemble de moyens structureacute en fonction drsquoobjectifs qui regroupe

notamment des personnes Les organisations sont multiforme elles peuvent ecirctre priveacutees

publiques vendre des produits des services ou ecirctre financeacutees par des cotisations ou lrsquoimpocirct

avoir ou non un but lucratif hellip

Lrsquoentreprise est une organisation particuliegravere qui vise la reacutealisation et le partage de beacuteneacutefices

(contrairement aux administrations publiques et aux associations sans but lucratif) Elle creacutee

une valeur ajouteacutee (diffeacuterence entre la valeur de la production et les consommations drsquoune

peacuteriode) qui est reacutepartie entre les salarieacutes les institutions financiegraveres les actionnaires lrsquoeacutetat

hellip)

Diriger piloter une entreprise consiste agrave deacutevelopper (au moins agrave conserver) sa valeur

eacuteconomique en tenant compte des contraintes et des opportuniteacutes internes (eacutequipements

disponibles compeacutetences hellip) et de celles de son environnement

Le terme socieacuteteacute correspond agrave une forme juridique de lrsquoentreprise (socieacuteteacute anonyme SARL

hellip)

Le terme firme drsquoorigine anglo-saxonne deacutesigne la place et le fonctionnement de

lrsquoentreprise sur le marcheacute dans les theacuteories eacuteconomiques notamment dans les theacuteories de la

firme deacuteveloppeacutees depuis les anneacutees 30 et qui connaissent de nouveaux deacuteveloppements

depuis une vingtaine drsquoanneacutees

12 ndash Lrsquoentreprise et son environnement

On distingue le microenvironnement de lrsquoentreprise (fournisseurs clients collectiviteacutes

concurrents banques hellip) et son macro environnement (eacuteconomique social juridique

politique hellip)

Lrsquoenvironnement est complexe changeant et il a des effets multiples sur lrsquoentreprise Le

marcheacute (client et concurrents) a un effet primordial car il conditionne la capaciteacute agrave vendre

des produits et donc agrave survivre

ENTREPRISE

Marcheacute (clients Marcheacute des capitaux Marcheacute du travail

concurrents)

Politique

Lois et regraveglements

Ecologie

Conditions

eacuteconomiques

geacuteneacuterales Facteurs socio

culturels modes

Fournisseurs Technologie Deacutemographie

partenaires

Urbanisme geacuteographie

ENVIRONNEMENT

Initiation au management copy CRCF ndash J Sornet Page 3 48

Exemple lrsquoeacutecologie (la preacuteoccupation environnementale) peut influencer lrsquoactiviteacute de

lrsquoentreprise en modifiant les coucircts (transports eacutelimination des deacutechets

perfectionnement des installations matiegraveres hellip) en imposant des regravegles (normes de

pollution emballages hellip) en induisant une pression commerciale (image de

lrsquoentreprise preacutesentation des produits) etc

2 ndash Le management

21 ndash Deacutefinition du management

Le management est une activiteacute dont la finaliteacute est de conduire une organisation vers son but

(reacutealiser des profits se deacutevelopper assurer un service public hellip) par la reacutealisation drsquoobjectifs

preacutedeacutefinis

Le management est lieacute au travail collectif (laquo ecirctre efficaces agrave plusieurs travailler avec les

autres faire travailler les autres raquo) et son importance grandit avec la taille de lrsquoorganisation

Il trouve (au terme pregraves) ses origines dans les premiers grands chantiers de lrsquoEgypte ancienne

et son vocabulaire emprunte aux premiegraveres grandes organisations occidentales que sont les

armeacutees (cadre strateacutegie tactique hellip)

Le terme laquo manager raquo deacutesigne ceux qui ont une responsabiliteacute drsquoencadrement (en France

cette notion ne se confond pas toujours avec celle de cadre qui correspond agrave un statut

mais pas obligatoirement agrave une fonction)

22 ndash Les apports du management

Diriger consiste agrave fixer des objectifs en effectuant des choix Geacuterer crsquoest organiser et

optimiser les moyens disponibles pour atteindre les objectifs

Le terme laquo management raquo qui tend agrave remplacer la laquo gestion raquo ou la laquo direction raquo dans le

vocabulaire eacuteconomique franccedilais englobe ces deux notions Cette eacutevolution terminologique

accompagne une eacutevolution dans la faccedilon de conduire les organisations

laquo Manager raquo suppose

UN CERTAIN REALISME

Le management repose sur le pragmatisme propre aux pays anglo-saxons ougrave

laquo management raquo et laquo manager raquo deacutesignent depuis longtemps ce ou ceux qui encadrent la

marche de lrsquoentreprise

UNE GRANDE REACTIVITE

Le manager a la capaciteacute drsquoagir en fonction des circonstances pour maicirctriser rapidement

des situations complexes Le management se distingue ainsi

- de lrsquo laquo administration raquo au sens courant du terme que lrsquoon retrouve par exemple dans

conseil drsquoadministration ou administration de lrsquoEtat qui eacutevoque plutocirct une action laquo haut

placeacutee raquo assez eacuteloigneacutee du quotidien (bien que lrsquoadministration drsquoentreprise inaugureacutee par

Fayol soit eacutequivalente au management ndash Voir les MBA initialiseacutes agrave Harvard et lrsquoIAE en France)

- de la laquo gestion raquo qui renvoie communeacutement agrave un ensemble de techniques (gestion

financiegravere gestion comptable gestion des ventes gestion de la treacutesorerie hellip)

UNE ACTION SUR LES HOMMES

De nombreux auteurs ont souligneacute cet aspect du management En particulier

Initiation au management copy CRCF ndash J Sornet Page 4 48

- Peter Drucker pour qui le management repose sur cinq principes fixer des objectifs

organiser le travail motiver et communiquer former les eacutequipes (au sens de lrsquoapprentissage

laquo former les autres et soi-mecircme raquo)

- Henry Mintzberg qui met lrsquoaccent sur le rocircle unificateur du manager laquo Ce qui distingue

avant tout une organisation formelle drsquoun quelconque rassemblement drsquohommes ndash drsquoune

foule drsquoun groupe informel ndash crsquoest la preacutesence drsquoun systegraveme drsquoautoriteacute et drsquoadministration

personnifieacute par un ou plusieurs managers dans une hieacuterarchie plus ou moins structureacutee et

dont la tacircche est drsquounir les efforts de tous dans un but donneacute raquo

On en deacuteduit le rocircle relationnel du manager et le poids de la gestion des ressources

humaines dans son activiteacute

LA PRIMAUTE DE LrsquoINFORMATION

Le management fixe des objectifs il a un rocircle deacutecisionnel et il creacutee les conditions neacutecessaires

pour atteindre les objectifs dont il controcircle la reacutealisation Tout ceci neacutecessite la mise en place

drsquoun systegraveme drsquoinformation fournissant les donneacutees neacutecessaires agrave des choix pertinents

(donneacutees qui remontent souvent par les managers eux-mecircmes) puis agrave la mise en œuvre et

au controcircle des reacutealisations

23 ndash Les dimensions du management

La fonction de management a de fait trois dimensions humaine (faire travailler des

personnes ensemble motiver) eacuteconomique (fonctionnement ou deacuteveloppement de

lrsquoorganisation aux meilleurs conditions) et informationnelle (dont la communication)

Le management est une activiteacute de synthegravese qui neacutecessite des capaciteacutes agrave traiter et

combiner des informations drsquoorigines diverses plus ou moins varieacutees selon le contexte

(financiegravere technique leacutegale eacuteconomique politique hellip) et agrave agir en conseacutequence dans les

trois dimensions

Humain

Economique

Information

Informations sur

lrsquoenvironnement

Informations sur

lrsquoorganisation

Management

Information

dirigeant lrsquoaction

des autres

Actions directes

du manager

Initiation au management copy CRCF ndash J Sornet Page 5 48

24 ndash La recherche de performance

Le management recherche la performance de lrsquoorganisation en rapport avec ses objectifs

qui peuvent ecirctre de diffeacuterentes natures (expansion profit hellip ou agrave un niveau plus deacutetailleacute

reacuteduction des deacutelais flexibiliteacute reacuteduction des coucircts accroissement de la qualiteacute hellip)

La performance peut ecirctre abordeacutee de deux faccedilons

- par lrsquoefficaciteacute qui conduit agrave la reacutealisation des objectifs

- par lrsquoefficience qui conduit agrave une utilisation optimale des moyens disponibles avec le

meilleur rendement

Exemple une entreprise qui atteint son objectif de croissance de 10 du chiffre

drsquoaffaires et de son beacuteneacutefice est efficace mais celle qui arrive au mecircme reacutesultat avec

moins drsquoactifs et moins de personnel est plus efficiente

25 ndash Le champ drsquoaction du management

Le management est mis en œuvre dans des peacuterimegravetres drsquoampleur variable un groupe une

entreprise une activiteacute particuliegravere de lrsquoentreprise un projet la tenue drsquoun magasin hellip Les

actions de management ont ainsi des conseacutequences plus ougrave moins importantes

Le management srsquoexerce agrave tous les niveaux drsquoencadrement de lrsquoorganisation Il est de

coutume de distinguer trois niveaux

Exemple

Management strateacutegique la direction geacuteneacuterale a fixeacute les objectifs de chaque filiale du

groupe en tenant compte des positions souhaiteacutees sur le marcheacute Il en reacutesulte que les

uniteacutes X et Y aux activiteacutes similaires et compleacutementaires doivent preacuteparer leur fusion

preacutevue dans les deux ans La filiale Z situeacutee dans une ville universitaire doit acqueacuterir des

compeacutetences pour innover dans lrsquoapplication des supra conducteurs drsquoici trois ans Les

budgets et les financements correspondants ont eacuteteacute globalement estimeacutes

Management intermeacutediaire en application de ces orientations strateacutegiques les

directions de X et Y planifient des reacuteunions de travail communes pour voir comment

harmoniser leurs ressources humaines dans les douze mois Z deacutecide de contractualiser

ses relations avec le centre universitaire pour atteindre ses objectifs et mettre en place

un partenariat en recherche appliqueacutee

Management strateacutegique ou geacuteneacuteral

(direction laquo top management raquo deacutefinition

des objectifs geacuteneacuteraux)

Management tactique

(intermeacutediaire laquo middle

managers raquo)

Management opeacuterationnel (de

terrain ou drsquouniteacute local

laquo executive manager raquo)

Initiation au management copy CRCF ndash J Sornet Page 6 48

Management opeacuterationnel X et Y mettent en place leur communication avec le

personnel et un plan de reconversion Z nomme un directeur de recherche qui prend

notamment en charge les relations avec lrsquouniversiteacute

Le management recouvre la totaliteacute des actes de conduite de lrsquoorganisation dans tous les

domaines (technique commercial financier hellip) mais il ne correspond agrave aucune cellule de

lrsquoorganigramme

Les actions de management sont par ailleurs contraintes par la disponibiliteacute des ressources

neacutecessaire pour atteindre les objectifs (financement savoir-faire profil du personnel

eacutequipements hellip)

3 ndash La meacutethode laquo management raquo

31 ndash La science du management

Le management nrsquoest pas une science exacte il srsquoapparente agrave une science humaine

expeacuterimentale qui traite de pheacutenomegravenes socio-eacuteconomiques eacutevolutifs et qui doit trouver

concregravetement son application dans la vie des organisations

Cette science traite notamment de lrsquoorganisation des entreprises et rassemble des meacutethodes

et des theacuteories qui peuvent ecirctre regroupeacutees en eacutecoles ou en courants

Les theacuteories marquent geacuteneacuteralement une eacutepoque et elles peuvent se recouper partiellement

parfois srsquoopposer avec des nuances qui doivent ecirctre bien identifieacutees Elles srsquoaccompagnent

souvent de modegraveles et de scheacutematisations qui en facilitent la compreacutehension et la

transposition agrave de nouvelles situations

Ces outils scientifiques guident le raisonnement permettent drsquoappreacutehender des reacutealiteacutes

complexes et structurent les connaissances ils sont peacutedagogiques et constituent des aides

pour lrsquoaction Leur application doit cependant ecirctre raisonneacutee car

- un modegravele nrsquoest qursquoune simplification de la reacutealiteacute

- une theacuteorie sortie de son contexte historique et eacuteconomique peut perdre de sa pertinence

Sa mise en œuvre doit tenir compte de la situation reacuteelle et une theacuteorie ne peut agrave elle seule

justifier une deacutecision de management (ce drsquoautant plus que chaque theacuteorie ne couvre que

tregraves partiellement le domaine du management ou de lrsquoorganisation)

- il est souvent difficile de disposer agrave temps drsquoinformations fiables et suffisantes pour appliquer

une theacuteorie dans les conditions ideacuteales Le manager est freacutequemment ameneacute agrave prendre ses

deacutecisions sur la base drsquoinformations incomplegravetes ou incertaines et il doit alors en mesurer les

risques et preacutevoir les ajustements neacutecessaires

ENTREPRISE

Management Administration

Gestion

Strateacutegique

Opeacuterationnel

Initiation au management copy CRCF ndash J Sornet Page 7 48

- des modes influencent le management Elles peuvent indiquer une veacuteritable eacutevolution

eacuteconomique mais aussi ecirctre sans lendemain voire introduire un danger ou un coucirct inutile

(se meacutefier des speacutecialistes dont le fond de commerce est la vente de nouvelles techniques

de management et des seacuteminaires associeacutes)

- seule la creacuteation drsquoune combinaison originale (dans le respect des regravegles) peut apporter un

avantage agrave lrsquoentreprise et non la reproduction de choix de management connus de tous

Exemple 1 la matrice de portefeuille drsquoactiviteacutes permet de classer les activiteacutes

strateacutegiques drsquoune entreprise en fonction de leur taux de croissance et de la part de

marcheacute deacutetenue (matrice laquo BCG raquo - Boston consulting group ndash 1975) Cet outil de

management neacutecessite de disposer drsquoinformations fiables concernant le marcheacute Il

conduit souvent agrave simplifier les conditions de concurrence et ignore la

compleacutementariteacute pouvant exister entre activiteacutes (synergie partage de techniciteacute

amortissement de charges fixes hellip) La matrice est un moyen de prendre conscience

du portefeuille et de ses eacuteventuels deacutefauts (portefeuille deacuteseacutequilibreacute avec

preacutedominance anormale drsquoune zone) mais il ne permet pas seul de deacutecider de lrsquoavenir

des activiteacutes

Exemple 2 le lancement drsquoun investissement lourd (lrsquoimplantation de nouvelles usines

le lancement drsquoune nouvelle activiteacute agrave fort taux de recherche ndash deacuteveloppement hellip) ne

peut ecirctre deacutecideacute que par le recoupement de diffeacuterentes approches (financiegravere

commerciale strateacutegique) Aucune theacuteorie du management appliqueacutee isoleacutement et

sans preacutecaution ne peut justifier un tel investissement

Exemple 3 une entreprise produisant des eacutequipements meacutedicaux deacutecide de renforcer

son offre commerciale par un service de conseil et drsquoassistance Cette deacutecision est

inspireacutee par le concept de creacuteation de valeur par une volonteacute de se diffeacuterencier de la

concurrence et par des pratiques existant depuis longtemps dans le domaine

informatique

Remarque parmi les facteurs influenccedilant les principes de management il est utile de

savoir deacutetecter pour srsquoen preacutemunir les ideacuteologies preacutejugeacutes et autres laquo valeurs raquo sans

rapport certain avec lrsquoefficaciteacute eacuteconomique

32 ndash Theacuteorie et pratique du management

La litteacuterature (ouvrages revues) aborde le management sous des angles diffeacuterents qui

peuvent suggeacuterer une concurrence entre des approches theacuteorique et pratique ou

psychologique et eacuteconomique de cette discipline

Taux de croissance

du domaine (cf

cycle de vie du

produit)

Part de marcheacute

(compareacutee au

principal

concurrent)

fort

faible

forte faible

Dilemmes (activiteacutes

en phase de

lancement

potentiel + coucircts+

risque+)

Vedettes

(activiteacutes en forte

croissance

autofinancement+)

Vaches agrave lait

(activiteacutes

stabiliseacutees et

concurrence faible

ou stable

rentabiliteacute+)

Poids morts

(activiteacutes en

deacuteclin)

Initiation au management copy CRCF ndash J Sornet Page 8 48

Le management a cependant une viseacutee unique et concregravete la bonne marche des

organisations et les travaux des universitaires rejoignent lrsquoaction des praticiens (les

universitaires reacutealisent drsquoailleurs bien souvent leurs recherches au sein des organisations)

Au plan peacutedagogique les deux visions du management se complegravetent

- la vision acadeacutemique met en perspective des concepts et des theacuteories Elle permet de

srsquoapproprier des raisonnements essentiels et stables notamment concernant la strateacutegie et

les reacuteactions humaines qui sont le moteur des organisations

- la vision pratique relate le veacutecu des praticiens preacutesente des techniques concregravetes et tente

parfois une vulgarisation favorisant la diffusion des concepts Elle integravegre de faccedilon

pragmatique les objectifs eacuteconomiques et les contraintes de fonctionnement des

organisations en se placcedilant parfois dans une vision agrave court terme

Sur le terrain le management reacutealise une synthegravese entre des techniques de psychologie

sociale et des techniques de gestion de diverses origines et de nouvelles meacutethodes

apparaissent aussi sous le label unique laquo management raquo

Le terme manageacuterial (approche manageacuteriale theacuteorie manageacuteriale pratique manageacuteriale

hellip) fait reacutefeacuterence agrave la vision aux preacuteoccupations et aux actions des managers qui doivent

emmener leur organisation vers la reacutealisation de ses objectifs

Exemples

- la deacutetermination des coucircts et des marges reacutesulte de techniques de gestion

indispensable aux deacutecisions des managers

- le laquo CRM raquo (customer relationship management en franccedilais GRC ndash gestion de la

relation client) est un concept reacutecent qui srsquoaccompagne de techniques lieacutee aux plus

reacutecents deacuteveloppements du management et des systegravemes drsquoinformation

- le laquo coaching raquo est une technique de management permettant une eacutevolution

personnelle dans le sens des objectifs de lrsquoentreprise

- le recentrage sur le laquo meacutetier raquo repose sur des techniques de management

Les techniques sont rassurantes mais pas suffisantes et si les theacuteories ne sont pas

indispensables au praticien elles aident agrave comprendre agrave anticiper et agrave bien utiliser les

techniques

La qualiteacute du management reacuteside beaucoup dans la capaciteacute agrave appliquer concepts et

techniques de faccedilon pertinente et agrave innover Cette capaciteacute relegraveve en partie drsquoun laquo art du

management raquo qui srsquoacquiert en grande partie par la pratique

4 ndash Management et expertise comptable

Lrsquoexpert comptable doit manager ses propres eacutequipes Il est par ailleurs supposeacute laquo hellip

conseiller et accompagner le chef drsquoentreprise dans toutes ses deacutecisions hellip raquo (selon lrsquoOEC)

Ce rocircle est particuliegraverement important dans ses relations avec les petites entreprises Il doit

donc avoir une capaciteacute au management

Organisation

Vision acadeacutemique Vision pratique

Initiation au management copy CRCF ndash J Sornet Page 9 48

Lrsquointervention de lrsquoexpert dans le management drsquoune entreprise peut toutefois poser

quelques problegravemes

- Il peut y avoir conflits drsquointeacuterecirct entre activiteacutes de certification des comptes et de conseil en

management (la tendance est agrave la seacuteparation des activiteacutes dans les plus grands cabinets)

- lrsquoactiviteacute de conseil neacutecessite des compeacutetences parfois tregraves speacutecifiques (conseil fiscal

conseil en RH conseil en systegravemes drsquoinformation hellip)

- le conseil est une activiteacute diffeacuterente par sa forme de lrsquoexpertise comptable (interventions

longues peu reacutepeacutetitives peu codifieacutees mises en concurrence) qui neacutecessite une

organisation particuliegravere du cabinet lorsqursquoelle deacutepasse lrsquointervention occasionnelle

APPLICATIONS IM

IM1 Analyser la profession de laquo manager raquo selon Henry Mintzberg (texte extrait de lrsquoouvrage

laquo Le management raquo Eyrolles - Editions drsquoOrganisation) et les principes du management de la

norme ISO

Faire ressortir les eacuteleacutements speacutecifiques agrave chacune de ces approches et mettre en eacutevidence

leurs points communs

Initiation au management copy CRCF ndash J Sornet Page 10 48

Principes du management drsquoapregraves la norme ISO 9001 (2000)

- Orientation vers le client (satisfaire ses attentes)

- Leadership (les dirigeants eacutetablissent les orientations de lrsquoorganisme Ils doivent creacuteer

un environnement interne ougrave les personnes peuvent clairement srsquoimpliquer dans la

reacutealisation des objectifs de lrsquoorganisme)

- Implication du personnel (les personnes sont agrave tout niveau lrsquoessence de lrsquoorganisme et

leur implication permet drsquoutiliser leurs aptitudes au profit de lrsquoorganisme)

- Approche laquo processus raquo (un reacutesultat est mieux atteint quand les ressources et les

activiteacutes neacutecessaires sont geacutereacutees comme un processus)

- Approche systegraveme (assimiler les processus correacuteleacutes agrave un systegraveme contribue agrave

lrsquoefficaciteacute et agrave lrsquoefficience de lrsquoorganisme vis-agrave-vis de ses objectifs)

- Ameacutelioration continue (objectif permanent de lrsquoorganisme)

- Prise de deacutecision efficace (par lrsquoanalyse de donneacutees et drsquoinformations)

- Relations mutuellement beacuteneacutefiques avec les fournisseurs (pour augmenter la capaciteacute

des deux organismes agrave creacuteer de la valeur)

IM2 Distinguer leader et manager

IM3 Compleacuteter le tableau ci-dessous en analysant chaque action preacutesenteacutee Faire ensuite

ressortir les domaines niveaux ou techniques de management pouvant ecirctre mobiliseacutes pour

chaque situation

Initiation au management copy CRCF ndash J Sornet Page 11 48

Caracteacuteristiques

de lrsquoaction

- reacutepeacutetition

- risque

- normes

- ampleur

Prise de

deacutecision

- opeacuterationnelle

strateacutegique

- deacutelai

Informations

neacutecessaires

- nature

- origine

- deacutelai obtention

Cleacutes pour la

reacuteussite

Intervention

exteacuterieure

possible

Assurer la

restauration du

soir

(restaurant

familial)

Construire un

viaduc

(autoroute)

Certifier les

comptes

annuels drsquoun

groupe

national

(cabinet

drsquoaudit)

Lancer une

ligne drsquoavions

(constructeur

aeacuteronautique)

Reacuteduire la

capaciteacute de

production

(groupe

industriel)

Acqueacuterir une

entreprise

concurrente

(teacuteleacutephonie

mobile)

Initiation au management copy CRCF ndash J Sornet Page 12 48

Fiche IM1 - Deacutefinitions du management

Dictionnaire anglais - franccedilais direction administration gestion intrigue manegravege

Wikipeacutedia Le management est lensemble des techniques dorganisation qui sont mises en

oeuvre pour ladministration dune entiteacute

Au point de vue eacutetymologique le verbe manage vient de litalien maneggiare (controcircler)

influenceacute par le mot franccedilais manegravege (faire tourner un cheval dans un manegravege) A cette

notion il faut aussi ajouter la notion de meacutenage (geacuterer les affaires du meacutenage) qui consiste agrave

geacuterer des ressources humaines et des moyens financiers

helliphellip

Fiche IM2 - Etudier le management

Concreacutetiser

Manager neacutecessite de syntheacutetiser des informations parfois complexes incomplegravetes et de

domaines tregraves divers pour en deacuteduire des actions Une approche trop parcellaire peut

conduire agrave lrsquoeacutechec et le savoir-faire est neacutecessaire pour agir vite avec un minimum de risque

Lrsquoeacutetudiant doit se preacuteparer simultaneacutement aux examens et agrave la pratique Il nrsquoa souvent connu

lrsquoentreprise que durant quelques semaines de stage et le manque de laquo recul raquo ne lui permet

pas toujours de concreacutetiser les theacuteories Il doit compenser par la lecture (ouvrages revues

journaux eacuteconomiques et boursiers) et en eacutetant attentif aux informations ambiantes (tout en

relativisant le style journalistique) en mettant en relation le cours les concepts les modegraveles

lrsquoactualiteacute les stages

Savoir traiter un exercice

Pour reacuteussir un examen ou traiter une application peacutedagogique (la conception les points 1 agrave

6 peut repreacutesenter le tiers du temps de travail)

1 ndash Identifier le type de sujet (faut-il trouver une solution pratique ou communiquer une

reacuteflexion geacuteneacuterale )

2 ndash Lire le sujet et relever les mots cleacutes

3 ndash Deacutefinir les mots cleacutes

4 ndash Reacutesumer la probleacutematique du sujet (en quelques lignes)

5 ndash Lister les connaissances reacutefeacuterences et raisonnements reacutepondant au problegraveme (par

recherche spontaneacutee ou raisonneacutee qui quoi ougrave quand comment combien hellip

listage des diffeacuterents points de vue) trouver des exemples (notamment dans les

documents fournis)

6 ndash Organiser la reacuteponse (deacutefinir le plan du deacuteveloppement ougrave des paragraphes bien

identifieacutes sont geacuteneacuteralement neacutecessaires en y liant les parties qui doivent ecirctre en nombre

limiteacute ndash de deux agrave quatre) Preacutevoir drsquoy inteacutegrer la deacutefinition des principales notions

induites par le sujet

7 ndash Reacutediger sous la forme adapteacutee (note technique ou recommandation solution

pratique exposeacute structureacute dissertation)

Introduction et conclusion sont indispensables agrave la dissertation ou agrave lrsquoexposeacute

- lrsquointroduction preacutesente le sujet traiteacute (phrase drsquoaccroche initiale) amorce la

probleacutematique (quelques sous - questions) et annonce le plan

- la conclusion syntheacutetise le deacuteveloppement (arguments) eacutelargit le sujet (prise de recul)

et apporte le point final (une phrase)

Une limite agrave la communication

Il est difficile de faire passer plus de 4 ou 5 ideacutees fortes dans un exposeacute unique

Initiation au management copy CRCF ndash J Sornet Page 13 48

Fiche IM3 - Bref historique

Antiquiteacute

3000 AJC

Peacuteriode greacuteco-

romaine

Transition

feacuteodale

12egraveme siegravecle

europe

15egraveme ndash 17egraveme

siegravecles

19egraveme siegravecle

20egraveme siegravecle

agriculture preacutedominante industrie limiteacutee aux besoins drsquoun individu ou drsquoun clan

pour la confection des outils des vecirctements et de la poterie Force motrice animale

ou humaine pour lrsquoessentiel

Grands travaux drsquoeacutetat en Egypte premiegravere laquo planification ndash organisation ndash controcircle raquo

Deacuteveloppement des communications essor industriel limiteacute peu de progregraves

technique (lrsquoesclavage supplante les innovations)

Deacuteveloppement progressif des eacutechanges commerciaux

La consommation indirecte atteint un bon niveau (surplus agricoles et

deacuteveloppement des villes) Apparition de nouveaux commerccedilants

Etat fort Evolutions technologiques (imprimerie bateaux performants instruments de

navigation) Extension geacuteographique de lrsquoeacuteconomie Apparition des corporations

drsquoartisans

Machine agrave vapeur chemin de fer passage de lrsquoartisanat au capitalisme

entrepreneurial producteur organisation des entreprises

Ecole classique (Taylor Fayol Weber) approche meacutecaniste bureaucratie

hieacuterarchie commandement fonctions et speacutecialisation laquo OS T raquo (organisation

scientifique du travail) organisation source de pouvoir rationaliteacute des individus bases

du management

Deacuteveloppement du capitalisme manageacuterial Electriciteacute peacutetrole puis communications

et information Consommation de masse mondialisation preacuteoccupations

eacutenergeacutetiques et environnementales 3 peacuteriodes

- standardisation grandes entreprises industrielles

- industries de consommation 30 glorieuses marketing multinationales protection

sociale

- deacutereacuteglementation monteacutee des services pays eacutemergents mondialisation et nouvelle

eacuteconomie (internet)

Ecole des relations humaines prise en compte de lrsquoindividu des motivations styles

de direction

Ecole neacuteo-classique et post-classique deacutecentralisation coordonneacutee DPO

management participatif zeacutero deacutefaut flux tendus

Approche systeacutemique partition de lrsquoentreprise eacutetude des interactions feacutedeacuteration

vers lrsquoobjectif controcircle et ajustement

Theacuteories de la deacutecision rationaliteacute limiteacutee contribution reacutetribution coalitions

Ecole socio-technique recherche de compromis technologie organisation

enrichissement des tacircches autonomie des groupes

Approche sociologique effets sociaux du travail jeux de pouvoir dans lrsquoentreprise

reacutegulation sociale

Theacuteories de la contingence facteurs contingents adaptation agrave lrsquoenvironnement

configurations organisationnelles

Theacuteories de la firme controcircle manageacuterial droits de proprieacuteteacute relation drsquoagence

Theacuteories contractualistes firme nœud de contrats coucircts de transaction

opportunisme externalisation internalisation

Approche eacutevolutioniste eacutecologie des organisations modegravele eacutevolutioniste

contraintes de sentier

Approche par les ressources valorisation des ressources compeacutetences cleacutes

apprentissage organisationnel

(Classement simplifieacute)

Initiation au management copy CRCF ndash J Sornet Page 14 48

ELEMENTS DE CORRIGE IM

IM1 Commenter la deacutefinition du management par la norme ISO et le manager de Mintzberg

Efficient = optimum avec les moyens disponibles

ISO (management objectifs) (manager moyens) HM

IM2 Le leader entraicircne naturellement derriegravere lui Le manager nrsquoest pas toujours leader

(mecircme si crsquoest souhaitable) Le leader nrsquoest pas toujours manager (plutocirct notion individuelle)

Leadership = faculteacute de diriger conjugaison drsquoune autoriteacute naturelle ou drsquoun savoir-faire

acquis drsquoune capaciteacute agrave entraicircner des personnes ou des groupes et drsquoune leacutegitimiteacute

statutaire (de position)

IM3 Compleacuteter le tableau ci-dessous en analysant chaque action preacutesenteacutee Faire ensuite

ressortir les domaines niveaux ou techniques de management pouvant ecirctre mobiliseacutes pour

chaque situation

Satisfaction client

Implication du personnel

Processus systegraveme

Ameacutelioration continue

Deacutecision efficace

Recherche de valeur

Image entreprise

Liaisons

Information

Reacutepartition ressources

Reacutegulation

Neacutegociation

Leadership

Initiation au management copy CRCF ndash J Sornet Page 15 48

Caracteacuteristiques

de lrsquoaction

- reacutepeacutetition

- risque

- normes

- ampleur

Prise de

deacutecision

- opeacuteration

- direction

- deacutelai

Informations

neacutecessaires

- nature

- origine

- deacutelai

obtention

Cleacute pour la

reacuteussite

Intervention

exteacuterieure

possible

Assurer la

restauration du

soir

(Restaurant

familial)

Technique

(fabrication)

Vente (terrain)

Appros

Reacutepeacutetitive

(quot)

Risque faible

Normes

drsquohygiegravene

Faible

Opeacuterationnelle

Geacuterant

responsable

Rapide (qq

jours menu et

appros)

Nombre de

couverts

Tarifs usuels

Calendrier

(fecirctes)

Clients docs

divers

expeacuterience

Qq jours

Varieacuteteacute menu

Plats phares

Accueil

Appros

Tarification

Vins

Gestion

congeacutelation

Qualiteacute cuisine

Fournisseurs

Extra

Publiciteacute

Construire un

viaduc

(autoroute)

Technique

Organisation

Appros

Uniteacute (ou peu)

Eleveacute (financier

technique)

Architecture

Eleveacutee

Direction

(aleacuteas)

Opeacuterationnelle

(conduite

chantier)

Immeacutediat agrave qq

semaines

Plans

plannings

Qualifications

Meacuteteacuteo

Disponibiliteacutes

Bureau eacutetudes

Qq sem agrave 24h

Techniciteacute

Appros

Qualifications

Preacutevision

GRH

Contrat juste

SS traitants

Organismes

certificateurs

Controcircle

client

Certifier les

comptes

annuels drsquoun

groupe national

(cabinet

drsquoaudit)

Technique

Relation client

Gestion des

connaissances

Annuelle

Moyen

Regravegles

comptables

fiscales

Moyenne (selon

importance du

cabinet)

Opeacuterationnelle

Qq jours agrave

semaines

Comptable

Juridique

Client

Etat

Qq jours agrave

semaines

Techniciteacute

Expeacuterience

Relation client

Systegraveme info client

Siegravege

Autre cabinet

Lancer une

ligne drsquoavions

(constructeur

aeacuteronautique)

Strateacutegique

RD

Etudes

Uniteacute

Tregraves eacuteleveacute

Aeacuteronautique

Tregraves eacuteleveacutee

Direction

Qq mois agrave

anneacutees

Marcheacute

Etudes

Compagnies

Qq mois agrave

anneacutees

Concept

Outil industriel

Coucirct exploitation

Tarif

Fiabiliteacute

Deacutelaisconcurrence

SI simulation

SS traitants

Bureaux

drsquoeacutetudes

speacutecialiseacutes

Compagnies

Conseils

Reacuteduire la

capaciteacute de

production

(groupe

industriel)

Strateacutegique

RH

Communication

Production

Uniteacute

Moyen

Leacutegislation

(dont RH)

Eleveacutee

Direction

Qq mois agrave

anneacutees

Financiegravere

Industrielle

Marcheacute

Organisation

Organismes

speacutecialiseacutes

DRH

Qq mois

Communication

Connaissance des

compeacutetences

Connaissance outil

industriel

Concurrence

Portefeuille

drsquoactiviteacutes

Cabinet

drsquoorganisation

Conseils

speacutecifiques

Acqueacuterir une

entreprise

concurrente

(teacuteleacutephonie

mobile)

Strateacutegique

Marketing

Production

(reacuteseau)

Financier

Communication

Uniteacute

Tregraves eacuteleveacute

Leacutegislation

telecom

Tregraves eacuteleveacutee

Direction

Qq mois

Financiegravere

Marcheacute

Reacuteseaux

(ampleur

recouvrement

hellip)

Organisations

Interne

Racheteacutee

Sources

speacutecialiseacutees

Qq mois

Communication

Marcheacute

Cours boursiers

Cabinet

drsquoorganisation

Conseils

speacutecifiques

Initiation au management copy CRCF ndash J Sornet Page 16 48

LE MANAGEMENT EN PRATIQUE

Pour assumer sa fonction le management doit couvrir sans discontinuiteacute lrsquoensemble de

lrsquoorganisation et inteacutegrer de nombreux facteurs dont nous allons reacutesumer lrsquoessentiel

1 ndash Les fonctions et activiteacutes du management

Pour Henri Fayol la fonction drsquoadministration de lrsquoentreprise (son management) reposait sur

cinq actions preacutevoir organiser commander coordonner et controcircler (laquo PO3C raquo)

Nous distinguerons cinq activiteacutes de management

- la conception (au plus haut niveau finaliteacute but ou vocation de lrsquoorganisation

meacutetiers dimension politique de croissance hellip)

- la planification (deacutefinition des objectifs eacutecheacuteances)

- lrsquoorganisation (reacutepartition du travail choix des modes de coordination)

- le pilotage de lrsquoaction opeacuterationnelle (motivation animation encadrement

assistance)

- lrsquoeacutevaluation (controcircle des reacutesultats obtenus ajustements)

Dans chacune de ces activiteacutes des deacutecisions et des arbitrages sont neacutecessaires avec des

enjeux plus ou moins importants

Remarques

- Les cinq activiteacutes du management peuvent se retrouver agrave tout niveau de

management si lrsquoentreprise laisse une certaine autonomie de deacutecision agrave ses diffeacuterentes

uniteacutes La conception est naturellement du ressort de la direction geacuteneacuterale et des

conseils drsquoadministration mais elle peut ecirctre preacutesente pregraves du terrain (latitude laisseacutee agrave

une filiale ou agrave un magasin par exemple) De mecircme lrsquoorganisation du travail concerne

un atelier mais aussi la direction qui structure lrsquoentreprise pour assurer ses activiteacutes sa

production

- La planification deacutefinit des objectifs ou des axes strateacutegiques (choix de produits

modaliteacutes de deacuteveloppement des ventes implantations alliances hellip) et les traduit en

donneacutees de gestion preacutevisionnelles syntheacutetiques et eacutechelonneacutees dans le temps afin de

valider les objectifs et de fixer des repegraveres

- Un laquo business plan raquo (plan drsquoaffaires)est notamment lrsquoeacutequivalent de la planification

dans le cas de creacuteation drsquoentreprise ou pour la preacutesentation de tout projet drsquoactiviteacute

Les activiteacutes du management srsquoinscrivent dans des cycles qui peuvent ecirctre scheacutematiseacute

comme suit (lrsquoeacutevaluation peut entraicircner une reacutevision du pilotage de lrsquoorganisation ou des

objectifs sans que lrsquoentreprise ne soit fondamentalement remise en cause)

conception

planification

organisation

pilotage

eacutevaluation

Initiation au management copy CRCF ndash J Sornet Page 17 48

2 ndash Les contextes de management

Le management est influenceacute par son contexte qui justifie des objectifs une organisation

des meacutethodes

Par exemple lrsquoentreprise admet de nombreuses variantes selon sa taille sa forme juridique

son controcircle par lrsquoeacutetat (entreprises publiques) ou par des inteacuterecircts priveacutes Il en va de mecircme des

organismes administratifs qui peuvent deacutependre de directives nationales ou reacutegionales des

associations qui ont des activiteacutes drsquoampleur tregraves variable

21 ndash La dimension de lrsquoentreprise

La dimension drsquoune entreprise se mesure principalement en fonction de son effectif ou de

son chiffre drsquoaffaires Des seuils sont deacutefinis par divers organismes et exploiteacutes agrave des fins

statistiques ou pour la deacutetermination de certaines obligations sociales ou fiscales

(repreacutesentation du personnel cotisations hellip) Il nrsquoy a bien entendu pas de laquo barriegravere de

tailleraquo absolue conditionnant le management drsquoune entreprise

LrsquoUE preacuteconise de distinguer les micro ndash entreprises (jusqursquoagrave 9 salarieacutes) les TPE ndash tregraves petites

entreprises (moins de 20 salarieacutes) les petites entreprises (moins de 50) et les moyennes

entreprises (de 50 agrave 250) Cependant les PME sont parfois situeacutees entre 10 et 500 salarieacutes

Remarques

- en France environ 40 des entreprises emploient de 1 agrave 50 salarieacutes (ce qui repreacutesente

plus de 50 des emplois) et 59 nrsquoen ont aucun

le pays compte environ 2 600 000 entreprises dont moins de 1 ont 250 employeacutes et

plus

- ancienneteacute et taille de lrsquoentreprise sont lieacutees si lrsquoon eacutecarte les restructurations et autres

eacutevolutions drsquoentreprises existantes

La dimension de lrsquoentreprise a une influence sur lrsquoorganisation et le laquo style raquo de son

management

- les PME sont souvent entrepreneuriales (les dirigeants eacutegalement apporteurs de capitaux

sont totalement engageacutes dans la marche de lrsquoentreprise) Elles ont une gestion flexible peu

formaliseacutee plus qualitative que quantitative Les PME sont freacutequemment focaliseacutees sur un seul

type drsquoactiviteacute Pour ne pas alourdir leur structure elles ont tendance agrave sous-traiter les

activiteacutes speacutecialiseacutees ne correspondant pas agrave leur meacutetier de base

- les grandes entreprises sont manageacuteriales (les dirigeants sont nommeacutes par les actionnaires

en raison de leurs compeacutetences) et moins reacuteactives

22 ndash Le type de production

On distingue industrie (production de biens mateacuteriels ou pour le moins de produits visibles ndash

comme un seacutejour touristique ou un film) et services (fourniture drsquoune prestation immateacuterielle)

Le type de production influence en principe le management de lrsquoentreprise

- lrsquoindustrie neacutecessite (si lrsquoon excepte lrsquoartisanat) un investissement relativement important

une organisation productive stable capable de reacutealiser plusieurs fois des produits identiques

(exemple un modegravele de reacutefrigeacuterateur) ou du moins similaires (exemple un bacirctiment) Le

produit de lrsquoindustrie consomme des matiegraveres et il doit geacuteneacuteralement ecirctre distribueacute jusqursquoau

client

- la production de services peut se satisfaire drsquoun investissement tregraves reacuteduit et neacutecessite un

contact permanent avec le client

Toutefois la standardisation des services et le deacuteveloppement des reacuteseaux informatiques

rapprochent la production de services de celle des biens industriels

- la production drsquoun service reacutepeacutetitif et technique peut imposer une structure lourde et une

organisation tregraves formaliseacutee (voir les grandes socieacuteteacutes drsquoaudit ou de conseil informatique)

Initiation au management copy CRCF ndash J Sornet Page 18 48

- certains services peuvent ecirctre fournis agrave distance sans contact direct avec le client et

distribueacutes par reacuteseau (tenue de comptabiliteacute affacturage gestion clientegravele centre drsquoappel

hellip)

Remarque les services repreacutesentent 75 de lrsquoactiviteacute eacuteconomique franccedilaise

23 ndash La nature de lrsquoorganisation

Les organisations publiques franccedilaises (administrations centrales collectiviteacutes territoriales

hocircpitaux hellip) repreacutesentent une part importante de lrsquoactiviteacute (environ 30 des emplois) La

fonction publique regroupe des organisations aux finaliteacutes diverses et qui ont des problegravemes

de gestion similaires agrave ceux des entreprises auxquelles elles peuvent emprunter des principes

de management Notamment

- pour controcircler les coucircts et assurer la qualiteacute des services

- pour communiquer avec les administreacutes ou les usagers

- pour motiver les personnels et geacuterer les ressources humaines

La transposition directe des techniques de gestion et de management nrsquoest cependant pas

toujours possible car

- la comptabiliteacute publique obeacuteit agrave des regravegles speacutecifiques (proceacutedure budgeacutetaire

notamment)

- le laquo client raquo ne paye pas toujours la prestation du moins directement

- la concurrence est parfois inexistante

- les grandes administrations centraliseacutees sont soumises agrave des choix politiques geacuteneacuteraux

parfois sans connexion eacutevidente avec les besoins opeacuterationnels

- le statut des personnels et les grilles de salaires limitent les possibiliteacutes de gestion des

ressources humaines

Remarque la LOLF (loi organique relative aux lois de finances) est entreacutee en vigueur en

2006 Elle alloue des moyens budgeacutetaires en fonction de programmes et remplace la

reconduction automatique de 90 des budgets Cette reacuteforme se heurte toutefois agrave la

lourdeur des grands ministegraveres ougrave la complexiteacute des activiteacutes est difficile agrave

appreacutehender et ougrave des inerties culturelles peuvent exister agrave tout niveau

Les associations loi de 1901 peuvent avoir une activiteacute comparable agrave celle de grandes

entreprises (voir par exemple les associations de santeacute ou professionnelles) et leur

management est alors similaire malgreacute lrsquoabsence de but lucratif (les beacuteneacutefices ne sont pas

distribuables) Elles ont drsquoailleurs en France un poids eacuteconomique important (elles emploient

environ 1 600 000 salarieacutes)

Cependant lrsquoadheacutesion agrave un systegraveme de valeurs fondateur de lrsquoassociation ou la limite de

lrsquoautoriteacute (quand un volant de beacuteneacutevoles important participe agrave lrsquoactiviteacute) peut introduire des

nuances

- le renforcement des objectifs socieacutetaux

- la faiblesse des relations hieacuterarchiques

- des contraintes de gestion du temps des beacuteneacutevoles

- des modaliteacutes particuliegraveres de recrutement et de motivation des dirigeants

24 ndash Les facteurs contingents

La theacuteorie de la contingence montre qursquoune structure drsquoentreprise nrsquoest efficace que dans

une situation deacutetermineacutee et qursquoil nrsquoexiste que des solutions de management construites dans

un contexte preacutecis

Le management doit ainsi srsquoadapter agrave des facteurs contingents qui ne peuvent ecirctre

controcircleacutes du moins agrave bregraveve eacutecheacuteance Ces facteurs sont par exemple

- lrsquoancienneteacute de lrsquoentreprise (plus elle est ancienne plus lrsquoentreprise a tendance agrave reacutepeacuteter

des comportements eacuteprouveacutes)

Initiation au management copy CRCF ndash J Sornet Page 19 48

- la taille de lrsquoentreprise (la grande entreprise a une composante administrative plus

deacuteveloppeacutee)

- le systegraveme de production (tregraves standardiseacute complexe automatiseacute hellip)

- lrsquoenvironnement

3 ndash Le management et les parties prenantes

Lrsquoentreprise a pour vocation premiegravere de mettre des produits agrave disposition de ses clients en

reacutealisant un profit Pour y arriver elle doit aussi satisfaire ses parties prenantes salarieacutes

actionnaires fournisseurs hellip

Est partie prenante agrave lrsquoentreprise laquo tout groupe ou individu qui peut ecirctre affecteacute ou est

affecteacute par les buts de lrsquoorganisation hellip raquo (Freeman ndash 1984)

Les parties prenantes attendent agrave des degreacutes divers de profiter drsquoune creacuteation de valeur en

provenance de lrsquoentreprise qui doit reacutepondre agrave ces attentes pour assurer sa peacuterenniteacute ou

favoriser son deacuteveloppement

On distingue les parties prenantes primaires ou principales qui sont essentielles agrave lrsquoentreprise

et qui ont geacuteneacuteralement une relation formelle avec elle (clients associeacutes et actionnaires

precircteurs salarieacutes fournisseurs collectiviteacutes) et les parties prenantes secondaires dont

lrsquoinfluence est diffuse (groupes de pression associations meacutedias instances europeacuteennes

agences de notation hellip)

Remarque la consideacuteration de lrsquoensemble des parties prenantes (laquo stakeholders raquo - les

deacutepositaires) fait contrepoids agrave lrsquoimportance accordeacutee aux seuls actionnaires

(laquo shareholders raquo)

Les organisations nrsquoayant pas drsquoobjectif de profit doivent aussi satisfaire leurs parties

prenantes apporter un service aux usagers dans les meilleures conditions eacuteconomiques

limiter un budget assurer la qualiteacute des relations avec les fournisseurs hellip

Dans cette optique le management doit organiser lrsquoaction de faccedilon agrave eacutequilibrer des forces

parfois divergentes

- le contexte fait pression sur lrsquoorganisation contrainte agrave optimiser ses reacutesultats

- lrsquoorganisation cherche par son action agrave assurer sa peacuterenniteacute son deacuteveloppement (en

reacutealisant des profits dans le cas de lrsquoentreprise) et agrave satisfaire ses parties prenantes

- le management agit en pilotant les actions pour contrebalancer la pression du contexte

Actions de

lrsquoorganisation

Management Contexte

Parties

prenantes

Initiation au management copy CRCF ndash J Sornet Page 20 48

APPLICATIONS MP

MP1 Deacutefinir contingent gestion budgeacutetaire

MP2 Deacuteterminer les parties prenantes drsquoun hocircpital public et leurs principales attentes

Mecircme question pour les organisations suivantes

- SNCF (entreprise publique)

- Peugeot

- MAIF (mutuelle drsquoassurance)

MP3 En les situant dans le cycle des activiteacutes du management trouver les actions agrave mener

dans les situations suivantes

- baisse de 10 des ventes dans une entreprise industrielle (produits meacutenagers le reacuteseau de

distribution vient drsquoecirctre reacuteorganiseacute)

- idem dans une entreprise de vente par correspondance soumise agrave la concurrence internet

(les ventes stagnaient depuis six mois malgreacute les efforts promotionnels)

- augmentation des deacutelais drsquoattente des consultations dans une clinique (lrsquohocircpital voisin a

fermeacute son service drsquourgences)

Initiation au management copy CRCF ndash J Sornet Page 21 48

ELEMENTS DE CORRIGE MP

MP1 Deacutefinir (dans le contexte drsquoune entreprise) contingent gestion budgeacutetaire

Contingent = imposeacute par lrsquoexteacuterieur Contingence = effet du hasard de la rencontre de

plusieurs eacuteveacutenements indeacutependants (variables explicatives que lrsquoon ne peut influencer)

Gestion budgeacutetaire = technique drsquoadministration des entreprises srsquoappuyant sur des

preacutevisions dont on deacuteduit apregraves accord des responsables des attributions de moyens sur une

dureacutee limiteacutee Une analyse reacuteguliegravere des eacutecarts entre preacutevisions et reacutealisations permet ensuite

le pilotage des activiteacutes Le budget est un cadre incitatif

La laquo planification budgeacutetaire raquo consiste agrave traduire en budgets une planification strateacutegique

avec systegraveme de reporting

MP2 Deacuteterminer les parties prenantes drsquoun hocircpital public et leurs principales attentes

Mecircme question pour les organisations suivantes

- SNCF (entreprise publique)

- Peugeot

- MAIF (mutuelle drsquoassurance)

Hocircpital

- patients (qualiteacute des soins)

- CNAM (baisse des coucircts)

- collectiviteacute locale (service aux administreacutes)

- eacutetat (ameacutenagement du territoire maicirctrise des budgets optimisation)

- employeacutes (salaire conditions de travail et satisfaction)

- fournisseurs ndash pharmacie autres (CA paiement reacutegulier)

- associations de patients (qualiteacute proximiteacute des soins)

SNCF

- usagers et associations drsquousagers (proximiteacute reacutegulariteacute prix du service)

- reacuteseau ferreacute de France (optimisation des lignes paiement adapteacute)

- fournisseurs (CA paiement reacutegulier)

- employeacutes (salaire conditions de travail seacutecuriteacute de lrsquoemploi)

- eacutetat (ameacutenagement du territoire)

- collectiviteacutes locales (service)

Peugeot

- clients (qualiteacute prix SAV relation commerciale)

- fournisseurs (CA reacutegulariteacute de lrsquoactiviteacute)

- employeacutes (salaire conditions de travail seacutecuriteacute de lrsquoemploi)

- eacutetat (taxes)

- collectiviteacute locale (emploi dynamisation eacuteconomique preacuteservation de lrsquoenvironnement)

- associations de protection de lrsquoenvironnement (activiteacute propre baisse des eacutemissions

nouvelles eacutenergies)

MAIF

- socieacutetaires (protection relation assureur tarif mesureacute)

- professionnels de lrsquoautomobile et autres (agreacutement marge de manœuvre reacuteparations tarifs

eacuteleveacutes)

- fournisseurs (CA paiement reacutegulier)

- eacutetat (taxes engagement pour la seacutecuriteacute)

- employeacutes (salaire conditions de travail seacutecuriteacute de lrsquoemploi)

Initiation au management copy CRCF ndash J Sornet Page 22 48

MP3 En les situant dans le cycle des activiteacutes du management trouver les actions agrave mener

dans les situations suivantes

- baisse de 10 des ventes dans une entreprise industrielle (produits meacutenagers le reacuteseau de

distribution vient drsquoecirctre reacuteorganiseacute)

Adapter le pilotage motiver cadrer si insuffisant retoucher une organisation deacutefectueuse

- idem dans une entreprise de vente par correspondance soumise agrave la concurrence internet

(les ventes stagnaient depuis six mois malgreacute les efforts promotionnels)

Voir pilotage et organisation si une eacutevolution du meacutetier a deacutejagrave eacuteteacute initialiseacutee Sinon re-

conception (adaptation au nouveau contexte) puis planification et reacuteorganisation

- augmentation des deacutelais drsquoattente des consultations dans une clinique (lrsquohocircpital voisin a

fermeacute son service drsquourgences)

Organisation Si insuffisant planification (nouveaux objectifs)

Initiation au management copy CRCF ndash J Sornet Page 23 48

ORGANISATION ET PROCESSUS

La performance de lrsquoentreprise deacutepend de son organisation et de son aptitude agrave produire

aux meilleures conditions Nous allons montrer comment organisation formelle et processus

de production peuvent contribuer agrave cette performance

1 ndash Vers lrsquooptimum

11 ndash Les eacuteconomies occidentales jusqursquoaux anneacutees 70

Jusqursquoen 1945 le principal problegraveme des entreprises eacutetait de produire des biens en quantiteacute

suffisante agrave un prix compatible avec le marcheacute Les grandes entreprises se sont multiplieacutees et

la standardisation a permis de reacuteduire les coucircts (exemple deacuteveloppement de Ford et de la

production agrave la chaicircne de 1908 agrave 1920 qui a permis une baisse du prix des voitures des 23)

On parle de laquo production pousseacutee vers le marcheacute raquo

Cette croissance de la production peu reacuteguleacutee a eacuteteacute marqueacutee par des surproductions en

1910 et 1920 puis par la crise de 1929 qui a prolongeacute ses effets jusqursquoagrave la guerre

De 1945 agrave 1975 environ (les laquo trente glorieuses raquo) la reconstruction la croissance de la

consommation de masse de nouvelles technologies et les eacutechanges internationaux

alimentent lrsquoeacuteconomie La standardisation srsquoeacutetend aux biens de consommation dont les

coucircts baissent fortement et de nouvelles reacutegulations sociales permettent une eacutevolution sans

heurt des revenus La saturation de certains marcheacutes conduit dans les anneacutees 60 agrave la

deacutemarche laquo marketing raquo et agrave la diffeacuterenciation des produits Le produit est laquo dirigeacute par le

marcheacute raquo mais les entreprises conservent une organisation assez classique et les plus grosses

srsquointernationalisent

12 ndash Lrsquoexpeacuterience japonaise et ses prolongements

Tregraves tocirct apregraves la guerre dans un Japon appauvri le constructeur automobile Toyota a ducirc

faire face agrave une restriction du marcheacute des moyens financiers et productifs et des

approvisionnements La firme a donc innoveacute dans un nouveau systegraveme de production

chassant les laquo gaspillages raquo (temps drsquoattente transports stocks deacutefauts hellip) consideacuterant que

seule la fabrication vendable creacutee de la valeur

Toyota srsquoorganise pour fabriquer la quantiteacute et la qualiteacute de produits juste neacutecessaires agrave la

satisfaction des clients la production est laquo tireacutee par le marcheacute raquo La mise en place de ce

systegraveme qui integravegre les fournisseurs ne sera acheveacutee que dans le milieu des anneacutees 70

En 1973 la hausse du peacutetrole inaugure un ralentissement de la croissance des eacuteconomies

occidentales La concurrence accrue provoque alors un inteacuterecirct pour le systegraveme deacuteveloppeacute

au Japon La production au plus juste se deacuteveloppe ainsi dans lrsquoindustrie automobile agrave partir

des anneacutees 80 et elle se reacutepand encore maintenant dans drsquoautres secteurs

Cette approche qui vise un objectif de zeacutero stock et zeacutero deacutefaut impose la maicirctrise de laquo bout

en bout raquo des processus de production et leur ameacutelioration

Initiation au management copy CRCF ndash J Sornet Page 24 48

2 ndash Organiser lrsquoentreprise

21 ndash Direction et organisation

Diriger une entreprise neacutecessite de lrsquoorganiser (de reacutepartir les tacircches) pour qursquoelle puisse

atteindre ses objectifs Lrsquoorganisation permet de satisfaire un marcheacute en tirant parti des

capaciteacutes actuelles de lrsquoentreprise tout en preacuteparant lrsquoavenir

Lrsquoorganisation reacutesulte freacutequemment drsquoun compromis entre des objectifs situeacutes agrave des niveaux

et des eacutecheacuteances diffeacuterents

Exemples

- le leader des chaises roulantes peut tirer profit de sa structure productive et de son

savoir faire pour entrer sur le marcheacute de la bicyclette eacutelectrique

- ecirctre parfaitement structureacute pour alimenter 90 du marcheacute des disquettes ne preacutepare

pas lrsquoavenir

- srsquoorganiser pour conqueacuterir le marcheacute des tire-bouchons eacutelectriques dans les deux ans

perd de son sens si cela altegravere les moyens neacutecessaires agrave la production drsquoappareils

manuels ancienne mais vitale dont la diminution agrave court terme risque de nuire agrave la

solvabiliteacute de lrsquoentreprise et de la conduire agrave la cessation de paiement

22 ndash Lrsquoorganisation fonctionnelle

La majoriteacute des entreprises adopte une laquo organisation fonctionnelle raquo (celle qui est visible

dans les organigrammes) ougrave des regroupements de personnels et drsquoeacutequipements se font

selon un modegravele hieacuterarchique (laquo line raquo) dans des uniteacutes des services ou des deacutepartements

speacutecialiseacutes Cette organisation peut se deacutecliner agrave lrsquointeacuterieur des divisions des grandes

entreprises quand elles scindent leur activiteacute par zone geacuteographique type drsquoactiviteacute

cateacutegorie de clients hellip

Remarque le terme laquo fonction raquo deacutesigne un rocircle particulier dans le fonctionnement de

lrsquoentreprise

Lrsquoorganisation fonctionnelle diffeacuterencie les activiteacutes de lrsquoentreprise en les regroupant par

meacutetier pour utiliser au mieux les compeacutetences et les moyens (meilleur rendement par la

speacutecialisation lrsquoeacutechange de compeacutetences dans une mecircme uniteacute ou gracircce agrave des eacuteconomies

drsquoeacutechelle)

23 ndash La notion de processus de production

Un processus de production se deacutefinit par la succession drsquoactiviteacutes permettant de satisfaire

un client en transformant des ressources (mateacuterielles financiegraveres humaines) en un produit

bien ou service Le processus doit creacuteer une valeur reconnue par le client

Un processus peut servir un client interne agrave lrsquoentreprise (par exemple en produisant un

composant intervenant dans plusieurs produits ou par la maintenance des machines) aussi

bien qursquoun client final On distingue usuellement

- les processus opeacuterationnels (ou maicirctres) aussi appeleacutes processus meacutetier (business process)

qui satisfont directement les clients finaux (conception et fabrication de produits vente hellip)

- les processus de support et de management (geacuterer les ressources humaines geacuterer

lrsquoinformation geacuterer les ressources financiegraveres hellip) qui ont les processus opeacuterationnels comme

clients

Toutes les actions internes agrave une organisation peuvent srsquointeacutegrer dans des processus qui

conditionnent directement ou indirectement la capaciteacute de lrsquoorganisation agrave satisfaire le

client final ou lrsquousager

Initiation au management copy CRCF ndash J Sornet Page 25 48

Aborder le fonctionnement de lrsquoentreprise par ses processus (approche processus) permet

de mettre en eacutevidence les chaicircnes drsquoactiviteacutes qui conduisent aux produits leurs

dysfonctionnements leurs coucircts la formation des deacutelais et la souplesse (la flexibiliteacute)

disponible pour satisfaire la clientegravele finale Lrsquoameacutelioration des processus a un impact visible

et direct sur chaque produit proposeacute aux clients

Lrsquoapproche processus provoque une eacutevolution de la faccedilon de travailler

- en faisant peacuteneacutetrer la laquo voix du client raquo au plus profond de lrsquoentreprise (et plus seulement

dans les services commerciaux et marketing)

- en mettant en eacutevidence des possibiliteacutes de rationalisation (par regroupement ou impartition

de certaines activiteacutes)

Remarque lrsquoapproche par les activiteacutes et les processus est agrave lrsquoorigine de la meacutethode

de deacutetermination des coucircts laquo ABC raquo - activity based costing

24 ndash Processus et fonctions

Le processus est transversal Il enchaicircne des activiteacutes qui traversent lrsquoentreprise en particulier

les services ou les deacutepartements drsquoune organisation fonctionnelle

Exemple

La division du travail par fonctions induit une charge de coordination pour assurer le

deacuteroulement du processus Elle peut geacuteneacuterer des attentes des erreurs ou des conflits drsquointeacuterecirct

(lrsquoobservation montre que des dysfonctionnements sont tregraves souvent constateacutes lors du

passage drsquoun service agrave un autre)

Organisation fonctionnelle et approche processus visent toutes deux un optimum

eacuteconomique mais leurs logiques sont diffeacuterentes

- le processus vise la satisfaction des clients (prix qualiteacute deacutelais service)

- le deacutecoupage fonctionnel cherche agrave optimiser les moyens (maximiser lrsquoeffet drsquoexpeacuterience

partager des infrastructures profiter de pocircles de compeacutetences hellip) Il apporte une ossature

hieacuterarchique stable souvent indispensable

Organisation fonctionnelle et approche processus sont donc compleacutementaires dans la

majoriteacute des cas et doivent ecirctre combineacutees judicieusement

APPLICATIONS OP

OP1 Deacutefinir flexibiliteacute systegraveme impartition

OP2 Citer huit exemples drsquoinformations essentielles pour optimiser un processus de

fabrication

Direction

Deacutepartement

commercial

(C)

Deacutepartement

administratif et

financier (AF)

Deacutepartement

Etudes (E)

Deacutepartement

Production (P)

Activiteacute

C-x Activiteacute

AF-x Activiteacute

E-x

Activiteacute

P-x

Processus x

Clie

nt

Initiation au management copy CRCF ndash J Sornet Page 26 48

OP3 Deacutegager les principes du toyotisme preacutesenteacute ci-dessous En quoi ce systegraveme est-il

initiateur de lrsquoapproche processus

Taiichi Ohno et le Toyotisme

1 - Extrait drsquoun article de Jacques BARRAUX - 1993 - LExpansion

Taiichi Ohno (1912 ndash 1990) hellip ne se prenait pas pour un visionnaire mais en imposant une

nouvelle faccedilon de produire il a reacuteinventeacute le management hellip tout le monde a entendu parler

des mots qui ont populariseacute le toyotisme dont il est le pegravere le juste-agrave-temps hellip Autant

doutils conccedilus pour lrsquoautomobile et qui ont aujourdhui une application universelle

hellip Taiichi Ohno jeune ingeacutenieur entre chez Toyota alors simple constructeur de machines

textiles Degraves 1926 apparaicirct la notion de jidoka hellip cest lart de transfeacuterer de lintelligence aux

machines pour mieux libeacuterer lintelligence des hommes Tout le contraire du taylorisme qui

juge la machine moins impreacutevisible que lhomme En 1933 Toyota se lance dans lautomobile

en sinspirant des meacutethodes ameacutericaines Mais en 1935 agrave loccasion dun voyage aux Etats-

Unis leacutetat-major de lentreprise revient fascineacute de sa visite dans un supermarcheacute La notion

de juste-agrave-temps va naicirctre de lobservation dune grande surface un lieu ougrave les clients ne

prennent que ce dont ils ont besoin et ougrave les rayons sont reacuteapprovisionneacutes pour compenser

les quantiteacutes preacuteleveacutees Ainsi le systegraveme Toyota est-il deacutejagrave dans la tecircte de ses dirigeants avant

mecircme la Seconde Guerre mondiale un demi-siegravecle avant la reacutevolution informatique et la

segmentation intensive des marcheacutes

hellip des esprits curieux comme Franccedilois Dalle en France tombent alors sous le charme des

formules et des paraboles de Taiichi Ohno En voici deux eacutechantillons

Penser agrave lenvers Cela signifie combattre les ideacutees reccedilues En lespegravece il sagit du fordisme et

du taylorisme Ohno ne croit pas agrave la planification aux effets deacutechelle et dexpeacuterience Il

propose un systegraveme industriel agrave lenvers qui permette de diversifier les produits et de les

fabriquer en petites quantiteacutes Nous ne devons plus ecirctre des paysans qui accumulent des

stocks mais des chasseurs On nimpose pas loffre On traque la demande et on la gegravere en

continu

Que les valleacutees soient hautes et les montagnes peu eacuteleveacutees Plutocirct que de concentrer tous

les efforts sur une production agrave un moment donneacute mieux vaut se doter de structures flexibles

permettant de passer agrave tout instant dune seacuterie agrave une autre Il faut eacuteviter les ruptures et les

secousses aplanir les cycles entretenir des flux reacuteguliers dactiviteacutes diversifieacutees Ce qui

implique de ne pas enfermer les hommes et les eacutequipements dans des speacutecialisations trop

eacutetroites

La flexibiliteacute le travail en groupe le refus de la dictature des machines la polyvalence et

surtout lattention constante aux signaux eacutemis par le marcheacute nappartiennent plus au

toyotisme Ces notions sont les fondements du nouvel art dorganiser de vendre et de

produire dans lindustrie comme dans les services hellip

2 - Quelques notions cleacutes

Taiichi Ohno a imagineacute la meacutethode des laquo cinq pourquoi raquo qui consiste agrave se poser cinq fois de

suite la question laquo pourquoi raquo sur le mecircme sujet de faccedilon agrave deacutecouvrir la veacuteritable cause

drsquoun problegraveme Cette meacutethode peut ecirctre appliqueacutee agrave tous les niveaux et permettre

notamment aux agents de fabrication de proposer de veacuteritables ameacuteliorations de la

production

La recherche de la qualiteacute totale (pas de deacutefaut des produits pas de rebuts pas de deacutefaut

des processus) accompagne la deacutemarche de Toyota La qualiteacute a un coucirct compenseacute par

des ventes accrues par lrsquoeacuteconomie des mesures palliatives aux deacutefauts

Initiation au management copy CRCF ndash J Sornet Page 27 48

Fiche OP1 ndash Benchmarking et processus

Le laquo benchmarking raquo consiste agrave comparer le fonctionnement de plusieurs systegravemes pour en

faire notamment ressortir les meilleures pratiques (laquo best practices raquo) Cette technique est

utiliseacutee depuis les anneacutees 80 pour ameacuteliorer la performance des entreprises Elle impose agrave

lrsquoentreprise drsquoeacutevaluer et de remettre en question ses propres modes de fonctionnement afin

de les faire eacutevoluer agrave la lueur de ce qui se fait ailleurs

Le benchmarking permet drsquoameacuteliorer les processus agrave moindre risque en fixant des objectifs

baseacutes sur des faits et donc plus facilement accepteacutes

Une classification des processus en tant que base de reacuteflexion a eacuteteacute eacutetablie aux USA par

lrsquolaquo International Benchmarking Clearinghouse raquo de lrsquoAPQC (american productivity and

quality center) en collaboration avec plusieurs dizaines drsquoentreprises

Elle se reacutesume ainsi

Le terme laquo reengineering raquo (la re-conception ou laquo reacuteingeacutenieacuterie raquo) des processus deacutesigne un

projet drsquoameacutelioration radicale des performances (de 20 agrave 50 ou plus) Il neacutecessite une

parfaite adheacutesion de la direction la constitution drsquoune petite eacutequipe de projet brillante

connaissant parfaitement les activiteacutes de lrsquoentreprise et il peut inclure un benchmarking

Le reengineering provoque geacuteneacuteralement la reacuteduction du nombre de niveaux hieacuterarchiques

(laquo delayering raquo) et lrsquoaccroissement du pouvoir de deacutecision des employeacutes (laquo empowerment raquo

ou laquo empouvoirement raquo) Bien qursquoy conduisant parfois il ne doit pas ecirctre confondu avec la

reacuteduction des activiteacutes (laquo downsizing raquo ou restructuration) et lrsquoexternalisation (laquo outsourcing raquo)

Pro

ce

ssu

s

op

eacutera

tio

nn

els

Pro

ce

ssu

s d

e m

an

ag

em

en

t e

t d

e

sup

po

rt

1 ndash

Comprendre

le marcheacute et

les clients (besoins

satisfaction)

2 ndash

Deacutevelopper

vision et

strateacutegie (contexte

concurrence)

3 ndash

Creacuteer

produits

services

processus

(concevoir

ameacuteliorer)

4 ndash

Marketing et

vente

5 ndash

Produire et

livrer (industrie

dont

ameacutelioration

processus)

6 ndash

Produire et

livrer (services)

7 ndash

Facturer et

servir les

clients (apregraves-

vente

reacuteclamations)

8 ndash Deacutevelopper et geacuterer les ressources humaines

9 ndash Geacuterer les systegravemes drsquoinformation

10 ndash Geacuterer les ressources financiegraveres et les actifs

11 ndash Appliquer un programme environnemental

12 ndash Geacuterer les relations exteacuterieures (actionnaires banques lois relations publiques hellip)

13 ndash Geacuterer lrsquoameacutelioration et le changement (eacutevaluer mesurer motiver qualiteacute totale)

Initiation au management copy CRCF ndash J Sornet Page 28 48

Fiche OP2 ndash Lrsquoorganisation par processus

Lrsquoeacutevolution drsquoune organisation aux activiteacutes reacutepeacutetitives vers lrsquoapproche processus est

geacuteneacuteralement progressive et se met en place par paliers

La mise en œuvre drsquoun veacuteritable management par processus doit ecirctre preacuteceacutedeacutee quand

lrsquoactiviteacute de lrsquoentreprise est complexe drsquoun recensement (une laquo cartographie des

processus raquo) pour mettre en eacutevidence les processus ou les familles de processus cleacutes critiques

pour le succegraves de lrsquoentreprise ougrave les efforts seront prioritaires

Des responsables de processus (laquo process owners raquo) sont ensuite deacutesigneacutes

Le responsable doit concevoir ses processus puis apregraves leur mise en œuvre assurer les

coordinations neacutecessaires les ameacuteliorer et les repreacutesenter aupregraves de la direction

Quand une structure par processus est mise en place des opeacuterateurs exeacutecutants

preacuteceacutedemment regroupeacutes dans les fonctions peuvent ecirctre affecteacutes aux processus et

drsquoanciens responsables de fonctions peuvent devenir des experts au service des processus

Lrsquoorganisation par processus peut imposer un degreacute eacuteleveacute drsquointeacutegration des activiteacutes donc

une polyvalence accrue des personnels et une reacuteduction des niveaux hieacuterarchiques

Elle neacutecessite pour le moins des compeacutetences eacutelargies au niveau des responsables de

processus (organisation administration technique hellip) dont le nombre doit rester limiteacute

(quelques dizaines au plus)

Sauf dans de tregraves petites structures lrsquoorganisation par processus se plaque geacuteneacuteralement sur

une structure plus classique

Initiation au management copy CRCF ndash J Sornet Page 29 48

ELEMENTS DE CORRIGE OP

OP1 Deacutefinir

Flexibiliteacute = adaptation au besoin (horaire variable chaicircnes robotiseacutees)

Systegraveme = ensemble organiseacute dans un but boicircte noire (sanguin nerveux meacutetrique laquo D raquo)

Impartition = sous-traitance ou externalisation (seacuteparation) drsquoactiviteacutes faire appel agrave des

partenaires plutocirct que faire soi-mecircme

OP2 Citer huit exemples drsquoinformations essentielles pour orienter lrsquooptimisation drsquoun processus

Montant des stocks (approvisionnements et produits finis)

Temps drsquoattente

Taux drsquoactiviteacute des ateliers

Rebuts

Deacutelai de production

Taux de reacuteclamations clients (qualiteacute)

Temps passeacutes en retouches finales

Turn over

Nombre drsquoarrecircts maladie

Accidents du travail

Dureacutee des arrecircts machines

OP3 Deacutegager les principes du toyotisme preacutesenteacute dans la fiche 31 En quoi ce systegraveme

repose trsquoil sur lrsquoapproche processus

Produire la quantiteacute juste neacutecessaire (agrave la demande) donc eacuteviter les stocks

Flexibiliteacute intelligence des chaicircnes de production

Qualiteacute (eacuteviter le coucirct de la non-qualiteacute)

La notion de processus est implicite ainsi que la chaicircne de valeur client

Initiation au management copy CRCF ndash J Sornet Page 30 48

DEFIS ET TENDANCES DU MANAGEMENT

Les meacutethodes de management se deacuteveloppent pour affronter le contexte eacuteconomique

Ce chapitre preacutesente les deacutefis auxquels le management contemporain doit faire face

1 ndash Lrsquoeacutevolution eacuteconomique contemporaine

A mesure que lrsquoactiviteacute eacuteconomique mondiale srsquoaccroicirct que la technologie eacutevolue les

changements sont de plus en plus rapides Ils introduisent des situations ineacutedites auxquelles les

entreprises doivent srsquoadapter en cherchant de nouvelles solutions de management Les trois

derniegraveres deacutecennies ont eacuteteacute notamment marqueacutees par les pheacutenomegravenes suivants (que nous

listons sans tenir compte des liens pouvant exister entre eux)

Pheacutenomegravene Traduction Effets

Deacute reacuteglementation

globalisation

financiegravere

titrisation

Libre circulation des capitaux accegraves

facile des particuliers au marcheacute

boursier (directement ou par

lrsquointermeacutediaire des OPCVM et SICAV)

Monteacutee en puissance du financement

des entreprises sur le marcheacute boursier

Fonds de pension

(retraites) et fonds

souverains (eacutetats)

Poids boursier important drsquoinvestisseurs

institutionnels qui cherchent un haut

rendement financier (dividendes ou

valorisation boursiegravere)

Pression sur les grandes entreprises

influence sur les strateacutegies

Mondialisation Liberteacute des eacutechanges internationaux Accroissement de la concurrence

recherche drsquoavantages eacuteconomiques

par la deacutelocalisation (biens et

services) la concentration des efforts

(recentrage) problegravemes drsquoemploi

multiplication des transports perte

drsquoinfluence des politiques

Baisse de lrsquoemploi

occidental

(notamment

industriel)

Moins de fabrications fabrications

automatiseacutees recours aux moyens

informatiques

Activiteacute reporteacutee sur le commerce la

conception et les services chocircmage

charge sociale

Restructurations Optimisation des entreprises

abaissement des coucircts augmentation

des marges recherche drsquoune taille

critique (eacuteconomies drsquoeacutechelle poids

sur le marcheacute)

Recentrages externalisations fusions

deacutelocalisations constitution de grands

groupes

NTIC (nouvelles

technologies de

lrsquoinformation et de

la communication)

Mise en œuvre des reacuteseaux (dont

internet) et drsquoapplications

informatiques communicantes

Nouvelles formes de commerce

marcheacute international deacutelocalisation

du travail intellectuel reacuteorganisation

de la distribution

Rareacutefaction relative

des matiegraveres

premiegraveres

Recherche de substituts exploration

miniegravere coucircts drsquoexploitation des

gisements accrus

Augmentation des coucircts variations

erratiques du cours des matiegraveres

deacutestabilisations politiques

Evolution

geacuteopolitique et

eacuteconomique

mondiale

Chute de lrsquoURSS transformation des

eacuteconomies collectivistes pays

eacutemergents (Chine Inde Breacutesil Russie)

()

Accroissement de la population

mondiale (4 agrave 6 7 milliards de 1970 agrave

2008)

Libeacuteralisme sans frein () nouvelles

puissances eacuteconomiques

opportuniteacutes de deacuteveloppement

nouveau partage des ressources

ineacutegaliteacutes baisse du soutien aux PVD

laquo Terrorisme raquo Actions armeacutees pression de groupes

armeacutes non gouvernementaux

Deacutestabilisations reacutegionales charge

des deacutepenses militaires

Deacuteveloppement

durable

Recherche drsquoune croissance eacutequitable

et respectueuse de lrsquoenvironnement

Pression sur les entreprises (eacutetats

associations de consommateurs

eacutecologistes ONG)

() Reacutecemment quelques affaires (Enron laquo subprimes raquo Vivendi Universal Socieacuteteacute

Geacuteneacuterale Airbus par exemple) et agrave plus grande eacutechelle la crise financiegravere de 2008 ont

montreacute les dangers drsquoune libeacuteralisation sans controcircles suffisants

Initiation au management copy CRCF ndash J Sornet Page 31 48

() Des alliances eacuteconomiques naissent entre pays eacutemergents (notamment en

ameacuterique centrale creacuteation de la Banque du Sud en 2008 par exemple) et lrsquoon

commence agrave imaginer une baisse progressive de lrsquoinfluence eacuteconomique des Etats

Unis

2 ndash Les deacutefis actuels du management

21 ndash Les grandes orientations

Lrsquoeacutevolution eacuteconomique suggegravere quelques pistes parfois concurrentes pour lrsquoaction du

manager contemporain On y retrouve au premier plan la construction drsquoune vision qui est

une composante commune du leadership

Objectif du manager

pour lrsquoentreprise

Justification Facteurs de reacuteussite

Construire une vision Eclairer lrsquoavenir de lrsquoentreprise partager

un but souder motiver

Effort de reacuteelle prospection

volontarisme de la direction

bonne communication

Reacuteactiviteacute et flexibiliteacute

(sous tous les aspects

agrave tous niveaux)

Srsquoadapter rapidement au marcheacute Bonne organisation des processus

personnel compeacutetent autonome

et motiveacute structure hieacuterarchique

alleacutegeacutee robotisation

Deacutegager des profits Reacutemuneacuterer les apporteurs de capitaux

srsquoautofinancer

Ajuster coucircts et structures

Exploiter les nouvelles

technologies

Reacuteactiviteacute ajuster coucircts et deacutelais

reacutepondre au marcheacute suivre les clients

Organiser le SI de faccedilon

pertinente eacuteviter le coucirct excessif

drsquoinvestissements trop en

laquo pointe raquo (laquo essuyer les placirctres raquo)

utiliser judicieusement les services

exteacuterieurs

Bacirctir des alliances

(contrats fusions)

Deacutevelopper une activiteacute limiter les coucircts

de transaction () atteindre la taille

critique et de meilleurs rendements se

recentrer sur une activiteacute profitable

Dominer les processus se donner

une identiteacute lisible externaliser se

doter drsquoune capaciteacute financiegravere

suffisante

Valoriser lrsquoimage Attirer les clients favoriser les alliances

donner confiance (apporteurs de fonds

employeacutes clients partenaires socieacuteteacute

civile)

Instaurer des regravegles de

gouvernance inteacutegrer le

deacuteveloppement durable

respecter lrsquoenvironnement

Geacuterer les risques Faire face aux aleacuteas eacuteconomiques et

technologiques (conjoncture politiques

accidents malveillance)

Creacuteer un systegraveme drsquoalerte geacuterer

la crise (reacuteaction raisonneacutee

sceacutenarios poursuite de

lrsquoexploitation dans un contexte

instable) mise en place de

proceacutedures drsquoapprentissage pour

ameacuteliorer les reacuteactions au fil du

temps

Geacuterer le changement Faire face agrave lrsquoeacutevolution de la demande

la pression sur les prix la variation des

performances financiegraveres la

concurrence la globalisation des

marcheacutes lrsquoeacutevolution technologique aux

fusions ou alliances aux changements

de reacuteglementation de direction hellip ()

Bonne communication pour

donner du sens au changement

et obtenir lrsquoadheacutesion du personnel

Rassembler et geacuterer les

connaissances former le

personnel

Innover Garder un avantage concurrentiel se

diffeacuterencier

Veille technologique et

commerciale investissement

Ouverture

internationale

Elargir le marcheacute saisir les opportuniteacutes Veille commerciale partenariats

() La theacuteorie des coucircts de transaction deacuteveloppeacutee par OE Williamson dans les

anneacutees 70 integravegre les coucircts lieacutes au recours au marcheacute (recherche et choix drsquoun

fournisseur neacutegociation reacutedaction de contrat suivi des eacutechanges risque de rupture

Initiation au management copy CRCF ndash J Sornet Page 32 48

drsquoapprovisionnement hellip) On peut en conclure que lrsquointeacutegration de diffeacuterentes

activiteacutes agrave lrsquoentreprise (la laquo firme raquo) preacutesente des avantages Mais des coucircts de

transaction internes doivent aussi ecirctre consideacutereacutes (preacuteparation organisation

surveillance hellip) et certaines formes de coopeacuteration continue avec les fournisseurs

permettent de reacuteduire le coucirct des transactions externes

() drsquoapregraves laquo Les meilleures pratiques de management raquo - Brilman Heacuterard ndash EO

Une eacutetude du Conference Board (2002) liste les deacutefis du management vus par 700 leaders

mondiaux Soit en reacutesumeacute avec indication du score correspondant

1 ndash Fideacuteliser les clients (42)

2 ndash Reacuteduire les coucircts (38)

3 ndash Accroicirctre flexibiliteacute et reacuteactiviteacute (29)

4 ndash Amener les employeacutes agrave adheacuterer aux valeurs et visions de lrsquoentreprise (26)

5 ndash Deacutevelopper et retenir les leaders (25)

6 ndash Geacuterer acquisitions et alliances (24)

7 ndash Accroicirctre lrsquoinnovation (20)

En fin de classement citoyenneteacute et reacuteputation (4) et ameacutelioration de la diversiteacute (3)

22 ndash Les techniques disponibles

Pour faire face aux deacutefis le manager dispose de nouveaux concepts et de nouvelles

techniques Le tableau ci-dessous en donne un reacutesumeacute et indique les domaines qursquoils

influencent principalement

Initiation au management copy CRCF ndash J Sornet Page 33 48

Principaux concepts techniques outils Incidence principale sur

Internet

- e-commerce (commerce eacutelectronique site

entreprise)

- CRM ou GRC (gestion de la relation client)

- e-procurement (gestion des approvisionnements

par le reacuteseau)

- messagerie eacutelectronique

- e-recrutement

Vente accegraves au marcheacute

Relation client reacuteactiviteacute personnalisation

fideacutelisation

Deacutelais coucircts

Communication transfert de donneacutees (piegraveces

jointes) tous domaines

Communication recrutement

Intranet reacuteseau drsquoentreprise SI

- knowledge management (gestion des

connaissances)

- e-learning (apprentissage en ligne)

- plateforme de travail collaboratif (groupware)

- workflow (circulation eacutelectronique de

documents enchaicircnement de processus)

- e-RH portail RH (libre accegraves aux postes agrave

pourvoir informations candidatures hellip)

- PGI (progiciel de gestion inteacutegreacute) ou ERP

Innovation capaciteacute au changement veille

documentaire

Formation du personnel accompagnement des

changements

Coordination communication interne

Coordination

Communication interne (voire internet en

externe) reacuteduction des coucircts climat drsquoentreprise

recrutement plans de carriegraveres hellip

Coucircts fiabiliteacute du systegraveme drsquoinformation deacutelais

processus (continuiteacute inteacutegration)

Logistique inteacutegreacutee

Supply Chain Management (SCM) gestion de la

logistique (incluant les approvisionnements)

Processus deacutelais coucircts

Externalisation

Valorisation du capital humain

GPEC (gestion preacutevisionnelle des emplois et

compeacutetences)

Coaching

Reacuteactiviteacute de lrsquoentreprise conservation des

compeacutetences rendements individuels turn-over

adaptation des compeacutetences motivation

Efficaciteacute individuelle controcircle reacutegulation

progregraves processus

Approche processus

Optimisation des processus

Deacutemarche qualiteacute totale (TQM ndash total quality

management)

Empowerment (empouvoirement)

Benchmarking reacuteingeacutenieacuterie

Coucircts marges qualiteacute deacutelais flexibiliteacute

externalisation eacutelargissement des compeacutetences

organisation

Ameacutelioration des processus (meacutetiers et supports)

Autonomie compeacutetences des employeacutes

Ameacutelioration des processus restructuration

Management par la valeur

Parties prenantes

Satisfaction des parties prenantes financement

motivation collaborations hellip

Collaboration inter organisations

Reacuteseaux drsquoentreprises alliances

EDI (eacutechange de donneacutees informatiseacutees) extranet

Impartition externalisation (outsourcing)

Coucircts recentrage investissements lancement

drsquoactiviteacute

Coucircts reacuteactiviteacute deacutelais relations avec

lrsquoadministration

Coucircts recentrage limitation des investissements

Ethique drsquoentreprise

Gouvernance drsquoentreprise (mode de direction

encadreacute par des regravegles)

Rocircle socieacutetal deacuteveloppement durable

environnement

Image de lrsquoentreprise reacutegulation du top

management relations actionnaires

Image peacutenaliteacutes et amendes objectifs

strateacutegiques

Initiation au management copy CRCF ndash J Sornet Page 34 48

23 ndash Le rocircle socieacutetal des entreprises

La responsabiliteacute socieacutetale de lrsquoentreprise (RSE) deacutesigne le rocircle qursquoelle prend dans la socieacuteteacute

au-delagrave de son activiteacute purement geacuteneacuteratrice de profit On parle aussi drsquoentreprise citoyenne

La RSE est indissociable du deacuteveloppement durable de porteacutee mondiale et dont les trois

piliers sont

- eacuteconomique (favoriser le deacuteveloppement les eacutechanges internationaux)

- social (accegraves aux soins eacuteducation conditions de travail hellip)

- environnemental (pollution preacuteservation des ressources hellip)

La RSE integravegre notamment une preacuteoccupation sociale de lrsquoentreprise vis-agrave-vis de ses salarieacutes

(seacutecuriteacute et santeacute au travail juste reacutemuneacuteration deacuteveloppement personnel hellip) Elle conduit agrave

tenir compte dans le management drsquoune vision exteacuterieure agrave lrsquoentreprise qui peut avoir des

reacutepercussions possibles sur son activiteacute eacuteconomique

Lrsquoentreprise peut aussi tirer avantage drsquoune deacutemarche responsable par la baisse de certains

coucircts (plus faibles consommations drsquoeacutenergies reacuteduction des transports hellip)

Le rocircle socieacutetal de lrsquoentreprise a eacuteteacute reconnu en France par la loi laquo NRE raquo de 2001 (loi sur les

nouvelles reacutegulations eacuteconomiques) qui oblige les socieacuteteacutes franccedilaise coteacutees sur un marcheacute

reacuteglementeacute agrave rendre compte dans leur rapport annuel de leur gestion sociale et

environnementale au travers de leur activiteacute

Article 116 de la loi Le rapport viseacute agrave larticle L 225-102 rend compte hellip laquo Il comprend

eacutegalement des informations dont la liste est fixeacutee par deacutecret en Conseil dEtat sur la

maniegravere dont la socieacuteteacute prend en compte les conseacutequences sociales et

environnementales de son activiteacute Le preacutesent alineacutea ne sapplique pas aux socieacuteteacutes

dont les titres ne sont pas admis aux neacutegociations sur un marcheacute reacuteglementeacute raquo

Une norme ISO 14000 integravegre ces preacuteoccupations et des taxes eacutecologiques sont

progressivement creacutees

3 ndash Le management par la valeur

31 ndash De lrsquoanalyse au management par la valeur

Lrsquoanalyse de la valeur est neacutee en 1947 aux Etats-Unis (General Electrics) Cette technique

consiste agrave eacutelaborer des produits conformes aux attentes de la clientegravele mais sans excegraves pour

trouver un bon compromis entre valeur pour le client et coucirct Le produit optimal est deacutefini agrave

partir drsquoenquecirctes qui deacuteterminent le besoin client (ou plutocirct drsquoun client laquo type raquo)

Exemple il est inutile de concevoir un petit veacutehicule citadin capable de parcourir

500 000 km sans avarie compte tenu des effets de mode et du faible kilomeacutetrage

annuel Par contre le marcheacute peut exiger un fonctionnement sans faille sur 150 000 km

soit dix ans en moyenne ce qui conditionne les coucircts de production

Cette recherche drsquoun ajustement de valeur au besoin des clients eacutetait un preacutecurseur du

management par la valeur qui recherche plus largement la creacuteation de valeur pour

chacune des parties prenantes de lrsquoentreprise tout en lui meacutenageant un reacutesultat suffisant

Plus geacuteneacuteralement le management par la valeur est deacutefini par une norme europeacuteenne (EN

12973)

Le management par la valeur est un style de management particuliegraverement destineacute agrave

mobiliser les individus agrave deacutevelopper les compeacutetences et agrave promouvoir les synergies et

Initiation au management copy CRCF ndash J Sornet Page 35 48

linnovation avec pour objectif la maximisation de la performance globale dun

organisme Le management par la valeur apporte une nouvelle faccedilon dutiliser nombre

de meacutethodes de management existantes Il est en coheacuterence avec le Management

de la qualiteacute

Cette approche du management pose de nombreuses questions notamment quelles

prioriteacutes et quelles valeurs attribuer aux parties prenantes comment appreacutehender la

perception par les parties prenantes de la valeur qui leur est affecteacutee

32 ndash La valeur client

Le processus drsquoeacutelaboration drsquoun produit qui consomme des ressources coucircteuses doit creacuteer

une valeur suffisante pour provoquer lrsquoachat par le client final La production drsquoune valeur

reconnue par le client est vitale pour lrsquoentreprise mais sa deacutetermination est parfois complexe

La valeur du produit perccedilue par le client integravegre des eacuteleacutements en partie subjectifs

- une valeur drsquousage (le produit reacutepond agrave un besoin)

- une valeur drsquoestime (lrsquoimage apporteacutee par le produit un aspect affectif)

- une valeur drsquoeacutechange (deacuteduite de lrsquoespoir de revente du produit)

Valeurs drsquousage drsquoestime et drsquoeacutechange deacutependent implicitement de la qualiteacute (un bien peu

fiable est impropre agrave lrsquousage attendu de mauvaise qualiteacute notoire il nrsquoapporte pas une

image positive et ses deacutefauts connus nuisent agrave sa revente) Une eacutevaluation de la qualiteacute

intervient donc dans la valeur perccedilue du produit

Par ailleurs le client considegravere le coucirct drsquoobtention du produit (les charges qursquoil doit supporter

pour acqueacuterir le produit lrsquoeffort qursquoil doit faire pour trouver le produit et les frais de mise agrave

disposition)

Le prix perccedilu par le client est geacuteneacuteralement supeacuterieur au prix de vente

Le client achegravete theacuteoriquement le produit qui preacutesente la diffeacuterence valeur perccedilue ndash prix

perccedilu la plus favorable ou le meilleur rapport prix perccedilu qualiteacute perccedilue et dans certains

cas celui qui a le prix produit le plus bas

Remarque les valeurs du scheacutema ci-dessus changent durant le cycle de vie du produit

(un nouveau produit peut avoir une valeur perccedilue plus eacuteleveacutee qursquoen fin de vie) La

valeur client ne peut ecirctre eacutevalueacutee que par enquecirctes et ne peut donc ecirctre deacutefinie avec

certitude

La notion de laquo satisfaction client raquo conseacutecutive agrave une vente influence aussi le prix produit et

le prix perccedilu

- lrsquoentreprise gagne sur les coucircts de recherche de clientegravele

- le client nrsquoa pas agrave rechercher un nouveau fournisseur et beacuteneacuteficie drsquoun coucirct drsquoobtention

plus bas

valeur perccedilue client

prix perccedilu client

coucirct produit Marge (valeur creacuteeacutee pour

lrsquoentreprise)

euros

prix produit

Valeur creacuteeacutee

pour le client

Initiation au management copy CRCF ndash J Sornet Page 36 48

La satisfaction du client deacutepend de facteurs qualitatifs aussi divers que la fiabiliteacute du produit

la vitesse de reacuteaction du fournisseur lrsquoattitude des commerciaux lrsquoefficaciteacute du service

apregraves-vente la netteteacute des contrats ou la justesse de la facture

Valeur perccedilue coucirct marge et satisfaction reacutesultent de processus allant de la conception du

produit jusqursquoagrave sa livraison et son apregraves-vente La deacutemarche laquo processus raquo et lrsquolaquo analyse de la

valeur raquo en forccedilant la recherche de solutions efficientes agrave tout niveau administratif

technique commercial et apregraves-vente sont donc neacutecessaires pour bien positionner

lrsquoentreprise sur son marcheacute

Pour autant le risque commercial ne peut jamais ecirctre annuleacute et lrsquooffre de lrsquoentreprise ne

satisfait geacuteneacuteralement pas en milieu concurrentiel tous ses clients potentiels

33 - La creacuteation de valeur pour les autres parties prenantes

Les salarieacutes

La creacuteation drsquoune valeur suffisante pour les salarieacutes est reconnue comme neacutecessaire car des

observations montrent que la satisfaction des clients en deacutepend Moins souvent eacutevoqueacutee en

peacuteriode de chocircmage elle nrsquoest prioritaire que pour les employeacutes dont lrsquoentreprise souhaite

conserver les compeacutetences

La laquo valeur salarieacute raquo ne comprend pas que le salaire Le sentiment drsquoappartenance agrave un

groupe la reconnaissance lrsquoaccomplissement de soi et la construction professionnelle en

sont des eacuteleacutements importants Comme pour les clients on doit ainsi distinguer la reacutetribution

perccedilue du salaire objectif

Les actionnaires

Lrsquoactionnaire apporte des fonds propres agrave lrsquoentreprise en contrepartie de titres parfois

neacutegociables en bourse et assortis drsquoun droit de vote en assembleacutee geacuteneacuterale La valeur

attribueacutee aux actionnaires est servie en termes moneacutetaires (dividende ou augmentation de la

valeur du titre neacutegociable)

Remarque des facteurs non moneacutetaires comme lrsquoimage de lrsquoentreprise qui deacutepend

en partie de sa communication peuvent influencer la deacutecision drsquoachat de vente ou

de conservation des titres par lrsquoactionnaire

Reacutetribution perccedilue euros

Salaire objectif

Avantage non

moneacutetaire de

lrsquoemploi

Initiation au management copy CRCF ndash J Sornet Page 37 48

Compte tenu de lrsquoimportance croissante de lrsquoactionnariat dans le financement des grandes

entreprises coteacutees en bourse et notamment des investisseurs institutionnels comme les fonds

de pension des indicateurs speacutecifiques ont eacuteteacute introduits pour appreacutecier la performance des

entreprises vue par les actionnaires Par exemple la valeur ajouteacutee eacuteconomique (EVA reg

economic value added marque deacuteposeacutee de Stern Stewart ou VAE ndash valeur ajouteacutee

eacuteconomique parfois deacutenommeacutee VEC ndash valeur eacuteconomique creacuteeacutee) qui prend en compte le

coucirct du capital

LrsquoEVA correspond tregraves scheacutematiquement au calcul suivant

EVA = (PO) profit opeacuterationnel ndash (C) coucirct du capital X (CE) capitaux employeacutes

LrsquoEVA neacutecessite en pratique des retraitements assez complexes Le PO peut se deacuteterminer

selon les principes suivants

- PO = reacutesultat drsquoexploitation (avant inteacuterecircts) ndash impocirct

- PO = beacuteneacutefice courant (tenant compte des inteacuterecircts) + inteacuterecircts ndash eacuteconomie drsquoimpocirct sur les

inteacuterecircts (on exclue les eacuteleacutements financiers et lrsquoimpocirct correspondant) ndash impocirct

- lrsquoimpocirct pris en compte correspond au profit opeacuterationnel consideacutereacute (dans les cas courants agrave

13 du PO)

C = taux moyen de reacutemuneacuteration du capital (reacutesultant par exemple du dividende exigeacute de

certains investisseurs et des taux drsquoemprunts bancaires)

CE = capitaux propres et dettes portant inteacuterecirct

Remarque le profit opeacuterationnel ou reacutesultat opeacuterationnel correspond au NOPAT ndash net

operating profit after tax - anglo-saxon LrsquoEVA est eacutegale au NOPAT diminueacute de la

reacutemuneacuteration des capitaux

Exemple lrsquoentreprise X dispose drsquoun capital de 2 500 000 euro et reacutealise un beacuteneacutefice net

drsquoimpocirct de 450 000 euro (taux 33 13) Un dividende de 6 doit ecirctre verseacute aux

actionnaires et la banque lui a accordeacute un precirct de 1 200 000 euro agrave 4 Les autres

constituants des reacutesultats financier et exceptionnel sont neacutegligeables

Reacutesultat opeacuterationnel = 450 000 + 004 x 1 200 000 x 23 = 482 000 euro

Coucirct du capital = 006 x 2 500 000 + 004 x 1 200 000 x 23 = 182 000 euro

EVA = 300 000 euro

Coucirct moyen pondeacutereacute du capital (C) = (004 x 1 200 000 x23 + 006 x 2 500 000)

3 700 000 Soit 492

Si lrsquoEVA est positive lrsquoentreprise creacuteeacutee de la valeur apregraves reacutemuneacuteration des capitaux et sa

valeur boursiegravere doit augmenter

Lrsquoutilisation de lrsquoEVA comme indicateur influence le management de lrsquoentreprise car il y a

trois moyens pratiques drsquoaugmenter lrsquoEVA

- augmenter le reacutesultat opeacuterationnel

- lancer des investissements ayant une rentabiliteacute supeacuterieure agrave C

- eacuteliminer les activiteacutes ayant une rentabiliteacute infeacuterieure agrave C

Remarque lrsquoutilisation sans nuance de lrsquoEVA comme critegravere de management peut

poser problegraveme Le calcul de lrsquoEVA repose sur des ajustements comptables il est donc

sujet agrave manipulations (provisions capitalisation ou non de la RD hellip) Par ailleurs le

critegravere laquo EVA raquo pris isoleacutement peut conduire agrave chercher la rentabiliteacute agrave court terme agrave

reacuteduire les investissements prospectifs et donc nuire agrave terme au deacuteveloppement de

lrsquoentreprise

Initiation au management copy CRCF ndash J Sornet Page 38 48

Les fournisseurs reccediloivent le paiement de leurs factures plus ou moins rapidement (le deacutelai

de paiement repreacutesente une valeur consentie au fournisseur)

Lrsquoentreprise peut accroicirctre la valeur apporteacutee agrave ses fournisseurs par des actions cibleacutees

comme une contribution agrave la formation de leurs personnels certains transferts de

technologie ou de savoir faire agrave des sous-traitants une coopeacuteration suivie favorisant leur

deacuteveloppement lrsquointeacutegration agrave des campagnes de promotion

A noter que la valeur consentie aux fournisseurs peut avoir une influence sur la qualiteacute et les

deacutelais de livraison des produits

La collectiviteacute reccediloit des taxes et parfois des prestations en nature par deacutefaut ou explicites

(effort de preacuteservation de lrsquoenvironnement ameacutenagement du territoire par les implantations

aide mateacuterielle agrave des projets participation agrave la formation par exemple)

APPLICATIONS DT

DT1 Deacutefinir expliquer deacutereacuteglementation socieacutetal eacuteconomies drsquoeacutechelle coaching EDI

gouvernance

DT2 Deacuteterminer en quoi la deacutemarche TQM srsquoinscrit dans les deacutefis actuels du management

DT3 Apregraves avoir consulteacute les documents ci-dessous extraits du site drsquoAir France

(httpdeveloppement-

durableairfrancecomFRfrlocaldemarcheN4_positionnement_pphtm)

exposer les enjeux et les limites de la RSE et de la gestion des parties prenantes

Initiation au management copy CRCF ndash J Sornet Page 39 48

Dialogue avec les parties prenantes

Initiation au management copy CRCF ndash J Sornet Page 40 48

Attentes des parties prenantes

Initiation au management copy CRCF ndash J Sornet Page 41 48

Creacuteation de valeur pour les parties prenantes

La creacuteation de valeur pour les parties prenantes est au cœur de la strateacutegie du Groupe Le scheacutema de

distribution financiegravere ci-dessous donne un aperccedilu de la distribution des recettes du Groupe aux

diffeacuterentes parties prenantes actionnaires collaborateurs fournisseurs pouvoirs publics

collectiviteacutes locales etc

Initiation au management copy CRCF ndash J Sornet Page 42 48

Fiche DT1 ndash Extrait du sommaire de laquo Problegravemes eacuteconomiques raquo No 2894

La gestion des entreprises bouleverseacutee par les technologies de linternet

Reacutealiteacutes industrielles - Annales des Mines Jean-Michel Yolin

Avec lavegravenement de linternet les processus de conception de production et de vente sont

radicalement remis en cause Quel que soit le secteur dactiviteacute les technologies de linternet

permettent en effet de reacuteduire les deacutelais et de passer dun processus discontinu agrave un processus

continu Lorganisation des entreprises et leur mode de gestion en sont profondeacutement bouleverseacutes

tant au niveau individuel que collectif Linternet rend ainsi possible la reacutealisation dobjectifs que les

entreprises cherchaient agrave atteindre depuis longtemps sans y parvenir meilleure eacutecoute du client

travail sans stocks en flux tendu hieacuterarchies plates autorisant une grande reacuteactiviteacute flexibiliteacute dans

lorganisation et loutil de production acceacuteleacuteration du renouvellement des produits entreprises en

reacuteseau ougrave chacune se recentre sur son cœur de meacutetier etc

Le laquo knowledge management raquo ou comment geacuterer les connaissances

Document de travail du LAMSADE - Michel Grundstein

Peter Drucker lavait preacutedit le capital immateacuteriel eacutetait voueacute agrave devenir un facteur de compeacutetitiviteacute

pour lentreprise La libeacuteralisation des eacutechanges acceacutelegravere les processus de deacutecision de lentreprise

et implique que lassimilation des informations soit agrave la fois de meilleure qualiteacute et plus rapide Ainsi

la fonction qui consiste agrave manager les connaissances au sein de lentreprise savegravere primordiale

Bien que la prise de conscience de limportance du capital immateacuteriel ait eacuteteacute tardive - le concept

de knowledge management est apparu en France aux Etats-Unis et au Japon au milieu des

anneacutees 1990 - agrave lheure actuelle lorganisation de leacutechange dinformations et le partage des

connaissances sont devenus des facteurs cleacutes dune gestion performante de lentreprise Ils

doivent sinscrire dans un projet global destineacute agrave mettre en valeur les savoirs et les savoir-faire

individuels et collectifs

Les leccedilons du laquo coaching raquo pour le management de la qualiteacute

Humanisme et Entreprise - Martine Brasseur

Parmi les nouvelles formes de management en vogue dans les entreprises le coaching figure en

bonne place Appliqueacute au management de la qualiteacute il sagit dune pratique

daccompagnement destineacutee agrave initier et agrave faciliter le processus de deacuteveloppement dun individu

La deacutemarche consiste agrave affirmer que tout individu est en quecircte de qualiteacute agrave condition toutefois

de ne pas lui imposer des contraintes lempecircchant de progresser On considegravere notamment les

erreurs comme potentiellement feacutecondes En deacutefinitive le coach donne au coacheacute la permission

de reacuteussir en lui donnant aussi la permission deacutechouer

Initiation au management copy CRCF ndash J Sornet Page 43 48

Fiche DT2 ndash Management strateacutegique les sept deacutefis agrave relever dici agrave 2016

Extrait drsquoun article du site wwwlentreprisecom -Sabine Blanc - Mis en ligne le 20032007

(httpwwwlentreprisecom325article11977html)

Une eacutetude anglaise publieacutee par lopeacuterateur Orange Grande-Bretagne deacutecrypte la mutation

des formes de travail et les enjeux majeurs pour les entreprises de demain afin decirctre au top

de la compeacutetitiviteacute Voici les challenges-cleacutes pour les managers qui veulent rester dans la

course hellip

1 - Future organisation du travail les quatre laquo mondes raquo possibles

La reacutealiteacute sera probablement un meacutelange de ces quatre sceacutenarios souligne lrsquoeacutetude

Les mondes mutuels Tout se passe dans le cadre des communauteacutes locales vie priveacutee

comme professionnelle Le modegravele coopeacuteratif preacutevaut au lieu du laquo big business raquo Oublieacutes

aussi dans ce systegraveme les trajets pour aller au bureau les gens preacutefegravereront travailler dans de

petites entreprises locales souvent connecteacutees au reacuteseau drsquoautres structures similaires

Les laquo reacutepondants raquo (en anglais laquo replicants raquo) La figure du consultant freelance deviendra

dominante tandis que celle du salarieacute deacuteclinera Il ne sera pas rare de travailler pour plusieurs

entreprises On perdra en seacutecuriteacute de lrsquoemploi en visibiliteacute et en routine ce que lrsquoon gagnera

en liberteacute La majeure partie des tacircches srsquoeffectuera chez soi avec la possibiliteacute de srsquoinstaller

temporairement dans les bureaux de son client du moment Dans un contexte dincertitude

sur lrsquoavenir les travailleurs alterneront peacuteriodes drsquoactiviteacute intense et repos Ce sera agrave eux

drsquoaller vers les entreprises et non lrsquoinverse mecircme si celles-ci devront veiller agrave rester attractives

Les cottages eacutelectroniques Comme ce nom le suggegravere le teacuteleacutetravail deviendrait la norme

univers priveacute et professionnel se confondant Plus besoin de subir une heure de transport les

salarieacutes se logueront de chez eux sur le reacuteseau de lrsquoentreprise Les reacuteunions se tiendront dans

de petits bureaux centraux situeacutes agrave courte distance La flexibiliteacute du temps de travail srsquoimpose

Les salarieacutes disposeront de plus de marge de liberteacute dans leur activiteacute

Les disciples de la nueacutee Cette appellation poeacutetique cache simplement une extension de

lrsquoorganisation actuelle des grandes entreprises avec des salarieacutes se rendant sur un lieu de

travail centraliseacute Le rocircle croissant des technologies de lrsquoinformation multipliera les faccedilons de

collaborer et accroicirctra lrsquoefficaciteacute Le controcircle du travail sera omnipreacutesent La frontiegravere entre

travail et vie priveacutee restera marqueacutee

2 - Sept deacutefis pour les entreprises et leur managers

Quoi qursquoil advienne les entreprises et leurs dirigeants devront concentrer leurs efforts sur sept

points-cleacutes pour srsquoadapter Voici quelques exemples de probleacutematiques souleveacutees par le

rapport et des pistes de solution

Le leadership Les managers devront entre autres savoir persuader et influencer des

travailleurs beaucoup plus indeacutependants Ils auront aussi agrave repenser les niveaux auxquels

prendre les deacutecisions strateacutegiques en haut ou au contraire agrave des degreacutes moins eacuteleveacutes de la

pyramide hieacuterarchique

gt Faire du management une force facilitant les activiteacutes transversales plutocirct que la reacuteduire agrave

la seule fonction de deacutecision

La culture drsquoentreprise Davantage de salarieacutes capables de reacutefleacutechir seront neacutecessaires

tandis que les tacircches qui peuvent ecirctre automatiseacutees ou scripteacutees diminueront Un des

enjeux creacuteer une culture agrave mecircme drsquoattirer et drsquoencourager les personnes preacutesentant ces

qualiteacutes de reacuteflexion requises dans un contexte de compeacutetition accrue et de plus grande

indeacutependance des travailleurs

Initiation au management copy CRCF ndash J Sornet Page 44 48

gt Passer si neacutecessaire drsquoune culture drsquoentreprise forte agrave un mode drsquoengagement plus

consensuel moins rebutant

La marque Conseacutequence du recours croissant agrave lrsquo laquo outsourcing raquo lrsquoimage drsquoune marque

deacutependra plus drsquoagents exteacuterieurs qui ne fonctionnent pas forceacutement selon le mecircme mode

drsquoorganisation Comment garder le controcircle dessus

gt Choisir le mode qui corresponde le plus agrave vos valeurs et preacutevoir un programme de risk

management qui mette en eacutevidence ougrave les conflits sont susceptibles de jaillir

Lrsquoinnovation Plus que jamais il faudra faire face agrave une acceacuteleacuteration du rythme de

lrsquoinnovation en proposant constamment des solutions adapteacutees

gt Tisser des partenariats strateacutegiques avec drsquoautres entreprises pour partager les coucircts et les

fruits de lrsquoinnovation

Le deacutefi opeacuterationnel et technologique De quelle faccedilon controcircler lrsquoinformation crsquoest-agrave-dire

faire en sorte que les bonnes personnes accegravedent facilement agrave une information toujours en

phase tout en maintenant la seacutecuriteacute

gt Recourir agrave des laquo feuilles de route des futurs raquo syntheacutetisant en une page les indicateurs

sociaux et de consommation ainsi que les eacutevolutions technologiques et leacutegislatives qui

influent sur les changements et indiquant comment ils modifient vos marcheacutes vos clients et

votre organisation

La qualiteacute Si de nouveaux proceacutedeacutes ont pu deacutegrader la qualiteacute comme le recours agrave des

centres drsquoappel externaliseacutes drsquoautres ideacutees se sont reacuteveacuteleacutees plus prometteuses comme en

teacutemoigne le succegraves de certaines compagnies aeacuteriennes low cost Elles ont su conjuguer prix

serreacutes et services eacuteleveacutes ce qui devra devenir la norme estime lrsquoeacutetude

gt Continuer de rechercher la qualiteacute Elaborez aussi une bonne prestation service qui inclut

une livraison de qualiteacute voire creacuteez-la en partenariat avec les consommateurs

La leacutegislation La question de la proprieacuteteacute intellectuelle pourrait ecirctre probleacutematique Elle est

deacutejagrave source de conflits comme en teacutemoigne le procegraves pour violation de brevet intenteacute agrave RIM

le fabricant canadien du Blackberry par NTP Que pourra-t-on et que faudra-t-il proteacuteger par

un brevet Il sera eacutegalement neacutecessaire drsquoadapter la leacutegislation aux nouveaux modes

drsquoorganisation

gt Collaborer avec les acteurs du mecircme secteur et les leacutegislateurs pour deacutevelopper les

modegraveles des lieux de travail du futur et bacirctir le droit le plus adeacutequat

Orange a-t-il vu juste dans ses preacutevisions Rendez-vous dans neuf ans pour la reacuteponsehellip

Initiation au management copy CRCF ndash J Sornet Page 45 48

Fiche DT3 ndash Le management par la qualiteacute totale

Extrait drsquoune lettre drsquoinformation du cabinet Baud Accordance Consulting AD2 consultants ndash

2002

1 - Le TQM (Total Quality Management) offre pour lentreprise une vision de la qualiteacute plus

large et transversale

Son principe est simple La finaliteacute de lEntreprise est de deacutevelopper la satisfaction de ses

clients tout en eacutetant beacuteneacuteficiaire cest agrave dire pas agrave nimporte quel prix Elle doit ameacuteliorer sa

rentabiliteacute au travers de la deacutemarche qualiteacute La Qualiteacute Totale vise agrave fournir aux clients

externes et internes une reacuteponse adeacutequate agrave leurs attentes dans le meilleur rapport qualiteacute

prix la meilleure efficience

Elle considegravere pour cela lensemble des processus de lentreprise ayant une incidence sur la

qualiteacute et la satisfaction des clients

Le TQM fait ainsi une large place agrave

la deacutefinition et la planification de la strateacutegie geacuteneacuterale

la coheacuterence de la politique qualiteacute avec la strateacutegie

la deacutemultiplication de la politique qualiteacute dans toutes les directions de lentreprise

la relation client fournisseur interne

la prise en compte de lenvironnement concurrentiel

la consideacuteration de lensemble des risques potentiels financiers sociaux concurrentielshellip

limplication et la motivation du personnel

lanalyse des besoins des clients et le positionnement marketing

la maicirctrise des processus transverses internes

les reacutesultats sous tous ses aspects y compris financiers commerciaux image

De nombreux reacutefeacuterentiels sont relatifs agrave la Qualiteacute Totale hellip Tous ces reacutefeacuterentiels imposent un

questionnement plus profond et indiscret sur le mode de fonctionnement de lentreprise et

son management

helliphellip

2 - LISO 9001 2000 au travers du deacuteploiement des processus (management supports

reacutealisation et ameacutelioration continue) reacutepond quelque peu agrave la mecircme logique

LISO est une ouverture indeacuteniable vers la logique du TQM mais ne se reacutefegravere pas agrave la notion

defficience

Les dirigeants sont cependant sensibles agrave la neacutecessaire reacuteduction des coucircts de non-qualiteacute

et dobtention de la qualiteacute agrave la rentabiliteacute du systegraveme de management de la qualiteacute

mais ne perccediloivent pas toujours la qualiteacute comme une deacutemarche globale

Les deacutemarches qualiteacute commencent bien souvent par la remise en cause de lorganisation

leacutevaluation critique de son efficaciteacute lexamen des processus et la mise en eacutevidence des

lourdeurs administratives

La qualiteacute devient laffaire de tous hellip

Initiation au management copy CRCF ndash J Sornet Page 46 48

Fiche DT4 ndash Le deacuteveloppement durable et la RSE

Extrait du site wwwvigeocom

(httpwwwvigeocomcsr-rating-agencyfrmethodologiecriteres-de-recherche37-

criteres-d-analysehtml)

Deacuteveloppement durable laquo un deacuteveloppement qui reacutepond aux besoins du preacutesent sans compromettre

la capaciteacute des geacuteneacuterations futures de reacutepondre aux leurs raquo (Commission mondiale sur lrsquoenvironnement

et le deacuteveloppement ndash 1987)

Reacutefeacuterentiel drsquoeacutevaluation des entreprises par le groupe Vigeacuteo (le groupe mesure les performances et le

niveau de maicirctrise des risques de responsabiliteacute sociale des entreprises et des organisations - site

wwwvigeocom)

1 Ressources Humaines Ameacutelioration continue des relations professionnelles des relations drsquoemploi et des conditions de travail 2 Droits humains sur les lieux de travail Respect de la liberteacute syndicale et promotion de la neacutegociation collective non discrimination et promotion de lrsquoeacutegaliteacute eacutelimination des formes de travail proscrites (enfants travail forceacute) preacutevention des traitements inhumains ou deacutegradants de type harcegravelements sexuels protection de la vie priveacutee et des donneacutees personnelles 3 Environnement Protection sauvegarde preacutevention des atteintes agrave lenvironnement mise en place drsquoune strateacutegie manageacuteriale approprieacutee eacuteco conception protection de la biodiversiteacute et maicirctrise rationnelle des impacts environnementaux sur lrsquoensemble du cycle de vie des produits ou services

4 Comportements sur les marcheacutes Prise en compte des droits et inteacuterecircts des clients inteacutegration de standards sociaux et environnementaux dans la seacutelection des fournisseurs et sur lrsquoensemble de la chaicircne drsquoapprovisionnement preacutevention effective de la corruption respect des regravegles concurrentielles 5 Gouvernement drsquoentreprise Efficience et probiteacute assurance de lrsquoindeacutependance et de lrsquoefficaciteacute du Conseil drsquoadministration effectiviteacute et efficience des meacutecanismes drsquoaudit et de controcircle et notamment inclusion des risques de responsabiliteacute sociale respect des droits des actionnaires et notamment des minoritaires transparence et rationaliteacute de la reacutemuneacuteration des dirigeants 6 Engagement socieacutetal Effectiviteacute inteacutegration manageacuteriale de lrsquoengagement contribution au deacuteveloppement eacuteconomique et social des territoires drsquoimplantation et de leurs communauteacutes humaines engagements concrets en faveur de la maicirctrise des impacts socieacutetaux des produits et des services contribution transparente et participative agrave des causes drsquointeacuterecirct geacuteneacuteral

Initiation au management copy CRCF ndash J Sornet Page 47 48

ELEMENTS DE CORRIGE DT DT1 Deacutefinir expliquer

Deacutereacuteglementation = suppression des contraintes eacuteconomiques (libre eacutechange des biens et

capitaux)

Socieacutetal = qui se rapporte agrave la structure agrave lrsquoorganisation ou au fonctionnement de la socieacuteteacute

Economies drsquoeacutechelle = reacuteduction des coucircts lieacutee au niveau drsquoactiviteacute (amortissement des

charges fixes)

Coaching = accompagnement de personnes ou deacutequipes pour le deacuteveloppement de leurs

potentiels

EDI = eacutechange de donneacutees informatiseacutees ET standardiseacutees (ex SWIFT bancaire edifact

documents deacuteclaratifs)

Gouvernance = exercice du pouvoir la bonne gouvernance est participative et eacutequitable

conforme agrave lrsquointeacuterecirct commun

DT2 Deacuteterminer en quoi la deacutemarche TQM srsquoinscrit dans les deacutefis actuels du management

Voir notamment fiche 43

Maicirctrise des processus reacuteduction des coucircts reacuteactiviteacute et satisfaction de la clientegravele = faire

face agrave la concurrence

Ameacutelioration de lrsquoimage motivation du personnel

DT3 Apregraves avoir consulteacute les documents ci-dessous extraits du site drsquoAir France

(httpdeveloppement-

durableairfrancecomFRfrlocaldemarcheN4_positionnement_pphtm)

exposer les enjeux et les limites de la RSE et de la gestion des parties prenantes

Trame geacuteneacuterale possible

Introduction

Les deacutefis contemporains (accroissement de la concurrence devenue mondiale recherche

de nouveaux avantages concurrentiels pression de la socieacuteteacute besoin drsquoimage et de projet

lisible pour mener lrsquoentreprise crise et scandales du libeacuteralisme hellip) RSE et PP

Deacuteveloppement (voir cours)

1 ndash Parties prenantes et management par la valeur

PP deacutefinir citer reacutesumer lrsquoavantage rechercheacute (fideacuteliser motiver recherche drsquoalliances

implicites)

PP moyens (dont exemples AF) et meacutethode de management par la valeur (reacutepartie)

2 ndash La responsabiliteacute socieacutetale de lrsquoentreprise

RSE 3 axes

- eacuteconomique (favoriser le deacuteveloppement les eacutechanges internationaux)

- social (accegraves aux soins eacuteducation conditions de travail hellip)

- environnemental (pollution preacuteservation des ressources hellip)

RSE gouvernance drsquoentreprise facteur drsquoimage inteacutegrable dans la deacutemarche PP

Article 116 de la loi Le rapport viseacute agrave larticle L 225-102 rend compte hellip laquo Il comprend

eacutegalement des informations dont la liste est fixeacutee par deacutecret en Conseil dEtat sur la maniegravere

dont la socieacuteteacute prend en compte les conseacutequences sociales et environnementales de son

activiteacute Le preacutesent alineacutea ne sapplique pas aux socieacuteteacutes dont les titres ne sont pas admis aux

neacutegociations sur un marcheacute reacuteglementeacute raquo

Initiation au management copy CRCF ndash J Sornet Page 48 48

RSE exemple AF (ONG fournisseurs)

3 ndash Liens entre PP et RSE

- la RSE introduit de nouvelles PP

- la RSE suppose le respect des PP usuelles (employeacutes clients notamment)

4 - Probleacutematique

- deacutefinir la valeur reacuteellement apporteacutee par une gestion des PP (confusion salaire ndash valeur

idem impocircts hellip ex laquo valeur ajouteacutee raquo)

- communication (neacutecessaire mais aller au-delagrave)

- marginaliteacute des deacutepenses RSE (efficaciteacute sinceacuteriteacute de lrsquoengagement marge de manœuvre)

- charge RSE reporteacutee sur des tiers (ex fournisseurs AF)

- inteacutegration de facteurs non visibles en comptabiliteacute (pertes drsquoemploi nuisances hellip)

Conclusion

Voies incontournables mais pouvant nrsquoavoir qursquoun effet superficiel et temporaire Voir utiliteacute

drsquoaccompagnement leacutegislatif de regravegles de gouvernance

Initiation au management copy CRCF ndash J Sornet Page 2 48

INTRODUCTION AU MANAGEMENT

Le management concerne tout groupe organiseacute en fonction drsquoun but Nous prendrons

toutefois le plus souvent lrsquoentreprise comme reacutefeacuterence courante sachant que les theacuteories et

les techniques preacutesenteacutees sont transposables agrave drsquoautres situations

1 ndash Les contextes du management

11 ndash Lrsquoorganisation

Une organisation est un ensemble de moyens structureacute en fonction drsquoobjectifs qui regroupe

notamment des personnes Les organisations sont multiforme elles peuvent ecirctre priveacutees

publiques vendre des produits des services ou ecirctre financeacutees par des cotisations ou lrsquoimpocirct

avoir ou non un but lucratif hellip

Lrsquoentreprise est une organisation particuliegravere qui vise la reacutealisation et le partage de beacuteneacutefices

(contrairement aux administrations publiques et aux associations sans but lucratif) Elle creacutee

une valeur ajouteacutee (diffeacuterence entre la valeur de la production et les consommations drsquoune

peacuteriode) qui est reacutepartie entre les salarieacutes les institutions financiegraveres les actionnaires lrsquoeacutetat

hellip)

Diriger piloter une entreprise consiste agrave deacutevelopper (au moins agrave conserver) sa valeur

eacuteconomique en tenant compte des contraintes et des opportuniteacutes internes (eacutequipements

disponibles compeacutetences hellip) et de celles de son environnement

Le terme socieacuteteacute correspond agrave une forme juridique de lrsquoentreprise (socieacuteteacute anonyme SARL

hellip)

Le terme firme drsquoorigine anglo-saxonne deacutesigne la place et le fonctionnement de

lrsquoentreprise sur le marcheacute dans les theacuteories eacuteconomiques notamment dans les theacuteories de la

firme deacuteveloppeacutees depuis les anneacutees 30 et qui connaissent de nouveaux deacuteveloppements

depuis une vingtaine drsquoanneacutees

12 ndash Lrsquoentreprise et son environnement

On distingue le microenvironnement de lrsquoentreprise (fournisseurs clients collectiviteacutes

concurrents banques hellip) et son macro environnement (eacuteconomique social juridique

politique hellip)

Lrsquoenvironnement est complexe changeant et il a des effets multiples sur lrsquoentreprise Le

marcheacute (client et concurrents) a un effet primordial car il conditionne la capaciteacute agrave vendre

des produits et donc agrave survivre

ENTREPRISE

Marcheacute (clients Marcheacute des capitaux Marcheacute du travail

concurrents)

Politique

Lois et regraveglements

Ecologie

Conditions

eacuteconomiques

geacuteneacuterales Facteurs socio

culturels modes

Fournisseurs Technologie Deacutemographie

partenaires

Urbanisme geacuteographie

ENVIRONNEMENT

Initiation au management copy CRCF ndash J Sornet Page 3 48

Exemple lrsquoeacutecologie (la preacuteoccupation environnementale) peut influencer lrsquoactiviteacute de

lrsquoentreprise en modifiant les coucircts (transports eacutelimination des deacutechets

perfectionnement des installations matiegraveres hellip) en imposant des regravegles (normes de

pollution emballages hellip) en induisant une pression commerciale (image de

lrsquoentreprise preacutesentation des produits) etc

2 ndash Le management

21 ndash Deacutefinition du management

Le management est une activiteacute dont la finaliteacute est de conduire une organisation vers son but

(reacutealiser des profits se deacutevelopper assurer un service public hellip) par la reacutealisation drsquoobjectifs

preacutedeacutefinis

Le management est lieacute au travail collectif (laquo ecirctre efficaces agrave plusieurs travailler avec les

autres faire travailler les autres raquo) et son importance grandit avec la taille de lrsquoorganisation

Il trouve (au terme pregraves) ses origines dans les premiers grands chantiers de lrsquoEgypte ancienne

et son vocabulaire emprunte aux premiegraveres grandes organisations occidentales que sont les

armeacutees (cadre strateacutegie tactique hellip)

Le terme laquo manager raquo deacutesigne ceux qui ont une responsabiliteacute drsquoencadrement (en France

cette notion ne se confond pas toujours avec celle de cadre qui correspond agrave un statut

mais pas obligatoirement agrave une fonction)

22 ndash Les apports du management

Diriger consiste agrave fixer des objectifs en effectuant des choix Geacuterer crsquoest organiser et

optimiser les moyens disponibles pour atteindre les objectifs

Le terme laquo management raquo qui tend agrave remplacer la laquo gestion raquo ou la laquo direction raquo dans le

vocabulaire eacuteconomique franccedilais englobe ces deux notions Cette eacutevolution terminologique

accompagne une eacutevolution dans la faccedilon de conduire les organisations

laquo Manager raquo suppose

UN CERTAIN REALISME

Le management repose sur le pragmatisme propre aux pays anglo-saxons ougrave

laquo management raquo et laquo manager raquo deacutesignent depuis longtemps ce ou ceux qui encadrent la

marche de lrsquoentreprise

UNE GRANDE REACTIVITE

Le manager a la capaciteacute drsquoagir en fonction des circonstances pour maicirctriser rapidement

des situations complexes Le management se distingue ainsi

- de lrsquo laquo administration raquo au sens courant du terme que lrsquoon retrouve par exemple dans

conseil drsquoadministration ou administration de lrsquoEtat qui eacutevoque plutocirct une action laquo haut

placeacutee raquo assez eacuteloigneacutee du quotidien (bien que lrsquoadministration drsquoentreprise inaugureacutee par

Fayol soit eacutequivalente au management ndash Voir les MBA initialiseacutes agrave Harvard et lrsquoIAE en France)

- de la laquo gestion raquo qui renvoie communeacutement agrave un ensemble de techniques (gestion

financiegravere gestion comptable gestion des ventes gestion de la treacutesorerie hellip)

UNE ACTION SUR LES HOMMES

De nombreux auteurs ont souligneacute cet aspect du management En particulier

Initiation au management copy CRCF ndash J Sornet Page 4 48

- Peter Drucker pour qui le management repose sur cinq principes fixer des objectifs

organiser le travail motiver et communiquer former les eacutequipes (au sens de lrsquoapprentissage

laquo former les autres et soi-mecircme raquo)

- Henry Mintzberg qui met lrsquoaccent sur le rocircle unificateur du manager laquo Ce qui distingue

avant tout une organisation formelle drsquoun quelconque rassemblement drsquohommes ndash drsquoune

foule drsquoun groupe informel ndash crsquoest la preacutesence drsquoun systegraveme drsquoautoriteacute et drsquoadministration

personnifieacute par un ou plusieurs managers dans une hieacuterarchie plus ou moins structureacutee et

dont la tacircche est drsquounir les efforts de tous dans un but donneacute raquo

On en deacuteduit le rocircle relationnel du manager et le poids de la gestion des ressources

humaines dans son activiteacute

LA PRIMAUTE DE LrsquoINFORMATION

Le management fixe des objectifs il a un rocircle deacutecisionnel et il creacutee les conditions neacutecessaires

pour atteindre les objectifs dont il controcircle la reacutealisation Tout ceci neacutecessite la mise en place

drsquoun systegraveme drsquoinformation fournissant les donneacutees neacutecessaires agrave des choix pertinents

(donneacutees qui remontent souvent par les managers eux-mecircmes) puis agrave la mise en œuvre et

au controcircle des reacutealisations

23 ndash Les dimensions du management

La fonction de management a de fait trois dimensions humaine (faire travailler des

personnes ensemble motiver) eacuteconomique (fonctionnement ou deacuteveloppement de

lrsquoorganisation aux meilleurs conditions) et informationnelle (dont la communication)

Le management est une activiteacute de synthegravese qui neacutecessite des capaciteacutes agrave traiter et

combiner des informations drsquoorigines diverses plus ou moins varieacutees selon le contexte

(financiegravere technique leacutegale eacuteconomique politique hellip) et agrave agir en conseacutequence dans les

trois dimensions

Humain

Economique

Information

Informations sur

lrsquoenvironnement

Informations sur

lrsquoorganisation

Management

Information

dirigeant lrsquoaction

des autres

Actions directes

du manager

Initiation au management copy CRCF ndash J Sornet Page 5 48

24 ndash La recherche de performance

Le management recherche la performance de lrsquoorganisation en rapport avec ses objectifs

qui peuvent ecirctre de diffeacuterentes natures (expansion profit hellip ou agrave un niveau plus deacutetailleacute

reacuteduction des deacutelais flexibiliteacute reacuteduction des coucircts accroissement de la qualiteacute hellip)

La performance peut ecirctre abordeacutee de deux faccedilons

- par lrsquoefficaciteacute qui conduit agrave la reacutealisation des objectifs

- par lrsquoefficience qui conduit agrave une utilisation optimale des moyens disponibles avec le

meilleur rendement

Exemple une entreprise qui atteint son objectif de croissance de 10 du chiffre

drsquoaffaires et de son beacuteneacutefice est efficace mais celle qui arrive au mecircme reacutesultat avec

moins drsquoactifs et moins de personnel est plus efficiente

25 ndash Le champ drsquoaction du management

Le management est mis en œuvre dans des peacuterimegravetres drsquoampleur variable un groupe une

entreprise une activiteacute particuliegravere de lrsquoentreprise un projet la tenue drsquoun magasin hellip Les

actions de management ont ainsi des conseacutequences plus ougrave moins importantes

Le management srsquoexerce agrave tous les niveaux drsquoencadrement de lrsquoorganisation Il est de

coutume de distinguer trois niveaux

Exemple

Management strateacutegique la direction geacuteneacuterale a fixeacute les objectifs de chaque filiale du

groupe en tenant compte des positions souhaiteacutees sur le marcheacute Il en reacutesulte que les

uniteacutes X et Y aux activiteacutes similaires et compleacutementaires doivent preacuteparer leur fusion

preacutevue dans les deux ans La filiale Z situeacutee dans une ville universitaire doit acqueacuterir des

compeacutetences pour innover dans lrsquoapplication des supra conducteurs drsquoici trois ans Les

budgets et les financements correspondants ont eacuteteacute globalement estimeacutes

Management intermeacutediaire en application de ces orientations strateacutegiques les

directions de X et Y planifient des reacuteunions de travail communes pour voir comment

harmoniser leurs ressources humaines dans les douze mois Z deacutecide de contractualiser

ses relations avec le centre universitaire pour atteindre ses objectifs et mettre en place

un partenariat en recherche appliqueacutee

Management strateacutegique ou geacuteneacuteral

(direction laquo top management raquo deacutefinition

des objectifs geacuteneacuteraux)

Management tactique

(intermeacutediaire laquo middle

managers raquo)

Management opeacuterationnel (de

terrain ou drsquouniteacute local

laquo executive manager raquo)

Initiation au management copy CRCF ndash J Sornet Page 6 48

Management opeacuterationnel X et Y mettent en place leur communication avec le

personnel et un plan de reconversion Z nomme un directeur de recherche qui prend

notamment en charge les relations avec lrsquouniversiteacute

Le management recouvre la totaliteacute des actes de conduite de lrsquoorganisation dans tous les

domaines (technique commercial financier hellip) mais il ne correspond agrave aucune cellule de

lrsquoorganigramme

Les actions de management sont par ailleurs contraintes par la disponibiliteacute des ressources

neacutecessaire pour atteindre les objectifs (financement savoir-faire profil du personnel

eacutequipements hellip)

3 ndash La meacutethode laquo management raquo

31 ndash La science du management

Le management nrsquoest pas une science exacte il srsquoapparente agrave une science humaine

expeacuterimentale qui traite de pheacutenomegravenes socio-eacuteconomiques eacutevolutifs et qui doit trouver

concregravetement son application dans la vie des organisations

Cette science traite notamment de lrsquoorganisation des entreprises et rassemble des meacutethodes

et des theacuteories qui peuvent ecirctre regroupeacutees en eacutecoles ou en courants

Les theacuteories marquent geacuteneacuteralement une eacutepoque et elles peuvent se recouper partiellement

parfois srsquoopposer avec des nuances qui doivent ecirctre bien identifieacutees Elles srsquoaccompagnent

souvent de modegraveles et de scheacutematisations qui en facilitent la compreacutehension et la

transposition agrave de nouvelles situations

Ces outils scientifiques guident le raisonnement permettent drsquoappreacutehender des reacutealiteacutes

complexes et structurent les connaissances ils sont peacutedagogiques et constituent des aides

pour lrsquoaction Leur application doit cependant ecirctre raisonneacutee car

- un modegravele nrsquoest qursquoune simplification de la reacutealiteacute

- une theacuteorie sortie de son contexte historique et eacuteconomique peut perdre de sa pertinence

Sa mise en œuvre doit tenir compte de la situation reacuteelle et une theacuteorie ne peut agrave elle seule

justifier une deacutecision de management (ce drsquoautant plus que chaque theacuteorie ne couvre que

tregraves partiellement le domaine du management ou de lrsquoorganisation)

- il est souvent difficile de disposer agrave temps drsquoinformations fiables et suffisantes pour appliquer

une theacuteorie dans les conditions ideacuteales Le manager est freacutequemment ameneacute agrave prendre ses

deacutecisions sur la base drsquoinformations incomplegravetes ou incertaines et il doit alors en mesurer les

risques et preacutevoir les ajustements neacutecessaires

ENTREPRISE

Management Administration

Gestion

Strateacutegique

Opeacuterationnel

Initiation au management copy CRCF ndash J Sornet Page 7 48

- des modes influencent le management Elles peuvent indiquer une veacuteritable eacutevolution

eacuteconomique mais aussi ecirctre sans lendemain voire introduire un danger ou un coucirct inutile

(se meacutefier des speacutecialistes dont le fond de commerce est la vente de nouvelles techniques

de management et des seacuteminaires associeacutes)

- seule la creacuteation drsquoune combinaison originale (dans le respect des regravegles) peut apporter un

avantage agrave lrsquoentreprise et non la reproduction de choix de management connus de tous

Exemple 1 la matrice de portefeuille drsquoactiviteacutes permet de classer les activiteacutes

strateacutegiques drsquoune entreprise en fonction de leur taux de croissance et de la part de

marcheacute deacutetenue (matrice laquo BCG raquo - Boston consulting group ndash 1975) Cet outil de

management neacutecessite de disposer drsquoinformations fiables concernant le marcheacute Il

conduit souvent agrave simplifier les conditions de concurrence et ignore la

compleacutementariteacute pouvant exister entre activiteacutes (synergie partage de techniciteacute

amortissement de charges fixes hellip) La matrice est un moyen de prendre conscience

du portefeuille et de ses eacuteventuels deacutefauts (portefeuille deacuteseacutequilibreacute avec

preacutedominance anormale drsquoune zone) mais il ne permet pas seul de deacutecider de lrsquoavenir

des activiteacutes

Exemple 2 le lancement drsquoun investissement lourd (lrsquoimplantation de nouvelles usines

le lancement drsquoune nouvelle activiteacute agrave fort taux de recherche ndash deacuteveloppement hellip) ne

peut ecirctre deacutecideacute que par le recoupement de diffeacuterentes approches (financiegravere

commerciale strateacutegique) Aucune theacuteorie du management appliqueacutee isoleacutement et

sans preacutecaution ne peut justifier un tel investissement

Exemple 3 une entreprise produisant des eacutequipements meacutedicaux deacutecide de renforcer

son offre commerciale par un service de conseil et drsquoassistance Cette deacutecision est

inspireacutee par le concept de creacuteation de valeur par une volonteacute de se diffeacuterencier de la

concurrence et par des pratiques existant depuis longtemps dans le domaine

informatique

Remarque parmi les facteurs influenccedilant les principes de management il est utile de

savoir deacutetecter pour srsquoen preacutemunir les ideacuteologies preacutejugeacutes et autres laquo valeurs raquo sans

rapport certain avec lrsquoefficaciteacute eacuteconomique

32 ndash Theacuteorie et pratique du management

La litteacuterature (ouvrages revues) aborde le management sous des angles diffeacuterents qui

peuvent suggeacuterer une concurrence entre des approches theacuteorique et pratique ou

psychologique et eacuteconomique de cette discipline

Taux de croissance

du domaine (cf

cycle de vie du

produit)

Part de marcheacute

(compareacutee au

principal

concurrent)

fort

faible

forte faible

Dilemmes (activiteacutes

en phase de

lancement

potentiel + coucircts+

risque+)

Vedettes

(activiteacutes en forte

croissance

autofinancement+)

Vaches agrave lait

(activiteacutes

stabiliseacutees et

concurrence faible

ou stable

rentabiliteacute+)

Poids morts

(activiteacutes en

deacuteclin)

Initiation au management copy CRCF ndash J Sornet Page 8 48

Le management a cependant une viseacutee unique et concregravete la bonne marche des

organisations et les travaux des universitaires rejoignent lrsquoaction des praticiens (les

universitaires reacutealisent drsquoailleurs bien souvent leurs recherches au sein des organisations)

Au plan peacutedagogique les deux visions du management se complegravetent

- la vision acadeacutemique met en perspective des concepts et des theacuteories Elle permet de

srsquoapproprier des raisonnements essentiels et stables notamment concernant la strateacutegie et

les reacuteactions humaines qui sont le moteur des organisations

- la vision pratique relate le veacutecu des praticiens preacutesente des techniques concregravetes et tente

parfois une vulgarisation favorisant la diffusion des concepts Elle integravegre de faccedilon

pragmatique les objectifs eacuteconomiques et les contraintes de fonctionnement des

organisations en se placcedilant parfois dans une vision agrave court terme

Sur le terrain le management reacutealise une synthegravese entre des techniques de psychologie

sociale et des techniques de gestion de diverses origines et de nouvelles meacutethodes

apparaissent aussi sous le label unique laquo management raquo

Le terme manageacuterial (approche manageacuteriale theacuteorie manageacuteriale pratique manageacuteriale

hellip) fait reacutefeacuterence agrave la vision aux preacuteoccupations et aux actions des managers qui doivent

emmener leur organisation vers la reacutealisation de ses objectifs

Exemples

- la deacutetermination des coucircts et des marges reacutesulte de techniques de gestion

indispensable aux deacutecisions des managers

- le laquo CRM raquo (customer relationship management en franccedilais GRC ndash gestion de la

relation client) est un concept reacutecent qui srsquoaccompagne de techniques lieacutee aux plus

reacutecents deacuteveloppements du management et des systegravemes drsquoinformation

- le laquo coaching raquo est une technique de management permettant une eacutevolution

personnelle dans le sens des objectifs de lrsquoentreprise

- le recentrage sur le laquo meacutetier raquo repose sur des techniques de management

Les techniques sont rassurantes mais pas suffisantes et si les theacuteories ne sont pas

indispensables au praticien elles aident agrave comprendre agrave anticiper et agrave bien utiliser les

techniques

La qualiteacute du management reacuteside beaucoup dans la capaciteacute agrave appliquer concepts et

techniques de faccedilon pertinente et agrave innover Cette capaciteacute relegraveve en partie drsquoun laquo art du

management raquo qui srsquoacquiert en grande partie par la pratique

4 ndash Management et expertise comptable

Lrsquoexpert comptable doit manager ses propres eacutequipes Il est par ailleurs supposeacute laquo hellip

conseiller et accompagner le chef drsquoentreprise dans toutes ses deacutecisions hellip raquo (selon lrsquoOEC)

Ce rocircle est particuliegraverement important dans ses relations avec les petites entreprises Il doit

donc avoir une capaciteacute au management

Organisation

Vision acadeacutemique Vision pratique

Initiation au management copy CRCF ndash J Sornet Page 9 48

Lrsquointervention de lrsquoexpert dans le management drsquoune entreprise peut toutefois poser

quelques problegravemes

- Il peut y avoir conflits drsquointeacuterecirct entre activiteacutes de certification des comptes et de conseil en

management (la tendance est agrave la seacuteparation des activiteacutes dans les plus grands cabinets)

- lrsquoactiviteacute de conseil neacutecessite des compeacutetences parfois tregraves speacutecifiques (conseil fiscal

conseil en RH conseil en systegravemes drsquoinformation hellip)

- le conseil est une activiteacute diffeacuterente par sa forme de lrsquoexpertise comptable (interventions

longues peu reacutepeacutetitives peu codifieacutees mises en concurrence) qui neacutecessite une

organisation particuliegravere du cabinet lorsqursquoelle deacutepasse lrsquointervention occasionnelle

APPLICATIONS IM

IM1 Analyser la profession de laquo manager raquo selon Henry Mintzberg (texte extrait de lrsquoouvrage

laquo Le management raquo Eyrolles - Editions drsquoOrganisation) et les principes du management de la

norme ISO

Faire ressortir les eacuteleacutements speacutecifiques agrave chacune de ces approches et mettre en eacutevidence

leurs points communs

Initiation au management copy CRCF ndash J Sornet Page 10 48

Principes du management drsquoapregraves la norme ISO 9001 (2000)

- Orientation vers le client (satisfaire ses attentes)

- Leadership (les dirigeants eacutetablissent les orientations de lrsquoorganisme Ils doivent creacuteer

un environnement interne ougrave les personnes peuvent clairement srsquoimpliquer dans la

reacutealisation des objectifs de lrsquoorganisme)

- Implication du personnel (les personnes sont agrave tout niveau lrsquoessence de lrsquoorganisme et

leur implication permet drsquoutiliser leurs aptitudes au profit de lrsquoorganisme)

- Approche laquo processus raquo (un reacutesultat est mieux atteint quand les ressources et les

activiteacutes neacutecessaires sont geacutereacutees comme un processus)

- Approche systegraveme (assimiler les processus correacuteleacutes agrave un systegraveme contribue agrave

lrsquoefficaciteacute et agrave lrsquoefficience de lrsquoorganisme vis-agrave-vis de ses objectifs)

- Ameacutelioration continue (objectif permanent de lrsquoorganisme)

- Prise de deacutecision efficace (par lrsquoanalyse de donneacutees et drsquoinformations)

- Relations mutuellement beacuteneacutefiques avec les fournisseurs (pour augmenter la capaciteacute

des deux organismes agrave creacuteer de la valeur)

IM2 Distinguer leader et manager

IM3 Compleacuteter le tableau ci-dessous en analysant chaque action preacutesenteacutee Faire ensuite

ressortir les domaines niveaux ou techniques de management pouvant ecirctre mobiliseacutes pour

chaque situation

Initiation au management copy CRCF ndash J Sornet Page 11 48

Caracteacuteristiques

de lrsquoaction

- reacutepeacutetition

- risque

- normes

- ampleur

Prise de

deacutecision

- opeacuterationnelle

strateacutegique

- deacutelai

Informations

neacutecessaires

- nature

- origine

- deacutelai obtention

Cleacutes pour la

reacuteussite

Intervention

exteacuterieure

possible

Assurer la

restauration du

soir

(restaurant

familial)

Construire un

viaduc

(autoroute)

Certifier les

comptes

annuels drsquoun

groupe

national

(cabinet

drsquoaudit)

Lancer une

ligne drsquoavions

(constructeur

aeacuteronautique)

Reacuteduire la

capaciteacute de

production

(groupe

industriel)

Acqueacuterir une

entreprise

concurrente

(teacuteleacutephonie

mobile)

Initiation au management copy CRCF ndash J Sornet Page 12 48

Fiche IM1 - Deacutefinitions du management

Dictionnaire anglais - franccedilais direction administration gestion intrigue manegravege

Wikipeacutedia Le management est lensemble des techniques dorganisation qui sont mises en

oeuvre pour ladministration dune entiteacute

Au point de vue eacutetymologique le verbe manage vient de litalien maneggiare (controcircler)

influenceacute par le mot franccedilais manegravege (faire tourner un cheval dans un manegravege) A cette

notion il faut aussi ajouter la notion de meacutenage (geacuterer les affaires du meacutenage) qui consiste agrave

geacuterer des ressources humaines et des moyens financiers

helliphellip

Fiche IM2 - Etudier le management

Concreacutetiser

Manager neacutecessite de syntheacutetiser des informations parfois complexes incomplegravetes et de

domaines tregraves divers pour en deacuteduire des actions Une approche trop parcellaire peut

conduire agrave lrsquoeacutechec et le savoir-faire est neacutecessaire pour agir vite avec un minimum de risque

Lrsquoeacutetudiant doit se preacuteparer simultaneacutement aux examens et agrave la pratique Il nrsquoa souvent connu

lrsquoentreprise que durant quelques semaines de stage et le manque de laquo recul raquo ne lui permet

pas toujours de concreacutetiser les theacuteories Il doit compenser par la lecture (ouvrages revues

journaux eacuteconomiques et boursiers) et en eacutetant attentif aux informations ambiantes (tout en

relativisant le style journalistique) en mettant en relation le cours les concepts les modegraveles

lrsquoactualiteacute les stages

Savoir traiter un exercice

Pour reacuteussir un examen ou traiter une application peacutedagogique (la conception les points 1 agrave

6 peut repreacutesenter le tiers du temps de travail)

1 ndash Identifier le type de sujet (faut-il trouver une solution pratique ou communiquer une

reacuteflexion geacuteneacuterale )

2 ndash Lire le sujet et relever les mots cleacutes

3 ndash Deacutefinir les mots cleacutes

4 ndash Reacutesumer la probleacutematique du sujet (en quelques lignes)

5 ndash Lister les connaissances reacutefeacuterences et raisonnements reacutepondant au problegraveme (par

recherche spontaneacutee ou raisonneacutee qui quoi ougrave quand comment combien hellip

listage des diffeacuterents points de vue) trouver des exemples (notamment dans les

documents fournis)

6 ndash Organiser la reacuteponse (deacutefinir le plan du deacuteveloppement ougrave des paragraphes bien

identifieacutes sont geacuteneacuteralement neacutecessaires en y liant les parties qui doivent ecirctre en nombre

limiteacute ndash de deux agrave quatre) Preacutevoir drsquoy inteacutegrer la deacutefinition des principales notions

induites par le sujet

7 ndash Reacutediger sous la forme adapteacutee (note technique ou recommandation solution

pratique exposeacute structureacute dissertation)

Introduction et conclusion sont indispensables agrave la dissertation ou agrave lrsquoexposeacute

- lrsquointroduction preacutesente le sujet traiteacute (phrase drsquoaccroche initiale) amorce la

probleacutematique (quelques sous - questions) et annonce le plan

- la conclusion syntheacutetise le deacuteveloppement (arguments) eacutelargit le sujet (prise de recul)

et apporte le point final (une phrase)

Une limite agrave la communication

Il est difficile de faire passer plus de 4 ou 5 ideacutees fortes dans un exposeacute unique

Initiation au management copy CRCF ndash J Sornet Page 13 48

Fiche IM3 - Bref historique

Antiquiteacute

3000 AJC

Peacuteriode greacuteco-

romaine

Transition

feacuteodale

12egraveme siegravecle

europe

15egraveme ndash 17egraveme

siegravecles

19egraveme siegravecle

20egraveme siegravecle

agriculture preacutedominante industrie limiteacutee aux besoins drsquoun individu ou drsquoun clan

pour la confection des outils des vecirctements et de la poterie Force motrice animale

ou humaine pour lrsquoessentiel

Grands travaux drsquoeacutetat en Egypte premiegravere laquo planification ndash organisation ndash controcircle raquo

Deacuteveloppement des communications essor industriel limiteacute peu de progregraves

technique (lrsquoesclavage supplante les innovations)

Deacuteveloppement progressif des eacutechanges commerciaux

La consommation indirecte atteint un bon niveau (surplus agricoles et

deacuteveloppement des villes) Apparition de nouveaux commerccedilants

Etat fort Evolutions technologiques (imprimerie bateaux performants instruments de

navigation) Extension geacuteographique de lrsquoeacuteconomie Apparition des corporations

drsquoartisans

Machine agrave vapeur chemin de fer passage de lrsquoartisanat au capitalisme

entrepreneurial producteur organisation des entreprises

Ecole classique (Taylor Fayol Weber) approche meacutecaniste bureaucratie

hieacuterarchie commandement fonctions et speacutecialisation laquo OS T raquo (organisation

scientifique du travail) organisation source de pouvoir rationaliteacute des individus bases

du management

Deacuteveloppement du capitalisme manageacuterial Electriciteacute peacutetrole puis communications

et information Consommation de masse mondialisation preacuteoccupations

eacutenergeacutetiques et environnementales 3 peacuteriodes

- standardisation grandes entreprises industrielles

- industries de consommation 30 glorieuses marketing multinationales protection

sociale

- deacutereacuteglementation monteacutee des services pays eacutemergents mondialisation et nouvelle

eacuteconomie (internet)

Ecole des relations humaines prise en compte de lrsquoindividu des motivations styles

de direction

Ecole neacuteo-classique et post-classique deacutecentralisation coordonneacutee DPO

management participatif zeacutero deacutefaut flux tendus

Approche systeacutemique partition de lrsquoentreprise eacutetude des interactions feacutedeacuteration

vers lrsquoobjectif controcircle et ajustement

Theacuteories de la deacutecision rationaliteacute limiteacutee contribution reacutetribution coalitions

Ecole socio-technique recherche de compromis technologie organisation

enrichissement des tacircches autonomie des groupes

Approche sociologique effets sociaux du travail jeux de pouvoir dans lrsquoentreprise

reacutegulation sociale

Theacuteories de la contingence facteurs contingents adaptation agrave lrsquoenvironnement

configurations organisationnelles

Theacuteories de la firme controcircle manageacuterial droits de proprieacuteteacute relation drsquoagence

Theacuteories contractualistes firme nœud de contrats coucircts de transaction

opportunisme externalisation internalisation

Approche eacutevolutioniste eacutecologie des organisations modegravele eacutevolutioniste

contraintes de sentier

Approche par les ressources valorisation des ressources compeacutetences cleacutes

apprentissage organisationnel

(Classement simplifieacute)

Initiation au management copy CRCF ndash J Sornet Page 14 48

ELEMENTS DE CORRIGE IM

IM1 Commenter la deacutefinition du management par la norme ISO et le manager de Mintzberg

Efficient = optimum avec les moyens disponibles

ISO (management objectifs) (manager moyens) HM

IM2 Le leader entraicircne naturellement derriegravere lui Le manager nrsquoest pas toujours leader

(mecircme si crsquoest souhaitable) Le leader nrsquoest pas toujours manager (plutocirct notion individuelle)

Leadership = faculteacute de diriger conjugaison drsquoune autoriteacute naturelle ou drsquoun savoir-faire

acquis drsquoune capaciteacute agrave entraicircner des personnes ou des groupes et drsquoune leacutegitimiteacute

statutaire (de position)

IM3 Compleacuteter le tableau ci-dessous en analysant chaque action preacutesenteacutee Faire ensuite

ressortir les domaines niveaux ou techniques de management pouvant ecirctre mobiliseacutes pour

chaque situation

Satisfaction client

Implication du personnel

Processus systegraveme

Ameacutelioration continue

Deacutecision efficace

Recherche de valeur

Image entreprise

Liaisons

Information

Reacutepartition ressources

Reacutegulation

Neacutegociation

Leadership

Initiation au management copy CRCF ndash J Sornet Page 15 48

Caracteacuteristiques

de lrsquoaction

- reacutepeacutetition

- risque

- normes

- ampleur

Prise de

deacutecision

- opeacuteration

- direction

- deacutelai

Informations

neacutecessaires

- nature

- origine

- deacutelai

obtention

Cleacute pour la

reacuteussite

Intervention

exteacuterieure

possible

Assurer la

restauration du

soir

(Restaurant

familial)

Technique

(fabrication)

Vente (terrain)

Appros

Reacutepeacutetitive

(quot)

Risque faible

Normes

drsquohygiegravene

Faible

Opeacuterationnelle

Geacuterant

responsable

Rapide (qq

jours menu et

appros)

Nombre de

couverts

Tarifs usuels

Calendrier

(fecirctes)

Clients docs

divers

expeacuterience

Qq jours

Varieacuteteacute menu

Plats phares

Accueil

Appros

Tarification

Vins

Gestion

congeacutelation

Qualiteacute cuisine

Fournisseurs

Extra

Publiciteacute

Construire un

viaduc

(autoroute)

Technique

Organisation

Appros

Uniteacute (ou peu)

Eleveacute (financier

technique)

Architecture

Eleveacutee

Direction

(aleacuteas)

Opeacuterationnelle

(conduite

chantier)

Immeacutediat agrave qq

semaines

Plans

plannings

Qualifications

Meacuteteacuteo

Disponibiliteacutes

Bureau eacutetudes

Qq sem agrave 24h

Techniciteacute

Appros

Qualifications

Preacutevision

GRH

Contrat juste

SS traitants

Organismes

certificateurs

Controcircle

client

Certifier les

comptes

annuels drsquoun

groupe national

(cabinet

drsquoaudit)

Technique

Relation client

Gestion des

connaissances

Annuelle

Moyen

Regravegles

comptables

fiscales

Moyenne (selon

importance du

cabinet)

Opeacuterationnelle

Qq jours agrave

semaines

Comptable

Juridique

Client

Etat

Qq jours agrave

semaines

Techniciteacute

Expeacuterience

Relation client

Systegraveme info client

Siegravege

Autre cabinet

Lancer une

ligne drsquoavions

(constructeur

aeacuteronautique)

Strateacutegique

RD

Etudes

Uniteacute

Tregraves eacuteleveacute

Aeacuteronautique

Tregraves eacuteleveacutee

Direction

Qq mois agrave

anneacutees

Marcheacute

Etudes

Compagnies

Qq mois agrave

anneacutees

Concept

Outil industriel

Coucirct exploitation

Tarif

Fiabiliteacute

Deacutelaisconcurrence

SI simulation

SS traitants

Bureaux

drsquoeacutetudes

speacutecialiseacutes

Compagnies

Conseils

Reacuteduire la

capaciteacute de

production

(groupe

industriel)

Strateacutegique

RH

Communication

Production

Uniteacute

Moyen

Leacutegislation

(dont RH)

Eleveacutee

Direction

Qq mois agrave

anneacutees

Financiegravere

Industrielle

Marcheacute

Organisation

Organismes

speacutecialiseacutes

DRH

Qq mois

Communication

Connaissance des

compeacutetences

Connaissance outil

industriel

Concurrence

Portefeuille

drsquoactiviteacutes

Cabinet

drsquoorganisation

Conseils

speacutecifiques

Acqueacuterir une

entreprise

concurrente

(teacuteleacutephonie

mobile)

Strateacutegique

Marketing

Production

(reacuteseau)

Financier

Communication

Uniteacute

Tregraves eacuteleveacute

Leacutegislation

telecom

Tregraves eacuteleveacutee

Direction

Qq mois

Financiegravere

Marcheacute

Reacuteseaux

(ampleur

recouvrement

hellip)

Organisations

Interne

Racheteacutee

Sources

speacutecialiseacutees

Qq mois

Communication

Marcheacute

Cours boursiers

Cabinet

drsquoorganisation

Conseils

speacutecifiques

Initiation au management copy CRCF ndash J Sornet Page 16 48

LE MANAGEMENT EN PRATIQUE

Pour assumer sa fonction le management doit couvrir sans discontinuiteacute lrsquoensemble de

lrsquoorganisation et inteacutegrer de nombreux facteurs dont nous allons reacutesumer lrsquoessentiel

1 ndash Les fonctions et activiteacutes du management

Pour Henri Fayol la fonction drsquoadministration de lrsquoentreprise (son management) reposait sur

cinq actions preacutevoir organiser commander coordonner et controcircler (laquo PO3C raquo)

Nous distinguerons cinq activiteacutes de management

- la conception (au plus haut niveau finaliteacute but ou vocation de lrsquoorganisation

meacutetiers dimension politique de croissance hellip)

- la planification (deacutefinition des objectifs eacutecheacuteances)

- lrsquoorganisation (reacutepartition du travail choix des modes de coordination)

- le pilotage de lrsquoaction opeacuterationnelle (motivation animation encadrement

assistance)

- lrsquoeacutevaluation (controcircle des reacutesultats obtenus ajustements)

Dans chacune de ces activiteacutes des deacutecisions et des arbitrages sont neacutecessaires avec des

enjeux plus ou moins importants

Remarques

- Les cinq activiteacutes du management peuvent se retrouver agrave tout niveau de

management si lrsquoentreprise laisse une certaine autonomie de deacutecision agrave ses diffeacuterentes

uniteacutes La conception est naturellement du ressort de la direction geacuteneacuterale et des

conseils drsquoadministration mais elle peut ecirctre preacutesente pregraves du terrain (latitude laisseacutee agrave

une filiale ou agrave un magasin par exemple) De mecircme lrsquoorganisation du travail concerne

un atelier mais aussi la direction qui structure lrsquoentreprise pour assurer ses activiteacutes sa

production

- La planification deacutefinit des objectifs ou des axes strateacutegiques (choix de produits

modaliteacutes de deacuteveloppement des ventes implantations alliances hellip) et les traduit en

donneacutees de gestion preacutevisionnelles syntheacutetiques et eacutechelonneacutees dans le temps afin de

valider les objectifs et de fixer des repegraveres

- Un laquo business plan raquo (plan drsquoaffaires)est notamment lrsquoeacutequivalent de la planification

dans le cas de creacuteation drsquoentreprise ou pour la preacutesentation de tout projet drsquoactiviteacute

Les activiteacutes du management srsquoinscrivent dans des cycles qui peuvent ecirctre scheacutematiseacute

comme suit (lrsquoeacutevaluation peut entraicircner une reacutevision du pilotage de lrsquoorganisation ou des

objectifs sans que lrsquoentreprise ne soit fondamentalement remise en cause)

conception

planification

organisation

pilotage

eacutevaluation

Initiation au management copy CRCF ndash J Sornet Page 17 48

2 ndash Les contextes de management

Le management est influenceacute par son contexte qui justifie des objectifs une organisation

des meacutethodes

Par exemple lrsquoentreprise admet de nombreuses variantes selon sa taille sa forme juridique

son controcircle par lrsquoeacutetat (entreprises publiques) ou par des inteacuterecircts priveacutes Il en va de mecircme des

organismes administratifs qui peuvent deacutependre de directives nationales ou reacutegionales des

associations qui ont des activiteacutes drsquoampleur tregraves variable

21 ndash La dimension de lrsquoentreprise

La dimension drsquoune entreprise se mesure principalement en fonction de son effectif ou de

son chiffre drsquoaffaires Des seuils sont deacutefinis par divers organismes et exploiteacutes agrave des fins

statistiques ou pour la deacutetermination de certaines obligations sociales ou fiscales

(repreacutesentation du personnel cotisations hellip) Il nrsquoy a bien entendu pas de laquo barriegravere de

tailleraquo absolue conditionnant le management drsquoune entreprise

LrsquoUE preacuteconise de distinguer les micro ndash entreprises (jusqursquoagrave 9 salarieacutes) les TPE ndash tregraves petites

entreprises (moins de 20 salarieacutes) les petites entreprises (moins de 50) et les moyennes

entreprises (de 50 agrave 250) Cependant les PME sont parfois situeacutees entre 10 et 500 salarieacutes

Remarques

- en France environ 40 des entreprises emploient de 1 agrave 50 salarieacutes (ce qui repreacutesente

plus de 50 des emplois) et 59 nrsquoen ont aucun

le pays compte environ 2 600 000 entreprises dont moins de 1 ont 250 employeacutes et

plus

- ancienneteacute et taille de lrsquoentreprise sont lieacutees si lrsquoon eacutecarte les restructurations et autres

eacutevolutions drsquoentreprises existantes

La dimension de lrsquoentreprise a une influence sur lrsquoorganisation et le laquo style raquo de son

management

- les PME sont souvent entrepreneuriales (les dirigeants eacutegalement apporteurs de capitaux

sont totalement engageacutes dans la marche de lrsquoentreprise) Elles ont une gestion flexible peu

formaliseacutee plus qualitative que quantitative Les PME sont freacutequemment focaliseacutees sur un seul

type drsquoactiviteacute Pour ne pas alourdir leur structure elles ont tendance agrave sous-traiter les

activiteacutes speacutecialiseacutees ne correspondant pas agrave leur meacutetier de base

- les grandes entreprises sont manageacuteriales (les dirigeants sont nommeacutes par les actionnaires

en raison de leurs compeacutetences) et moins reacuteactives

22 ndash Le type de production

On distingue industrie (production de biens mateacuteriels ou pour le moins de produits visibles ndash

comme un seacutejour touristique ou un film) et services (fourniture drsquoune prestation immateacuterielle)

Le type de production influence en principe le management de lrsquoentreprise

- lrsquoindustrie neacutecessite (si lrsquoon excepte lrsquoartisanat) un investissement relativement important

une organisation productive stable capable de reacutealiser plusieurs fois des produits identiques

(exemple un modegravele de reacutefrigeacuterateur) ou du moins similaires (exemple un bacirctiment) Le

produit de lrsquoindustrie consomme des matiegraveres et il doit geacuteneacuteralement ecirctre distribueacute jusqursquoau

client

- la production de services peut se satisfaire drsquoun investissement tregraves reacuteduit et neacutecessite un

contact permanent avec le client

Toutefois la standardisation des services et le deacuteveloppement des reacuteseaux informatiques

rapprochent la production de services de celle des biens industriels

- la production drsquoun service reacutepeacutetitif et technique peut imposer une structure lourde et une

organisation tregraves formaliseacutee (voir les grandes socieacuteteacutes drsquoaudit ou de conseil informatique)

Initiation au management copy CRCF ndash J Sornet Page 18 48

- certains services peuvent ecirctre fournis agrave distance sans contact direct avec le client et

distribueacutes par reacuteseau (tenue de comptabiliteacute affacturage gestion clientegravele centre drsquoappel

hellip)

Remarque les services repreacutesentent 75 de lrsquoactiviteacute eacuteconomique franccedilaise

23 ndash La nature de lrsquoorganisation

Les organisations publiques franccedilaises (administrations centrales collectiviteacutes territoriales

hocircpitaux hellip) repreacutesentent une part importante de lrsquoactiviteacute (environ 30 des emplois) La

fonction publique regroupe des organisations aux finaliteacutes diverses et qui ont des problegravemes

de gestion similaires agrave ceux des entreprises auxquelles elles peuvent emprunter des principes

de management Notamment

- pour controcircler les coucircts et assurer la qualiteacute des services

- pour communiquer avec les administreacutes ou les usagers

- pour motiver les personnels et geacuterer les ressources humaines

La transposition directe des techniques de gestion et de management nrsquoest cependant pas

toujours possible car

- la comptabiliteacute publique obeacuteit agrave des regravegles speacutecifiques (proceacutedure budgeacutetaire

notamment)

- le laquo client raquo ne paye pas toujours la prestation du moins directement

- la concurrence est parfois inexistante

- les grandes administrations centraliseacutees sont soumises agrave des choix politiques geacuteneacuteraux

parfois sans connexion eacutevidente avec les besoins opeacuterationnels

- le statut des personnels et les grilles de salaires limitent les possibiliteacutes de gestion des

ressources humaines

Remarque la LOLF (loi organique relative aux lois de finances) est entreacutee en vigueur en

2006 Elle alloue des moyens budgeacutetaires en fonction de programmes et remplace la

reconduction automatique de 90 des budgets Cette reacuteforme se heurte toutefois agrave la

lourdeur des grands ministegraveres ougrave la complexiteacute des activiteacutes est difficile agrave

appreacutehender et ougrave des inerties culturelles peuvent exister agrave tout niveau

Les associations loi de 1901 peuvent avoir une activiteacute comparable agrave celle de grandes

entreprises (voir par exemple les associations de santeacute ou professionnelles) et leur

management est alors similaire malgreacute lrsquoabsence de but lucratif (les beacuteneacutefices ne sont pas

distribuables) Elles ont drsquoailleurs en France un poids eacuteconomique important (elles emploient

environ 1 600 000 salarieacutes)

Cependant lrsquoadheacutesion agrave un systegraveme de valeurs fondateur de lrsquoassociation ou la limite de

lrsquoautoriteacute (quand un volant de beacuteneacutevoles important participe agrave lrsquoactiviteacute) peut introduire des

nuances

- le renforcement des objectifs socieacutetaux

- la faiblesse des relations hieacuterarchiques

- des contraintes de gestion du temps des beacuteneacutevoles

- des modaliteacutes particuliegraveres de recrutement et de motivation des dirigeants

24 ndash Les facteurs contingents

La theacuteorie de la contingence montre qursquoune structure drsquoentreprise nrsquoest efficace que dans

une situation deacutetermineacutee et qursquoil nrsquoexiste que des solutions de management construites dans

un contexte preacutecis

Le management doit ainsi srsquoadapter agrave des facteurs contingents qui ne peuvent ecirctre

controcircleacutes du moins agrave bregraveve eacutecheacuteance Ces facteurs sont par exemple

- lrsquoancienneteacute de lrsquoentreprise (plus elle est ancienne plus lrsquoentreprise a tendance agrave reacutepeacuteter

des comportements eacuteprouveacutes)

Initiation au management copy CRCF ndash J Sornet Page 19 48

- la taille de lrsquoentreprise (la grande entreprise a une composante administrative plus

deacuteveloppeacutee)

- le systegraveme de production (tregraves standardiseacute complexe automatiseacute hellip)

- lrsquoenvironnement

3 ndash Le management et les parties prenantes

Lrsquoentreprise a pour vocation premiegravere de mettre des produits agrave disposition de ses clients en

reacutealisant un profit Pour y arriver elle doit aussi satisfaire ses parties prenantes salarieacutes

actionnaires fournisseurs hellip

Est partie prenante agrave lrsquoentreprise laquo tout groupe ou individu qui peut ecirctre affecteacute ou est

affecteacute par les buts de lrsquoorganisation hellip raquo (Freeman ndash 1984)

Les parties prenantes attendent agrave des degreacutes divers de profiter drsquoune creacuteation de valeur en

provenance de lrsquoentreprise qui doit reacutepondre agrave ces attentes pour assurer sa peacuterenniteacute ou

favoriser son deacuteveloppement

On distingue les parties prenantes primaires ou principales qui sont essentielles agrave lrsquoentreprise

et qui ont geacuteneacuteralement une relation formelle avec elle (clients associeacutes et actionnaires

precircteurs salarieacutes fournisseurs collectiviteacutes) et les parties prenantes secondaires dont

lrsquoinfluence est diffuse (groupes de pression associations meacutedias instances europeacuteennes

agences de notation hellip)

Remarque la consideacuteration de lrsquoensemble des parties prenantes (laquo stakeholders raquo - les

deacutepositaires) fait contrepoids agrave lrsquoimportance accordeacutee aux seuls actionnaires

(laquo shareholders raquo)

Les organisations nrsquoayant pas drsquoobjectif de profit doivent aussi satisfaire leurs parties

prenantes apporter un service aux usagers dans les meilleures conditions eacuteconomiques

limiter un budget assurer la qualiteacute des relations avec les fournisseurs hellip

Dans cette optique le management doit organiser lrsquoaction de faccedilon agrave eacutequilibrer des forces

parfois divergentes

- le contexte fait pression sur lrsquoorganisation contrainte agrave optimiser ses reacutesultats

- lrsquoorganisation cherche par son action agrave assurer sa peacuterenniteacute son deacuteveloppement (en

reacutealisant des profits dans le cas de lrsquoentreprise) et agrave satisfaire ses parties prenantes

- le management agit en pilotant les actions pour contrebalancer la pression du contexte

Actions de

lrsquoorganisation

Management Contexte

Parties

prenantes

Initiation au management copy CRCF ndash J Sornet Page 20 48

APPLICATIONS MP

MP1 Deacutefinir contingent gestion budgeacutetaire

MP2 Deacuteterminer les parties prenantes drsquoun hocircpital public et leurs principales attentes

Mecircme question pour les organisations suivantes

- SNCF (entreprise publique)

- Peugeot

- MAIF (mutuelle drsquoassurance)

MP3 En les situant dans le cycle des activiteacutes du management trouver les actions agrave mener

dans les situations suivantes

- baisse de 10 des ventes dans une entreprise industrielle (produits meacutenagers le reacuteseau de

distribution vient drsquoecirctre reacuteorganiseacute)

- idem dans une entreprise de vente par correspondance soumise agrave la concurrence internet

(les ventes stagnaient depuis six mois malgreacute les efforts promotionnels)

- augmentation des deacutelais drsquoattente des consultations dans une clinique (lrsquohocircpital voisin a

fermeacute son service drsquourgences)

Initiation au management copy CRCF ndash J Sornet Page 21 48

ELEMENTS DE CORRIGE MP

MP1 Deacutefinir (dans le contexte drsquoune entreprise) contingent gestion budgeacutetaire

Contingent = imposeacute par lrsquoexteacuterieur Contingence = effet du hasard de la rencontre de

plusieurs eacuteveacutenements indeacutependants (variables explicatives que lrsquoon ne peut influencer)

Gestion budgeacutetaire = technique drsquoadministration des entreprises srsquoappuyant sur des

preacutevisions dont on deacuteduit apregraves accord des responsables des attributions de moyens sur une

dureacutee limiteacutee Une analyse reacuteguliegravere des eacutecarts entre preacutevisions et reacutealisations permet ensuite

le pilotage des activiteacutes Le budget est un cadre incitatif

La laquo planification budgeacutetaire raquo consiste agrave traduire en budgets une planification strateacutegique

avec systegraveme de reporting

MP2 Deacuteterminer les parties prenantes drsquoun hocircpital public et leurs principales attentes

Mecircme question pour les organisations suivantes

- SNCF (entreprise publique)

- Peugeot

- MAIF (mutuelle drsquoassurance)

Hocircpital

- patients (qualiteacute des soins)

- CNAM (baisse des coucircts)

- collectiviteacute locale (service aux administreacutes)

- eacutetat (ameacutenagement du territoire maicirctrise des budgets optimisation)

- employeacutes (salaire conditions de travail et satisfaction)

- fournisseurs ndash pharmacie autres (CA paiement reacutegulier)

- associations de patients (qualiteacute proximiteacute des soins)

SNCF

- usagers et associations drsquousagers (proximiteacute reacutegulariteacute prix du service)

- reacuteseau ferreacute de France (optimisation des lignes paiement adapteacute)

- fournisseurs (CA paiement reacutegulier)

- employeacutes (salaire conditions de travail seacutecuriteacute de lrsquoemploi)

- eacutetat (ameacutenagement du territoire)

- collectiviteacutes locales (service)

Peugeot

- clients (qualiteacute prix SAV relation commerciale)

- fournisseurs (CA reacutegulariteacute de lrsquoactiviteacute)

- employeacutes (salaire conditions de travail seacutecuriteacute de lrsquoemploi)

- eacutetat (taxes)

- collectiviteacute locale (emploi dynamisation eacuteconomique preacuteservation de lrsquoenvironnement)

- associations de protection de lrsquoenvironnement (activiteacute propre baisse des eacutemissions

nouvelles eacutenergies)

MAIF

- socieacutetaires (protection relation assureur tarif mesureacute)

- professionnels de lrsquoautomobile et autres (agreacutement marge de manœuvre reacuteparations tarifs

eacuteleveacutes)

- fournisseurs (CA paiement reacutegulier)

- eacutetat (taxes engagement pour la seacutecuriteacute)

- employeacutes (salaire conditions de travail seacutecuriteacute de lrsquoemploi)

Initiation au management copy CRCF ndash J Sornet Page 22 48

MP3 En les situant dans le cycle des activiteacutes du management trouver les actions agrave mener

dans les situations suivantes

- baisse de 10 des ventes dans une entreprise industrielle (produits meacutenagers le reacuteseau de

distribution vient drsquoecirctre reacuteorganiseacute)

Adapter le pilotage motiver cadrer si insuffisant retoucher une organisation deacutefectueuse

- idem dans une entreprise de vente par correspondance soumise agrave la concurrence internet

(les ventes stagnaient depuis six mois malgreacute les efforts promotionnels)

Voir pilotage et organisation si une eacutevolution du meacutetier a deacutejagrave eacuteteacute initialiseacutee Sinon re-

conception (adaptation au nouveau contexte) puis planification et reacuteorganisation

- augmentation des deacutelais drsquoattente des consultations dans une clinique (lrsquohocircpital voisin a

fermeacute son service drsquourgences)

Organisation Si insuffisant planification (nouveaux objectifs)

Initiation au management copy CRCF ndash J Sornet Page 23 48

ORGANISATION ET PROCESSUS

La performance de lrsquoentreprise deacutepend de son organisation et de son aptitude agrave produire

aux meilleures conditions Nous allons montrer comment organisation formelle et processus

de production peuvent contribuer agrave cette performance

1 ndash Vers lrsquooptimum

11 ndash Les eacuteconomies occidentales jusqursquoaux anneacutees 70

Jusqursquoen 1945 le principal problegraveme des entreprises eacutetait de produire des biens en quantiteacute

suffisante agrave un prix compatible avec le marcheacute Les grandes entreprises se sont multiplieacutees et

la standardisation a permis de reacuteduire les coucircts (exemple deacuteveloppement de Ford et de la

production agrave la chaicircne de 1908 agrave 1920 qui a permis une baisse du prix des voitures des 23)

On parle de laquo production pousseacutee vers le marcheacute raquo

Cette croissance de la production peu reacuteguleacutee a eacuteteacute marqueacutee par des surproductions en

1910 et 1920 puis par la crise de 1929 qui a prolongeacute ses effets jusqursquoagrave la guerre

De 1945 agrave 1975 environ (les laquo trente glorieuses raquo) la reconstruction la croissance de la

consommation de masse de nouvelles technologies et les eacutechanges internationaux

alimentent lrsquoeacuteconomie La standardisation srsquoeacutetend aux biens de consommation dont les

coucircts baissent fortement et de nouvelles reacutegulations sociales permettent une eacutevolution sans

heurt des revenus La saturation de certains marcheacutes conduit dans les anneacutees 60 agrave la

deacutemarche laquo marketing raquo et agrave la diffeacuterenciation des produits Le produit est laquo dirigeacute par le

marcheacute raquo mais les entreprises conservent une organisation assez classique et les plus grosses

srsquointernationalisent

12 ndash Lrsquoexpeacuterience japonaise et ses prolongements

Tregraves tocirct apregraves la guerre dans un Japon appauvri le constructeur automobile Toyota a ducirc

faire face agrave une restriction du marcheacute des moyens financiers et productifs et des

approvisionnements La firme a donc innoveacute dans un nouveau systegraveme de production

chassant les laquo gaspillages raquo (temps drsquoattente transports stocks deacutefauts hellip) consideacuterant que

seule la fabrication vendable creacutee de la valeur

Toyota srsquoorganise pour fabriquer la quantiteacute et la qualiteacute de produits juste neacutecessaires agrave la

satisfaction des clients la production est laquo tireacutee par le marcheacute raquo La mise en place de ce

systegraveme qui integravegre les fournisseurs ne sera acheveacutee que dans le milieu des anneacutees 70

En 1973 la hausse du peacutetrole inaugure un ralentissement de la croissance des eacuteconomies

occidentales La concurrence accrue provoque alors un inteacuterecirct pour le systegraveme deacuteveloppeacute

au Japon La production au plus juste se deacuteveloppe ainsi dans lrsquoindustrie automobile agrave partir

des anneacutees 80 et elle se reacutepand encore maintenant dans drsquoautres secteurs

Cette approche qui vise un objectif de zeacutero stock et zeacutero deacutefaut impose la maicirctrise de laquo bout

en bout raquo des processus de production et leur ameacutelioration

Initiation au management copy CRCF ndash J Sornet Page 24 48

2 ndash Organiser lrsquoentreprise

21 ndash Direction et organisation

Diriger une entreprise neacutecessite de lrsquoorganiser (de reacutepartir les tacircches) pour qursquoelle puisse

atteindre ses objectifs Lrsquoorganisation permet de satisfaire un marcheacute en tirant parti des

capaciteacutes actuelles de lrsquoentreprise tout en preacuteparant lrsquoavenir

Lrsquoorganisation reacutesulte freacutequemment drsquoun compromis entre des objectifs situeacutes agrave des niveaux

et des eacutecheacuteances diffeacuterents

Exemples

- le leader des chaises roulantes peut tirer profit de sa structure productive et de son

savoir faire pour entrer sur le marcheacute de la bicyclette eacutelectrique

- ecirctre parfaitement structureacute pour alimenter 90 du marcheacute des disquettes ne preacutepare

pas lrsquoavenir

- srsquoorganiser pour conqueacuterir le marcheacute des tire-bouchons eacutelectriques dans les deux ans

perd de son sens si cela altegravere les moyens neacutecessaires agrave la production drsquoappareils

manuels ancienne mais vitale dont la diminution agrave court terme risque de nuire agrave la

solvabiliteacute de lrsquoentreprise et de la conduire agrave la cessation de paiement

22 ndash Lrsquoorganisation fonctionnelle

La majoriteacute des entreprises adopte une laquo organisation fonctionnelle raquo (celle qui est visible

dans les organigrammes) ougrave des regroupements de personnels et drsquoeacutequipements se font

selon un modegravele hieacuterarchique (laquo line raquo) dans des uniteacutes des services ou des deacutepartements

speacutecialiseacutes Cette organisation peut se deacutecliner agrave lrsquointeacuterieur des divisions des grandes

entreprises quand elles scindent leur activiteacute par zone geacuteographique type drsquoactiviteacute

cateacutegorie de clients hellip

Remarque le terme laquo fonction raquo deacutesigne un rocircle particulier dans le fonctionnement de

lrsquoentreprise

Lrsquoorganisation fonctionnelle diffeacuterencie les activiteacutes de lrsquoentreprise en les regroupant par

meacutetier pour utiliser au mieux les compeacutetences et les moyens (meilleur rendement par la

speacutecialisation lrsquoeacutechange de compeacutetences dans une mecircme uniteacute ou gracircce agrave des eacuteconomies

drsquoeacutechelle)

23 ndash La notion de processus de production

Un processus de production se deacutefinit par la succession drsquoactiviteacutes permettant de satisfaire

un client en transformant des ressources (mateacuterielles financiegraveres humaines) en un produit

bien ou service Le processus doit creacuteer une valeur reconnue par le client

Un processus peut servir un client interne agrave lrsquoentreprise (par exemple en produisant un

composant intervenant dans plusieurs produits ou par la maintenance des machines) aussi

bien qursquoun client final On distingue usuellement

- les processus opeacuterationnels (ou maicirctres) aussi appeleacutes processus meacutetier (business process)

qui satisfont directement les clients finaux (conception et fabrication de produits vente hellip)

- les processus de support et de management (geacuterer les ressources humaines geacuterer

lrsquoinformation geacuterer les ressources financiegraveres hellip) qui ont les processus opeacuterationnels comme

clients

Toutes les actions internes agrave une organisation peuvent srsquointeacutegrer dans des processus qui

conditionnent directement ou indirectement la capaciteacute de lrsquoorganisation agrave satisfaire le

client final ou lrsquousager

Initiation au management copy CRCF ndash J Sornet Page 25 48

Aborder le fonctionnement de lrsquoentreprise par ses processus (approche processus) permet

de mettre en eacutevidence les chaicircnes drsquoactiviteacutes qui conduisent aux produits leurs

dysfonctionnements leurs coucircts la formation des deacutelais et la souplesse (la flexibiliteacute)

disponible pour satisfaire la clientegravele finale Lrsquoameacutelioration des processus a un impact visible

et direct sur chaque produit proposeacute aux clients

Lrsquoapproche processus provoque une eacutevolution de la faccedilon de travailler

- en faisant peacuteneacutetrer la laquo voix du client raquo au plus profond de lrsquoentreprise (et plus seulement

dans les services commerciaux et marketing)

- en mettant en eacutevidence des possibiliteacutes de rationalisation (par regroupement ou impartition

de certaines activiteacutes)

Remarque lrsquoapproche par les activiteacutes et les processus est agrave lrsquoorigine de la meacutethode

de deacutetermination des coucircts laquo ABC raquo - activity based costing

24 ndash Processus et fonctions

Le processus est transversal Il enchaicircne des activiteacutes qui traversent lrsquoentreprise en particulier

les services ou les deacutepartements drsquoune organisation fonctionnelle

Exemple

La division du travail par fonctions induit une charge de coordination pour assurer le

deacuteroulement du processus Elle peut geacuteneacuterer des attentes des erreurs ou des conflits drsquointeacuterecirct

(lrsquoobservation montre que des dysfonctionnements sont tregraves souvent constateacutes lors du

passage drsquoun service agrave un autre)

Organisation fonctionnelle et approche processus visent toutes deux un optimum

eacuteconomique mais leurs logiques sont diffeacuterentes

- le processus vise la satisfaction des clients (prix qualiteacute deacutelais service)

- le deacutecoupage fonctionnel cherche agrave optimiser les moyens (maximiser lrsquoeffet drsquoexpeacuterience

partager des infrastructures profiter de pocircles de compeacutetences hellip) Il apporte une ossature

hieacuterarchique stable souvent indispensable

Organisation fonctionnelle et approche processus sont donc compleacutementaires dans la

majoriteacute des cas et doivent ecirctre combineacutees judicieusement

APPLICATIONS OP

OP1 Deacutefinir flexibiliteacute systegraveme impartition

OP2 Citer huit exemples drsquoinformations essentielles pour optimiser un processus de

fabrication

Direction

Deacutepartement

commercial

(C)

Deacutepartement

administratif et

financier (AF)

Deacutepartement

Etudes (E)

Deacutepartement

Production (P)

Activiteacute

C-x Activiteacute

AF-x Activiteacute

E-x

Activiteacute

P-x

Processus x

Clie

nt

Initiation au management copy CRCF ndash J Sornet Page 26 48

OP3 Deacutegager les principes du toyotisme preacutesenteacute ci-dessous En quoi ce systegraveme est-il

initiateur de lrsquoapproche processus

Taiichi Ohno et le Toyotisme

1 - Extrait drsquoun article de Jacques BARRAUX - 1993 - LExpansion

Taiichi Ohno (1912 ndash 1990) hellip ne se prenait pas pour un visionnaire mais en imposant une

nouvelle faccedilon de produire il a reacuteinventeacute le management hellip tout le monde a entendu parler

des mots qui ont populariseacute le toyotisme dont il est le pegravere le juste-agrave-temps hellip Autant

doutils conccedilus pour lrsquoautomobile et qui ont aujourdhui une application universelle

hellip Taiichi Ohno jeune ingeacutenieur entre chez Toyota alors simple constructeur de machines

textiles Degraves 1926 apparaicirct la notion de jidoka hellip cest lart de transfeacuterer de lintelligence aux

machines pour mieux libeacuterer lintelligence des hommes Tout le contraire du taylorisme qui

juge la machine moins impreacutevisible que lhomme En 1933 Toyota se lance dans lautomobile

en sinspirant des meacutethodes ameacutericaines Mais en 1935 agrave loccasion dun voyage aux Etats-

Unis leacutetat-major de lentreprise revient fascineacute de sa visite dans un supermarcheacute La notion

de juste-agrave-temps va naicirctre de lobservation dune grande surface un lieu ougrave les clients ne

prennent que ce dont ils ont besoin et ougrave les rayons sont reacuteapprovisionneacutes pour compenser

les quantiteacutes preacuteleveacutees Ainsi le systegraveme Toyota est-il deacutejagrave dans la tecircte de ses dirigeants avant

mecircme la Seconde Guerre mondiale un demi-siegravecle avant la reacutevolution informatique et la

segmentation intensive des marcheacutes

hellip des esprits curieux comme Franccedilois Dalle en France tombent alors sous le charme des

formules et des paraboles de Taiichi Ohno En voici deux eacutechantillons

Penser agrave lenvers Cela signifie combattre les ideacutees reccedilues En lespegravece il sagit du fordisme et

du taylorisme Ohno ne croit pas agrave la planification aux effets deacutechelle et dexpeacuterience Il

propose un systegraveme industriel agrave lenvers qui permette de diversifier les produits et de les

fabriquer en petites quantiteacutes Nous ne devons plus ecirctre des paysans qui accumulent des

stocks mais des chasseurs On nimpose pas loffre On traque la demande et on la gegravere en

continu

Que les valleacutees soient hautes et les montagnes peu eacuteleveacutees Plutocirct que de concentrer tous

les efforts sur une production agrave un moment donneacute mieux vaut se doter de structures flexibles

permettant de passer agrave tout instant dune seacuterie agrave une autre Il faut eacuteviter les ruptures et les

secousses aplanir les cycles entretenir des flux reacuteguliers dactiviteacutes diversifieacutees Ce qui

implique de ne pas enfermer les hommes et les eacutequipements dans des speacutecialisations trop

eacutetroites

La flexibiliteacute le travail en groupe le refus de la dictature des machines la polyvalence et

surtout lattention constante aux signaux eacutemis par le marcheacute nappartiennent plus au

toyotisme Ces notions sont les fondements du nouvel art dorganiser de vendre et de

produire dans lindustrie comme dans les services hellip

2 - Quelques notions cleacutes

Taiichi Ohno a imagineacute la meacutethode des laquo cinq pourquoi raquo qui consiste agrave se poser cinq fois de

suite la question laquo pourquoi raquo sur le mecircme sujet de faccedilon agrave deacutecouvrir la veacuteritable cause

drsquoun problegraveme Cette meacutethode peut ecirctre appliqueacutee agrave tous les niveaux et permettre

notamment aux agents de fabrication de proposer de veacuteritables ameacuteliorations de la

production

La recherche de la qualiteacute totale (pas de deacutefaut des produits pas de rebuts pas de deacutefaut

des processus) accompagne la deacutemarche de Toyota La qualiteacute a un coucirct compenseacute par

des ventes accrues par lrsquoeacuteconomie des mesures palliatives aux deacutefauts

Initiation au management copy CRCF ndash J Sornet Page 27 48

Fiche OP1 ndash Benchmarking et processus

Le laquo benchmarking raquo consiste agrave comparer le fonctionnement de plusieurs systegravemes pour en

faire notamment ressortir les meilleures pratiques (laquo best practices raquo) Cette technique est

utiliseacutee depuis les anneacutees 80 pour ameacuteliorer la performance des entreprises Elle impose agrave

lrsquoentreprise drsquoeacutevaluer et de remettre en question ses propres modes de fonctionnement afin

de les faire eacutevoluer agrave la lueur de ce qui se fait ailleurs

Le benchmarking permet drsquoameacuteliorer les processus agrave moindre risque en fixant des objectifs

baseacutes sur des faits et donc plus facilement accepteacutes

Une classification des processus en tant que base de reacuteflexion a eacuteteacute eacutetablie aux USA par

lrsquolaquo International Benchmarking Clearinghouse raquo de lrsquoAPQC (american productivity and

quality center) en collaboration avec plusieurs dizaines drsquoentreprises

Elle se reacutesume ainsi

Le terme laquo reengineering raquo (la re-conception ou laquo reacuteingeacutenieacuterie raquo) des processus deacutesigne un

projet drsquoameacutelioration radicale des performances (de 20 agrave 50 ou plus) Il neacutecessite une

parfaite adheacutesion de la direction la constitution drsquoune petite eacutequipe de projet brillante

connaissant parfaitement les activiteacutes de lrsquoentreprise et il peut inclure un benchmarking

Le reengineering provoque geacuteneacuteralement la reacuteduction du nombre de niveaux hieacuterarchiques

(laquo delayering raquo) et lrsquoaccroissement du pouvoir de deacutecision des employeacutes (laquo empowerment raquo

ou laquo empouvoirement raquo) Bien qursquoy conduisant parfois il ne doit pas ecirctre confondu avec la

reacuteduction des activiteacutes (laquo downsizing raquo ou restructuration) et lrsquoexternalisation (laquo outsourcing raquo)

Pro

ce

ssu

s

op

eacutera

tio

nn

els

Pro

ce

ssu

s d

e m

an

ag

em

en

t e

t d

e

sup

po

rt

1 ndash

Comprendre

le marcheacute et

les clients (besoins

satisfaction)

2 ndash

Deacutevelopper

vision et

strateacutegie (contexte

concurrence)

3 ndash

Creacuteer

produits

services

processus

(concevoir

ameacuteliorer)

4 ndash

Marketing et

vente

5 ndash

Produire et

livrer (industrie

dont

ameacutelioration

processus)

6 ndash

Produire et

livrer (services)

7 ndash

Facturer et

servir les

clients (apregraves-

vente

reacuteclamations)

8 ndash Deacutevelopper et geacuterer les ressources humaines

9 ndash Geacuterer les systegravemes drsquoinformation

10 ndash Geacuterer les ressources financiegraveres et les actifs

11 ndash Appliquer un programme environnemental

12 ndash Geacuterer les relations exteacuterieures (actionnaires banques lois relations publiques hellip)

13 ndash Geacuterer lrsquoameacutelioration et le changement (eacutevaluer mesurer motiver qualiteacute totale)

Initiation au management copy CRCF ndash J Sornet Page 28 48

Fiche OP2 ndash Lrsquoorganisation par processus

Lrsquoeacutevolution drsquoune organisation aux activiteacutes reacutepeacutetitives vers lrsquoapproche processus est

geacuteneacuteralement progressive et se met en place par paliers

La mise en œuvre drsquoun veacuteritable management par processus doit ecirctre preacuteceacutedeacutee quand

lrsquoactiviteacute de lrsquoentreprise est complexe drsquoun recensement (une laquo cartographie des

processus raquo) pour mettre en eacutevidence les processus ou les familles de processus cleacutes critiques

pour le succegraves de lrsquoentreprise ougrave les efforts seront prioritaires

Des responsables de processus (laquo process owners raquo) sont ensuite deacutesigneacutes

Le responsable doit concevoir ses processus puis apregraves leur mise en œuvre assurer les

coordinations neacutecessaires les ameacuteliorer et les repreacutesenter aupregraves de la direction

Quand une structure par processus est mise en place des opeacuterateurs exeacutecutants

preacuteceacutedemment regroupeacutes dans les fonctions peuvent ecirctre affecteacutes aux processus et

drsquoanciens responsables de fonctions peuvent devenir des experts au service des processus

Lrsquoorganisation par processus peut imposer un degreacute eacuteleveacute drsquointeacutegration des activiteacutes donc

une polyvalence accrue des personnels et une reacuteduction des niveaux hieacuterarchiques

Elle neacutecessite pour le moins des compeacutetences eacutelargies au niveau des responsables de

processus (organisation administration technique hellip) dont le nombre doit rester limiteacute

(quelques dizaines au plus)

Sauf dans de tregraves petites structures lrsquoorganisation par processus se plaque geacuteneacuteralement sur

une structure plus classique

Initiation au management copy CRCF ndash J Sornet Page 29 48

ELEMENTS DE CORRIGE OP

OP1 Deacutefinir

Flexibiliteacute = adaptation au besoin (horaire variable chaicircnes robotiseacutees)

Systegraveme = ensemble organiseacute dans un but boicircte noire (sanguin nerveux meacutetrique laquo D raquo)

Impartition = sous-traitance ou externalisation (seacuteparation) drsquoactiviteacutes faire appel agrave des

partenaires plutocirct que faire soi-mecircme

OP2 Citer huit exemples drsquoinformations essentielles pour orienter lrsquooptimisation drsquoun processus

Montant des stocks (approvisionnements et produits finis)

Temps drsquoattente

Taux drsquoactiviteacute des ateliers

Rebuts

Deacutelai de production

Taux de reacuteclamations clients (qualiteacute)

Temps passeacutes en retouches finales

Turn over

Nombre drsquoarrecircts maladie

Accidents du travail

Dureacutee des arrecircts machines

OP3 Deacutegager les principes du toyotisme preacutesenteacute dans la fiche 31 En quoi ce systegraveme

repose trsquoil sur lrsquoapproche processus

Produire la quantiteacute juste neacutecessaire (agrave la demande) donc eacuteviter les stocks

Flexibiliteacute intelligence des chaicircnes de production

Qualiteacute (eacuteviter le coucirct de la non-qualiteacute)

La notion de processus est implicite ainsi que la chaicircne de valeur client

Initiation au management copy CRCF ndash J Sornet Page 30 48

DEFIS ET TENDANCES DU MANAGEMENT

Les meacutethodes de management se deacuteveloppent pour affronter le contexte eacuteconomique

Ce chapitre preacutesente les deacutefis auxquels le management contemporain doit faire face

1 ndash Lrsquoeacutevolution eacuteconomique contemporaine

A mesure que lrsquoactiviteacute eacuteconomique mondiale srsquoaccroicirct que la technologie eacutevolue les

changements sont de plus en plus rapides Ils introduisent des situations ineacutedites auxquelles les

entreprises doivent srsquoadapter en cherchant de nouvelles solutions de management Les trois

derniegraveres deacutecennies ont eacuteteacute notamment marqueacutees par les pheacutenomegravenes suivants (que nous

listons sans tenir compte des liens pouvant exister entre eux)

Pheacutenomegravene Traduction Effets

Deacute reacuteglementation

globalisation

financiegravere

titrisation

Libre circulation des capitaux accegraves

facile des particuliers au marcheacute

boursier (directement ou par

lrsquointermeacutediaire des OPCVM et SICAV)

Monteacutee en puissance du financement

des entreprises sur le marcheacute boursier

Fonds de pension

(retraites) et fonds

souverains (eacutetats)

Poids boursier important drsquoinvestisseurs

institutionnels qui cherchent un haut

rendement financier (dividendes ou

valorisation boursiegravere)

Pression sur les grandes entreprises

influence sur les strateacutegies

Mondialisation Liberteacute des eacutechanges internationaux Accroissement de la concurrence

recherche drsquoavantages eacuteconomiques

par la deacutelocalisation (biens et

services) la concentration des efforts

(recentrage) problegravemes drsquoemploi

multiplication des transports perte

drsquoinfluence des politiques

Baisse de lrsquoemploi

occidental

(notamment

industriel)

Moins de fabrications fabrications

automatiseacutees recours aux moyens

informatiques

Activiteacute reporteacutee sur le commerce la

conception et les services chocircmage

charge sociale

Restructurations Optimisation des entreprises

abaissement des coucircts augmentation

des marges recherche drsquoune taille

critique (eacuteconomies drsquoeacutechelle poids

sur le marcheacute)

Recentrages externalisations fusions

deacutelocalisations constitution de grands

groupes

NTIC (nouvelles

technologies de

lrsquoinformation et de

la communication)

Mise en œuvre des reacuteseaux (dont

internet) et drsquoapplications

informatiques communicantes

Nouvelles formes de commerce

marcheacute international deacutelocalisation

du travail intellectuel reacuteorganisation

de la distribution

Rareacutefaction relative

des matiegraveres

premiegraveres

Recherche de substituts exploration

miniegravere coucircts drsquoexploitation des

gisements accrus

Augmentation des coucircts variations

erratiques du cours des matiegraveres

deacutestabilisations politiques

Evolution

geacuteopolitique et

eacuteconomique

mondiale

Chute de lrsquoURSS transformation des

eacuteconomies collectivistes pays

eacutemergents (Chine Inde Breacutesil Russie)

()

Accroissement de la population

mondiale (4 agrave 6 7 milliards de 1970 agrave

2008)

Libeacuteralisme sans frein () nouvelles

puissances eacuteconomiques

opportuniteacutes de deacuteveloppement

nouveau partage des ressources

ineacutegaliteacutes baisse du soutien aux PVD

laquo Terrorisme raquo Actions armeacutees pression de groupes

armeacutes non gouvernementaux

Deacutestabilisations reacutegionales charge

des deacutepenses militaires

Deacuteveloppement

durable

Recherche drsquoune croissance eacutequitable

et respectueuse de lrsquoenvironnement

Pression sur les entreprises (eacutetats

associations de consommateurs

eacutecologistes ONG)

() Reacutecemment quelques affaires (Enron laquo subprimes raquo Vivendi Universal Socieacuteteacute

Geacuteneacuterale Airbus par exemple) et agrave plus grande eacutechelle la crise financiegravere de 2008 ont

montreacute les dangers drsquoune libeacuteralisation sans controcircles suffisants

Initiation au management copy CRCF ndash J Sornet Page 31 48

() Des alliances eacuteconomiques naissent entre pays eacutemergents (notamment en

ameacuterique centrale creacuteation de la Banque du Sud en 2008 par exemple) et lrsquoon

commence agrave imaginer une baisse progressive de lrsquoinfluence eacuteconomique des Etats

Unis

2 ndash Les deacutefis actuels du management

21 ndash Les grandes orientations

Lrsquoeacutevolution eacuteconomique suggegravere quelques pistes parfois concurrentes pour lrsquoaction du

manager contemporain On y retrouve au premier plan la construction drsquoune vision qui est

une composante commune du leadership

Objectif du manager

pour lrsquoentreprise

Justification Facteurs de reacuteussite

Construire une vision Eclairer lrsquoavenir de lrsquoentreprise partager

un but souder motiver

Effort de reacuteelle prospection

volontarisme de la direction

bonne communication

Reacuteactiviteacute et flexibiliteacute

(sous tous les aspects

agrave tous niveaux)

Srsquoadapter rapidement au marcheacute Bonne organisation des processus

personnel compeacutetent autonome

et motiveacute structure hieacuterarchique

alleacutegeacutee robotisation

Deacutegager des profits Reacutemuneacuterer les apporteurs de capitaux

srsquoautofinancer

Ajuster coucircts et structures

Exploiter les nouvelles

technologies

Reacuteactiviteacute ajuster coucircts et deacutelais

reacutepondre au marcheacute suivre les clients

Organiser le SI de faccedilon

pertinente eacuteviter le coucirct excessif

drsquoinvestissements trop en

laquo pointe raquo (laquo essuyer les placirctres raquo)

utiliser judicieusement les services

exteacuterieurs

Bacirctir des alliances

(contrats fusions)

Deacutevelopper une activiteacute limiter les coucircts

de transaction () atteindre la taille

critique et de meilleurs rendements se

recentrer sur une activiteacute profitable

Dominer les processus se donner

une identiteacute lisible externaliser se

doter drsquoune capaciteacute financiegravere

suffisante

Valoriser lrsquoimage Attirer les clients favoriser les alliances

donner confiance (apporteurs de fonds

employeacutes clients partenaires socieacuteteacute

civile)

Instaurer des regravegles de

gouvernance inteacutegrer le

deacuteveloppement durable

respecter lrsquoenvironnement

Geacuterer les risques Faire face aux aleacuteas eacuteconomiques et

technologiques (conjoncture politiques

accidents malveillance)

Creacuteer un systegraveme drsquoalerte geacuterer

la crise (reacuteaction raisonneacutee

sceacutenarios poursuite de

lrsquoexploitation dans un contexte

instable) mise en place de

proceacutedures drsquoapprentissage pour

ameacuteliorer les reacuteactions au fil du

temps

Geacuterer le changement Faire face agrave lrsquoeacutevolution de la demande

la pression sur les prix la variation des

performances financiegraveres la

concurrence la globalisation des

marcheacutes lrsquoeacutevolution technologique aux

fusions ou alliances aux changements

de reacuteglementation de direction hellip ()

Bonne communication pour

donner du sens au changement

et obtenir lrsquoadheacutesion du personnel

Rassembler et geacuterer les

connaissances former le

personnel

Innover Garder un avantage concurrentiel se

diffeacuterencier

Veille technologique et

commerciale investissement

Ouverture

internationale

Elargir le marcheacute saisir les opportuniteacutes Veille commerciale partenariats

() La theacuteorie des coucircts de transaction deacuteveloppeacutee par OE Williamson dans les

anneacutees 70 integravegre les coucircts lieacutes au recours au marcheacute (recherche et choix drsquoun

fournisseur neacutegociation reacutedaction de contrat suivi des eacutechanges risque de rupture

Initiation au management copy CRCF ndash J Sornet Page 32 48

drsquoapprovisionnement hellip) On peut en conclure que lrsquointeacutegration de diffeacuterentes

activiteacutes agrave lrsquoentreprise (la laquo firme raquo) preacutesente des avantages Mais des coucircts de

transaction internes doivent aussi ecirctre consideacutereacutes (preacuteparation organisation

surveillance hellip) et certaines formes de coopeacuteration continue avec les fournisseurs

permettent de reacuteduire le coucirct des transactions externes

() drsquoapregraves laquo Les meilleures pratiques de management raquo - Brilman Heacuterard ndash EO

Une eacutetude du Conference Board (2002) liste les deacutefis du management vus par 700 leaders

mondiaux Soit en reacutesumeacute avec indication du score correspondant

1 ndash Fideacuteliser les clients (42)

2 ndash Reacuteduire les coucircts (38)

3 ndash Accroicirctre flexibiliteacute et reacuteactiviteacute (29)

4 ndash Amener les employeacutes agrave adheacuterer aux valeurs et visions de lrsquoentreprise (26)

5 ndash Deacutevelopper et retenir les leaders (25)

6 ndash Geacuterer acquisitions et alliances (24)

7 ndash Accroicirctre lrsquoinnovation (20)

En fin de classement citoyenneteacute et reacuteputation (4) et ameacutelioration de la diversiteacute (3)

22 ndash Les techniques disponibles

Pour faire face aux deacutefis le manager dispose de nouveaux concepts et de nouvelles

techniques Le tableau ci-dessous en donne un reacutesumeacute et indique les domaines qursquoils

influencent principalement

Initiation au management copy CRCF ndash J Sornet Page 33 48

Principaux concepts techniques outils Incidence principale sur

Internet

- e-commerce (commerce eacutelectronique site

entreprise)

- CRM ou GRC (gestion de la relation client)

- e-procurement (gestion des approvisionnements

par le reacuteseau)

- messagerie eacutelectronique

- e-recrutement

Vente accegraves au marcheacute

Relation client reacuteactiviteacute personnalisation

fideacutelisation

Deacutelais coucircts

Communication transfert de donneacutees (piegraveces

jointes) tous domaines

Communication recrutement

Intranet reacuteseau drsquoentreprise SI

- knowledge management (gestion des

connaissances)

- e-learning (apprentissage en ligne)

- plateforme de travail collaboratif (groupware)

- workflow (circulation eacutelectronique de

documents enchaicircnement de processus)

- e-RH portail RH (libre accegraves aux postes agrave

pourvoir informations candidatures hellip)

- PGI (progiciel de gestion inteacutegreacute) ou ERP

Innovation capaciteacute au changement veille

documentaire

Formation du personnel accompagnement des

changements

Coordination communication interne

Coordination

Communication interne (voire internet en

externe) reacuteduction des coucircts climat drsquoentreprise

recrutement plans de carriegraveres hellip

Coucircts fiabiliteacute du systegraveme drsquoinformation deacutelais

processus (continuiteacute inteacutegration)

Logistique inteacutegreacutee

Supply Chain Management (SCM) gestion de la

logistique (incluant les approvisionnements)

Processus deacutelais coucircts

Externalisation

Valorisation du capital humain

GPEC (gestion preacutevisionnelle des emplois et

compeacutetences)

Coaching

Reacuteactiviteacute de lrsquoentreprise conservation des

compeacutetences rendements individuels turn-over

adaptation des compeacutetences motivation

Efficaciteacute individuelle controcircle reacutegulation

progregraves processus

Approche processus

Optimisation des processus

Deacutemarche qualiteacute totale (TQM ndash total quality

management)

Empowerment (empouvoirement)

Benchmarking reacuteingeacutenieacuterie

Coucircts marges qualiteacute deacutelais flexibiliteacute

externalisation eacutelargissement des compeacutetences

organisation

Ameacutelioration des processus (meacutetiers et supports)

Autonomie compeacutetences des employeacutes

Ameacutelioration des processus restructuration

Management par la valeur

Parties prenantes

Satisfaction des parties prenantes financement

motivation collaborations hellip

Collaboration inter organisations

Reacuteseaux drsquoentreprises alliances

EDI (eacutechange de donneacutees informatiseacutees) extranet

Impartition externalisation (outsourcing)

Coucircts recentrage investissements lancement

drsquoactiviteacute

Coucircts reacuteactiviteacute deacutelais relations avec

lrsquoadministration

Coucircts recentrage limitation des investissements

Ethique drsquoentreprise

Gouvernance drsquoentreprise (mode de direction

encadreacute par des regravegles)

Rocircle socieacutetal deacuteveloppement durable

environnement

Image de lrsquoentreprise reacutegulation du top

management relations actionnaires

Image peacutenaliteacutes et amendes objectifs

strateacutegiques

Initiation au management copy CRCF ndash J Sornet Page 34 48

23 ndash Le rocircle socieacutetal des entreprises

La responsabiliteacute socieacutetale de lrsquoentreprise (RSE) deacutesigne le rocircle qursquoelle prend dans la socieacuteteacute

au-delagrave de son activiteacute purement geacuteneacuteratrice de profit On parle aussi drsquoentreprise citoyenne

La RSE est indissociable du deacuteveloppement durable de porteacutee mondiale et dont les trois

piliers sont

- eacuteconomique (favoriser le deacuteveloppement les eacutechanges internationaux)

- social (accegraves aux soins eacuteducation conditions de travail hellip)

- environnemental (pollution preacuteservation des ressources hellip)

La RSE integravegre notamment une preacuteoccupation sociale de lrsquoentreprise vis-agrave-vis de ses salarieacutes

(seacutecuriteacute et santeacute au travail juste reacutemuneacuteration deacuteveloppement personnel hellip) Elle conduit agrave

tenir compte dans le management drsquoune vision exteacuterieure agrave lrsquoentreprise qui peut avoir des

reacutepercussions possibles sur son activiteacute eacuteconomique

Lrsquoentreprise peut aussi tirer avantage drsquoune deacutemarche responsable par la baisse de certains

coucircts (plus faibles consommations drsquoeacutenergies reacuteduction des transports hellip)

Le rocircle socieacutetal de lrsquoentreprise a eacuteteacute reconnu en France par la loi laquo NRE raquo de 2001 (loi sur les

nouvelles reacutegulations eacuteconomiques) qui oblige les socieacuteteacutes franccedilaise coteacutees sur un marcheacute

reacuteglementeacute agrave rendre compte dans leur rapport annuel de leur gestion sociale et

environnementale au travers de leur activiteacute

Article 116 de la loi Le rapport viseacute agrave larticle L 225-102 rend compte hellip laquo Il comprend

eacutegalement des informations dont la liste est fixeacutee par deacutecret en Conseil dEtat sur la

maniegravere dont la socieacuteteacute prend en compte les conseacutequences sociales et

environnementales de son activiteacute Le preacutesent alineacutea ne sapplique pas aux socieacuteteacutes

dont les titres ne sont pas admis aux neacutegociations sur un marcheacute reacuteglementeacute raquo

Une norme ISO 14000 integravegre ces preacuteoccupations et des taxes eacutecologiques sont

progressivement creacutees

3 ndash Le management par la valeur

31 ndash De lrsquoanalyse au management par la valeur

Lrsquoanalyse de la valeur est neacutee en 1947 aux Etats-Unis (General Electrics) Cette technique

consiste agrave eacutelaborer des produits conformes aux attentes de la clientegravele mais sans excegraves pour

trouver un bon compromis entre valeur pour le client et coucirct Le produit optimal est deacutefini agrave

partir drsquoenquecirctes qui deacuteterminent le besoin client (ou plutocirct drsquoun client laquo type raquo)

Exemple il est inutile de concevoir un petit veacutehicule citadin capable de parcourir

500 000 km sans avarie compte tenu des effets de mode et du faible kilomeacutetrage

annuel Par contre le marcheacute peut exiger un fonctionnement sans faille sur 150 000 km

soit dix ans en moyenne ce qui conditionne les coucircts de production

Cette recherche drsquoun ajustement de valeur au besoin des clients eacutetait un preacutecurseur du

management par la valeur qui recherche plus largement la creacuteation de valeur pour

chacune des parties prenantes de lrsquoentreprise tout en lui meacutenageant un reacutesultat suffisant

Plus geacuteneacuteralement le management par la valeur est deacutefini par une norme europeacuteenne (EN

12973)

Le management par la valeur est un style de management particuliegraverement destineacute agrave

mobiliser les individus agrave deacutevelopper les compeacutetences et agrave promouvoir les synergies et

Initiation au management copy CRCF ndash J Sornet Page 35 48

linnovation avec pour objectif la maximisation de la performance globale dun

organisme Le management par la valeur apporte une nouvelle faccedilon dutiliser nombre

de meacutethodes de management existantes Il est en coheacuterence avec le Management

de la qualiteacute

Cette approche du management pose de nombreuses questions notamment quelles

prioriteacutes et quelles valeurs attribuer aux parties prenantes comment appreacutehender la

perception par les parties prenantes de la valeur qui leur est affecteacutee

32 ndash La valeur client

Le processus drsquoeacutelaboration drsquoun produit qui consomme des ressources coucircteuses doit creacuteer

une valeur suffisante pour provoquer lrsquoachat par le client final La production drsquoune valeur

reconnue par le client est vitale pour lrsquoentreprise mais sa deacutetermination est parfois complexe

La valeur du produit perccedilue par le client integravegre des eacuteleacutements en partie subjectifs

- une valeur drsquousage (le produit reacutepond agrave un besoin)

- une valeur drsquoestime (lrsquoimage apporteacutee par le produit un aspect affectif)

- une valeur drsquoeacutechange (deacuteduite de lrsquoespoir de revente du produit)

Valeurs drsquousage drsquoestime et drsquoeacutechange deacutependent implicitement de la qualiteacute (un bien peu

fiable est impropre agrave lrsquousage attendu de mauvaise qualiteacute notoire il nrsquoapporte pas une

image positive et ses deacutefauts connus nuisent agrave sa revente) Une eacutevaluation de la qualiteacute

intervient donc dans la valeur perccedilue du produit

Par ailleurs le client considegravere le coucirct drsquoobtention du produit (les charges qursquoil doit supporter

pour acqueacuterir le produit lrsquoeffort qursquoil doit faire pour trouver le produit et les frais de mise agrave

disposition)

Le prix perccedilu par le client est geacuteneacuteralement supeacuterieur au prix de vente

Le client achegravete theacuteoriquement le produit qui preacutesente la diffeacuterence valeur perccedilue ndash prix

perccedilu la plus favorable ou le meilleur rapport prix perccedilu qualiteacute perccedilue et dans certains

cas celui qui a le prix produit le plus bas

Remarque les valeurs du scheacutema ci-dessus changent durant le cycle de vie du produit

(un nouveau produit peut avoir une valeur perccedilue plus eacuteleveacutee qursquoen fin de vie) La

valeur client ne peut ecirctre eacutevalueacutee que par enquecirctes et ne peut donc ecirctre deacutefinie avec

certitude

La notion de laquo satisfaction client raquo conseacutecutive agrave une vente influence aussi le prix produit et

le prix perccedilu

- lrsquoentreprise gagne sur les coucircts de recherche de clientegravele

- le client nrsquoa pas agrave rechercher un nouveau fournisseur et beacuteneacuteficie drsquoun coucirct drsquoobtention

plus bas

valeur perccedilue client

prix perccedilu client

coucirct produit Marge (valeur creacuteeacutee pour

lrsquoentreprise)

euros

prix produit

Valeur creacuteeacutee

pour le client

Initiation au management copy CRCF ndash J Sornet Page 36 48

La satisfaction du client deacutepend de facteurs qualitatifs aussi divers que la fiabiliteacute du produit

la vitesse de reacuteaction du fournisseur lrsquoattitude des commerciaux lrsquoefficaciteacute du service

apregraves-vente la netteteacute des contrats ou la justesse de la facture

Valeur perccedilue coucirct marge et satisfaction reacutesultent de processus allant de la conception du

produit jusqursquoagrave sa livraison et son apregraves-vente La deacutemarche laquo processus raquo et lrsquolaquo analyse de la

valeur raquo en forccedilant la recherche de solutions efficientes agrave tout niveau administratif

technique commercial et apregraves-vente sont donc neacutecessaires pour bien positionner

lrsquoentreprise sur son marcheacute

Pour autant le risque commercial ne peut jamais ecirctre annuleacute et lrsquooffre de lrsquoentreprise ne

satisfait geacuteneacuteralement pas en milieu concurrentiel tous ses clients potentiels

33 - La creacuteation de valeur pour les autres parties prenantes

Les salarieacutes

La creacuteation drsquoune valeur suffisante pour les salarieacutes est reconnue comme neacutecessaire car des

observations montrent que la satisfaction des clients en deacutepend Moins souvent eacutevoqueacutee en

peacuteriode de chocircmage elle nrsquoest prioritaire que pour les employeacutes dont lrsquoentreprise souhaite

conserver les compeacutetences

La laquo valeur salarieacute raquo ne comprend pas que le salaire Le sentiment drsquoappartenance agrave un

groupe la reconnaissance lrsquoaccomplissement de soi et la construction professionnelle en

sont des eacuteleacutements importants Comme pour les clients on doit ainsi distinguer la reacutetribution

perccedilue du salaire objectif

Les actionnaires

Lrsquoactionnaire apporte des fonds propres agrave lrsquoentreprise en contrepartie de titres parfois

neacutegociables en bourse et assortis drsquoun droit de vote en assembleacutee geacuteneacuterale La valeur

attribueacutee aux actionnaires est servie en termes moneacutetaires (dividende ou augmentation de la

valeur du titre neacutegociable)

Remarque des facteurs non moneacutetaires comme lrsquoimage de lrsquoentreprise qui deacutepend

en partie de sa communication peuvent influencer la deacutecision drsquoachat de vente ou

de conservation des titres par lrsquoactionnaire

Reacutetribution perccedilue euros

Salaire objectif

Avantage non

moneacutetaire de

lrsquoemploi

Initiation au management copy CRCF ndash J Sornet Page 37 48

Compte tenu de lrsquoimportance croissante de lrsquoactionnariat dans le financement des grandes

entreprises coteacutees en bourse et notamment des investisseurs institutionnels comme les fonds

de pension des indicateurs speacutecifiques ont eacuteteacute introduits pour appreacutecier la performance des

entreprises vue par les actionnaires Par exemple la valeur ajouteacutee eacuteconomique (EVA reg

economic value added marque deacuteposeacutee de Stern Stewart ou VAE ndash valeur ajouteacutee

eacuteconomique parfois deacutenommeacutee VEC ndash valeur eacuteconomique creacuteeacutee) qui prend en compte le

coucirct du capital

LrsquoEVA correspond tregraves scheacutematiquement au calcul suivant

EVA = (PO) profit opeacuterationnel ndash (C) coucirct du capital X (CE) capitaux employeacutes

LrsquoEVA neacutecessite en pratique des retraitements assez complexes Le PO peut se deacuteterminer

selon les principes suivants

- PO = reacutesultat drsquoexploitation (avant inteacuterecircts) ndash impocirct

- PO = beacuteneacutefice courant (tenant compte des inteacuterecircts) + inteacuterecircts ndash eacuteconomie drsquoimpocirct sur les

inteacuterecircts (on exclue les eacuteleacutements financiers et lrsquoimpocirct correspondant) ndash impocirct

- lrsquoimpocirct pris en compte correspond au profit opeacuterationnel consideacutereacute (dans les cas courants agrave

13 du PO)

C = taux moyen de reacutemuneacuteration du capital (reacutesultant par exemple du dividende exigeacute de

certains investisseurs et des taux drsquoemprunts bancaires)

CE = capitaux propres et dettes portant inteacuterecirct

Remarque le profit opeacuterationnel ou reacutesultat opeacuterationnel correspond au NOPAT ndash net

operating profit after tax - anglo-saxon LrsquoEVA est eacutegale au NOPAT diminueacute de la

reacutemuneacuteration des capitaux

Exemple lrsquoentreprise X dispose drsquoun capital de 2 500 000 euro et reacutealise un beacuteneacutefice net

drsquoimpocirct de 450 000 euro (taux 33 13) Un dividende de 6 doit ecirctre verseacute aux

actionnaires et la banque lui a accordeacute un precirct de 1 200 000 euro agrave 4 Les autres

constituants des reacutesultats financier et exceptionnel sont neacutegligeables

Reacutesultat opeacuterationnel = 450 000 + 004 x 1 200 000 x 23 = 482 000 euro

Coucirct du capital = 006 x 2 500 000 + 004 x 1 200 000 x 23 = 182 000 euro

EVA = 300 000 euro

Coucirct moyen pondeacutereacute du capital (C) = (004 x 1 200 000 x23 + 006 x 2 500 000)

3 700 000 Soit 492

Si lrsquoEVA est positive lrsquoentreprise creacuteeacutee de la valeur apregraves reacutemuneacuteration des capitaux et sa

valeur boursiegravere doit augmenter

Lrsquoutilisation de lrsquoEVA comme indicateur influence le management de lrsquoentreprise car il y a

trois moyens pratiques drsquoaugmenter lrsquoEVA

- augmenter le reacutesultat opeacuterationnel

- lancer des investissements ayant une rentabiliteacute supeacuterieure agrave C

- eacuteliminer les activiteacutes ayant une rentabiliteacute infeacuterieure agrave C

Remarque lrsquoutilisation sans nuance de lrsquoEVA comme critegravere de management peut

poser problegraveme Le calcul de lrsquoEVA repose sur des ajustements comptables il est donc

sujet agrave manipulations (provisions capitalisation ou non de la RD hellip) Par ailleurs le

critegravere laquo EVA raquo pris isoleacutement peut conduire agrave chercher la rentabiliteacute agrave court terme agrave

reacuteduire les investissements prospectifs et donc nuire agrave terme au deacuteveloppement de

lrsquoentreprise

Initiation au management copy CRCF ndash J Sornet Page 38 48

Les fournisseurs reccediloivent le paiement de leurs factures plus ou moins rapidement (le deacutelai

de paiement repreacutesente une valeur consentie au fournisseur)

Lrsquoentreprise peut accroicirctre la valeur apporteacutee agrave ses fournisseurs par des actions cibleacutees

comme une contribution agrave la formation de leurs personnels certains transferts de

technologie ou de savoir faire agrave des sous-traitants une coopeacuteration suivie favorisant leur

deacuteveloppement lrsquointeacutegration agrave des campagnes de promotion

A noter que la valeur consentie aux fournisseurs peut avoir une influence sur la qualiteacute et les

deacutelais de livraison des produits

La collectiviteacute reccediloit des taxes et parfois des prestations en nature par deacutefaut ou explicites

(effort de preacuteservation de lrsquoenvironnement ameacutenagement du territoire par les implantations

aide mateacuterielle agrave des projets participation agrave la formation par exemple)

APPLICATIONS DT

DT1 Deacutefinir expliquer deacutereacuteglementation socieacutetal eacuteconomies drsquoeacutechelle coaching EDI

gouvernance

DT2 Deacuteterminer en quoi la deacutemarche TQM srsquoinscrit dans les deacutefis actuels du management

DT3 Apregraves avoir consulteacute les documents ci-dessous extraits du site drsquoAir France

(httpdeveloppement-

durableairfrancecomFRfrlocaldemarcheN4_positionnement_pphtm)

exposer les enjeux et les limites de la RSE et de la gestion des parties prenantes

Initiation au management copy CRCF ndash J Sornet Page 39 48

Dialogue avec les parties prenantes

Initiation au management copy CRCF ndash J Sornet Page 40 48

Attentes des parties prenantes

Initiation au management copy CRCF ndash J Sornet Page 41 48

Creacuteation de valeur pour les parties prenantes

La creacuteation de valeur pour les parties prenantes est au cœur de la strateacutegie du Groupe Le scheacutema de

distribution financiegravere ci-dessous donne un aperccedilu de la distribution des recettes du Groupe aux

diffeacuterentes parties prenantes actionnaires collaborateurs fournisseurs pouvoirs publics

collectiviteacutes locales etc

Initiation au management copy CRCF ndash J Sornet Page 42 48

Fiche DT1 ndash Extrait du sommaire de laquo Problegravemes eacuteconomiques raquo No 2894

La gestion des entreprises bouleverseacutee par les technologies de linternet

Reacutealiteacutes industrielles - Annales des Mines Jean-Michel Yolin

Avec lavegravenement de linternet les processus de conception de production et de vente sont

radicalement remis en cause Quel que soit le secteur dactiviteacute les technologies de linternet

permettent en effet de reacuteduire les deacutelais et de passer dun processus discontinu agrave un processus

continu Lorganisation des entreprises et leur mode de gestion en sont profondeacutement bouleverseacutes

tant au niveau individuel que collectif Linternet rend ainsi possible la reacutealisation dobjectifs que les

entreprises cherchaient agrave atteindre depuis longtemps sans y parvenir meilleure eacutecoute du client

travail sans stocks en flux tendu hieacuterarchies plates autorisant une grande reacuteactiviteacute flexibiliteacute dans

lorganisation et loutil de production acceacuteleacuteration du renouvellement des produits entreprises en

reacuteseau ougrave chacune se recentre sur son cœur de meacutetier etc

Le laquo knowledge management raquo ou comment geacuterer les connaissances

Document de travail du LAMSADE - Michel Grundstein

Peter Drucker lavait preacutedit le capital immateacuteriel eacutetait voueacute agrave devenir un facteur de compeacutetitiviteacute

pour lentreprise La libeacuteralisation des eacutechanges acceacutelegravere les processus de deacutecision de lentreprise

et implique que lassimilation des informations soit agrave la fois de meilleure qualiteacute et plus rapide Ainsi

la fonction qui consiste agrave manager les connaissances au sein de lentreprise savegravere primordiale

Bien que la prise de conscience de limportance du capital immateacuteriel ait eacuteteacute tardive - le concept

de knowledge management est apparu en France aux Etats-Unis et au Japon au milieu des

anneacutees 1990 - agrave lheure actuelle lorganisation de leacutechange dinformations et le partage des

connaissances sont devenus des facteurs cleacutes dune gestion performante de lentreprise Ils

doivent sinscrire dans un projet global destineacute agrave mettre en valeur les savoirs et les savoir-faire

individuels et collectifs

Les leccedilons du laquo coaching raquo pour le management de la qualiteacute

Humanisme et Entreprise - Martine Brasseur

Parmi les nouvelles formes de management en vogue dans les entreprises le coaching figure en

bonne place Appliqueacute au management de la qualiteacute il sagit dune pratique

daccompagnement destineacutee agrave initier et agrave faciliter le processus de deacuteveloppement dun individu

La deacutemarche consiste agrave affirmer que tout individu est en quecircte de qualiteacute agrave condition toutefois

de ne pas lui imposer des contraintes lempecircchant de progresser On considegravere notamment les

erreurs comme potentiellement feacutecondes En deacutefinitive le coach donne au coacheacute la permission

de reacuteussir en lui donnant aussi la permission deacutechouer

Initiation au management copy CRCF ndash J Sornet Page 43 48

Fiche DT2 ndash Management strateacutegique les sept deacutefis agrave relever dici agrave 2016

Extrait drsquoun article du site wwwlentreprisecom -Sabine Blanc - Mis en ligne le 20032007

(httpwwwlentreprisecom325article11977html)

Une eacutetude anglaise publieacutee par lopeacuterateur Orange Grande-Bretagne deacutecrypte la mutation

des formes de travail et les enjeux majeurs pour les entreprises de demain afin decirctre au top

de la compeacutetitiviteacute Voici les challenges-cleacutes pour les managers qui veulent rester dans la

course hellip

1 - Future organisation du travail les quatre laquo mondes raquo possibles

La reacutealiteacute sera probablement un meacutelange de ces quatre sceacutenarios souligne lrsquoeacutetude

Les mondes mutuels Tout se passe dans le cadre des communauteacutes locales vie priveacutee

comme professionnelle Le modegravele coopeacuteratif preacutevaut au lieu du laquo big business raquo Oublieacutes

aussi dans ce systegraveme les trajets pour aller au bureau les gens preacutefegravereront travailler dans de

petites entreprises locales souvent connecteacutees au reacuteseau drsquoautres structures similaires

Les laquo reacutepondants raquo (en anglais laquo replicants raquo) La figure du consultant freelance deviendra

dominante tandis que celle du salarieacute deacuteclinera Il ne sera pas rare de travailler pour plusieurs

entreprises On perdra en seacutecuriteacute de lrsquoemploi en visibiliteacute et en routine ce que lrsquoon gagnera

en liberteacute La majeure partie des tacircches srsquoeffectuera chez soi avec la possibiliteacute de srsquoinstaller

temporairement dans les bureaux de son client du moment Dans un contexte dincertitude

sur lrsquoavenir les travailleurs alterneront peacuteriodes drsquoactiviteacute intense et repos Ce sera agrave eux

drsquoaller vers les entreprises et non lrsquoinverse mecircme si celles-ci devront veiller agrave rester attractives

Les cottages eacutelectroniques Comme ce nom le suggegravere le teacuteleacutetravail deviendrait la norme

univers priveacute et professionnel se confondant Plus besoin de subir une heure de transport les

salarieacutes se logueront de chez eux sur le reacuteseau de lrsquoentreprise Les reacuteunions se tiendront dans

de petits bureaux centraux situeacutes agrave courte distance La flexibiliteacute du temps de travail srsquoimpose

Les salarieacutes disposeront de plus de marge de liberteacute dans leur activiteacute

Les disciples de la nueacutee Cette appellation poeacutetique cache simplement une extension de

lrsquoorganisation actuelle des grandes entreprises avec des salarieacutes se rendant sur un lieu de

travail centraliseacute Le rocircle croissant des technologies de lrsquoinformation multipliera les faccedilons de

collaborer et accroicirctra lrsquoefficaciteacute Le controcircle du travail sera omnipreacutesent La frontiegravere entre

travail et vie priveacutee restera marqueacutee

2 - Sept deacutefis pour les entreprises et leur managers

Quoi qursquoil advienne les entreprises et leurs dirigeants devront concentrer leurs efforts sur sept

points-cleacutes pour srsquoadapter Voici quelques exemples de probleacutematiques souleveacutees par le

rapport et des pistes de solution

Le leadership Les managers devront entre autres savoir persuader et influencer des

travailleurs beaucoup plus indeacutependants Ils auront aussi agrave repenser les niveaux auxquels

prendre les deacutecisions strateacutegiques en haut ou au contraire agrave des degreacutes moins eacuteleveacutes de la

pyramide hieacuterarchique

gt Faire du management une force facilitant les activiteacutes transversales plutocirct que la reacuteduire agrave

la seule fonction de deacutecision

La culture drsquoentreprise Davantage de salarieacutes capables de reacutefleacutechir seront neacutecessaires

tandis que les tacircches qui peuvent ecirctre automatiseacutees ou scripteacutees diminueront Un des

enjeux creacuteer une culture agrave mecircme drsquoattirer et drsquoencourager les personnes preacutesentant ces

qualiteacutes de reacuteflexion requises dans un contexte de compeacutetition accrue et de plus grande

indeacutependance des travailleurs

Initiation au management copy CRCF ndash J Sornet Page 44 48

gt Passer si neacutecessaire drsquoune culture drsquoentreprise forte agrave un mode drsquoengagement plus

consensuel moins rebutant

La marque Conseacutequence du recours croissant agrave lrsquo laquo outsourcing raquo lrsquoimage drsquoune marque

deacutependra plus drsquoagents exteacuterieurs qui ne fonctionnent pas forceacutement selon le mecircme mode

drsquoorganisation Comment garder le controcircle dessus

gt Choisir le mode qui corresponde le plus agrave vos valeurs et preacutevoir un programme de risk

management qui mette en eacutevidence ougrave les conflits sont susceptibles de jaillir

Lrsquoinnovation Plus que jamais il faudra faire face agrave une acceacuteleacuteration du rythme de

lrsquoinnovation en proposant constamment des solutions adapteacutees

gt Tisser des partenariats strateacutegiques avec drsquoautres entreprises pour partager les coucircts et les

fruits de lrsquoinnovation

Le deacutefi opeacuterationnel et technologique De quelle faccedilon controcircler lrsquoinformation crsquoest-agrave-dire

faire en sorte que les bonnes personnes accegravedent facilement agrave une information toujours en

phase tout en maintenant la seacutecuriteacute

gt Recourir agrave des laquo feuilles de route des futurs raquo syntheacutetisant en une page les indicateurs

sociaux et de consommation ainsi que les eacutevolutions technologiques et leacutegislatives qui

influent sur les changements et indiquant comment ils modifient vos marcheacutes vos clients et

votre organisation

La qualiteacute Si de nouveaux proceacutedeacutes ont pu deacutegrader la qualiteacute comme le recours agrave des

centres drsquoappel externaliseacutes drsquoautres ideacutees se sont reacuteveacuteleacutees plus prometteuses comme en

teacutemoigne le succegraves de certaines compagnies aeacuteriennes low cost Elles ont su conjuguer prix

serreacutes et services eacuteleveacutes ce qui devra devenir la norme estime lrsquoeacutetude

gt Continuer de rechercher la qualiteacute Elaborez aussi une bonne prestation service qui inclut

une livraison de qualiteacute voire creacuteez-la en partenariat avec les consommateurs

La leacutegislation La question de la proprieacuteteacute intellectuelle pourrait ecirctre probleacutematique Elle est

deacutejagrave source de conflits comme en teacutemoigne le procegraves pour violation de brevet intenteacute agrave RIM

le fabricant canadien du Blackberry par NTP Que pourra-t-on et que faudra-t-il proteacuteger par

un brevet Il sera eacutegalement neacutecessaire drsquoadapter la leacutegislation aux nouveaux modes

drsquoorganisation

gt Collaborer avec les acteurs du mecircme secteur et les leacutegislateurs pour deacutevelopper les

modegraveles des lieux de travail du futur et bacirctir le droit le plus adeacutequat

Orange a-t-il vu juste dans ses preacutevisions Rendez-vous dans neuf ans pour la reacuteponsehellip

Initiation au management copy CRCF ndash J Sornet Page 45 48

Fiche DT3 ndash Le management par la qualiteacute totale

Extrait drsquoune lettre drsquoinformation du cabinet Baud Accordance Consulting AD2 consultants ndash

2002

1 - Le TQM (Total Quality Management) offre pour lentreprise une vision de la qualiteacute plus

large et transversale

Son principe est simple La finaliteacute de lEntreprise est de deacutevelopper la satisfaction de ses

clients tout en eacutetant beacuteneacuteficiaire cest agrave dire pas agrave nimporte quel prix Elle doit ameacuteliorer sa

rentabiliteacute au travers de la deacutemarche qualiteacute La Qualiteacute Totale vise agrave fournir aux clients

externes et internes une reacuteponse adeacutequate agrave leurs attentes dans le meilleur rapport qualiteacute

prix la meilleure efficience

Elle considegravere pour cela lensemble des processus de lentreprise ayant une incidence sur la

qualiteacute et la satisfaction des clients

Le TQM fait ainsi une large place agrave

la deacutefinition et la planification de la strateacutegie geacuteneacuterale

la coheacuterence de la politique qualiteacute avec la strateacutegie

la deacutemultiplication de la politique qualiteacute dans toutes les directions de lentreprise

la relation client fournisseur interne

la prise en compte de lenvironnement concurrentiel

la consideacuteration de lensemble des risques potentiels financiers sociaux concurrentielshellip

limplication et la motivation du personnel

lanalyse des besoins des clients et le positionnement marketing

la maicirctrise des processus transverses internes

les reacutesultats sous tous ses aspects y compris financiers commerciaux image

De nombreux reacutefeacuterentiels sont relatifs agrave la Qualiteacute Totale hellip Tous ces reacutefeacuterentiels imposent un

questionnement plus profond et indiscret sur le mode de fonctionnement de lentreprise et

son management

helliphellip

2 - LISO 9001 2000 au travers du deacuteploiement des processus (management supports

reacutealisation et ameacutelioration continue) reacutepond quelque peu agrave la mecircme logique

LISO est une ouverture indeacuteniable vers la logique du TQM mais ne se reacutefegravere pas agrave la notion

defficience

Les dirigeants sont cependant sensibles agrave la neacutecessaire reacuteduction des coucircts de non-qualiteacute

et dobtention de la qualiteacute agrave la rentabiliteacute du systegraveme de management de la qualiteacute

mais ne perccediloivent pas toujours la qualiteacute comme une deacutemarche globale

Les deacutemarches qualiteacute commencent bien souvent par la remise en cause de lorganisation

leacutevaluation critique de son efficaciteacute lexamen des processus et la mise en eacutevidence des

lourdeurs administratives

La qualiteacute devient laffaire de tous hellip

Initiation au management copy CRCF ndash J Sornet Page 46 48

Fiche DT4 ndash Le deacuteveloppement durable et la RSE

Extrait du site wwwvigeocom

(httpwwwvigeocomcsr-rating-agencyfrmethodologiecriteres-de-recherche37-

criteres-d-analysehtml)

Deacuteveloppement durable laquo un deacuteveloppement qui reacutepond aux besoins du preacutesent sans compromettre

la capaciteacute des geacuteneacuterations futures de reacutepondre aux leurs raquo (Commission mondiale sur lrsquoenvironnement

et le deacuteveloppement ndash 1987)

Reacutefeacuterentiel drsquoeacutevaluation des entreprises par le groupe Vigeacuteo (le groupe mesure les performances et le

niveau de maicirctrise des risques de responsabiliteacute sociale des entreprises et des organisations - site

wwwvigeocom)

1 Ressources Humaines Ameacutelioration continue des relations professionnelles des relations drsquoemploi et des conditions de travail 2 Droits humains sur les lieux de travail Respect de la liberteacute syndicale et promotion de la neacutegociation collective non discrimination et promotion de lrsquoeacutegaliteacute eacutelimination des formes de travail proscrites (enfants travail forceacute) preacutevention des traitements inhumains ou deacutegradants de type harcegravelements sexuels protection de la vie priveacutee et des donneacutees personnelles 3 Environnement Protection sauvegarde preacutevention des atteintes agrave lenvironnement mise en place drsquoune strateacutegie manageacuteriale approprieacutee eacuteco conception protection de la biodiversiteacute et maicirctrise rationnelle des impacts environnementaux sur lrsquoensemble du cycle de vie des produits ou services

4 Comportements sur les marcheacutes Prise en compte des droits et inteacuterecircts des clients inteacutegration de standards sociaux et environnementaux dans la seacutelection des fournisseurs et sur lrsquoensemble de la chaicircne drsquoapprovisionnement preacutevention effective de la corruption respect des regravegles concurrentielles 5 Gouvernement drsquoentreprise Efficience et probiteacute assurance de lrsquoindeacutependance et de lrsquoefficaciteacute du Conseil drsquoadministration effectiviteacute et efficience des meacutecanismes drsquoaudit et de controcircle et notamment inclusion des risques de responsabiliteacute sociale respect des droits des actionnaires et notamment des minoritaires transparence et rationaliteacute de la reacutemuneacuteration des dirigeants 6 Engagement socieacutetal Effectiviteacute inteacutegration manageacuteriale de lrsquoengagement contribution au deacuteveloppement eacuteconomique et social des territoires drsquoimplantation et de leurs communauteacutes humaines engagements concrets en faveur de la maicirctrise des impacts socieacutetaux des produits et des services contribution transparente et participative agrave des causes drsquointeacuterecirct geacuteneacuteral

Initiation au management copy CRCF ndash J Sornet Page 47 48

ELEMENTS DE CORRIGE DT DT1 Deacutefinir expliquer

Deacutereacuteglementation = suppression des contraintes eacuteconomiques (libre eacutechange des biens et

capitaux)

Socieacutetal = qui se rapporte agrave la structure agrave lrsquoorganisation ou au fonctionnement de la socieacuteteacute

Economies drsquoeacutechelle = reacuteduction des coucircts lieacutee au niveau drsquoactiviteacute (amortissement des

charges fixes)

Coaching = accompagnement de personnes ou deacutequipes pour le deacuteveloppement de leurs

potentiels

EDI = eacutechange de donneacutees informatiseacutees ET standardiseacutees (ex SWIFT bancaire edifact

documents deacuteclaratifs)

Gouvernance = exercice du pouvoir la bonne gouvernance est participative et eacutequitable

conforme agrave lrsquointeacuterecirct commun

DT2 Deacuteterminer en quoi la deacutemarche TQM srsquoinscrit dans les deacutefis actuels du management

Voir notamment fiche 43

Maicirctrise des processus reacuteduction des coucircts reacuteactiviteacute et satisfaction de la clientegravele = faire

face agrave la concurrence

Ameacutelioration de lrsquoimage motivation du personnel

DT3 Apregraves avoir consulteacute les documents ci-dessous extraits du site drsquoAir France

(httpdeveloppement-

durableairfrancecomFRfrlocaldemarcheN4_positionnement_pphtm)

exposer les enjeux et les limites de la RSE et de la gestion des parties prenantes

Trame geacuteneacuterale possible

Introduction

Les deacutefis contemporains (accroissement de la concurrence devenue mondiale recherche

de nouveaux avantages concurrentiels pression de la socieacuteteacute besoin drsquoimage et de projet

lisible pour mener lrsquoentreprise crise et scandales du libeacuteralisme hellip) RSE et PP

Deacuteveloppement (voir cours)

1 ndash Parties prenantes et management par la valeur

PP deacutefinir citer reacutesumer lrsquoavantage rechercheacute (fideacuteliser motiver recherche drsquoalliances

implicites)

PP moyens (dont exemples AF) et meacutethode de management par la valeur (reacutepartie)

2 ndash La responsabiliteacute socieacutetale de lrsquoentreprise

RSE 3 axes

- eacuteconomique (favoriser le deacuteveloppement les eacutechanges internationaux)

- social (accegraves aux soins eacuteducation conditions de travail hellip)

- environnemental (pollution preacuteservation des ressources hellip)

RSE gouvernance drsquoentreprise facteur drsquoimage inteacutegrable dans la deacutemarche PP

Article 116 de la loi Le rapport viseacute agrave larticle L 225-102 rend compte hellip laquo Il comprend

eacutegalement des informations dont la liste est fixeacutee par deacutecret en Conseil dEtat sur la maniegravere

dont la socieacuteteacute prend en compte les conseacutequences sociales et environnementales de son

activiteacute Le preacutesent alineacutea ne sapplique pas aux socieacuteteacutes dont les titres ne sont pas admis aux

neacutegociations sur un marcheacute reacuteglementeacute raquo

Initiation au management copy CRCF ndash J Sornet Page 48 48

RSE exemple AF (ONG fournisseurs)

3 ndash Liens entre PP et RSE

- la RSE introduit de nouvelles PP

- la RSE suppose le respect des PP usuelles (employeacutes clients notamment)

4 - Probleacutematique

- deacutefinir la valeur reacuteellement apporteacutee par une gestion des PP (confusion salaire ndash valeur

idem impocircts hellip ex laquo valeur ajouteacutee raquo)

- communication (neacutecessaire mais aller au-delagrave)

- marginaliteacute des deacutepenses RSE (efficaciteacute sinceacuteriteacute de lrsquoengagement marge de manœuvre)

- charge RSE reporteacutee sur des tiers (ex fournisseurs AF)

- inteacutegration de facteurs non visibles en comptabiliteacute (pertes drsquoemploi nuisances hellip)

Conclusion

Voies incontournables mais pouvant nrsquoavoir qursquoun effet superficiel et temporaire Voir utiliteacute

drsquoaccompagnement leacutegislatif de regravegles de gouvernance

Initiation au management copy CRCF ndash J Sornet Page 3 48

Exemple lrsquoeacutecologie (la preacuteoccupation environnementale) peut influencer lrsquoactiviteacute de

lrsquoentreprise en modifiant les coucircts (transports eacutelimination des deacutechets

perfectionnement des installations matiegraveres hellip) en imposant des regravegles (normes de

pollution emballages hellip) en induisant une pression commerciale (image de

lrsquoentreprise preacutesentation des produits) etc

2 ndash Le management

21 ndash Deacutefinition du management

Le management est une activiteacute dont la finaliteacute est de conduire une organisation vers son but

(reacutealiser des profits se deacutevelopper assurer un service public hellip) par la reacutealisation drsquoobjectifs

preacutedeacutefinis

Le management est lieacute au travail collectif (laquo ecirctre efficaces agrave plusieurs travailler avec les

autres faire travailler les autres raquo) et son importance grandit avec la taille de lrsquoorganisation

Il trouve (au terme pregraves) ses origines dans les premiers grands chantiers de lrsquoEgypte ancienne

et son vocabulaire emprunte aux premiegraveres grandes organisations occidentales que sont les

armeacutees (cadre strateacutegie tactique hellip)

Le terme laquo manager raquo deacutesigne ceux qui ont une responsabiliteacute drsquoencadrement (en France

cette notion ne se confond pas toujours avec celle de cadre qui correspond agrave un statut

mais pas obligatoirement agrave une fonction)

22 ndash Les apports du management

Diriger consiste agrave fixer des objectifs en effectuant des choix Geacuterer crsquoest organiser et

optimiser les moyens disponibles pour atteindre les objectifs

Le terme laquo management raquo qui tend agrave remplacer la laquo gestion raquo ou la laquo direction raquo dans le

vocabulaire eacuteconomique franccedilais englobe ces deux notions Cette eacutevolution terminologique

accompagne une eacutevolution dans la faccedilon de conduire les organisations

laquo Manager raquo suppose

UN CERTAIN REALISME

Le management repose sur le pragmatisme propre aux pays anglo-saxons ougrave

laquo management raquo et laquo manager raquo deacutesignent depuis longtemps ce ou ceux qui encadrent la

marche de lrsquoentreprise

UNE GRANDE REACTIVITE

Le manager a la capaciteacute drsquoagir en fonction des circonstances pour maicirctriser rapidement

des situations complexes Le management se distingue ainsi

- de lrsquo laquo administration raquo au sens courant du terme que lrsquoon retrouve par exemple dans

conseil drsquoadministration ou administration de lrsquoEtat qui eacutevoque plutocirct une action laquo haut

placeacutee raquo assez eacuteloigneacutee du quotidien (bien que lrsquoadministration drsquoentreprise inaugureacutee par

Fayol soit eacutequivalente au management ndash Voir les MBA initialiseacutes agrave Harvard et lrsquoIAE en France)

- de la laquo gestion raquo qui renvoie communeacutement agrave un ensemble de techniques (gestion

financiegravere gestion comptable gestion des ventes gestion de la treacutesorerie hellip)

UNE ACTION SUR LES HOMMES

De nombreux auteurs ont souligneacute cet aspect du management En particulier

Initiation au management copy CRCF ndash J Sornet Page 4 48

- Peter Drucker pour qui le management repose sur cinq principes fixer des objectifs

organiser le travail motiver et communiquer former les eacutequipes (au sens de lrsquoapprentissage

laquo former les autres et soi-mecircme raquo)

- Henry Mintzberg qui met lrsquoaccent sur le rocircle unificateur du manager laquo Ce qui distingue

avant tout une organisation formelle drsquoun quelconque rassemblement drsquohommes ndash drsquoune

foule drsquoun groupe informel ndash crsquoest la preacutesence drsquoun systegraveme drsquoautoriteacute et drsquoadministration

personnifieacute par un ou plusieurs managers dans une hieacuterarchie plus ou moins structureacutee et

dont la tacircche est drsquounir les efforts de tous dans un but donneacute raquo

On en deacuteduit le rocircle relationnel du manager et le poids de la gestion des ressources

humaines dans son activiteacute

LA PRIMAUTE DE LrsquoINFORMATION

Le management fixe des objectifs il a un rocircle deacutecisionnel et il creacutee les conditions neacutecessaires

pour atteindre les objectifs dont il controcircle la reacutealisation Tout ceci neacutecessite la mise en place

drsquoun systegraveme drsquoinformation fournissant les donneacutees neacutecessaires agrave des choix pertinents

(donneacutees qui remontent souvent par les managers eux-mecircmes) puis agrave la mise en œuvre et

au controcircle des reacutealisations

23 ndash Les dimensions du management

La fonction de management a de fait trois dimensions humaine (faire travailler des

personnes ensemble motiver) eacuteconomique (fonctionnement ou deacuteveloppement de

lrsquoorganisation aux meilleurs conditions) et informationnelle (dont la communication)

Le management est une activiteacute de synthegravese qui neacutecessite des capaciteacutes agrave traiter et

combiner des informations drsquoorigines diverses plus ou moins varieacutees selon le contexte

(financiegravere technique leacutegale eacuteconomique politique hellip) et agrave agir en conseacutequence dans les

trois dimensions

Humain

Economique

Information

Informations sur

lrsquoenvironnement

Informations sur

lrsquoorganisation

Management

Information

dirigeant lrsquoaction

des autres

Actions directes

du manager

Initiation au management copy CRCF ndash J Sornet Page 5 48

24 ndash La recherche de performance

Le management recherche la performance de lrsquoorganisation en rapport avec ses objectifs

qui peuvent ecirctre de diffeacuterentes natures (expansion profit hellip ou agrave un niveau plus deacutetailleacute

reacuteduction des deacutelais flexibiliteacute reacuteduction des coucircts accroissement de la qualiteacute hellip)

La performance peut ecirctre abordeacutee de deux faccedilons

- par lrsquoefficaciteacute qui conduit agrave la reacutealisation des objectifs

- par lrsquoefficience qui conduit agrave une utilisation optimale des moyens disponibles avec le

meilleur rendement

Exemple une entreprise qui atteint son objectif de croissance de 10 du chiffre

drsquoaffaires et de son beacuteneacutefice est efficace mais celle qui arrive au mecircme reacutesultat avec

moins drsquoactifs et moins de personnel est plus efficiente

25 ndash Le champ drsquoaction du management

Le management est mis en œuvre dans des peacuterimegravetres drsquoampleur variable un groupe une

entreprise une activiteacute particuliegravere de lrsquoentreprise un projet la tenue drsquoun magasin hellip Les

actions de management ont ainsi des conseacutequences plus ougrave moins importantes

Le management srsquoexerce agrave tous les niveaux drsquoencadrement de lrsquoorganisation Il est de

coutume de distinguer trois niveaux

Exemple

Management strateacutegique la direction geacuteneacuterale a fixeacute les objectifs de chaque filiale du

groupe en tenant compte des positions souhaiteacutees sur le marcheacute Il en reacutesulte que les

uniteacutes X et Y aux activiteacutes similaires et compleacutementaires doivent preacuteparer leur fusion

preacutevue dans les deux ans La filiale Z situeacutee dans une ville universitaire doit acqueacuterir des

compeacutetences pour innover dans lrsquoapplication des supra conducteurs drsquoici trois ans Les

budgets et les financements correspondants ont eacuteteacute globalement estimeacutes

Management intermeacutediaire en application de ces orientations strateacutegiques les

directions de X et Y planifient des reacuteunions de travail communes pour voir comment

harmoniser leurs ressources humaines dans les douze mois Z deacutecide de contractualiser

ses relations avec le centre universitaire pour atteindre ses objectifs et mettre en place

un partenariat en recherche appliqueacutee

Management strateacutegique ou geacuteneacuteral

(direction laquo top management raquo deacutefinition

des objectifs geacuteneacuteraux)

Management tactique

(intermeacutediaire laquo middle

managers raquo)

Management opeacuterationnel (de

terrain ou drsquouniteacute local

laquo executive manager raquo)

Initiation au management copy CRCF ndash J Sornet Page 6 48

Management opeacuterationnel X et Y mettent en place leur communication avec le

personnel et un plan de reconversion Z nomme un directeur de recherche qui prend

notamment en charge les relations avec lrsquouniversiteacute

Le management recouvre la totaliteacute des actes de conduite de lrsquoorganisation dans tous les

domaines (technique commercial financier hellip) mais il ne correspond agrave aucune cellule de

lrsquoorganigramme

Les actions de management sont par ailleurs contraintes par la disponibiliteacute des ressources

neacutecessaire pour atteindre les objectifs (financement savoir-faire profil du personnel

eacutequipements hellip)

3 ndash La meacutethode laquo management raquo

31 ndash La science du management

Le management nrsquoest pas une science exacte il srsquoapparente agrave une science humaine

expeacuterimentale qui traite de pheacutenomegravenes socio-eacuteconomiques eacutevolutifs et qui doit trouver

concregravetement son application dans la vie des organisations

Cette science traite notamment de lrsquoorganisation des entreprises et rassemble des meacutethodes

et des theacuteories qui peuvent ecirctre regroupeacutees en eacutecoles ou en courants

Les theacuteories marquent geacuteneacuteralement une eacutepoque et elles peuvent se recouper partiellement

parfois srsquoopposer avec des nuances qui doivent ecirctre bien identifieacutees Elles srsquoaccompagnent

souvent de modegraveles et de scheacutematisations qui en facilitent la compreacutehension et la

transposition agrave de nouvelles situations

Ces outils scientifiques guident le raisonnement permettent drsquoappreacutehender des reacutealiteacutes

complexes et structurent les connaissances ils sont peacutedagogiques et constituent des aides

pour lrsquoaction Leur application doit cependant ecirctre raisonneacutee car

- un modegravele nrsquoest qursquoune simplification de la reacutealiteacute

- une theacuteorie sortie de son contexte historique et eacuteconomique peut perdre de sa pertinence

Sa mise en œuvre doit tenir compte de la situation reacuteelle et une theacuteorie ne peut agrave elle seule

justifier une deacutecision de management (ce drsquoautant plus que chaque theacuteorie ne couvre que

tregraves partiellement le domaine du management ou de lrsquoorganisation)

- il est souvent difficile de disposer agrave temps drsquoinformations fiables et suffisantes pour appliquer

une theacuteorie dans les conditions ideacuteales Le manager est freacutequemment ameneacute agrave prendre ses

deacutecisions sur la base drsquoinformations incomplegravetes ou incertaines et il doit alors en mesurer les

risques et preacutevoir les ajustements neacutecessaires

ENTREPRISE

Management Administration

Gestion

Strateacutegique

Opeacuterationnel

Initiation au management copy CRCF ndash J Sornet Page 7 48

- des modes influencent le management Elles peuvent indiquer une veacuteritable eacutevolution

eacuteconomique mais aussi ecirctre sans lendemain voire introduire un danger ou un coucirct inutile

(se meacutefier des speacutecialistes dont le fond de commerce est la vente de nouvelles techniques

de management et des seacuteminaires associeacutes)

- seule la creacuteation drsquoune combinaison originale (dans le respect des regravegles) peut apporter un

avantage agrave lrsquoentreprise et non la reproduction de choix de management connus de tous

Exemple 1 la matrice de portefeuille drsquoactiviteacutes permet de classer les activiteacutes

strateacutegiques drsquoune entreprise en fonction de leur taux de croissance et de la part de

marcheacute deacutetenue (matrice laquo BCG raquo - Boston consulting group ndash 1975) Cet outil de

management neacutecessite de disposer drsquoinformations fiables concernant le marcheacute Il

conduit souvent agrave simplifier les conditions de concurrence et ignore la

compleacutementariteacute pouvant exister entre activiteacutes (synergie partage de techniciteacute

amortissement de charges fixes hellip) La matrice est un moyen de prendre conscience

du portefeuille et de ses eacuteventuels deacutefauts (portefeuille deacuteseacutequilibreacute avec

preacutedominance anormale drsquoune zone) mais il ne permet pas seul de deacutecider de lrsquoavenir

des activiteacutes

Exemple 2 le lancement drsquoun investissement lourd (lrsquoimplantation de nouvelles usines

le lancement drsquoune nouvelle activiteacute agrave fort taux de recherche ndash deacuteveloppement hellip) ne

peut ecirctre deacutecideacute que par le recoupement de diffeacuterentes approches (financiegravere

commerciale strateacutegique) Aucune theacuteorie du management appliqueacutee isoleacutement et

sans preacutecaution ne peut justifier un tel investissement

Exemple 3 une entreprise produisant des eacutequipements meacutedicaux deacutecide de renforcer

son offre commerciale par un service de conseil et drsquoassistance Cette deacutecision est

inspireacutee par le concept de creacuteation de valeur par une volonteacute de se diffeacuterencier de la

concurrence et par des pratiques existant depuis longtemps dans le domaine

informatique

Remarque parmi les facteurs influenccedilant les principes de management il est utile de

savoir deacutetecter pour srsquoen preacutemunir les ideacuteologies preacutejugeacutes et autres laquo valeurs raquo sans

rapport certain avec lrsquoefficaciteacute eacuteconomique

32 ndash Theacuteorie et pratique du management

La litteacuterature (ouvrages revues) aborde le management sous des angles diffeacuterents qui

peuvent suggeacuterer une concurrence entre des approches theacuteorique et pratique ou

psychologique et eacuteconomique de cette discipline

Taux de croissance

du domaine (cf

cycle de vie du

produit)

Part de marcheacute

(compareacutee au

principal

concurrent)

fort

faible

forte faible

Dilemmes (activiteacutes

en phase de

lancement

potentiel + coucircts+

risque+)

Vedettes

(activiteacutes en forte

croissance

autofinancement+)

Vaches agrave lait

(activiteacutes

stabiliseacutees et

concurrence faible

ou stable

rentabiliteacute+)

Poids morts

(activiteacutes en

deacuteclin)

Initiation au management copy CRCF ndash J Sornet Page 8 48

Le management a cependant une viseacutee unique et concregravete la bonne marche des

organisations et les travaux des universitaires rejoignent lrsquoaction des praticiens (les

universitaires reacutealisent drsquoailleurs bien souvent leurs recherches au sein des organisations)

Au plan peacutedagogique les deux visions du management se complegravetent

- la vision acadeacutemique met en perspective des concepts et des theacuteories Elle permet de

srsquoapproprier des raisonnements essentiels et stables notamment concernant la strateacutegie et

les reacuteactions humaines qui sont le moteur des organisations

- la vision pratique relate le veacutecu des praticiens preacutesente des techniques concregravetes et tente

parfois une vulgarisation favorisant la diffusion des concepts Elle integravegre de faccedilon

pragmatique les objectifs eacuteconomiques et les contraintes de fonctionnement des

organisations en se placcedilant parfois dans une vision agrave court terme

Sur le terrain le management reacutealise une synthegravese entre des techniques de psychologie

sociale et des techniques de gestion de diverses origines et de nouvelles meacutethodes

apparaissent aussi sous le label unique laquo management raquo

Le terme manageacuterial (approche manageacuteriale theacuteorie manageacuteriale pratique manageacuteriale

hellip) fait reacutefeacuterence agrave la vision aux preacuteoccupations et aux actions des managers qui doivent

emmener leur organisation vers la reacutealisation de ses objectifs

Exemples

- la deacutetermination des coucircts et des marges reacutesulte de techniques de gestion

indispensable aux deacutecisions des managers

- le laquo CRM raquo (customer relationship management en franccedilais GRC ndash gestion de la

relation client) est un concept reacutecent qui srsquoaccompagne de techniques lieacutee aux plus

reacutecents deacuteveloppements du management et des systegravemes drsquoinformation

- le laquo coaching raquo est une technique de management permettant une eacutevolution

personnelle dans le sens des objectifs de lrsquoentreprise

- le recentrage sur le laquo meacutetier raquo repose sur des techniques de management

Les techniques sont rassurantes mais pas suffisantes et si les theacuteories ne sont pas

indispensables au praticien elles aident agrave comprendre agrave anticiper et agrave bien utiliser les

techniques

La qualiteacute du management reacuteside beaucoup dans la capaciteacute agrave appliquer concepts et

techniques de faccedilon pertinente et agrave innover Cette capaciteacute relegraveve en partie drsquoun laquo art du

management raquo qui srsquoacquiert en grande partie par la pratique

4 ndash Management et expertise comptable

Lrsquoexpert comptable doit manager ses propres eacutequipes Il est par ailleurs supposeacute laquo hellip

conseiller et accompagner le chef drsquoentreprise dans toutes ses deacutecisions hellip raquo (selon lrsquoOEC)

Ce rocircle est particuliegraverement important dans ses relations avec les petites entreprises Il doit

donc avoir une capaciteacute au management

Organisation

Vision acadeacutemique Vision pratique

Initiation au management copy CRCF ndash J Sornet Page 9 48

Lrsquointervention de lrsquoexpert dans le management drsquoune entreprise peut toutefois poser

quelques problegravemes

- Il peut y avoir conflits drsquointeacuterecirct entre activiteacutes de certification des comptes et de conseil en

management (la tendance est agrave la seacuteparation des activiteacutes dans les plus grands cabinets)

- lrsquoactiviteacute de conseil neacutecessite des compeacutetences parfois tregraves speacutecifiques (conseil fiscal

conseil en RH conseil en systegravemes drsquoinformation hellip)

- le conseil est une activiteacute diffeacuterente par sa forme de lrsquoexpertise comptable (interventions

longues peu reacutepeacutetitives peu codifieacutees mises en concurrence) qui neacutecessite une

organisation particuliegravere du cabinet lorsqursquoelle deacutepasse lrsquointervention occasionnelle

APPLICATIONS IM

IM1 Analyser la profession de laquo manager raquo selon Henry Mintzberg (texte extrait de lrsquoouvrage

laquo Le management raquo Eyrolles - Editions drsquoOrganisation) et les principes du management de la

norme ISO

Faire ressortir les eacuteleacutements speacutecifiques agrave chacune de ces approches et mettre en eacutevidence

leurs points communs

Initiation au management copy CRCF ndash J Sornet Page 10 48

Principes du management drsquoapregraves la norme ISO 9001 (2000)

- Orientation vers le client (satisfaire ses attentes)

- Leadership (les dirigeants eacutetablissent les orientations de lrsquoorganisme Ils doivent creacuteer

un environnement interne ougrave les personnes peuvent clairement srsquoimpliquer dans la

reacutealisation des objectifs de lrsquoorganisme)

- Implication du personnel (les personnes sont agrave tout niveau lrsquoessence de lrsquoorganisme et

leur implication permet drsquoutiliser leurs aptitudes au profit de lrsquoorganisme)

- Approche laquo processus raquo (un reacutesultat est mieux atteint quand les ressources et les

activiteacutes neacutecessaires sont geacutereacutees comme un processus)

- Approche systegraveme (assimiler les processus correacuteleacutes agrave un systegraveme contribue agrave

lrsquoefficaciteacute et agrave lrsquoefficience de lrsquoorganisme vis-agrave-vis de ses objectifs)

- Ameacutelioration continue (objectif permanent de lrsquoorganisme)

- Prise de deacutecision efficace (par lrsquoanalyse de donneacutees et drsquoinformations)

- Relations mutuellement beacuteneacutefiques avec les fournisseurs (pour augmenter la capaciteacute

des deux organismes agrave creacuteer de la valeur)

IM2 Distinguer leader et manager

IM3 Compleacuteter le tableau ci-dessous en analysant chaque action preacutesenteacutee Faire ensuite

ressortir les domaines niveaux ou techniques de management pouvant ecirctre mobiliseacutes pour

chaque situation

Initiation au management copy CRCF ndash J Sornet Page 11 48

Caracteacuteristiques

de lrsquoaction

- reacutepeacutetition

- risque

- normes

- ampleur

Prise de

deacutecision

- opeacuterationnelle

strateacutegique

- deacutelai

Informations

neacutecessaires

- nature

- origine

- deacutelai obtention

Cleacutes pour la

reacuteussite

Intervention

exteacuterieure

possible

Assurer la

restauration du

soir

(restaurant

familial)

Construire un

viaduc

(autoroute)

Certifier les

comptes

annuels drsquoun

groupe

national

(cabinet

drsquoaudit)

Lancer une

ligne drsquoavions

(constructeur

aeacuteronautique)

Reacuteduire la

capaciteacute de

production

(groupe

industriel)

Acqueacuterir une

entreprise

concurrente

(teacuteleacutephonie

mobile)

Initiation au management copy CRCF ndash J Sornet Page 12 48

Fiche IM1 - Deacutefinitions du management

Dictionnaire anglais - franccedilais direction administration gestion intrigue manegravege

Wikipeacutedia Le management est lensemble des techniques dorganisation qui sont mises en

oeuvre pour ladministration dune entiteacute

Au point de vue eacutetymologique le verbe manage vient de litalien maneggiare (controcircler)

influenceacute par le mot franccedilais manegravege (faire tourner un cheval dans un manegravege) A cette

notion il faut aussi ajouter la notion de meacutenage (geacuterer les affaires du meacutenage) qui consiste agrave

geacuterer des ressources humaines et des moyens financiers

helliphellip

Fiche IM2 - Etudier le management

Concreacutetiser

Manager neacutecessite de syntheacutetiser des informations parfois complexes incomplegravetes et de

domaines tregraves divers pour en deacuteduire des actions Une approche trop parcellaire peut

conduire agrave lrsquoeacutechec et le savoir-faire est neacutecessaire pour agir vite avec un minimum de risque

Lrsquoeacutetudiant doit se preacuteparer simultaneacutement aux examens et agrave la pratique Il nrsquoa souvent connu

lrsquoentreprise que durant quelques semaines de stage et le manque de laquo recul raquo ne lui permet

pas toujours de concreacutetiser les theacuteories Il doit compenser par la lecture (ouvrages revues

journaux eacuteconomiques et boursiers) et en eacutetant attentif aux informations ambiantes (tout en

relativisant le style journalistique) en mettant en relation le cours les concepts les modegraveles

lrsquoactualiteacute les stages

Savoir traiter un exercice

Pour reacuteussir un examen ou traiter une application peacutedagogique (la conception les points 1 agrave

6 peut repreacutesenter le tiers du temps de travail)

1 ndash Identifier le type de sujet (faut-il trouver une solution pratique ou communiquer une

reacuteflexion geacuteneacuterale )

2 ndash Lire le sujet et relever les mots cleacutes

3 ndash Deacutefinir les mots cleacutes

4 ndash Reacutesumer la probleacutematique du sujet (en quelques lignes)

5 ndash Lister les connaissances reacutefeacuterences et raisonnements reacutepondant au problegraveme (par

recherche spontaneacutee ou raisonneacutee qui quoi ougrave quand comment combien hellip

listage des diffeacuterents points de vue) trouver des exemples (notamment dans les

documents fournis)

6 ndash Organiser la reacuteponse (deacutefinir le plan du deacuteveloppement ougrave des paragraphes bien

identifieacutes sont geacuteneacuteralement neacutecessaires en y liant les parties qui doivent ecirctre en nombre

limiteacute ndash de deux agrave quatre) Preacutevoir drsquoy inteacutegrer la deacutefinition des principales notions

induites par le sujet

7 ndash Reacutediger sous la forme adapteacutee (note technique ou recommandation solution

pratique exposeacute structureacute dissertation)

Introduction et conclusion sont indispensables agrave la dissertation ou agrave lrsquoexposeacute

- lrsquointroduction preacutesente le sujet traiteacute (phrase drsquoaccroche initiale) amorce la

probleacutematique (quelques sous - questions) et annonce le plan

- la conclusion syntheacutetise le deacuteveloppement (arguments) eacutelargit le sujet (prise de recul)

et apporte le point final (une phrase)

Une limite agrave la communication

Il est difficile de faire passer plus de 4 ou 5 ideacutees fortes dans un exposeacute unique

Initiation au management copy CRCF ndash J Sornet Page 13 48

Fiche IM3 - Bref historique

Antiquiteacute

3000 AJC

Peacuteriode greacuteco-

romaine

Transition

feacuteodale

12egraveme siegravecle

europe

15egraveme ndash 17egraveme

siegravecles

19egraveme siegravecle

20egraveme siegravecle

agriculture preacutedominante industrie limiteacutee aux besoins drsquoun individu ou drsquoun clan

pour la confection des outils des vecirctements et de la poterie Force motrice animale

ou humaine pour lrsquoessentiel

Grands travaux drsquoeacutetat en Egypte premiegravere laquo planification ndash organisation ndash controcircle raquo

Deacuteveloppement des communications essor industriel limiteacute peu de progregraves

technique (lrsquoesclavage supplante les innovations)

Deacuteveloppement progressif des eacutechanges commerciaux

La consommation indirecte atteint un bon niveau (surplus agricoles et

deacuteveloppement des villes) Apparition de nouveaux commerccedilants

Etat fort Evolutions technologiques (imprimerie bateaux performants instruments de

navigation) Extension geacuteographique de lrsquoeacuteconomie Apparition des corporations

drsquoartisans

Machine agrave vapeur chemin de fer passage de lrsquoartisanat au capitalisme

entrepreneurial producteur organisation des entreprises

Ecole classique (Taylor Fayol Weber) approche meacutecaniste bureaucratie

hieacuterarchie commandement fonctions et speacutecialisation laquo OS T raquo (organisation

scientifique du travail) organisation source de pouvoir rationaliteacute des individus bases

du management

Deacuteveloppement du capitalisme manageacuterial Electriciteacute peacutetrole puis communications

et information Consommation de masse mondialisation preacuteoccupations

eacutenergeacutetiques et environnementales 3 peacuteriodes

- standardisation grandes entreprises industrielles

- industries de consommation 30 glorieuses marketing multinationales protection

sociale

- deacutereacuteglementation monteacutee des services pays eacutemergents mondialisation et nouvelle

eacuteconomie (internet)

Ecole des relations humaines prise en compte de lrsquoindividu des motivations styles

de direction

Ecole neacuteo-classique et post-classique deacutecentralisation coordonneacutee DPO

management participatif zeacutero deacutefaut flux tendus

Approche systeacutemique partition de lrsquoentreprise eacutetude des interactions feacutedeacuteration

vers lrsquoobjectif controcircle et ajustement

Theacuteories de la deacutecision rationaliteacute limiteacutee contribution reacutetribution coalitions

Ecole socio-technique recherche de compromis technologie organisation

enrichissement des tacircches autonomie des groupes

Approche sociologique effets sociaux du travail jeux de pouvoir dans lrsquoentreprise

reacutegulation sociale

Theacuteories de la contingence facteurs contingents adaptation agrave lrsquoenvironnement

configurations organisationnelles

Theacuteories de la firme controcircle manageacuterial droits de proprieacuteteacute relation drsquoagence

Theacuteories contractualistes firme nœud de contrats coucircts de transaction

opportunisme externalisation internalisation

Approche eacutevolutioniste eacutecologie des organisations modegravele eacutevolutioniste

contraintes de sentier

Approche par les ressources valorisation des ressources compeacutetences cleacutes

apprentissage organisationnel

(Classement simplifieacute)

Initiation au management copy CRCF ndash J Sornet Page 14 48

ELEMENTS DE CORRIGE IM

IM1 Commenter la deacutefinition du management par la norme ISO et le manager de Mintzberg

Efficient = optimum avec les moyens disponibles

ISO (management objectifs) (manager moyens) HM

IM2 Le leader entraicircne naturellement derriegravere lui Le manager nrsquoest pas toujours leader

(mecircme si crsquoest souhaitable) Le leader nrsquoest pas toujours manager (plutocirct notion individuelle)

Leadership = faculteacute de diriger conjugaison drsquoune autoriteacute naturelle ou drsquoun savoir-faire

acquis drsquoune capaciteacute agrave entraicircner des personnes ou des groupes et drsquoune leacutegitimiteacute

statutaire (de position)

IM3 Compleacuteter le tableau ci-dessous en analysant chaque action preacutesenteacutee Faire ensuite

ressortir les domaines niveaux ou techniques de management pouvant ecirctre mobiliseacutes pour

chaque situation

Satisfaction client

Implication du personnel

Processus systegraveme

Ameacutelioration continue

Deacutecision efficace

Recherche de valeur

Image entreprise

Liaisons

Information

Reacutepartition ressources

Reacutegulation

Neacutegociation

Leadership

Initiation au management copy CRCF ndash J Sornet Page 15 48

Caracteacuteristiques

de lrsquoaction

- reacutepeacutetition

- risque

- normes

- ampleur

Prise de

deacutecision

- opeacuteration

- direction

- deacutelai

Informations

neacutecessaires

- nature

- origine

- deacutelai

obtention

Cleacute pour la

reacuteussite

Intervention

exteacuterieure

possible

Assurer la

restauration du

soir

(Restaurant

familial)

Technique

(fabrication)

Vente (terrain)

Appros

Reacutepeacutetitive

(quot)

Risque faible

Normes

drsquohygiegravene

Faible

Opeacuterationnelle

Geacuterant

responsable

Rapide (qq

jours menu et

appros)

Nombre de

couverts

Tarifs usuels

Calendrier

(fecirctes)

Clients docs

divers

expeacuterience

Qq jours

Varieacuteteacute menu

Plats phares

Accueil

Appros

Tarification

Vins

Gestion

congeacutelation

Qualiteacute cuisine

Fournisseurs

Extra

Publiciteacute

Construire un

viaduc

(autoroute)

Technique

Organisation

Appros

Uniteacute (ou peu)

Eleveacute (financier

technique)

Architecture

Eleveacutee

Direction

(aleacuteas)

Opeacuterationnelle

(conduite

chantier)

Immeacutediat agrave qq

semaines

Plans

plannings

Qualifications

Meacuteteacuteo

Disponibiliteacutes

Bureau eacutetudes

Qq sem agrave 24h

Techniciteacute

Appros

Qualifications

Preacutevision

GRH

Contrat juste

SS traitants

Organismes

certificateurs

Controcircle

client

Certifier les

comptes

annuels drsquoun

groupe national

(cabinet

drsquoaudit)

Technique

Relation client

Gestion des

connaissances

Annuelle

Moyen

Regravegles

comptables

fiscales

Moyenne (selon

importance du

cabinet)

Opeacuterationnelle

Qq jours agrave

semaines

Comptable

Juridique

Client

Etat

Qq jours agrave

semaines

Techniciteacute

Expeacuterience

Relation client

Systegraveme info client

Siegravege

Autre cabinet

Lancer une

ligne drsquoavions

(constructeur

aeacuteronautique)

Strateacutegique

RD

Etudes

Uniteacute

Tregraves eacuteleveacute

Aeacuteronautique

Tregraves eacuteleveacutee

Direction

Qq mois agrave

anneacutees

Marcheacute

Etudes

Compagnies

Qq mois agrave

anneacutees

Concept

Outil industriel

Coucirct exploitation

Tarif

Fiabiliteacute

Deacutelaisconcurrence

SI simulation

SS traitants

Bureaux

drsquoeacutetudes

speacutecialiseacutes

Compagnies

Conseils

Reacuteduire la

capaciteacute de

production

(groupe

industriel)

Strateacutegique

RH

Communication

Production

Uniteacute

Moyen

Leacutegislation

(dont RH)

Eleveacutee

Direction

Qq mois agrave

anneacutees

Financiegravere

Industrielle

Marcheacute

Organisation

Organismes

speacutecialiseacutes

DRH

Qq mois

Communication

Connaissance des

compeacutetences

Connaissance outil

industriel

Concurrence

Portefeuille

drsquoactiviteacutes

Cabinet

drsquoorganisation

Conseils

speacutecifiques

Acqueacuterir une

entreprise

concurrente

(teacuteleacutephonie

mobile)

Strateacutegique

Marketing

Production

(reacuteseau)

Financier

Communication

Uniteacute

Tregraves eacuteleveacute

Leacutegislation

telecom

Tregraves eacuteleveacutee

Direction

Qq mois

Financiegravere

Marcheacute

Reacuteseaux

(ampleur

recouvrement

hellip)

Organisations

Interne

Racheteacutee

Sources

speacutecialiseacutees

Qq mois

Communication

Marcheacute

Cours boursiers

Cabinet

drsquoorganisation

Conseils

speacutecifiques

Initiation au management copy CRCF ndash J Sornet Page 16 48

LE MANAGEMENT EN PRATIQUE

Pour assumer sa fonction le management doit couvrir sans discontinuiteacute lrsquoensemble de

lrsquoorganisation et inteacutegrer de nombreux facteurs dont nous allons reacutesumer lrsquoessentiel

1 ndash Les fonctions et activiteacutes du management

Pour Henri Fayol la fonction drsquoadministration de lrsquoentreprise (son management) reposait sur

cinq actions preacutevoir organiser commander coordonner et controcircler (laquo PO3C raquo)

Nous distinguerons cinq activiteacutes de management

- la conception (au plus haut niveau finaliteacute but ou vocation de lrsquoorganisation

meacutetiers dimension politique de croissance hellip)

- la planification (deacutefinition des objectifs eacutecheacuteances)

- lrsquoorganisation (reacutepartition du travail choix des modes de coordination)

- le pilotage de lrsquoaction opeacuterationnelle (motivation animation encadrement

assistance)

- lrsquoeacutevaluation (controcircle des reacutesultats obtenus ajustements)

Dans chacune de ces activiteacutes des deacutecisions et des arbitrages sont neacutecessaires avec des

enjeux plus ou moins importants

Remarques

- Les cinq activiteacutes du management peuvent se retrouver agrave tout niveau de

management si lrsquoentreprise laisse une certaine autonomie de deacutecision agrave ses diffeacuterentes

uniteacutes La conception est naturellement du ressort de la direction geacuteneacuterale et des

conseils drsquoadministration mais elle peut ecirctre preacutesente pregraves du terrain (latitude laisseacutee agrave

une filiale ou agrave un magasin par exemple) De mecircme lrsquoorganisation du travail concerne

un atelier mais aussi la direction qui structure lrsquoentreprise pour assurer ses activiteacutes sa

production

- La planification deacutefinit des objectifs ou des axes strateacutegiques (choix de produits

modaliteacutes de deacuteveloppement des ventes implantations alliances hellip) et les traduit en

donneacutees de gestion preacutevisionnelles syntheacutetiques et eacutechelonneacutees dans le temps afin de

valider les objectifs et de fixer des repegraveres

- Un laquo business plan raquo (plan drsquoaffaires)est notamment lrsquoeacutequivalent de la planification

dans le cas de creacuteation drsquoentreprise ou pour la preacutesentation de tout projet drsquoactiviteacute

Les activiteacutes du management srsquoinscrivent dans des cycles qui peuvent ecirctre scheacutematiseacute

comme suit (lrsquoeacutevaluation peut entraicircner une reacutevision du pilotage de lrsquoorganisation ou des

objectifs sans que lrsquoentreprise ne soit fondamentalement remise en cause)

conception

planification

organisation

pilotage

eacutevaluation

Initiation au management copy CRCF ndash J Sornet Page 17 48

2 ndash Les contextes de management

Le management est influenceacute par son contexte qui justifie des objectifs une organisation

des meacutethodes

Par exemple lrsquoentreprise admet de nombreuses variantes selon sa taille sa forme juridique

son controcircle par lrsquoeacutetat (entreprises publiques) ou par des inteacuterecircts priveacutes Il en va de mecircme des

organismes administratifs qui peuvent deacutependre de directives nationales ou reacutegionales des

associations qui ont des activiteacutes drsquoampleur tregraves variable

21 ndash La dimension de lrsquoentreprise

La dimension drsquoune entreprise se mesure principalement en fonction de son effectif ou de

son chiffre drsquoaffaires Des seuils sont deacutefinis par divers organismes et exploiteacutes agrave des fins

statistiques ou pour la deacutetermination de certaines obligations sociales ou fiscales

(repreacutesentation du personnel cotisations hellip) Il nrsquoy a bien entendu pas de laquo barriegravere de

tailleraquo absolue conditionnant le management drsquoune entreprise

LrsquoUE preacuteconise de distinguer les micro ndash entreprises (jusqursquoagrave 9 salarieacutes) les TPE ndash tregraves petites

entreprises (moins de 20 salarieacutes) les petites entreprises (moins de 50) et les moyennes

entreprises (de 50 agrave 250) Cependant les PME sont parfois situeacutees entre 10 et 500 salarieacutes

Remarques

- en France environ 40 des entreprises emploient de 1 agrave 50 salarieacutes (ce qui repreacutesente

plus de 50 des emplois) et 59 nrsquoen ont aucun

le pays compte environ 2 600 000 entreprises dont moins de 1 ont 250 employeacutes et

plus

- ancienneteacute et taille de lrsquoentreprise sont lieacutees si lrsquoon eacutecarte les restructurations et autres

eacutevolutions drsquoentreprises existantes

La dimension de lrsquoentreprise a une influence sur lrsquoorganisation et le laquo style raquo de son

management

- les PME sont souvent entrepreneuriales (les dirigeants eacutegalement apporteurs de capitaux

sont totalement engageacutes dans la marche de lrsquoentreprise) Elles ont une gestion flexible peu

formaliseacutee plus qualitative que quantitative Les PME sont freacutequemment focaliseacutees sur un seul

type drsquoactiviteacute Pour ne pas alourdir leur structure elles ont tendance agrave sous-traiter les

activiteacutes speacutecialiseacutees ne correspondant pas agrave leur meacutetier de base

- les grandes entreprises sont manageacuteriales (les dirigeants sont nommeacutes par les actionnaires

en raison de leurs compeacutetences) et moins reacuteactives

22 ndash Le type de production

On distingue industrie (production de biens mateacuteriels ou pour le moins de produits visibles ndash

comme un seacutejour touristique ou un film) et services (fourniture drsquoune prestation immateacuterielle)

Le type de production influence en principe le management de lrsquoentreprise

- lrsquoindustrie neacutecessite (si lrsquoon excepte lrsquoartisanat) un investissement relativement important

une organisation productive stable capable de reacutealiser plusieurs fois des produits identiques

(exemple un modegravele de reacutefrigeacuterateur) ou du moins similaires (exemple un bacirctiment) Le

produit de lrsquoindustrie consomme des matiegraveres et il doit geacuteneacuteralement ecirctre distribueacute jusqursquoau

client

- la production de services peut se satisfaire drsquoun investissement tregraves reacuteduit et neacutecessite un

contact permanent avec le client

Toutefois la standardisation des services et le deacuteveloppement des reacuteseaux informatiques

rapprochent la production de services de celle des biens industriels

- la production drsquoun service reacutepeacutetitif et technique peut imposer une structure lourde et une

organisation tregraves formaliseacutee (voir les grandes socieacuteteacutes drsquoaudit ou de conseil informatique)

Initiation au management copy CRCF ndash J Sornet Page 18 48

- certains services peuvent ecirctre fournis agrave distance sans contact direct avec le client et

distribueacutes par reacuteseau (tenue de comptabiliteacute affacturage gestion clientegravele centre drsquoappel

hellip)

Remarque les services repreacutesentent 75 de lrsquoactiviteacute eacuteconomique franccedilaise

23 ndash La nature de lrsquoorganisation

Les organisations publiques franccedilaises (administrations centrales collectiviteacutes territoriales

hocircpitaux hellip) repreacutesentent une part importante de lrsquoactiviteacute (environ 30 des emplois) La

fonction publique regroupe des organisations aux finaliteacutes diverses et qui ont des problegravemes

de gestion similaires agrave ceux des entreprises auxquelles elles peuvent emprunter des principes

de management Notamment

- pour controcircler les coucircts et assurer la qualiteacute des services

- pour communiquer avec les administreacutes ou les usagers

- pour motiver les personnels et geacuterer les ressources humaines

La transposition directe des techniques de gestion et de management nrsquoest cependant pas

toujours possible car

- la comptabiliteacute publique obeacuteit agrave des regravegles speacutecifiques (proceacutedure budgeacutetaire

notamment)

- le laquo client raquo ne paye pas toujours la prestation du moins directement

- la concurrence est parfois inexistante

- les grandes administrations centraliseacutees sont soumises agrave des choix politiques geacuteneacuteraux

parfois sans connexion eacutevidente avec les besoins opeacuterationnels

- le statut des personnels et les grilles de salaires limitent les possibiliteacutes de gestion des

ressources humaines

Remarque la LOLF (loi organique relative aux lois de finances) est entreacutee en vigueur en

2006 Elle alloue des moyens budgeacutetaires en fonction de programmes et remplace la

reconduction automatique de 90 des budgets Cette reacuteforme se heurte toutefois agrave la

lourdeur des grands ministegraveres ougrave la complexiteacute des activiteacutes est difficile agrave

appreacutehender et ougrave des inerties culturelles peuvent exister agrave tout niveau

Les associations loi de 1901 peuvent avoir une activiteacute comparable agrave celle de grandes

entreprises (voir par exemple les associations de santeacute ou professionnelles) et leur

management est alors similaire malgreacute lrsquoabsence de but lucratif (les beacuteneacutefices ne sont pas

distribuables) Elles ont drsquoailleurs en France un poids eacuteconomique important (elles emploient

environ 1 600 000 salarieacutes)

Cependant lrsquoadheacutesion agrave un systegraveme de valeurs fondateur de lrsquoassociation ou la limite de

lrsquoautoriteacute (quand un volant de beacuteneacutevoles important participe agrave lrsquoactiviteacute) peut introduire des

nuances

- le renforcement des objectifs socieacutetaux

- la faiblesse des relations hieacuterarchiques

- des contraintes de gestion du temps des beacuteneacutevoles

- des modaliteacutes particuliegraveres de recrutement et de motivation des dirigeants

24 ndash Les facteurs contingents

La theacuteorie de la contingence montre qursquoune structure drsquoentreprise nrsquoest efficace que dans

une situation deacutetermineacutee et qursquoil nrsquoexiste que des solutions de management construites dans

un contexte preacutecis

Le management doit ainsi srsquoadapter agrave des facteurs contingents qui ne peuvent ecirctre

controcircleacutes du moins agrave bregraveve eacutecheacuteance Ces facteurs sont par exemple

- lrsquoancienneteacute de lrsquoentreprise (plus elle est ancienne plus lrsquoentreprise a tendance agrave reacutepeacuteter

des comportements eacuteprouveacutes)

Initiation au management copy CRCF ndash J Sornet Page 19 48

- la taille de lrsquoentreprise (la grande entreprise a une composante administrative plus

deacuteveloppeacutee)

- le systegraveme de production (tregraves standardiseacute complexe automatiseacute hellip)

- lrsquoenvironnement

3 ndash Le management et les parties prenantes

Lrsquoentreprise a pour vocation premiegravere de mettre des produits agrave disposition de ses clients en

reacutealisant un profit Pour y arriver elle doit aussi satisfaire ses parties prenantes salarieacutes

actionnaires fournisseurs hellip

Est partie prenante agrave lrsquoentreprise laquo tout groupe ou individu qui peut ecirctre affecteacute ou est

affecteacute par les buts de lrsquoorganisation hellip raquo (Freeman ndash 1984)

Les parties prenantes attendent agrave des degreacutes divers de profiter drsquoune creacuteation de valeur en

provenance de lrsquoentreprise qui doit reacutepondre agrave ces attentes pour assurer sa peacuterenniteacute ou

favoriser son deacuteveloppement

On distingue les parties prenantes primaires ou principales qui sont essentielles agrave lrsquoentreprise

et qui ont geacuteneacuteralement une relation formelle avec elle (clients associeacutes et actionnaires

precircteurs salarieacutes fournisseurs collectiviteacutes) et les parties prenantes secondaires dont

lrsquoinfluence est diffuse (groupes de pression associations meacutedias instances europeacuteennes

agences de notation hellip)

Remarque la consideacuteration de lrsquoensemble des parties prenantes (laquo stakeholders raquo - les

deacutepositaires) fait contrepoids agrave lrsquoimportance accordeacutee aux seuls actionnaires

(laquo shareholders raquo)

Les organisations nrsquoayant pas drsquoobjectif de profit doivent aussi satisfaire leurs parties

prenantes apporter un service aux usagers dans les meilleures conditions eacuteconomiques

limiter un budget assurer la qualiteacute des relations avec les fournisseurs hellip

Dans cette optique le management doit organiser lrsquoaction de faccedilon agrave eacutequilibrer des forces

parfois divergentes

- le contexte fait pression sur lrsquoorganisation contrainte agrave optimiser ses reacutesultats

- lrsquoorganisation cherche par son action agrave assurer sa peacuterenniteacute son deacuteveloppement (en

reacutealisant des profits dans le cas de lrsquoentreprise) et agrave satisfaire ses parties prenantes

- le management agit en pilotant les actions pour contrebalancer la pression du contexte

Actions de

lrsquoorganisation

Management Contexte

Parties

prenantes

Initiation au management copy CRCF ndash J Sornet Page 20 48

APPLICATIONS MP

MP1 Deacutefinir contingent gestion budgeacutetaire

MP2 Deacuteterminer les parties prenantes drsquoun hocircpital public et leurs principales attentes

Mecircme question pour les organisations suivantes

- SNCF (entreprise publique)

- Peugeot

- MAIF (mutuelle drsquoassurance)

MP3 En les situant dans le cycle des activiteacutes du management trouver les actions agrave mener

dans les situations suivantes

- baisse de 10 des ventes dans une entreprise industrielle (produits meacutenagers le reacuteseau de

distribution vient drsquoecirctre reacuteorganiseacute)

- idem dans une entreprise de vente par correspondance soumise agrave la concurrence internet

(les ventes stagnaient depuis six mois malgreacute les efforts promotionnels)

- augmentation des deacutelais drsquoattente des consultations dans une clinique (lrsquohocircpital voisin a

fermeacute son service drsquourgences)

Initiation au management copy CRCF ndash J Sornet Page 21 48

ELEMENTS DE CORRIGE MP

MP1 Deacutefinir (dans le contexte drsquoune entreprise) contingent gestion budgeacutetaire

Contingent = imposeacute par lrsquoexteacuterieur Contingence = effet du hasard de la rencontre de

plusieurs eacuteveacutenements indeacutependants (variables explicatives que lrsquoon ne peut influencer)

Gestion budgeacutetaire = technique drsquoadministration des entreprises srsquoappuyant sur des

preacutevisions dont on deacuteduit apregraves accord des responsables des attributions de moyens sur une

dureacutee limiteacutee Une analyse reacuteguliegravere des eacutecarts entre preacutevisions et reacutealisations permet ensuite

le pilotage des activiteacutes Le budget est un cadre incitatif

La laquo planification budgeacutetaire raquo consiste agrave traduire en budgets une planification strateacutegique

avec systegraveme de reporting

MP2 Deacuteterminer les parties prenantes drsquoun hocircpital public et leurs principales attentes

Mecircme question pour les organisations suivantes

- SNCF (entreprise publique)

- Peugeot

- MAIF (mutuelle drsquoassurance)

Hocircpital

- patients (qualiteacute des soins)

- CNAM (baisse des coucircts)

- collectiviteacute locale (service aux administreacutes)

- eacutetat (ameacutenagement du territoire maicirctrise des budgets optimisation)

- employeacutes (salaire conditions de travail et satisfaction)

- fournisseurs ndash pharmacie autres (CA paiement reacutegulier)

- associations de patients (qualiteacute proximiteacute des soins)

SNCF

- usagers et associations drsquousagers (proximiteacute reacutegulariteacute prix du service)

- reacuteseau ferreacute de France (optimisation des lignes paiement adapteacute)

- fournisseurs (CA paiement reacutegulier)

- employeacutes (salaire conditions de travail seacutecuriteacute de lrsquoemploi)

- eacutetat (ameacutenagement du territoire)

- collectiviteacutes locales (service)

Peugeot

- clients (qualiteacute prix SAV relation commerciale)

- fournisseurs (CA reacutegulariteacute de lrsquoactiviteacute)

- employeacutes (salaire conditions de travail seacutecuriteacute de lrsquoemploi)

- eacutetat (taxes)

- collectiviteacute locale (emploi dynamisation eacuteconomique preacuteservation de lrsquoenvironnement)

- associations de protection de lrsquoenvironnement (activiteacute propre baisse des eacutemissions

nouvelles eacutenergies)

MAIF

- socieacutetaires (protection relation assureur tarif mesureacute)

- professionnels de lrsquoautomobile et autres (agreacutement marge de manœuvre reacuteparations tarifs

eacuteleveacutes)

- fournisseurs (CA paiement reacutegulier)

- eacutetat (taxes engagement pour la seacutecuriteacute)

- employeacutes (salaire conditions de travail seacutecuriteacute de lrsquoemploi)

Initiation au management copy CRCF ndash J Sornet Page 22 48

MP3 En les situant dans le cycle des activiteacutes du management trouver les actions agrave mener

dans les situations suivantes

- baisse de 10 des ventes dans une entreprise industrielle (produits meacutenagers le reacuteseau de

distribution vient drsquoecirctre reacuteorganiseacute)

Adapter le pilotage motiver cadrer si insuffisant retoucher une organisation deacutefectueuse

- idem dans une entreprise de vente par correspondance soumise agrave la concurrence internet

(les ventes stagnaient depuis six mois malgreacute les efforts promotionnels)

Voir pilotage et organisation si une eacutevolution du meacutetier a deacutejagrave eacuteteacute initialiseacutee Sinon re-

conception (adaptation au nouveau contexte) puis planification et reacuteorganisation

- augmentation des deacutelais drsquoattente des consultations dans une clinique (lrsquohocircpital voisin a

fermeacute son service drsquourgences)

Organisation Si insuffisant planification (nouveaux objectifs)

Initiation au management copy CRCF ndash J Sornet Page 23 48

ORGANISATION ET PROCESSUS

La performance de lrsquoentreprise deacutepend de son organisation et de son aptitude agrave produire

aux meilleures conditions Nous allons montrer comment organisation formelle et processus

de production peuvent contribuer agrave cette performance

1 ndash Vers lrsquooptimum

11 ndash Les eacuteconomies occidentales jusqursquoaux anneacutees 70

Jusqursquoen 1945 le principal problegraveme des entreprises eacutetait de produire des biens en quantiteacute

suffisante agrave un prix compatible avec le marcheacute Les grandes entreprises se sont multiplieacutees et

la standardisation a permis de reacuteduire les coucircts (exemple deacuteveloppement de Ford et de la

production agrave la chaicircne de 1908 agrave 1920 qui a permis une baisse du prix des voitures des 23)

On parle de laquo production pousseacutee vers le marcheacute raquo

Cette croissance de la production peu reacuteguleacutee a eacuteteacute marqueacutee par des surproductions en

1910 et 1920 puis par la crise de 1929 qui a prolongeacute ses effets jusqursquoagrave la guerre

De 1945 agrave 1975 environ (les laquo trente glorieuses raquo) la reconstruction la croissance de la

consommation de masse de nouvelles technologies et les eacutechanges internationaux

alimentent lrsquoeacuteconomie La standardisation srsquoeacutetend aux biens de consommation dont les

coucircts baissent fortement et de nouvelles reacutegulations sociales permettent une eacutevolution sans

heurt des revenus La saturation de certains marcheacutes conduit dans les anneacutees 60 agrave la

deacutemarche laquo marketing raquo et agrave la diffeacuterenciation des produits Le produit est laquo dirigeacute par le

marcheacute raquo mais les entreprises conservent une organisation assez classique et les plus grosses

srsquointernationalisent

12 ndash Lrsquoexpeacuterience japonaise et ses prolongements

Tregraves tocirct apregraves la guerre dans un Japon appauvri le constructeur automobile Toyota a ducirc

faire face agrave une restriction du marcheacute des moyens financiers et productifs et des

approvisionnements La firme a donc innoveacute dans un nouveau systegraveme de production

chassant les laquo gaspillages raquo (temps drsquoattente transports stocks deacutefauts hellip) consideacuterant que

seule la fabrication vendable creacutee de la valeur

Toyota srsquoorganise pour fabriquer la quantiteacute et la qualiteacute de produits juste neacutecessaires agrave la

satisfaction des clients la production est laquo tireacutee par le marcheacute raquo La mise en place de ce

systegraveme qui integravegre les fournisseurs ne sera acheveacutee que dans le milieu des anneacutees 70

En 1973 la hausse du peacutetrole inaugure un ralentissement de la croissance des eacuteconomies

occidentales La concurrence accrue provoque alors un inteacuterecirct pour le systegraveme deacuteveloppeacute

au Japon La production au plus juste se deacuteveloppe ainsi dans lrsquoindustrie automobile agrave partir

des anneacutees 80 et elle se reacutepand encore maintenant dans drsquoautres secteurs

Cette approche qui vise un objectif de zeacutero stock et zeacutero deacutefaut impose la maicirctrise de laquo bout

en bout raquo des processus de production et leur ameacutelioration

Initiation au management copy CRCF ndash J Sornet Page 24 48

2 ndash Organiser lrsquoentreprise

21 ndash Direction et organisation

Diriger une entreprise neacutecessite de lrsquoorganiser (de reacutepartir les tacircches) pour qursquoelle puisse

atteindre ses objectifs Lrsquoorganisation permet de satisfaire un marcheacute en tirant parti des

capaciteacutes actuelles de lrsquoentreprise tout en preacuteparant lrsquoavenir

Lrsquoorganisation reacutesulte freacutequemment drsquoun compromis entre des objectifs situeacutes agrave des niveaux

et des eacutecheacuteances diffeacuterents

Exemples

- le leader des chaises roulantes peut tirer profit de sa structure productive et de son

savoir faire pour entrer sur le marcheacute de la bicyclette eacutelectrique

- ecirctre parfaitement structureacute pour alimenter 90 du marcheacute des disquettes ne preacutepare

pas lrsquoavenir

- srsquoorganiser pour conqueacuterir le marcheacute des tire-bouchons eacutelectriques dans les deux ans

perd de son sens si cela altegravere les moyens neacutecessaires agrave la production drsquoappareils

manuels ancienne mais vitale dont la diminution agrave court terme risque de nuire agrave la

solvabiliteacute de lrsquoentreprise et de la conduire agrave la cessation de paiement

22 ndash Lrsquoorganisation fonctionnelle

La majoriteacute des entreprises adopte une laquo organisation fonctionnelle raquo (celle qui est visible

dans les organigrammes) ougrave des regroupements de personnels et drsquoeacutequipements se font

selon un modegravele hieacuterarchique (laquo line raquo) dans des uniteacutes des services ou des deacutepartements

speacutecialiseacutes Cette organisation peut se deacutecliner agrave lrsquointeacuterieur des divisions des grandes

entreprises quand elles scindent leur activiteacute par zone geacuteographique type drsquoactiviteacute

cateacutegorie de clients hellip

Remarque le terme laquo fonction raquo deacutesigne un rocircle particulier dans le fonctionnement de

lrsquoentreprise

Lrsquoorganisation fonctionnelle diffeacuterencie les activiteacutes de lrsquoentreprise en les regroupant par

meacutetier pour utiliser au mieux les compeacutetences et les moyens (meilleur rendement par la

speacutecialisation lrsquoeacutechange de compeacutetences dans une mecircme uniteacute ou gracircce agrave des eacuteconomies

drsquoeacutechelle)

23 ndash La notion de processus de production

Un processus de production se deacutefinit par la succession drsquoactiviteacutes permettant de satisfaire

un client en transformant des ressources (mateacuterielles financiegraveres humaines) en un produit

bien ou service Le processus doit creacuteer une valeur reconnue par le client

Un processus peut servir un client interne agrave lrsquoentreprise (par exemple en produisant un

composant intervenant dans plusieurs produits ou par la maintenance des machines) aussi

bien qursquoun client final On distingue usuellement

- les processus opeacuterationnels (ou maicirctres) aussi appeleacutes processus meacutetier (business process)

qui satisfont directement les clients finaux (conception et fabrication de produits vente hellip)

- les processus de support et de management (geacuterer les ressources humaines geacuterer

lrsquoinformation geacuterer les ressources financiegraveres hellip) qui ont les processus opeacuterationnels comme

clients

Toutes les actions internes agrave une organisation peuvent srsquointeacutegrer dans des processus qui

conditionnent directement ou indirectement la capaciteacute de lrsquoorganisation agrave satisfaire le

client final ou lrsquousager

Initiation au management copy CRCF ndash J Sornet Page 25 48

Aborder le fonctionnement de lrsquoentreprise par ses processus (approche processus) permet

de mettre en eacutevidence les chaicircnes drsquoactiviteacutes qui conduisent aux produits leurs

dysfonctionnements leurs coucircts la formation des deacutelais et la souplesse (la flexibiliteacute)

disponible pour satisfaire la clientegravele finale Lrsquoameacutelioration des processus a un impact visible

et direct sur chaque produit proposeacute aux clients

Lrsquoapproche processus provoque une eacutevolution de la faccedilon de travailler

- en faisant peacuteneacutetrer la laquo voix du client raquo au plus profond de lrsquoentreprise (et plus seulement

dans les services commerciaux et marketing)

- en mettant en eacutevidence des possibiliteacutes de rationalisation (par regroupement ou impartition

de certaines activiteacutes)

Remarque lrsquoapproche par les activiteacutes et les processus est agrave lrsquoorigine de la meacutethode

de deacutetermination des coucircts laquo ABC raquo - activity based costing

24 ndash Processus et fonctions

Le processus est transversal Il enchaicircne des activiteacutes qui traversent lrsquoentreprise en particulier

les services ou les deacutepartements drsquoune organisation fonctionnelle

Exemple

La division du travail par fonctions induit une charge de coordination pour assurer le

deacuteroulement du processus Elle peut geacuteneacuterer des attentes des erreurs ou des conflits drsquointeacuterecirct

(lrsquoobservation montre que des dysfonctionnements sont tregraves souvent constateacutes lors du

passage drsquoun service agrave un autre)

Organisation fonctionnelle et approche processus visent toutes deux un optimum

eacuteconomique mais leurs logiques sont diffeacuterentes

- le processus vise la satisfaction des clients (prix qualiteacute deacutelais service)

- le deacutecoupage fonctionnel cherche agrave optimiser les moyens (maximiser lrsquoeffet drsquoexpeacuterience

partager des infrastructures profiter de pocircles de compeacutetences hellip) Il apporte une ossature

hieacuterarchique stable souvent indispensable

Organisation fonctionnelle et approche processus sont donc compleacutementaires dans la

majoriteacute des cas et doivent ecirctre combineacutees judicieusement

APPLICATIONS OP

OP1 Deacutefinir flexibiliteacute systegraveme impartition

OP2 Citer huit exemples drsquoinformations essentielles pour optimiser un processus de

fabrication

Direction

Deacutepartement

commercial

(C)

Deacutepartement

administratif et

financier (AF)

Deacutepartement

Etudes (E)

Deacutepartement

Production (P)

Activiteacute

C-x Activiteacute

AF-x Activiteacute

E-x

Activiteacute

P-x

Processus x

Clie

nt

Initiation au management copy CRCF ndash J Sornet Page 26 48

OP3 Deacutegager les principes du toyotisme preacutesenteacute ci-dessous En quoi ce systegraveme est-il

initiateur de lrsquoapproche processus

Taiichi Ohno et le Toyotisme

1 - Extrait drsquoun article de Jacques BARRAUX - 1993 - LExpansion

Taiichi Ohno (1912 ndash 1990) hellip ne se prenait pas pour un visionnaire mais en imposant une

nouvelle faccedilon de produire il a reacuteinventeacute le management hellip tout le monde a entendu parler

des mots qui ont populariseacute le toyotisme dont il est le pegravere le juste-agrave-temps hellip Autant

doutils conccedilus pour lrsquoautomobile et qui ont aujourdhui une application universelle

hellip Taiichi Ohno jeune ingeacutenieur entre chez Toyota alors simple constructeur de machines

textiles Degraves 1926 apparaicirct la notion de jidoka hellip cest lart de transfeacuterer de lintelligence aux

machines pour mieux libeacuterer lintelligence des hommes Tout le contraire du taylorisme qui

juge la machine moins impreacutevisible que lhomme En 1933 Toyota se lance dans lautomobile

en sinspirant des meacutethodes ameacutericaines Mais en 1935 agrave loccasion dun voyage aux Etats-

Unis leacutetat-major de lentreprise revient fascineacute de sa visite dans un supermarcheacute La notion

de juste-agrave-temps va naicirctre de lobservation dune grande surface un lieu ougrave les clients ne

prennent que ce dont ils ont besoin et ougrave les rayons sont reacuteapprovisionneacutes pour compenser

les quantiteacutes preacuteleveacutees Ainsi le systegraveme Toyota est-il deacutejagrave dans la tecircte de ses dirigeants avant

mecircme la Seconde Guerre mondiale un demi-siegravecle avant la reacutevolution informatique et la

segmentation intensive des marcheacutes

hellip des esprits curieux comme Franccedilois Dalle en France tombent alors sous le charme des

formules et des paraboles de Taiichi Ohno En voici deux eacutechantillons

Penser agrave lenvers Cela signifie combattre les ideacutees reccedilues En lespegravece il sagit du fordisme et

du taylorisme Ohno ne croit pas agrave la planification aux effets deacutechelle et dexpeacuterience Il

propose un systegraveme industriel agrave lenvers qui permette de diversifier les produits et de les

fabriquer en petites quantiteacutes Nous ne devons plus ecirctre des paysans qui accumulent des

stocks mais des chasseurs On nimpose pas loffre On traque la demande et on la gegravere en

continu

Que les valleacutees soient hautes et les montagnes peu eacuteleveacutees Plutocirct que de concentrer tous

les efforts sur une production agrave un moment donneacute mieux vaut se doter de structures flexibles

permettant de passer agrave tout instant dune seacuterie agrave une autre Il faut eacuteviter les ruptures et les

secousses aplanir les cycles entretenir des flux reacuteguliers dactiviteacutes diversifieacutees Ce qui

implique de ne pas enfermer les hommes et les eacutequipements dans des speacutecialisations trop

eacutetroites

La flexibiliteacute le travail en groupe le refus de la dictature des machines la polyvalence et

surtout lattention constante aux signaux eacutemis par le marcheacute nappartiennent plus au

toyotisme Ces notions sont les fondements du nouvel art dorganiser de vendre et de

produire dans lindustrie comme dans les services hellip

2 - Quelques notions cleacutes

Taiichi Ohno a imagineacute la meacutethode des laquo cinq pourquoi raquo qui consiste agrave se poser cinq fois de

suite la question laquo pourquoi raquo sur le mecircme sujet de faccedilon agrave deacutecouvrir la veacuteritable cause

drsquoun problegraveme Cette meacutethode peut ecirctre appliqueacutee agrave tous les niveaux et permettre

notamment aux agents de fabrication de proposer de veacuteritables ameacuteliorations de la

production

La recherche de la qualiteacute totale (pas de deacutefaut des produits pas de rebuts pas de deacutefaut

des processus) accompagne la deacutemarche de Toyota La qualiteacute a un coucirct compenseacute par

des ventes accrues par lrsquoeacuteconomie des mesures palliatives aux deacutefauts

Initiation au management copy CRCF ndash J Sornet Page 27 48

Fiche OP1 ndash Benchmarking et processus

Le laquo benchmarking raquo consiste agrave comparer le fonctionnement de plusieurs systegravemes pour en

faire notamment ressortir les meilleures pratiques (laquo best practices raquo) Cette technique est

utiliseacutee depuis les anneacutees 80 pour ameacuteliorer la performance des entreprises Elle impose agrave

lrsquoentreprise drsquoeacutevaluer et de remettre en question ses propres modes de fonctionnement afin

de les faire eacutevoluer agrave la lueur de ce qui se fait ailleurs

Le benchmarking permet drsquoameacuteliorer les processus agrave moindre risque en fixant des objectifs

baseacutes sur des faits et donc plus facilement accepteacutes

Une classification des processus en tant que base de reacuteflexion a eacuteteacute eacutetablie aux USA par

lrsquolaquo International Benchmarking Clearinghouse raquo de lrsquoAPQC (american productivity and

quality center) en collaboration avec plusieurs dizaines drsquoentreprises

Elle se reacutesume ainsi

Le terme laquo reengineering raquo (la re-conception ou laquo reacuteingeacutenieacuterie raquo) des processus deacutesigne un

projet drsquoameacutelioration radicale des performances (de 20 agrave 50 ou plus) Il neacutecessite une

parfaite adheacutesion de la direction la constitution drsquoune petite eacutequipe de projet brillante

connaissant parfaitement les activiteacutes de lrsquoentreprise et il peut inclure un benchmarking

Le reengineering provoque geacuteneacuteralement la reacuteduction du nombre de niveaux hieacuterarchiques

(laquo delayering raquo) et lrsquoaccroissement du pouvoir de deacutecision des employeacutes (laquo empowerment raquo

ou laquo empouvoirement raquo) Bien qursquoy conduisant parfois il ne doit pas ecirctre confondu avec la

reacuteduction des activiteacutes (laquo downsizing raquo ou restructuration) et lrsquoexternalisation (laquo outsourcing raquo)

Pro

ce

ssu

s

op

eacutera

tio

nn

els

Pro

ce

ssu

s d

e m

an

ag

em

en

t e

t d

e

sup

po

rt

1 ndash

Comprendre

le marcheacute et

les clients (besoins

satisfaction)

2 ndash

Deacutevelopper

vision et

strateacutegie (contexte

concurrence)

3 ndash

Creacuteer

produits

services

processus

(concevoir

ameacuteliorer)

4 ndash

Marketing et

vente

5 ndash

Produire et

livrer (industrie

dont

ameacutelioration

processus)

6 ndash

Produire et

livrer (services)

7 ndash

Facturer et

servir les

clients (apregraves-

vente

reacuteclamations)

8 ndash Deacutevelopper et geacuterer les ressources humaines

9 ndash Geacuterer les systegravemes drsquoinformation

10 ndash Geacuterer les ressources financiegraveres et les actifs

11 ndash Appliquer un programme environnemental

12 ndash Geacuterer les relations exteacuterieures (actionnaires banques lois relations publiques hellip)

13 ndash Geacuterer lrsquoameacutelioration et le changement (eacutevaluer mesurer motiver qualiteacute totale)

Initiation au management copy CRCF ndash J Sornet Page 28 48

Fiche OP2 ndash Lrsquoorganisation par processus

Lrsquoeacutevolution drsquoune organisation aux activiteacutes reacutepeacutetitives vers lrsquoapproche processus est

geacuteneacuteralement progressive et se met en place par paliers

La mise en œuvre drsquoun veacuteritable management par processus doit ecirctre preacuteceacutedeacutee quand

lrsquoactiviteacute de lrsquoentreprise est complexe drsquoun recensement (une laquo cartographie des

processus raquo) pour mettre en eacutevidence les processus ou les familles de processus cleacutes critiques

pour le succegraves de lrsquoentreprise ougrave les efforts seront prioritaires

Des responsables de processus (laquo process owners raquo) sont ensuite deacutesigneacutes

Le responsable doit concevoir ses processus puis apregraves leur mise en œuvre assurer les

coordinations neacutecessaires les ameacuteliorer et les repreacutesenter aupregraves de la direction

Quand une structure par processus est mise en place des opeacuterateurs exeacutecutants

preacuteceacutedemment regroupeacutes dans les fonctions peuvent ecirctre affecteacutes aux processus et

drsquoanciens responsables de fonctions peuvent devenir des experts au service des processus

Lrsquoorganisation par processus peut imposer un degreacute eacuteleveacute drsquointeacutegration des activiteacutes donc

une polyvalence accrue des personnels et une reacuteduction des niveaux hieacuterarchiques

Elle neacutecessite pour le moins des compeacutetences eacutelargies au niveau des responsables de

processus (organisation administration technique hellip) dont le nombre doit rester limiteacute

(quelques dizaines au plus)

Sauf dans de tregraves petites structures lrsquoorganisation par processus se plaque geacuteneacuteralement sur

une structure plus classique

Initiation au management copy CRCF ndash J Sornet Page 29 48

ELEMENTS DE CORRIGE OP

OP1 Deacutefinir

Flexibiliteacute = adaptation au besoin (horaire variable chaicircnes robotiseacutees)

Systegraveme = ensemble organiseacute dans un but boicircte noire (sanguin nerveux meacutetrique laquo D raquo)

Impartition = sous-traitance ou externalisation (seacuteparation) drsquoactiviteacutes faire appel agrave des

partenaires plutocirct que faire soi-mecircme

OP2 Citer huit exemples drsquoinformations essentielles pour orienter lrsquooptimisation drsquoun processus

Montant des stocks (approvisionnements et produits finis)

Temps drsquoattente

Taux drsquoactiviteacute des ateliers

Rebuts

Deacutelai de production

Taux de reacuteclamations clients (qualiteacute)

Temps passeacutes en retouches finales

Turn over

Nombre drsquoarrecircts maladie

Accidents du travail

Dureacutee des arrecircts machines

OP3 Deacutegager les principes du toyotisme preacutesenteacute dans la fiche 31 En quoi ce systegraveme

repose trsquoil sur lrsquoapproche processus

Produire la quantiteacute juste neacutecessaire (agrave la demande) donc eacuteviter les stocks

Flexibiliteacute intelligence des chaicircnes de production

Qualiteacute (eacuteviter le coucirct de la non-qualiteacute)

La notion de processus est implicite ainsi que la chaicircne de valeur client

Initiation au management copy CRCF ndash J Sornet Page 30 48

DEFIS ET TENDANCES DU MANAGEMENT

Les meacutethodes de management se deacuteveloppent pour affronter le contexte eacuteconomique

Ce chapitre preacutesente les deacutefis auxquels le management contemporain doit faire face

1 ndash Lrsquoeacutevolution eacuteconomique contemporaine

A mesure que lrsquoactiviteacute eacuteconomique mondiale srsquoaccroicirct que la technologie eacutevolue les

changements sont de plus en plus rapides Ils introduisent des situations ineacutedites auxquelles les

entreprises doivent srsquoadapter en cherchant de nouvelles solutions de management Les trois

derniegraveres deacutecennies ont eacuteteacute notamment marqueacutees par les pheacutenomegravenes suivants (que nous

listons sans tenir compte des liens pouvant exister entre eux)

Pheacutenomegravene Traduction Effets

Deacute reacuteglementation

globalisation

financiegravere

titrisation

Libre circulation des capitaux accegraves

facile des particuliers au marcheacute

boursier (directement ou par

lrsquointermeacutediaire des OPCVM et SICAV)

Monteacutee en puissance du financement

des entreprises sur le marcheacute boursier

Fonds de pension

(retraites) et fonds

souverains (eacutetats)

Poids boursier important drsquoinvestisseurs

institutionnels qui cherchent un haut

rendement financier (dividendes ou

valorisation boursiegravere)

Pression sur les grandes entreprises

influence sur les strateacutegies

Mondialisation Liberteacute des eacutechanges internationaux Accroissement de la concurrence

recherche drsquoavantages eacuteconomiques

par la deacutelocalisation (biens et

services) la concentration des efforts

(recentrage) problegravemes drsquoemploi

multiplication des transports perte

drsquoinfluence des politiques

Baisse de lrsquoemploi

occidental

(notamment

industriel)

Moins de fabrications fabrications

automatiseacutees recours aux moyens

informatiques

Activiteacute reporteacutee sur le commerce la

conception et les services chocircmage

charge sociale

Restructurations Optimisation des entreprises

abaissement des coucircts augmentation

des marges recherche drsquoune taille

critique (eacuteconomies drsquoeacutechelle poids

sur le marcheacute)

Recentrages externalisations fusions

deacutelocalisations constitution de grands

groupes

NTIC (nouvelles

technologies de

lrsquoinformation et de

la communication)

Mise en œuvre des reacuteseaux (dont

internet) et drsquoapplications

informatiques communicantes

Nouvelles formes de commerce

marcheacute international deacutelocalisation

du travail intellectuel reacuteorganisation

de la distribution

Rareacutefaction relative

des matiegraveres

premiegraveres

Recherche de substituts exploration

miniegravere coucircts drsquoexploitation des

gisements accrus

Augmentation des coucircts variations

erratiques du cours des matiegraveres

deacutestabilisations politiques

Evolution

geacuteopolitique et

eacuteconomique

mondiale

Chute de lrsquoURSS transformation des

eacuteconomies collectivistes pays

eacutemergents (Chine Inde Breacutesil Russie)

()

Accroissement de la population

mondiale (4 agrave 6 7 milliards de 1970 agrave

2008)

Libeacuteralisme sans frein () nouvelles

puissances eacuteconomiques

opportuniteacutes de deacuteveloppement

nouveau partage des ressources

ineacutegaliteacutes baisse du soutien aux PVD

laquo Terrorisme raquo Actions armeacutees pression de groupes

armeacutes non gouvernementaux

Deacutestabilisations reacutegionales charge

des deacutepenses militaires

Deacuteveloppement

durable

Recherche drsquoune croissance eacutequitable

et respectueuse de lrsquoenvironnement

Pression sur les entreprises (eacutetats

associations de consommateurs

eacutecologistes ONG)

() Reacutecemment quelques affaires (Enron laquo subprimes raquo Vivendi Universal Socieacuteteacute

Geacuteneacuterale Airbus par exemple) et agrave plus grande eacutechelle la crise financiegravere de 2008 ont

montreacute les dangers drsquoune libeacuteralisation sans controcircles suffisants

Initiation au management copy CRCF ndash J Sornet Page 31 48

() Des alliances eacuteconomiques naissent entre pays eacutemergents (notamment en

ameacuterique centrale creacuteation de la Banque du Sud en 2008 par exemple) et lrsquoon

commence agrave imaginer une baisse progressive de lrsquoinfluence eacuteconomique des Etats

Unis

2 ndash Les deacutefis actuels du management

21 ndash Les grandes orientations

Lrsquoeacutevolution eacuteconomique suggegravere quelques pistes parfois concurrentes pour lrsquoaction du

manager contemporain On y retrouve au premier plan la construction drsquoune vision qui est

une composante commune du leadership

Objectif du manager

pour lrsquoentreprise

Justification Facteurs de reacuteussite

Construire une vision Eclairer lrsquoavenir de lrsquoentreprise partager

un but souder motiver

Effort de reacuteelle prospection

volontarisme de la direction

bonne communication

Reacuteactiviteacute et flexibiliteacute

(sous tous les aspects

agrave tous niveaux)

Srsquoadapter rapidement au marcheacute Bonne organisation des processus

personnel compeacutetent autonome

et motiveacute structure hieacuterarchique

alleacutegeacutee robotisation

Deacutegager des profits Reacutemuneacuterer les apporteurs de capitaux

srsquoautofinancer

Ajuster coucircts et structures

Exploiter les nouvelles

technologies

Reacuteactiviteacute ajuster coucircts et deacutelais

reacutepondre au marcheacute suivre les clients

Organiser le SI de faccedilon

pertinente eacuteviter le coucirct excessif

drsquoinvestissements trop en

laquo pointe raquo (laquo essuyer les placirctres raquo)

utiliser judicieusement les services

exteacuterieurs

Bacirctir des alliances

(contrats fusions)

Deacutevelopper une activiteacute limiter les coucircts

de transaction () atteindre la taille

critique et de meilleurs rendements se

recentrer sur une activiteacute profitable

Dominer les processus se donner

une identiteacute lisible externaliser se

doter drsquoune capaciteacute financiegravere

suffisante

Valoriser lrsquoimage Attirer les clients favoriser les alliances

donner confiance (apporteurs de fonds

employeacutes clients partenaires socieacuteteacute

civile)

Instaurer des regravegles de

gouvernance inteacutegrer le

deacuteveloppement durable

respecter lrsquoenvironnement

Geacuterer les risques Faire face aux aleacuteas eacuteconomiques et

technologiques (conjoncture politiques

accidents malveillance)

Creacuteer un systegraveme drsquoalerte geacuterer

la crise (reacuteaction raisonneacutee

sceacutenarios poursuite de

lrsquoexploitation dans un contexte

instable) mise en place de

proceacutedures drsquoapprentissage pour

ameacuteliorer les reacuteactions au fil du

temps

Geacuterer le changement Faire face agrave lrsquoeacutevolution de la demande

la pression sur les prix la variation des

performances financiegraveres la

concurrence la globalisation des

marcheacutes lrsquoeacutevolution technologique aux

fusions ou alliances aux changements

de reacuteglementation de direction hellip ()

Bonne communication pour

donner du sens au changement

et obtenir lrsquoadheacutesion du personnel

Rassembler et geacuterer les

connaissances former le

personnel

Innover Garder un avantage concurrentiel se

diffeacuterencier

Veille technologique et

commerciale investissement

Ouverture

internationale

Elargir le marcheacute saisir les opportuniteacutes Veille commerciale partenariats

() La theacuteorie des coucircts de transaction deacuteveloppeacutee par OE Williamson dans les

anneacutees 70 integravegre les coucircts lieacutes au recours au marcheacute (recherche et choix drsquoun

fournisseur neacutegociation reacutedaction de contrat suivi des eacutechanges risque de rupture

Initiation au management copy CRCF ndash J Sornet Page 32 48

drsquoapprovisionnement hellip) On peut en conclure que lrsquointeacutegration de diffeacuterentes

activiteacutes agrave lrsquoentreprise (la laquo firme raquo) preacutesente des avantages Mais des coucircts de

transaction internes doivent aussi ecirctre consideacutereacutes (preacuteparation organisation

surveillance hellip) et certaines formes de coopeacuteration continue avec les fournisseurs

permettent de reacuteduire le coucirct des transactions externes

() drsquoapregraves laquo Les meilleures pratiques de management raquo - Brilman Heacuterard ndash EO

Une eacutetude du Conference Board (2002) liste les deacutefis du management vus par 700 leaders

mondiaux Soit en reacutesumeacute avec indication du score correspondant

1 ndash Fideacuteliser les clients (42)

2 ndash Reacuteduire les coucircts (38)

3 ndash Accroicirctre flexibiliteacute et reacuteactiviteacute (29)

4 ndash Amener les employeacutes agrave adheacuterer aux valeurs et visions de lrsquoentreprise (26)

5 ndash Deacutevelopper et retenir les leaders (25)

6 ndash Geacuterer acquisitions et alliances (24)

7 ndash Accroicirctre lrsquoinnovation (20)

En fin de classement citoyenneteacute et reacuteputation (4) et ameacutelioration de la diversiteacute (3)

22 ndash Les techniques disponibles

Pour faire face aux deacutefis le manager dispose de nouveaux concepts et de nouvelles

techniques Le tableau ci-dessous en donne un reacutesumeacute et indique les domaines qursquoils

influencent principalement

Initiation au management copy CRCF ndash J Sornet Page 33 48

Principaux concepts techniques outils Incidence principale sur

Internet

- e-commerce (commerce eacutelectronique site

entreprise)

- CRM ou GRC (gestion de la relation client)

- e-procurement (gestion des approvisionnements

par le reacuteseau)

- messagerie eacutelectronique

- e-recrutement

Vente accegraves au marcheacute

Relation client reacuteactiviteacute personnalisation

fideacutelisation

Deacutelais coucircts

Communication transfert de donneacutees (piegraveces

jointes) tous domaines

Communication recrutement

Intranet reacuteseau drsquoentreprise SI

- knowledge management (gestion des

connaissances)

- e-learning (apprentissage en ligne)

- plateforme de travail collaboratif (groupware)

- workflow (circulation eacutelectronique de

documents enchaicircnement de processus)

- e-RH portail RH (libre accegraves aux postes agrave

pourvoir informations candidatures hellip)

- PGI (progiciel de gestion inteacutegreacute) ou ERP

Innovation capaciteacute au changement veille

documentaire

Formation du personnel accompagnement des

changements

Coordination communication interne

Coordination

Communication interne (voire internet en

externe) reacuteduction des coucircts climat drsquoentreprise

recrutement plans de carriegraveres hellip

Coucircts fiabiliteacute du systegraveme drsquoinformation deacutelais

processus (continuiteacute inteacutegration)

Logistique inteacutegreacutee

Supply Chain Management (SCM) gestion de la

logistique (incluant les approvisionnements)

Processus deacutelais coucircts

Externalisation

Valorisation du capital humain

GPEC (gestion preacutevisionnelle des emplois et

compeacutetences)

Coaching

Reacuteactiviteacute de lrsquoentreprise conservation des

compeacutetences rendements individuels turn-over

adaptation des compeacutetences motivation

Efficaciteacute individuelle controcircle reacutegulation

progregraves processus

Approche processus

Optimisation des processus

Deacutemarche qualiteacute totale (TQM ndash total quality

management)

Empowerment (empouvoirement)

Benchmarking reacuteingeacutenieacuterie

Coucircts marges qualiteacute deacutelais flexibiliteacute

externalisation eacutelargissement des compeacutetences

organisation

Ameacutelioration des processus (meacutetiers et supports)

Autonomie compeacutetences des employeacutes

Ameacutelioration des processus restructuration

Management par la valeur

Parties prenantes

Satisfaction des parties prenantes financement

motivation collaborations hellip

Collaboration inter organisations

Reacuteseaux drsquoentreprises alliances

EDI (eacutechange de donneacutees informatiseacutees) extranet

Impartition externalisation (outsourcing)

Coucircts recentrage investissements lancement

drsquoactiviteacute

Coucircts reacuteactiviteacute deacutelais relations avec

lrsquoadministration

Coucircts recentrage limitation des investissements

Ethique drsquoentreprise

Gouvernance drsquoentreprise (mode de direction

encadreacute par des regravegles)

Rocircle socieacutetal deacuteveloppement durable

environnement

Image de lrsquoentreprise reacutegulation du top

management relations actionnaires

Image peacutenaliteacutes et amendes objectifs

strateacutegiques

Initiation au management copy CRCF ndash J Sornet Page 34 48

23 ndash Le rocircle socieacutetal des entreprises

La responsabiliteacute socieacutetale de lrsquoentreprise (RSE) deacutesigne le rocircle qursquoelle prend dans la socieacuteteacute

au-delagrave de son activiteacute purement geacuteneacuteratrice de profit On parle aussi drsquoentreprise citoyenne

La RSE est indissociable du deacuteveloppement durable de porteacutee mondiale et dont les trois

piliers sont

- eacuteconomique (favoriser le deacuteveloppement les eacutechanges internationaux)

- social (accegraves aux soins eacuteducation conditions de travail hellip)

- environnemental (pollution preacuteservation des ressources hellip)

La RSE integravegre notamment une preacuteoccupation sociale de lrsquoentreprise vis-agrave-vis de ses salarieacutes

(seacutecuriteacute et santeacute au travail juste reacutemuneacuteration deacuteveloppement personnel hellip) Elle conduit agrave

tenir compte dans le management drsquoune vision exteacuterieure agrave lrsquoentreprise qui peut avoir des

reacutepercussions possibles sur son activiteacute eacuteconomique

Lrsquoentreprise peut aussi tirer avantage drsquoune deacutemarche responsable par la baisse de certains

coucircts (plus faibles consommations drsquoeacutenergies reacuteduction des transports hellip)

Le rocircle socieacutetal de lrsquoentreprise a eacuteteacute reconnu en France par la loi laquo NRE raquo de 2001 (loi sur les

nouvelles reacutegulations eacuteconomiques) qui oblige les socieacuteteacutes franccedilaise coteacutees sur un marcheacute

reacuteglementeacute agrave rendre compte dans leur rapport annuel de leur gestion sociale et

environnementale au travers de leur activiteacute

Article 116 de la loi Le rapport viseacute agrave larticle L 225-102 rend compte hellip laquo Il comprend

eacutegalement des informations dont la liste est fixeacutee par deacutecret en Conseil dEtat sur la

maniegravere dont la socieacuteteacute prend en compte les conseacutequences sociales et

environnementales de son activiteacute Le preacutesent alineacutea ne sapplique pas aux socieacuteteacutes

dont les titres ne sont pas admis aux neacutegociations sur un marcheacute reacuteglementeacute raquo

Une norme ISO 14000 integravegre ces preacuteoccupations et des taxes eacutecologiques sont

progressivement creacutees

3 ndash Le management par la valeur

31 ndash De lrsquoanalyse au management par la valeur

Lrsquoanalyse de la valeur est neacutee en 1947 aux Etats-Unis (General Electrics) Cette technique

consiste agrave eacutelaborer des produits conformes aux attentes de la clientegravele mais sans excegraves pour

trouver un bon compromis entre valeur pour le client et coucirct Le produit optimal est deacutefini agrave

partir drsquoenquecirctes qui deacuteterminent le besoin client (ou plutocirct drsquoun client laquo type raquo)

Exemple il est inutile de concevoir un petit veacutehicule citadin capable de parcourir

500 000 km sans avarie compte tenu des effets de mode et du faible kilomeacutetrage

annuel Par contre le marcheacute peut exiger un fonctionnement sans faille sur 150 000 km

soit dix ans en moyenne ce qui conditionne les coucircts de production

Cette recherche drsquoun ajustement de valeur au besoin des clients eacutetait un preacutecurseur du

management par la valeur qui recherche plus largement la creacuteation de valeur pour

chacune des parties prenantes de lrsquoentreprise tout en lui meacutenageant un reacutesultat suffisant

Plus geacuteneacuteralement le management par la valeur est deacutefini par une norme europeacuteenne (EN

12973)

Le management par la valeur est un style de management particuliegraverement destineacute agrave

mobiliser les individus agrave deacutevelopper les compeacutetences et agrave promouvoir les synergies et

Initiation au management copy CRCF ndash J Sornet Page 35 48

linnovation avec pour objectif la maximisation de la performance globale dun

organisme Le management par la valeur apporte une nouvelle faccedilon dutiliser nombre

de meacutethodes de management existantes Il est en coheacuterence avec le Management

de la qualiteacute

Cette approche du management pose de nombreuses questions notamment quelles

prioriteacutes et quelles valeurs attribuer aux parties prenantes comment appreacutehender la

perception par les parties prenantes de la valeur qui leur est affecteacutee

32 ndash La valeur client

Le processus drsquoeacutelaboration drsquoun produit qui consomme des ressources coucircteuses doit creacuteer

une valeur suffisante pour provoquer lrsquoachat par le client final La production drsquoune valeur

reconnue par le client est vitale pour lrsquoentreprise mais sa deacutetermination est parfois complexe

La valeur du produit perccedilue par le client integravegre des eacuteleacutements en partie subjectifs

- une valeur drsquousage (le produit reacutepond agrave un besoin)

- une valeur drsquoestime (lrsquoimage apporteacutee par le produit un aspect affectif)

- une valeur drsquoeacutechange (deacuteduite de lrsquoespoir de revente du produit)

Valeurs drsquousage drsquoestime et drsquoeacutechange deacutependent implicitement de la qualiteacute (un bien peu

fiable est impropre agrave lrsquousage attendu de mauvaise qualiteacute notoire il nrsquoapporte pas une

image positive et ses deacutefauts connus nuisent agrave sa revente) Une eacutevaluation de la qualiteacute

intervient donc dans la valeur perccedilue du produit

Par ailleurs le client considegravere le coucirct drsquoobtention du produit (les charges qursquoil doit supporter

pour acqueacuterir le produit lrsquoeffort qursquoil doit faire pour trouver le produit et les frais de mise agrave

disposition)

Le prix perccedilu par le client est geacuteneacuteralement supeacuterieur au prix de vente

Le client achegravete theacuteoriquement le produit qui preacutesente la diffeacuterence valeur perccedilue ndash prix

perccedilu la plus favorable ou le meilleur rapport prix perccedilu qualiteacute perccedilue et dans certains

cas celui qui a le prix produit le plus bas

Remarque les valeurs du scheacutema ci-dessus changent durant le cycle de vie du produit

(un nouveau produit peut avoir une valeur perccedilue plus eacuteleveacutee qursquoen fin de vie) La

valeur client ne peut ecirctre eacutevalueacutee que par enquecirctes et ne peut donc ecirctre deacutefinie avec

certitude

La notion de laquo satisfaction client raquo conseacutecutive agrave une vente influence aussi le prix produit et

le prix perccedilu

- lrsquoentreprise gagne sur les coucircts de recherche de clientegravele

- le client nrsquoa pas agrave rechercher un nouveau fournisseur et beacuteneacuteficie drsquoun coucirct drsquoobtention

plus bas

valeur perccedilue client

prix perccedilu client

coucirct produit Marge (valeur creacuteeacutee pour

lrsquoentreprise)

euros

prix produit

Valeur creacuteeacutee

pour le client

Initiation au management copy CRCF ndash J Sornet Page 36 48

La satisfaction du client deacutepend de facteurs qualitatifs aussi divers que la fiabiliteacute du produit

la vitesse de reacuteaction du fournisseur lrsquoattitude des commerciaux lrsquoefficaciteacute du service

apregraves-vente la netteteacute des contrats ou la justesse de la facture

Valeur perccedilue coucirct marge et satisfaction reacutesultent de processus allant de la conception du

produit jusqursquoagrave sa livraison et son apregraves-vente La deacutemarche laquo processus raquo et lrsquolaquo analyse de la

valeur raquo en forccedilant la recherche de solutions efficientes agrave tout niveau administratif

technique commercial et apregraves-vente sont donc neacutecessaires pour bien positionner

lrsquoentreprise sur son marcheacute

Pour autant le risque commercial ne peut jamais ecirctre annuleacute et lrsquooffre de lrsquoentreprise ne

satisfait geacuteneacuteralement pas en milieu concurrentiel tous ses clients potentiels

33 - La creacuteation de valeur pour les autres parties prenantes

Les salarieacutes

La creacuteation drsquoune valeur suffisante pour les salarieacutes est reconnue comme neacutecessaire car des

observations montrent que la satisfaction des clients en deacutepend Moins souvent eacutevoqueacutee en

peacuteriode de chocircmage elle nrsquoest prioritaire que pour les employeacutes dont lrsquoentreprise souhaite

conserver les compeacutetences

La laquo valeur salarieacute raquo ne comprend pas que le salaire Le sentiment drsquoappartenance agrave un

groupe la reconnaissance lrsquoaccomplissement de soi et la construction professionnelle en

sont des eacuteleacutements importants Comme pour les clients on doit ainsi distinguer la reacutetribution

perccedilue du salaire objectif

Les actionnaires

Lrsquoactionnaire apporte des fonds propres agrave lrsquoentreprise en contrepartie de titres parfois

neacutegociables en bourse et assortis drsquoun droit de vote en assembleacutee geacuteneacuterale La valeur

attribueacutee aux actionnaires est servie en termes moneacutetaires (dividende ou augmentation de la

valeur du titre neacutegociable)

Remarque des facteurs non moneacutetaires comme lrsquoimage de lrsquoentreprise qui deacutepend

en partie de sa communication peuvent influencer la deacutecision drsquoachat de vente ou

de conservation des titres par lrsquoactionnaire

Reacutetribution perccedilue euros

Salaire objectif

Avantage non

moneacutetaire de

lrsquoemploi

Initiation au management copy CRCF ndash J Sornet Page 37 48

Compte tenu de lrsquoimportance croissante de lrsquoactionnariat dans le financement des grandes

entreprises coteacutees en bourse et notamment des investisseurs institutionnels comme les fonds

de pension des indicateurs speacutecifiques ont eacuteteacute introduits pour appreacutecier la performance des

entreprises vue par les actionnaires Par exemple la valeur ajouteacutee eacuteconomique (EVA reg

economic value added marque deacuteposeacutee de Stern Stewart ou VAE ndash valeur ajouteacutee

eacuteconomique parfois deacutenommeacutee VEC ndash valeur eacuteconomique creacuteeacutee) qui prend en compte le

coucirct du capital

LrsquoEVA correspond tregraves scheacutematiquement au calcul suivant

EVA = (PO) profit opeacuterationnel ndash (C) coucirct du capital X (CE) capitaux employeacutes

LrsquoEVA neacutecessite en pratique des retraitements assez complexes Le PO peut se deacuteterminer

selon les principes suivants

- PO = reacutesultat drsquoexploitation (avant inteacuterecircts) ndash impocirct

- PO = beacuteneacutefice courant (tenant compte des inteacuterecircts) + inteacuterecircts ndash eacuteconomie drsquoimpocirct sur les

inteacuterecircts (on exclue les eacuteleacutements financiers et lrsquoimpocirct correspondant) ndash impocirct

- lrsquoimpocirct pris en compte correspond au profit opeacuterationnel consideacutereacute (dans les cas courants agrave

13 du PO)

C = taux moyen de reacutemuneacuteration du capital (reacutesultant par exemple du dividende exigeacute de

certains investisseurs et des taux drsquoemprunts bancaires)

CE = capitaux propres et dettes portant inteacuterecirct

Remarque le profit opeacuterationnel ou reacutesultat opeacuterationnel correspond au NOPAT ndash net

operating profit after tax - anglo-saxon LrsquoEVA est eacutegale au NOPAT diminueacute de la

reacutemuneacuteration des capitaux

Exemple lrsquoentreprise X dispose drsquoun capital de 2 500 000 euro et reacutealise un beacuteneacutefice net

drsquoimpocirct de 450 000 euro (taux 33 13) Un dividende de 6 doit ecirctre verseacute aux

actionnaires et la banque lui a accordeacute un precirct de 1 200 000 euro agrave 4 Les autres

constituants des reacutesultats financier et exceptionnel sont neacutegligeables

Reacutesultat opeacuterationnel = 450 000 + 004 x 1 200 000 x 23 = 482 000 euro

Coucirct du capital = 006 x 2 500 000 + 004 x 1 200 000 x 23 = 182 000 euro

EVA = 300 000 euro

Coucirct moyen pondeacutereacute du capital (C) = (004 x 1 200 000 x23 + 006 x 2 500 000)

3 700 000 Soit 492

Si lrsquoEVA est positive lrsquoentreprise creacuteeacutee de la valeur apregraves reacutemuneacuteration des capitaux et sa

valeur boursiegravere doit augmenter

Lrsquoutilisation de lrsquoEVA comme indicateur influence le management de lrsquoentreprise car il y a

trois moyens pratiques drsquoaugmenter lrsquoEVA

- augmenter le reacutesultat opeacuterationnel

- lancer des investissements ayant une rentabiliteacute supeacuterieure agrave C

- eacuteliminer les activiteacutes ayant une rentabiliteacute infeacuterieure agrave C

Remarque lrsquoutilisation sans nuance de lrsquoEVA comme critegravere de management peut

poser problegraveme Le calcul de lrsquoEVA repose sur des ajustements comptables il est donc

sujet agrave manipulations (provisions capitalisation ou non de la RD hellip) Par ailleurs le

critegravere laquo EVA raquo pris isoleacutement peut conduire agrave chercher la rentabiliteacute agrave court terme agrave

reacuteduire les investissements prospectifs et donc nuire agrave terme au deacuteveloppement de

lrsquoentreprise

Initiation au management copy CRCF ndash J Sornet Page 38 48

Les fournisseurs reccediloivent le paiement de leurs factures plus ou moins rapidement (le deacutelai

de paiement repreacutesente une valeur consentie au fournisseur)

Lrsquoentreprise peut accroicirctre la valeur apporteacutee agrave ses fournisseurs par des actions cibleacutees

comme une contribution agrave la formation de leurs personnels certains transferts de

technologie ou de savoir faire agrave des sous-traitants une coopeacuteration suivie favorisant leur

deacuteveloppement lrsquointeacutegration agrave des campagnes de promotion

A noter que la valeur consentie aux fournisseurs peut avoir une influence sur la qualiteacute et les

deacutelais de livraison des produits

La collectiviteacute reccediloit des taxes et parfois des prestations en nature par deacutefaut ou explicites

(effort de preacuteservation de lrsquoenvironnement ameacutenagement du territoire par les implantations

aide mateacuterielle agrave des projets participation agrave la formation par exemple)

APPLICATIONS DT

DT1 Deacutefinir expliquer deacutereacuteglementation socieacutetal eacuteconomies drsquoeacutechelle coaching EDI

gouvernance

DT2 Deacuteterminer en quoi la deacutemarche TQM srsquoinscrit dans les deacutefis actuels du management

DT3 Apregraves avoir consulteacute les documents ci-dessous extraits du site drsquoAir France

(httpdeveloppement-

durableairfrancecomFRfrlocaldemarcheN4_positionnement_pphtm)

exposer les enjeux et les limites de la RSE et de la gestion des parties prenantes

Initiation au management copy CRCF ndash J Sornet Page 39 48

Dialogue avec les parties prenantes

Initiation au management copy CRCF ndash J Sornet Page 40 48

Attentes des parties prenantes

Initiation au management copy CRCF ndash J Sornet Page 41 48

Creacuteation de valeur pour les parties prenantes

La creacuteation de valeur pour les parties prenantes est au cœur de la strateacutegie du Groupe Le scheacutema de

distribution financiegravere ci-dessous donne un aperccedilu de la distribution des recettes du Groupe aux

diffeacuterentes parties prenantes actionnaires collaborateurs fournisseurs pouvoirs publics

collectiviteacutes locales etc

Initiation au management copy CRCF ndash J Sornet Page 42 48

Fiche DT1 ndash Extrait du sommaire de laquo Problegravemes eacuteconomiques raquo No 2894

La gestion des entreprises bouleverseacutee par les technologies de linternet

Reacutealiteacutes industrielles - Annales des Mines Jean-Michel Yolin

Avec lavegravenement de linternet les processus de conception de production et de vente sont

radicalement remis en cause Quel que soit le secteur dactiviteacute les technologies de linternet

permettent en effet de reacuteduire les deacutelais et de passer dun processus discontinu agrave un processus

continu Lorganisation des entreprises et leur mode de gestion en sont profondeacutement bouleverseacutes

tant au niveau individuel que collectif Linternet rend ainsi possible la reacutealisation dobjectifs que les

entreprises cherchaient agrave atteindre depuis longtemps sans y parvenir meilleure eacutecoute du client

travail sans stocks en flux tendu hieacuterarchies plates autorisant une grande reacuteactiviteacute flexibiliteacute dans

lorganisation et loutil de production acceacuteleacuteration du renouvellement des produits entreprises en

reacuteseau ougrave chacune se recentre sur son cœur de meacutetier etc

Le laquo knowledge management raquo ou comment geacuterer les connaissances

Document de travail du LAMSADE - Michel Grundstein

Peter Drucker lavait preacutedit le capital immateacuteriel eacutetait voueacute agrave devenir un facteur de compeacutetitiviteacute

pour lentreprise La libeacuteralisation des eacutechanges acceacutelegravere les processus de deacutecision de lentreprise

et implique que lassimilation des informations soit agrave la fois de meilleure qualiteacute et plus rapide Ainsi

la fonction qui consiste agrave manager les connaissances au sein de lentreprise savegravere primordiale

Bien que la prise de conscience de limportance du capital immateacuteriel ait eacuteteacute tardive - le concept

de knowledge management est apparu en France aux Etats-Unis et au Japon au milieu des

anneacutees 1990 - agrave lheure actuelle lorganisation de leacutechange dinformations et le partage des

connaissances sont devenus des facteurs cleacutes dune gestion performante de lentreprise Ils

doivent sinscrire dans un projet global destineacute agrave mettre en valeur les savoirs et les savoir-faire

individuels et collectifs

Les leccedilons du laquo coaching raquo pour le management de la qualiteacute

Humanisme et Entreprise - Martine Brasseur

Parmi les nouvelles formes de management en vogue dans les entreprises le coaching figure en

bonne place Appliqueacute au management de la qualiteacute il sagit dune pratique

daccompagnement destineacutee agrave initier et agrave faciliter le processus de deacuteveloppement dun individu

La deacutemarche consiste agrave affirmer que tout individu est en quecircte de qualiteacute agrave condition toutefois

de ne pas lui imposer des contraintes lempecircchant de progresser On considegravere notamment les

erreurs comme potentiellement feacutecondes En deacutefinitive le coach donne au coacheacute la permission

de reacuteussir en lui donnant aussi la permission deacutechouer

Initiation au management copy CRCF ndash J Sornet Page 43 48

Fiche DT2 ndash Management strateacutegique les sept deacutefis agrave relever dici agrave 2016

Extrait drsquoun article du site wwwlentreprisecom -Sabine Blanc - Mis en ligne le 20032007

(httpwwwlentreprisecom325article11977html)

Une eacutetude anglaise publieacutee par lopeacuterateur Orange Grande-Bretagne deacutecrypte la mutation

des formes de travail et les enjeux majeurs pour les entreprises de demain afin decirctre au top

de la compeacutetitiviteacute Voici les challenges-cleacutes pour les managers qui veulent rester dans la

course hellip

1 - Future organisation du travail les quatre laquo mondes raquo possibles

La reacutealiteacute sera probablement un meacutelange de ces quatre sceacutenarios souligne lrsquoeacutetude

Les mondes mutuels Tout se passe dans le cadre des communauteacutes locales vie priveacutee

comme professionnelle Le modegravele coopeacuteratif preacutevaut au lieu du laquo big business raquo Oublieacutes

aussi dans ce systegraveme les trajets pour aller au bureau les gens preacutefegravereront travailler dans de

petites entreprises locales souvent connecteacutees au reacuteseau drsquoautres structures similaires

Les laquo reacutepondants raquo (en anglais laquo replicants raquo) La figure du consultant freelance deviendra

dominante tandis que celle du salarieacute deacuteclinera Il ne sera pas rare de travailler pour plusieurs

entreprises On perdra en seacutecuriteacute de lrsquoemploi en visibiliteacute et en routine ce que lrsquoon gagnera

en liberteacute La majeure partie des tacircches srsquoeffectuera chez soi avec la possibiliteacute de srsquoinstaller

temporairement dans les bureaux de son client du moment Dans un contexte dincertitude

sur lrsquoavenir les travailleurs alterneront peacuteriodes drsquoactiviteacute intense et repos Ce sera agrave eux

drsquoaller vers les entreprises et non lrsquoinverse mecircme si celles-ci devront veiller agrave rester attractives

Les cottages eacutelectroniques Comme ce nom le suggegravere le teacuteleacutetravail deviendrait la norme

univers priveacute et professionnel se confondant Plus besoin de subir une heure de transport les

salarieacutes se logueront de chez eux sur le reacuteseau de lrsquoentreprise Les reacuteunions se tiendront dans

de petits bureaux centraux situeacutes agrave courte distance La flexibiliteacute du temps de travail srsquoimpose

Les salarieacutes disposeront de plus de marge de liberteacute dans leur activiteacute

Les disciples de la nueacutee Cette appellation poeacutetique cache simplement une extension de

lrsquoorganisation actuelle des grandes entreprises avec des salarieacutes se rendant sur un lieu de

travail centraliseacute Le rocircle croissant des technologies de lrsquoinformation multipliera les faccedilons de

collaborer et accroicirctra lrsquoefficaciteacute Le controcircle du travail sera omnipreacutesent La frontiegravere entre

travail et vie priveacutee restera marqueacutee

2 - Sept deacutefis pour les entreprises et leur managers

Quoi qursquoil advienne les entreprises et leurs dirigeants devront concentrer leurs efforts sur sept

points-cleacutes pour srsquoadapter Voici quelques exemples de probleacutematiques souleveacutees par le

rapport et des pistes de solution

Le leadership Les managers devront entre autres savoir persuader et influencer des

travailleurs beaucoup plus indeacutependants Ils auront aussi agrave repenser les niveaux auxquels

prendre les deacutecisions strateacutegiques en haut ou au contraire agrave des degreacutes moins eacuteleveacutes de la

pyramide hieacuterarchique

gt Faire du management une force facilitant les activiteacutes transversales plutocirct que la reacuteduire agrave

la seule fonction de deacutecision

La culture drsquoentreprise Davantage de salarieacutes capables de reacutefleacutechir seront neacutecessaires

tandis que les tacircches qui peuvent ecirctre automatiseacutees ou scripteacutees diminueront Un des

enjeux creacuteer une culture agrave mecircme drsquoattirer et drsquoencourager les personnes preacutesentant ces

qualiteacutes de reacuteflexion requises dans un contexte de compeacutetition accrue et de plus grande

indeacutependance des travailleurs

Initiation au management copy CRCF ndash J Sornet Page 44 48

gt Passer si neacutecessaire drsquoune culture drsquoentreprise forte agrave un mode drsquoengagement plus

consensuel moins rebutant

La marque Conseacutequence du recours croissant agrave lrsquo laquo outsourcing raquo lrsquoimage drsquoune marque

deacutependra plus drsquoagents exteacuterieurs qui ne fonctionnent pas forceacutement selon le mecircme mode

drsquoorganisation Comment garder le controcircle dessus

gt Choisir le mode qui corresponde le plus agrave vos valeurs et preacutevoir un programme de risk

management qui mette en eacutevidence ougrave les conflits sont susceptibles de jaillir

Lrsquoinnovation Plus que jamais il faudra faire face agrave une acceacuteleacuteration du rythme de

lrsquoinnovation en proposant constamment des solutions adapteacutees

gt Tisser des partenariats strateacutegiques avec drsquoautres entreprises pour partager les coucircts et les

fruits de lrsquoinnovation

Le deacutefi opeacuterationnel et technologique De quelle faccedilon controcircler lrsquoinformation crsquoest-agrave-dire

faire en sorte que les bonnes personnes accegravedent facilement agrave une information toujours en

phase tout en maintenant la seacutecuriteacute

gt Recourir agrave des laquo feuilles de route des futurs raquo syntheacutetisant en une page les indicateurs

sociaux et de consommation ainsi que les eacutevolutions technologiques et leacutegislatives qui

influent sur les changements et indiquant comment ils modifient vos marcheacutes vos clients et

votre organisation

La qualiteacute Si de nouveaux proceacutedeacutes ont pu deacutegrader la qualiteacute comme le recours agrave des

centres drsquoappel externaliseacutes drsquoautres ideacutees se sont reacuteveacuteleacutees plus prometteuses comme en

teacutemoigne le succegraves de certaines compagnies aeacuteriennes low cost Elles ont su conjuguer prix

serreacutes et services eacuteleveacutes ce qui devra devenir la norme estime lrsquoeacutetude

gt Continuer de rechercher la qualiteacute Elaborez aussi une bonne prestation service qui inclut

une livraison de qualiteacute voire creacuteez-la en partenariat avec les consommateurs

La leacutegislation La question de la proprieacuteteacute intellectuelle pourrait ecirctre probleacutematique Elle est

deacutejagrave source de conflits comme en teacutemoigne le procegraves pour violation de brevet intenteacute agrave RIM

le fabricant canadien du Blackberry par NTP Que pourra-t-on et que faudra-t-il proteacuteger par

un brevet Il sera eacutegalement neacutecessaire drsquoadapter la leacutegislation aux nouveaux modes

drsquoorganisation

gt Collaborer avec les acteurs du mecircme secteur et les leacutegislateurs pour deacutevelopper les

modegraveles des lieux de travail du futur et bacirctir le droit le plus adeacutequat

Orange a-t-il vu juste dans ses preacutevisions Rendez-vous dans neuf ans pour la reacuteponsehellip

Initiation au management copy CRCF ndash J Sornet Page 45 48

Fiche DT3 ndash Le management par la qualiteacute totale

Extrait drsquoune lettre drsquoinformation du cabinet Baud Accordance Consulting AD2 consultants ndash

2002

1 - Le TQM (Total Quality Management) offre pour lentreprise une vision de la qualiteacute plus

large et transversale

Son principe est simple La finaliteacute de lEntreprise est de deacutevelopper la satisfaction de ses

clients tout en eacutetant beacuteneacuteficiaire cest agrave dire pas agrave nimporte quel prix Elle doit ameacuteliorer sa

rentabiliteacute au travers de la deacutemarche qualiteacute La Qualiteacute Totale vise agrave fournir aux clients

externes et internes une reacuteponse adeacutequate agrave leurs attentes dans le meilleur rapport qualiteacute

prix la meilleure efficience

Elle considegravere pour cela lensemble des processus de lentreprise ayant une incidence sur la

qualiteacute et la satisfaction des clients

Le TQM fait ainsi une large place agrave

la deacutefinition et la planification de la strateacutegie geacuteneacuterale

la coheacuterence de la politique qualiteacute avec la strateacutegie

la deacutemultiplication de la politique qualiteacute dans toutes les directions de lentreprise

la relation client fournisseur interne

la prise en compte de lenvironnement concurrentiel

la consideacuteration de lensemble des risques potentiels financiers sociaux concurrentielshellip

limplication et la motivation du personnel

lanalyse des besoins des clients et le positionnement marketing

la maicirctrise des processus transverses internes

les reacutesultats sous tous ses aspects y compris financiers commerciaux image

De nombreux reacutefeacuterentiels sont relatifs agrave la Qualiteacute Totale hellip Tous ces reacutefeacuterentiels imposent un

questionnement plus profond et indiscret sur le mode de fonctionnement de lentreprise et

son management

helliphellip

2 - LISO 9001 2000 au travers du deacuteploiement des processus (management supports

reacutealisation et ameacutelioration continue) reacutepond quelque peu agrave la mecircme logique

LISO est une ouverture indeacuteniable vers la logique du TQM mais ne se reacutefegravere pas agrave la notion

defficience

Les dirigeants sont cependant sensibles agrave la neacutecessaire reacuteduction des coucircts de non-qualiteacute

et dobtention de la qualiteacute agrave la rentabiliteacute du systegraveme de management de la qualiteacute

mais ne perccediloivent pas toujours la qualiteacute comme une deacutemarche globale

Les deacutemarches qualiteacute commencent bien souvent par la remise en cause de lorganisation

leacutevaluation critique de son efficaciteacute lexamen des processus et la mise en eacutevidence des

lourdeurs administratives

La qualiteacute devient laffaire de tous hellip

Initiation au management copy CRCF ndash J Sornet Page 46 48

Fiche DT4 ndash Le deacuteveloppement durable et la RSE

Extrait du site wwwvigeocom

(httpwwwvigeocomcsr-rating-agencyfrmethodologiecriteres-de-recherche37-

criteres-d-analysehtml)

Deacuteveloppement durable laquo un deacuteveloppement qui reacutepond aux besoins du preacutesent sans compromettre

la capaciteacute des geacuteneacuterations futures de reacutepondre aux leurs raquo (Commission mondiale sur lrsquoenvironnement

et le deacuteveloppement ndash 1987)

Reacutefeacuterentiel drsquoeacutevaluation des entreprises par le groupe Vigeacuteo (le groupe mesure les performances et le

niveau de maicirctrise des risques de responsabiliteacute sociale des entreprises et des organisations - site

wwwvigeocom)

1 Ressources Humaines Ameacutelioration continue des relations professionnelles des relations drsquoemploi et des conditions de travail 2 Droits humains sur les lieux de travail Respect de la liberteacute syndicale et promotion de la neacutegociation collective non discrimination et promotion de lrsquoeacutegaliteacute eacutelimination des formes de travail proscrites (enfants travail forceacute) preacutevention des traitements inhumains ou deacutegradants de type harcegravelements sexuels protection de la vie priveacutee et des donneacutees personnelles 3 Environnement Protection sauvegarde preacutevention des atteintes agrave lenvironnement mise en place drsquoune strateacutegie manageacuteriale approprieacutee eacuteco conception protection de la biodiversiteacute et maicirctrise rationnelle des impacts environnementaux sur lrsquoensemble du cycle de vie des produits ou services

4 Comportements sur les marcheacutes Prise en compte des droits et inteacuterecircts des clients inteacutegration de standards sociaux et environnementaux dans la seacutelection des fournisseurs et sur lrsquoensemble de la chaicircne drsquoapprovisionnement preacutevention effective de la corruption respect des regravegles concurrentielles 5 Gouvernement drsquoentreprise Efficience et probiteacute assurance de lrsquoindeacutependance et de lrsquoefficaciteacute du Conseil drsquoadministration effectiviteacute et efficience des meacutecanismes drsquoaudit et de controcircle et notamment inclusion des risques de responsabiliteacute sociale respect des droits des actionnaires et notamment des minoritaires transparence et rationaliteacute de la reacutemuneacuteration des dirigeants 6 Engagement socieacutetal Effectiviteacute inteacutegration manageacuteriale de lrsquoengagement contribution au deacuteveloppement eacuteconomique et social des territoires drsquoimplantation et de leurs communauteacutes humaines engagements concrets en faveur de la maicirctrise des impacts socieacutetaux des produits et des services contribution transparente et participative agrave des causes drsquointeacuterecirct geacuteneacuteral

Initiation au management copy CRCF ndash J Sornet Page 47 48

ELEMENTS DE CORRIGE DT DT1 Deacutefinir expliquer

Deacutereacuteglementation = suppression des contraintes eacuteconomiques (libre eacutechange des biens et

capitaux)

Socieacutetal = qui se rapporte agrave la structure agrave lrsquoorganisation ou au fonctionnement de la socieacuteteacute

Economies drsquoeacutechelle = reacuteduction des coucircts lieacutee au niveau drsquoactiviteacute (amortissement des

charges fixes)

Coaching = accompagnement de personnes ou deacutequipes pour le deacuteveloppement de leurs

potentiels

EDI = eacutechange de donneacutees informatiseacutees ET standardiseacutees (ex SWIFT bancaire edifact

documents deacuteclaratifs)

Gouvernance = exercice du pouvoir la bonne gouvernance est participative et eacutequitable

conforme agrave lrsquointeacuterecirct commun

DT2 Deacuteterminer en quoi la deacutemarche TQM srsquoinscrit dans les deacutefis actuels du management

Voir notamment fiche 43

Maicirctrise des processus reacuteduction des coucircts reacuteactiviteacute et satisfaction de la clientegravele = faire

face agrave la concurrence

Ameacutelioration de lrsquoimage motivation du personnel

DT3 Apregraves avoir consulteacute les documents ci-dessous extraits du site drsquoAir France

(httpdeveloppement-

durableairfrancecomFRfrlocaldemarcheN4_positionnement_pphtm)

exposer les enjeux et les limites de la RSE et de la gestion des parties prenantes

Trame geacuteneacuterale possible

Introduction

Les deacutefis contemporains (accroissement de la concurrence devenue mondiale recherche

de nouveaux avantages concurrentiels pression de la socieacuteteacute besoin drsquoimage et de projet

lisible pour mener lrsquoentreprise crise et scandales du libeacuteralisme hellip) RSE et PP

Deacuteveloppement (voir cours)

1 ndash Parties prenantes et management par la valeur

PP deacutefinir citer reacutesumer lrsquoavantage rechercheacute (fideacuteliser motiver recherche drsquoalliances

implicites)

PP moyens (dont exemples AF) et meacutethode de management par la valeur (reacutepartie)

2 ndash La responsabiliteacute socieacutetale de lrsquoentreprise

RSE 3 axes

- eacuteconomique (favoriser le deacuteveloppement les eacutechanges internationaux)

- social (accegraves aux soins eacuteducation conditions de travail hellip)

- environnemental (pollution preacuteservation des ressources hellip)

RSE gouvernance drsquoentreprise facteur drsquoimage inteacutegrable dans la deacutemarche PP

Article 116 de la loi Le rapport viseacute agrave larticle L 225-102 rend compte hellip laquo Il comprend

eacutegalement des informations dont la liste est fixeacutee par deacutecret en Conseil dEtat sur la maniegravere

dont la socieacuteteacute prend en compte les conseacutequences sociales et environnementales de son

activiteacute Le preacutesent alineacutea ne sapplique pas aux socieacuteteacutes dont les titres ne sont pas admis aux

neacutegociations sur un marcheacute reacuteglementeacute raquo

Initiation au management copy CRCF ndash J Sornet Page 48 48

RSE exemple AF (ONG fournisseurs)

3 ndash Liens entre PP et RSE

- la RSE introduit de nouvelles PP

- la RSE suppose le respect des PP usuelles (employeacutes clients notamment)

4 - Probleacutematique

- deacutefinir la valeur reacuteellement apporteacutee par une gestion des PP (confusion salaire ndash valeur

idem impocircts hellip ex laquo valeur ajouteacutee raquo)

- communication (neacutecessaire mais aller au-delagrave)

- marginaliteacute des deacutepenses RSE (efficaciteacute sinceacuteriteacute de lrsquoengagement marge de manœuvre)

- charge RSE reporteacutee sur des tiers (ex fournisseurs AF)

- inteacutegration de facteurs non visibles en comptabiliteacute (pertes drsquoemploi nuisances hellip)

Conclusion

Voies incontournables mais pouvant nrsquoavoir qursquoun effet superficiel et temporaire Voir utiliteacute

drsquoaccompagnement leacutegislatif de regravegles de gouvernance

Initiation au management copy CRCF ndash J Sornet Page 4 48

- Peter Drucker pour qui le management repose sur cinq principes fixer des objectifs

organiser le travail motiver et communiquer former les eacutequipes (au sens de lrsquoapprentissage

laquo former les autres et soi-mecircme raquo)

- Henry Mintzberg qui met lrsquoaccent sur le rocircle unificateur du manager laquo Ce qui distingue

avant tout une organisation formelle drsquoun quelconque rassemblement drsquohommes ndash drsquoune

foule drsquoun groupe informel ndash crsquoest la preacutesence drsquoun systegraveme drsquoautoriteacute et drsquoadministration

personnifieacute par un ou plusieurs managers dans une hieacuterarchie plus ou moins structureacutee et

dont la tacircche est drsquounir les efforts de tous dans un but donneacute raquo

On en deacuteduit le rocircle relationnel du manager et le poids de la gestion des ressources

humaines dans son activiteacute

LA PRIMAUTE DE LrsquoINFORMATION

Le management fixe des objectifs il a un rocircle deacutecisionnel et il creacutee les conditions neacutecessaires

pour atteindre les objectifs dont il controcircle la reacutealisation Tout ceci neacutecessite la mise en place

drsquoun systegraveme drsquoinformation fournissant les donneacutees neacutecessaires agrave des choix pertinents

(donneacutees qui remontent souvent par les managers eux-mecircmes) puis agrave la mise en œuvre et

au controcircle des reacutealisations

23 ndash Les dimensions du management

La fonction de management a de fait trois dimensions humaine (faire travailler des

personnes ensemble motiver) eacuteconomique (fonctionnement ou deacuteveloppement de

lrsquoorganisation aux meilleurs conditions) et informationnelle (dont la communication)

Le management est une activiteacute de synthegravese qui neacutecessite des capaciteacutes agrave traiter et

combiner des informations drsquoorigines diverses plus ou moins varieacutees selon le contexte

(financiegravere technique leacutegale eacuteconomique politique hellip) et agrave agir en conseacutequence dans les

trois dimensions

Humain

Economique

Information

Informations sur

lrsquoenvironnement

Informations sur

lrsquoorganisation

Management

Information

dirigeant lrsquoaction

des autres

Actions directes

du manager

Initiation au management copy CRCF ndash J Sornet Page 5 48

24 ndash La recherche de performance

Le management recherche la performance de lrsquoorganisation en rapport avec ses objectifs

qui peuvent ecirctre de diffeacuterentes natures (expansion profit hellip ou agrave un niveau plus deacutetailleacute

reacuteduction des deacutelais flexibiliteacute reacuteduction des coucircts accroissement de la qualiteacute hellip)

La performance peut ecirctre abordeacutee de deux faccedilons

- par lrsquoefficaciteacute qui conduit agrave la reacutealisation des objectifs

- par lrsquoefficience qui conduit agrave une utilisation optimale des moyens disponibles avec le

meilleur rendement

Exemple une entreprise qui atteint son objectif de croissance de 10 du chiffre

drsquoaffaires et de son beacuteneacutefice est efficace mais celle qui arrive au mecircme reacutesultat avec

moins drsquoactifs et moins de personnel est plus efficiente

25 ndash Le champ drsquoaction du management

Le management est mis en œuvre dans des peacuterimegravetres drsquoampleur variable un groupe une

entreprise une activiteacute particuliegravere de lrsquoentreprise un projet la tenue drsquoun magasin hellip Les

actions de management ont ainsi des conseacutequences plus ougrave moins importantes

Le management srsquoexerce agrave tous les niveaux drsquoencadrement de lrsquoorganisation Il est de

coutume de distinguer trois niveaux

Exemple

Management strateacutegique la direction geacuteneacuterale a fixeacute les objectifs de chaque filiale du

groupe en tenant compte des positions souhaiteacutees sur le marcheacute Il en reacutesulte que les

uniteacutes X et Y aux activiteacutes similaires et compleacutementaires doivent preacuteparer leur fusion

preacutevue dans les deux ans La filiale Z situeacutee dans une ville universitaire doit acqueacuterir des

compeacutetences pour innover dans lrsquoapplication des supra conducteurs drsquoici trois ans Les

budgets et les financements correspondants ont eacuteteacute globalement estimeacutes

Management intermeacutediaire en application de ces orientations strateacutegiques les

directions de X et Y planifient des reacuteunions de travail communes pour voir comment

harmoniser leurs ressources humaines dans les douze mois Z deacutecide de contractualiser

ses relations avec le centre universitaire pour atteindre ses objectifs et mettre en place

un partenariat en recherche appliqueacutee

Management strateacutegique ou geacuteneacuteral

(direction laquo top management raquo deacutefinition

des objectifs geacuteneacuteraux)

Management tactique

(intermeacutediaire laquo middle

managers raquo)

Management opeacuterationnel (de

terrain ou drsquouniteacute local

laquo executive manager raquo)

Initiation au management copy CRCF ndash J Sornet Page 6 48

Management opeacuterationnel X et Y mettent en place leur communication avec le

personnel et un plan de reconversion Z nomme un directeur de recherche qui prend

notamment en charge les relations avec lrsquouniversiteacute

Le management recouvre la totaliteacute des actes de conduite de lrsquoorganisation dans tous les

domaines (technique commercial financier hellip) mais il ne correspond agrave aucune cellule de

lrsquoorganigramme

Les actions de management sont par ailleurs contraintes par la disponibiliteacute des ressources

neacutecessaire pour atteindre les objectifs (financement savoir-faire profil du personnel

eacutequipements hellip)

3 ndash La meacutethode laquo management raquo

31 ndash La science du management

Le management nrsquoest pas une science exacte il srsquoapparente agrave une science humaine

expeacuterimentale qui traite de pheacutenomegravenes socio-eacuteconomiques eacutevolutifs et qui doit trouver

concregravetement son application dans la vie des organisations

Cette science traite notamment de lrsquoorganisation des entreprises et rassemble des meacutethodes

et des theacuteories qui peuvent ecirctre regroupeacutees en eacutecoles ou en courants

Les theacuteories marquent geacuteneacuteralement une eacutepoque et elles peuvent se recouper partiellement

parfois srsquoopposer avec des nuances qui doivent ecirctre bien identifieacutees Elles srsquoaccompagnent

souvent de modegraveles et de scheacutematisations qui en facilitent la compreacutehension et la

transposition agrave de nouvelles situations

Ces outils scientifiques guident le raisonnement permettent drsquoappreacutehender des reacutealiteacutes

complexes et structurent les connaissances ils sont peacutedagogiques et constituent des aides

pour lrsquoaction Leur application doit cependant ecirctre raisonneacutee car

- un modegravele nrsquoest qursquoune simplification de la reacutealiteacute

- une theacuteorie sortie de son contexte historique et eacuteconomique peut perdre de sa pertinence

Sa mise en œuvre doit tenir compte de la situation reacuteelle et une theacuteorie ne peut agrave elle seule

justifier une deacutecision de management (ce drsquoautant plus que chaque theacuteorie ne couvre que

tregraves partiellement le domaine du management ou de lrsquoorganisation)

- il est souvent difficile de disposer agrave temps drsquoinformations fiables et suffisantes pour appliquer

une theacuteorie dans les conditions ideacuteales Le manager est freacutequemment ameneacute agrave prendre ses

deacutecisions sur la base drsquoinformations incomplegravetes ou incertaines et il doit alors en mesurer les

risques et preacutevoir les ajustements neacutecessaires

ENTREPRISE

Management Administration

Gestion

Strateacutegique

Opeacuterationnel

Initiation au management copy CRCF ndash J Sornet Page 7 48

- des modes influencent le management Elles peuvent indiquer une veacuteritable eacutevolution

eacuteconomique mais aussi ecirctre sans lendemain voire introduire un danger ou un coucirct inutile

(se meacutefier des speacutecialistes dont le fond de commerce est la vente de nouvelles techniques

de management et des seacuteminaires associeacutes)

- seule la creacuteation drsquoune combinaison originale (dans le respect des regravegles) peut apporter un

avantage agrave lrsquoentreprise et non la reproduction de choix de management connus de tous

Exemple 1 la matrice de portefeuille drsquoactiviteacutes permet de classer les activiteacutes

strateacutegiques drsquoune entreprise en fonction de leur taux de croissance et de la part de

marcheacute deacutetenue (matrice laquo BCG raquo - Boston consulting group ndash 1975) Cet outil de

management neacutecessite de disposer drsquoinformations fiables concernant le marcheacute Il

conduit souvent agrave simplifier les conditions de concurrence et ignore la

compleacutementariteacute pouvant exister entre activiteacutes (synergie partage de techniciteacute

amortissement de charges fixes hellip) La matrice est un moyen de prendre conscience

du portefeuille et de ses eacuteventuels deacutefauts (portefeuille deacuteseacutequilibreacute avec

preacutedominance anormale drsquoune zone) mais il ne permet pas seul de deacutecider de lrsquoavenir

des activiteacutes

Exemple 2 le lancement drsquoun investissement lourd (lrsquoimplantation de nouvelles usines

le lancement drsquoune nouvelle activiteacute agrave fort taux de recherche ndash deacuteveloppement hellip) ne

peut ecirctre deacutecideacute que par le recoupement de diffeacuterentes approches (financiegravere

commerciale strateacutegique) Aucune theacuteorie du management appliqueacutee isoleacutement et

sans preacutecaution ne peut justifier un tel investissement

Exemple 3 une entreprise produisant des eacutequipements meacutedicaux deacutecide de renforcer

son offre commerciale par un service de conseil et drsquoassistance Cette deacutecision est

inspireacutee par le concept de creacuteation de valeur par une volonteacute de se diffeacuterencier de la

concurrence et par des pratiques existant depuis longtemps dans le domaine

informatique

Remarque parmi les facteurs influenccedilant les principes de management il est utile de

savoir deacutetecter pour srsquoen preacutemunir les ideacuteologies preacutejugeacutes et autres laquo valeurs raquo sans

rapport certain avec lrsquoefficaciteacute eacuteconomique

32 ndash Theacuteorie et pratique du management

La litteacuterature (ouvrages revues) aborde le management sous des angles diffeacuterents qui

peuvent suggeacuterer une concurrence entre des approches theacuteorique et pratique ou

psychologique et eacuteconomique de cette discipline

Taux de croissance

du domaine (cf

cycle de vie du

produit)

Part de marcheacute

(compareacutee au

principal

concurrent)

fort

faible

forte faible

Dilemmes (activiteacutes

en phase de

lancement

potentiel + coucircts+

risque+)

Vedettes

(activiteacutes en forte

croissance

autofinancement+)

Vaches agrave lait

(activiteacutes

stabiliseacutees et

concurrence faible

ou stable

rentabiliteacute+)

Poids morts

(activiteacutes en

deacuteclin)

Initiation au management copy CRCF ndash J Sornet Page 8 48

Le management a cependant une viseacutee unique et concregravete la bonne marche des

organisations et les travaux des universitaires rejoignent lrsquoaction des praticiens (les

universitaires reacutealisent drsquoailleurs bien souvent leurs recherches au sein des organisations)

Au plan peacutedagogique les deux visions du management se complegravetent

- la vision acadeacutemique met en perspective des concepts et des theacuteories Elle permet de

srsquoapproprier des raisonnements essentiels et stables notamment concernant la strateacutegie et

les reacuteactions humaines qui sont le moteur des organisations

- la vision pratique relate le veacutecu des praticiens preacutesente des techniques concregravetes et tente

parfois une vulgarisation favorisant la diffusion des concepts Elle integravegre de faccedilon

pragmatique les objectifs eacuteconomiques et les contraintes de fonctionnement des

organisations en se placcedilant parfois dans une vision agrave court terme

Sur le terrain le management reacutealise une synthegravese entre des techniques de psychologie

sociale et des techniques de gestion de diverses origines et de nouvelles meacutethodes

apparaissent aussi sous le label unique laquo management raquo

Le terme manageacuterial (approche manageacuteriale theacuteorie manageacuteriale pratique manageacuteriale

hellip) fait reacutefeacuterence agrave la vision aux preacuteoccupations et aux actions des managers qui doivent

emmener leur organisation vers la reacutealisation de ses objectifs

Exemples

- la deacutetermination des coucircts et des marges reacutesulte de techniques de gestion

indispensable aux deacutecisions des managers

- le laquo CRM raquo (customer relationship management en franccedilais GRC ndash gestion de la

relation client) est un concept reacutecent qui srsquoaccompagne de techniques lieacutee aux plus

reacutecents deacuteveloppements du management et des systegravemes drsquoinformation

- le laquo coaching raquo est une technique de management permettant une eacutevolution

personnelle dans le sens des objectifs de lrsquoentreprise

- le recentrage sur le laquo meacutetier raquo repose sur des techniques de management

Les techniques sont rassurantes mais pas suffisantes et si les theacuteories ne sont pas

indispensables au praticien elles aident agrave comprendre agrave anticiper et agrave bien utiliser les

techniques

La qualiteacute du management reacuteside beaucoup dans la capaciteacute agrave appliquer concepts et

techniques de faccedilon pertinente et agrave innover Cette capaciteacute relegraveve en partie drsquoun laquo art du

management raquo qui srsquoacquiert en grande partie par la pratique

4 ndash Management et expertise comptable

Lrsquoexpert comptable doit manager ses propres eacutequipes Il est par ailleurs supposeacute laquo hellip

conseiller et accompagner le chef drsquoentreprise dans toutes ses deacutecisions hellip raquo (selon lrsquoOEC)

Ce rocircle est particuliegraverement important dans ses relations avec les petites entreprises Il doit

donc avoir une capaciteacute au management

Organisation

Vision acadeacutemique Vision pratique

Initiation au management copy CRCF ndash J Sornet Page 9 48

Lrsquointervention de lrsquoexpert dans le management drsquoune entreprise peut toutefois poser

quelques problegravemes

- Il peut y avoir conflits drsquointeacuterecirct entre activiteacutes de certification des comptes et de conseil en

management (la tendance est agrave la seacuteparation des activiteacutes dans les plus grands cabinets)

- lrsquoactiviteacute de conseil neacutecessite des compeacutetences parfois tregraves speacutecifiques (conseil fiscal

conseil en RH conseil en systegravemes drsquoinformation hellip)

- le conseil est une activiteacute diffeacuterente par sa forme de lrsquoexpertise comptable (interventions

longues peu reacutepeacutetitives peu codifieacutees mises en concurrence) qui neacutecessite une

organisation particuliegravere du cabinet lorsqursquoelle deacutepasse lrsquointervention occasionnelle

APPLICATIONS IM

IM1 Analyser la profession de laquo manager raquo selon Henry Mintzberg (texte extrait de lrsquoouvrage

laquo Le management raquo Eyrolles - Editions drsquoOrganisation) et les principes du management de la

norme ISO

Faire ressortir les eacuteleacutements speacutecifiques agrave chacune de ces approches et mettre en eacutevidence

leurs points communs

Initiation au management copy CRCF ndash J Sornet Page 10 48

Principes du management drsquoapregraves la norme ISO 9001 (2000)

- Orientation vers le client (satisfaire ses attentes)

- Leadership (les dirigeants eacutetablissent les orientations de lrsquoorganisme Ils doivent creacuteer

un environnement interne ougrave les personnes peuvent clairement srsquoimpliquer dans la

reacutealisation des objectifs de lrsquoorganisme)

- Implication du personnel (les personnes sont agrave tout niveau lrsquoessence de lrsquoorganisme et

leur implication permet drsquoutiliser leurs aptitudes au profit de lrsquoorganisme)

- Approche laquo processus raquo (un reacutesultat est mieux atteint quand les ressources et les

activiteacutes neacutecessaires sont geacutereacutees comme un processus)

- Approche systegraveme (assimiler les processus correacuteleacutes agrave un systegraveme contribue agrave

lrsquoefficaciteacute et agrave lrsquoefficience de lrsquoorganisme vis-agrave-vis de ses objectifs)

- Ameacutelioration continue (objectif permanent de lrsquoorganisme)

- Prise de deacutecision efficace (par lrsquoanalyse de donneacutees et drsquoinformations)

- Relations mutuellement beacuteneacutefiques avec les fournisseurs (pour augmenter la capaciteacute

des deux organismes agrave creacuteer de la valeur)

IM2 Distinguer leader et manager

IM3 Compleacuteter le tableau ci-dessous en analysant chaque action preacutesenteacutee Faire ensuite

ressortir les domaines niveaux ou techniques de management pouvant ecirctre mobiliseacutes pour

chaque situation

Initiation au management copy CRCF ndash J Sornet Page 11 48

Caracteacuteristiques

de lrsquoaction

- reacutepeacutetition

- risque

- normes

- ampleur

Prise de

deacutecision

- opeacuterationnelle

strateacutegique

- deacutelai

Informations

neacutecessaires

- nature

- origine

- deacutelai obtention

Cleacutes pour la

reacuteussite

Intervention

exteacuterieure

possible

Assurer la

restauration du

soir

(restaurant

familial)

Construire un

viaduc

(autoroute)

Certifier les

comptes

annuels drsquoun

groupe

national

(cabinet

drsquoaudit)

Lancer une

ligne drsquoavions

(constructeur

aeacuteronautique)

Reacuteduire la

capaciteacute de

production

(groupe

industriel)

Acqueacuterir une

entreprise

concurrente

(teacuteleacutephonie

mobile)

Initiation au management copy CRCF ndash J Sornet Page 12 48

Fiche IM1 - Deacutefinitions du management

Dictionnaire anglais - franccedilais direction administration gestion intrigue manegravege

Wikipeacutedia Le management est lensemble des techniques dorganisation qui sont mises en

oeuvre pour ladministration dune entiteacute

Au point de vue eacutetymologique le verbe manage vient de litalien maneggiare (controcircler)

influenceacute par le mot franccedilais manegravege (faire tourner un cheval dans un manegravege) A cette

notion il faut aussi ajouter la notion de meacutenage (geacuterer les affaires du meacutenage) qui consiste agrave

geacuterer des ressources humaines et des moyens financiers

helliphellip

Fiche IM2 - Etudier le management

Concreacutetiser

Manager neacutecessite de syntheacutetiser des informations parfois complexes incomplegravetes et de

domaines tregraves divers pour en deacuteduire des actions Une approche trop parcellaire peut

conduire agrave lrsquoeacutechec et le savoir-faire est neacutecessaire pour agir vite avec un minimum de risque

Lrsquoeacutetudiant doit se preacuteparer simultaneacutement aux examens et agrave la pratique Il nrsquoa souvent connu

lrsquoentreprise que durant quelques semaines de stage et le manque de laquo recul raquo ne lui permet

pas toujours de concreacutetiser les theacuteories Il doit compenser par la lecture (ouvrages revues

journaux eacuteconomiques et boursiers) et en eacutetant attentif aux informations ambiantes (tout en

relativisant le style journalistique) en mettant en relation le cours les concepts les modegraveles

lrsquoactualiteacute les stages

Savoir traiter un exercice

Pour reacuteussir un examen ou traiter une application peacutedagogique (la conception les points 1 agrave

6 peut repreacutesenter le tiers du temps de travail)

1 ndash Identifier le type de sujet (faut-il trouver une solution pratique ou communiquer une

reacuteflexion geacuteneacuterale )

2 ndash Lire le sujet et relever les mots cleacutes

3 ndash Deacutefinir les mots cleacutes

4 ndash Reacutesumer la probleacutematique du sujet (en quelques lignes)

5 ndash Lister les connaissances reacutefeacuterences et raisonnements reacutepondant au problegraveme (par

recherche spontaneacutee ou raisonneacutee qui quoi ougrave quand comment combien hellip

listage des diffeacuterents points de vue) trouver des exemples (notamment dans les

documents fournis)

6 ndash Organiser la reacuteponse (deacutefinir le plan du deacuteveloppement ougrave des paragraphes bien

identifieacutes sont geacuteneacuteralement neacutecessaires en y liant les parties qui doivent ecirctre en nombre

limiteacute ndash de deux agrave quatre) Preacutevoir drsquoy inteacutegrer la deacutefinition des principales notions

induites par le sujet

7 ndash Reacutediger sous la forme adapteacutee (note technique ou recommandation solution

pratique exposeacute structureacute dissertation)

Introduction et conclusion sont indispensables agrave la dissertation ou agrave lrsquoexposeacute

- lrsquointroduction preacutesente le sujet traiteacute (phrase drsquoaccroche initiale) amorce la

probleacutematique (quelques sous - questions) et annonce le plan

- la conclusion syntheacutetise le deacuteveloppement (arguments) eacutelargit le sujet (prise de recul)

et apporte le point final (une phrase)

Une limite agrave la communication

Il est difficile de faire passer plus de 4 ou 5 ideacutees fortes dans un exposeacute unique

Initiation au management copy CRCF ndash J Sornet Page 13 48

Fiche IM3 - Bref historique

Antiquiteacute

3000 AJC

Peacuteriode greacuteco-

romaine

Transition

feacuteodale

12egraveme siegravecle

europe

15egraveme ndash 17egraveme

siegravecles

19egraveme siegravecle

20egraveme siegravecle

agriculture preacutedominante industrie limiteacutee aux besoins drsquoun individu ou drsquoun clan

pour la confection des outils des vecirctements et de la poterie Force motrice animale

ou humaine pour lrsquoessentiel

Grands travaux drsquoeacutetat en Egypte premiegravere laquo planification ndash organisation ndash controcircle raquo

Deacuteveloppement des communications essor industriel limiteacute peu de progregraves

technique (lrsquoesclavage supplante les innovations)

Deacuteveloppement progressif des eacutechanges commerciaux

La consommation indirecte atteint un bon niveau (surplus agricoles et

deacuteveloppement des villes) Apparition de nouveaux commerccedilants

Etat fort Evolutions technologiques (imprimerie bateaux performants instruments de

navigation) Extension geacuteographique de lrsquoeacuteconomie Apparition des corporations

drsquoartisans

Machine agrave vapeur chemin de fer passage de lrsquoartisanat au capitalisme

entrepreneurial producteur organisation des entreprises

Ecole classique (Taylor Fayol Weber) approche meacutecaniste bureaucratie

hieacuterarchie commandement fonctions et speacutecialisation laquo OS T raquo (organisation

scientifique du travail) organisation source de pouvoir rationaliteacute des individus bases

du management

Deacuteveloppement du capitalisme manageacuterial Electriciteacute peacutetrole puis communications

et information Consommation de masse mondialisation preacuteoccupations

eacutenergeacutetiques et environnementales 3 peacuteriodes

- standardisation grandes entreprises industrielles

- industries de consommation 30 glorieuses marketing multinationales protection

sociale

- deacutereacuteglementation monteacutee des services pays eacutemergents mondialisation et nouvelle

eacuteconomie (internet)

Ecole des relations humaines prise en compte de lrsquoindividu des motivations styles

de direction

Ecole neacuteo-classique et post-classique deacutecentralisation coordonneacutee DPO

management participatif zeacutero deacutefaut flux tendus

Approche systeacutemique partition de lrsquoentreprise eacutetude des interactions feacutedeacuteration

vers lrsquoobjectif controcircle et ajustement

Theacuteories de la deacutecision rationaliteacute limiteacutee contribution reacutetribution coalitions

Ecole socio-technique recherche de compromis technologie organisation

enrichissement des tacircches autonomie des groupes

Approche sociologique effets sociaux du travail jeux de pouvoir dans lrsquoentreprise

reacutegulation sociale

Theacuteories de la contingence facteurs contingents adaptation agrave lrsquoenvironnement

configurations organisationnelles

Theacuteories de la firme controcircle manageacuterial droits de proprieacuteteacute relation drsquoagence

Theacuteories contractualistes firme nœud de contrats coucircts de transaction

opportunisme externalisation internalisation

Approche eacutevolutioniste eacutecologie des organisations modegravele eacutevolutioniste

contraintes de sentier

Approche par les ressources valorisation des ressources compeacutetences cleacutes

apprentissage organisationnel

(Classement simplifieacute)

Initiation au management copy CRCF ndash J Sornet Page 14 48

ELEMENTS DE CORRIGE IM

IM1 Commenter la deacutefinition du management par la norme ISO et le manager de Mintzberg

Efficient = optimum avec les moyens disponibles

ISO (management objectifs) (manager moyens) HM

IM2 Le leader entraicircne naturellement derriegravere lui Le manager nrsquoest pas toujours leader

(mecircme si crsquoest souhaitable) Le leader nrsquoest pas toujours manager (plutocirct notion individuelle)

Leadership = faculteacute de diriger conjugaison drsquoune autoriteacute naturelle ou drsquoun savoir-faire

acquis drsquoune capaciteacute agrave entraicircner des personnes ou des groupes et drsquoune leacutegitimiteacute

statutaire (de position)

IM3 Compleacuteter le tableau ci-dessous en analysant chaque action preacutesenteacutee Faire ensuite

ressortir les domaines niveaux ou techniques de management pouvant ecirctre mobiliseacutes pour

chaque situation

Satisfaction client

Implication du personnel

Processus systegraveme

Ameacutelioration continue

Deacutecision efficace

Recherche de valeur

Image entreprise

Liaisons

Information

Reacutepartition ressources

Reacutegulation

Neacutegociation

Leadership

Initiation au management copy CRCF ndash J Sornet Page 15 48

Caracteacuteristiques

de lrsquoaction

- reacutepeacutetition

- risque

- normes

- ampleur

Prise de

deacutecision

- opeacuteration

- direction

- deacutelai

Informations

neacutecessaires

- nature

- origine

- deacutelai

obtention

Cleacute pour la

reacuteussite

Intervention

exteacuterieure

possible

Assurer la

restauration du

soir

(Restaurant

familial)

Technique

(fabrication)

Vente (terrain)

Appros

Reacutepeacutetitive

(quot)

Risque faible

Normes

drsquohygiegravene

Faible

Opeacuterationnelle

Geacuterant

responsable

Rapide (qq

jours menu et

appros)

Nombre de

couverts

Tarifs usuels

Calendrier

(fecirctes)

Clients docs

divers

expeacuterience

Qq jours

Varieacuteteacute menu

Plats phares

Accueil

Appros

Tarification

Vins

Gestion

congeacutelation

Qualiteacute cuisine

Fournisseurs

Extra

Publiciteacute

Construire un

viaduc

(autoroute)

Technique

Organisation

Appros

Uniteacute (ou peu)

Eleveacute (financier

technique)

Architecture

Eleveacutee

Direction

(aleacuteas)

Opeacuterationnelle

(conduite

chantier)

Immeacutediat agrave qq

semaines

Plans

plannings

Qualifications

Meacuteteacuteo

Disponibiliteacutes

Bureau eacutetudes

Qq sem agrave 24h

Techniciteacute

Appros

Qualifications

Preacutevision

GRH

Contrat juste

SS traitants

Organismes

certificateurs

Controcircle

client

Certifier les

comptes

annuels drsquoun

groupe national

(cabinet

drsquoaudit)

Technique

Relation client

Gestion des

connaissances

Annuelle

Moyen

Regravegles

comptables

fiscales

Moyenne (selon

importance du

cabinet)

Opeacuterationnelle

Qq jours agrave

semaines

Comptable

Juridique

Client

Etat

Qq jours agrave

semaines

Techniciteacute

Expeacuterience

Relation client

Systegraveme info client

Siegravege

Autre cabinet

Lancer une

ligne drsquoavions

(constructeur

aeacuteronautique)

Strateacutegique

RD

Etudes

Uniteacute

Tregraves eacuteleveacute

Aeacuteronautique

Tregraves eacuteleveacutee

Direction

Qq mois agrave

anneacutees

Marcheacute

Etudes

Compagnies

Qq mois agrave

anneacutees

Concept

Outil industriel

Coucirct exploitation

Tarif

Fiabiliteacute

Deacutelaisconcurrence

SI simulation

SS traitants

Bureaux

drsquoeacutetudes

speacutecialiseacutes

Compagnies

Conseils

Reacuteduire la

capaciteacute de

production

(groupe

industriel)

Strateacutegique

RH

Communication

Production

Uniteacute

Moyen

Leacutegislation

(dont RH)

Eleveacutee

Direction

Qq mois agrave

anneacutees

Financiegravere

Industrielle

Marcheacute

Organisation

Organismes

speacutecialiseacutes

DRH

Qq mois

Communication

Connaissance des

compeacutetences

Connaissance outil

industriel

Concurrence

Portefeuille

drsquoactiviteacutes

Cabinet

drsquoorganisation

Conseils

speacutecifiques

Acqueacuterir une

entreprise

concurrente

(teacuteleacutephonie

mobile)

Strateacutegique

Marketing

Production

(reacuteseau)

Financier

Communication

Uniteacute

Tregraves eacuteleveacute

Leacutegislation

telecom

Tregraves eacuteleveacutee

Direction

Qq mois

Financiegravere

Marcheacute

Reacuteseaux

(ampleur

recouvrement

hellip)

Organisations

Interne

Racheteacutee

Sources

speacutecialiseacutees

Qq mois

Communication

Marcheacute

Cours boursiers

Cabinet

drsquoorganisation

Conseils

speacutecifiques

Initiation au management copy CRCF ndash J Sornet Page 16 48

LE MANAGEMENT EN PRATIQUE

Pour assumer sa fonction le management doit couvrir sans discontinuiteacute lrsquoensemble de

lrsquoorganisation et inteacutegrer de nombreux facteurs dont nous allons reacutesumer lrsquoessentiel

1 ndash Les fonctions et activiteacutes du management

Pour Henri Fayol la fonction drsquoadministration de lrsquoentreprise (son management) reposait sur

cinq actions preacutevoir organiser commander coordonner et controcircler (laquo PO3C raquo)

Nous distinguerons cinq activiteacutes de management

- la conception (au plus haut niveau finaliteacute but ou vocation de lrsquoorganisation

meacutetiers dimension politique de croissance hellip)

- la planification (deacutefinition des objectifs eacutecheacuteances)

- lrsquoorganisation (reacutepartition du travail choix des modes de coordination)

- le pilotage de lrsquoaction opeacuterationnelle (motivation animation encadrement

assistance)

- lrsquoeacutevaluation (controcircle des reacutesultats obtenus ajustements)

Dans chacune de ces activiteacutes des deacutecisions et des arbitrages sont neacutecessaires avec des

enjeux plus ou moins importants

Remarques

- Les cinq activiteacutes du management peuvent se retrouver agrave tout niveau de

management si lrsquoentreprise laisse une certaine autonomie de deacutecision agrave ses diffeacuterentes

uniteacutes La conception est naturellement du ressort de la direction geacuteneacuterale et des

conseils drsquoadministration mais elle peut ecirctre preacutesente pregraves du terrain (latitude laisseacutee agrave

une filiale ou agrave un magasin par exemple) De mecircme lrsquoorganisation du travail concerne

un atelier mais aussi la direction qui structure lrsquoentreprise pour assurer ses activiteacutes sa

production

- La planification deacutefinit des objectifs ou des axes strateacutegiques (choix de produits

modaliteacutes de deacuteveloppement des ventes implantations alliances hellip) et les traduit en

donneacutees de gestion preacutevisionnelles syntheacutetiques et eacutechelonneacutees dans le temps afin de

valider les objectifs et de fixer des repegraveres

- Un laquo business plan raquo (plan drsquoaffaires)est notamment lrsquoeacutequivalent de la planification

dans le cas de creacuteation drsquoentreprise ou pour la preacutesentation de tout projet drsquoactiviteacute

Les activiteacutes du management srsquoinscrivent dans des cycles qui peuvent ecirctre scheacutematiseacute

comme suit (lrsquoeacutevaluation peut entraicircner une reacutevision du pilotage de lrsquoorganisation ou des

objectifs sans que lrsquoentreprise ne soit fondamentalement remise en cause)

conception

planification

organisation

pilotage

eacutevaluation

Initiation au management copy CRCF ndash J Sornet Page 17 48

2 ndash Les contextes de management

Le management est influenceacute par son contexte qui justifie des objectifs une organisation

des meacutethodes

Par exemple lrsquoentreprise admet de nombreuses variantes selon sa taille sa forme juridique

son controcircle par lrsquoeacutetat (entreprises publiques) ou par des inteacuterecircts priveacutes Il en va de mecircme des

organismes administratifs qui peuvent deacutependre de directives nationales ou reacutegionales des

associations qui ont des activiteacutes drsquoampleur tregraves variable

21 ndash La dimension de lrsquoentreprise

La dimension drsquoune entreprise se mesure principalement en fonction de son effectif ou de

son chiffre drsquoaffaires Des seuils sont deacutefinis par divers organismes et exploiteacutes agrave des fins

statistiques ou pour la deacutetermination de certaines obligations sociales ou fiscales

(repreacutesentation du personnel cotisations hellip) Il nrsquoy a bien entendu pas de laquo barriegravere de

tailleraquo absolue conditionnant le management drsquoune entreprise

LrsquoUE preacuteconise de distinguer les micro ndash entreprises (jusqursquoagrave 9 salarieacutes) les TPE ndash tregraves petites

entreprises (moins de 20 salarieacutes) les petites entreprises (moins de 50) et les moyennes

entreprises (de 50 agrave 250) Cependant les PME sont parfois situeacutees entre 10 et 500 salarieacutes

Remarques

- en France environ 40 des entreprises emploient de 1 agrave 50 salarieacutes (ce qui repreacutesente

plus de 50 des emplois) et 59 nrsquoen ont aucun

le pays compte environ 2 600 000 entreprises dont moins de 1 ont 250 employeacutes et

plus

- ancienneteacute et taille de lrsquoentreprise sont lieacutees si lrsquoon eacutecarte les restructurations et autres

eacutevolutions drsquoentreprises existantes

La dimension de lrsquoentreprise a une influence sur lrsquoorganisation et le laquo style raquo de son

management

- les PME sont souvent entrepreneuriales (les dirigeants eacutegalement apporteurs de capitaux

sont totalement engageacutes dans la marche de lrsquoentreprise) Elles ont une gestion flexible peu

formaliseacutee plus qualitative que quantitative Les PME sont freacutequemment focaliseacutees sur un seul

type drsquoactiviteacute Pour ne pas alourdir leur structure elles ont tendance agrave sous-traiter les

activiteacutes speacutecialiseacutees ne correspondant pas agrave leur meacutetier de base

- les grandes entreprises sont manageacuteriales (les dirigeants sont nommeacutes par les actionnaires

en raison de leurs compeacutetences) et moins reacuteactives

22 ndash Le type de production

On distingue industrie (production de biens mateacuteriels ou pour le moins de produits visibles ndash

comme un seacutejour touristique ou un film) et services (fourniture drsquoune prestation immateacuterielle)

Le type de production influence en principe le management de lrsquoentreprise

- lrsquoindustrie neacutecessite (si lrsquoon excepte lrsquoartisanat) un investissement relativement important

une organisation productive stable capable de reacutealiser plusieurs fois des produits identiques

(exemple un modegravele de reacutefrigeacuterateur) ou du moins similaires (exemple un bacirctiment) Le

produit de lrsquoindustrie consomme des matiegraveres et il doit geacuteneacuteralement ecirctre distribueacute jusqursquoau

client

- la production de services peut se satisfaire drsquoun investissement tregraves reacuteduit et neacutecessite un

contact permanent avec le client

Toutefois la standardisation des services et le deacuteveloppement des reacuteseaux informatiques

rapprochent la production de services de celle des biens industriels

- la production drsquoun service reacutepeacutetitif et technique peut imposer une structure lourde et une

organisation tregraves formaliseacutee (voir les grandes socieacuteteacutes drsquoaudit ou de conseil informatique)

Initiation au management copy CRCF ndash J Sornet Page 18 48

- certains services peuvent ecirctre fournis agrave distance sans contact direct avec le client et

distribueacutes par reacuteseau (tenue de comptabiliteacute affacturage gestion clientegravele centre drsquoappel

hellip)

Remarque les services repreacutesentent 75 de lrsquoactiviteacute eacuteconomique franccedilaise

23 ndash La nature de lrsquoorganisation

Les organisations publiques franccedilaises (administrations centrales collectiviteacutes territoriales

hocircpitaux hellip) repreacutesentent une part importante de lrsquoactiviteacute (environ 30 des emplois) La

fonction publique regroupe des organisations aux finaliteacutes diverses et qui ont des problegravemes

de gestion similaires agrave ceux des entreprises auxquelles elles peuvent emprunter des principes

de management Notamment

- pour controcircler les coucircts et assurer la qualiteacute des services

- pour communiquer avec les administreacutes ou les usagers

- pour motiver les personnels et geacuterer les ressources humaines

La transposition directe des techniques de gestion et de management nrsquoest cependant pas

toujours possible car

- la comptabiliteacute publique obeacuteit agrave des regravegles speacutecifiques (proceacutedure budgeacutetaire

notamment)

- le laquo client raquo ne paye pas toujours la prestation du moins directement

- la concurrence est parfois inexistante

- les grandes administrations centraliseacutees sont soumises agrave des choix politiques geacuteneacuteraux

parfois sans connexion eacutevidente avec les besoins opeacuterationnels

- le statut des personnels et les grilles de salaires limitent les possibiliteacutes de gestion des

ressources humaines

Remarque la LOLF (loi organique relative aux lois de finances) est entreacutee en vigueur en

2006 Elle alloue des moyens budgeacutetaires en fonction de programmes et remplace la

reconduction automatique de 90 des budgets Cette reacuteforme se heurte toutefois agrave la

lourdeur des grands ministegraveres ougrave la complexiteacute des activiteacutes est difficile agrave

appreacutehender et ougrave des inerties culturelles peuvent exister agrave tout niveau

Les associations loi de 1901 peuvent avoir une activiteacute comparable agrave celle de grandes

entreprises (voir par exemple les associations de santeacute ou professionnelles) et leur

management est alors similaire malgreacute lrsquoabsence de but lucratif (les beacuteneacutefices ne sont pas

distribuables) Elles ont drsquoailleurs en France un poids eacuteconomique important (elles emploient

environ 1 600 000 salarieacutes)

Cependant lrsquoadheacutesion agrave un systegraveme de valeurs fondateur de lrsquoassociation ou la limite de

lrsquoautoriteacute (quand un volant de beacuteneacutevoles important participe agrave lrsquoactiviteacute) peut introduire des

nuances

- le renforcement des objectifs socieacutetaux

- la faiblesse des relations hieacuterarchiques

- des contraintes de gestion du temps des beacuteneacutevoles

- des modaliteacutes particuliegraveres de recrutement et de motivation des dirigeants

24 ndash Les facteurs contingents

La theacuteorie de la contingence montre qursquoune structure drsquoentreprise nrsquoest efficace que dans

une situation deacutetermineacutee et qursquoil nrsquoexiste que des solutions de management construites dans

un contexte preacutecis

Le management doit ainsi srsquoadapter agrave des facteurs contingents qui ne peuvent ecirctre

controcircleacutes du moins agrave bregraveve eacutecheacuteance Ces facteurs sont par exemple

- lrsquoancienneteacute de lrsquoentreprise (plus elle est ancienne plus lrsquoentreprise a tendance agrave reacutepeacuteter

des comportements eacuteprouveacutes)

Initiation au management copy CRCF ndash J Sornet Page 19 48

- la taille de lrsquoentreprise (la grande entreprise a une composante administrative plus

deacuteveloppeacutee)

- le systegraveme de production (tregraves standardiseacute complexe automatiseacute hellip)

- lrsquoenvironnement

3 ndash Le management et les parties prenantes

Lrsquoentreprise a pour vocation premiegravere de mettre des produits agrave disposition de ses clients en

reacutealisant un profit Pour y arriver elle doit aussi satisfaire ses parties prenantes salarieacutes

actionnaires fournisseurs hellip

Est partie prenante agrave lrsquoentreprise laquo tout groupe ou individu qui peut ecirctre affecteacute ou est

affecteacute par les buts de lrsquoorganisation hellip raquo (Freeman ndash 1984)

Les parties prenantes attendent agrave des degreacutes divers de profiter drsquoune creacuteation de valeur en

provenance de lrsquoentreprise qui doit reacutepondre agrave ces attentes pour assurer sa peacuterenniteacute ou

favoriser son deacuteveloppement

On distingue les parties prenantes primaires ou principales qui sont essentielles agrave lrsquoentreprise

et qui ont geacuteneacuteralement une relation formelle avec elle (clients associeacutes et actionnaires

precircteurs salarieacutes fournisseurs collectiviteacutes) et les parties prenantes secondaires dont

lrsquoinfluence est diffuse (groupes de pression associations meacutedias instances europeacuteennes

agences de notation hellip)

Remarque la consideacuteration de lrsquoensemble des parties prenantes (laquo stakeholders raquo - les

deacutepositaires) fait contrepoids agrave lrsquoimportance accordeacutee aux seuls actionnaires

(laquo shareholders raquo)

Les organisations nrsquoayant pas drsquoobjectif de profit doivent aussi satisfaire leurs parties

prenantes apporter un service aux usagers dans les meilleures conditions eacuteconomiques

limiter un budget assurer la qualiteacute des relations avec les fournisseurs hellip

Dans cette optique le management doit organiser lrsquoaction de faccedilon agrave eacutequilibrer des forces

parfois divergentes

- le contexte fait pression sur lrsquoorganisation contrainte agrave optimiser ses reacutesultats

- lrsquoorganisation cherche par son action agrave assurer sa peacuterenniteacute son deacuteveloppement (en

reacutealisant des profits dans le cas de lrsquoentreprise) et agrave satisfaire ses parties prenantes

- le management agit en pilotant les actions pour contrebalancer la pression du contexte

Actions de

lrsquoorganisation

Management Contexte

Parties

prenantes

Initiation au management copy CRCF ndash J Sornet Page 20 48

APPLICATIONS MP

MP1 Deacutefinir contingent gestion budgeacutetaire

MP2 Deacuteterminer les parties prenantes drsquoun hocircpital public et leurs principales attentes

Mecircme question pour les organisations suivantes

- SNCF (entreprise publique)

- Peugeot

- MAIF (mutuelle drsquoassurance)

MP3 En les situant dans le cycle des activiteacutes du management trouver les actions agrave mener

dans les situations suivantes

- baisse de 10 des ventes dans une entreprise industrielle (produits meacutenagers le reacuteseau de

distribution vient drsquoecirctre reacuteorganiseacute)

- idem dans une entreprise de vente par correspondance soumise agrave la concurrence internet

(les ventes stagnaient depuis six mois malgreacute les efforts promotionnels)

- augmentation des deacutelais drsquoattente des consultations dans une clinique (lrsquohocircpital voisin a

fermeacute son service drsquourgences)

Initiation au management copy CRCF ndash J Sornet Page 21 48

ELEMENTS DE CORRIGE MP

MP1 Deacutefinir (dans le contexte drsquoune entreprise) contingent gestion budgeacutetaire

Contingent = imposeacute par lrsquoexteacuterieur Contingence = effet du hasard de la rencontre de

plusieurs eacuteveacutenements indeacutependants (variables explicatives que lrsquoon ne peut influencer)

Gestion budgeacutetaire = technique drsquoadministration des entreprises srsquoappuyant sur des

preacutevisions dont on deacuteduit apregraves accord des responsables des attributions de moyens sur une

dureacutee limiteacutee Une analyse reacuteguliegravere des eacutecarts entre preacutevisions et reacutealisations permet ensuite

le pilotage des activiteacutes Le budget est un cadre incitatif

La laquo planification budgeacutetaire raquo consiste agrave traduire en budgets une planification strateacutegique

avec systegraveme de reporting

MP2 Deacuteterminer les parties prenantes drsquoun hocircpital public et leurs principales attentes

Mecircme question pour les organisations suivantes

- SNCF (entreprise publique)

- Peugeot

- MAIF (mutuelle drsquoassurance)

Hocircpital

- patients (qualiteacute des soins)

- CNAM (baisse des coucircts)

- collectiviteacute locale (service aux administreacutes)

- eacutetat (ameacutenagement du territoire maicirctrise des budgets optimisation)

- employeacutes (salaire conditions de travail et satisfaction)

- fournisseurs ndash pharmacie autres (CA paiement reacutegulier)

- associations de patients (qualiteacute proximiteacute des soins)

SNCF

- usagers et associations drsquousagers (proximiteacute reacutegulariteacute prix du service)

- reacuteseau ferreacute de France (optimisation des lignes paiement adapteacute)

- fournisseurs (CA paiement reacutegulier)

- employeacutes (salaire conditions de travail seacutecuriteacute de lrsquoemploi)

- eacutetat (ameacutenagement du territoire)

- collectiviteacutes locales (service)

Peugeot

- clients (qualiteacute prix SAV relation commerciale)

- fournisseurs (CA reacutegulariteacute de lrsquoactiviteacute)

- employeacutes (salaire conditions de travail seacutecuriteacute de lrsquoemploi)

- eacutetat (taxes)

- collectiviteacute locale (emploi dynamisation eacuteconomique preacuteservation de lrsquoenvironnement)

- associations de protection de lrsquoenvironnement (activiteacute propre baisse des eacutemissions

nouvelles eacutenergies)

MAIF

- socieacutetaires (protection relation assureur tarif mesureacute)

- professionnels de lrsquoautomobile et autres (agreacutement marge de manœuvre reacuteparations tarifs

eacuteleveacutes)

- fournisseurs (CA paiement reacutegulier)

- eacutetat (taxes engagement pour la seacutecuriteacute)

- employeacutes (salaire conditions de travail seacutecuriteacute de lrsquoemploi)

Initiation au management copy CRCF ndash J Sornet Page 22 48

MP3 En les situant dans le cycle des activiteacutes du management trouver les actions agrave mener

dans les situations suivantes

- baisse de 10 des ventes dans une entreprise industrielle (produits meacutenagers le reacuteseau de

distribution vient drsquoecirctre reacuteorganiseacute)

Adapter le pilotage motiver cadrer si insuffisant retoucher une organisation deacutefectueuse

- idem dans une entreprise de vente par correspondance soumise agrave la concurrence internet

(les ventes stagnaient depuis six mois malgreacute les efforts promotionnels)

Voir pilotage et organisation si une eacutevolution du meacutetier a deacutejagrave eacuteteacute initialiseacutee Sinon re-

conception (adaptation au nouveau contexte) puis planification et reacuteorganisation

- augmentation des deacutelais drsquoattente des consultations dans une clinique (lrsquohocircpital voisin a

fermeacute son service drsquourgences)

Organisation Si insuffisant planification (nouveaux objectifs)

Initiation au management copy CRCF ndash J Sornet Page 23 48

ORGANISATION ET PROCESSUS

La performance de lrsquoentreprise deacutepend de son organisation et de son aptitude agrave produire

aux meilleures conditions Nous allons montrer comment organisation formelle et processus

de production peuvent contribuer agrave cette performance

1 ndash Vers lrsquooptimum

11 ndash Les eacuteconomies occidentales jusqursquoaux anneacutees 70

Jusqursquoen 1945 le principal problegraveme des entreprises eacutetait de produire des biens en quantiteacute

suffisante agrave un prix compatible avec le marcheacute Les grandes entreprises se sont multiplieacutees et

la standardisation a permis de reacuteduire les coucircts (exemple deacuteveloppement de Ford et de la

production agrave la chaicircne de 1908 agrave 1920 qui a permis une baisse du prix des voitures des 23)

On parle de laquo production pousseacutee vers le marcheacute raquo

Cette croissance de la production peu reacuteguleacutee a eacuteteacute marqueacutee par des surproductions en

1910 et 1920 puis par la crise de 1929 qui a prolongeacute ses effets jusqursquoagrave la guerre

De 1945 agrave 1975 environ (les laquo trente glorieuses raquo) la reconstruction la croissance de la

consommation de masse de nouvelles technologies et les eacutechanges internationaux

alimentent lrsquoeacuteconomie La standardisation srsquoeacutetend aux biens de consommation dont les

coucircts baissent fortement et de nouvelles reacutegulations sociales permettent une eacutevolution sans

heurt des revenus La saturation de certains marcheacutes conduit dans les anneacutees 60 agrave la

deacutemarche laquo marketing raquo et agrave la diffeacuterenciation des produits Le produit est laquo dirigeacute par le

marcheacute raquo mais les entreprises conservent une organisation assez classique et les plus grosses

srsquointernationalisent

12 ndash Lrsquoexpeacuterience japonaise et ses prolongements

Tregraves tocirct apregraves la guerre dans un Japon appauvri le constructeur automobile Toyota a ducirc

faire face agrave une restriction du marcheacute des moyens financiers et productifs et des

approvisionnements La firme a donc innoveacute dans un nouveau systegraveme de production

chassant les laquo gaspillages raquo (temps drsquoattente transports stocks deacutefauts hellip) consideacuterant que

seule la fabrication vendable creacutee de la valeur

Toyota srsquoorganise pour fabriquer la quantiteacute et la qualiteacute de produits juste neacutecessaires agrave la

satisfaction des clients la production est laquo tireacutee par le marcheacute raquo La mise en place de ce

systegraveme qui integravegre les fournisseurs ne sera acheveacutee que dans le milieu des anneacutees 70

En 1973 la hausse du peacutetrole inaugure un ralentissement de la croissance des eacuteconomies

occidentales La concurrence accrue provoque alors un inteacuterecirct pour le systegraveme deacuteveloppeacute

au Japon La production au plus juste se deacuteveloppe ainsi dans lrsquoindustrie automobile agrave partir

des anneacutees 80 et elle se reacutepand encore maintenant dans drsquoautres secteurs

Cette approche qui vise un objectif de zeacutero stock et zeacutero deacutefaut impose la maicirctrise de laquo bout

en bout raquo des processus de production et leur ameacutelioration

Initiation au management copy CRCF ndash J Sornet Page 24 48

2 ndash Organiser lrsquoentreprise

21 ndash Direction et organisation

Diriger une entreprise neacutecessite de lrsquoorganiser (de reacutepartir les tacircches) pour qursquoelle puisse

atteindre ses objectifs Lrsquoorganisation permet de satisfaire un marcheacute en tirant parti des

capaciteacutes actuelles de lrsquoentreprise tout en preacuteparant lrsquoavenir

Lrsquoorganisation reacutesulte freacutequemment drsquoun compromis entre des objectifs situeacutes agrave des niveaux

et des eacutecheacuteances diffeacuterents

Exemples

- le leader des chaises roulantes peut tirer profit de sa structure productive et de son

savoir faire pour entrer sur le marcheacute de la bicyclette eacutelectrique

- ecirctre parfaitement structureacute pour alimenter 90 du marcheacute des disquettes ne preacutepare

pas lrsquoavenir

- srsquoorganiser pour conqueacuterir le marcheacute des tire-bouchons eacutelectriques dans les deux ans

perd de son sens si cela altegravere les moyens neacutecessaires agrave la production drsquoappareils

manuels ancienne mais vitale dont la diminution agrave court terme risque de nuire agrave la

solvabiliteacute de lrsquoentreprise et de la conduire agrave la cessation de paiement

22 ndash Lrsquoorganisation fonctionnelle

La majoriteacute des entreprises adopte une laquo organisation fonctionnelle raquo (celle qui est visible

dans les organigrammes) ougrave des regroupements de personnels et drsquoeacutequipements se font

selon un modegravele hieacuterarchique (laquo line raquo) dans des uniteacutes des services ou des deacutepartements

speacutecialiseacutes Cette organisation peut se deacutecliner agrave lrsquointeacuterieur des divisions des grandes

entreprises quand elles scindent leur activiteacute par zone geacuteographique type drsquoactiviteacute

cateacutegorie de clients hellip

Remarque le terme laquo fonction raquo deacutesigne un rocircle particulier dans le fonctionnement de

lrsquoentreprise

Lrsquoorganisation fonctionnelle diffeacuterencie les activiteacutes de lrsquoentreprise en les regroupant par

meacutetier pour utiliser au mieux les compeacutetences et les moyens (meilleur rendement par la

speacutecialisation lrsquoeacutechange de compeacutetences dans une mecircme uniteacute ou gracircce agrave des eacuteconomies

drsquoeacutechelle)

23 ndash La notion de processus de production

Un processus de production se deacutefinit par la succession drsquoactiviteacutes permettant de satisfaire

un client en transformant des ressources (mateacuterielles financiegraveres humaines) en un produit

bien ou service Le processus doit creacuteer une valeur reconnue par le client

Un processus peut servir un client interne agrave lrsquoentreprise (par exemple en produisant un

composant intervenant dans plusieurs produits ou par la maintenance des machines) aussi

bien qursquoun client final On distingue usuellement

- les processus opeacuterationnels (ou maicirctres) aussi appeleacutes processus meacutetier (business process)

qui satisfont directement les clients finaux (conception et fabrication de produits vente hellip)

- les processus de support et de management (geacuterer les ressources humaines geacuterer

lrsquoinformation geacuterer les ressources financiegraveres hellip) qui ont les processus opeacuterationnels comme

clients

Toutes les actions internes agrave une organisation peuvent srsquointeacutegrer dans des processus qui

conditionnent directement ou indirectement la capaciteacute de lrsquoorganisation agrave satisfaire le

client final ou lrsquousager

Initiation au management copy CRCF ndash J Sornet Page 25 48

Aborder le fonctionnement de lrsquoentreprise par ses processus (approche processus) permet

de mettre en eacutevidence les chaicircnes drsquoactiviteacutes qui conduisent aux produits leurs

dysfonctionnements leurs coucircts la formation des deacutelais et la souplesse (la flexibiliteacute)

disponible pour satisfaire la clientegravele finale Lrsquoameacutelioration des processus a un impact visible

et direct sur chaque produit proposeacute aux clients

Lrsquoapproche processus provoque une eacutevolution de la faccedilon de travailler

- en faisant peacuteneacutetrer la laquo voix du client raquo au plus profond de lrsquoentreprise (et plus seulement

dans les services commerciaux et marketing)

- en mettant en eacutevidence des possibiliteacutes de rationalisation (par regroupement ou impartition

de certaines activiteacutes)

Remarque lrsquoapproche par les activiteacutes et les processus est agrave lrsquoorigine de la meacutethode

de deacutetermination des coucircts laquo ABC raquo - activity based costing

24 ndash Processus et fonctions

Le processus est transversal Il enchaicircne des activiteacutes qui traversent lrsquoentreprise en particulier

les services ou les deacutepartements drsquoune organisation fonctionnelle

Exemple

La division du travail par fonctions induit une charge de coordination pour assurer le

deacuteroulement du processus Elle peut geacuteneacuterer des attentes des erreurs ou des conflits drsquointeacuterecirct

(lrsquoobservation montre que des dysfonctionnements sont tregraves souvent constateacutes lors du

passage drsquoun service agrave un autre)

Organisation fonctionnelle et approche processus visent toutes deux un optimum

eacuteconomique mais leurs logiques sont diffeacuterentes

- le processus vise la satisfaction des clients (prix qualiteacute deacutelais service)

- le deacutecoupage fonctionnel cherche agrave optimiser les moyens (maximiser lrsquoeffet drsquoexpeacuterience

partager des infrastructures profiter de pocircles de compeacutetences hellip) Il apporte une ossature

hieacuterarchique stable souvent indispensable

Organisation fonctionnelle et approche processus sont donc compleacutementaires dans la

majoriteacute des cas et doivent ecirctre combineacutees judicieusement

APPLICATIONS OP

OP1 Deacutefinir flexibiliteacute systegraveme impartition

OP2 Citer huit exemples drsquoinformations essentielles pour optimiser un processus de

fabrication

Direction

Deacutepartement

commercial

(C)

Deacutepartement

administratif et

financier (AF)

Deacutepartement

Etudes (E)

Deacutepartement

Production (P)

Activiteacute

C-x Activiteacute

AF-x Activiteacute

E-x

Activiteacute

P-x

Processus x

Clie

nt

Initiation au management copy CRCF ndash J Sornet Page 26 48

OP3 Deacutegager les principes du toyotisme preacutesenteacute ci-dessous En quoi ce systegraveme est-il

initiateur de lrsquoapproche processus

Taiichi Ohno et le Toyotisme

1 - Extrait drsquoun article de Jacques BARRAUX - 1993 - LExpansion

Taiichi Ohno (1912 ndash 1990) hellip ne se prenait pas pour un visionnaire mais en imposant une

nouvelle faccedilon de produire il a reacuteinventeacute le management hellip tout le monde a entendu parler

des mots qui ont populariseacute le toyotisme dont il est le pegravere le juste-agrave-temps hellip Autant

doutils conccedilus pour lrsquoautomobile et qui ont aujourdhui une application universelle

hellip Taiichi Ohno jeune ingeacutenieur entre chez Toyota alors simple constructeur de machines

textiles Degraves 1926 apparaicirct la notion de jidoka hellip cest lart de transfeacuterer de lintelligence aux

machines pour mieux libeacuterer lintelligence des hommes Tout le contraire du taylorisme qui

juge la machine moins impreacutevisible que lhomme En 1933 Toyota se lance dans lautomobile

en sinspirant des meacutethodes ameacutericaines Mais en 1935 agrave loccasion dun voyage aux Etats-

Unis leacutetat-major de lentreprise revient fascineacute de sa visite dans un supermarcheacute La notion

de juste-agrave-temps va naicirctre de lobservation dune grande surface un lieu ougrave les clients ne

prennent que ce dont ils ont besoin et ougrave les rayons sont reacuteapprovisionneacutes pour compenser

les quantiteacutes preacuteleveacutees Ainsi le systegraveme Toyota est-il deacutejagrave dans la tecircte de ses dirigeants avant

mecircme la Seconde Guerre mondiale un demi-siegravecle avant la reacutevolution informatique et la

segmentation intensive des marcheacutes

hellip des esprits curieux comme Franccedilois Dalle en France tombent alors sous le charme des

formules et des paraboles de Taiichi Ohno En voici deux eacutechantillons

Penser agrave lenvers Cela signifie combattre les ideacutees reccedilues En lespegravece il sagit du fordisme et

du taylorisme Ohno ne croit pas agrave la planification aux effets deacutechelle et dexpeacuterience Il

propose un systegraveme industriel agrave lenvers qui permette de diversifier les produits et de les

fabriquer en petites quantiteacutes Nous ne devons plus ecirctre des paysans qui accumulent des

stocks mais des chasseurs On nimpose pas loffre On traque la demande et on la gegravere en

continu

Que les valleacutees soient hautes et les montagnes peu eacuteleveacutees Plutocirct que de concentrer tous

les efforts sur une production agrave un moment donneacute mieux vaut se doter de structures flexibles

permettant de passer agrave tout instant dune seacuterie agrave une autre Il faut eacuteviter les ruptures et les

secousses aplanir les cycles entretenir des flux reacuteguliers dactiviteacutes diversifieacutees Ce qui

implique de ne pas enfermer les hommes et les eacutequipements dans des speacutecialisations trop

eacutetroites

La flexibiliteacute le travail en groupe le refus de la dictature des machines la polyvalence et

surtout lattention constante aux signaux eacutemis par le marcheacute nappartiennent plus au

toyotisme Ces notions sont les fondements du nouvel art dorganiser de vendre et de

produire dans lindustrie comme dans les services hellip

2 - Quelques notions cleacutes

Taiichi Ohno a imagineacute la meacutethode des laquo cinq pourquoi raquo qui consiste agrave se poser cinq fois de

suite la question laquo pourquoi raquo sur le mecircme sujet de faccedilon agrave deacutecouvrir la veacuteritable cause

drsquoun problegraveme Cette meacutethode peut ecirctre appliqueacutee agrave tous les niveaux et permettre

notamment aux agents de fabrication de proposer de veacuteritables ameacuteliorations de la

production

La recherche de la qualiteacute totale (pas de deacutefaut des produits pas de rebuts pas de deacutefaut

des processus) accompagne la deacutemarche de Toyota La qualiteacute a un coucirct compenseacute par

des ventes accrues par lrsquoeacuteconomie des mesures palliatives aux deacutefauts

Initiation au management copy CRCF ndash J Sornet Page 27 48

Fiche OP1 ndash Benchmarking et processus

Le laquo benchmarking raquo consiste agrave comparer le fonctionnement de plusieurs systegravemes pour en

faire notamment ressortir les meilleures pratiques (laquo best practices raquo) Cette technique est

utiliseacutee depuis les anneacutees 80 pour ameacuteliorer la performance des entreprises Elle impose agrave

lrsquoentreprise drsquoeacutevaluer et de remettre en question ses propres modes de fonctionnement afin

de les faire eacutevoluer agrave la lueur de ce qui se fait ailleurs

Le benchmarking permet drsquoameacuteliorer les processus agrave moindre risque en fixant des objectifs

baseacutes sur des faits et donc plus facilement accepteacutes

Une classification des processus en tant que base de reacuteflexion a eacuteteacute eacutetablie aux USA par

lrsquolaquo International Benchmarking Clearinghouse raquo de lrsquoAPQC (american productivity and

quality center) en collaboration avec plusieurs dizaines drsquoentreprises

Elle se reacutesume ainsi

Le terme laquo reengineering raquo (la re-conception ou laquo reacuteingeacutenieacuterie raquo) des processus deacutesigne un

projet drsquoameacutelioration radicale des performances (de 20 agrave 50 ou plus) Il neacutecessite une

parfaite adheacutesion de la direction la constitution drsquoune petite eacutequipe de projet brillante

connaissant parfaitement les activiteacutes de lrsquoentreprise et il peut inclure un benchmarking

Le reengineering provoque geacuteneacuteralement la reacuteduction du nombre de niveaux hieacuterarchiques

(laquo delayering raquo) et lrsquoaccroissement du pouvoir de deacutecision des employeacutes (laquo empowerment raquo

ou laquo empouvoirement raquo) Bien qursquoy conduisant parfois il ne doit pas ecirctre confondu avec la

reacuteduction des activiteacutes (laquo downsizing raquo ou restructuration) et lrsquoexternalisation (laquo outsourcing raquo)

Pro

ce

ssu

s

op

eacutera

tio

nn

els

Pro

ce

ssu

s d

e m

an

ag

em

en

t e

t d

e

sup

po

rt

1 ndash

Comprendre

le marcheacute et

les clients (besoins

satisfaction)

2 ndash

Deacutevelopper

vision et

strateacutegie (contexte

concurrence)

3 ndash

Creacuteer

produits

services

processus

(concevoir

ameacuteliorer)

4 ndash

Marketing et

vente

5 ndash

Produire et

livrer (industrie

dont

ameacutelioration

processus)

6 ndash

Produire et

livrer (services)

7 ndash

Facturer et

servir les

clients (apregraves-

vente

reacuteclamations)

8 ndash Deacutevelopper et geacuterer les ressources humaines

9 ndash Geacuterer les systegravemes drsquoinformation

10 ndash Geacuterer les ressources financiegraveres et les actifs

11 ndash Appliquer un programme environnemental

12 ndash Geacuterer les relations exteacuterieures (actionnaires banques lois relations publiques hellip)

13 ndash Geacuterer lrsquoameacutelioration et le changement (eacutevaluer mesurer motiver qualiteacute totale)

Initiation au management copy CRCF ndash J Sornet Page 28 48

Fiche OP2 ndash Lrsquoorganisation par processus

Lrsquoeacutevolution drsquoune organisation aux activiteacutes reacutepeacutetitives vers lrsquoapproche processus est

geacuteneacuteralement progressive et se met en place par paliers

La mise en œuvre drsquoun veacuteritable management par processus doit ecirctre preacuteceacutedeacutee quand

lrsquoactiviteacute de lrsquoentreprise est complexe drsquoun recensement (une laquo cartographie des

processus raquo) pour mettre en eacutevidence les processus ou les familles de processus cleacutes critiques

pour le succegraves de lrsquoentreprise ougrave les efforts seront prioritaires

Des responsables de processus (laquo process owners raquo) sont ensuite deacutesigneacutes

Le responsable doit concevoir ses processus puis apregraves leur mise en œuvre assurer les

coordinations neacutecessaires les ameacuteliorer et les repreacutesenter aupregraves de la direction

Quand une structure par processus est mise en place des opeacuterateurs exeacutecutants

preacuteceacutedemment regroupeacutes dans les fonctions peuvent ecirctre affecteacutes aux processus et

drsquoanciens responsables de fonctions peuvent devenir des experts au service des processus

Lrsquoorganisation par processus peut imposer un degreacute eacuteleveacute drsquointeacutegration des activiteacutes donc

une polyvalence accrue des personnels et une reacuteduction des niveaux hieacuterarchiques

Elle neacutecessite pour le moins des compeacutetences eacutelargies au niveau des responsables de

processus (organisation administration technique hellip) dont le nombre doit rester limiteacute

(quelques dizaines au plus)

Sauf dans de tregraves petites structures lrsquoorganisation par processus se plaque geacuteneacuteralement sur

une structure plus classique

Initiation au management copy CRCF ndash J Sornet Page 29 48

ELEMENTS DE CORRIGE OP

OP1 Deacutefinir

Flexibiliteacute = adaptation au besoin (horaire variable chaicircnes robotiseacutees)

Systegraveme = ensemble organiseacute dans un but boicircte noire (sanguin nerveux meacutetrique laquo D raquo)

Impartition = sous-traitance ou externalisation (seacuteparation) drsquoactiviteacutes faire appel agrave des

partenaires plutocirct que faire soi-mecircme

OP2 Citer huit exemples drsquoinformations essentielles pour orienter lrsquooptimisation drsquoun processus

Montant des stocks (approvisionnements et produits finis)

Temps drsquoattente

Taux drsquoactiviteacute des ateliers

Rebuts

Deacutelai de production

Taux de reacuteclamations clients (qualiteacute)

Temps passeacutes en retouches finales

Turn over

Nombre drsquoarrecircts maladie

Accidents du travail

Dureacutee des arrecircts machines

OP3 Deacutegager les principes du toyotisme preacutesenteacute dans la fiche 31 En quoi ce systegraveme

repose trsquoil sur lrsquoapproche processus

Produire la quantiteacute juste neacutecessaire (agrave la demande) donc eacuteviter les stocks

Flexibiliteacute intelligence des chaicircnes de production

Qualiteacute (eacuteviter le coucirct de la non-qualiteacute)

La notion de processus est implicite ainsi que la chaicircne de valeur client

Initiation au management copy CRCF ndash J Sornet Page 30 48

DEFIS ET TENDANCES DU MANAGEMENT

Les meacutethodes de management se deacuteveloppent pour affronter le contexte eacuteconomique

Ce chapitre preacutesente les deacutefis auxquels le management contemporain doit faire face

1 ndash Lrsquoeacutevolution eacuteconomique contemporaine

A mesure que lrsquoactiviteacute eacuteconomique mondiale srsquoaccroicirct que la technologie eacutevolue les

changements sont de plus en plus rapides Ils introduisent des situations ineacutedites auxquelles les

entreprises doivent srsquoadapter en cherchant de nouvelles solutions de management Les trois

derniegraveres deacutecennies ont eacuteteacute notamment marqueacutees par les pheacutenomegravenes suivants (que nous

listons sans tenir compte des liens pouvant exister entre eux)

Pheacutenomegravene Traduction Effets

Deacute reacuteglementation

globalisation

financiegravere

titrisation

Libre circulation des capitaux accegraves

facile des particuliers au marcheacute

boursier (directement ou par

lrsquointermeacutediaire des OPCVM et SICAV)

Monteacutee en puissance du financement

des entreprises sur le marcheacute boursier

Fonds de pension

(retraites) et fonds

souverains (eacutetats)

Poids boursier important drsquoinvestisseurs

institutionnels qui cherchent un haut

rendement financier (dividendes ou

valorisation boursiegravere)

Pression sur les grandes entreprises

influence sur les strateacutegies

Mondialisation Liberteacute des eacutechanges internationaux Accroissement de la concurrence

recherche drsquoavantages eacuteconomiques

par la deacutelocalisation (biens et

services) la concentration des efforts

(recentrage) problegravemes drsquoemploi

multiplication des transports perte

drsquoinfluence des politiques

Baisse de lrsquoemploi

occidental

(notamment

industriel)

Moins de fabrications fabrications

automatiseacutees recours aux moyens

informatiques

Activiteacute reporteacutee sur le commerce la

conception et les services chocircmage

charge sociale

Restructurations Optimisation des entreprises

abaissement des coucircts augmentation

des marges recherche drsquoune taille

critique (eacuteconomies drsquoeacutechelle poids

sur le marcheacute)

Recentrages externalisations fusions

deacutelocalisations constitution de grands

groupes

NTIC (nouvelles

technologies de

lrsquoinformation et de

la communication)

Mise en œuvre des reacuteseaux (dont

internet) et drsquoapplications

informatiques communicantes

Nouvelles formes de commerce

marcheacute international deacutelocalisation

du travail intellectuel reacuteorganisation

de la distribution

Rareacutefaction relative

des matiegraveres

premiegraveres

Recherche de substituts exploration

miniegravere coucircts drsquoexploitation des

gisements accrus

Augmentation des coucircts variations

erratiques du cours des matiegraveres

deacutestabilisations politiques

Evolution

geacuteopolitique et

eacuteconomique

mondiale

Chute de lrsquoURSS transformation des

eacuteconomies collectivistes pays

eacutemergents (Chine Inde Breacutesil Russie)

()

Accroissement de la population

mondiale (4 agrave 6 7 milliards de 1970 agrave

2008)

Libeacuteralisme sans frein () nouvelles

puissances eacuteconomiques

opportuniteacutes de deacuteveloppement

nouveau partage des ressources

ineacutegaliteacutes baisse du soutien aux PVD

laquo Terrorisme raquo Actions armeacutees pression de groupes

armeacutes non gouvernementaux

Deacutestabilisations reacutegionales charge

des deacutepenses militaires

Deacuteveloppement

durable

Recherche drsquoune croissance eacutequitable

et respectueuse de lrsquoenvironnement

Pression sur les entreprises (eacutetats

associations de consommateurs

eacutecologistes ONG)

() Reacutecemment quelques affaires (Enron laquo subprimes raquo Vivendi Universal Socieacuteteacute

Geacuteneacuterale Airbus par exemple) et agrave plus grande eacutechelle la crise financiegravere de 2008 ont

montreacute les dangers drsquoune libeacuteralisation sans controcircles suffisants

Initiation au management copy CRCF ndash J Sornet Page 31 48

() Des alliances eacuteconomiques naissent entre pays eacutemergents (notamment en

ameacuterique centrale creacuteation de la Banque du Sud en 2008 par exemple) et lrsquoon

commence agrave imaginer une baisse progressive de lrsquoinfluence eacuteconomique des Etats

Unis

2 ndash Les deacutefis actuels du management

21 ndash Les grandes orientations

Lrsquoeacutevolution eacuteconomique suggegravere quelques pistes parfois concurrentes pour lrsquoaction du

manager contemporain On y retrouve au premier plan la construction drsquoune vision qui est

une composante commune du leadership

Objectif du manager

pour lrsquoentreprise

Justification Facteurs de reacuteussite

Construire une vision Eclairer lrsquoavenir de lrsquoentreprise partager

un but souder motiver

Effort de reacuteelle prospection

volontarisme de la direction

bonne communication

Reacuteactiviteacute et flexibiliteacute

(sous tous les aspects

agrave tous niveaux)

Srsquoadapter rapidement au marcheacute Bonne organisation des processus

personnel compeacutetent autonome

et motiveacute structure hieacuterarchique

alleacutegeacutee robotisation

Deacutegager des profits Reacutemuneacuterer les apporteurs de capitaux

srsquoautofinancer

Ajuster coucircts et structures

Exploiter les nouvelles

technologies

Reacuteactiviteacute ajuster coucircts et deacutelais

reacutepondre au marcheacute suivre les clients

Organiser le SI de faccedilon

pertinente eacuteviter le coucirct excessif

drsquoinvestissements trop en

laquo pointe raquo (laquo essuyer les placirctres raquo)

utiliser judicieusement les services

exteacuterieurs

Bacirctir des alliances

(contrats fusions)

Deacutevelopper une activiteacute limiter les coucircts

de transaction () atteindre la taille

critique et de meilleurs rendements se

recentrer sur une activiteacute profitable

Dominer les processus se donner

une identiteacute lisible externaliser se

doter drsquoune capaciteacute financiegravere

suffisante

Valoriser lrsquoimage Attirer les clients favoriser les alliances

donner confiance (apporteurs de fonds

employeacutes clients partenaires socieacuteteacute

civile)

Instaurer des regravegles de

gouvernance inteacutegrer le

deacuteveloppement durable

respecter lrsquoenvironnement

Geacuterer les risques Faire face aux aleacuteas eacuteconomiques et

technologiques (conjoncture politiques

accidents malveillance)

Creacuteer un systegraveme drsquoalerte geacuterer

la crise (reacuteaction raisonneacutee

sceacutenarios poursuite de

lrsquoexploitation dans un contexte

instable) mise en place de

proceacutedures drsquoapprentissage pour

ameacuteliorer les reacuteactions au fil du

temps

Geacuterer le changement Faire face agrave lrsquoeacutevolution de la demande

la pression sur les prix la variation des

performances financiegraveres la

concurrence la globalisation des

marcheacutes lrsquoeacutevolution technologique aux

fusions ou alliances aux changements

de reacuteglementation de direction hellip ()

Bonne communication pour

donner du sens au changement

et obtenir lrsquoadheacutesion du personnel

Rassembler et geacuterer les

connaissances former le

personnel

Innover Garder un avantage concurrentiel se

diffeacuterencier

Veille technologique et

commerciale investissement

Ouverture

internationale

Elargir le marcheacute saisir les opportuniteacutes Veille commerciale partenariats

() La theacuteorie des coucircts de transaction deacuteveloppeacutee par OE Williamson dans les

anneacutees 70 integravegre les coucircts lieacutes au recours au marcheacute (recherche et choix drsquoun

fournisseur neacutegociation reacutedaction de contrat suivi des eacutechanges risque de rupture

Initiation au management copy CRCF ndash J Sornet Page 32 48

drsquoapprovisionnement hellip) On peut en conclure que lrsquointeacutegration de diffeacuterentes

activiteacutes agrave lrsquoentreprise (la laquo firme raquo) preacutesente des avantages Mais des coucircts de

transaction internes doivent aussi ecirctre consideacutereacutes (preacuteparation organisation

surveillance hellip) et certaines formes de coopeacuteration continue avec les fournisseurs

permettent de reacuteduire le coucirct des transactions externes

() drsquoapregraves laquo Les meilleures pratiques de management raquo - Brilman Heacuterard ndash EO

Une eacutetude du Conference Board (2002) liste les deacutefis du management vus par 700 leaders

mondiaux Soit en reacutesumeacute avec indication du score correspondant

1 ndash Fideacuteliser les clients (42)

2 ndash Reacuteduire les coucircts (38)

3 ndash Accroicirctre flexibiliteacute et reacuteactiviteacute (29)

4 ndash Amener les employeacutes agrave adheacuterer aux valeurs et visions de lrsquoentreprise (26)

5 ndash Deacutevelopper et retenir les leaders (25)

6 ndash Geacuterer acquisitions et alliances (24)

7 ndash Accroicirctre lrsquoinnovation (20)

En fin de classement citoyenneteacute et reacuteputation (4) et ameacutelioration de la diversiteacute (3)

22 ndash Les techniques disponibles

Pour faire face aux deacutefis le manager dispose de nouveaux concepts et de nouvelles

techniques Le tableau ci-dessous en donne un reacutesumeacute et indique les domaines qursquoils

influencent principalement

Initiation au management copy CRCF ndash J Sornet Page 33 48

Principaux concepts techniques outils Incidence principale sur

Internet

- e-commerce (commerce eacutelectronique site

entreprise)

- CRM ou GRC (gestion de la relation client)

- e-procurement (gestion des approvisionnements

par le reacuteseau)

- messagerie eacutelectronique

- e-recrutement

Vente accegraves au marcheacute

Relation client reacuteactiviteacute personnalisation

fideacutelisation

Deacutelais coucircts

Communication transfert de donneacutees (piegraveces

jointes) tous domaines

Communication recrutement

Intranet reacuteseau drsquoentreprise SI

- knowledge management (gestion des

connaissances)

- e-learning (apprentissage en ligne)

- plateforme de travail collaboratif (groupware)

- workflow (circulation eacutelectronique de

documents enchaicircnement de processus)

- e-RH portail RH (libre accegraves aux postes agrave

pourvoir informations candidatures hellip)

- PGI (progiciel de gestion inteacutegreacute) ou ERP

Innovation capaciteacute au changement veille

documentaire

Formation du personnel accompagnement des

changements

Coordination communication interne

Coordination

Communication interne (voire internet en

externe) reacuteduction des coucircts climat drsquoentreprise

recrutement plans de carriegraveres hellip

Coucircts fiabiliteacute du systegraveme drsquoinformation deacutelais

processus (continuiteacute inteacutegration)

Logistique inteacutegreacutee

Supply Chain Management (SCM) gestion de la

logistique (incluant les approvisionnements)

Processus deacutelais coucircts

Externalisation

Valorisation du capital humain

GPEC (gestion preacutevisionnelle des emplois et

compeacutetences)

Coaching

Reacuteactiviteacute de lrsquoentreprise conservation des

compeacutetences rendements individuels turn-over

adaptation des compeacutetences motivation

Efficaciteacute individuelle controcircle reacutegulation

progregraves processus

Approche processus

Optimisation des processus

Deacutemarche qualiteacute totale (TQM ndash total quality

management)

Empowerment (empouvoirement)

Benchmarking reacuteingeacutenieacuterie

Coucircts marges qualiteacute deacutelais flexibiliteacute

externalisation eacutelargissement des compeacutetences

organisation

Ameacutelioration des processus (meacutetiers et supports)

Autonomie compeacutetences des employeacutes

Ameacutelioration des processus restructuration

Management par la valeur

Parties prenantes

Satisfaction des parties prenantes financement

motivation collaborations hellip

Collaboration inter organisations

Reacuteseaux drsquoentreprises alliances

EDI (eacutechange de donneacutees informatiseacutees) extranet

Impartition externalisation (outsourcing)

Coucircts recentrage investissements lancement

drsquoactiviteacute

Coucircts reacuteactiviteacute deacutelais relations avec

lrsquoadministration

Coucircts recentrage limitation des investissements

Ethique drsquoentreprise

Gouvernance drsquoentreprise (mode de direction

encadreacute par des regravegles)

Rocircle socieacutetal deacuteveloppement durable

environnement

Image de lrsquoentreprise reacutegulation du top

management relations actionnaires

Image peacutenaliteacutes et amendes objectifs

strateacutegiques

Initiation au management copy CRCF ndash J Sornet Page 34 48

23 ndash Le rocircle socieacutetal des entreprises

La responsabiliteacute socieacutetale de lrsquoentreprise (RSE) deacutesigne le rocircle qursquoelle prend dans la socieacuteteacute

au-delagrave de son activiteacute purement geacuteneacuteratrice de profit On parle aussi drsquoentreprise citoyenne

La RSE est indissociable du deacuteveloppement durable de porteacutee mondiale et dont les trois

piliers sont

- eacuteconomique (favoriser le deacuteveloppement les eacutechanges internationaux)

- social (accegraves aux soins eacuteducation conditions de travail hellip)

- environnemental (pollution preacuteservation des ressources hellip)

La RSE integravegre notamment une preacuteoccupation sociale de lrsquoentreprise vis-agrave-vis de ses salarieacutes

(seacutecuriteacute et santeacute au travail juste reacutemuneacuteration deacuteveloppement personnel hellip) Elle conduit agrave

tenir compte dans le management drsquoune vision exteacuterieure agrave lrsquoentreprise qui peut avoir des

reacutepercussions possibles sur son activiteacute eacuteconomique

Lrsquoentreprise peut aussi tirer avantage drsquoune deacutemarche responsable par la baisse de certains

coucircts (plus faibles consommations drsquoeacutenergies reacuteduction des transports hellip)

Le rocircle socieacutetal de lrsquoentreprise a eacuteteacute reconnu en France par la loi laquo NRE raquo de 2001 (loi sur les

nouvelles reacutegulations eacuteconomiques) qui oblige les socieacuteteacutes franccedilaise coteacutees sur un marcheacute

reacuteglementeacute agrave rendre compte dans leur rapport annuel de leur gestion sociale et

environnementale au travers de leur activiteacute

Article 116 de la loi Le rapport viseacute agrave larticle L 225-102 rend compte hellip laquo Il comprend

eacutegalement des informations dont la liste est fixeacutee par deacutecret en Conseil dEtat sur la

maniegravere dont la socieacuteteacute prend en compte les conseacutequences sociales et

environnementales de son activiteacute Le preacutesent alineacutea ne sapplique pas aux socieacuteteacutes

dont les titres ne sont pas admis aux neacutegociations sur un marcheacute reacuteglementeacute raquo

Une norme ISO 14000 integravegre ces preacuteoccupations et des taxes eacutecologiques sont

progressivement creacutees

3 ndash Le management par la valeur

31 ndash De lrsquoanalyse au management par la valeur

Lrsquoanalyse de la valeur est neacutee en 1947 aux Etats-Unis (General Electrics) Cette technique

consiste agrave eacutelaborer des produits conformes aux attentes de la clientegravele mais sans excegraves pour

trouver un bon compromis entre valeur pour le client et coucirct Le produit optimal est deacutefini agrave

partir drsquoenquecirctes qui deacuteterminent le besoin client (ou plutocirct drsquoun client laquo type raquo)

Exemple il est inutile de concevoir un petit veacutehicule citadin capable de parcourir

500 000 km sans avarie compte tenu des effets de mode et du faible kilomeacutetrage

annuel Par contre le marcheacute peut exiger un fonctionnement sans faille sur 150 000 km

soit dix ans en moyenne ce qui conditionne les coucircts de production

Cette recherche drsquoun ajustement de valeur au besoin des clients eacutetait un preacutecurseur du

management par la valeur qui recherche plus largement la creacuteation de valeur pour

chacune des parties prenantes de lrsquoentreprise tout en lui meacutenageant un reacutesultat suffisant

Plus geacuteneacuteralement le management par la valeur est deacutefini par une norme europeacuteenne (EN

12973)

Le management par la valeur est un style de management particuliegraverement destineacute agrave

mobiliser les individus agrave deacutevelopper les compeacutetences et agrave promouvoir les synergies et

Initiation au management copy CRCF ndash J Sornet Page 35 48

linnovation avec pour objectif la maximisation de la performance globale dun

organisme Le management par la valeur apporte une nouvelle faccedilon dutiliser nombre

de meacutethodes de management existantes Il est en coheacuterence avec le Management

de la qualiteacute

Cette approche du management pose de nombreuses questions notamment quelles

prioriteacutes et quelles valeurs attribuer aux parties prenantes comment appreacutehender la

perception par les parties prenantes de la valeur qui leur est affecteacutee

32 ndash La valeur client

Le processus drsquoeacutelaboration drsquoun produit qui consomme des ressources coucircteuses doit creacuteer

une valeur suffisante pour provoquer lrsquoachat par le client final La production drsquoune valeur

reconnue par le client est vitale pour lrsquoentreprise mais sa deacutetermination est parfois complexe

La valeur du produit perccedilue par le client integravegre des eacuteleacutements en partie subjectifs

- une valeur drsquousage (le produit reacutepond agrave un besoin)

- une valeur drsquoestime (lrsquoimage apporteacutee par le produit un aspect affectif)

- une valeur drsquoeacutechange (deacuteduite de lrsquoespoir de revente du produit)

Valeurs drsquousage drsquoestime et drsquoeacutechange deacutependent implicitement de la qualiteacute (un bien peu

fiable est impropre agrave lrsquousage attendu de mauvaise qualiteacute notoire il nrsquoapporte pas une

image positive et ses deacutefauts connus nuisent agrave sa revente) Une eacutevaluation de la qualiteacute

intervient donc dans la valeur perccedilue du produit

Par ailleurs le client considegravere le coucirct drsquoobtention du produit (les charges qursquoil doit supporter

pour acqueacuterir le produit lrsquoeffort qursquoil doit faire pour trouver le produit et les frais de mise agrave

disposition)

Le prix perccedilu par le client est geacuteneacuteralement supeacuterieur au prix de vente

Le client achegravete theacuteoriquement le produit qui preacutesente la diffeacuterence valeur perccedilue ndash prix

perccedilu la plus favorable ou le meilleur rapport prix perccedilu qualiteacute perccedilue et dans certains

cas celui qui a le prix produit le plus bas

Remarque les valeurs du scheacutema ci-dessus changent durant le cycle de vie du produit

(un nouveau produit peut avoir une valeur perccedilue plus eacuteleveacutee qursquoen fin de vie) La

valeur client ne peut ecirctre eacutevalueacutee que par enquecirctes et ne peut donc ecirctre deacutefinie avec

certitude

La notion de laquo satisfaction client raquo conseacutecutive agrave une vente influence aussi le prix produit et

le prix perccedilu

- lrsquoentreprise gagne sur les coucircts de recherche de clientegravele

- le client nrsquoa pas agrave rechercher un nouveau fournisseur et beacuteneacuteficie drsquoun coucirct drsquoobtention

plus bas

valeur perccedilue client

prix perccedilu client

coucirct produit Marge (valeur creacuteeacutee pour

lrsquoentreprise)

euros

prix produit

Valeur creacuteeacutee

pour le client

Initiation au management copy CRCF ndash J Sornet Page 36 48

La satisfaction du client deacutepend de facteurs qualitatifs aussi divers que la fiabiliteacute du produit

la vitesse de reacuteaction du fournisseur lrsquoattitude des commerciaux lrsquoefficaciteacute du service

apregraves-vente la netteteacute des contrats ou la justesse de la facture

Valeur perccedilue coucirct marge et satisfaction reacutesultent de processus allant de la conception du

produit jusqursquoagrave sa livraison et son apregraves-vente La deacutemarche laquo processus raquo et lrsquolaquo analyse de la

valeur raquo en forccedilant la recherche de solutions efficientes agrave tout niveau administratif

technique commercial et apregraves-vente sont donc neacutecessaires pour bien positionner

lrsquoentreprise sur son marcheacute

Pour autant le risque commercial ne peut jamais ecirctre annuleacute et lrsquooffre de lrsquoentreprise ne

satisfait geacuteneacuteralement pas en milieu concurrentiel tous ses clients potentiels

33 - La creacuteation de valeur pour les autres parties prenantes

Les salarieacutes

La creacuteation drsquoune valeur suffisante pour les salarieacutes est reconnue comme neacutecessaire car des

observations montrent que la satisfaction des clients en deacutepend Moins souvent eacutevoqueacutee en

peacuteriode de chocircmage elle nrsquoest prioritaire que pour les employeacutes dont lrsquoentreprise souhaite

conserver les compeacutetences

La laquo valeur salarieacute raquo ne comprend pas que le salaire Le sentiment drsquoappartenance agrave un

groupe la reconnaissance lrsquoaccomplissement de soi et la construction professionnelle en

sont des eacuteleacutements importants Comme pour les clients on doit ainsi distinguer la reacutetribution

perccedilue du salaire objectif

Les actionnaires

Lrsquoactionnaire apporte des fonds propres agrave lrsquoentreprise en contrepartie de titres parfois

neacutegociables en bourse et assortis drsquoun droit de vote en assembleacutee geacuteneacuterale La valeur

attribueacutee aux actionnaires est servie en termes moneacutetaires (dividende ou augmentation de la

valeur du titre neacutegociable)

Remarque des facteurs non moneacutetaires comme lrsquoimage de lrsquoentreprise qui deacutepend

en partie de sa communication peuvent influencer la deacutecision drsquoachat de vente ou

de conservation des titres par lrsquoactionnaire

Reacutetribution perccedilue euros

Salaire objectif

Avantage non

moneacutetaire de

lrsquoemploi

Initiation au management copy CRCF ndash J Sornet Page 37 48

Compte tenu de lrsquoimportance croissante de lrsquoactionnariat dans le financement des grandes

entreprises coteacutees en bourse et notamment des investisseurs institutionnels comme les fonds

de pension des indicateurs speacutecifiques ont eacuteteacute introduits pour appreacutecier la performance des

entreprises vue par les actionnaires Par exemple la valeur ajouteacutee eacuteconomique (EVA reg

economic value added marque deacuteposeacutee de Stern Stewart ou VAE ndash valeur ajouteacutee

eacuteconomique parfois deacutenommeacutee VEC ndash valeur eacuteconomique creacuteeacutee) qui prend en compte le

coucirct du capital

LrsquoEVA correspond tregraves scheacutematiquement au calcul suivant

EVA = (PO) profit opeacuterationnel ndash (C) coucirct du capital X (CE) capitaux employeacutes

LrsquoEVA neacutecessite en pratique des retraitements assez complexes Le PO peut se deacuteterminer

selon les principes suivants

- PO = reacutesultat drsquoexploitation (avant inteacuterecircts) ndash impocirct

- PO = beacuteneacutefice courant (tenant compte des inteacuterecircts) + inteacuterecircts ndash eacuteconomie drsquoimpocirct sur les

inteacuterecircts (on exclue les eacuteleacutements financiers et lrsquoimpocirct correspondant) ndash impocirct

- lrsquoimpocirct pris en compte correspond au profit opeacuterationnel consideacutereacute (dans les cas courants agrave

13 du PO)

C = taux moyen de reacutemuneacuteration du capital (reacutesultant par exemple du dividende exigeacute de

certains investisseurs et des taux drsquoemprunts bancaires)

CE = capitaux propres et dettes portant inteacuterecirct

Remarque le profit opeacuterationnel ou reacutesultat opeacuterationnel correspond au NOPAT ndash net

operating profit after tax - anglo-saxon LrsquoEVA est eacutegale au NOPAT diminueacute de la

reacutemuneacuteration des capitaux

Exemple lrsquoentreprise X dispose drsquoun capital de 2 500 000 euro et reacutealise un beacuteneacutefice net

drsquoimpocirct de 450 000 euro (taux 33 13) Un dividende de 6 doit ecirctre verseacute aux

actionnaires et la banque lui a accordeacute un precirct de 1 200 000 euro agrave 4 Les autres

constituants des reacutesultats financier et exceptionnel sont neacutegligeables

Reacutesultat opeacuterationnel = 450 000 + 004 x 1 200 000 x 23 = 482 000 euro

Coucirct du capital = 006 x 2 500 000 + 004 x 1 200 000 x 23 = 182 000 euro

EVA = 300 000 euro

Coucirct moyen pondeacutereacute du capital (C) = (004 x 1 200 000 x23 + 006 x 2 500 000)

3 700 000 Soit 492

Si lrsquoEVA est positive lrsquoentreprise creacuteeacutee de la valeur apregraves reacutemuneacuteration des capitaux et sa

valeur boursiegravere doit augmenter

Lrsquoutilisation de lrsquoEVA comme indicateur influence le management de lrsquoentreprise car il y a

trois moyens pratiques drsquoaugmenter lrsquoEVA

- augmenter le reacutesultat opeacuterationnel

- lancer des investissements ayant une rentabiliteacute supeacuterieure agrave C

- eacuteliminer les activiteacutes ayant une rentabiliteacute infeacuterieure agrave C

Remarque lrsquoutilisation sans nuance de lrsquoEVA comme critegravere de management peut

poser problegraveme Le calcul de lrsquoEVA repose sur des ajustements comptables il est donc

sujet agrave manipulations (provisions capitalisation ou non de la RD hellip) Par ailleurs le

critegravere laquo EVA raquo pris isoleacutement peut conduire agrave chercher la rentabiliteacute agrave court terme agrave

reacuteduire les investissements prospectifs et donc nuire agrave terme au deacuteveloppement de

lrsquoentreprise

Initiation au management copy CRCF ndash J Sornet Page 38 48

Les fournisseurs reccediloivent le paiement de leurs factures plus ou moins rapidement (le deacutelai

de paiement repreacutesente une valeur consentie au fournisseur)

Lrsquoentreprise peut accroicirctre la valeur apporteacutee agrave ses fournisseurs par des actions cibleacutees

comme une contribution agrave la formation de leurs personnels certains transferts de

technologie ou de savoir faire agrave des sous-traitants une coopeacuteration suivie favorisant leur

deacuteveloppement lrsquointeacutegration agrave des campagnes de promotion

A noter que la valeur consentie aux fournisseurs peut avoir une influence sur la qualiteacute et les

deacutelais de livraison des produits

La collectiviteacute reccediloit des taxes et parfois des prestations en nature par deacutefaut ou explicites

(effort de preacuteservation de lrsquoenvironnement ameacutenagement du territoire par les implantations

aide mateacuterielle agrave des projets participation agrave la formation par exemple)

APPLICATIONS DT

DT1 Deacutefinir expliquer deacutereacuteglementation socieacutetal eacuteconomies drsquoeacutechelle coaching EDI

gouvernance

DT2 Deacuteterminer en quoi la deacutemarche TQM srsquoinscrit dans les deacutefis actuels du management

DT3 Apregraves avoir consulteacute les documents ci-dessous extraits du site drsquoAir France

(httpdeveloppement-

durableairfrancecomFRfrlocaldemarcheN4_positionnement_pphtm)

exposer les enjeux et les limites de la RSE et de la gestion des parties prenantes

Initiation au management copy CRCF ndash J Sornet Page 39 48

Dialogue avec les parties prenantes

Initiation au management copy CRCF ndash J Sornet Page 40 48

Attentes des parties prenantes

Initiation au management copy CRCF ndash J Sornet Page 41 48

Creacuteation de valeur pour les parties prenantes

La creacuteation de valeur pour les parties prenantes est au cœur de la strateacutegie du Groupe Le scheacutema de

distribution financiegravere ci-dessous donne un aperccedilu de la distribution des recettes du Groupe aux

diffeacuterentes parties prenantes actionnaires collaborateurs fournisseurs pouvoirs publics

collectiviteacutes locales etc

Initiation au management copy CRCF ndash J Sornet Page 42 48

Fiche DT1 ndash Extrait du sommaire de laquo Problegravemes eacuteconomiques raquo No 2894

La gestion des entreprises bouleverseacutee par les technologies de linternet

Reacutealiteacutes industrielles - Annales des Mines Jean-Michel Yolin

Avec lavegravenement de linternet les processus de conception de production et de vente sont

radicalement remis en cause Quel que soit le secteur dactiviteacute les technologies de linternet

permettent en effet de reacuteduire les deacutelais et de passer dun processus discontinu agrave un processus

continu Lorganisation des entreprises et leur mode de gestion en sont profondeacutement bouleverseacutes

tant au niveau individuel que collectif Linternet rend ainsi possible la reacutealisation dobjectifs que les

entreprises cherchaient agrave atteindre depuis longtemps sans y parvenir meilleure eacutecoute du client

travail sans stocks en flux tendu hieacuterarchies plates autorisant une grande reacuteactiviteacute flexibiliteacute dans

lorganisation et loutil de production acceacuteleacuteration du renouvellement des produits entreprises en

reacuteseau ougrave chacune se recentre sur son cœur de meacutetier etc

Le laquo knowledge management raquo ou comment geacuterer les connaissances

Document de travail du LAMSADE - Michel Grundstein

Peter Drucker lavait preacutedit le capital immateacuteriel eacutetait voueacute agrave devenir un facteur de compeacutetitiviteacute

pour lentreprise La libeacuteralisation des eacutechanges acceacutelegravere les processus de deacutecision de lentreprise

et implique que lassimilation des informations soit agrave la fois de meilleure qualiteacute et plus rapide Ainsi

la fonction qui consiste agrave manager les connaissances au sein de lentreprise savegravere primordiale

Bien que la prise de conscience de limportance du capital immateacuteriel ait eacuteteacute tardive - le concept

de knowledge management est apparu en France aux Etats-Unis et au Japon au milieu des

anneacutees 1990 - agrave lheure actuelle lorganisation de leacutechange dinformations et le partage des

connaissances sont devenus des facteurs cleacutes dune gestion performante de lentreprise Ils

doivent sinscrire dans un projet global destineacute agrave mettre en valeur les savoirs et les savoir-faire

individuels et collectifs

Les leccedilons du laquo coaching raquo pour le management de la qualiteacute

Humanisme et Entreprise - Martine Brasseur

Parmi les nouvelles formes de management en vogue dans les entreprises le coaching figure en

bonne place Appliqueacute au management de la qualiteacute il sagit dune pratique

daccompagnement destineacutee agrave initier et agrave faciliter le processus de deacuteveloppement dun individu

La deacutemarche consiste agrave affirmer que tout individu est en quecircte de qualiteacute agrave condition toutefois

de ne pas lui imposer des contraintes lempecircchant de progresser On considegravere notamment les

erreurs comme potentiellement feacutecondes En deacutefinitive le coach donne au coacheacute la permission

de reacuteussir en lui donnant aussi la permission deacutechouer

Initiation au management copy CRCF ndash J Sornet Page 43 48

Fiche DT2 ndash Management strateacutegique les sept deacutefis agrave relever dici agrave 2016

Extrait drsquoun article du site wwwlentreprisecom -Sabine Blanc - Mis en ligne le 20032007

(httpwwwlentreprisecom325article11977html)

Une eacutetude anglaise publieacutee par lopeacuterateur Orange Grande-Bretagne deacutecrypte la mutation

des formes de travail et les enjeux majeurs pour les entreprises de demain afin decirctre au top

de la compeacutetitiviteacute Voici les challenges-cleacutes pour les managers qui veulent rester dans la

course hellip

1 - Future organisation du travail les quatre laquo mondes raquo possibles

La reacutealiteacute sera probablement un meacutelange de ces quatre sceacutenarios souligne lrsquoeacutetude

Les mondes mutuels Tout se passe dans le cadre des communauteacutes locales vie priveacutee

comme professionnelle Le modegravele coopeacuteratif preacutevaut au lieu du laquo big business raquo Oublieacutes

aussi dans ce systegraveme les trajets pour aller au bureau les gens preacutefegravereront travailler dans de

petites entreprises locales souvent connecteacutees au reacuteseau drsquoautres structures similaires

Les laquo reacutepondants raquo (en anglais laquo replicants raquo) La figure du consultant freelance deviendra

dominante tandis que celle du salarieacute deacuteclinera Il ne sera pas rare de travailler pour plusieurs

entreprises On perdra en seacutecuriteacute de lrsquoemploi en visibiliteacute et en routine ce que lrsquoon gagnera

en liberteacute La majeure partie des tacircches srsquoeffectuera chez soi avec la possibiliteacute de srsquoinstaller

temporairement dans les bureaux de son client du moment Dans un contexte dincertitude

sur lrsquoavenir les travailleurs alterneront peacuteriodes drsquoactiviteacute intense et repos Ce sera agrave eux

drsquoaller vers les entreprises et non lrsquoinverse mecircme si celles-ci devront veiller agrave rester attractives

Les cottages eacutelectroniques Comme ce nom le suggegravere le teacuteleacutetravail deviendrait la norme

univers priveacute et professionnel se confondant Plus besoin de subir une heure de transport les

salarieacutes se logueront de chez eux sur le reacuteseau de lrsquoentreprise Les reacuteunions se tiendront dans

de petits bureaux centraux situeacutes agrave courte distance La flexibiliteacute du temps de travail srsquoimpose

Les salarieacutes disposeront de plus de marge de liberteacute dans leur activiteacute

Les disciples de la nueacutee Cette appellation poeacutetique cache simplement une extension de

lrsquoorganisation actuelle des grandes entreprises avec des salarieacutes se rendant sur un lieu de

travail centraliseacute Le rocircle croissant des technologies de lrsquoinformation multipliera les faccedilons de

collaborer et accroicirctra lrsquoefficaciteacute Le controcircle du travail sera omnipreacutesent La frontiegravere entre

travail et vie priveacutee restera marqueacutee

2 - Sept deacutefis pour les entreprises et leur managers

Quoi qursquoil advienne les entreprises et leurs dirigeants devront concentrer leurs efforts sur sept

points-cleacutes pour srsquoadapter Voici quelques exemples de probleacutematiques souleveacutees par le

rapport et des pistes de solution

Le leadership Les managers devront entre autres savoir persuader et influencer des

travailleurs beaucoup plus indeacutependants Ils auront aussi agrave repenser les niveaux auxquels

prendre les deacutecisions strateacutegiques en haut ou au contraire agrave des degreacutes moins eacuteleveacutes de la

pyramide hieacuterarchique

gt Faire du management une force facilitant les activiteacutes transversales plutocirct que la reacuteduire agrave

la seule fonction de deacutecision

La culture drsquoentreprise Davantage de salarieacutes capables de reacutefleacutechir seront neacutecessaires

tandis que les tacircches qui peuvent ecirctre automatiseacutees ou scripteacutees diminueront Un des

enjeux creacuteer une culture agrave mecircme drsquoattirer et drsquoencourager les personnes preacutesentant ces

qualiteacutes de reacuteflexion requises dans un contexte de compeacutetition accrue et de plus grande

indeacutependance des travailleurs

Initiation au management copy CRCF ndash J Sornet Page 44 48

gt Passer si neacutecessaire drsquoune culture drsquoentreprise forte agrave un mode drsquoengagement plus

consensuel moins rebutant

La marque Conseacutequence du recours croissant agrave lrsquo laquo outsourcing raquo lrsquoimage drsquoune marque

deacutependra plus drsquoagents exteacuterieurs qui ne fonctionnent pas forceacutement selon le mecircme mode

drsquoorganisation Comment garder le controcircle dessus

gt Choisir le mode qui corresponde le plus agrave vos valeurs et preacutevoir un programme de risk

management qui mette en eacutevidence ougrave les conflits sont susceptibles de jaillir

Lrsquoinnovation Plus que jamais il faudra faire face agrave une acceacuteleacuteration du rythme de

lrsquoinnovation en proposant constamment des solutions adapteacutees

gt Tisser des partenariats strateacutegiques avec drsquoautres entreprises pour partager les coucircts et les

fruits de lrsquoinnovation

Le deacutefi opeacuterationnel et technologique De quelle faccedilon controcircler lrsquoinformation crsquoest-agrave-dire

faire en sorte que les bonnes personnes accegravedent facilement agrave une information toujours en

phase tout en maintenant la seacutecuriteacute

gt Recourir agrave des laquo feuilles de route des futurs raquo syntheacutetisant en une page les indicateurs

sociaux et de consommation ainsi que les eacutevolutions technologiques et leacutegislatives qui

influent sur les changements et indiquant comment ils modifient vos marcheacutes vos clients et

votre organisation

La qualiteacute Si de nouveaux proceacutedeacutes ont pu deacutegrader la qualiteacute comme le recours agrave des

centres drsquoappel externaliseacutes drsquoautres ideacutees se sont reacuteveacuteleacutees plus prometteuses comme en

teacutemoigne le succegraves de certaines compagnies aeacuteriennes low cost Elles ont su conjuguer prix

serreacutes et services eacuteleveacutes ce qui devra devenir la norme estime lrsquoeacutetude

gt Continuer de rechercher la qualiteacute Elaborez aussi une bonne prestation service qui inclut

une livraison de qualiteacute voire creacuteez-la en partenariat avec les consommateurs

La leacutegislation La question de la proprieacuteteacute intellectuelle pourrait ecirctre probleacutematique Elle est

deacutejagrave source de conflits comme en teacutemoigne le procegraves pour violation de brevet intenteacute agrave RIM

le fabricant canadien du Blackberry par NTP Que pourra-t-on et que faudra-t-il proteacuteger par

un brevet Il sera eacutegalement neacutecessaire drsquoadapter la leacutegislation aux nouveaux modes

drsquoorganisation

gt Collaborer avec les acteurs du mecircme secteur et les leacutegislateurs pour deacutevelopper les

modegraveles des lieux de travail du futur et bacirctir le droit le plus adeacutequat

Orange a-t-il vu juste dans ses preacutevisions Rendez-vous dans neuf ans pour la reacuteponsehellip

Initiation au management copy CRCF ndash J Sornet Page 45 48

Fiche DT3 ndash Le management par la qualiteacute totale

Extrait drsquoune lettre drsquoinformation du cabinet Baud Accordance Consulting AD2 consultants ndash

2002

1 - Le TQM (Total Quality Management) offre pour lentreprise une vision de la qualiteacute plus

large et transversale

Son principe est simple La finaliteacute de lEntreprise est de deacutevelopper la satisfaction de ses

clients tout en eacutetant beacuteneacuteficiaire cest agrave dire pas agrave nimporte quel prix Elle doit ameacuteliorer sa

rentabiliteacute au travers de la deacutemarche qualiteacute La Qualiteacute Totale vise agrave fournir aux clients

externes et internes une reacuteponse adeacutequate agrave leurs attentes dans le meilleur rapport qualiteacute

prix la meilleure efficience

Elle considegravere pour cela lensemble des processus de lentreprise ayant une incidence sur la

qualiteacute et la satisfaction des clients

Le TQM fait ainsi une large place agrave

la deacutefinition et la planification de la strateacutegie geacuteneacuterale

la coheacuterence de la politique qualiteacute avec la strateacutegie

la deacutemultiplication de la politique qualiteacute dans toutes les directions de lentreprise

la relation client fournisseur interne

la prise en compte de lenvironnement concurrentiel

la consideacuteration de lensemble des risques potentiels financiers sociaux concurrentielshellip

limplication et la motivation du personnel

lanalyse des besoins des clients et le positionnement marketing

la maicirctrise des processus transverses internes

les reacutesultats sous tous ses aspects y compris financiers commerciaux image

De nombreux reacutefeacuterentiels sont relatifs agrave la Qualiteacute Totale hellip Tous ces reacutefeacuterentiels imposent un

questionnement plus profond et indiscret sur le mode de fonctionnement de lentreprise et

son management

helliphellip

2 - LISO 9001 2000 au travers du deacuteploiement des processus (management supports

reacutealisation et ameacutelioration continue) reacutepond quelque peu agrave la mecircme logique

LISO est une ouverture indeacuteniable vers la logique du TQM mais ne se reacutefegravere pas agrave la notion

defficience

Les dirigeants sont cependant sensibles agrave la neacutecessaire reacuteduction des coucircts de non-qualiteacute

et dobtention de la qualiteacute agrave la rentabiliteacute du systegraveme de management de la qualiteacute

mais ne perccediloivent pas toujours la qualiteacute comme une deacutemarche globale

Les deacutemarches qualiteacute commencent bien souvent par la remise en cause de lorganisation

leacutevaluation critique de son efficaciteacute lexamen des processus et la mise en eacutevidence des

lourdeurs administratives

La qualiteacute devient laffaire de tous hellip

Initiation au management copy CRCF ndash J Sornet Page 46 48

Fiche DT4 ndash Le deacuteveloppement durable et la RSE

Extrait du site wwwvigeocom

(httpwwwvigeocomcsr-rating-agencyfrmethodologiecriteres-de-recherche37-

criteres-d-analysehtml)

Deacuteveloppement durable laquo un deacuteveloppement qui reacutepond aux besoins du preacutesent sans compromettre

la capaciteacute des geacuteneacuterations futures de reacutepondre aux leurs raquo (Commission mondiale sur lrsquoenvironnement

et le deacuteveloppement ndash 1987)

Reacutefeacuterentiel drsquoeacutevaluation des entreprises par le groupe Vigeacuteo (le groupe mesure les performances et le

niveau de maicirctrise des risques de responsabiliteacute sociale des entreprises et des organisations - site

wwwvigeocom)

1 Ressources Humaines Ameacutelioration continue des relations professionnelles des relations drsquoemploi et des conditions de travail 2 Droits humains sur les lieux de travail Respect de la liberteacute syndicale et promotion de la neacutegociation collective non discrimination et promotion de lrsquoeacutegaliteacute eacutelimination des formes de travail proscrites (enfants travail forceacute) preacutevention des traitements inhumains ou deacutegradants de type harcegravelements sexuels protection de la vie priveacutee et des donneacutees personnelles 3 Environnement Protection sauvegarde preacutevention des atteintes agrave lenvironnement mise en place drsquoune strateacutegie manageacuteriale approprieacutee eacuteco conception protection de la biodiversiteacute et maicirctrise rationnelle des impacts environnementaux sur lrsquoensemble du cycle de vie des produits ou services

4 Comportements sur les marcheacutes Prise en compte des droits et inteacuterecircts des clients inteacutegration de standards sociaux et environnementaux dans la seacutelection des fournisseurs et sur lrsquoensemble de la chaicircne drsquoapprovisionnement preacutevention effective de la corruption respect des regravegles concurrentielles 5 Gouvernement drsquoentreprise Efficience et probiteacute assurance de lrsquoindeacutependance et de lrsquoefficaciteacute du Conseil drsquoadministration effectiviteacute et efficience des meacutecanismes drsquoaudit et de controcircle et notamment inclusion des risques de responsabiliteacute sociale respect des droits des actionnaires et notamment des minoritaires transparence et rationaliteacute de la reacutemuneacuteration des dirigeants 6 Engagement socieacutetal Effectiviteacute inteacutegration manageacuteriale de lrsquoengagement contribution au deacuteveloppement eacuteconomique et social des territoires drsquoimplantation et de leurs communauteacutes humaines engagements concrets en faveur de la maicirctrise des impacts socieacutetaux des produits et des services contribution transparente et participative agrave des causes drsquointeacuterecirct geacuteneacuteral

Initiation au management copy CRCF ndash J Sornet Page 47 48

ELEMENTS DE CORRIGE DT DT1 Deacutefinir expliquer

Deacutereacuteglementation = suppression des contraintes eacuteconomiques (libre eacutechange des biens et

capitaux)

Socieacutetal = qui se rapporte agrave la structure agrave lrsquoorganisation ou au fonctionnement de la socieacuteteacute

Economies drsquoeacutechelle = reacuteduction des coucircts lieacutee au niveau drsquoactiviteacute (amortissement des

charges fixes)

Coaching = accompagnement de personnes ou deacutequipes pour le deacuteveloppement de leurs

potentiels

EDI = eacutechange de donneacutees informatiseacutees ET standardiseacutees (ex SWIFT bancaire edifact

documents deacuteclaratifs)

Gouvernance = exercice du pouvoir la bonne gouvernance est participative et eacutequitable

conforme agrave lrsquointeacuterecirct commun

DT2 Deacuteterminer en quoi la deacutemarche TQM srsquoinscrit dans les deacutefis actuels du management

Voir notamment fiche 43

Maicirctrise des processus reacuteduction des coucircts reacuteactiviteacute et satisfaction de la clientegravele = faire

face agrave la concurrence

Ameacutelioration de lrsquoimage motivation du personnel

DT3 Apregraves avoir consulteacute les documents ci-dessous extraits du site drsquoAir France

(httpdeveloppement-

durableairfrancecomFRfrlocaldemarcheN4_positionnement_pphtm)

exposer les enjeux et les limites de la RSE et de la gestion des parties prenantes

Trame geacuteneacuterale possible

Introduction

Les deacutefis contemporains (accroissement de la concurrence devenue mondiale recherche

de nouveaux avantages concurrentiels pression de la socieacuteteacute besoin drsquoimage et de projet

lisible pour mener lrsquoentreprise crise et scandales du libeacuteralisme hellip) RSE et PP

Deacuteveloppement (voir cours)

1 ndash Parties prenantes et management par la valeur

PP deacutefinir citer reacutesumer lrsquoavantage rechercheacute (fideacuteliser motiver recherche drsquoalliances

implicites)

PP moyens (dont exemples AF) et meacutethode de management par la valeur (reacutepartie)

2 ndash La responsabiliteacute socieacutetale de lrsquoentreprise

RSE 3 axes

- eacuteconomique (favoriser le deacuteveloppement les eacutechanges internationaux)

- social (accegraves aux soins eacuteducation conditions de travail hellip)

- environnemental (pollution preacuteservation des ressources hellip)

RSE gouvernance drsquoentreprise facteur drsquoimage inteacutegrable dans la deacutemarche PP

Article 116 de la loi Le rapport viseacute agrave larticle L 225-102 rend compte hellip laquo Il comprend

eacutegalement des informations dont la liste est fixeacutee par deacutecret en Conseil dEtat sur la maniegravere

dont la socieacuteteacute prend en compte les conseacutequences sociales et environnementales de son

activiteacute Le preacutesent alineacutea ne sapplique pas aux socieacuteteacutes dont les titres ne sont pas admis aux

neacutegociations sur un marcheacute reacuteglementeacute raquo

Initiation au management copy CRCF ndash J Sornet Page 48 48

RSE exemple AF (ONG fournisseurs)

3 ndash Liens entre PP et RSE

- la RSE introduit de nouvelles PP

- la RSE suppose le respect des PP usuelles (employeacutes clients notamment)

4 - Probleacutematique

- deacutefinir la valeur reacuteellement apporteacutee par une gestion des PP (confusion salaire ndash valeur

idem impocircts hellip ex laquo valeur ajouteacutee raquo)

- communication (neacutecessaire mais aller au-delagrave)

- marginaliteacute des deacutepenses RSE (efficaciteacute sinceacuteriteacute de lrsquoengagement marge de manœuvre)

- charge RSE reporteacutee sur des tiers (ex fournisseurs AF)

- inteacutegration de facteurs non visibles en comptabiliteacute (pertes drsquoemploi nuisances hellip)

Conclusion

Voies incontournables mais pouvant nrsquoavoir qursquoun effet superficiel et temporaire Voir utiliteacute

drsquoaccompagnement leacutegislatif de regravegles de gouvernance

Initiation au management copy CRCF ndash J Sornet Page 5 48

24 ndash La recherche de performance

Le management recherche la performance de lrsquoorganisation en rapport avec ses objectifs

qui peuvent ecirctre de diffeacuterentes natures (expansion profit hellip ou agrave un niveau plus deacutetailleacute

reacuteduction des deacutelais flexibiliteacute reacuteduction des coucircts accroissement de la qualiteacute hellip)

La performance peut ecirctre abordeacutee de deux faccedilons

- par lrsquoefficaciteacute qui conduit agrave la reacutealisation des objectifs

- par lrsquoefficience qui conduit agrave une utilisation optimale des moyens disponibles avec le

meilleur rendement

Exemple une entreprise qui atteint son objectif de croissance de 10 du chiffre

drsquoaffaires et de son beacuteneacutefice est efficace mais celle qui arrive au mecircme reacutesultat avec

moins drsquoactifs et moins de personnel est plus efficiente

25 ndash Le champ drsquoaction du management

Le management est mis en œuvre dans des peacuterimegravetres drsquoampleur variable un groupe une

entreprise une activiteacute particuliegravere de lrsquoentreprise un projet la tenue drsquoun magasin hellip Les

actions de management ont ainsi des conseacutequences plus ougrave moins importantes

Le management srsquoexerce agrave tous les niveaux drsquoencadrement de lrsquoorganisation Il est de

coutume de distinguer trois niveaux

Exemple

Management strateacutegique la direction geacuteneacuterale a fixeacute les objectifs de chaque filiale du

groupe en tenant compte des positions souhaiteacutees sur le marcheacute Il en reacutesulte que les

uniteacutes X et Y aux activiteacutes similaires et compleacutementaires doivent preacuteparer leur fusion

preacutevue dans les deux ans La filiale Z situeacutee dans une ville universitaire doit acqueacuterir des

compeacutetences pour innover dans lrsquoapplication des supra conducteurs drsquoici trois ans Les

budgets et les financements correspondants ont eacuteteacute globalement estimeacutes

Management intermeacutediaire en application de ces orientations strateacutegiques les

directions de X et Y planifient des reacuteunions de travail communes pour voir comment

harmoniser leurs ressources humaines dans les douze mois Z deacutecide de contractualiser

ses relations avec le centre universitaire pour atteindre ses objectifs et mettre en place

un partenariat en recherche appliqueacutee

Management strateacutegique ou geacuteneacuteral

(direction laquo top management raquo deacutefinition

des objectifs geacuteneacuteraux)

Management tactique

(intermeacutediaire laquo middle

managers raquo)

Management opeacuterationnel (de

terrain ou drsquouniteacute local

laquo executive manager raquo)

Initiation au management copy CRCF ndash J Sornet Page 6 48

Management opeacuterationnel X et Y mettent en place leur communication avec le

personnel et un plan de reconversion Z nomme un directeur de recherche qui prend

notamment en charge les relations avec lrsquouniversiteacute

Le management recouvre la totaliteacute des actes de conduite de lrsquoorganisation dans tous les

domaines (technique commercial financier hellip) mais il ne correspond agrave aucune cellule de

lrsquoorganigramme

Les actions de management sont par ailleurs contraintes par la disponibiliteacute des ressources

neacutecessaire pour atteindre les objectifs (financement savoir-faire profil du personnel

eacutequipements hellip)

3 ndash La meacutethode laquo management raquo

31 ndash La science du management

Le management nrsquoest pas une science exacte il srsquoapparente agrave une science humaine

expeacuterimentale qui traite de pheacutenomegravenes socio-eacuteconomiques eacutevolutifs et qui doit trouver

concregravetement son application dans la vie des organisations

Cette science traite notamment de lrsquoorganisation des entreprises et rassemble des meacutethodes

et des theacuteories qui peuvent ecirctre regroupeacutees en eacutecoles ou en courants

Les theacuteories marquent geacuteneacuteralement une eacutepoque et elles peuvent se recouper partiellement

parfois srsquoopposer avec des nuances qui doivent ecirctre bien identifieacutees Elles srsquoaccompagnent

souvent de modegraveles et de scheacutematisations qui en facilitent la compreacutehension et la

transposition agrave de nouvelles situations

Ces outils scientifiques guident le raisonnement permettent drsquoappreacutehender des reacutealiteacutes

complexes et structurent les connaissances ils sont peacutedagogiques et constituent des aides

pour lrsquoaction Leur application doit cependant ecirctre raisonneacutee car

- un modegravele nrsquoest qursquoune simplification de la reacutealiteacute

- une theacuteorie sortie de son contexte historique et eacuteconomique peut perdre de sa pertinence

Sa mise en œuvre doit tenir compte de la situation reacuteelle et une theacuteorie ne peut agrave elle seule

justifier une deacutecision de management (ce drsquoautant plus que chaque theacuteorie ne couvre que

tregraves partiellement le domaine du management ou de lrsquoorganisation)

- il est souvent difficile de disposer agrave temps drsquoinformations fiables et suffisantes pour appliquer

une theacuteorie dans les conditions ideacuteales Le manager est freacutequemment ameneacute agrave prendre ses

deacutecisions sur la base drsquoinformations incomplegravetes ou incertaines et il doit alors en mesurer les

risques et preacutevoir les ajustements neacutecessaires

ENTREPRISE

Management Administration

Gestion

Strateacutegique

Opeacuterationnel

Initiation au management copy CRCF ndash J Sornet Page 7 48

- des modes influencent le management Elles peuvent indiquer une veacuteritable eacutevolution

eacuteconomique mais aussi ecirctre sans lendemain voire introduire un danger ou un coucirct inutile

(se meacutefier des speacutecialistes dont le fond de commerce est la vente de nouvelles techniques

de management et des seacuteminaires associeacutes)

- seule la creacuteation drsquoune combinaison originale (dans le respect des regravegles) peut apporter un

avantage agrave lrsquoentreprise et non la reproduction de choix de management connus de tous

Exemple 1 la matrice de portefeuille drsquoactiviteacutes permet de classer les activiteacutes

strateacutegiques drsquoune entreprise en fonction de leur taux de croissance et de la part de

marcheacute deacutetenue (matrice laquo BCG raquo - Boston consulting group ndash 1975) Cet outil de

management neacutecessite de disposer drsquoinformations fiables concernant le marcheacute Il

conduit souvent agrave simplifier les conditions de concurrence et ignore la

compleacutementariteacute pouvant exister entre activiteacutes (synergie partage de techniciteacute

amortissement de charges fixes hellip) La matrice est un moyen de prendre conscience

du portefeuille et de ses eacuteventuels deacutefauts (portefeuille deacuteseacutequilibreacute avec

preacutedominance anormale drsquoune zone) mais il ne permet pas seul de deacutecider de lrsquoavenir

des activiteacutes

Exemple 2 le lancement drsquoun investissement lourd (lrsquoimplantation de nouvelles usines

le lancement drsquoune nouvelle activiteacute agrave fort taux de recherche ndash deacuteveloppement hellip) ne

peut ecirctre deacutecideacute que par le recoupement de diffeacuterentes approches (financiegravere

commerciale strateacutegique) Aucune theacuteorie du management appliqueacutee isoleacutement et

sans preacutecaution ne peut justifier un tel investissement

Exemple 3 une entreprise produisant des eacutequipements meacutedicaux deacutecide de renforcer

son offre commerciale par un service de conseil et drsquoassistance Cette deacutecision est

inspireacutee par le concept de creacuteation de valeur par une volonteacute de se diffeacuterencier de la

concurrence et par des pratiques existant depuis longtemps dans le domaine

informatique

Remarque parmi les facteurs influenccedilant les principes de management il est utile de

savoir deacutetecter pour srsquoen preacutemunir les ideacuteologies preacutejugeacutes et autres laquo valeurs raquo sans

rapport certain avec lrsquoefficaciteacute eacuteconomique

32 ndash Theacuteorie et pratique du management

La litteacuterature (ouvrages revues) aborde le management sous des angles diffeacuterents qui

peuvent suggeacuterer une concurrence entre des approches theacuteorique et pratique ou

psychologique et eacuteconomique de cette discipline

Taux de croissance

du domaine (cf

cycle de vie du

produit)

Part de marcheacute

(compareacutee au

principal

concurrent)

fort

faible

forte faible

Dilemmes (activiteacutes

en phase de

lancement

potentiel + coucircts+

risque+)

Vedettes

(activiteacutes en forte

croissance

autofinancement+)

Vaches agrave lait

(activiteacutes

stabiliseacutees et

concurrence faible

ou stable

rentabiliteacute+)

Poids morts

(activiteacutes en

deacuteclin)

Initiation au management copy CRCF ndash J Sornet Page 8 48

Le management a cependant une viseacutee unique et concregravete la bonne marche des

organisations et les travaux des universitaires rejoignent lrsquoaction des praticiens (les

universitaires reacutealisent drsquoailleurs bien souvent leurs recherches au sein des organisations)

Au plan peacutedagogique les deux visions du management se complegravetent

- la vision acadeacutemique met en perspective des concepts et des theacuteories Elle permet de

srsquoapproprier des raisonnements essentiels et stables notamment concernant la strateacutegie et

les reacuteactions humaines qui sont le moteur des organisations

- la vision pratique relate le veacutecu des praticiens preacutesente des techniques concregravetes et tente

parfois une vulgarisation favorisant la diffusion des concepts Elle integravegre de faccedilon

pragmatique les objectifs eacuteconomiques et les contraintes de fonctionnement des

organisations en se placcedilant parfois dans une vision agrave court terme

Sur le terrain le management reacutealise une synthegravese entre des techniques de psychologie

sociale et des techniques de gestion de diverses origines et de nouvelles meacutethodes

apparaissent aussi sous le label unique laquo management raquo

Le terme manageacuterial (approche manageacuteriale theacuteorie manageacuteriale pratique manageacuteriale

hellip) fait reacutefeacuterence agrave la vision aux preacuteoccupations et aux actions des managers qui doivent

emmener leur organisation vers la reacutealisation de ses objectifs

Exemples

- la deacutetermination des coucircts et des marges reacutesulte de techniques de gestion

indispensable aux deacutecisions des managers

- le laquo CRM raquo (customer relationship management en franccedilais GRC ndash gestion de la

relation client) est un concept reacutecent qui srsquoaccompagne de techniques lieacutee aux plus

reacutecents deacuteveloppements du management et des systegravemes drsquoinformation

- le laquo coaching raquo est une technique de management permettant une eacutevolution

personnelle dans le sens des objectifs de lrsquoentreprise

- le recentrage sur le laquo meacutetier raquo repose sur des techniques de management

Les techniques sont rassurantes mais pas suffisantes et si les theacuteories ne sont pas

indispensables au praticien elles aident agrave comprendre agrave anticiper et agrave bien utiliser les

techniques

La qualiteacute du management reacuteside beaucoup dans la capaciteacute agrave appliquer concepts et

techniques de faccedilon pertinente et agrave innover Cette capaciteacute relegraveve en partie drsquoun laquo art du

management raquo qui srsquoacquiert en grande partie par la pratique

4 ndash Management et expertise comptable

Lrsquoexpert comptable doit manager ses propres eacutequipes Il est par ailleurs supposeacute laquo hellip

conseiller et accompagner le chef drsquoentreprise dans toutes ses deacutecisions hellip raquo (selon lrsquoOEC)

Ce rocircle est particuliegraverement important dans ses relations avec les petites entreprises Il doit

donc avoir une capaciteacute au management

Organisation

Vision acadeacutemique Vision pratique

Initiation au management copy CRCF ndash J Sornet Page 9 48

Lrsquointervention de lrsquoexpert dans le management drsquoune entreprise peut toutefois poser

quelques problegravemes

- Il peut y avoir conflits drsquointeacuterecirct entre activiteacutes de certification des comptes et de conseil en

management (la tendance est agrave la seacuteparation des activiteacutes dans les plus grands cabinets)

- lrsquoactiviteacute de conseil neacutecessite des compeacutetences parfois tregraves speacutecifiques (conseil fiscal

conseil en RH conseil en systegravemes drsquoinformation hellip)

- le conseil est une activiteacute diffeacuterente par sa forme de lrsquoexpertise comptable (interventions

longues peu reacutepeacutetitives peu codifieacutees mises en concurrence) qui neacutecessite une

organisation particuliegravere du cabinet lorsqursquoelle deacutepasse lrsquointervention occasionnelle

APPLICATIONS IM

IM1 Analyser la profession de laquo manager raquo selon Henry Mintzberg (texte extrait de lrsquoouvrage

laquo Le management raquo Eyrolles - Editions drsquoOrganisation) et les principes du management de la

norme ISO

Faire ressortir les eacuteleacutements speacutecifiques agrave chacune de ces approches et mettre en eacutevidence

leurs points communs

Initiation au management copy CRCF ndash J Sornet Page 10 48

Principes du management drsquoapregraves la norme ISO 9001 (2000)

- Orientation vers le client (satisfaire ses attentes)

- Leadership (les dirigeants eacutetablissent les orientations de lrsquoorganisme Ils doivent creacuteer

un environnement interne ougrave les personnes peuvent clairement srsquoimpliquer dans la

reacutealisation des objectifs de lrsquoorganisme)

- Implication du personnel (les personnes sont agrave tout niveau lrsquoessence de lrsquoorganisme et

leur implication permet drsquoutiliser leurs aptitudes au profit de lrsquoorganisme)

- Approche laquo processus raquo (un reacutesultat est mieux atteint quand les ressources et les

activiteacutes neacutecessaires sont geacutereacutees comme un processus)

- Approche systegraveme (assimiler les processus correacuteleacutes agrave un systegraveme contribue agrave

lrsquoefficaciteacute et agrave lrsquoefficience de lrsquoorganisme vis-agrave-vis de ses objectifs)

- Ameacutelioration continue (objectif permanent de lrsquoorganisme)

- Prise de deacutecision efficace (par lrsquoanalyse de donneacutees et drsquoinformations)

- Relations mutuellement beacuteneacutefiques avec les fournisseurs (pour augmenter la capaciteacute

des deux organismes agrave creacuteer de la valeur)

IM2 Distinguer leader et manager

IM3 Compleacuteter le tableau ci-dessous en analysant chaque action preacutesenteacutee Faire ensuite

ressortir les domaines niveaux ou techniques de management pouvant ecirctre mobiliseacutes pour

chaque situation

Initiation au management copy CRCF ndash J Sornet Page 11 48

Caracteacuteristiques

de lrsquoaction

- reacutepeacutetition

- risque

- normes

- ampleur

Prise de

deacutecision

- opeacuterationnelle

strateacutegique

- deacutelai

Informations

neacutecessaires

- nature

- origine

- deacutelai obtention

Cleacutes pour la

reacuteussite

Intervention

exteacuterieure

possible

Assurer la

restauration du

soir

(restaurant

familial)

Construire un

viaduc

(autoroute)

Certifier les

comptes

annuels drsquoun

groupe

national

(cabinet

drsquoaudit)

Lancer une

ligne drsquoavions

(constructeur

aeacuteronautique)

Reacuteduire la

capaciteacute de

production

(groupe

industriel)

Acqueacuterir une

entreprise

concurrente

(teacuteleacutephonie

mobile)

Initiation au management copy CRCF ndash J Sornet Page 12 48

Fiche IM1 - Deacutefinitions du management

Dictionnaire anglais - franccedilais direction administration gestion intrigue manegravege

Wikipeacutedia Le management est lensemble des techniques dorganisation qui sont mises en

oeuvre pour ladministration dune entiteacute

Au point de vue eacutetymologique le verbe manage vient de litalien maneggiare (controcircler)

influenceacute par le mot franccedilais manegravege (faire tourner un cheval dans un manegravege) A cette

notion il faut aussi ajouter la notion de meacutenage (geacuterer les affaires du meacutenage) qui consiste agrave

geacuterer des ressources humaines et des moyens financiers

helliphellip

Fiche IM2 - Etudier le management

Concreacutetiser

Manager neacutecessite de syntheacutetiser des informations parfois complexes incomplegravetes et de

domaines tregraves divers pour en deacuteduire des actions Une approche trop parcellaire peut

conduire agrave lrsquoeacutechec et le savoir-faire est neacutecessaire pour agir vite avec un minimum de risque

Lrsquoeacutetudiant doit se preacuteparer simultaneacutement aux examens et agrave la pratique Il nrsquoa souvent connu

lrsquoentreprise que durant quelques semaines de stage et le manque de laquo recul raquo ne lui permet

pas toujours de concreacutetiser les theacuteories Il doit compenser par la lecture (ouvrages revues

journaux eacuteconomiques et boursiers) et en eacutetant attentif aux informations ambiantes (tout en

relativisant le style journalistique) en mettant en relation le cours les concepts les modegraveles

lrsquoactualiteacute les stages

Savoir traiter un exercice

Pour reacuteussir un examen ou traiter une application peacutedagogique (la conception les points 1 agrave

6 peut repreacutesenter le tiers du temps de travail)

1 ndash Identifier le type de sujet (faut-il trouver une solution pratique ou communiquer une

reacuteflexion geacuteneacuterale )

2 ndash Lire le sujet et relever les mots cleacutes

3 ndash Deacutefinir les mots cleacutes

4 ndash Reacutesumer la probleacutematique du sujet (en quelques lignes)

5 ndash Lister les connaissances reacutefeacuterences et raisonnements reacutepondant au problegraveme (par

recherche spontaneacutee ou raisonneacutee qui quoi ougrave quand comment combien hellip

listage des diffeacuterents points de vue) trouver des exemples (notamment dans les

documents fournis)

6 ndash Organiser la reacuteponse (deacutefinir le plan du deacuteveloppement ougrave des paragraphes bien

identifieacutes sont geacuteneacuteralement neacutecessaires en y liant les parties qui doivent ecirctre en nombre

limiteacute ndash de deux agrave quatre) Preacutevoir drsquoy inteacutegrer la deacutefinition des principales notions

induites par le sujet

7 ndash Reacutediger sous la forme adapteacutee (note technique ou recommandation solution

pratique exposeacute structureacute dissertation)

Introduction et conclusion sont indispensables agrave la dissertation ou agrave lrsquoexposeacute

- lrsquointroduction preacutesente le sujet traiteacute (phrase drsquoaccroche initiale) amorce la

probleacutematique (quelques sous - questions) et annonce le plan

- la conclusion syntheacutetise le deacuteveloppement (arguments) eacutelargit le sujet (prise de recul)

et apporte le point final (une phrase)

Une limite agrave la communication

Il est difficile de faire passer plus de 4 ou 5 ideacutees fortes dans un exposeacute unique

Initiation au management copy CRCF ndash J Sornet Page 13 48

Fiche IM3 - Bref historique

Antiquiteacute

3000 AJC

Peacuteriode greacuteco-

romaine

Transition

feacuteodale

12egraveme siegravecle

europe

15egraveme ndash 17egraveme

siegravecles

19egraveme siegravecle

20egraveme siegravecle

agriculture preacutedominante industrie limiteacutee aux besoins drsquoun individu ou drsquoun clan

pour la confection des outils des vecirctements et de la poterie Force motrice animale

ou humaine pour lrsquoessentiel

Grands travaux drsquoeacutetat en Egypte premiegravere laquo planification ndash organisation ndash controcircle raquo

Deacuteveloppement des communications essor industriel limiteacute peu de progregraves

technique (lrsquoesclavage supplante les innovations)

Deacuteveloppement progressif des eacutechanges commerciaux

La consommation indirecte atteint un bon niveau (surplus agricoles et

deacuteveloppement des villes) Apparition de nouveaux commerccedilants

Etat fort Evolutions technologiques (imprimerie bateaux performants instruments de

navigation) Extension geacuteographique de lrsquoeacuteconomie Apparition des corporations

drsquoartisans

Machine agrave vapeur chemin de fer passage de lrsquoartisanat au capitalisme

entrepreneurial producteur organisation des entreprises

Ecole classique (Taylor Fayol Weber) approche meacutecaniste bureaucratie

hieacuterarchie commandement fonctions et speacutecialisation laquo OS T raquo (organisation

scientifique du travail) organisation source de pouvoir rationaliteacute des individus bases

du management

Deacuteveloppement du capitalisme manageacuterial Electriciteacute peacutetrole puis communications

et information Consommation de masse mondialisation preacuteoccupations

eacutenergeacutetiques et environnementales 3 peacuteriodes

- standardisation grandes entreprises industrielles

- industries de consommation 30 glorieuses marketing multinationales protection

sociale

- deacutereacuteglementation monteacutee des services pays eacutemergents mondialisation et nouvelle

eacuteconomie (internet)

Ecole des relations humaines prise en compte de lrsquoindividu des motivations styles

de direction

Ecole neacuteo-classique et post-classique deacutecentralisation coordonneacutee DPO

management participatif zeacutero deacutefaut flux tendus

Approche systeacutemique partition de lrsquoentreprise eacutetude des interactions feacutedeacuteration

vers lrsquoobjectif controcircle et ajustement

Theacuteories de la deacutecision rationaliteacute limiteacutee contribution reacutetribution coalitions

Ecole socio-technique recherche de compromis technologie organisation

enrichissement des tacircches autonomie des groupes

Approche sociologique effets sociaux du travail jeux de pouvoir dans lrsquoentreprise

reacutegulation sociale

Theacuteories de la contingence facteurs contingents adaptation agrave lrsquoenvironnement

configurations organisationnelles

Theacuteories de la firme controcircle manageacuterial droits de proprieacuteteacute relation drsquoagence

Theacuteories contractualistes firme nœud de contrats coucircts de transaction

opportunisme externalisation internalisation

Approche eacutevolutioniste eacutecologie des organisations modegravele eacutevolutioniste

contraintes de sentier

Approche par les ressources valorisation des ressources compeacutetences cleacutes

apprentissage organisationnel

(Classement simplifieacute)

Initiation au management copy CRCF ndash J Sornet Page 14 48

ELEMENTS DE CORRIGE IM

IM1 Commenter la deacutefinition du management par la norme ISO et le manager de Mintzberg

Efficient = optimum avec les moyens disponibles

ISO (management objectifs) (manager moyens) HM

IM2 Le leader entraicircne naturellement derriegravere lui Le manager nrsquoest pas toujours leader

(mecircme si crsquoest souhaitable) Le leader nrsquoest pas toujours manager (plutocirct notion individuelle)

Leadership = faculteacute de diriger conjugaison drsquoune autoriteacute naturelle ou drsquoun savoir-faire

acquis drsquoune capaciteacute agrave entraicircner des personnes ou des groupes et drsquoune leacutegitimiteacute

statutaire (de position)

IM3 Compleacuteter le tableau ci-dessous en analysant chaque action preacutesenteacutee Faire ensuite

ressortir les domaines niveaux ou techniques de management pouvant ecirctre mobiliseacutes pour

chaque situation

Satisfaction client

Implication du personnel

Processus systegraveme

Ameacutelioration continue

Deacutecision efficace

Recherche de valeur

Image entreprise

Liaisons

Information

Reacutepartition ressources

Reacutegulation

Neacutegociation

Leadership

Initiation au management copy CRCF ndash J Sornet Page 15 48

Caracteacuteristiques

de lrsquoaction

- reacutepeacutetition

- risque

- normes

- ampleur

Prise de

deacutecision

- opeacuteration

- direction

- deacutelai

Informations

neacutecessaires

- nature

- origine

- deacutelai

obtention

Cleacute pour la

reacuteussite

Intervention

exteacuterieure

possible

Assurer la

restauration du

soir

(Restaurant

familial)

Technique

(fabrication)

Vente (terrain)

Appros

Reacutepeacutetitive

(quot)

Risque faible

Normes

drsquohygiegravene

Faible

Opeacuterationnelle

Geacuterant

responsable

Rapide (qq

jours menu et

appros)

Nombre de

couverts

Tarifs usuels

Calendrier

(fecirctes)

Clients docs

divers

expeacuterience

Qq jours

Varieacuteteacute menu

Plats phares

Accueil

Appros

Tarification

Vins

Gestion

congeacutelation

Qualiteacute cuisine

Fournisseurs

Extra

Publiciteacute

Construire un

viaduc

(autoroute)

Technique

Organisation

Appros

Uniteacute (ou peu)

Eleveacute (financier

technique)

Architecture

Eleveacutee

Direction

(aleacuteas)

Opeacuterationnelle

(conduite

chantier)

Immeacutediat agrave qq

semaines

Plans

plannings

Qualifications

Meacuteteacuteo

Disponibiliteacutes

Bureau eacutetudes

Qq sem agrave 24h

Techniciteacute

Appros

Qualifications

Preacutevision

GRH

Contrat juste

SS traitants

Organismes

certificateurs

Controcircle

client

Certifier les

comptes

annuels drsquoun

groupe national

(cabinet

drsquoaudit)

Technique

Relation client

Gestion des

connaissances

Annuelle

Moyen

Regravegles

comptables

fiscales

Moyenne (selon

importance du

cabinet)

Opeacuterationnelle

Qq jours agrave

semaines

Comptable

Juridique

Client

Etat

Qq jours agrave

semaines

Techniciteacute

Expeacuterience

Relation client

Systegraveme info client

Siegravege

Autre cabinet

Lancer une

ligne drsquoavions

(constructeur

aeacuteronautique)

Strateacutegique

RD

Etudes

Uniteacute

Tregraves eacuteleveacute

Aeacuteronautique

Tregraves eacuteleveacutee

Direction

Qq mois agrave

anneacutees

Marcheacute

Etudes

Compagnies

Qq mois agrave

anneacutees

Concept

Outil industriel

Coucirct exploitation

Tarif

Fiabiliteacute

Deacutelaisconcurrence

SI simulation

SS traitants

Bureaux

drsquoeacutetudes

speacutecialiseacutes

Compagnies

Conseils

Reacuteduire la

capaciteacute de

production

(groupe

industriel)

Strateacutegique

RH

Communication

Production

Uniteacute

Moyen

Leacutegislation

(dont RH)

Eleveacutee

Direction

Qq mois agrave

anneacutees

Financiegravere

Industrielle

Marcheacute

Organisation

Organismes

speacutecialiseacutes

DRH

Qq mois

Communication

Connaissance des

compeacutetences

Connaissance outil

industriel

Concurrence

Portefeuille

drsquoactiviteacutes

Cabinet

drsquoorganisation

Conseils

speacutecifiques

Acqueacuterir une

entreprise

concurrente

(teacuteleacutephonie

mobile)

Strateacutegique

Marketing

Production

(reacuteseau)

Financier

Communication

Uniteacute

Tregraves eacuteleveacute

Leacutegislation

telecom

Tregraves eacuteleveacutee

Direction

Qq mois

Financiegravere

Marcheacute

Reacuteseaux

(ampleur

recouvrement

hellip)

Organisations

Interne

Racheteacutee

Sources

speacutecialiseacutees

Qq mois

Communication

Marcheacute

Cours boursiers

Cabinet

drsquoorganisation

Conseils

speacutecifiques

Initiation au management copy CRCF ndash J Sornet Page 16 48

LE MANAGEMENT EN PRATIQUE

Pour assumer sa fonction le management doit couvrir sans discontinuiteacute lrsquoensemble de

lrsquoorganisation et inteacutegrer de nombreux facteurs dont nous allons reacutesumer lrsquoessentiel

1 ndash Les fonctions et activiteacutes du management

Pour Henri Fayol la fonction drsquoadministration de lrsquoentreprise (son management) reposait sur

cinq actions preacutevoir organiser commander coordonner et controcircler (laquo PO3C raquo)

Nous distinguerons cinq activiteacutes de management

- la conception (au plus haut niveau finaliteacute but ou vocation de lrsquoorganisation

meacutetiers dimension politique de croissance hellip)

- la planification (deacutefinition des objectifs eacutecheacuteances)

- lrsquoorganisation (reacutepartition du travail choix des modes de coordination)

- le pilotage de lrsquoaction opeacuterationnelle (motivation animation encadrement

assistance)

- lrsquoeacutevaluation (controcircle des reacutesultats obtenus ajustements)

Dans chacune de ces activiteacutes des deacutecisions et des arbitrages sont neacutecessaires avec des

enjeux plus ou moins importants

Remarques

- Les cinq activiteacutes du management peuvent se retrouver agrave tout niveau de

management si lrsquoentreprise laisse une certaine autonomie de deacutecision agrave ses diffeacuterentes

uniteacutes La conception est naturellement du ressort de la direction geacuteneacuterale et des

conseils drsquoadministration mais elle peut ecirctre preacutesente pregraves du terrain (latitude laisseacutee agrave

une filiale ou agrave un magasin par exemple) De mecircme lrsquoorganisation du travail concerne

un atelier mais aussi la direction qui structure lrsquoentreprise pour assurer ses activiteacutes sa

production

- La planification deacutefinit des objectifs ou des axes strateacutegiques (choix de produits

modaliteacutes de deacuteveloppement des ventes implantations alliances hellip) et les traduit en

donneacutees de gestion preacutevisionnelles syntheacutetiques et eacutechelonneacutees dans le temps afin de

valider les objectifs et de fixer des repegraveres

- Un laquo business plan raquo (plan drsquoaffaires)est notamment lrsquoeacutequivalent de la planification

dans le cas de creacuteation drsquoentreprise ou pour la preacutesentation de tout projet drsquoactiviteacute

Les activiteacutes du management srsquoinscrivent dans des cycles qui peuvent ecirctre scheacutematiseacute

comme suit (lrsquoeacutevaluation peut entraicircner une reacutevision du pilotage de lrsquoorganisation ou des

objectifs sans que lrsquoentreprise ne soit fondamentalement remise en cause)

conception

planification

organisation

pilotage

eacutevaluation

Initiation au management copy CRCF ndash J Sornet Page 17 48

2 ndash Les contextes de management

Le management est influenceacute par son contexte qui justifie des objectifs une organisation

des meacutethodes

Par exemple lrsquoentreprise admet de nombreuses variantes selon sa taille sa forme juridique

son controcircle par lrsquoeacutetat (entreprises publiques) ou par des inteacuterecircts priveacutes Il en va de mecircme des

organismes administratifs qui peuvent deacutependre de directives nationales ou reacutegionales des

associations qui ont des activiteacutes drsquoampleur tregraves variable

21 ndash La dimension de lrsquoentreprise

La dimension drsquoune entreprise se mesure principalement en fonction de son effectif ou de

son chiffre drsquoaffaires Des seuils sont deacutefinis par divers organismes et exploiteacutes agrave des fins

statistiques ou pour la deacutetermination de certaines obligations sociales ou fiscales

(repreacutesentation du personnel cotisations hellip) Il nrsquoy a bien entendu pas de laquo barriegravere de

tailleraquo absolue conditionnant le management drsquoune entreprise

LrsquoUE preacuteconise de distinguer les micro ndash entreprises (jusqursquoagrave 9 salarieacutes) les TPE ndash tregraves petites

entreprises (moins de 20 salarieacutes) les petites entreprises (moins de 50) et les moyennes

entreprises (de 50 agrave 250) Cependant les PME sont parfois situeacutees entre 10 et 500 salarieacutes

Remarques

- en France environ 40 des entreprises emploient de 1 agrave 50 salarieacutes (ce qui repreacutesente

plus de 50 des emplois) et 59 nrsquoen ont aucun

le pays compte environ 2 600 000 entreprises dont moins de 1 ont 250 employeacutes et

plus

- ancienneteacute et taille de lrsquoentreprise sont lieacutees si lrsquoon eacutecarte les restructurations et autres

eacutevolutions drsquoentreprises existantes

La dimension de lrsquoentreprise a une influence sur lrsquoorganisation et le laquo style raquo de son

management

- les PME sont souvent entrepreneuriales (les dirigeants eacutegalement apporteurs de capitaux

sont totalement engageacutes dans la marche de lrsquoentreprise) Elles ont une gestion flexible peu

formaliseacutee plus qualitative que quantitative Les PME sont freacutequemment focaliseacutees sur un seul

type drsquoactiviteacute Pour ne pas alourdir leur structure elles ont tendance agrave sous-traiter les

activiteacutes speacutecialiseacutees ne correspondant pas agrave leur meacutetier de base

- les grandes entreprises sont manageacuteriales (les dirigeants sont nommeacutes par les actionnaires

en raison de leurs compeacutetences) et moins reacuteactives

22 ndash Le type de production

On distingue industrie (production de biens mateacuteriels ou pour le moins de produits visibles ndash

comme un seacutejour touristique ou un film) et services (fourniture drsquoune prestation immateacuterielle)

Le type de production influence en principe le management de lrsquoentreprise

- lrsquoindustrie neacutecessite (si lrsquoon excepte lrsquoartisanat) un investissement relativement important

une organisation productive stable capable de reacutealiser plusieurs fois des produits identiques

(exemple un modegravele de reacutefrigeacuterateur) ou du moins similaires (exemple un bacirctiment) Le

produit de lrsquoindustrie consomme des matiegraveres et il doit geacuteneacuteralement ecirctre distribueacute jusqursquoau

client

- la production de services peut se satisfaire drsquoun investissement tregraves reacuteduit et neacutecessite un

contact permanent avec le client

Toutefois la standardisation des services et le deacuteveloppement des reacuteseaux informatiques

rapprochent la production de services de celle des biens industriels

- la production drsquoun service reacutepeacutetitif et technique peut imposer une structure lourde et une

organisation tregraves formaliseacutee (voir les grandes socieacuteteacutes drsquoaudit ou de conseil informatique)

Initiation au management copy CRCF ndash J Sornet Page 18 48

- certains services peuvent ecirctre fournis agrave distance sans contact direct avec le client et

distribueacutes par reacuteseau (tenue de comptabiliteacute affacturage gestion clientegravele centre drsquoappel

hellip)

Remarque les services repreacutesentent 75 de lrsquoactiviteacute eacuteconomique franccedilaise

23 ndash La nature de lrsquoorganisation

Les organisations publiques franccedilaises (administrations centrales collectiviteacutes territoriales

hocircpitaux hellip) repreacutesentent une part importante de lrsquoactiviteacute (environ 30 des emplois) La

fonction publique regroupe des organisations aux finaliteacutes diverses et qui ont des problegravemes

de gestion similaires agrave ceux des entreprises auxquelles elles peuvent emprunter des principes

de management Notamment

- pour controcircler les coucircts et assurer la qualiteacute des services

- pour communiquer avec les administreacutes ou les usagers

- pour motiver les personnels et geacuterer les ressources humaines

La transposition directe des techniques de gestion et de management nrsquoest cependant pas

toujours possible car

- la comptabiliteacute publique obeacuteit agrave des regravegles speacutecifiques (proceacutedure budgeacutetaire

notamment)

- le laquo client raquo ne paye pas toujours la prestation du moins directement

- la concurrence est parfois inexistante

- les grandes administrations centraliseacutees sont soumises agrave des choix politiques geacuteneacuteraux

parfois sans connexion eacutevidente avec les besoins opeacuterationnels

- le statut des personnels et les grilles de salaires limitent les possibiliteacutes de gestion des

ressources humaines

Remarque la LOLF (loi organique relative aux lois de finances) est entreacutee en vigueur en

2006 Elle alloue des moyens budgeacutetaires en fonction de programmes et remplace la

reconduction automatique de 90 des budgets Cette reacuteforme se heurte toutefois agrave la

lourdeur des grands ministegraveres ougrave la complexiteacute des activiteacutes est difficile agrave

appreacutehender et ougrave des inerties culturelles peuvent exister agrave tout niveau

Les associations loi de 1901 peuvent avoir une activiteacute comparable agrave celle de grandes

entreprises (voir par exemple les associations de santeacute ou professionnelles) et leur

management est alors similaire malgreacute lrsquoabsence de but lucratif (les beacuteneacutefices ne sont pas

distribuables) Elles ont drsquoailleurs en France un poids eacuteconomique important (elles emploient

environ 1 600 000 salarieacutes)

Cependant lrsquoadheacutesion agrave un systegraveme de valeurs fondateur de lrsquoassociation ou la limite de

lrsquoautoriteacute (quand un volant de beacuteneacutevoles important participe agrave lrsquoactiviteacute) peut introduire des

nuances

- le renforcement des objectifs socieacutetaux

- la faiblesse des relations hieacuterarchiques

- des contraintes de gestion du temps des beacuteneacutevoles

- des modaliteacutes particuliegraveres de recrutement et de motivation des dirigeants

24 ndash Les facteurs contingents

La theacuteorie de la contingence montre qursquoune structure drsquoentreprise nrsquoest efficace que dans

une situation deacutetermineacutee et qursquoil nrsquoexiste que des solutions de management construites dans

un contexte preacutecis

Le management doit ainsi srsquoadapter agrave des facteurs contingents qui ne peuvent ecirctre

controcircleacutes du moins agrave bregraveve eacutecheacuteance Ces facteurs sont par exemple

- lrsquoancienneteacute de lrsquoentreprise (plus elle est ancienne plus lrsquoentreprise a tendance agrave reacutepeacuteter

des comportements eacuteprouveacutes)

Initiation au management copy CRCF ndash J Sornet Page 19 48

- la taille de lrsquoentreprise (la grande entreprise a une composante administrative plus

deacuteveloppeacutee)

- le systegraveme de production (tregraves standardiseacute complexe automatiseacute hellip)

- lrsquoenvironnement

3 ndash Le management et les parties prenantes

Lrsquoentreprise a pour vocation premiegravere de mettre des produits agrave disposition de ses clients en

reacutealisant un profit Pour y arriver elle doit aussi satisfaire ses parties prenantes salarieacutes

actionnaires fournisseurs hellip

Est partie prenante agrave lrsquoentreprise laquo tout groupe ou individu qui peut ecirctre affecteacute ou est

affecteacute par les buts de lrsquoorganisation hellip raquo (Freeman ndash 1984)

Les parties prenantes attendent agrave des degreacutes divers de profiter drsquoune creacuteation de valeur en

provenance de lrsquoentreprise qui doit reacutepondre agrave ces attentes pour assurer sa peacuterenniteacute ou

favoriser son deacuteveloppement

On distingue les parties prenantes primaires ou principales qui sont essentielles agrave lrsquoentreprise

et qui ont geacuteneacuteralement une relation formelle avec elle (clients associeacutes et actionnaires

precircteurs salarieacutes fournisseurs collectiviteacutes) et les parties prenantes secondaires dont

lrsquoinfluence est diffuse (groupes de pression associations meacutedias instances europeacuteennes

agences de notation hellip)

Remarque la consideacuteration de lrsquoensemble des parties prenantes (laquo stakeholders raquo - les

deacutepositaires) fait contrepoids agrave lrsquoimportance accordeacutee aux seuls actionnaires

(laquo shareholders raquo)

Les organisations nrsquoayant pas drsquoobjectif de profit doivent aussi satisfaire leurs parties

prenantes apporter un service aux usagers dans les meilleures conditions eacuteconomiques

limiter un budget assurer la qualiteacute des relations avec les fournisseurs hellip

Dans cette optique le management doit organiser lrsquoaction de faccedilon agrave eacutequilibrer des forces

parfois divergentes

- le contexte fait pression sur lrsquoorganisation contrainte agrave optimiser ses reacutesultats

- lrsquoorganisation cherche par son action agrave assurer sa peacuterenniteacute son deacuteveloppement (en

reacutealisant des profits dans le cas de lrsquoentreprise) et agrave satisfaire ses parties prenantes

- le management agit en pilotant les actions pour contrebalancer la pression du contexte

Actions de

lrsquoorganisation

Management Contexte

Parties

prenantes

Initiation au management copy CRCF ndash J Sornet Page 20 48

APPLICATIONS MP

MP1 Deacutefinir contingent gestion budgeacutetaire

MP2 Deacuteterminer les parties prenantes drsquoun hocircpital public et leurs principales attentes

Mecircme question pour les organisations suivantes

- SNCF (entreprise publique)

- Peugeot

- MAIF (mutuelle drsquoassurance)

MP3 En les situant dans le cycle des activiteacutes du management trouver les actions agrave mener

dans les situations suivantes

- baisse de 10 des ventes dans une entreprise industrielle (produits meacutenagers le reacuteseau de

distribution vient drsquoecirctre reacuteorganiseacute)

- idem dans une entreprise de vente par correspondance soumise agrave la concurrence internet

(les ventes stagnaient depuis six mois malgreacute les efforts promotionnels)

- augmentation des deacutelais drsquoattente des consultations dans une clinique (lrsquohocircpital voisin a

fermeacute son service drsquourgences)

Initiation au management copy CRCF ndash J Sornet Page 21 48

ELEMENTS DE CORRIGE MP

MP1 Deacutefinir (dans le contexte drsquoune entreprise) contingent gestion budgeacutetaire

Contingent = imposeacute par lrsquoexteacuterieur Contingence = effet du hasard de la rencontre de

plusieurs eacuteveacutenements indeacutependants (variables explicatives que lrsquoon ne peut influencer)

Gestion budgeacutetaire = technique drsquoadministration des entreprises srsquoappuyant sur des

preacutevisions dont on deacuteduit apregraves accord des responsables des attributions de moyens sur une

dureacutee limiteacutee Une analyse reacuteguliegravere des eacutecarts entre preacutevisions et reacutealisations permet ensuite

le pilotage des activiteacutes Le budget est un cadre incitatif

La laquo planification budgeacutetaire raquo consiste agrave traduire en budgets une planification strateacutegique

avec systegraveme de reporting

MP2 Deacuteterminer les parties prenantes drsquoun hocircpital public et leurs principales attentes

Mecircme question pour les organisations suivantes

- SNCF (entreprise publique)

- Peugeot

- MAIF (mutuelle drsquoassurance)

Hocircpital

- patients (qualiteacute des soins)

- CNAM (baisse des coucircts)

- collectiviteacute locale (service aux administreacutes)

- eacutetat (ameacutenagement du territoire maicirctrise des budgets optimisation)

- employeacutes (salaire conditions de travail et satisfaction)

- fournisseurs ndash pharmacie autres (CA paiement reacutegulier)

- associations de patients (qualiteacute proximiteacute des soins)

SNCF

- usagers et associations drsquousagers (proximiteacute reacutegulariteacute prix du service)

- reacuteseau ferreacute de France (optimisation des lignes paiement adapteacute)

- fournisseurs (CA paiement reacutegulier)

- employeacutes (salaire conditions de travail seacutecuriteacute de lrsquoemploi)

- eacutetat (ameacutenagement du territoire)

- collectiviteacutes locales (service)

Peugeot

- clients (qualiteacute prix SAV relation commerciale)

- fournisseurs (CA reacutegulariteacute de lrsquoactiviteacute)

- employeacutes (salaire conditions de travail seacutecuriteacute de lrsquoemploi)

- eacutetat (taxes)

- collectiviteacute locale (emploi dynamisation eacuteconomique preacuteservation de lrsquoenvironnement)

- associations de protection de lrsquoenvironnement (activiteacute propre baisse des eacutemissions

nouvelles eacutenergies)

MAIF

- socieacutetaires (protection relation assureur tarif mesureacute)

- professionnels de lrsquoautomobile et autres (agreacutement marge de manœuvre reacuteparations tarifs

eacuteleveacutes)

- fournisseurs (CA paiement reacutegulier)

- eacutetat (taxes engagement pour la seacutecuriteacute)

- employeacutes (salaire conditions de travail seacutecuriteacute de lrsquoemploi)

Initiation au management copy CRCF ndash J Sornet Page 22 48

MP3 En les situant dans le cycle des activiteacutes du management trouver les actions agrave mener

dans les situations suivantes

- baisse de 10 des ventes dans une entreprise industrielle (produits meacutenagers le reacuteseau de

distribution vient drsquoecirctre reacuteorganiseacute)

Adapter le pilotage motiver cadrer si insuffisant retoucher une organisation deacutefectueuse

- idem dans une entreprise de vente par correspondance soumise agrave la concurrence internet

(les ventes stagnaient depuis six mois malgreacute les efforts promotionnels)

Voir pilotage et organisation si une eacutevolution du meacutetier a deacutejagrave eacuteteacute initialiseacutee Sinon re-

conception (adaptation au nouveau contexte) puis planification et reacuteorganisation

- augmentation des deacutelais drsquoattente des consultations dans une clinique (lrsquohocircpital voisin a

fermeacute son service drsquourgences)

Organisation Si insuffisant planification (nouveaux objectifs)

Initiation au management copy CRCF ndash J Sornet Page 23 48

ORGANISATION ET PROCESSUS

La performance de lrsquoentreprise deacutepend de son organisation et de son aptitude agrave produire

aux meilleures conditions Nous allons montrer comment organisation formelle et processus

de production peuvent contribuer agrave cette performance

1 ndash Vers lrsquooptimum

11 ndash Les eacuteconomies occidentales jusqursquoaux anneacutees 70

Jusqursquoen 1945 le principal problegraveme des entreprises eacutetait de produire des biens en quantiteacute

suffisante agrave un prix compatible avec le marcheacute Les grandes entreprises se sont multiplieacutees et

la standardisation a permis de reacuteduire les coucircts (exemple deacuteveloppement de Ford et de la

production agrave la chaicircne de 1908 agrave 1920 qui a permis une baisse du prix des voitures des 23)

On parle de laquo production pousseacutee vers le marcheacute raquo

Cette croissance de la production peu reacuteguleacutee a eacuteteacute marqueacutee par des surproductions en

1910 et 1920 puis par la crise de 1929 qui a prolongeacute ses effets jusqursquoagrave la guerre

De 1945 agrave 1975 environ (les laquo trente glorieuses raquo) la reconstruction la croissance de la

consommation de masse de nouvelles technologies et les eacutechanges internationaux

alimentent lrsquoeacuteconomie La standardisation srsquoeacutetend aux biens de consommation dont les

coucircts baissent fortement et de nouvelles reacutegulations sociales permettent une eacutevolution sans

heurt des revenus La saturation de certains marcheacutes conduit dans les anneacutees 60 agrave la

deacutemarche laquo marketing raquo et agrave la diffeacuterenciation des produits Le produit est laquo dirigeacute par le

marcheacute raquo mais les entreprises conservent une organisation assez classique et les plus grosses

srsquointernationalisent

12 ndash Lrsquoexpeacuterience japonaise et ses prolongements

Tregraves tocirct apregraves la guerre dans un Japon appauvri le constructeur automobile Toyota a ducirc

faire face agrave une restriction du marcheacute des moyens financiers et productifs et des

approvisionnements La firme a donc innoveacute dans un nouveau systegraveme de production

chassant les laquo gaspillages raquo (temps drsquoattente transports stocks deacutefauts hellip) consideacuterant que

seule la fabrication vendable creacutee de la valeur

Toyota srsquoorganise pour fabriquer la quantiteacute et la qualiteacute de produits juste neacutecessaires agrave la

satisfaction des clients la production est laquo tireacutee par le marcheacute raquo La mise en place de ce

systegraveme qui integravegre les fournisseurs ne sera acheveacutee que dans le milieu des anneacutees 70

En 1973 la hausse du peacutetrole inaugure un ralentissement de la croissance des eacuteconomies

occidentales La concurrence accrue provoque alors un inteacuterecirct pour le systegraveme deacuteveloppeacute

au Japon La production au plus juste se deacuteveloppe ainsi dans lrsquoindustrie automobile agrave partir

des anneacutees 80 et elle se reacutepand encore maintenant dans drsquoautres secteurs

Cette approche qui vise un objectif de zeacutero stock et zeacutero deacutefaut impose la maicirctrise de laquo bout

en bout raquo des processus de production et leur ameacutelioration

Initiation au management copy CRCF ndash J Sornet Page 24 48

2 ndash Organiser lrsquoentreprise

21 ndash Direction et organisation

Diriger une entreprise neacutecessite de lrsquoorganiser (de reacutepartir les tacircches) pour qursquoelle puisse

atteindre ses objectifs Lrsquoorganisation permet de satisfaire un marcheacute en tirant parti des

capaciteacutes actuelles de lrsquoentreprise tout en preacuteparant lrsquoavenir

Lrsquoorganisation reacutesulte freacutequemment drsquoun compromis entre des objectifs situeacutes agrave des niveaux

et des eacutecheacuteances diffeacuterents

Exemples

- le leader des chaises roulantes peut tirer profit de sa structure productive et de son

savoir faire pour entrer sur le marcheacute de la bicyclette eacutelectrique

- ecirctre parfaitement structureacute pour alimenter 90 du marcheacute des disquettes ne preacutepare

pas lrsquoavenir

- srsquoorganiser pour conqueacuterir le marcheacute des tire-bouchons eacutelectriques dans les deux ans

perd de son sens si cela altegravere les moyens neacutecessaires agrave la production drsquoappareils

manuels ancienne mais vitale dont la diminution agrave court terme risque de nuire agrave la

solvabiliteacute de lrsquoentreprise et de la conduire agrave la cessation de paiement

22 ndash Lrsquoorganisation fonctionnelle

La majoriteacute des entreprises adopte une laquo organisation fonctionnelle raquo (celle qui est visible

dans les organigrammes) ougrave des regroupements de personnels et drsquoeacutequipements se font

selon un modegravele hieacuterarchique (laquo line raquo) dans des uniteacutes des services ou des deacutepartements

speacutecialiseacutes Cette organisation peut se deacutecliner agrave lrsquointeacuterieur des divisions des grandes

entreprises quand elles scindent leur activiteacute par zone geacuteographique type drsquoactiviteacute

cateacutegorie de clients hellip

Remarque le terme laquo fonction raquo deacutesigne un rocircle particulier dans le fonctionnement de

lrsquoentreprise

Lrsquoorganisation fonctionnelle diffeacuterencie les activiteacutes de lrsquoentreprise en les regroupant par

meacutetier pour utiliser au mieux les compeacutetences et les moyens (meilleur rendement par la

speacutecialisation lrsquoeacutechange de compeacutetences dans une mecircme uniteacute ou gracircce agrave des eacuteconomies

drsquoeacutechelle)

23 ndash La notion de processus de production

Un processus de production se deacutefinit par la succession drsquoactiviteacutes permettant de satisfaire

un client en transformant des ressources (mateacuterielles financiegraveres humaines) en un produit

bien ou service Le processus doit creacuteer une valeur reconnue par le client

Un processus peut servir un client interne agrave lrsquoentreprise (par exemple en produisant un

composant intervenant dans plusieurs produits ou par la maintenance des machines) aussi

bien qursquoun client final On distingue usuellement

- les processus opeacuterationnels (ou maicirctres) aussi appeleacutes processus meacutetier (business process)

qui satisfont directement les clients finaux (conception et fabrication de produits vente hellip)

- les processus de support et de management (geacuterer les ressources humaines geacuterer

lrsquoinformation geacuterer les ressources financiegraveres hellip) qui ont les processus opeacuterationnels comme

clients

Toutes les actions internes agrave une organisation peuvent srsquointeacutegrer dans des processus qui

conditionnent directement ou indirectement la capaciteacute de lrsquoorganisation agrave satisfaire le

client final ou lrsquousager

Initiation au management copy CRCF ndash J Sornet Page 25 48

Aborder le fonctionnement de lrsquoentreprise par ses processus (approche processus) permet

de mettre en eacutevidence les chaicircnes drsquoactiviteacutes qui conduisent aux produits leurs

dysfonctionnements leurs coucircts la formation des deacutelais et la souplesse (la flexibiliteacute)

disponible pour satisfaire la clientegravele finale Lrsquoameacutelioration des processus a un impact visible

et direct sur chaque produit proposeacute aux clients

Lrsquoapproche processus provoque une eacutevolution de la faccedilon de travailler

- en faisant peacuteneacutetrer la laquo voix du client raquo au plus profond de lrsquoentreprise (et plus seulement

dans les services commerciaux et marketing)

- en mettant en eacutevidence des possibiliteacutes de rationalisation (par regroupement ou impartition

de certaines activiteacutes)

Remarque lrsquoapproche par les activiteacutes et les processus est agrave lrsquoorigine de la meacutethode

de deacutetermination des coucircts laquo ABC raquo - activity based costing

24 ndash Processus et fonctions

Le processus est transversal Il enchaicircne des activiteacutes qui traversent lrsquoentreprise en particulier

les services ou les deacutepartements drsquoune organisation fonctionnelle

Exemple

La division du travail par fonctions induit une charge de coordination pour assurer le

deacuteroulement du processus Elle peut geacuteneacuterer des attentes des erreurs ou des conflits drsquointeacuterecirct

(lrsquoobservation montre que des dysfonctionnements sont tregraves souvent constateacutes lors du

passage drsquoun service agrave un autre)

Organisation fonctionnelle et approche processus visent toutes deux un optimum

eacuteconomique mais leurs logiques sont diffeacuterentes

- le processus vise la satisfaction des clients (prix qualiteacute deacutelais service)

- le deacutecoupage fonctionnel cherche agrave optimiser les moyens (maximiser lrsquoeffet drsquoexpeacuterience

partager des infrastructures profiter de pocircles de compeacutetences hellip) Il apporte une ossature

hieacuterarchique stable souvent indispensable

Organisation fonctionnelle et approche processus sont donc compleacutementaires dans la

majoriteacute des cas et doivent ecirctre combineacutees judicieusement

APPLICATIONS OP

OP1 Deacutefinir flexibiliteacute systegraveme impartition

OP2 Citer huit exemples drsquoinformations essentielles pour optimiser un processus de

fabrication

Direction

Deacutepartement

commercial

(C)

Deacutepartement

administratif et

financier (AF)

Deacutepartement

Etudes (E)

Deacutepartement

Production (P)

Activiteacute

C-x Activiteacute

AF-x Activiteacute

E-x

Activiteacute

P-x

Processus x

Clie

nt

Initiation au management copy CRCF ndash J Sornet Page 26 48

OP3 Deacutegager les principes du toyotisme preacutesenteacute ci-dessous En quoi ce systegraveme est-il

initiateur de lrsquoapproche processus

Taiichi Ohno et le Toyotisme

1 - Extrait drsquoun article de Jacques BARRAUX - 1993 - LExpansion

Taiichi Ohno (1912 ndash 1990) hellip ne se prenait pas pour un visionnaire mais en imposant une

nouvelle faccedilon de produire il a reacuteinventeacute le management hellip tout le monde a entendu parler

des mots qui ont populariseacute le toyotisme dont il est le pegravere le juste-agrave-temps hellip Autant

doutils conccedilus pour lrsquoautomobile et qui ont aujourdhui une application universelle

hellip Taiichi Ohno jeune ingeacutenieur entre chez Toyota alors simple constructeur de machines

textiles Degraves 1926 apparaicirct la notion de jidoka hellip cest lart de transfeacuterer de lintelligence aux

machines pour mieux libeacuterer lintelligence des hommes Tout le contraire du taylorisme qui

juge la machine moins impreacutevisible que lhomme En 1933 Toyota se lance dans lautomobile

en sinspirant des meacutethodes ameacutericaines Mais en 1935 agrave loccasion dun voyage aux Etats-

Unis leacutetat-major de lentreprise revient fascineacute de sa visite dans un supermarcheacute La notion

de juste-agrave-temps va naicirctre de lobservation dune grande surface un lieu ougrave les clients ne

prennent que ce dont ils ont besoin et ougrave les rayons sont reacuteapprovisionneacutes pour compenser

les quantiteacutes preacuteleveacutees Ainsi le systegraveme Toyota est-il deacutejagrave dans la tecircte de ses dirigeants avant

mecircme la Seconde Guerre mondiale un demi-siegravecle avant la reacutevolution informatique et la

segmentation intensive des marcheacutes

hellip des esprits curieux comme Franccedilois Dalle en France tombent alors sous le charme des

formules et des paraboles de Taiichi Ohno En voici deux eacutechantillons

Penser agrave lenvers Cela signifie combattre les ideacutees reccedilues En lespegravece il sagit du fordisme et

du taylorisme Ohno ne croit pas agrave la planification aux effets deacutechelle et dexpeacuterience Il

propose un systegraveme industriel agrave lenvers qui permette de diversifier les produits et de les

fabriquer en petites quantiteacutes Nous ne devons plus ecirctre des paysans qui accumulent des

stocks mais des chasseurs On nimpose pas loffre On traque la demande et on la gegravere en

continu

Que les valleacutees soient hautes et les montagnes peu eacuteleveacutees Plutocirct que de concentrer tous

les efforts sur une production agrave un moment donneacute mieux vaut se doter de structures flexibles

permettant de passer agrave tout instant dune seacuterie agrave une autre Il faut eacuteviter les ruptures et les

secousses aplanir les cycles entretenir des flux reacuteguliers dactiviteacutes diversifieacutees Ce qui

implique de ne pas enfermer les hommes et les eacutequipements dans des speacutecialisations trop

eacutetroites

La flexibiliteacute le travail en groupe le refus de la dictature des machines la polyvalence et

surtout lattention constante aux signaux eacutemis par le marcheacute nappartiennent plus au

toyotisme Ces notions sont les fondements du nouvel art dorganiser de vendre et de

produire dans lindustrie comme dans les services hellip

2 - Quelques notions cleacutes

Taiichi Ohno a imagineacute la meacutethode des laquo cinq pourquoi raquo qui consiste agrave se poser cinq fois de

suite la question laquo pourquoi raquo sur le mecircme sujet de faccedilon agrave deacutecouvrir la veacuteritable cause

drsquoun problegraveme Cette meacutethode peut ecirctre appliqueacutee agrave tous les niveaux et permettre

notamment aux agents de fabrication de proposer de veacuteritables ameacuteliorations de la

production

La recherche de la qualiteacute totale (pas de deacutefaut des produits pas de rebuts pas de deacutefaut

des processus) accompagne la deacutemarche de Toyota La qualiteacute a un coucirct compenseacute par

des ventes accrues par lrsquoeacuteconomie des mesures palliatives aux deacutefauts

Initiation au management copy CRCF ndash J Sornet Page 27 48

Fiche OP1 ndash Benchmarking et processus

Le laquo benchmarking raquo consiste agrave comparer le fonctionnement de plusieurs systegravemes pour en

faire notamment ressortir les meilleures pratiques (laquo best practices raquo) Cette technique est

utiliseacutee depuis les anneacutees 80 pour ameacuteliorer la performance des entreprises Elle impose agrave

lrsquoentreprise drsquoeacutevaluer et de remettre en question ses propres modes de fonctionnement afin

de les faire eacutevoluer agrave la lueur de ce qui se fait ailleurs

Le benchmarking permet drsquoameacuteliorer les processus agrave moindre risque en fixant des objectifs

baseacutes sur des faits et donc plus facilement accepteacutes

Une classification des processus en tant que base de reacuteflexion a eacuteteacute eacutetablie aux USA par

lrsquolaquo International Benchmarking Clearinghouse raquo de lrsquoAPQC (american productivity and

quality center) en collaboration avec plusieurs dizaines drsquoentreprises

Elle se reacutesume ainsi

Le terme laquo reengineering raquo (la re-conception ou laquo reacuteingeacutenieacuterie raquo) des processus deacutesigne un

projet drsquoameacutelioration radicale des performances (de 20 agrave 50 ou plus) Il neacutecessite une

parfaite adheacutesion de la direction la constitution drsquoune petite eacutequipe de projet brillante

connaissant parfaitement les activiteacutes de lrsquoentreprise et il peut inclure un benchmarking

Le reengineering provoque geacuteneacuteralement la reacuteduction du nombre de niveaux hieacuterarchiques

(laquo delayering raquo) et lrsquoaccroissement du pouvoir de deacutecision des employeacutes (laquo empowerment raquo

ou laquo empouvoirement raquo) Bien qursquoy conduisant parfois il ne doit pas ecirctre confondu avec la

reacuteduction des activiteacutes (laquo downsizing raquo ou restructuration) et lrsquoexternalisation (laquo outsourcing raquo)

Pro

ce

ssu

s

op

eacutera

tio

nn

els

Pro

ce

ssu

s d

e m

an

ag

em

en

t e

t d

e

sup

po

rt

1 ndash

Comprendre

le marcheacute et

les clients (besoins

satisfaction)

2 ndash

Deacutevelopper

vision et

strateacutegie (contexte

concurrence)

3 ndash

Creacuteer

produits

services

processus

(concevoir

ameacuteliorer)

4 ndash

Marketing et

vente

5 ndash

Produire et

livrer (industrie

dont

ameacutelioration

processus)

6 ndash

Produire et

livrer (services)

7 ndash

Facturer et

servir les

clients (apregraves-

vente

reacuteclamations)

8 ndash Deacutevelopper et geacuterer les ressources humaines

9 ndash Geacuterer les systegravemes drsquoinformation

10 ndash Geacuterer les ressources financiegraveres et les actifs

11 ndash Appliquer un programme environnemental

12 ndash Geacuterer les relations exteacuterieures (actionnaires banques lois relations publiques hellip)

13 ndash Geacuterer lrsquoameacutelioration et le changement (eacutevaluer mesurer motiver qualiteacute totale)

Initiation au management copy CRCF ndash J Sornet Page 28 48

Fiche OP2 ndash Lrsquoorganisation par processus

Lrsquoeacutevolution drsquoune organisation aux activiteacutes reacutepeacutetitives vers lrsquoapproche processus est

geacuteneacuteralement progressive et se met en place par paliers

La mise en œuvre drsquoun veacuteritable management par processus doit ecirctre preacuteceacutedeacutee quand

lrsquoactiviteacute de lrsquoentreprise est complexe drsquoun recensement (une laquo cartographie des

processus raquo) pour mettre en eacutevidence les processus ou les familles de processus cleacutes critiques

pour le succegraves de lrsquoentreprise ougrave les efforts seront prioritaires

Des responsables de processus (laquo process owners raquo) sont ensuite deacutesigneacutes

Le responsable doit concevoir ses processus puis apregraves leur mise en œuvre assurer les

coordinations neacutecessaires les ameacuteliorer et les repreacutesenter aupregraves de la direction

Quand une structure par processus est mise en place des opeacuterateurs exeacutecutants

preacuteceacutedemment regroupeacutes dans les fonctions peuvent ecirctre affecteacutes aux processus et

drsquoanciens responsables de fonctions peuvent devenir des experts au service des processus

Lrsquoorganisation par processus peut imposer un degreacute eacuteleveacute drsquointeacutegration des activiteacutes donc

une polyvalence accrue des personnels et une reacuteduction des niveaux hieacuterarchiques

Elle neacutecessite pour le moins des compeacutetences eacutelargies au niveau des responsables de

processus (organisation administration technique hellip) dont le nombre doit rester limiteacute

(quelques dizaines au plus)

Sauf dans de tregraves petites structures lrsquoorganisation par processus se plaque geacuteneacuteralement sur

une structure plus classique

Initiation au management copy CRCF ndash J Sornet Page 29 48

ELEMENTS DE CORRIGE OP

OP1 Deacutefinir

Flexibiliteacute = adaptation au besoin (horaire variable chaicircnes robotiseacutees)

Systegraveme = ensemble organiseacute dans un but boicircte noire (sanguin nerveux meacutetrique laquo D raquo)

Impartition = sous-traitance ou externalisation (seacuteparation) drsquoactiviteacutes faire appel agrave des

partenaires plutocirct que faire soi-mecircme

OP2 Citer huit exemples drsquoinformations essentielles pour orienter lrsquooptimisation drsquoun processus

Montant des stocks (approvisionnements et produits finis)

Temps drsquoattente

Taux drsquoactiviteacute des ateliers

Rebuts

Deacutelai de production

Taux de reacuteclamations clients (qualiteacute)

Temps passeacutes en retouches finales

Turn over

Nombre drsquoarrecircts maladie

Accidents du travail

Dureacutee des arrecircts machines

OP3 Deacutegager les principes du toyotisme preacutesenteacute dans la fiche 31 En quoi ce systegraveme

repose trsquoil sur lrsquoapproche processus

Produire la quantiteacute juste neacutecessaire (agrave la demande) donc eacuteviter les stocks

Flexibiliteacute intelligence des chaicircnes de production

Qualiteacute (eacuteviter le coucirct de la non-qualiteacute)

La notion de processus est implicite ainsi que la chaicircne de valeur client

Initiation au management copy CRCF ndash J Sornet Page 30 48

DEFIS ET TENDANCES DU MANAGEMENT

Les meacutethodes de management se deacuteveloppent pour affronter le contexte eacuteconomique

Ce chapitre preacutesente les deacutefis auxquels le management contemporain doit faire face

1 ndash Lrsquoeacutevolution eacuteconomique contemporaine

A mesure que lrsquoactiviteacute eacuteconomique mondiale srsquoaccroicirct que la technologie eacutevolue les

changements sont de plus en plus rapides Ils introduisent des situations ineacutedites auxquelles les

entreprises doivent srsquoadapter en cherchant de nouvelles solutions de management Les trois

derniegraveres deacutecennies ont eacuteteacute notamment marqueacutees par les pheacutenomegravenes suivants (que nous

listons sans tenir compte des liens pouvant exister entre eux)

Pheacutenomegravene Traduction Effets

Deacute reacuteglementation

globalisation

financiegravere

titrisation

Libre circulation des capitaux accegraves

facile des particuliers au marcheacute

boursier (directement ou par

lrsquointermeacutediaire des OPCVM et SICAV)

Monteacutee en puissance du financement

des entreprises sur le marcheacute boursier

Fonds de pension

(retraites) et fonds

souverains (eacutetats)

Poids boursier important drsquoinvestisseurs

institutionnels qui cherchent un haut

rendement financier (dividendes ou

valorisation boursiegravere)

Pression sur les grandes entreprises

influence sur les strateacutegies

Mondialisation Liberteacute des eacutechanges internationaux Accroissement de la concurrence

recherche drsquoavantages eacuteconomiques

par la deacutelocalisation (biens et

services) la concentration des efforts

(recentrage) problegravemes drsquoemploi

multiplication des transports perte

drsquoinfluence des politiques

Baisse de lrsquoemploi

occidental

(notamment

industriel)

Moins de fabrications fabrications

automatiseacutees recours aux moyens

informatiques

Activiteacute reporteacutee sur le commerce la

conception et les services chocircmage

charge sociale

Restructurations Optimisation des entreprises

abaissement des coucircts augmentation

des marges recherche drsquoune taille

critique (eacuteconomies drsquoeacutechelle poids

sur le marcheacute)

Recentrages externalisations fusions

deacutelocalisations constitution de grands

groupes

NTIC (nouvelles

technologies de

lrsquoinformation et de

la communication)

Mise en œuvre des reacuteseaux (dont

internet) et drsquoapplications

informatiques communicantes

Nouvelles formes de commerce

marcheacute international deacutelocalisation

du travail intellectuel reacuteorganisation

de la distribution

Rareacutefaction relative

des matiegraveres

premiegraveres

Recherche de substituts exploration

miniegravere coucircts drsquoexploitation des

gisements accrus

Augmentation des coucircts variations

erratiques du cours des matiegraveres

deacutestabilisations politiques

Evolution

geacuteopolitique et

eacuteconomique

mondiale

Chute de lrsquoURSS transformation des

eacuteconomies collectivistes pays

eacutemergents (Chine Inde Breacutesil Russie)

()

Accroissement de la population

mondiale (4 agrave 6 7 milliards de 1970 agrave

2008)

Libeacuteralisme sans frein () nouvelles

puissances eacuteconomiques

opportuniteacutes de deacuteveloppement

nouveau partage des ressources

ineacutegaliteacutes baisse du soutien aux PVD

laquo Terrorisme raquo Actions armeacutees pression de groupes

armeacutes non gouvernementaux

Deacutestabilisations reacutegionales charge

des deacutepenses militaires

Deacuteveloppement

durable

Recherche drsquoune croissance eacutequitable

et respectueuse de lrsquoenvironnement

Pression sur les entreprises (eacutetats

associations de consommateurs

eacutecologistes ONG)

() Reacutecemment quelques affaires (Enron laquo subprimes raquo Vivendi Universal Socieacuteteacute

Geacuteneacuterale Airbus par exemple) et agrave plus grande eacutechelle la crise financiegravere de 2008 ont

montreacute les dangers drsquoune libeacuteralisation sans controcircles suffisants

Initiation au management copy CRCF ndash J Sornet Page 31 48

() Des alliances eacuteconomiques naissent entre pays eacutemergents (notamment en

ameacuterique centrale creacuteation de la Banque du Sud en 2008 par exemple) et lrsquoon

commence agrave imaginer une baisse progressive de lrsquoinfluence eacuteconomique des Etats

Unis

2 ndash Les deacutefis actuels du management

21 ndash Les grandes orientations

Lrsquoeacutevolution eacuteconomique suggegravere quelques pistes parfois concurrentes pour lrsquoaction du

manager contemporain On y retrouve au premier plan la construction drsquoune vision qui est

une composante commune du leadership

Objectif du manager

pour lrsquoentreprise

Justification Facteurs de reacuteussite

Construire une vision Eclairer lrsquoavenir de lrsquoentreprise partager

un but souder motiver

Effort de reacuteelle prospection

volontarisme de la direction

bonne communication

Reacuteactiviteacute et flexibiliteacute

(sous tous les aspects

agrave tous niveaux)

Srsquoadapter rapidement au marcheacute Bonne organisation des processus

personnel compeacutetent autonome

et motiveacute structure hieacuterarchique

alleacutegeacutee robotisation

Deacutegager des profits Reacutemuneacuterer les apporteurs de capitaux

srsquoautofinancer

Ajuster coucircts et structures

Exploiter les nouvelles

technologies

Reacuteactiviteacute ajuster coucircts et deacutelais

reacutepondre au marcheacute suivre les clients

Organiser le SI de faccedilon

pertinente eacuteviter le coucirct excessif

drsquoinvestissements trop en

laquo pointe raquo (laquo essuyer les placirctres raquo)

utiliser judicieusement les services

exteacuterieurs

Bacirctir des alliances

(contrats fusions)

Deacutevelopper une activiteacute limiter les coucircts

de transaction () atteindre la taille

critique et de meilleurs rendements se

recentrer sur une activiteacute profitable

Dominer les processus se donner

une identiteacute lisible externaliser se

doter drsquoune capaciteacute financiegravere

suffisante

Valoriser lrsquoimage Attirer les clients favoriser les alliances

donner confiance (apporteurs de fonds

employeacutes clients partenaires socieacuteteacute

civile)

Instaurer des regravegles de

gouvernance inteacutegrer le

deacuteveloppement durable

respecter lrsquoenvironnement

Geacuterer les risques Faire face aux aleacuteas eacuteconomiques et

technologiques (conjoncture politiques

accidents malveillance)

Creacuteer un systegraveme drsquoalerte geacuterer

la crise (reacuteaction raisonneacutee

sceacutenarios poursuite de

lrsquoexploitation dans un contexte

instable) mise en place de

proceacutedures drsquoapprentissage pour

ameacuteliorer les reacuteactions au fil du

temps

Geacuterer le changement Faire face agrave lrsquoeacutevolution de la demande

la pression sur les prix la variation des

performances financiegraveres la

concurrence la globalisation des

marcheacutes lrsquoeacutevolution technologique aux

fusions ou alliances aux changements

de reacuteglementation de direction hellip ()

Bonne communication pour

donner du sens au changement

et obtenir lrsquoadheacutesion du personnel

Rassembler et geacuterer les

connaissances former le

personnel

Innover Garder un avantage concurrentiel se

diffeacuterencier

Veille technologique et

commerciale investissement

Ouverture

internationale

Elargir le marcheacute saisir les opportuniteacutes Veille commerciale partenariats

() La theacuteorie des coucircts de transaction deacuteveloppeacutee par OE Williamson dans les

anneacutees 70 integravegre les coucircts lieacutes au recours au marcheacute (recherche et choix drsquoun

fournisseur neacutegociation reacutedaction de contrat suivi des eacutechanges risque de rupture

Initiation au management copy CRCF ndash J Sornet Page 32 48

drsquoapprovisionnement hellip) On peut en conclure que lrsquointeacutegration de diffeacuterentes

activiteacutes agrave lrsquoentreprise (la laquo firme raquo) preacutesente des avantages Mais des coucircts de

transaction internes doivent aussi ecirctre consideacutereacutes (preacuteparation organisation

surveillance hellip) et certaines formes de coopeacuteration continue avec les fournisseurs

permettent de reacuteduire le coucirct des transactions externes

() drsquoapregraves laquo Les meilleures pratiques de management raquo - Brilman Heacuterard ndash EO

Une eacutetude du Conference Board (2002) liste les deacutefis du management vus par 700 leaders

mondiaux Soit en reacutesumeacute avec indication du score correspondant

1 ndash Fideacuteliser les clients (42)

2 ndash Reacuteduire les coucircts (38)

3 ndash Accroicirctre flexibiliteacute et reacuteactiviteacute (29)

4 ndash Amener les employeacutes agrave adheacuterer aux valeurs et visions de lrsquoentreprise (26)

5 ndash Deacutevelopper et retenir les leaders (25)

6 ndash Geacuterer acquisitions et alliances (24)

7 ndash Accroicirctre lrsquoinnovation (20)

En fin de classement citoyenneteacute et reacuteputation (4) et ameacutelioration de la diversiteacute (3)

22 ndash Les techniques disponibles

Pour faire face aux deacutefis le manager dispose de nouveaux concepts et de nouvelles

techniques Le tableau ci-dessous en donne un reacutesumeacute et indique les domaines qursquoils

influencent principalement

Initiation au management copy CRCF ndash J Sornet Page 33 48

Principaux concepts techniques outils Incidence principale sur

Internet

- e-commerce (commerce eacutelectronique site

entreprise)

- CRM ou GRC (gestion de la relation client)

- e-procurement (gestion des approvisionnements

par le reacuteseau)

- messagerie eacutelectronique

- e-recrutement

Vente accegraves au marcheacute

Relation client reacuteactiviteacute personnalisation

fideacutelisation

Deacutelais coucircts

Communication transfert de donneacutees (piegraveces

jointes) tous domaines

Communication recrutement

Intranet reacuteseau drsquoentreprise SI

- knowledge management (gestion des

connaissances)

- e-learning (apprentissage en ligne)

- plateforme de travail collaboratif (groupware)

- workflow (circulation eacutelectronique de

documents enchaicircnement de processus)

- e-RH portail RH (libre accegraves aux postes agrave

pourvoir informations candidatures hellip)

- PGI (progiciel de gestion inteacutegreacute) ou ERP

Innovation capaciteacute au changement veille

documentaire

Formation du personnel accompagnement des

changements

Coordination communication interne

Coordination

Communication interne (voire internet en

externe) reacuteduction des coucircts climat drsquoentreprise

recrutement plans de carriegraveres hellip

Coucircts fiabiliteacute du systegraveme drsquoinformation deacutelais

processus (continuiteacute inteacutegration)

Logistique inteacutegreacutee

Supply Chain Management (SCM) gestion de la

logistique (incluant les approvisionnements)

Processus deacutelais coucircts

Externalisation

Valorisation du capital humain

GPEC (gestion preacutevisionnelle des emplois et

compeacutetences)

Coaching

Reacuteactiviteacute de lrsquoentreprise conservation des

compeacutetences rendements individuels turn-over

adaptation des compeacutetences motivation

Efficaciteacute individuelle controcircle reacutegulation

progregraves processus

Approche processus

Optimisation des processus

Deacutemarche qualiteacute totale (TQM ndash total quality

management)

Empowerment (empouvoirement)

Benchmarking reacuteingeacutenieacuterie

Coucircts marges qualiteacute deacutelais flexibiliteacute

externalisation eacutelargissement des compeacutetences

organisation

Ameacutelioration des processus (meacutetiers et supports)

Autonomie compeacutetences des employeacutes

Ameacutelioration des processus restructuration

Management par la valeur

Parties prenantes

Satisfaction des parties prenantes financement

motivation collaborations hellip

Collaboration inter organisations

Reacuteseaux drsquoentreprises alliances

EDI (eacutechange de donneacutees informatiseacutees) extranet

Impartition externalisation (outsourcing)

Coucircts recentrage investissements lancement

drsquoactiviteacute

Coucircts reacuteactiviteacute deacutelais relations avec

lrsquoadministration

Coucircts recentrage limitation des investissements

Ethique drsquoentreprise

Gouvernance drsquoentreprise (mode de direction

encadreacute par des regravegles)

Rocircle socieacutetal deacuteveloppement durable

environnement

Image de lrsquoentreprise reacutegulation du top

management relations actionnaires

Image peacutenaliteacutes et amendes objectifs

strateacutegiques

Initiation au management copy CRCF ndash J Sornet Page 34 48

23 ndash Le rocircle socieacutetal des entreprises

La responsabiliteacute socieacutetale de lrsquoentreprise (RSE) deacutesigne le rocircle qursquoelle prend dans la socieacuteteacute

au-delagrave de son activiteacute purement geacuteneacuteratrice de profit On parle aussi drsquoentreprise citoyenne

La RSE est indissociable du deacuteveloppement durable de porteacutee mondiale et dont les trois

piliers sont

- eacuteconomique (favoriser le deacuteveloppement les eacutechanges internationaux)

- social (accegraves aux soins eacuteducation conditions de travail hellip)

- environnemental (pollution preacuteservation des ressources hellip)

La RSE integravegre notamment une preacuteoccupation sociale de lrsquoentreprise vis-agrave-vis de ses salarieacutes

(seacutecuriteacute et santeacute au travail juste reacutemuneacuteration deacuteveloppement personnel hellip) Elle conduit agrave

tenir compte dans le management drsquoune vision exteacuterieure agrave lrsquoentreprise qui peut avoir des

reacutepercussions possibles sur son activiteacute eacuteconomique

Lrsquoentreprise peut aussi tirer avantage drsquoune deacutemarche responsable par la baisse de certains

coucircts (plus faibles consommations drsquoeacutenergies reacuteduction des transports hellip)

Le rocircle socieacutetal de lrsquoentreprise a eacuteteacute reconnu en France par la loi laquo NRE raquo de 2001 (loi sur les

nouvelles reacutegulations eacuteconomiques) qui oblige les socieacuteteacutes franccedilaise coteacutees sur un marcheacute

reacuteglementeacute agrave rendre compte dans leur rapport annuel de leur gestion sociale et

environnementale au travers de leur activiteacute

Article 116 de la loi Le rapport viseacute agrave larticle L 225-102 rend compte hellip laquo Il comprend

eacutegalement des informations dont la liste est fixeacutee par deacutecret en Conseil dEtat sur la

maniegravere dont la socieacuteteacute prend en compte les conseacutequences sociales et

environnementales de son activiteacute Le preacutesent alineacutea ne sapplique pas aux socieacuteteacutes

dont les titres ne sont pas admis aux neacutegociations sur un marcheacute reacuteglementeacute raquo

Une norme ISO 14000 integravegre ces preacuteoccupations et des taxes eacutecologiques sont

progressivement creacutees

3 ndash Le management par la valeur

31 ndash De lrsquoanalyse au management par la valeur

Lrsquoanalyse de la valeur est neacutee en 1947 aux Etats-Unis (General Electrics) Cette technique

consiste agrave eacutelaborer des produits conformes aux attentes de la clientegravele mais sans excegraves pour

trouver un bon compromis entre valeur pour le client et coucirct Le produit optimal est deacutefini agrave

partir drsquoenquecirctes qui deacuteterminent le besoin client (ou plutocirct drsquoun client laquo type raquo)

Exemple il est inutile de concevoir un petit veacutehicule citadin capable de parcourir

500 000 km sans avarie compte tenu des effets de mode et du faible kilomeacutetrage

annuel Par contre le marcheacute peut exiger un fonctionnement sans faille sur 150 000 km

soit dix ans en moyenne ce qui conditionne les coucircts de production

Cette recherche drsquoun ajustement de valeur au besoin des clients eacutetait un preacutecurseur du

management par la valeur qui recherche plus largement la creacuteation de valeur pour

chacune des parties prenantes de lrsquoentreprise tout en lui meacutenageant un reacutesultat suffisant

Plus geacuteneacuteralement le management par la valeur est deacutefini par une norme europeacuteenne (EN

12973)

Le management par la valeur est un style de management particuliegraverement destineacute agrave

mobiliser les individus agrave deacutevelopper les compeacutetences et agrave promouvoir les synergies et

Initiation au management copy CRCF ndash J Sornet Page 35 48

linnovation avec pour objectif la maximisation de la performance globale dun

organisme Le management par la valeur apporte une nouvelle faccedilon dutiliser nombre

de meacutethodes de management existantes Il est en coheacuterence avec le Management

de la qualiteacute

Cette approche du management pose de nombreuses questions notamment quelles

prioriteacutes et quelles valeurs attribuer aux parties prenantes comment appreacutehender la

perception par les parties prenantes de la valeur qui leur est affecteacutee

32 ndash La valeur client

Le processus drsquoeacutelaboration drsquoun produit qui consomme des ressources coucircteuses doit creacuteer

une valeur suffisante pour provoquer lrsquoachat par le client final La production drsquoune valeur

reconnue par le client est vitale pour lrsquoentreprise mais sa deacutetermination est parfois complexe

La valeur du produit perccedilue par le client integravegre des eacuteleacutements en partie subjectifs

- une valeur drsquousage (le produit reacutepond agrave un besoin)

- une valeur drsquoestime (lrsquoimage apporteacutee par le produit un aspect affectif)

- une valeur drsquoeacutechange (deacuteduite de lrsquoespoir de revente du produit)

Valeurs drsquousage drsquoestime et drsquoeacutechange deacutependent implicitement de la qualiteacute (un bien peu

fiable est impropre agrave lrsquousage attendu de mauvaise qualiteacute notoire il nrsquoapporte pas une

image positive et ses deacutefauts connus nuisent agrave sa revente) Une eacutevaluation de la qualiteacute

intervient donc dans la valeur perccedilue du produit

Par ailleurs le client considegravere le coucirct drsquoobtention du produit (les charges qursquoil doit supporter

pour acqueacuterir le produit lrsquoeffort qursquoil doit faire pour trouver le produit et les frais de mise agrave

disposition)

Le prix perccedilu par le client est geacuteneacuteralement supeacuterieur au prix de vente

Le client achegravete theacuteoriquement le produit qui preacutesente la diffeacuterence valeur perccedilue ndash prix

perccedilu la plus favorable ou le meilleur rapport prix perccedilu qualiteacute perccedilue et dans certains

cas celui qui a le prix produit le plus bas

Remarque les valeurs du scheacutema ci-dessus changent durant le cycle de vie du produit

(un nouveau produit peut avoir une valeur perccedilue plus eacuteleveacutee qursquoen fin de vie) La

valeur client ne peut ecirctre eacutevalueacutee que par enquecirctes et ne peut donc ecirctre deacutefinie avec

certitude

La notion de laquo satisfaction client raquo conseacutecutive agrave une vente influence aussi le prix produit et

le prix perccedilu

- lrsquoentreprise gagne sur les coucircts de recherche de clientegravele

- le client nrsquoa pas agrave rechercher un nouveau fournisseur et beacuteneacuteficie drsquoun coucirct drsquoobtention

plus bas

valeur perccedilue client

prix perccedilu client

coucirct produit Marge (valeur creacuteeacutee pour

lrsquoentreprise)

euros

prix produit

Valeur creacuteeacutee

pour le client

Initiation au management copy CRCF ndash J Sornet Page 36 48

La satisfaction du client deacutepend de facteurs qualitatifs aussi divers que la fiabiliteacute du produit

la vitesse de reacuteaction du fournisseur lrsquoattitude des commerciaux lrsquoefficaciteacute du service

apregraves-vente la netteteacute des contrats ou la justesse de la facture

Valeur perccedilue coucirct marge et satisfaction reacutesultent de processus allant de la conception du

produit jusqursquoagrave sa livraison et son apregraves-vente La deacutemarche laquo processus raquo et lrsquolaquo analyse de la

valeur raquo en forccedilant la recherche de solutions efficientes agrave tout niveau administratif

technique commercial et apregraves-vente sont donc neacutecessaires pour bien positionner

lrsquoentreprise sur son marcheacute

Pour autant le risque commercial ne peut jamais ecirctre annuleacute et lrsquooffre de lrsquoentreprise ne

satisfait geacuteneacuteralement pas en milieu concurrentiel tous ses clients potentiels

33 - La creacuteation de valeur pour les autres parties prenantes

Les salarieacutes

La creacuteation drsquoune valeur suffisante pour les salarieacutes est reconnue comme neacutecessaire car des

observations montrent que la satisfaction des clients en deacutepend Moins souvent eacutevoqueacutee en

peacuteriode de chocircmage elle nrsquoest prioritaire que pour les employeacutes dont lrsquoentreprise souhaite

conserver les compeacutetences

La laquo valeur salarieacute raquo ne comprend pas que le salaire Le sentiment drsquoappartenance agrave un

groupe la reconnaissance lrsquoaccomplissement de soi et la construction professionnelle en

sont des eacuteleacutements importants Comme pour les clients on doit ainsi distinguer la reacutetribution

perccedilue du salaire objectif

Les actionnaires

Lrsquoactionnaire apporte des fonds propres agrave lrsquoentreprise en contrepartie de titres parfois

neacutegociables en bourse et assortis drsquoun droit de vote en assembleacutee geacuteneacuterale La valeur

attribueacutee aux actionnaires est servie en termes moneacutetaires (dividende ou augmentation de la

valeur du titre neacutegociable)

Remarque des facteurs non moneacutetaires comme lrsquoimage de lrsquoentreprise qui deacutepend

en partie de sa communication peuvent influencer la deacutecision drsquoachat de vente ou

de conservation des titres par lrsquoactionnaire

Reacutetribution perccedilue euros

Salaire objectif

Avantage non

moneacutetaire de

lrsquoemploi

Initiation au management copy CRCF ndash J Sornet Page 37 48

Compte tenu de lrsquoimportance croissante de lrsquoactionnariat dans le financement des grandes

entreprises coteacutees en bourse et notamment des investisseurs institutionnels comme les fonds

de pension des indicateurs speacutecifiques ont eacuteteacute introduits pour appreacutecier la performance des

entreprises vue par les actionnaires Par exemple la valeur ajouteacutee eacuteconomique (EVA reg

economic value added marque deacuteposeacutee de Stern Stewart ou VAE ndash valeur ajouteacutee

eacuteconomique parfois deacutenommeacutee VEC ndash valeur eacuteconomique creacuteeacutee) qui prend en compte le

coucirct du capital

LrsquoEVA correspond tregraves scheacutematiquement au calcul suivant

EVA = (PO) profit opeacuterationnel ndash (C) coucirct du capital X (CE) capitaux employeacutes

LrsquoEVA neacutecessite en pratique des retraitements assez complexes Le PO peut se deacuteterminer

selon les principes suivants

- PO = reacutesultat drsquoexploitation (avant inteacuterecircts) ndash impocirct

- PO = beacuteneacutefice courant (tenant compte des inteacuterecircts) + inteacuterecircts ndash eacuteconomie drsquoimpocirct sur les

inteacuterecircts (on exclue les eacuteleacutements financiers et lrsquoimpocirct correspondant) ndash impocirct

- lrsquoimpocirct pris en compte correspond au profit opeacuterationnel consideacutereacute (dans les cas courants agrave

13 du PO)

C = taux moyen de reacutemuneacuteration du capital (reacutesultant par exemple du dividende exigeacute de

certains investisseurs et des taux drsquoemprunts bancaires)

CE = capitaux propres et dettes portant inteacuterecirct

Remarque le profit opeacuterationnel ou reacutesultat opeacuterationnel correspond au NOPAT ndash net

operating profit after tax - anglo-saxon LrsquoEVA est eacutegale au NOPAT diminueacute de la

reacutemuneacuteration des capitaux

Exemple lrsquoentreprise X dispose drsquoun capital de 2 500 000 euro et reacutealise un beacuteneacutefice net

drsquoimpocirct de 450 000 euro (taux 33 13) Un dividende de 6 doit ecirctre verseacute aux

actionnaires et la banque lui a accordeacute un precirct de 1 200 000 euro agrave 4 Les autres

constituants des reacutesultats financier et exceptionnel sont neacutegligeables

Reacutesultat opeacuterationnel = 450 000 + 004 x 1 200 000 x 23 = 482 000 euro

Coucirct du capital = 006 x 2 500 000 + 004 x 1 200 000 x 23 = 182 000 euro

EVA = 300 000 euro

Coucirct moyen pondeacutereacute du capital (C) = (004 x 1 200 000 x23 + 006 x 2 500 000)

3 700 000 Soit 492

Si lrsquoEVA est positive lrsquoentreprise creacuteeacutee de la valeur apregraves reacutemuneacuteration des capitaux et sa

valeur boursiegravere doit augmenter

Lrsquoutilisation de lrsquoEVA comme indicateur influence le management de lrsquoentreprise car il y a

trois moyens pratiques drsquoaugmenter lrsquoEVA

- augmenter le reacutesultat opeacuterationnel

- lancer des investissements ayant une rentabiliteacute supeacuterieure agrave C

- eacuteliminer les activiteacutes ayant une rentabiliteacute infeacuterieure agrave C

Remarque lrsquoutilisation sans nuance de lrsquoEVA comme critegravere de management peut

poser problegraveme Le calcul de lrsquoEVA repose sur des ajustements comptables il est donc

sujet agrave manipulations (provisions capitalisation ou non de la RD hellip) Par ailleurs le

critegravere laquo EVA raquo pris isoleacutement peut conduire agrave chercher la rentabiliteacute agrave court terme agrave

reacuteduire les investissements prospectifs et donc nuire agrave terme au deacuteveloppement de

lrsquoentreprise

Initiation au management copy CRCF ndash J Sornet Page 38 48

Les fournisseurs reccediloivent le paiement de leurs factures plus ou moins rapidement (le deacutelai

de paiement repreacutesente une valeur consentie au fournisseur)

Lrsquoentreprise peut accroicirctre la valeur apporteacutee agrave ses fournisseurs par des actions cibleacutees

comme une contribution agrave la formation de leurs personnels certains transferts de

technologie ou de savoir faire agrave des sous-traitants une coopeacuteration suivie favorisant leur

deacuteveloppement lrsquointeacutegration agrave des campagnes de promotion

A noter que la valeur consentie aux fournisseurs peut avoir une influence sur la qualiteacute et les

deacutelais de livraison des produits

La collectiviteacute reccediloit des taxes et parfois des prestations en nature par deacutefaut ou explicites

(effort de preacuteservation de lrsquoenvironnement ameacutenagement du territoire par les implantations

aide mateacuterielle agrave des projets participation agrave la formation par exemple)

APPLICATIONS DT

DT1 Deacutefinir expliquer deacutereacuteglementation socieacutetal eacuteconomies drsquoeacutechelle coaching EDI

gouvernance

DT2 Deacuteterminer en quoi la deacutemarche TQM srsquoinscrit dans les deacutefis actuels du management

DT3 Apregraves avoir consulteacute les documents ci-dessous extraits du site drsquoAir France

(httpdeveloppement-

durableairfrancecomFRfrlocaldemarcheN4_positionnement_pphtm)

exposer les enjeux et les limites de la RSE et de la gestion des parties prenantes

Initiation au management copy CRCF ndash J Sornet Page 39 48

Dialogue avec les parties prenantes

Initiation au management copy CRCF ndash J Sornet Page 40 48

Attentes des parties prenantes

Initiation au management copy CRCF ndash J Sornet Page 41 48

Creacuteation de valeur pour les parties prenantes

La creacuteation de valeur pour les parties prenantes est au cœur de la strateacutegie du Groupe Le scheacutema de

distribution financiegravere ci-dessous donne un aperccedilu de la distribution des recettes du Groupe aux

diffeacuterentes parties prenantes actionnaires collaborateurs fournisseurs pouvoirs publics

collectiviteacutes locales etc

Initiation au management copy CRCF ndash J Sornet Page 42 48

Fiche DT1 ndash Extrait du sommaire de laquo Problegravemes eacuteconomiques raquo No 2894

La gestion des entreprises bouleverseacutee par les technologies de linternet

Reacutealiteacutes industrielles - Annales des Mines Jean-Michel Yolin

Avec lavegravenement de linternet les processus de conception de production et de vente sont

radicalement remis en cause Quel que soit le secteur dactiviteacute les technologies de linternet

permettent en effet de reacuteduire les deacutelais et de passer dun processus discontinu agrave un processus

continu Lorganisation des entreprises et leur mode de gestion en sont profondeacutement bouleverseacutes

tant au niveau individuel que collectif Linternet rend ainsi possible la reacutealisation dobjectifs que les

entreprises cherchaient agrave atteindre depuis longtemps sans y parvenir meilleure eacutecoute du client

travail sans stocks en flux tendu hieacuterarchies plates autorisant une grande reacuteactiviteacute flexibiliteacute dans

lorganisation et loutil de production acceacuteleacuteration du renouvellement des produits entreprises en

reacuteseau ougrave chacune se recentre sur son cœur de meacutetier etc

Le laquo knowledge management raquo ou comment geacuterer les connaissances

Document de travail du LAMSADE - Michel Grundstein

Peter Drucker lavait preacutedit le capital immateacuteriel eacutetait voueacute agrave devenir un facteur de compeacutetitiviteacute

pour lentreprise La libeacuteralisation des eacutechanges acceacutelegravere les processus de deacutecision de lentreprise

et implique que lassimilation des informations soit agrave la fois de meilleure qualiteacute et plus rapide Ainsi

la fonction qui consiste agrave manager les connaissances au sein de lentreprise savegravere primordiale

Bien que la prise de conscience de limportance du capital immateacuteriel ait eacuteteacute tardive - le concept

de knowledge management est apparu en France aux Etats-Unis et au Japon au milieu des

anneacutees 1990 - agrave lheure actuelle lorganisation de leacutechange dinformations et le partage des

connaissances sont devenus des facteurs cleacutes dune gestion performante de lentreprise Ils

doivent sinscrire dans un projet global destineacute agrave mettre en valeur les savoirs et les savoir-faire

individuels et collectifs

Les leccedilons du laquo coaching raquo pour le management de la qualiteacute

Humanisme et Entreprise - Martine Brasseur

Parmi les nouvelles formes de management en vogue dans les entreprises le coaching figure en

bonne place Appliqueacute au management de la qualiteacute il sagit dune pratique

daccompagnement destineacutee agrave initier et agrave faciliter le processus de deacuteveloppement dun individu

La deacutemarche consiste agrave affirmer que tout individu est en quecircte de qualiteacute agrave condition toutefois

de ne pas lui imposer des contraintes lempecircchant de progresser On considegravere notamment les

erreurs comme potentiellement feacutecondes En deacutefinitive le coach donne au coacheacute la permission

de reacuteussir en lui donnant aussi la permission deacutechouer

Initiation au management copy CRCF ndash J Sornet Page 43 48

Fiche DT2 ndash Management strateacutegique les sept deacutefis agrave relever dici agrave 2016

Extrait drsquoun article du site wwwlentreprisecom -Sabine Blanc - Mis en ligne le 20032007

(httpwwwlentreprisecom325article11977html)

Une eacutetude anglaise publieacutee par lopeacuterateur Orange Grande-Bretagne deacutecrypte la mutation

des formes de travail et les enjeux majeurs pour les entreprises de demain afin decirctre au top

de la compeacutetitiviteacute Voici les challenges-cleacutes pour les managers qui veulent rester dans la

course hellip

1 - Future organisation du travail les quatre laquo mondes raquo possibles

La reacutealiteacute sera probablement un meacutelange de ces quatre sceacutenarios souligne lrsquoeacutetude

Les mondes mutuels Tout se passe dans le cadre des communauteacutes locales vie priveacutee

comme professionnelle Le modegravele coopeacuteratif preacutevaut au lieu du laquo big business raquo Oublieacutes

aussi dans ce systegraveme les trajets pour aller au bureau les gens preacutefegravereront travailler dans de

petites entreprises locales souvent connecteacutees au reacuteseau drsquoautres structures similaires

Les laquo reacutepondants raquo (en anglais laquo replicants raquo) La figure du consultant freelance deviendra

dominante tandis que celle du salarieacute deacuteclinera Il ne sera pas rare de travailler pour plusieurs

entreprises On perdra en seacutecuriteacute de lrsquoemploi en visibiliteacute et en routine ce que lrsquoon gagnera

en liberteacute La majeure partie des tacircches srsquoeffectuera chez soi avec la possibiliteacute de srsquoinstaller

temporairement dans les bureaux de son client du moment Dans un contexte dincertitude

sur lrsquoavenir les travailleurs alterneront peacuteriodes drsquoactiviteacute intense et repos Ce sera agrave eux

drsquoaller vers les entreprises et non lrsquoinverse mecircme si celles-ci devront veiller agrave rester attractives

Les cottages eacutelectroniques Comme ce nom le suggegravere le teacuteleacutetravail deviendrait la norme

univers priveacute et professionnel se confondant Plus besoin de subir une heure de transport les

salarieacutes se logueront de chez eux sur le reacuteseau de lrsquoentreprise Les reacuteunions se tiendront dans

de petits bureaux centraux situeacutes agrave courte distance La flexibiliteacute du temps de travail srsquoimpose

Les salarieacutes disposeront de plus de marge de liberteacute dans leur activiteacute

Les disciples de la nueacutee Cette appellation poeacutetique cache simplement une extension de

lrsquoorganisation actuelle des grandes entreprises avec des salarieacutes se rendant sur un lieu de

travail centraliseacute Le rocircle croissant des technologies de lrsquoinformation multipliera les faccedilons de

collaborer et accroicirctra lrsquoefficaciteacute Le controcircle du travail sera omnipreacutesent La frontiegravere entre

travail et vie priveacutee restera marqueacutee

2 - Sept deacutefis pour les entreprises et leur managers

Quoi qursquoil advienne les entreprises et leurs dirigeants devront concentrer leurs efforts sur sept

points-cleacutes pour srsquoadapter Voici quelques exemples de probleacutematiques souleveacutees par le

rapport et des pistes de solution

Le leadership Les managers devront entre autres savoir persuader et influencer des

travailleurs beaucoup plus indeacutependants Ils auront aussi agrave repenser les niveaux auxquels

prendre les deacutecisions strateacutegiques en haut ou au contraire agrave des degreacutes moins eacuteleveacutes de la

pyramide hieacuterarchique

gt Faire du management une force facilitant les activiteacutes transversales plutocirct que la reacuteduire agrave

la seule fonction de deacutecision

La culture drsquoentreprise Davantage de salarieacutes capables de reacutefleacutechir seront neacutecessaires

tandis que les tacircches qui peuvent ecirctre automatiseacutees ou scripteacutees diminueront Un des

enjeux creacuteer une culture agrave mecircme drsquoattirer et drsquoencourager les personnes preacutesentant ces

qualiteacutes de reacuteflexion requises dans un contexte de compeacutetition accrue et de plus grande

indeacutependance des travailleurs

Initiation au management copy CRCF ndash J Sornet Page 44 48

gt Passer si neacutecessaire drsquoune culture drsquoentreprise forte agrave un mode drsquoengagement plus

consensuel moins rebutant

La marque Conseacutequence du recours croissant agrave lrsquo laquo outsourcing raquo lrsquoimage drsquoune marque

deacutependra plus drsquoagents exteacuterieurs qui ne fonctionnent pas forceacutement selon le mecircme mode

drsquoorganisation Comment garder le controcircle dessus

gt Choisir le mode qui corresponde le plus agrave vos valeurs et preacutevoir un programme de risk

management qui mette en eacutevidence ougrave les conflits sont susceptibles de jaillir

Lrsquoinnovation Plus que jamais il faudra faire face agrave une acceacuteleacuteration du rythme de

lrsquoinnovation en proposant constamment des solutions adapteacutees

gt Tisser des partenariats strateacutegiques avec drsquoautres entreprises pour partager les coucircts et les

fruits de lrsquoinnovation

Le deacutefi opeacuterationnel et technologique De quelle faccedilon controcircler lrsquoinformation crsquoest-agrave-dire

faire en sorte que les bonnes personnes accegravedent facilement agrave une information toujours en

phase tout en maintenant la seacutecuriteacute

gt Recourir agrave des laquo feuilles de route des futurs raquo syntheacutetisant en une page les indicateurs

sociaux et de consommation ainsi que les eacutevolutions technologiques et leacutegislatives qui

influent sur les changements et indiquant comment ils modifient vos marcheacutes vos clients et

votre organisation

La qualiteacute Si de nouveaux proceacutedeacutes ont pu deacutegrader la qualiteacute comme le recours agrave des

centres drsquoappel externaliseacutes drsquoautres ideacutees se sont reacuteveacuteleacutees plus prometteuses comme en

teacutemoigne le succegraves de certaines compagnies aeacuteriennes low cost Elles ont su conjuguer prix

serreacutes et services eacuteleveacutes ce qui devra devenir la norme estime lrsquoeacutetude

gt Continuer de rechercher la qualiteacute Elaborez aussi une bonne prestation service qui inclut

une livraison de qualiteacute voire creacuteez-la en partenariat avec les consommateurs

La leacutegislation La question de la proprieacuteteacute intellectuelle pourrait ecirctre probleacutematique Elle est

deacutejagrave source de conflits comme en teacutemoigne le procegraves pour violation de brevet intenteacute agrave RIM

le fabricant canadien du Blackberry par NTP Que pourra-t-on et que faudra-t-il proteacuteger par

un brevet Il sera eacutegalement neacutecessaire drsquoadapter la leacutegislation aux nouveaux modes

drsquoorganisation

gt Collaborer avec les acteurs du mecircme secteur et les leacutegislateurs pour deacutevelopper les

modegraveles des lieux de travail du futur et bacirctir le droit le plus adeacutequat

Orange a-t-il vu juste dans ses preacutevisions Rendez-vous dans neuf ans pour la reacuteponsehellip

Initiation au management copy CRCF ndash J Sornet Page 45 48

Fiche DT3 ndash Le management par la qualiteacute totale

Extrait drsquoune lettre drsquoinformation du cabinet Baud Accordance Consulting AD2 consultants ndash

2002

1 - Le TQM (Total Quality Management) offre pour lentreprise une vision de la qualiteacute plus

large et transversale

Son principe est simple La finaliteacute de lEntreprise est de deacutevelopper la satisfaction de ses

clients tout en eacutetant beacuteneacuteficiaire cest agrave dire pas agrave nimporte quel prix Elle doit ameacuteliorer sa

rentabiliteacute au travers de la deacutemarche qualiteacute La Qualiteacute Totale vise agrave fournir aux clients

externes et internes une reacuteponse adeacutequate agrave leurs attentes dans le meilleur rapport qualiteacute

prix la meilleure efficience

Elle considegravere pour cela lensemble des processus de lentreprise ayant une incidence sur la

qualiteacute et la satisfaction des clients

Le TQM fait ainsi une large place agrave

la deacutefinition et la planification de la strateacutegie geacuteneacuterale

la coheacuterence de la politique qualiteacute avec la strateacutegie

la deacutemultiplication de la politique qualiteacute dans toutes les directions de lentreprise

la relation client fournisseur interne

la prise en compte de lenvironnement concurrentiel

la consideacuteration de lensemble des risques potentiels financiers sociaux concurrentielshellip

limplication et la motivation du personnel

lanalyse des besoins des clients et le positionnement marketing

la maicirctrise des processus transverses internes

les reacutesultats sous tous ses aspects y compris financiers commerciaux image

De nombreux reacutefeacuterentiels sont relatifs agrave la Qualiteacute Totale hellip Tous ces reacutefeacuterentiels imposent un

questionnement plus profond et indiscret sur le mode de fonctionnement de lentreprise et

son management

helliphellip

2 - LISO 9001 2000 au travers du deacuteploiement des processus (management supports

reacutealisation et ameacutelioration continue) reacutepond quelque peu agrave la mecircme logique

LISO est une ouverture indeacuteniable vers la logique du TQM mais ne se reacutefegravere pas agrave la notion

defficience

Les dirigeants sont cependant sensibles agrave la neacutecessaire reacuteduction des coucircts de non-qualiteacute

et dobtention de la qualiteacute agrave la rentabiliteacute du systegraveme de management de la qualiteacute

mais ne perccediloivent pas toujours la qualiteacute comme une deacutemarche globale

Les deacutemarches qualiteacute commencent bien souvent par la remise en cause de lorganisation

leacutevaluation critique de son efficaciteacute lexamen des processus et la mise en eacutevidence des

lourdeurs administratives

La qualiteacute devient laffaire de tous hellip

Initiation au management copy CRCF ndash J Sornet Page 46 48

Fiche DT4 ndash Le deacuteveloppement durable et la RSE

Extrait du site wwwvigeocom

(httpwwwvigeocomcsr-rating-agencyfrmethodologiecriteres-de-recherche37-

criteres-d-analysehtml)

Deacuteveloppement durable laquo un deacuteveloppement qui reacutepond aux besoins du preacutesent sans compromettre

la capaciteacute des geacuteneacuterations futures de reacutepondre aux leurs raquo (Commission mondiale sur lrsquoenvironnement

et le deacuteveloppement ndash 1987)

Reacutefeacuterentiel drsquoeacutevaluation des entreprises par le groupe Vigeacuteo (le groupe mesure les performances et le

niveau de maicirctrise des risques de responsabiliteacute sociale des entreprises et des organisations - site

wwwvigeocom)

1 Ressources Humaines Ameacutelioration continue des relations professionnelles des relations drsquoemploi et des conditions de travail 2 Droits humains sur les lieux de travail Respect de la liberteacute syndicale et promotion de la neacutegociation collective non discrimination et promotion de lrsquoeacutegaliteacute eacutelimination des formes de travail proscrites (enfants travail forceacute) preacutevention des traitements inhumains ou deacutegradants de type harcegravelements sexuels protection de la vie priveacutee et des donneacutees personnelles 3 Environnement Protection sauvegarde preacutevention des atteintes agrave lenvironnement mise en place drsquoune strateacutegie manageacuteriale approprieacutee eacuteco conception protection de la biodiversiteacute et maicirctrise rationnelle des impacts environnementaux sur lrsquoensemble du cycle de vie des produits ou services

4 Comportements sur les marcheacutes Prise en compte des droits et inteacuterecircts des clients inteacutegration de standards sociaux et environnementaux dans la seacutelection des fournisseurs et sur lrsquoensemble de la chaicircne drsquoapprovisionnement preacutevention effective de la corruption respect des regravegles concurrentielles 5 Gouvernement drsquoentreprise Efficience et probiteacute assurance de lrsquoindeacutependance et de lrsquoefficaciteacute du Conseil drsquoadministration effectiviteacute et efficience des meacutecanismes drsquoaudit et de controcircle et notamment inclusion des risques de responsabiliteacute sociale respect des droits des actionnaires et notamment des minoritaires transparence et rationaliteacute de la reacutemuneacuteration des dirigeants 6 Engagement socieacutetal Effectiviteacute inteacutegration manageacuteriale de lrsquoengagement contribution au deacuteveloppement eacuteconomique et social des territoires drsquoimplantation et de leurs communauteacutes humaines engagements concrets en faveur de la maicirctrise des impacts socieacutetaux des produits et des services contribution transparente et participative agrave des causes drsquointeacuterecirct geacuteneacuteral

Initiation au management copy CRCF ndash J Sornet Page 47 48

ELEMENTS DE CORRIGE DT DT1 Deacutefinir expliquer

Deacutereacuteglementation = suppression des contraintes eacuteconomiques (libre eacutechange des biens et

capitaux)

Socieacutetal = qui se rapporte agrave la structure agrave lrsquoorganisation ou au fonctionnement de la socieacuteteacute

Economies drsquoeacutechelle = reacuteduction des coucircts lieacutee au niveau drsquoactiviteacute (amortissement des

charges fixes)

Coaching = accompagnement de personnes ou deacutequipes pour le deacuteveloppement de leurs

potentiels

EDI = eacutechange de donneacutees informatiseacutees ET standardiseacutees (ex SWIFT bancaire edifact

documents deacuteclaratifs)

Gouvernance = exercice du pouvoir la bonne gouvernance est participative et eacutequitable

conforme agrave lrsquointeacuterecirct commun

DT2 Deacuteterminer en quoi la deacutemarche TQM srsquoinscrit dans les deacutefis actuels du management

Voir notamment fiche 43

Maicirctrise des processus reacuteduction des coucircts reacuteactiviteacute et satisfaction de la clientegravele = faire

face agrave la concurrence

Ameacutelioration de lrsquoimage motivation du personnel

DT3 Apregraves avoir consulteacute les documents ci-dessous extraits du site drsquoAir France

(httpdeveloppement-

durableairfrancecomFRfrlocaldemarcheN4_positionnement_pphtm)

exposer les enjeux et les limites de la RSE et de la gestion des parties prenantes

Trame geacuteneacuterale possible

Introduction

Les deacutefis contemporains (accroissement de la concurrence devenue mondiale recherche

de nouveaux avantages concurrentiels pression de la socieacuteteacute besoin drsquoimage et de projet

lisible pour mener lrsquoentreprise crise et scandales du libeacuteralisme hellip) RSE et PP

Deacuteveloppement (voir cours)

1 ndash Parties prenantes et management par la valeur

PP deacutefinir citer reacutesumer lrsquoavantage rechercheacute (fideacuteliser motiver recherche drsquoalliances

implicites)

PP moyens (dont exemples AF) et meacutethode de management par la valeur (reacutepartie)

2 ndash La responsabiliteacute socieacutetale de lrsquoentreprise

RSE 3 axes

- eacuteconomique (favoriser le deacuteveloppement les eacutechanges internationaux)

- social (accegraves aux soins eacuteducation conditions de travail hellip)

- environnemental (pollution preacuteservation des ressources hellip)

RSE gouvernance drsquoentreprise facteur drsquoimage inteacutegrable dans la deacutemarche PP

Article 116 de la loi Le rapport viseacute agrave larticle L 225-102 rend compte hellip laquo Il comprend

eacutegalement des informations dont la liste est fixeacutee par deacutecret en Conseil dEtat sur la maniegravere

dont la socieacuteteacute prend en compte les conseacutequences sociales et environnementales de son

activiteacute Le preacutesent alineacutea ne sapplique pas aux socieacuteteacutes dont les titres ne sont pas admis aux

neacutegociations sur un marcheacute reacuteglementeacute raquo

Initiation au management copy CRCF ndash J Sornet Page 48 48

RSE exemple AF (ONG fournisseurs)

3 ndash Liens entre PP et RSE

- la RSE introduit de nouvelles PP

- la RSE suppose le respect des PP usuelles (employeacutes clients notamment)

4 - Probleacutematique

- deacutefinir la valeur reacuteellement apporteacutee par une gestion des PP (confusion salaire ndash valeur

idem impocircts hellip ex laquo valeur ajouteacutee raquo)

- communication (neacutecessaire mais aller au-delagrave)

- marginaliteacute des deacutepenses RSE (efficaciteacute sinceacuteriteacute de lrsquoengagement marge de manœuvre)

- charge RSE reporteacutee sur des tiers (ex fournisseurs AF)

- inteacutegration de facteurs non visibles en comptabiliteacute (pertes drsquoemploi nuisances hellip)

Conclusion

Voies incontournables mais pouvant nrsquoavoir qursquoun effet superficiel et temporaire Voir utiliteacute

drsquoaccompagnement leacutegislatif de regravegles de gouvernance

Initiation au management copy CRCF ndash J Sornet Page 6 48

Management opeacuterationnel X et Y mettent en place leur communication avec le

personnel et un plan de reconversion Z nomme un directeur de recherche qui prend

notamment en charge les relations avec lrsquouniversiteacute

Le management recouvre la totaliteacute des actes de conduite de lrsquoorganisation dans tous les

domaines (technique commercial financier hellip) mais il ne correspond agrave aucune cellule de

lrsquoorganigramme

Les actions de management sont par ailleurs contraintes par la disponibiliteacute des ressources

neacutecessaire pour atteindre les objectifs (financement savoir-faire profil du personnel

eacutequipements hellip)

3 ndash La meacutethode laquo management raquo

31 ndash La science du management

Le management nrsquoest pas une science exacte il srsquoapparente agrave une science humaine

expeacuterimentale qui traite de pheacutenomegravenes socio-eacuteconomiques eacutevolutifs et qui doit trouver

concregravetement son application dans la vie des organisations

Cette science traite notamment de lrsquoorganisation des entreprises et rassemble des meacutethodes

et des theacuteories qui peuvent ecirctre regroupeacutees en eacutecoles ou en courants

Les theacuteories marquent geacuteneacuteralement une eacutepoque et elles peuvent se recouper partiellement

parfois srsquoopposer avec des nuances qui doivent ecirctre bien identifieacutees Elles srsquoaccompagnent

souvent de modegraveles et de scheacutematisations qui en facilitent la compreacutehension et la

transposition agrave de nouvelles situations

Ces outils scientifiques guident le raisonnement permettent drsquoappreacutehender des reacutealiteacutes

complexes et structurent les connaissances ils sont peacutedagogiques et constituent des aides

pour lrsquoaction Leur application doit cependant ecirctre raisonneacutee car

- un modegravele nrsquoest qursquoune simplification de la reacutealiteacute

- une theacuteorie sortie de son contexte historique et eacuteconomique peut perdre de sa pertinence

Sa mise en œuvre doit tenir compte de la situation reacuteelle et une theacuteorie ne peut agrave elle seule

justifier une deacutecision de management (ce drsquoautant plus que chaque theacuteorie ne couvre que

tregraves partiellement le domaine du management ou de lrsquoorganisation)

- il est souvent difficile de disposer agrave temps drsquoinformations fiables et suffisantes pour appliquer

une theacuteorie dans les conditions ideacuteales Le manager est freacutequemment ameneacute agrave prendre ses

deacutecisions sur la base drsquoinformations incomplegravetes ou incertaines et il doit alors en mesurer les

risques et preacutevoir les ajustements neacutecessaires

ENTREPRISE

Management Administration

Gestion

Strateacutegique

Opeacuterationnel

Initiation au management copy CRCF ndash J Sornet Page 7 48

- des modes influencent le management Elles peuvent indiquer une veacuteritable eacutevolution

eacuteconomique mais aussi ecirctre sans lendemain voire introduire un danger ou un coucirct inutile

(se meacutefier des speacutecialistes dont le fond de commerce est la vente de nouvelles techniques

de management et des seacuteminaires associeacutes)

- seule la creacuteation drsquoune combinaison originale (dans le respect des regravegles) peut apporter un

avantage agrave lrsquoentreprise et non la reproduction de choix de management connus de tous

Exemple 1 la matrice de portefeuille drsquoactiviteacutes permet de classer les activiteacutes

strateacutegiques drsquoune entreprise en fonction de leur taux de croissance et de la part de

marcheacute deacutetenue (matrice laquo BCG raquo - Boston consulting group ndash 1975) Cet outil de

management neacutecessite de disposer drsquoinformations fiables concernant le marcheacute Il

conduit souvent agrave simplifier les conditions de concurrence et ignore la

compleacutementariteacute pouvant exister entre activiteacutes (synergie partage de techniciteacute

amortissement de charges fixes hellip) La matrice est un moyen de prendre conscience

du portefeuille et de ses eacuteventuels deacutefauts (portefeuille deacuteseacutequilibreacute avec

preacutedominance anormale drsquoune zone) mais il ne permet pas seul de deacutecider de lrsquoavenir

des activiteacutes

Exemple 2 le lancement drsquoun investissement lourd (lrsquoimplantation de nouvelles usines

le lancement drsquoune nouvelle activiteacute agrave fort taux de recherche ndash deacuteveloppement hellip) ne

peut ecirctre deacutecideacute que par le recoupement de diffeacuterentes approches (financiegravere

commerciale strateacutegique) Aucune theacuteorie du management appliqueacutee isoleacutement et

sans preacutecaution ne peut justifier un tel investissement

Exemple 3 une entreprise produisant des eacutequipements meacutedicaux deacutecide de renforcer

son offre commerciale par un service de conseil et drsquoassistance Cette deacutecision est

inspireacutee par le concept de creacuteation de valeur par une volonteacute de se diffeacuterencier de la

concurrence et par des pratiques existant depuis longtemps dans le domaine

informatique

Remarque parmi les facteurs influenccedilant les principes de management il est utile de

savoir deacutetecter pour srsquoen preacutemunir les ideacuteologies preacutejugeacutes et autres laquo valeurs raquo sans

rapport certain avec lrsquoefficaciteacute eacuteconomique

32 ndash Theacuteorie et pratique du management

La litteacuterature (ouvrages revues) aborde le management sous des angles diffeacuterents qui

peuvent suggeacuterer une concurrence entre des approches theacuteorique et pratique ou

psychologique et eacuteconomique de cette discipline

Taux de croissance

du domaine (cf

cycle de vie du

produit)

Part de marcheacute

(compareacutee au

principal

concurrent)

fort

faible

forte faible

Dilemmes (activiteacutes

en phase de

lancement

potentiel + coucircts+

risque+)

Vedettes

(activiteacutes en forte

croissance

autofinancement+)

Vaches agrave lait

(activiteacutes

stabiliseacutees et

concurrence faible

ou stable

rentabiliteacute+)

Poids morts

(activiteacutes en

deacuteclin)

Initiation au management copy CRCF ndash J Sornet Page 8 48

Le management a cependant une viseacutee unique et concregravete la bonne marche des

organisations et les travaux des universitaires rejoignent lrsquoaction des praticiens (les

universitaires reacutealisent drsquoailleurs bien souvent leurs recherches au sein des organisations)

Au plan peacutedagogique les deux visions du management se complegravetent

- la vision acadeacutemique met en perspective des concepts et des theacuteories Elle permet de

srsquoapproprier des raisonnements essentiels et stables notamment concernant la strateacutegie et

les reacuteactions humaines qui sont le moteur des organisations

- la vision pratique relate le veacutecu des praticiens preacutesente des techniques concregravetes et tente

parfois une vulgarisation favorisant la diffusion des concepts Elle integravegre de faccedilon

pragmatique les objectifs eacuteconomiques et les contraintes de fonctionnement des

organisations en se placcedilant parfois dans une vision agrave court terme

Sur le terrain le management reacutealise une synthegravese entre des techniques de psychologie

sociale et des techniques de gestion de diverses origines et de nouvelles meacutethodes

apparaissent aussi sous le label unique laquo management raquo

Le terme manageacuterial (approche manageacuteriale theacuteorie manageacuteriale pratique manageacuteriale

hellip) fait reacutefeacuterence agrave la vision aux preacuteoccupations et aux actions des managers qui doivent

emmener leur organisation vers la reacutealisation de ses objectifs

Exemples

- la deacutetermination des coucircts et des marges reacutesulte de techniques de gestion

indispensable aux deacutecisions des managers

- le laquo CRM raquo (customer relationship management en franccedilais GRC ndash gestion de la

relation client) est un concept reacutecent qui srsquoaccompagne de techniques lieacutee aux plus

reacutecents deacuteveloppements du management et des systegravemes drsquoinformation

- le laquo coaching raquo est une technique de management permettant une eacutevolution

personnelle dans le sens des objectifs de lrsquoentreprise

- le recentrage sur le laquo meacutetier raquo repose sur des techniques de management

Les techniques sont rassurantes mais pas suffisantes et si les theacuteories ne sont pas

indispensables au praticien elles aident agrave comprendre agrave anticiper et agrave bien utiliser les

techniques

La qualiteacute du management reacuteside beaucoup dans la capaciteacute agrave appliquer concepts et

techniques de faccedilon pertinente et agrave innover Cette capaciteacute relegraveve en partie drsquoun laquo art du

management raquo qui srsquoacquiert en grande partie par la pratique

4 ndash Management et expertise comptable

Lrsquoexpert comptable doit manager ses propres eacutequipes Il est par ailleurs supposeacute laquo hellip

conseiller et accompagner le chef drsquoentreprise dans toutes ses deacutecisions hellip raquo (selon lrsquoOEC)

Ce rocircle est particuliegraverement important dans ses relations avec les petites entreprises Il doit

donc avoir une capaciteacute au management

Organisation

Vision acadeacutemique Vision pratique

Initiation au management copy CRCF ndash J Sornet Page 9 48

Lrsquointervention de lrsquoexpert dans le management drsquoune entreprise peut toutefois poser

quelques problegravemes

- Il peut y avoir conflits drsquointeacuterecirct entre activiteacutes de certification des comptes et de conseil en

management (la tendance est agrave la seacuteparation des activiteacutes dans les plus grands cabinets)

- lrsquoactiviteacute de conseil neacutecessite des compeacutetences parfois tregraves speacutecifiques (conseil fiscal

conseil en RH conseil en systegravemes drsquoinformation hellip)

- le conseil est une activiteacute diffeacuterente par sa forme de lrsquoexpertise comptable (interventions

longues peu reacutepeacutetitives peu codifieacutees mises en concurrence) qui neacutecessite une

organisation particuliegravere du cabinet lorsqursquoelle deacutepasse lrsquointervention occasionnelle

APPLICATIONS IM

IM1 Analyser la profession de laquo manager raquo selon Henry Mintzberg (texte extrait de lrsquoouvrage

laquo Le management raquo Eyrolles - Editions drsquoOrganisation) et les principes du management de la

norme ISO

Faire ressortir les eacuteleacutements speacutecifiques agrave chacune de ces approches et mettre en eacutevidence

leurs points communs

Initiation au management copy CRCF ndash J Sornet Page 10 48

Principes du management drsquoapregraves la norme ISO 9001 (2000)

- Orientation vers le client (satisfaire ses attentes)

- Leadership (les dirigeants eacutetablissent les orientations de lrsquoorganisme Ils doivent creacuteer

un environnement interne ougrave les personnes peuvent clairement srsquoimpliquer dans la

reacutealisation des objectifs de lrsquoorganisme)

- Implication du personnel (les personnes sont agrave tout niveau lrsquoessence de lrsquoorganisme et

leur implication permet drsquoutiliser leurs aptitudes au profit de lrsquoorganisme)

- Approche laquo processus raquo (un reacutesultat est mieux atteint quand les ressources et les

activiteacutes neacutecessaires sont geacutereacutees comme un processus)

- Approche systegraveme (assimiler les processus correacuteleacutes agrave un systegraveme contribue agrave

lrsquoefficaciteacute et agrave lrsquoefficience de lrsquoorganisme vis-agrave-vis de ses objectifs)

- Ameacutelioration continue (objectif permanent de lrsquoorganisme)

- Prise de deacutecision efficace (par lrsquoanalyse de donneacutees et drsquoinformations)

- Relations mutuellement beacuteneacutefiques avec les fournisseurs (pour augmenter la capaciteacute

des deux organismes agrave creacuteer de la valeur)

IM2 Distinguer leader et manager

IM3 Compleacuteter le tableau ci-dessous en analysant chaque action preacutesenteacutee Faire ensuite

ressortir les domaines niveaux ou techniques de management pouvant ecirctre mobiliseacutes pour

chaque situation

Initiation au management copy CRCF ndash J Sornet Page 11 48

Caracteacuteristiques

de lrsquoaction

- reacutepeacutetition

- risque

- normes

- ampleur

Prise de

deacutecision

- opeacuterationnelle

strateacutegique

- deacutelai

Informations

neacutecessaires

- nature

- origine

- deacutelai obtention

Cleacutes pour la

reacuteussite

Intervention

exteacuterieure

possible

Assurer la

restauration du

soir

(restaurant

familial)

Construire un

viaduc

(autoroute)

Certifier les

comptes

annuels drsquoun

groupe

national

(cabinet

drsquoaudit)

Lancer une

ligne drsquoavions

(constructeur

aeacuteronautique)

Reacuteduire la

capaciteacute de

production

(groupe

industriel)

Acqueacuterir une

entreprise

concurrente

(teacuteleacutephonie

mobile)

Initiation au management copy CRCF ndash J Sornet Page 12 48

Fiche IM1 - Deacutefinitions du management

Dictionnaire anglais - franccedilais direction administration gestion intrigue manegravege

Wikipeacutedia Le management est lensemble des techniques dorganisation qui sont mises en

oeuvre pour ladministration dune entiteacute

Au point de vue eacutetymologique le verbe manage vient de litalien maneggiare (controcircler)

influenceacute par le mot franccedilais manegravege (faire tourner un cheval dans un manegravege) A cette

notion il faut aussi ajouter la notion de meacutenage (geacuterer les affaires du meacutenage) qui consiste agrave

geacuterer des ressources humaines et des moyens financiers

helliphellip

Fiche IM2 - Etudier le management

Concreacutetiser

Manager neacutecessite de syntheacutetiser des informations parfois complexes incomplegravetes et de

domaines tregraves divers pour en deacuteduire des actions Une approche trop parcellaire peut

conduire agrave lrsquoeacutechec et le savoir-faire est neacutecessaire pour agir vite avec un minimum de risque

Lrsquoeacutetudiant doit se preacuteparer simultaneacutement aux examens et agrave la pratique Il nrsquoa souvent connu

lrsquoentreprise que durant quelques semaines de stage et le manque de laquo recul raquo ne lui permet

pas toujours de concreacutetiser les theacuteories Il doit compenser par la lecture (ouvrages revues

journaux eacuteconomiques et boursiers) et en eacutetant attentif aux informations ambiantes (tout en

relativisant le style journalistique) en mettant en relation le cours les concepts les modegraveles

lrsquoactualiteacute les stages

Savoir traiter un exercice

Pour reacuteussir un examen ou traiter une application peacutedagogique (la conception les points 1 agrave

6 peut repreacutesenter le tiers du temps de travail)

1 ndash Identifier le type de sujet (faut-il trouver une solution pratique ou communiquer une

reacuteflexion geacuteneacuterale )

2 ndash Lire le sujet et relever les mots cleacutes

3 ndash Deacutefinir les mots cleacutes

4 ndash Reacutesumer la probleacutematique du sujet (en quelques lignes)

5 ndash Lister les connaissances reacutefeacuterences et raisonnements reacutepondant au problegraveme (par

recherche spontaneacutee ou raisonneacutee qui quoi ougrave quand comment combien hellip

listage des diffeacuterents points de vue) trouver des exemples (notamment dans les

documents fournis)

6 ndash Organiser la reacuteponse (deacutefinir le plan du deacuteveloppement ougrave des paragraphes bien

identifieacutes sont geacuteneacuteralement neacutecessaires en y liant les parties qui doivent ecirctre en nombre

limiteacute ndash de deux agrave quatre) Preacutevoir drsquoy inteacutegrer la deacutefinition des principales notions

induites par le sujet

7 ndash Reacutediger sous la forme adapteacutee (note technique ou recommandation solution

pratique exposeacute structureacute dissertation)

Introduction et conclusion sont indispensables agrave la dissertation ou agrave lrsquoexposeacute

- lrsquointroduction preacutesente le sujet traiteacute (phrase drsquoaccroche initiale) amorce la

probleacutematique (quelques sous - questions) et annonce le plan

- la conclusion syntheacutetise le deacuteveloppement (arguments) eacutelargit le sujet (prise de recul)

et apporte le point final (une phrase)

Une limite agrave la communication

Il est difficile de faire passer plus de 4 ou 5 ideacutees fortes dans un exposeacute unique

Initiation au management copy CRCF ndash J Sornet Page 13 48

Fiche IM3 - Bref historique

Antiquiteacute

3000 AJC

Peacuteriode greacuteco-

romaine

Transition

feacuteodale

12egraveme siegravecle

europe

15egraveme ndash 17egraveme

siegravecles

19egraveme siegravecle

20egraveme siegravecle

agriculture preacutedominante industrie limiteacutee aux besoins drsquoun individu ou drsquoun clan

pour la confection des outils des vecirctements et de la poterie Force motrice animale

ou humaine pour lrsquoessentiel

Grands travaux drsquoeacutetat en Egypte premiegravere laquo planification ndash organisation ndash controcircle raquo

Deacuteveloppement des communications essor industriel limiteacute peu de progregraves

technique (lrsquoesclavage supplante les innovations)

Deacuteveloppement progressif des eacutechanges commerciaux

La consommation indirecte atteint un bon niveau (surplus agricoles et

deacuteveloppement des villes) Apparition de nouveaux commerccedilants

Etat fort Evolutions technologiques (imprimerie bateaux performants instruments de

navigation) Extension geacuteographique de lrsquoeacuteconomie Apparition des corporations

drsquoartisans

Machine agrave vapeur chemin de fer passage de lrsquoartisanat au capitalisme

entrepreneurial producteur organisation des entreprises

Ecole classique (Taylor Fayol Weber) approche meacutecaniste bureaucratie

hieacuterarchie commandement fonctions et speacutecialisation laquo OS T raquo (organisation

scientifique du travail) organisation source de pouvoir rationaliteacute des individus bases

du management

Deacuteveloppement du capitalisme manageacuterial Electriciteacute peacutetrole puis communications

et information Consommation de masse mondialisation preacuteoccupations

eacutenergeacutetiques et environnementales 3 peacuteriodes

- standardisation grandes entreprises industrielles

- industries de consommation 30 glorieuses marketing multinationales protection

sociale

- deacutereacuteglementation monteacutee des services pays eacutemergents mondialisation et nouvelle

eacuteconomie (internet)

Ecole des relations humaines prise en compte de lrsquoindividu des motivations styles

de direction

Ecole neacuteo-classique et post-classique deacutecentralisation coordonneacutee DPO

management participatif zeacutero deacutefaut flux tendus

Approche systeacutemique partition de lrsquoentreprise eacutetude des interactions feacutedeacuteration

vers lrsquoobjectif controcircle et ajustement

Theacuteories de la deacutecision rationaliteacute limiteacutee contribution reacutetribution coalitions

Ecole socio-technique recherche de compromis technologie organisation

enrichissement des tacircches autonomie des groupes

Approche sociologique effets sociaux du travail jeux de pouvoir dans lrsquoentreprise

reacutegulation sociale

Theacuteories de la contingence facteurs contingents adaptation agrave lrsquoenvironnement

configurations organisationnelles

Theacuteories de la firme controcircle manageacuterial droits de proprieacuteteacute relation drsquoagence

Theacuteories contractualistes firme nœud de contrats coucircts de transaction

opportunisme externalisation internalisation

Approche eacutevolutioniste eacutecologie des organisations modegravele eacutevolutioniste

contraintes de sentier

Approche par les ressources valorisation des ressources compeacutetences cleacutes

apprentissage organisationnel

(Classement simplifieacute)

Initiation au management copy CRCF ndash J Sornet Page 14 48

ELEMENTS DE CORRIGE IM

IM1 Commenter la deacutefinition du management par la norme ISO et le manager de Mintzberg

Efficient = optimum avec les moyens disponibles

ISO (management objectifs) (manager moyens) HM

IM2 Le leader entraicircne naturellement derriegravere lui Le manager nrsquoest pas toujours leader

(mecircme si crsquoest souhaitable) Le leader nrsquoest pas toujours manager (plutocirct notion individuelle)

Leadership = faculteacute de diriger conjugaison drsquoune autoriteacute naturelle ou drsquoun savoir-faire

acquis drsquoune capaciteacute agrave entraicircner des personnes ou des groupes et drsquoune leacutegitimiteacute

statutaire (de position)

IM3 Compleacuteter le tableau ci-dessous en analysant chaque action preacutesenteacutee Faire ensuite

ressortir les domaines niveaux ou techniques de management pouvant ecirctre mobiliseacutes pour

chaque situation

Satisfaction client

Implication du personnel

Processus systegraveme

Ameacutelioration continue

Deacutecision efficace

Recherche de valeur

Image entreprise

Liaisons

Information

Reacutepartition ressources

Reacutegulation

Neacutegociation

Leadership

Initiation au management copy CRCF ndash J Sornet Page 15 48

Caracteacuteristiques

de lrsquoaction

- reacutepeacutetition

- risque

- normes

- ampleur

Prise de

deacutecision

- opeacuteration

- direction

- deacutelai

Informations

neacutecessaires

- nature

- origine

- deacutelai

obtention

Cleacute pour la

reacuteussite

Intervention

exteacuterieure

possible

Assurer la

restauration du

soir

(Restaurant

familial)

Technique

(fabrication)

Vente (terrain)

Appros

Reacutepeacutetitive

(quot)

Risque faible

Normes

drsquohygiegravene

Faible

Opeacuterationnelle

Geacuterant

responsable

Rapide (qq

jours menu et

appros)

Nombre de

couverts

Tarifs usuels

Calendrier

(fecirctes)

Clients docs

divers

expeacuterience

Qq jours

Varieacuteteacute menu

Plats phares

Accueil

Appros

Tarification

Vins

Gestion

congeacutelation

Qualiteacute cuisine

Fournisseurs

Extra

Publiciteacute

Construire un

viaduc

(autoroute)

Technique

Organisation

Appros

Uniteacute (ou peu)

Eleveacute (financier

technique)

Architecture

Eleveacutee

Direction

(aleacuteas)

Opeacuterationnelle

(conduite

chantier)

Immeacutediat agrave qq

semaines

Plans

plannings

Qualifications

Meacuteteacuteo

Disponibiliteacutes

Bureau eacutetudes

Qq sem agrave 24h

Techniciteacute

Appros

Qualifications

Preacutevision

GRH

Contrat juste

SS traitants

Organismes

certificateurs

Controcircle

client

Certifier les

comptes

annuels drsquoun

groupe national

(cabinet

drsquoaudit)

Technique

Relation client

Gestion des

connaissances

Annuelle

Moyen

Regravegles

comptables

fiscales

Moyenne (selon

importance du

cabinet)

Opeacuterationnelle

Qq jours agrave

semaines

Comptable

Juridique

Client

Etat

Qq jours agrave

semaines

Techniciteacute

Expeacuterience

Relation client

Systegraveme info client

Siegravege

Autre cabinet

Lancer une

ligne drsquoavions

(constructeur

aeacuteronautique)

Strateacutegique

RD

Etudes

Uniteacute

Tregraves eacuteleveacute

Aeacuteronautique

Tregraves eacuteleveacutee

Direction

Qq mois agrave

anneacutees

Marcheacute

Etudes

Compagnies

Qq mois agrave

anneacutees

Concept

Outil industriel

Coucirct exploitation

Tarif

Fiabiliteacute

Deacutelaisconcurrence

SI simulation

SS traitants

Bureaux

drsquoeacutetudes

speacutecialiseacutes

Compagnies

Conseils

Reacuteduire la

capaciteacute de

production

(groupe

industriel)

Strateacutegique

RH

Communication

Production

Uniteacute

Moyen

Leacutegislation

(dont RH)

Eleveacutee

Direction

Qq mois agrave

anneacutees

Financiegravere

Industrielle

Marcheacute

Organisation

Organismes

speacutecialiseacutes

DRH

Qq mois

Communication

Connaissance des

compeacutetences

Connaissance outil

industriel

Concurrence

Portefeuille

drsquoactiviteacutes

Cabinet

drsquoorganisation

Conseils

speacutecifiques

Acqueacuterir une

entreprise

concurrente

(teacuteleacutephonie

mobile)

Strateacutegique

Marketing

Production

(reacuteseau)

Financier

Communication

Uniteacute

Tregraves eacuteleveacute

Leacutegislation

telecom

Tregraves eacuteleveacutee

Direction

Qq mois

Financiegravere

Marcheacute

Reacuteseaux

(ampleur

recouvrement

hellip)

Organisations

Interne

Racheteacutee

Sources

speacutecialiseacutees

Qq mois

Communication

Marcheacute

Cours boursiers

Cabinet

drsquoorganisation

Conseils

speacutecifiques

Initiation au management copy CRCF ndash J Sornet Page 16 48

LE MANAGEMENT EN PRATIQUE

Pour assumer sa fonction le management doit couvrir sans discontinuiteacute lrsquoensemble de

lrsquoorganisation et inteacutegrer de nombreux facteurs dont nous allons reacutesumer lrsquoessentiel

1 ndash Les fonctions et activiteacutes du management

Pour Henri Fayol la fonction drsquoadministration de lrsquoentreprise (son management) reposait sur

cinq actions preacutevoir organiser commander coordonner et controcircler (laquo PO3C raquo)

Nous distinguerons cinq activiteacutes de management

- la conception (au plus haut niveau finaliteacute but ou vocation de lrsquoorganisation

meacutetiers dimension politique de croissance hellip)

- la planification (deacutefinition des objectifs eacutecheacuteances)

- lrsquoorganisation (reacutepartition du travail choix des modes de coordination)

- le pilotage de lrsquoaction opeacuterationnelle (motivation animation encadrement

assistance)

- lrsquoeacutevaluation (controcircle des reacutesultats obtenus ajustements)

Dans chacune de ces activiteacutes des deacutecisions et des arbitrages sont neacutecessaires avec des

enjeux plus ou moins importants

Remarques

- Les cinq activiteacutes du management peuvent se retrouver agrave tout niveau de

management si lrsquoentreprise laisse une certaine autonomie de deacutecision agrave ses diffeacuterentes

uniteacutes La conception est naturellement du ressort de la direction geacuteneacuterale et des

conseils drsquoadministration mais elle peut ecirctre preacutesente pregraves du terrain (latitude laisseacutee agrave

une filiale ou agrave un magasin par exemple) De mecircme lrsquoorganisation du travail concerne

un atelier mais aussi la direction qui structure lrsquoentreprise pour assurer ses activiteacutes sa

production

- La planification deacutefinit des objectifs ou des axes strateacutegiques (choix de produits

modaliteacutes de deacuteveloppement des ventes implantations alliances hellip) et les traduit en

donneacutees de gestion preacutevisionnelles syntheacutetiques et eacutechelonneacutees dans le temps afin de

valider les objectifs et de fixer des repegraveres

- Un laquo business plan raquo (plan drsquoaffaires)est notamment lrsquoeacutequivalent de la planification

dans le cas de creacuteation drsquoentreprise ou pour la preacutesentation de tout projet drsquoactiviteacute

Les activiteacutes du management srsquoinscrivent dans des cycles qui peuvent ecirctre scheacutematiseacute

comme suit (lrsquoeacutevaluation peut entraicircner une reacutevision du pilotage de lrsquoorganisation ou des

objectifs sans que lrsquoentreprise ne soit fondamentalement remise en cause)

conception

planification

organisation

pilotage

eacutevaluation

Initiation au management copy CRCF ndash J Sornet Page 17 48

2 ndash Les contextes de management

Le management est influenceacute par son contexte qui justifie des objectifs une organisation

des meacutethodes

Par exemple lrsquoentreprise admet de nombreuses variantes selon sa taille sa forme juridique

son controcircle par lrsquoeacutetat (entreprises publiques) ou par des inteacuterecircts priveacutes Il en va de mecircme des

organismes administratifs qui peuvent deacutependre de directives nationales ou reacutegionales des

associations qui ont des activiteacutes drsquoampleur tregraves variable

21 ndash La dimension de lrsquoentreprise

La dimension drsquoune entreprise se mesure principalement en fonction de son effectif ou de

son chiffre drsquoaffaires Des seuils sont deacutefinis par divers organismes et exploiteacutes agrave des fins

statistiques ou pour la deacutetermination de certaines obligations sociales ou fiscales

(repreacutesentation du personnel cotisations hellip) Il nrsquoy a bien entendu pas de laquo barriegravere de

tailleraquo absolue conditionnant le management drsquoune entreprise

LrsquoUE preacuteconise de distinguer les micro ndash entreprises (jusqursquoagrave 9 salarieacutes) les TPE ndash tregraves petites

entreprises (moins de 20 salarieacutes) les petites entreprises (moins de 50) et les moyennes

entreprises (de 50 agrave 250) Cependant les PME sont parfois situeacutees entre 10 et 500 salarieacutes

Remarques

- en France environ 40 des entreprises emploient de 1 agrave 50 salarieacutes (ce qui repreacutesente

plus de 50 des emplois) et 59 nrsquoen ont aucun

le pays compte environ 2 600 000 entreprises dont moins de 1 ont 250 employeacutes et

plus

- ancienneteacute et taille de lrsquoentreprise sont lieacutees si lrsquoon eacutecarte les restructurations et autres

eacutevolutions drsquoentreprises existantes

La dimension de lrsquoentreprise a une influence sur lrsquoorganisation et le laquo style raquo de son

management

- les PME sont souvent entrepreneuriales (les dirigeants eacutegalement apporteurs de capitaux

sont totalement engageacutes dans la marche de lrsquoentreprise) Elles ont une gestion flexible peu

formaliseacutee plus qualitative que quantitative Les PME sont freacutequemment focaliseacutees sur un seul

type drsquoactiviteacute Pour ne pas alourdir leur structure elles ont tendance agrave sous-traiter les

activiteacutes speacutecialiseacutees ne correspondant pas agrave leur meacutetier de base

- les grandes entreprises sont manageacuteriales (les dirigeants sont nommeacutes par les actionnaires

en raison de leurs compeacutetences) et moins reacuteactives

22 ndash Le type de production

On distingue industrie (production de biens mateacuteriels ou pour le moins de produits visibles ndash

comme un seacutejour touristique ou un film) et services (fourniture drsquoune prestation immateacuterielle)

Le type de production influence en principe le management de lrsquoentreprise

- lrsquoindustrie neacutecessite (si lrsquoon excepte lrsquoartisanat) un investissement relativement important

une organisation productive stable capable de reacutealiser plusieurs fois des produits identiques

(exemple un modegravele de reacutefrigeacuterateur) ou du moins similaires (exemple un bacirctiment) Le

produit de lrsquoindustrie consomme des matiegraveres et il doit geacuteneacuteralement ecirctre distribueacute jusqursquoau

client

- la production de services peut se satisfaire drsquoun investissement tregraves reacuteduit et neacutecessite un

contact permanent avec le client

Toutefois la standardisation des services et le deacuteveloppement des reacuteseaux informatiques

rapprochent la production de services de celle des biens industriels

- la production drsquoun service reacutepeacutetitif et technique peut imposer une structure lourde et une

organisation tregraves formaliseacutee (voir les grandes socieacuteteacutes drsquoaudit ou de conseil informatique)

Initiation au management copy CRCF ndash J Sornet Page 18 48

- certains services peuvent ecirctre fournis agrave distance sans contact direct avec le client et

distribueacutes par reacuteseau (tenue de comptabiliteacute affacturage gestion clientegravele centre drsquoappel

hellip)

Remarque les services repreacutesentent 75 de lrsquoactiviteacute eacuteconomique franccedilaise

23 ndash La nature de lrsquoorganisation

Les organisations publiques franccedilaises (administrations centrales collectiviteacutes territoriales

hocircpitaux hellip) repreacutesentent une part importante de lrsquoactiviteacute (environ 30 des emplois) La

fonction publique regroupe des organisations aux finaliteacutes diverses et qui ont des problegravemes

de gestion similaires agrave ceux des entreprises auxquelles elles peuvent emprunter des principes

de management Notamment

- pour controcircler les coucircts et assurer la qualiteacute des services

- pour communiquer avec les administreacutes ou les usagers

- pour motiver les personnels et geacuterer les ressources humaines

La transposition directe des techniques de gestion et de management nrsquoest cependant pas

toujours possible car

- la comptabiliteacute publique obeacuteit agrave des regravegles speacutecifiques (proceacutedure budgeacutetaire

notamment)

- le laquo client raquo ne paye pas toujours la prestation du moins directement

- la concurrence est parfois inexistante

- les grandes administrations centraliseacutees sont soumises agrave des choix politiques geacuteneacuteraux

parfois sans connexion eacutevidente avec les besoins opeacuterationnels

- le statut des personnels et les grilles de salaires limitent les possibiliteacutes de gestion des

ressources humaines

Remarque la LOLF (loi organique relative aux lois de finances) est entreacutee en vigueur en

2006 Elle alloue des moyens budgeacutetaires en fonction de programmes et remplace la

reconduction automatique de 90 des budgets Cette reacuteforme se heurte toutefois agrave la

lourdeur des grands ministegraveres ougrave la complexiteacute des activiteacutes est difficile agrave

appreacutehender et ougrave des inerties culturelles peuvent exister agrave tout niveau

Les associations loi de 1901 peuvent avoir une activiteacute comparable agrave celle de grandes

entreprises (voir par exemple les associations de santeacute ou professionnelles) et leur

management est alors similaire malgreacute lrsquoabsence de but lucratif (les beacuteneacutefices ne sont pas

distribuables) Elles ont drsquoailleurs en France un poids eacuteconomique important (elles emploient

environ 1 600 000 salarieacutes)

Cependant lrsquoadheacutesion agrave un systegraveme de valeurs fondateur de lrsquoassociation ou la limite de

lrsquoautoriteacute (quand un volant de beacuteneacutevoles important participe agrave lrsquoactiviteacute) peut introduire des

nuances

- le renforcement des objectifs socieacutetaux

- la faiblesse des relations hieacuterarchiques

- des contraintes de gestion du temps des beacuteneacutevoles

- des modaliteacutes particuliegraveres de recrutement et de motivation des dirigeants

24 ndash Les facteurs contingents

La theacuteorie de la contingence montre qursquoune structure drsquoentreprise nrsquoest efficace que dans

une situation deacutetermineacutee et qursquoil nrsquoexiste que des solutions de management construites dans

un contexte preacutecis

Le management doit ainsi srsquoadapter agrave des facteurs contingents qui ne peuvent ecirctre

controcircleacutes du moins agrave bregraveve eacutecheacuteance Ces facteurs sont par exemple

- lrsquoancienneteacute de lrsquoentreprise (plus elle est ancienne plus lrsquoentreprise a tendance agrave reacutepeacuteter

des comportements eacuteprouveacutes)

Initiation au management copy CRCF ndash J Sornet Page 19 48

- la taille de lrsquoentreprise (la grande entreprise a une composante administrative plus

deacuteveloppeacutee)

- le systegraveme de production (tregraves standardiseacute complexe automatiseacute hellip)

- lrsquoenvironnement

3 ndash Le management et les parties prenantes

Lrsquoentreprise a pour vocation premiegravere de mettre des produits agrave disposition de ses clients en

reacutealisant un profit Pour y arriver elle doit aussi satisfaire ses parties prenantes salarieacutes

actionnaires fournisseurs hellip

Est partie prenante agrave lrsquoentreprise laquo tout groupe ou individu qui peut ecirctre affecteacute ou est

affecteacute par les buts de lrsquoorganisation hellip raquo (Freeman ndash 1984)

Les parties prenantes attendent agrave des degreacutes divers de profiter drsquoune creacuteation de valeur en

provenance de lrsquoentreprise qui doit reacutepondre agrave ces attentes pour assurer sa peacuterenniteacute ou

favoriser son deacuteveloppement

On distingue les parties prenantes primaires ou principales qui sont essentielles agrave lrsquoentreprise

et qui ont geacuteneacuteralement une relation formelle avec elle (clients associeacutes et actionnaires

precircteurs salarieacutes fournisseurs collectiviteacutes) et les parties prenantes secondaires dont

lrsquoinfluence est diffuse (groupes de pression associations meacutedias instances europeacuteennes

agences de notation hellip)

Remarque la consideacuteration de lrsquoensemble des parties prenantes (laquo stakeholders raquo - les

deacutepositaires) fait contrepoids agrave lrsquoimportance accordeacutee aux seuls actionnaires

(laquo shareholders raquo)

Les organisations nrsquoayant pas drsquoobjectif de profit doivent aussi satisfaire leurs parties

prenantes apporter un service aux usagers dans les meilleures conditions eacuteconomiques

limiter un budget assurer la qualiteacute des relations avec les fournisseurs hellip

Dans cette optique le management doit organiser lrsquoaction de faccedilon agrave eacutequilibrer des forces

parfois divergentes

- le contexte fait pression sur lrsquoorganisation contrainte agrave optimiser ses reacutesultats

- lrsquoorganisation cherche par son action agrave assurer sa peacuterenniteacute son deacuteveloppement (en

reacutealisant des profits dans le cas de lrsquoentreprise) et agrave satisfaire ses parties prenantes

- le management agit en pilotant les actions pour contrebalancer la pression du contexte

Actions de

lrsquoorganisation

Management Contexte

Parties

prenantes

Initiation au management copy CRCF ndash J Sornet Page 20 48

APPLICATIONS MP

MP1 Deacutefinir contingent gestion budgeacutetaire

MP2 Deacuteterminer les parties prenantes drsquoun hocircpital public et leurs principales attentes

Mecircme question pour les organisations suivantes

- SNCF (entreprise publique)

- Peugeot

- MAIF (mutuelle drsquoassurance)

MP3 En les situant dans le cycle des activiteacutes du management trouver les actions agrave mener

dans les situations suivantes

- baisse de 10 des ventes dans une entreprise industrielle (produits meacutenagers le reacuteseau de

distribution vient drsquoecirctre reacuteorganiseacute)

- idem dans une entreprise de vente par correspondance soumise agrave la concurrence internet

(les ventes stagnaient depuis six mois malgreacute les efforts promotionnels)

- augmentation des deacutelais drsquoattente des consultations dans une clinique (lrsquohocircpital voisin a

fermeacute son service drsquourgences)

Initiation au management copy CRCF ndash J Sornet Page 21 48

ELEMENTS DE CORRIGE MP

MP1 Deacutefinir (dans le contexte drsquoune entreprise) contingent gestion budgeacutetaire

Contingent = imposeacute par lrsquoexteacuterieur Contingence = effet du hasard de la rencontre de

plusieurs eacuteveacutenements indeacutependants (variables explicatives que lrsquoon ne peut influencer)

Gestion budgeacutetaire = technique drsquoadministration des entreprises srsquoappuyant sur des

preacutevisions dont on deacuteduit apregraves accord des responsables des attributions de moyens sur une

dureacutee limiteacutee Une analyse reacuteguliegravere des eacutecarts entre preacutevisions et reacutealisations permet ensuite

le pilotage des activiteacutes Le budget est un cadre incitatif

La laquo planification budgeacutetaire raquo consiste agrave traduire en budgets une planification strateacutegique

avec systegraveme de reporting

MP2 Deacuteterminer les parties prenantes drsquoun hocircpital public et leurs principales attentes

Mecircme question pour les organisations suivantes

- SNCF (entreprise publique)

- Peugeot

- MAIF (mutuelle drsquoassurance)

Hocircpital

- patients (qualiteacute des soins)

- CNAM (baisse des coucircts)

- collectiviteacute locale (service aux administreacutes)

- eacutetat (ameacutenagement du territoire maicirctrise des budgets optimisation)

- employeacutes (salaire conditions de travail et satisfaction)

- fournisseurs ndash pharmacie autres (CA paiement reacutegulier)

- associations de patients (qualiteacute proximiteacute des soins)

SNCF

- usagers et associations drsquousagers (proximiteacute reacutegulariteacute prix du service)

- reacuteseau ferreacute de France (optimisation des lignes paiement adapteacute)

- fournisseurs (CA paiement reacutegulier)

- employeacutes (salaire conditions de travail seacutecuriteacute de lrsquoemploi)

- eacutetat (ameacutenagement du territoire)

- collectiviteacutes locales (service)

Peugeot

- clients (qualiteacute prix SAV relation commerciale)

- fournisseurs (CA reacutegulariteacute de lrsquoactiviteacute)

- employeacutes (salaire conditions de travail seacutecuriteacute de lrsquoemploi)

- eacutetat (taxes)

- collectiviteacute locale (emploi dynamisation eacuteconomique preacuteservation de lrsquoenvironnement)

- associations de protection de lrsquoenvironnement (activiteacute propre baisse des eacutemissions

nouvelles eacutenergies)

MAIF

- socieacutetaires (protection relation assureur tarif mesureacute)

- professionnels de lrsquoautomobile et autres (agreacutement marge de manœuvre reacuteparations tarifs

eacuteleveacutes)

- fournisseurs (CA paiement reacutegulier)

- eacutetat (taxes engagement pour la seacutecuriteacute)

- employeacutes (salaire conditions de travail seacutecuriteacute de lrsquoemploi)

Initiation au management copy CRCF ndash J Sornet Page 22 48

MP3 En les situant dans le cycle des activiteacutes du management trouver les actions agrave mener

dans les situations suivantes

- baisse de 10 des ventes dans une entreprise industrielle (produits meacutenagers le reacuteseau de

distribution vient drsquoecirctre reacuteorganiseacute)

Adapter le pilotage motiver cadrer si insuffisant retoucher une organisation deacutefectueuse

- idem dans une entreprise de vente par correspondance soumise agrave la concurrence internet

(les ventes stagnaient depuis six mois malgreacute les efforts promotionnels)

Voir pilotage et organisation si une eacutevolution du meacutetier a deacutejagrave eacuteteacute initialiseacutee Sinon re-

conception (adaptation au nouveau contexte) puis planification et reacuteorganisation

- augmentation des deacutelais drsquoattente des consultations dans une clinique (lrsquohocircpital voisin a

fermeacute son service drsquourgences)

Organisation Si insuffisant planification (nouveaux objectifs)

Initiation au management copy CRCF ndash J Sornet Page 23 48

ORGANISATION ET PROCESSUS

La performance de lrsquoentreprise deacutepend de son organisation et de son aptitude agrave produire

aux meilleures conditions Nous allons montrer comment organisation formelle et processus

de production peuvent contribuer agrave cette performance

1 ndash Vers lrsquooptimum

11 ndash Les eacuteconomies occidentales jusqursquoaux anneacutees 70

Jusqursquoen 1945 le principal problegraveme des entreprises eacutetait de produire des biens en quantiteacute

suffisante agrave un prix compatible avec le marcheacute Les grandes entreprises se sont multiplieacutees et

la standardisation a permis de reacuteduire les coucircts (exemple deacuteveloppement de Ford et de la

production agrave la chaicircne de 1908 agrave 1920 qui a permis une baisse du prix des voitures des 23)

On parle de laquo production pousseacutee vers le marcheacute raquo

Cette croissance de la production peu reacuteguleacutee a eacuteteacute marqueacutee par des surproductions en

1910 et 1920 puis par la crise de 1929 qui a prolongeacute ses effets jusqursquoagrave la guerre

De 1945 agrave 1975 environ (les laquo trente glorieuses raquo) la reconstruction la croissance de la

consommation de masse de nouvelles technologies et les eacutechanges internationaux

alimentent lrsquoeacuteconomie La standardisation srsquoeacutetend aux biens de consommation dont les

coucircts baissent fortement et de nouvelles reacutegulations sociales permettent une eacutevolution sans

heurt des revenus La saturation de certains marcheacutes conduit dans les anneacutees 60 agrave la

deacutemarche laquo marketing raquo et agrave la diffeacuterenciation des produits Le produit est laquo dirigeacute par le

marcheacute raquo mais les entreprises conservent une organisation assez classique et les plus grosses

srsquointernationalisent

12 ndash Lrsquoexpeacuterience japonaise et ses prolongements

Tregraves tocirct apregraves la guerre dans un Japon appauvri le constructeur automobile Toyota a ducirc

faire face agrave une restriction du marcheacute des moyens financiers et productifs et des

approvisionnements La firme a donc innoveacute dans un nouveau systegraveme de production

chassant les laquo gaspillages raquo (temps drsquoattente transports stocks deacutefauts hellip) consideacuterant que

seule la fabrication vendable creacutee de la valeur

Toyota srsquoorganise pour fabriquer la quantiteacute et la qualiteacute de produits juste neacutecessaires agrave la

satisfaction des clients la production est laquo tireacutee par le marcheacute raquo La mise en place de ce

systegraveme qui integravegre les fournisseurs ne sera acheveacutee que dans le milieu des anneacutees 70

En 1973 la hausse du peacutetrole inaugure un ralentissement de la croissance des eacuteconomies

occidentales La concurrence accrue provoque alors un inteacuterecirct pour le systegraveme deacuteveloppeacute

au Japon La production au plus juste se deacuteveloppe ainsi dans lrsquoindustrie automobile agrave partir

des anneacutees 80 et elle se reacutepand encore maintenant dans drsquoautres secteurs

Cette approche qui vise un objectif de zeacutero stock et zeacutero deacutefaut impose la maicirctrise de laquo bout

en bout raquo des processus de production et leur ameacutelioration

Initiation au management copy CRCF ndash J Sornet Page 24 48

2 ndash Organiser lrsquoentreprise

21 ndash Direction et organisation

Diriger une entreprise neacutecessite de lrsquoorganiser (de reacutepartir les tacircches) pour qursquoelle puisse

atteindre ses objectifs Lrsquoorganisation permet de satisfaire un marcheacute en tirant parti des

capaciteacutes actuelles de lrsquoentreprise tout en preacuteparant lrsquoavenir

Lrsquoorganisation reacutesulte freacutequemment drsquoun compromis entre des objectifs situeacutes agrave des niveaux

et des eacutecheacuteances diffeacuterents

Exemples

- le leader des chaises roulantes peut tirer profit de sa structure productive et de son

savoir faire pour entrer sur le marcheacute de la bicyclette eacutelectrique

- ecirctre parfaitement structureacute pour alimenter 90 du marcheacute des disquettes ne preacutepare

pas lrsquoavenir

- srsquoorganiser pour conqueacuterir le marcheacute des tire-bouchons eacutelectriques dans les deux ans

perd de son sens si cela altegravere les moyens neacutecessaires agrave la production drsquoappareils

manuels ancienne mais vitale dont la diminution agrave court terme risque de nuire agrave la

solvabiliteacute de lrsquoentreprise et de la conduire agrave la cessation de paiement

22 ndash Lrsquoorganisation fonctionnelle

La majoriteacute des entreprises adopte une laquo organisation fonctionnelle raquo (celle qui est visible

dans les organigrammes) ougrave des regroupements de personnels et drsquoeacutequipements se font

selon un modegravele hieacuterarchique (laquo line raquo) dans des uniteacutes des services ou des deacutepartements

speacutecialiseacutes Cette organisation peut se deacutecliner agrave lrsquointeacuterieur des divisions des grandes

entreprises quand elles scindent leur activiteacute par zone geacuteographique type drsquoactiviteacute

cateacutegorie de clients hellip

Remarque le terme laquo fonction raquo deacutesigne un rocircle particulier dans le fonctionnement de

lrsquoentreprise

Lrsquoorganisation fonctionnelle diffeacuterencie les activiteacutes de lrsquoentreprise en les regroupant par

meacutetier pour utiliser au mieux les compeacutetences et les moyens (meilleur rendement par la

speacutecialisation lrsquoeacutechange de compeacutetences dans une mecircme uniteacute ou gracircce agrave des eacuteconomies

drsquoeacutechelle)

23 ndash La notion de processus de production

Un processus de production se deacutefinit par la succession drsquoactiviteacutes permettant de satisfaire

un client en transformant des ressources (mateacuterielles financiegraveres humaines) en un produit

bien ou service Le processus doit creacuteer une valeur reconnue par le client

Un processus peut servir un client interne agrave lrsquoentreprise (par exemple en produisant un

composant intervenant dans plusieurs produits ou par la maintenance des machines) aussi

bien qursquoun client final On distingue usuellement

- les processus opeacuterationnels (ou maicirctres) aussi appeleacutes processus meacutetier (business process)

qui satisfont directement les clients finaux (conception et fabrication de produits vente hellip)

- les processus de support et de management (geacuterer les ressources humaines geacuterer

lrsquoinformation geacuterer les ressources financiegraveres hellip) qui ont les processus opeacuterationnels comme

clients

Toutes les actions internes agrave une organisation peuvent srsquointeacutegrer dans des processus qui

conditionnent directement ou indirectement la capaciteacute de lrsquoorganisation agrave satisfaire le

client final ou lrsquousager

Initiation au management copy CRCF ndash J Sornet Page 25 48

Aborder le fonctionnement de lrsquoentreprise par ses processus (approche processus) permet

de mettre en eacutevidence les chaicircnes drsquoactiviteacutes qui conduisent aux produits leurs

dysfonctionnements leurs coucircts la formation des deacutelais et la souplesse (la flexibiliteacute)

disponible pour satisfaire la clientegravele finale Lrsquoameacutelioration des processus a un impact visible

et direct sur chaque produit proposeacute aux clients

Lrsquoapproche processus provoque une eacutevolution de la faccedilon de travailler

- en faisant peacuteneacutetrer la laquo voix du client raquo au plus profond de lrsquoentreprise (et plus seulement

dans les services commerciaux et marketing)

- en mettant en eacutevidence des possibiliteacutes de rationalisation (par regroupement ou impartition

de certaines activiteacutes)

Remarque lrsquoapproche par les activiteacutes et les processus est agrave lrsquoorigine de la meacutethode

de deacutetermination des coucircts laquo ABC raquo - activity based costing

24 ndash Processus et fonctions

Le processus est transversal Il enchaicircne des activiteacutes qui traversent lrsquoentreprise en particulier

les services ou les deacutepartements drsquoune organisation fonctionnelle

Exemple

La division du travail par fonctions induit une charge de coordination pour assurer le

deacuteroulement du processus Elle peut geacuteneacuterer des attentes des erreurs ou des conflits drsquointeacuterecirct

(lrsquoobservation montre que des dysfonctionnements sont tregraves souvent constateacutes lors du

passage drsquoun service agrave un autre)

Organisation fonctionnelle et approche processus visent toutes deux un optimum

eacuteconomique mais leurs logiques sont diffeacuterentes

- le processus vise la satisfaction des clients (prix qualiteacute deacutelais service)

- le deacutecoupage fonctionnel cherche agrave optimiser les moyens (maximiser lrsquoeffet drsquoexpeacuterience

partager des infrastructures profiter de pocircles de compeacutetences hellip) Il apporte une ossature

hieacuterarchique stable souvent indispensable

Organisation fonctionnelle et approche processus sont donc compleacutementaires dans la

majoriteacute des cas et doivent ecirctre combineacutees judicieusement

APPLICATIONS OP

OP1 Deacutefinir flexibiliteacute systegraveme impartition

OP2 Citer huit exemples drsquoinformations essentielles pour optimiser un processus de

fabrication

Direction

Deacutepartement

commercial

(C)

Deacutepartement

administratif et

financier (AF)

Deacutepartement

Etudes (E)

Deacutepartement

Production (P)

Activiteacute

C-x Activiteacute

AF-x Activiteacute

E-x

Activiteacute

P-x

Processus x

Clie

nt

Initiation au management copy CRCF ndash J Sornet Page 26 48

OP3 Deacutegager les principes du toyotisme preacutesenteacute ci-dessous En quoi ce systegraveme est-il

initiateur de lrsquoapproche processus

Taiichi Ohno et le Toyotisme

1 - Extrait drsquoun article de Jacques BARRAUX - 1993 - LExpansion

Taiichi Ohno (1912 ndash 1990) hellip ne se prenait pas pour un visionnaire mais en imposant une

nouvelle faccedilon de produire il a reacuteinventeacute le management hellip tout le monde a entendu parler

des mots qui ont populariseacute le toyotisme dont il est le pegravere le juste-agrave-temps hellip Autant

doutils conccedilus pour lrsquoautomobile et qui ont aujourdhui une application universelle

hellip Taiichi Ohno jeune ingeacutenieur entre chez Toyota alors simple constructeur de machines

textiles Degraves 1926 apparaicirct la notion de jidoka hellip cest lart de transfeacuterer de lintelligence aux

machines pour mieux libeacuterer lintelligence des hommes Tout le contraire du taylorisme qui

juge la machine moins impreacutevisible que lhomme En 1933 Toyota se lance dans lautomobile

en sinspirant des meacutethodes ameacutericaines Mais en 1935 agrave loccasion dun voyage aux Etats-

Unis leacutetat-major de lentreprise revient fascineacute de sa visite dans un supermarcheacute La notion

de juste-agrave-temps va naicirctre de lobservation dune grande surface un lieu ougrave les clients ne

prennent que ce dont ils ont besoin et ougrave les rayons sont reacuteapprovisionneacutes pour compenser

les quantiteacutes preacuteleveacutees Ainsi le systegraveme Toyota est-il deacutejagrave dans la tecircte de ses dirigeants avant

mecircme la Seconde Guerre mondiale un demi-siegravecle avant la reacutevolution informatique et la

segmentation intensive des marcheacutes

hellip des esprits curieux comme Franccedilois Dalle en France tombent alors sous le charme des

formules et des paraboles de Taiichi Ohno En voici deux eacutechantillons

Penser agrave lenvers Cela signifie combattre les ideacutees reccedilues En lespegravece il sagit du fordisme et

du taylorisme Ohno ne croit pas agrave la planification aux effets deacutechelle et dexpeacuterience Il

propose un systegraveme industriel agrave lenvers qui permette de diversifier les produits et de les

fabriquer en petites quantiteacutes Nous ne devons plus ecirctre des paysans qui accumulent des

stocks mais des chasseurs On nimpose pas loffre On traque la demande et on la gegravere en

continu

Que les valleacutees soient hautes et les montagnes peu eacuteleveacutees Plutocirct que de concentrer tous

les efforts sur une production agrave un moment donneacute mieux vaut se doter de structures flexibles

permettant de passer agrave tout instant dune seacuterie agrave une autre Il faut eacuteviter les ruptures et les

secousses aplanir les cycles entretenir des flux reacuteguliers dactiviteacutes diversifieacutees Ce qui

implique de ne pas enfermer les hommes et les eacutequipements dans des speacutecialisations trop

eacutetroites

La flexibiliteacute le travail en groupe le refus de la dictature des machines la polyvalence et

surtout lattention constante aux signaux eacutemis par le marcheacute nappartiennent plus au

toyotisme Ces notions sont les fondements du nouvel art dorganiser de vendre et de

produire dans lindustrie comme dans les services hellip

2 - Quelques notions cleacutes

Taiichi Ohno a imagineacute la meacutethode des laquo cinq pourquoi raquo qui consiste agrave se poser cinq fois de

suite la question laquo pourquoi raquo sur le mecircme sujet de faccedilon agrave deacutecouvrir la veacuteritable cause

drsquoun problegraveme Cette meacutethode peut ecirctre appliqueacutee agrave tous les niveaux et permettre

notamment aux agents de fabrication de proposer de veacuteritables ameacuteliorations de la

production

La recherche de la qualiteacute totale (pas de deacutefaut des produits pas de rebuts pas de deacutefaut

des processus) accompagne la deacutemarche de Toyota La qualiteacute a un coucirct compenseacute par

des ventes accrues par lrsquoeacuteconomie des mesures palliatives aux deacutefauts

Initiation au management copy CRCF ndash J Sornet Page 27 48

Fiche OP1 ndash Benchmarking et processus

Le laquo benchmarking raquo consiste agrave comparer le fonctionnement de plusieurs systegravemes pour en

faire notamment ressortir les meilleures pratiques (laquo best practices raquo) Cette technique est

utiliseacutee depuis les anneacutees 80 pour ameacuteliorer la performance des entreprises Elle impose agrave

lrsquoentreprise drsquoeacutevaluer et de remettre en question ses propres modes de fonctionnement afin

de les faire eacutevoluer agrave la lueur de ce qui se fait ailleurs

Le benchmarking permet drsquoameacuteliorer les processus agrave moindre risque en fixant des objectifs

baseacutes sur des faits et donc plus facilement accepteacutes

Une classification des processus en tant que base de reacuteflexion a eacuteteacute eacutetablie aux USA par

lrsquolaquo International Benchmarking Clearinghouse raquo de lrsquoAPQC (american productivity and

quality center) en collaboration avec plusieurs dizaines drsquoentreprises

Elle se reacutesume ainsi

Le terme laquo reengineering raquo (la re-conception ou laquo reacuteingeacutenieacuterie raquo) des processus deacutesigne un

projet drsquoameacutelioration radicale des performances (de 20 agrave 50 ou plus) Il neacutecessite une

parfaite adheacutesion de la direction la constitution drsquoune petite eacutequipe de projet brillante

connaissant parfaitement les activiteacutes de lrsquoentreprise et il peut inclure un benchmarking

Le reengineering provoque geacuteneacuteralement la reacuteduction du nombre de niveaux hieacuterarchiques

(laquo delayering raquo) et lrsquoaccroissement du pouvoir de deacutecision des employeacutes (laquo empowerment raquo

ou laquo empouvoirement raquo) Bien qursquoy conduisant parfois il ne doit pas ecirctre confondu avec la

reacuteduction des activiteacutes (laquo downsizing raquo ou restructuration) et lrsquoexternalisation (laquo outsourcing raquo)

Pro

ce

ssu

s

op

eacutera

tio

nn

els

Pro

ce

ssu

s d

e m

an

ag

em

en

t e

t d

e

sup

po

rt

1 ndash

Comprendre

le marcheacute et

les clients (besoins

satisfaction)

2 ndash

Deacutevelopper

vision et

strateacutegie (contexte

concurrence)

3 ndash

Creacuteer

produits

services

processus

(concevoir

ameacuteliorer)

4 ndash

Marketing et

vente

5 ndash

Produire et

livrer (industrie

dont

ameacutelioration

processus)

6 ndash

Produire et

livrer (services)

7 ndash

Facturer et

servir les

clients (apregraves-

vente

reacuteclamations)

8 ndash Deacutevelopper et geacuterer les ressources humaines

9 ndash Geacuterer les systegravemes drsquoinformation

10 ndash Geacuterer les ressources financiegraveres et les actifs

11 ndash Appliquer un programme environnemental

12 ndash Geacuterer les relations exteacuterieures (actionnaires banques lois relations publiques hellip)

13 ndash Geacuterer lrsquoameacutelioration et le changement (eacutevaluer mesurer motiver qualiteacute totale)

Initiation au management copy CRCF ndash J Sornet Page 28 48

Fiche OP2 ndash Lrsquoorganisation par processus

Lrsquoeacutevolution drsquoune organisation aux activiteacutes reacutepeacutetitives vers lrsquoapproche processus est

geacuteneacuteralement progressive et se met en place par paliers

La mise en œuvre drsquoun veacuteritable management par processus doit ecirctre preacuteceacutedeacutee quand

lrsquoactiviteacute de lrsquoentreprise est complexe drsquoun recensement (une laquo cartographie des

processus raquo) pour mettre en eacutevidence les processus ou les familles de processus cleacutes critiques

pour le succegraves de lrsquoentreprise ougrave les efforts seront prioritaires

Des responsables de processus (laquo process owners raquo) sont ensuite deacutesigneacutes

Le responsable doit concevoir ses processus puis apregraves leur mise en œuvre assurer les

coordinations neacutecessaires les ameacuteliorer et les repreacutesenter aupregraves de la direction

Quand une structure par processus est mise en place des opeacuterateurs exeacutecutants

preacuteceacutedemment regroupeacutes dans les fonctions peuvent ecirctre affecteacutes aux processus et

drsquoanciens responsables de fonctions peuvent devenir des experts au service des processus

Lrsquoorganisation par processus peut imposer un degreacute eacuteleveacute drsquointeacutegration des activiteacutes donc

une polyvalence accrue des personnels et une reacuteduction des niveaux hieacuterarchiques

Elle neacutecessite pour le moins des compeacutetences eacutelargies au niveau des responsables de

processus (organisation administration technique hellip) dont le nombre doit rester limiteacute

(quelques dizaines au plus)

Sauf dans de tregraves petites structures lrsquoorganisation par processus se plaque geacuteneacuteralement sur

une structure plus classique

Initiation au management copy CRCF ndash J Sornet Page 29 48

ELEMENTS DE CORRIGE OP

OP1 Deacutefinir

Flexibiliteacute = adaptation au besoin (horaire variable chaicircnes robotiseacutees)

Systegraveme = ensemble organiseacute dans un but boicircte noire (sanguin nerveux meacutetrique laquo D raquo)

Impartition = sous-traitance ou externalisation (seacuteparation) drsquoactiviteacutes faire appel agrave des

partenaires plutocirct que faire soi-mecircme

OP2 Citer huit exemples drsquoinformations essentielles pour orienter lrsquooptimisation drsquoun processus

Montant des stocks (approvisionnements et produits finis)

Temps drsquoattente

Taux drsquoactiviteacute des ateliers

Rebuts

Deacutelai de production

Taux de reacuteclamations clients (qualiteacute)

Temps passeacutes en retouches finales

Turn over

Nombre drsquoarrecircts maladie

Accidents du travail

Dureacutee des arrecircts machines

OP3 Deacutegager les principes du toyotisme preacutesenteacute dans la fiche 31 En quoi ce systegraveme

repose trsquoil sur lrsquoapproche processus

Produire la quantiteacute juste neacutecessaire (agrave la demande) donc eacuteviter les stocks

Flexibiliteacute intelligence des chaicircnes de production

Qualiteacute (eacuteviter le coucirct de la non-qualiteacute)

La notion de processus est implicite ainsi que la chaicircne de valeur client

Initiation au management copy CRCF ndash J Sornet Page 30 48

DEFIS ET TENDANCES DU MANAGEMENT

Les meacutethodes de management se deacuteveloppent pour affronter le contexte eacuteconomique

Ce chapitre preacutesente les deacutefis auxquels le management contemporain doit faire face

1 ndash Lrsquoeacutevolution eacuteconomique contemporaine

A mesure que lrsquoactiviteacute eacuteconomique mondiale srsquoaccroicirct que la technologie eacutevolue les

changements sont de plus en plus rapides Ils introduisent des situations ineacutedites auxquelles les

entreprises doivent srsquoadapter en cherchant de nouvelles solutions de management Les trois

derniegraveres deacutecennies ont eacuteteacute notamment marqueacutees par les pheacutenomegravenes suivants (que nous

listons sans tenir compte des liens pouvant exister entre eux)

Pheacutenomegravene Traduction Effets

Deacute reacuteglementation

globalisation

financiegravere

titrisation

Libre circulation des capitaux accegraves

facile des particuliers au marcheacute

boursier (directement ou par

lrsquointermeacutediaire des OPCVM et SICAV)

Monteacutee en puissance du financement

des entreprises sur le marcheacute boursier

Fonds de pension

(retraites) et fonds

souverains (eacutetats)

Poids boursier important drsquoinvestisseurs

institutionnels qui cherchent un haut

rendement financier (dividendes ou

valorisation boursiegravere)

Pression sur les grandes entreprises

influence sur les strateacutegies

Mondialisation Liberteacute des eacutechanges internationaux Accroissement de la concurrence

recherche drsquoavantages eacuteconomiques

par la deacutelocalisation (biens et

services) la concentration des efforts

(recentrage) problegravemes drsquoemploi

multiplication des transports perte

drsquoinfluence des politiques

Baisse de lrsquoemploi

occidental

(notamment

industriel)

Moins de fabrications fabrications

automatiseacutees recours aux moyens

informatiques

Activiteacute reporteacutee sur le commerce la

conception et les services chocircmage

charge sociale

Restructurations Optimisation des entreprises

abaissement des coucircts augmentation

des marges recherche drsquoune taille

critique (eacuteconomies drsquoeacutechelle poids

sur le marcheacute)

Recentrages externalisations fusions

deacutelocalisations constitution de grands

groupes

NTIC (nouvelles

technologies de

lrsquoinformation et de

la communication)

Mise en œuvre des reacuteseaux (dont

internet) et drsquoapplications

informatiques communicantes

Nouvelles formes de commerce

marcheacute international deacutelocalisation

du travail intellectuel reacuteorganisation

de la distribution

Rareacutefaction relative

des matiegraveres

premiegraveres

Recherche de substituts exploration

miniegravere coucircts drsquoexploitation des

gisements accrus

Augmentation des coucircts variations

erratiques du cours des matiegraveres

deacutestabilisations politiques

Evolution

geacuteopolitique et

eacuteconomique

mondiale

Chute de lrsquoURSS transformation des

eacuteconomies collectivistes pays

eacutemergents (Chine Inde Breacutesil Russie)

()

Accroissement de la population

mondiale (4 agrave 6 7 milliards de 1970 agrave

2008)

Libeacuteralisme sans frein () nouvelles

puissances eacuteconomiques

opportuniteacutes de deacuteveloppement

nouveau partage des ressources

ineacutegaliteacutes baisse du soutien aux PVD

laquo Terrorisme raquo Actions armeacutees pression de groupes

armeacutes non gouvernementaux

Deacutestabilisations reacutegionales charge

des deacutepenses militaires

Deacuteveloppement

durable

Recherche drsquoune croissance eacutequitable

et respectueuse de lrsquoenvironnement

Pression sur les entreprises (eacutetats

associations de consommateurs

eacutecologistes ONG)

() Reacutecemment quelques affaires (Enron laquo subprimes raquo Vivendi Universal Socieacuteteacute

Geacuteneacuterale Airbus par exemple) et agrave plus grande eacutechelle la crise financiegravere de 2008 ont

montreacute les dangers drsquoune libeacuteralisation sans controcircles suffisants

Initiation au management copy CRCF ndash J Sornet Page 31 48

() Des alliances eacuteconomiques naissent entre pays eacutemergents (notamment en

ameacuterique centrale creacuteation de la Banque du Sud en 2008 par exemple) et lrsquoon

commence agrave imaginer une baisse progressive de lrsquoinfluence eacuteconomique des Etats

Unis

2 ndash Les deacutefis actuels du management

21 ndash Les grandes orientations

Lrsquoeacutevolution eacuteconomique suggegravere quelques pistes parfois concurrentes pour lrsquoaction du

manager contemporain On y retrouve au premier plan la construction drsquoune vision qui est

une composante commune du leadership

Objectif du manager

pour lrsquoentreprise

Justification Facteurs de reacuteussite

Construire une vision Eclairer lrsquoavenir de lrsquoentreprise partager

un but souder motiver

Effort de reacuteelle prospection

volontarisme de la direction

bonne communication

Reacuteactiviteacute et flexibiliteacute

(sous tous les aspects

agrave tous niveaux)

Srsquoadapter rapidement au marcheacute Bonne organisation des processus

personnel compeacutetent autonome

et motiveacute structure hieacuterarchique

alleacutegeacutee robotisation

Deacutegager des profits Reacutemuneacuterer les apporteurs de capitaux

srsquoautofinancer

Ajuster coucircts et structures

Exploiter les nouvelles

technologies

Reacuteactiviteacute ajuster coucircts et deacutelais

reacutepondre au marcheacute suivre les clients

Organiser le SI de faccedilon

pertinente eacuteviter le coucirct excessif

drsquoinvestissements trop en

laquo pointe raquo (laquo essuyer les placirctres raquo)

utiliser judicieusement les services

exteacuterieurs

Bacirctir des alliances

(contrats fusions)

Deacutevelopper une activiteacute limiter les coucircts

de transaction () atteindre la taille

critique et de meilleurs rendements se

recentrer sur une activiteacute profitable

Dominer les processus se donner

une identiteacute lisible externaliser se

doter drsquoune capaciteacute financiegravere

suffisante

Valoriser lrsquoimage Attirer les clients favoriser les alliances

donner confiance (apporteurs de fonds

employeacutes clients partenaires socieacuteteacute

civile)

Instaurer des regravegles de

gouvernance inteacutegrer le

deacuteveloppement durable

respecter lrsquoenvironnement

Geacuterer les risques Faire face aux aleacuteas eacuteconomiques et

technologiques (conjoncture politiques

accidents malveillance)

Creacuteer un systegraveme drsquoalerte geacuterer

la crise (reacuteaction raisonneacutee

sceacutenarios poursuite de

lrsquoexploitation dans un contexte

instable) mise en place de

proceacutedures drsquoapprentissage pour

ameacuteliorer les reacuteactions au fil du

temps

Geacuterer le changement Faire face agrave lrsquoeacutevolution de la demande

la pression sur les prix la variation des

performances financiegraveres la

concurrence la globalisation des

marcheacutes lrsquoeacutevolution technologique aux

fusions ou alliances aux changements

de reacuteglementation de direction hellip ()

Bonne communication pour

donner du sens au changement

et obtenir lrsquoadheacutesion du personnel

Rassembler et geacuterer les

connaissances former le

personnel

Innover Garder un avantage concurrentiel se

diffeacuterencier

Veille technologique et

commerciale investissement

Ouverture

internationale

Elargir le marcheacute saisir les opportuniteacutes Veille commerciale partenariats

() La theacuteorie des coucircts de transaction deacuteveloppeacutee par OE Williamson dans les

anneacutees 70 integravegre les coucircts lieacutes au recours au marcheacute (recherche et choix drsquoun

fournisseur neacutegociation reacutedaction de contrat suivi des eacutechanges risque de rupture

Initiation au management copy CRCF ndash J Sornet Page 32 48

drsquoapprovisionnement hellip) On peut en conclure que lrsquointeacutegration de diffeacuterentes

activiteacutes agrave lrsquoentreprise (la laquo firme raquo) preacutesente des avantages Mais des coucircts de

transaction internes doivent aussi ecirctre consideacutereacutes (preacuteparation organisation

surveillance hellip) et certaines formes de coopeacuteration continue avec les fournisseurs

permettent de reacuteduire le coucirct des transactions externes

() drsquoapregraves laquo Les meilleures pratiques de management raquo - Brilman Heacuterard ndash EO

Une eacutetude du Conference Board (2002) liste les deacutefis du management vus par 700 leaders

mondiaux Soit en reacutesumeacute avec indication du score correspondant

1 ndash Fideacuteliser les clients (42)

2 ndash Reacuteduire les coucircts (38)

3 ndash Accroicirctre flexibiliteacute et reacuteactiviteacute (29)

4 ndash Amener les employeacutes agrave adheacuterer aux valeurs et visions de lrsquoentreprise (26)

5 ndash Deacutevelopper et retenir les leaders (25)

6 ndash Geacuterer acquisitions et alliances (24)

7 ndash Accroicirctre lrsquoinnovation (20)

En fin de classement citoyenneteacute et reacuteputation (4) et ameacutelioration de la diversiteacute (3)

22 ndash Les techniques disponibles

Pour faire face aux deacutefis le manager dispose de nouveaux concepts et de nouvelles

techniques Le tableau ci-dessous en donne un reacutesumeacute et indique les domaines qursquoils

influencent principalement

Initiation au management copy CRCF ndash J Sornet Page 33 48

Principaux concepts techniques outils Incidence principale sur

Internet

- e-commerce (commerce eacutelectronique site

entreprise)

- CRM ou GRC (gestion de la relation client)

- e-procurement (gestion des approvisionnements

par le reacuteseau)

- messagerie eacutelectronique

- e-recrutement

Vente accegraves au marcheacute

Relation client reacuteactiviteacute personnalisation

fideacutelisation

Deacutelais coucircts

Communication transfert de donneacutees (piegraveces

jointes) tous domaines

Communication recrutement

Intranet reacuteseau drsquoentreprise SI

- knowledge management (gestion des

connaissances)

- e-learning (apprentissage en ligne)

- plateforme de travail collaboratif (groupware)

- workflow (circulation eacutelectronique de

documents enchaicircnement de processus)

- e-RH portail RH (libre accegraves aux postes agrave

pourvoir informations candidatures hellip)

- PGI (progiciel de gestion inteacutegreacute) ou ERP

Innovation capaciteacute au changement veille

documentaire

Formation du personnel accompagnement des

changements

Coordination communication interne

Coordination

Communication interne (voire internet en

externe) reacuteduction des coucircts climat drsquoentreprise

recrutement plans de carriegraveres hellip

Coucircts fiabiliteacute du systegraveme drsquoinformation deacutelais

processus (continuiteacute inteacutegration)

Logistique inteacutegreacutee

Supply Chain Management (SCM) gestion de la

logistique (incluant les approvisionnements)

Processus deacutelais coucircts

Externalisation

Valorisation du capital humain

GPEC (gestion preacutevisionnelle des emplois et

compeacutetences)

Coaching

Reacuteactiviteacute de lrsquoentreprise conservation des

compeacutetences rendements individuels turn-over

adaptation des compeacutetences motivation

Efficaciteacute individuelle controcircle reacutegulation

progregraves processus

Approche processus

Optimisation des processus

Deacutemarche qualiteacute totale (TQM ndash total quality

management)

Empowerment (empouvoirement)

Benchmarking reacuteingeacutenieacuterie

Coucircts marges qualiteacute deacutelais flexibiliteacute

externalisation eacutelargissement des compeacutetences

organisation

Ameacutelioration des processus (meacutetiers et supports)

Autonomie compeacutetences des employeacutes

Ameacutelioration des processus restructuration

Management par la valeur

Parties prenantes

Satisfaction des parties prenantes financement

motivation collaborations hellip

Collaboration inter organisations

Reacuteseaux drsquoentreprises alliances

EDI (eacutechange de donneacutees informatiseacutees) extranet

Impartition externalisation (outsourcing)

Coucircts recentrage investissements lancement

drsquoactiviteacute

Coucircts reacuteactiviteacute deacutelais relations avec

lrsquoadministration

Coucircts recentrage limitation des investissements

Ethique drsquoentreprise

Gouvernance drsquoentreprise (mode de direction

encadreacute par des regravegles)

Rocircle socieacutetal deacuteveloppement durable

environnement

Image de lrsquoentreprise reacutegulation du top

management relations actionnaires

Image peacutenaliteacutes et amendes objectifs

strateacutegiques

Initiation au management copy CRCF ndash J Sornet Page 34 48

23 ndash Le rocircle socieacutetal des entreprises

La responsabiliteacute socieacutetale de lrsquoentreprise (RSE) deacutesigne le rocircle qursquoelle prend dans la socieacuteteacute

au-delagrave de son activiteacute purement geacuteneacuteratrice de profit On parle aussi drsquoentreprise citoyenne

La RSE est indissociable du deacuteveloppement durable de porteacutee mondiale et dont les trois

piliers sont

- eacuteconomique (favoriser le deacuteveloppement les eacutechanges internationaux)

- social (accegraves aux soins eacuteducation conditions de travail hellip)

- environnemental (pollution preacuteservation des ressources hellip)

La RSE integravegre notamment une preacuteoccupation sociale de lrsquoentreprise vis-agrave-vis de ses salarieacutes

(seacutecuriteacute et santeacute au travail juste reacutemuneacuteration deacuteveloppement personnel hellip) Elle conduit agrave

tenir compte dans le management drsquoune vision exteacuterieure agrave lrsquoentreprise qui peut avoir des

reacutepercussions possibles sur son activiteacute eacuteconomique

Lrsquoentreprise peut aussi tirer avantage drsquoune deacutemarche responsable par la baisse de certains

coucircts (plus faibles consommations drsquoeacutenergies reacuteduction des transports hellip)

Le rocircle socieacutetal de lrsquoentreprise a eacuteteacute reconnu en France par la loi laquo NRE raquo de 2001 (loi sur les

nouvelles reacutegulations eacuteconomiques) qui oblige les socieacuteteacutes franccedilaise coteacutees sur un marcheacute

reacuteglementeacute agrave rendre compte dans leur rapport annuel de leur gestion sociale et

environnementale au travers de leur activiteacute

Article 116 de la loi Le rapport viseacute agrave larticle L 225-102 rend compte hellip laquo Il comprend

eacutegalement des informations dont la liste est fixeacutee par deacutecret en Conseil dEtat sur la

maniegravere dont la socieacuteteacute prend en compte les conseacutequences sociales et

environnementales de son activiteacute Le preacutesent alineacutea ne sapplique pas aux socieacuteteacutes

dont les titres ne sont pas admis aux neacutegociations sur un marcheacute reacuteglementeacute raquo

Une norme ISO 14000 integravegre ces preacuteoccupations et des taxes eacutecologiques sont

progressivement creacutees

3 ndash Le management par la valeur

31 ndash De lrsquoanalyse au management par la valeur

Lrsquoanalyse de la valeur est neacutee en 1947 aux Etats-Unis (General Electrics) Cette technique

consiste agrave eacutelaborer des produits conformes aux attentes de la clientegravele mais sans excegraves pour

trouver un bon compromis entre valeur pour le client et coucirct Le produit optimal est deacutefini agrave

partir drsquoenquecirctes qui deacuteterminent le besoin client (ou plutocirct drsquoun client laquo type raquo)

Exemple il est inutile de concevoir un petit veacutehicule citadin capable de parcourir

500 000 km sans avarie compte tenu des effets de mode et du faible kilomeacutetrage

annuel Par contre le marcheacute peut exiger un fonctionnement sans faille sur 150 000 km

soit dix ans en moyenne ce qui conditionne les coucircts de production

Cette recherche drsquoun ajustement de valeur au besoin des clients eacutetait un preacutecurseur du

management par la valeur qui recherche plus largement la creacuteation de valeur pour

chacune des parties prenantes de lrsquoentreprise tout en lui meacutenageant un reacutesultat suffisant

Plus geacuteneacuteralement le management par la valeur est deacutefini par une norme europeacuteenne (EN

12973)

Le management par la valeur est un style de management particuliegraverement destineacute agrave

mobiliser les individus agrave deacutevelopper les compeacutetences et agrave promouvoir les synergies et

Initiation au management copy CRCF ndash J Sornet Page 35 48

linnovation avec pour objectif la maximisation de la performance globale dun

organisme Le management par la valeur apporte une nouvelle faccedilon dutiliser nombre

de meacutethodes de management existantes Il est en coheacuterence avec le Management

de la qualiteacute

Cette approche du management pose de nombreuses questions notamment quelles

prioriteacutes et quelles valeurs attribuer aux parties prenantes comment appreacutehender la

perception par les parties prenantes de la valeur qui leur est affecteacutee

32 ndash La valeur client

Le processus drsquoeacutelaboration drsquoun produit qui consomme des ressources coucircteuses doit creacuteer

une valeur suffisante pour provoquer lrsquoachat par le client final La production drsquoune valeur

reconnue par le client est vitale pour lrsquoentreprise mais sa deacutetermination est parfois complexe

La valeur du produit perccedilue par le client integravegre des eacuteleacutements en partie subjectifs

- une valeur drsquousage (le produit reacutepond agrave un besoin)

- une valeur drsquoestime (lrsquoimage apporteacutee par le produit un aspect affectif)

- une valeur drsquoeacutechange (deacuteduite de lrsquoespoir de revente du produit)

Valeurs drsquousage drsquoestime et drsquoeacutechange deacutependent implicitement de la qualiteacute (un bien peu

fiable est impropre agrave lrsquousage attendu de mauvaise qualiteacute notoire il nrsquoapporte pas une

image positive et ses deacutefauts connus nuisent agrave sa revente) Une eacutevaluation de la qualiteacute

intervient donc dans la valeur perccedilue du produit

Par ailleurs le client considegravere le coucirct drsquoobtention du produit (les charges qursquoil doit supporter

pour acqueacuterir le produit lrsquoeffort qursquoil doit faire pour trouver le produit et les frais de mise agrave

disposition)

Le prix perccedilu par le client est geacuteneacuteralement supeacuterieur au prix de vente

Le client achegravete theacuteoriquement le produit qui preacutesente la diffeacuterence valeur perccedilue ndash prix

perccedilu la plus favorable ou le meilleur rapport prix perccedilu qualiteacute perccedilue et dans certains

cas celui qui a le prix produit le plus bas

Remarque les valeurs du scheacutema ci-dessus changent durant le cycle de vie du produit

(un nouveau produit peut avoir une valeur perccedilue plus eacuteleveacutee qursquoen fin de vie) La

valeur client ne peut ecirctre eacutevalueacutee que par enquecirctes et ne peut donc ecirctre deacutefinie avec

certitude

La notion de laquo satisfaction client raquo conseacutecutive agrave une vente influence aussi le prix produit et

le prix perccedilu

- lrsquoentreprise gagne sur les coucircts de recherche de clientegravele

- le client nrsquoa pas agrave rechercher un nouveau fournisseur et beacuteneacuteficie drsquoun coucirct drsquoobtention

plus bas

valeur perccedilue client

prix perccedilu client

coucirct produit Marge (valeur creacuteeacutee pour

lrsquoentreprise)

euros

prix produit

Valeur creacuteeacutee

pour le client

Initiation au management copy CRCF ndash J Sornet Page 36 48

La satisfaction du client deacutepend de facteurs qualitatifs aussi divers que la fiabiliteacute du produit

la vitesse de reacuteaction du fournisseur lrsquoattitude des commerciaux lrsquoefficaciteacute du service

apregraves-vente la netteteacute des contrats ou la justesse de la facture

Valeur perccedilue coucirct marge et satisfaction reacutesultent de processus allant de la conception du

produit jusqursquoagrave sa livraison et son apregraves-vente La deacutemarche laquo processus raquo et lrsquolaquo analyse de la

valeur raquo en forccedilant la recherche de solutions efficientes agrave tout niveau administratif

technique commercial et apregraves-vente sont donc neacutecessaires pour bien positionner

lrsquoentreprise sur son marcheacute

Pour autant le risque commercial ne peut jamais ecirctre annuleacute et lrsquooffre de lrsquoentreprise ne

satisfait geacuteneacuteralement pas en milieu concurrentiel tous ses clients potentiels

33 - La creacuteation de valeur pour les autres parties prenantes

Les salarieacutes

La creacuteation drsquoune valeur suffisante pour les salarieacutes est reconnue comme neacutecessaire car des

observations montrent que la satisfaction des clients en deacutepend Moins souvent eacutevoqueacutee en

peacuteriode de chocircmage elle nrsquoest prioritaire que pour les employeacutes dont lrsquoentreprise souhaite

conserver les compeacutetences

La laquo valeur salarieacute raquo ne comprend pas que le salaire Le sentiment drsquoappartenance agrave un

groupe la reconnaissance lrsquoaccomplissement de soi et la construction professionnelle en

sont des eacuteleacutements importants Comme pour les clients on doit ainsi distinguer la reacutetribution

perccedilue du salaire objectif

Les actionnaires

Lrsquoactionnaire apporte des fonds propres agrave lrsquoentreprise en contrepartie de titres parfois

neacutegociables en bourse et assortis drsquoun droit de vote en assembleacutee geacuteneacuterale La valeur

attribueacutee aux actionnaires est servie en termes moneacutetaires (dividende ou augmentation de la

valeur du titre neacutegociable)

Remarque des facteurs non moneacutetaires comme lrsquoimage de lrsquoentreprise qui deacutepend

en partie de sa communication peuvent influencer la deacutecision drsquoachat de vente ou

de conservation des titres par lrsquoactionnaire

Reacutetribution perccedilue euros

Salaire objectif

Avantage non

moneacutetaire de

lrsquoemploi

Initiation au management copy CRCF ndash J Sornet Page 37 48

Compte tenu de lrsquoimportance croissante de lrsquoactionnariat dans le financement des grandes

entreprises coteacutees en bourse et notamment des investisseurs institutionnels comme les fonds

de pension des indicateurs speacutecifiques ont eacuteteacute introduits pour appreacutecier la performance des

entreprises vue par les actionnaires Par exemple la valeur ajouteacutee eacuteconomique (EVA reg

economic value added marque deacuteposeacutee de Stern Stewart ou VAE ndash valeur ajouteacutee

eacuteconomique parfois deacutenommeacutee VEC ndash valeur eacuteconomique creacuteeacutee) qui prend en compte le

coucirct du capital

LrsquoEVA correspond tregraves scheacutematiquement au calcul suivant

EVA = (PO) profit opeacuterationnel ndash (C) coucirct du capital X (CE) capitaux employeacutes

LrsquoEVA neacutecessite en pratique des retraitements assez complexes Le PO peut se deacuteterminer

selon les principes suivants

- PO = reacutesultat drsquoexploitation (avant inteacuterecircts) ndash impocirct

- PO = beacuteneacutefice courant (tenant compte des inteacuterecircts) + inteacuterecircts ndash eacuteconomie drsquoimpocirct sur les

inteacuterecircts (on exclue les eacuteleacutements financiers et lrsquoimpocirct correspondant) ndash impocirct

- lrsquoimpocirct pris en compte correspond au profit opeacuterationnel consideacutereacute (dans les cas courants agrave

13 du PO)

C = taux moyen de reacutemuneacuteration du capital (reacutesultant par exemple du dividende exigeacute de

certains investisseurs et des taux drsquoemprunts bancaires)

CE = capitaux propres et dettes portant inteacuterecirct

Remarque le profit opeacuterationnel ou reacutesultat opeacuterationnel correspond au NOPAT ndash net

operating profit after tax - anglo-saxon LrsquoEVA est eacutegale au NOPAT diminueacute de la

reacutemuneacuteration des capitaux

Exemple lrsquoentreprise X dispose drsquoun capital de 2 500 000 euro et reacutealise un beacuteneacutefice net

drsquoimpocirct de 450 000 euro (taux 33 13) Un dividende de 6 doit ecirctre verseacute aux

actionnaires et la banque lui a accordeacute un precirct de 1 200 000 euro agrave 4 Les autres

constituants des reacutesultats financier et exceptionnel sont neacutegligeables

Reacutesultat opeacuterationnel = 450 000 + 004 x 1 200 000 x 23 = 482 000 euro

Coucirct du capital = 006 x 2 500 000 + 004 x 1 200 000 x 23 = 182 000 euro

EVA = 300 000 euro

Coucirct moyen pondeacutereacute du capital (C) = (004 x 1 200 000 x23 + 006 x 2 500 000)

3 700 000 Soit 492

Si lrsquoEVA est positive lrsquoentreprise creacuteeacutee de la valeur apregraves reacutemuneacuteration des capitaux et sa

valeur boursiegravere doit augmenter

Lrsquoutilisation de lrsquoEVA comme indicateur influence le management de lrsquoentreprise car il y a

trois moyens pratiques drsquoaugmenter lrsquoEVA

- augmenter le reacutesultat opeacuterationnel

- lancer des investissements ayant une rentabiliteacute supeacuterieure agrave C

- eacuteliminer les activiteacutes ayant une rentabiliteacute infeacuterieure agrave C

Remarque lrsquoutilisation sans nuance de lrsquoEVA comme critegravere de management peut

poser problegraveme Le calcul de lrsquoEVA repose sur des ajustements comptables il est donc

sujet agrave manipulations (provisions capitalisation ou non de la RD hellip) Par ailleurs le

critegravere laquo EVA raquo pris isoleacutement peut conduire agrave chercher la rentabiliteacute agrave court terme agrave

reacuteduire les investissements prospectifs et donc nuire agrave terme au deacuteveloppement de

lrsquoentreprise

Initiation au management copy CRCF ndash J Sornet Page 38 48

Les fournisseurs reccediloivent le paiement de leurs factures plus ou moins rapidement (le deacutelai

de paiement repreacutesente une valeur consentie au fournisseur)

Lrsquoentreprise peut accroicirctre la valeur apporteacutee agrave ses fournisseurs par des actions cibleacutees

comme une contribution agrave la formation de leurs personnels certains transferts de

technologie ou de savoir faire agrave des sous-traitants une coopeacuteration suivie favorisant leur

deacuteveloppement lrsquointeacutegration agrave des campagnes de promotion

A noter que la valeur consentie aux fournisseurs peut avoir une influence sur la qualiteacute et les

deacutelais de livraison des produits

La collectiviteacute reccediloit des taxes et parfois des prestations en nature par deacutefaut ou explicites

(effort de preacuteservation de lrsquoenvironnement ameacutenagement du territoire par les implantations

aide mateacuterielle agrave des projets participation agrave la formation par exemple)

APPLICATIONS DT

DT1 Deacutefinir expliquer deacutereacuteglementation socieacutetal eacuteconomies drsquoeacutechelle coaching EDI

gouvernance

DT2 Deacuteterminer en quoi la deacutemarche TQM srsquoinscrit dans les deacutefis actuels du management

DT3 Apregraves avoir consulteacute les documents ci-dessous extraits du site drsquoAir France

(httpdeveloppement-

durableairfrancecomFRfrlocaldemarcheN4_positionnement_pphtm)

exposer les enjeux et les limites de la RSE et de la gestion des parties prenantes

Initiation au management copy CRCF ndash J Sornet Page 39 48

Dialogue avec les parties prenantes

Initiation au management copy CRCF ndash J Sornet Page 40 48

Attentes des parties prenantes

Initiation au management copy CRCF ndash J Sornet Page 41 48

Creacuteation de valeur pour les parties prenantes

La creacuteation de valeur pour les parties prenantes est au cœur de la strateacutegie du Groupe Le scheacutema de

distribution financiegravere ci-dessous donne un aperccedilu de la distribution des recettes du Groupe aux

diffeacuterentes parties prenantes actionnaires collaborateurs fournisseurs pouvoirs publics

collectiviteacutes locales etc

Initiation au management copy CRCF ndash J Sornet Page 42 48

Fiche DT1 ndash Extrait du sommaire de laquo Problegravemes eacuteconomiques raquo No 2894

La gestion des entreprises bouleverseacutee par les technologies de linternet

Reacutealiteacutes industrielles - Annales des Mines Jean-Michel Yolin

Avec lavegravenement de linternet les processus de conception de production et de vente sont

radicalement remis en cause Quel que soit le secteur dactiviteacute les technologies de linternet

permettent en effet de reacuteduire les deacutelais et de passer dun processus discontinu agrave un processus

continu Lorganisation des entreprises et leur mode de gestion en sont profondeacutement bouleverseacutes

tant au niveau individuel que collectif Linternet rend ainsi possible la reacutealisation dobjectifs que les

entreprises cherchaient agrave atteindre depuis longtemps sans y parvenir meilleure eacutecoute du client

travail sans stocks en flux tendu hieacuterarchies plates autorisant une grande reacuteactiviteacute flexibiliteacute dans

lorganisation et loutil de production acceacuteleacuteration du renouvellement des produits entreprises en

reacuteseau ougrave chacune se recentre sur son cœur de meacutetier etc

Le laquo knowledge management raquo ou comment geacuterer les connaissances

Document de travail du LAMSADE - Michel Grundstein

Peter Drucker lavait preacutedit le capital immateacuteriel eacutetait voueacute agrave devenir un facteur de compeacutetitiviteacute

pour lentreprise La libeacuteralisation des eacutechanges acceacutelegravere les processus de deacutecision de lentreprise

et implique que lassimilation des informations soit agrave la fois de meilleure qualiteacute et plus rapide Ainsi

la fonction qui consiste agrave manager les connaissances au sein de lentreprise savegravere primordiale

Bien que la prise de conscience de limportance du capital immateacuteriel ait eacuteteacute tardive - le concept

de knowledge management est apparu en France aux Etats-Unis et au Japon au milieu des

anneacutees 1990 - agrave lheure actuelle lorganisation de leacutechange dinformations et le partage des

connaissances sont devenus des facteurs cleacutes dune gestion performante de lentreprise Ils

doivent sinscrire dans un projet global destineacute agrave mettre en valeur les savoirs et les savoir-faire

individuels et collectifs

Les leccedilons du laquo coaching raquo pour le management de la qualiteacute

Humanisme et Entreprise - Martine Brasseur

Parmi les nouvelles formes de management en vogue dans les entreprises le coaching figure en

bonne place Appliqueacute au management de la qualiteacute il sagit dune pratique

daccompagnement destineacutee agrave initier et agrave faciliter le processus de deacuteveloppement dun individu

La deacutemarche consiste agrave affirmer que tout individu est en quecircte de qualiteacute agrave condition toutefois

de ne pas lui imposer des contraintes lempecircchant de progresser On considegravere notamment les

erreurs comme potentiellement feacutecondes En deacutefinitive le coach donne au coacheacute la permission

de reacuteussir en lui donnant aussi la permission deacutechouer

Initiation au management copy CRCF ndash J Sornet Page 43 48

Fiche DT2 ndash Management strateacutegique les sept deacutefis agrave relever dici agrave 2016

Extrait drsquoun article du site wwwlentreprisecom -Sabine Blanc - Mis en ligne le 20032007

(httpwwwlentreprisecom325article11977html)

Une eacutetude anglaise publieacutee par lopeacuterateur Orange Grande-Bretagne deacutecrypte la mutation

des formes de travail et les enjeux majeurs pour les entreprises de demain afin decirctre au top

de la compeacutetitiviteacute Voici les challenges-cleacutes pour les managers qui veulent rester dans la

course hellip

1 - Future organisation du travail les quatre laquo mondes raquo possibles

La reacutealiteacute sera probablement un meacutelange de ces quatre sceacutenarios souligne lrsquoeacutetude

Les mondes mutuels Tout se passe dans le cadre des communauteacutes locales vie priveacutee

comme professionnelle Le modegravele coopeacuteratif preacutevaut au lieu du laquo big business raquo Oublieacutes

aussi dans ce systegraveme les trajets pour aller au bureau les gens preacutefegravereront travailler dans de

petites entreprises locales souvent connecteacutees au reacuteseau drsquoautres structures similaires

Les laquo reacutepondants raquo (en anglais laquo replicants raquo) La figure du consultant freelance deviendra

dominante tandis que celle du salarieacute deacuteclinera Il ne sera pas rare de travailler pour plusieurs

entreprises On perdra en seacutecuriteacute de lrsquoemploi en visibiliteacute et en routine ce que lrsquoon gagnera

en liberteacute La majeure partie des tacircches srsquoeffectuera chez soi avec la possibiliteacute de srsquoinstaller

temporairement dans les bureaux de son client du moment Dans un contexte dincertitude

sur lrsquoavenir les travailleurs alterneront peacuteriodes drsquoactiviteacute intense et repos Ce sera agrave eux

drsquoaller vers les entreprises et non lrsquoinverse mecircme si celles-ci devront veiller agrave rester attractives

Les cottages eacutelectroniques Comme ce nom le suggegravere le teacuteleacutetravail deviendrait la norme

univers priveacute et professionnel se confondant Plus besoin de subir une heure de transport les

salarieacutes se logueront de chez eux sur le reacuteseau de lrsquoentreprise Les reacuteunions se tiendront dans

de petits bureaux centraux situeacutes agrave courte distance La flexibiliteacute du temps de travail srsquoimpose

Les salarieacutes disposeront de plus de marge de liberteacute dans leur activiteacute

Les disciples de la nueacutee Cette appellation poeacutetique cache simplement une extension de

lrsquoorganisation actuelle des grandes entreprises avec des salarieacutes se rendant sur un lieu de

travail centraliseacute Le rocircle croissant des technologies de lrsquoinformation multipliera les faccedilons de

collaborer et accroicirctra lrsquoefficaciteacute Le controcircle du travail sera omnipreacutesent La frontiegravere entre

travail et vie priveacutee restera marqueacutee

2 - Sept deacutefis pour les entreprises et leur managers

Quoi qursquoil advienne les entreprises et leurs dirigeants devront concentrer leurs efforts sur sept

points-cleacutes pour srsquoadapter Voici quelques exemples de probleacutematiques souleveacutees par le

rapport et des pistes de solution

Le leadership Les managers devront entre autres savoir persuader et influencer des

travailleurs beaucoup plus indeacutependants Ils auront aussi agrave repenser les niveaux auxquels

prendre les deacutecisions strateacutegiques en haut ou au contraire agrave des degreacutes moins eacuteleveacutes de la

pyramide hieacuterarchique

gt Faire du management une force facilitant les activiteacutes transversales plutocirct que la reacuteduire agrave

la seule fonction de deacutecision

La culture drsquoentreprise Davantage de salarieacutes capables de reacutefleacutechir seront neacutecessaires

tandis que les tacircches qui peuvent ecirctre automatiseacutees ou scripteacutees diminueront Un des

enjeux creacuteer une culture agrave mecircme drsquoattirer et drsquoencourager les personnes preacutesentant ces

qualiteacutes de reacuteflexion requises dans un contexte de compeacutetition accrue et de plus grande

indeacutependance des travailleurs

Initiation au management copy CRCF ndash J Sornet Page 44 48

gt Passer si neacutecessaire drsquoune culture drsquoentreprise forte agrave un mode drsquoengagement plus

consensuel moins rebutant

La marque Conseacutequence du recours croissant agrave lrsquo laquo outsourcing raquo lrsquoimage drsquoune marque

deacutependra plus drsquoagents exteacuterieurs qui ne fonctionnent pas forceacutement selon le mecircme mode

drsquoorganisation Comment garder le controcircle dessus

gt Choisir le mode qui corresponde le plus agrave vos valeurs et preacutevoir un programme de risk

management qui mette en eacutevidence ougrave les conflits sont susceptibles de jaillir

Lrsquoinnovation Plus que jamais il faudra faire face agrave une acceacuteleacuteration du rythme de

lrsquoinnovation en proposant constamment des solutions adapteacutees

gt Tisser des partenariats strateacutegiques avec drsquoautres entreprises pour partager les coucircts et les

fruits de lrsquoinnovation

Le deacutefi opeacuterationnel et technologique De quelle faccedilon controcircler lrsquoinformation crsquoest-agrave-dire

faire en sorte que les bonnes personnes accegravedent facilement agrave une information toujours en

phase tout en maintenant la seacutecuriteacute

gt Recourir agrave des laquo feuilles de route des futurs raquo syntheacutetisant en une page les indicateurs

sociaux et de consommation ainsi que les eacutevolutions technologiques et leacutegislatives qui

influent sur les changements et indiquant comment ils modifient vos marcheacutes vos clients et

votre organisation

La qualiteacute Si de nouveaux proceacutedeacutes ont pu deacutegrader la qualiteacute comme le recours agrave des

centres drsquoappel externaliseacutes drsquoautres ideacutees se sont reacuteveacuteleacutees plus prometteuses comme en

teacutemoigne le succegraves de certaines compagnies aeacuteriennes low cost Elles ont su conjuguer prix

serreacutes et services eacuteleveacutes ce qui devra devenir la norme estime lrsquoeacutetude

gt Continuer de rechercher la qualiteacute Elaborez aussi une bonne prestation service qui inclut

une livraison de qualiteacute voire creacuteez-la en partenariat avec les consommateurs

La leacutegislation La question de la proprieacuteteacute intellectuelle pourrait ecirctre probleacutematique Elle est

deacutejagrave source de conflits comme en teacutemoigne le procegraves pour violation de brevet intenteacute agrave RIM

le fabricant canadien du Blackberry par NTP Que pourra-t-on et que faudra-t-il proteacuteger par

un brevet Il sera eacutegalement neacutecessaire drsquoadapter la leacutegislation aux nouveaux modes

drsquoorganisation

gt Collaborer avec les acteurs du mecircme secteur et les leacutegislateurs pour deacutevelopper les

modegraveles des lieux de travail du futur et bacirctir le droit le plus adeacutequat

Orange a-t-il vu juste dans ses preacutevisions Rendez-vous dans neuf ans pour la reacuteponsehellip

Initiation au management copy CRCF ndash J Sornet Page 45 48

Fiche DT3 ndash Le management par la qualiteacute totale

Extrait drsquoune lettre drsquoinformation du cabinet Baud Accordance Consulting AD2 consultants ndash

2002

1 - Le TQM (Total Quality Management) offre pour lentreprise une vision de la qualiteacute plus

large et transversale

Son principe est simple La finaliteacute de lEntreprise est de deacutevelopper la satisfaction de ses

clients tout en eacutetant beacuteneacuteficiaire cest agrave dire pas agrave nimporte quel prix Elle doit ameacuteliorer sa

rentabiliteacute au travers de la deacutemarche qualiteacute La Qualiteacute Totale vise agrave fournir aux clients

externes et internes une reacuteponse adeacutequate agrave leurs attentes dans le meilleur rapport qualiteacute

prix la meilleure efficience

Elle considegravere pour cela lensemble des processus de lentreprise ayant une incidence sur la

qualiteacute et la satisfaction des clients

Le TQM fait ainsi une large place agrave

la deacutefinition et la planification de la strateacutegie geacuteneacuterale

la coheacuterence de la politique qualiteacute avec la strateacutegie

la deacutemultiplication de la politique qualiteacute dans toutes les directions de lentreprise

la relation client fournisseur interne

la prise en compte de lenvironnement concurrentiel

la consideacuteration de lensemble des risques potentiels financiers sociaux concurrentielshellip

limplication et la motivation du personnel

lanalyse des besoins des clients et le positionnement marketing

la maicirctrise des processus transverses internes

les reacutesultats sous tous ses aspects y compris financiers commerciaux image

De nombreux reacutefeacuterentiels sont relatifs agrave la Qualiteacute Totale hellip Tous ces reacutefeacuterentiels imposent un

questionnement plus profond et indiscret sur le mode de fonctionnement de lentreprise et

son management

helliphellip

2 - LISO 9001 2000 au travers du deacuteploiement des processus (management supports

reacutealisation et ameacutelioration continue) reacutepond quelque peu agrave la mecircme logique

LISO est une ouverture indeacuteniable vers la logique du TQM mais ne se reacutefegravere pas agrave la notion

defficience

Les dirigeants sont cependant sensibles agrave la neacutecessaire reacuteduction des coucircts de non-qualiteacute

et dobtention de la qualiteacute agrave la rentabiliteacute du systegraveme de management de la qualiteacute

mais ne perccediloivent pas toujours la qualiteacute comme une deacutemarche globale

Les deacutemarches qualiteacute commencent bien souvent par la remise en cause de lorganisation

leacutevaluation critique de son efficaciteacute lexamen des processus et la mise en eacutevidence des

lourdeurs administratives

La qualiteacute devient laffaire de tous hellip

Initiation au management copy CRCF ndash J Sornet Page 46 48

Fiche DT4 ndash Le deacuteveloppement durable et la RSE

Extrait du site wwwvigeocom

(httpwwwvigeocomcsr-rating-agencyfrmethodologiecriteres-de-recherche37-

criteres-d-analysehtml)

Deacuteveloppement durable laquo un deacuteveloppement qui reacutepond aux besoins du preacutesent sans compromettre

la capaciteacute des geacuteneacuterations futures de reacutepondre aux leurs raquo (Commission mondiale sur lrsquoenvironnement

et le deacuteveloppement ndash 1987)

Reacutefeacuterentiel drsquoeacutevaluation des entreprises par le groupe Vigeacuteo (le groupe mesure les performances et le

niveau de maicirctrise des risques de responsabiliteacute sociale des entreprises et des organisations - site

wwwvigeocom)

1 Ressources Humaines Ameacutelioration continue des relations professionnelles des relations drsquoemploi et des conditions de travail 2 Droits humains sur les lieux de travail Respect de la liberteacute syndicale et promotion de la neacutegociation collective non discrimination et promotion de lrsquoeacutegaliteacute eacutelimination des formes de travail proscrites (enfants travail forceacute) preacutevention des traitements inhumains ou deacutegradants de type harcegravelements sexuels protection de la vie priveacutee et des donneacutees personnelles 3 Environnement Protection sauvegarde preacutevention des atteintes agrave lenvironnement mise en place drsquoune strateacutegie manageacuteriale approprieacutee eacuteco conception protection de la biodiversiteacute et maicirctrise rationnelle des impacts environnementaux sur lrsquoensemble du cycle de vie des produits ou services

4 Comportements sur les marcheacutes Prise en compte des droits et inteacuterecircts des clients inteacutegration de standards sociaux et environnementaux dans la seacutelection des fournisseurs et sur lrsquoensemble de la chaicircne drsquoapprovisionnement preacutevention effective de la corruption respect des regravegles concurrentielles 5 Gouvernement drsquoentreprise Efficience et probiteacute assurance de lrsquoindeacutependance et de lrsquoefficaciteacute du Conseil drsquoadministration effectiviteacute et efficience des meacutecanismes drsquoaudit et de controcircle et notamment inclusion des risques de responsabiliteacute sociale respect des droits des actionnaires et notamment des minoritaires transparence et rationaliteacute de la reacutemuneacuteration des dirigeants 6 Engagement socieacutetal Effectiviteacute inteacutegration manageacuteriale de lrsquoengagement contribution au deacuteveloppement eacuteconomique et social des territoires drsquoimplantation et de leurs communauteacutes humaines engagements concrets en faveur de la maicirctrise des impacts socieacutetaux des produits et des services contribution transparente et participative agrave des causes drsquointeacuterecirct geacuteneacuteral

Initiation au management copy CRCF ndash J Sornet Page 47 48

ELEMENTS DE CORRIGE DT DT1 Deacutefinir expliquer

Deacutereacuteglementation = suppression des contraintes eacuteconomiques (libre eacutechange des biens et

capitaux)

Socieacutetal = qui se rapporte agrave la structure agrave lrsquoorganisation ou au fonctionnement de la socieacuteteacute

Economies drsquoeacutechelle = reacuteduction des coucircts lieacutee au niveau drsquoactiviteacute (amortissement des

charges fixes)

Coaching = accompagnement de personnes ou deacutequipes pour le deacuteveloppement de leurs

potentiels

EDI = eacutechange de donneacutees informatiseacutees ET standardiseacutees (ex SWIFT bancaire edifact

documents deacuteclaratifs)

Gouvernance = exercice du pouvoir la bonne gouvernance est participative et eacutequitable

conforme agrave lrsquointeacuterecirct commun

DT2 Deacuteterminer en quoi la deacutemarche TQM srsquoinscrit dans les deacutefis actuels du management

Voir notamment fiche 43

Maicirctrise des processus reacuteduction des coucircts reacuteactiviteacute et satisfaction de la clientegravele = faire

face agrave la concurrence

Ameacutelioration de lrsquoimage motivation du personnel

DT3 Apregraves avoir consulteacute les documents ci-dessous extraits du site drsquoAir France

(httpdeveloppement-

durableairfrancecomFRfrlocaldemarcheN4_positionnement_pphtm)

exposer les enjeux et les limites de la RSE et de la gestion des parties prenantes

Trame geacuteneacuterale possible

Introduction

Les deacutefis contemporains (accroissement de la concurrence devenue mondiale recherche

de nouveaux avantages concurrentiels pression de la socieacuteteacute besoin drsquoimage et de projet

lisible pour mener lrsquoentreprise crise et scandales du libeacuteralisme hellip) RSE et PP

Deacuteveloppement (voir cours)

1 ndash Parties prenantes et management par la valeur

PP deacutefinir citer reacutesumer lrsquoavantage rechercheacute (fideacuteliser motiver recherche drsquoalliances

implicites)

PP moyens (dont exemples AF) et meacutethode de management par la valeur (reacutepartie)

2 ndash La responsabiliteacute socieacutetale de lrsquoentreprise

RSE 3 axes

- eacuteconomique (favoriser le deacuteveloppement les eacutechanges internationaux)

- social (accegraves aux soins eacuteducation conditions de travail hellip)

- environnemental (pollution preacuteservation des ressources hellip)

RSE gouvernance drsquoentreprise facteur drsquoimage inteacutegrable dans la deacutemarche PP

Article 116 de la loi Le rapport viseacute agrave larticle L 225-102 rend compte hellip laquo Il comprend

eacutegalement des informations dont la liste est fixeacutee par deacutecret en Conseil dEtat sur la maniegravere

dont la socieacuteteacute prend en compte les conseacutequences sociales et environnementales de son

activiteacute Le preacutesent alineacutea ne sapplique pas aux socieacuteteacutes dont les titres ne sont pas admis aux

neacutegociations sur un marcheacute reacuteglementeacute raquo

Initiation au management copy CRCF ndash J Sornet Page 48 48

RSE exemple AF (ONG fournisseurs)

3 ndash Liens entre PP et RSE

- la RSE introduit de nouvelles PP

- la RSE suppose le respect des PP usuelles (employeacutes clients notamment)

4 - Probleacutematique

- deacutefinir la valeur reacuteellement apporteacutee par une gestion des PP (confusion salaire ndash valeur

idem impocircts hellip ex laquo valeur ajouteacutee raquo)

- communication (neacutecessaire mais aller au-delagrave)

- marginaliteacute des deacutepenses RSE (efficaciteacute sinceacuteriteacute de lrsquoengagement marge de manœuvre)

- charge RSE reporteacutee sur des tiers (ex fournisseurs AF)

- inteacutegration de facteurs non visibles en comptabiliteacute (pertes drsquoemploi nuisances hellip)

Conclusion

Voies incontournables mais pouvant nrsquoavoir qursquoun effet superficiel et temporaire Voir utiliteacute

drsquoaccompagnement leacutegislatif de regravegles de gouvernance

Initiation au management copy CRCF ndash J Sornet Page 7 48

- des modes influencent le management Elles peuvent indiquer une veacuteritable eacutevolution

eacuteconomique mais aussi ecirctre sans lendemain voire introduire un danger ou un coucirct inutile

(se meacutefier des speacutecialistes dont le fond de commerce est la vente de nouvelles techniques

de management et des seacuteminaires associeacutes)

- seule la creacuteation drsquoune combinaison originale (dans le respect des regravegles) peut apporter un

avantage agrave lrsquoentreprise et non la reproduction de choix de management connus de tous

Exemple 1 la matrice de portefeuille drsquoactiviteacutes permet de classer les activiteacutes

strateacutegiques drsquoune entreprise en fonction de leur taux de croissance et de la part de

marcheacute deacutetenue (matrice laquo BCG raquo - Boston consulting group ndash 1975) Cet outil de

management neacutecessite de disposer drsquoinformations fiables concernant le marcheacute Il

conduit souvent agrave simplifier les conditions de concurrence et ignore la

compleacutementariteacute pouvant exister entre activiteacutes (synergie partage de techniciteacute

amortissement de charges fixes hellip) La matrice est un moyen de prendre conscience

du portefeuille et de ses eacuteventuels deacutefauts (portefeuille deacuteseacutequilibreacute avec

preacutedominance anormale drsquoune zone) mais il ne permet pas seul de deacutecider de lrsquoavenir

des activiteacutes

Exemple 2 le lancement drsquoun investissement lourd (lrsquoimplantation de nouvelles usines

le lancement drsquoune nouvelle activiteacute agrave fort taux de recherche ndash deacuteveloppement hellip) ne

peut ecirctre deacutecideacute que par le recoupement de diffeacuterentes approches (financiegravere

commerciale strateacutegique) Aucune theacuteorie du management appliqueacutee isoleacutement et

sans preacutecaution ne peut justifier un tel investissement

Exemple 3 une entreprise produisant des eacutequipements meacutedicaux deacutecide de renforcer

son offre commerciale par un service de conseil et drsquoassistance Cette deacutecision est

inspireacutee par le concept de creacuteation de valeur par une volonteacute de se diffeacuterencier de la

concurrence et par des pratiques existant depuis longtemps dans le domaine

informatique

Remarque parmi les facteurs influenccedilant les principes de management il est utile de

savoir deacutetecter pour srsquoen preacutemunir les ideacuteologies preacutejugeacutes et autres laquo valeurs raquo sans

rapport certain avec lrsquoefficaciteacute eacuteconomique

32 ndash Theacuteorie et pratique du management

La litteacuterature (ouvrages revues) aborde le management sous des angles diffeacuterents qui

peuvent suggeacuterer une concurrence entre des approches theacuteorique et pratique ou

psychologique et eacuteconomique de cette discipline

Taux de croissance

du domaine (cf

cycle de vie du

produit)

Part de marcheacute

(compareacutee au

principal

concurrent)

fort

faible

forte faible

Dilemmes (activiteacutes

en phase de

lancement

potentiel + coucircts+

risque+)

Vedettes

(activiteacutes en forte

croissance

autofinancement+)

Vaches agrave lait

(activiteacutes

stabiliseacutees et

concurrence faible

ou stable

rentabiliteacute+)

Poids morts

(activiteacutes en

deacuteclin)

Initiation au management copy CRCF ndash J Sornet Page 8 48

Le management a cependant une viseacutee unique et concregravete la bonne marche des

organisations et les travaux des universitaires rejoignent lrsquoaction des praticiens (les

universitaires reacutealisent drsquoailleurs bien souvent leurs recherches au sein des organisations)

Au plan peacutedagogique les deux visions du management se complegravetent

- la vision acadeacutemique met en perspective des concepts et des theacuteories Elle permet de

srsquoapproprier des raisonnements essentiels et stables notamment concernant la strateacutegie et

les reacuteactions humaines qui sont le moteur des organisations

- la vision pratique relate le veacutecu des praticiens preacutesente des techniques concregravetes et tente

parfois une vulgarisation favorisant la diffusion des concepts Elle integravegre de faccedilon

pragmatique les objectifs eacuteconomiques et les contraintes de fonctionnement des

organisations en se placcedilant parfois dans une vision agrave court terme

Sur le terrain le management reacutealise une synthegravese entre des techniques de psychologie

sociale et des techniques de gestion de diverses origines et de nouvelles meacutethodes

apparaissent aussi sous le label unique laquo management raquo

Le terme manageacuterial (approche manageacuteriale theacuteorie manageacuteriale pratique manageacuteriale

hellip) fait reacutefeacuterence agrave la vision aux preacuteoccupations et aux actions des managers qui doivent

emmener leur organisation vers la reacutealisation de ses objectifs

Exemples

- la deacutetermination des coucircts et des marges reacutesulte de techniques de gestion

indispensable aux deacutecisions des managers

- le laquo CRM raquo (customer relationship management en franccedilais GRC ndash gestion de la

relation client) est un concept reacutecent qui srsquoaccompagne de techniques lieacutee aux plus

reacutecents deacuteveloppements du management et des systegravemes drsquoinformation

- le laquo coaching raquo est une technique de management permettant une eacutevolution

personnelle dans le sens des objectifs de lrsquoentreprise

- le recentrage sur le laquo meacutetier raquo repose sur des techniques de management

Les techniques sont rassurantes mais pas suffisantes et si les theacuteories ne sont pas

indispensables au praticien elles aident agrave comprendre agrave anticiper et agrave bien utiliser les

techniques

La qualiteacute du management reacuteside beaucoup dans la capaciteacute agrave appliquer concepts et

techniques de faccedilon pertinente et agrave innover Cette capaciteacute relegraveve en partie drsquoun laquo art du

management raquo qui srsquoacquiert en grande partie par la pratique

4 ndash Management et expertise comptable

Lrsquoexpert comptable doit manager ses propres eacutequipes Il est par ailleurs supposeacute laquo hellip

conseiller et accompagner le chef drsquoentreprise dans toutes ses deacutecisions hellip raquo (selon lrsquoOEC)

Ce rocircle est particuliegraverement important dans ses relations avec les petites entreprises Il doit

donc avoir une capaciteacute au management

Organisation

Vision acadeacutemique Vision pratique

Initiation au management copy CRCF ndash J Sornet Page 9 48

Lrsquointervention de lrsquoexpert dans le management drsquoune entreprise peut toutefois poser

quelques problegravemes

- Il peut y avoir conflits drsquointeacuterecirct entre activiteacutes de certification des comptes et de conseil en

management (la tendance est agrave la seacuteparation des activiteacutes dans les plus grands cabinets)

- lrsquoactiviteacute de conseil neacutecessite des compeacutetences parfois tregraves speacutecifiques (conseil fiscal

conseil en RH conseil en systegravemes drsquoinformation hellip)

- le conseil est une activiteacute diffeacuterente par sa forme de lrsquoexpertise comptable (interventions

longues peu reacutepeacutetitives peu codifieacutees mises en concurrence) qui neacutecessite une

organisation particuliegravere du cabinet lorsqursquoelle deacutepasse lrsquointervention occasionnelle

APPLICATIONS IM

IM1 Analyser la profession de laquo manager raquo selon Henry Mintzberg (texte extrait de lrsquoouvrage

laquo Le management raquo Eyrolles - Editions drsquoOrganisation) et les principes du management de la

norme ISO

Faire ressortir les eacuteleacutements speacutecifiques agrave chacune de ces approches et mettre en eacutevidence

leurs points communs

Initiation au management copy CRCF ndash J Sornet Page 10 48

Principes du management drsquoapregraves la norme ISO 9001 (2000)

- Orientation vers le client (satisfaire ses attentes)

- Leadership (les dirigeants eacutetablissent les orientations de lrsquoorganisme Ils doivent creacuteer

un environnement interne ougrave les personnes peuvent clairement srsquoimpliquer dans la

reacutealisation des objectifs de lrsquoorganisme)

- Implication du personnel (les personnes sont agrave tout niveau lrsquoessence de lrsquoorganisme et

leur implication permet drsquoutiliser leurs aptitudes au profit de lrsquoorganisme)

- Approche laquo processus raquo (un reacutesultat est mieux atteint quand les ressources et les

activiteacutes neacutecessaires sont geacutereacutees comme un processus)

- Approche systegraveme (assimiler les processus correacuteleacutes agrave un systegraveme contribue agrave

lrsquoefficaciteacute et agrave lrsquoefficience de lrsquoorganisme vis-agrave-vis de ses objectifs)

- Ameacutelioration continue (objectif permanent de lrsquoorganisme)

- Prise de deacutecision efficace (par lrsquoanalyse de donneacutees et drsquoinformations)

- Relations mutuellement beacuteneacutefiques avec les fournisseurs (pour augmenter la capaciteacute

des deux organismes agrave creacuteer de la valeur)

IM2 Distinguer leader et manager

IM3 Compleacuteter le tableau ci-dessous en analysant chaque action preacutesenteacutee Faire ensuite

ressortir les domaines niveaux ou techniques de management pouvant ecirctre mobiliseacutes pour

chaque situation

Initiation au management copy CRCF ndash J Sornet Page 11 48

Caracteacuteristiques

de lrsquoaction

- reacutepeacutetition

- risque

- normes

- ampleur

Prise de

deacutecision

- opeacuterationnelle

strateacutegique

- deacutelai

Informations

neacutecessaires

- nature

- origine

- deacutelai obtention

Cleacutes pour la

reacuteussite

Intervention

exteacuterieure

possible

Assurer la

restauration du

soir

(restaurant

familial)

Construire un

viaduc

(autoroute)

Certifier les

comptes

annuels drsquoun

groupe

national

(cabinet

drsquoaudit)

Lancer une

ligne drsquoavions

(constructeur

aeacuteronautique)

Reacuteduire la

capaciteacute de

production

(groupe

industriel)

Acqueacuterir une

entreprise

concurrente

(teacuteleacutephonie

mobile)

Initiation au management copy CRCF ndash J Sornet Page 12 48

Fiche IM1 - Deacutefinitions du management

Dictionnaire anglais - franccedilais direction administration gestion intrigue manegravege

Wikipeacutedia Le management est lensemble des techniques dorganisation qui sont mises en

oeuvre pour ladministration dune entiteacute

Au point de vue eacutetymologique le verbe manage vient de litalien maneggiare (controcircler)

influenceacute par le mot franccedilais manegravege (faire tourner un cheval dans un manegravege) A cette

notion il faut aussi ajouter la notion de meacutenage (geacuterer les affaires du meacutenage) qui consiste agrave

geacuterer des ressources humaines et des moyens financiers

helliphellip

Fiche IM2 - Etudier le management

Concreacutetiser

Manager neacutecessite de syntheacutetiser des informations parfois complexes incomplegravetes et de

domaines tregraves divers pour en deacuteduire des actions Une approche trop parcellaire peut

conduire agrave lrsquoeacutechec et le savoir-faire est neacutecessaire pour agir vite avec un minimum de risque

Lrsquoeacutetudiant doit se preacuteparer simultaneacutement aux examens et agrave la pratique Il nrsquoa souvent connu

lrsquoentreprise que durant quelques semaines de stage et le manque de laquo recul raquo ne lui permet

pas toujours de concreacutetiser les theacuteories Il doit compenser par la lecture (ouvrages revues

journaux eacuteconomiques et boursiers) et en eacutetant attentif aux informations ambiantes (tout en

relativisant le style journalistique) en mettant en relation le cours les concepts les modegraveles

lrsquoactualiteacute les stages

Savoir traiter un exercice

Pour reacuteussir un examen ou traiter une application peacutedagogique (la conception les points 1 agrave

6 peut repreacutesenter le tiers du temps de travail)

1 ndash Identifier le type de sujet (faut-il trouver une solution pratique ou communiquer une

reacuteflexion geacuteneacuterale )

2 ndash Lire le sujet et relever les mots cleacutes

3 ndash Deacutefinir les mots cleacutes

4 ndash Reacutesumer la probleacutematique du sujet (en quelques lignes)

5 ndash Lister les connaissances reacutefeacuterences et raisonnements reacutepondant au problegraveme (par

recherche spontaneacutee ou raisonneacutee qui quoi ougrave quand comment combien hellip

listage des diffeacuterents points de vue) trouver des exemples (notamment dans les

documents fournis)

6 ndash Organiser la reacuteponse (deacutefinir le plan du deacuteveloppement ougrave des paragraphes bien

identifieacutes sont geacuteneacuteralement neacutecessaires en y liant les parties qui doivent ecirctre en nombre

limiteacute ndash de deux agrave quatre) Preacutevoir drsquoy inteacutegrer la deacutefinition des principales notions

induites par le sujet

7 ndash Reacutediger sous la forme adapteacutee (note technique ou recommandation solution

pratique exposeacute structureacute dissertation)

Introduction et conclusion sont indispensables agrave la dissertation ou agrave lrsquoexposeacute

- lrsquointroduction preacutesente le sujet traiteacute (phrase drsquoaccroche initiale) amorce la

probleacutematique (quelques sous - questions) et annonce le plan

- la conclusion syntheacutetise le deacuteveloppement (arguments) eacutelargit le sujet (prise de recul)

et apporte le point final (une phrase)

Une limite agrave la communication

Il est difficile de faire passer plus de 4 ou 5 ideacutees fortes dans un exposeacute unique

Initiation au management copy CRCF ndash J Sornet Page 13 48

Fiche IM3 - Bref historique

Antiquiteacute

3000 AJC

Peacuteriode greacuteco-

romaine

Transition

feacuteodale

12egraveme siegravecle

europe

15egraveme ndash 17egraveme

siegravecles

19egraveme siegravecle

20egraveme siegravecle

agriculture preacutedominante industrie limiteacutee aux besoins drsquoun individu ou drsquoun clan

pour la confection des outils des vecirctements et de la poterie Force motrice animale

ou humaine pour lrsquoessentiel

Grands travaux drsquoeacutetat en Egypte premiegravere laquo planification ndash organisation ndash controcircle raquo

Deacuteveloppement des communications essor industriel limiteacute peu de progregraves

technique (lrsquoesclavage supplante les innovations)

Deacuteveloppement progressif des eacutechanges commerciaux

La consommation indirecte atteint un bon niveau (surplus agricoles et

deacuteveloppement des villes) Apparition de nouveaux commerccedilants

Etat fort Evolutions technologiques (imprimerie bateaux performants instruments de

navigation) Extension geacuteographique de lrsquoeacuteconomie Apparition des corporations

drsquoartisans

Machine agrave vapeur chemin de fer passage de lrsquoartisanat au capitalisme

entrepreneurial producteur organisation des entreprises

Ecole classique (Taylor Fayol Weber) approche meacutecaniste bureaucratie

hieacuterarchie commandement fonctions et speacutecialisation laquo OS T raquo (organisation

scientifique du travail) organisation source de pouvoir rationaliteacute des individus bases

du management

Deacuteveloppement du capitalisme manageacuterial Electriciteacute peacutetrole puis communications

et information Consommation de masse mondialisation preacuteoccupations

eacutenergeacutetiques et environnementales 3 peacuteriodes

- standardisation grandes entreprises industrielles

- industries de consommation 30 glorieuses marketing multinationales protection

sociale

- deacutereacuteglementation monteacutee des services pays eacutemergents mondialisation et nouvelle

eacuteconomie (internet)

Ecole des relations humaines prise en compte de lrsquoindividu des motivations styles

de direction

Ecole neacuteo-classique et post-classique deacutecentralisation coordonneacutee DPO

management participatif zeacutero deacutefaut flux tendus

Approche systeacutemique partition de lrsquoentreprise eacutetude des interactions feacutedeacuteration

vers lrsquoobjectif controcircle et ajustement

Theacuteories de la deacutecision rationaliteacute limiteacutee contribution reacutetribution coalitions

Ecole socio-technique recherche de compromis technologie organisation

enrichissement des tacircches autonomie des groupes

Approche sociologique effets sociaux du travail jeux de pouvoir dans lrsquoentreprise

reacutegulation sociale

Theacuteories de la contingence facteurs contingents adaptation agrave lrsquoenvironnement

configurations organisationnelles

Theacuteories de la firme controcircle manageacuterial droits de proprieacuteteacute relation drsquoagence

Theacuteories contractualistes firme nœud de contrats coucircts de transaction

opportunisme externalisation internalisation

Approche eacutevolutioniste eacutecologie des organisations modegravele eacutevolutioniste

contraintes de sentier

Approche par les ressources valorisation des ressources compeacutetences cleacutes

apprentissage organisationnel

(Classement simplifieacute)

Initiation au management copy CRCF ndash J Sornet Page 14 48

ELEMENTS DE CORRIGE IM

IM1 Commenter la deacutefinition du management par la norme ISO et le manager de Mintzberg

Efficient = optimum avec les moyens disponibles

ISO (management objectifs) (manager moyens) HM

IM2 Le leader entraicircne naturellement derriegravere lui Le manager nrsquoest pas toujours leader

(mecircme si crsquoest souhaitable) Le leader nrsquoest pas toujours manager (plutocirct notion individuelle)

Leadership = faculteacute de diriger conjugaison drsquoune autoriteacute naturelle ou drsquoun savoir-faire

acquis drsquoune capaciteacute agrave entraicircner des personnes ou des groupes et drsquoune leacutegitimiteacute

statutaire (de position)

IM3 Compleacuteter le tableau ci-dessous en analysant chaque action preacutesenteacutee Faire ensuite

ressortir les domaines niveaux ou techniques de management pouvant ecirctre mobiliseacutes pour

chaque situation

Satisfaction client

Implication du personnel

Processus systegraveme

Ameacutelioration continue

Deacutecision efficace

Recherche de valeur

Image entreprise

Liaisons

Information

Reacutepartition ressources

Reacutegulation

Neacutegociation

Leadership

Initiation au management copy CRCF ndash J Sornet Page 15 48

Caracteacuteristiques

de lrsquoaction

- reacutepeacutetition

- risque

- normes

- ampleur

Prise de

deacutecision

- opeacuteration

- direction

- deacutelai

Informations

neacutecessaires

- nature

- origine

- deacutelai

obtention

Cleacute pour la

reacuteussite

Intervention

exteacuterieure

possible

Assurer la

restauration du

soir

(Restaurant

familial)

Technique

(fabrication)

Vente (terrain)

Appros

Reacutepeacutetitive

(quot)

Risque faible

Normes

drsquohygiegravene

Faible

Opeacuterationnelle

Geacuterant

responsable

Rapide (qq

jours menu et

appros)

Nombre de

couverts

Tarifs usuels

Calendrier

(fecirctes)

Clients docs

divers

expeacuterience

Qq jours

Varieacuteteacute menu

Plats phares

Accueil

Appros

Tarification

Vins

Gestion

congeacutelation

Qualiteacute cuisine

Fournisseurs

Extra

Publiciteacute

Construire un

viaduc

(autoroute)

Technique

Organisation

Appros

Uniteacute (ou peu)

Eleveacute (financier

technique)

Architecture

Eleveacutee

Direction

(aleacuteas)

Opeacuterationnelle

(conduite

chantier)

Immeacutediat agrave qq

semaines

Plans

plannings

Qualifications

Meacuteteacuteo

Disponibiliteacutes

Bureau eacutetudes

Qq sem agrave 24h

Techniciteacute

Appros

Qualifications

Preacutevision

GRH

Contrat juste

SS traitants

Organismes

certificateurs

Controcircle

client

Certifier les

comptes

annuels drsquoun

groupe national

(cabinet

drsquoaudit)

Technique

Relation client

Gestion des

connaissances

Annuelle

Moyen

Regravegles

comptables

fiscales

Moyenne (selon

importance du

cabinet)

Opeacuterationnelle

Qq jours agrave

semaines

Comptable

Juridique

Client

Etat

Qq jours agrave

semaines

Techniciteacute

Expeacuterience

Relation client

Systegraveme info client

Siegravege

Autre cabinet

Lancer une

ligne drsquoavions

(constructeur

aeacuteronautique)

Strateacutegique

RD

Etudes

Uniteacute

Tregraves eacuteleveacute

Aeacuteronautique

Tregraves eacuteleveacutee

Direction

Qq mois agrave

anneacutees

Marcheacute

Etudes

Compagnies

Qq mois agrave

anneacutees

Concept

Outil industriel

Coucirct exploitation

Tarif

Fiabiliteacute

Deacutelaisconcurrence

SI simulation

SS traitants

Bureaux

drsquoeacutetudes

speacutecialiseacutes

Compagnies

Conseils

Reacuteduire la

capaciteacute de

production

(groupe

industriel)

Strateacutegique

RH

Communication

Production

Uniteacute

Moyen

Leacutegislation

(dont RH)

Eleveacutee

Direction

Qq mois agrave

anneacutees

Financiegravere

Industrielle

Marcheacute

Organisation

Organismes

speacutecialiseacutes

DRH

Qq mois

Communication

Connaissance des

compeacutetences

Connaissance outil

industriel

Concurrence

Portefeuille

drsquoactiviteacutes

Cabinet

drsquoorganisation

Conseils

speacutecifiques

Acqueacuterir une

entreprise

concurrente

(teacuteleacutephonie

mobile)

Strateacutegique

Marketing

Production

(reacuteseau)

Financier

Communication

Uniteacute

Tregraves eacuteleveacute

Leacutegislation

telecom

Tregraves eacuteleveacutee

Direction

Qq mois

Financiegravere

Marcheacute

Reacuteseaux

(ampleur

recouvrement

hellip)

Organisations

Interne

Racheteacutee

Sources

speacutecialiseacutees

Qq mois

Communication

Marcheacute

Cours boursiers

Cabinet

drsquoorganisation

Conseils

speacutecifiques

Initiation au management copy CRCF ndash J Sornet Page 16 48

LE MANAGEMENT EN PRATIQUE

Pour assumer sa fonction le management doit couvrir sans discontinuiteacute lrsquoensemble de

lrsquoorganisation et inteacutegrer de nombreux facteurs dont nous allons reacutesumer lrsquoessentiel

1 ndash Les fonctions et activiteacutes du management

Pour Henri Fayol la fonction drsquoadministration de lrsquoentreprise (son management) reposait sur

cinq actions preacutevoir organiser commander coordonner et controcircler (laquo PO3C raquo)

Nous distinguerons cinq activiteacutes de management

- la conception (au plus haut niveau finaliteacute but ou vocation de lrsquoorganisation

meacutetiers dimension politique de croissance hellip)

- la planification (deacutefinition des objectifs eacutecheacuteances)

- lrsquoorganisation (reacutepartition du travail choix des modes de coordination)

- le pilotage de lrsquoaction opeacuterationnelle (motivation animation encadrement

assistance)

- lrsquoeacutevaluation (controcircle des reacutesultats obtenus ajustements)

Dans chacune de ces activiteacutes des deacutecisions et des arbitrages sont neacutecessaires avec des

enjeux plus ou moins importants

Remarques

- Les cinq activiteacutes du management peuvent se retrouver agrave tout niveau de

management si lrsquoentreprise laisse une certaine autonomie de deacutecision agrave ses diffeacuterentes

uniteacutes La conception est naturellement du ressort de la direction geacuteneacuterale et des

conseils drsquoadministration mais elle peut ecirctre preacutesente pregraves du terrain (latitude laisseacutee agrave

une filiale ou agrave un magasin par exemple) De mecircme lrsquoorganisation du travail concerne

un atelier mais aussi la direction qui structure lrsquoentreprise pour assurer ses activiteacutes sa

production

- La planification deacutefinit des objectifs ou des axes strateacutegiques (choix de produits

modaliteacutes de deacuteveloppement des ventes implantations alliances hellip) et les traduit en

donneacutees de gestion preacutevisionnelles syntheacutetiques et eacutechelonneacutees dans le temps afin de

valider les objectifs et de fixer des repegraveres

- Un laquo business plan raquo (plan drsquoaffaires)est notamment lrsquoeacutequivalent de la planification

dans le cas de creacuteation drsquoentreprise ou pour la preacutesentation de tout projet drsquoactiviteacute

Les activiteacutes du management srsquoinscrivent dans des cycles qui peuvent ecirctre scheacutematiseacute

comme suit (lrsquoeacutevaluation peut entraicircner une reacutevision du pilotage de lrsquoorganisation ou des

objectifs sans que lrsquoentreprise ne soit fondamentalement remise en cause)

conception

planification

organisation

pilotage

eacutevaluation

Initiation au management copy CRCF ndash J Sornet Page 17 48

2 ndash Les contextes de management

Le management est influenceacute par son contexte qui justifie des objectifs une organisation

des meacutethodes

Par exemple lrsquoentreprise admet de nombreuses variantes selon sa taille sa forme juridique

son controcircle par lrsquoeacutetat (entreprises publiques) ou par des inteacuterecircts priveacutes Il en va de mecircme des

organismes administratifs qui peuvent deacutependre de directives nationales ou reacutegionales des

associations qui ont des activiteacutes drsquoampleur tregraves variable

21 ndash La dimension de lrsquoentreprise

La dimension drsquoune entreprise se mesure principalement en fonction de son effectif ou de

son chiffre drsquoaffaires Des seuils sont deacutefinis par divers organismes et exploiteacutes agrave des fins

statistiques ou pour la deacutetermination de certaines obligations sociales ou fiscales

(repreacutesentation du personnel cotisations hellip) Il nrsquoy a bien entendu pas de laquo barriegravere de

tailleraquo absolue conditionnant le management drsquoune entreprise

LrsquoUE preacuteconise de distinguer les micro ndash entreprises (jusqursquoagrave 9 salarieacutes) les TPE ndash tregraves petites

entreprises (moins de 20 salarieacutes) les petites entreprises (moins de 50) et les moyennes

entreprises (de 50 agrave 250) Cependant les PME sont parfois situeacutees entre 10 et 500 salarieacutes

Remarques

- en France environ 40 des entreprises emploient de 1 agrave 50 salarieacutes (ce qui repreacutesente

plus de 50 des emplois) et 59 nrsquoen ont aucun

le pays compte environ 2 600 000 entreprises dont moins de 1 ont 250 employeacutes et

plus

- ancienneteacute et taille de lrsquoentreprise sont lieacutees si lrsquoon eacutecarte les restructurations et autres

eacutevolutions drsquoentreprises existantes

La dimension de lrsquoentreprise a une influence sur lrsquoorganisation et le laquo style raquo de son

management

- les PME sont souvent entrepreneuriales (les dirigeants eacutegalement apporteurs de capitaux

sont totalement engageacutes dans la marche de lrsquoentreprise) Elles ont une gestion flexible peu

formaliseacutee plus qualitative que quantitative Les PME sont freacutequemment focaliseacutees sur un seul

type drsquoactiviteacute Pour ne pas alourdir leur structure elles ont tendance agrave sous-traiter les

activiteacutes speacutecialiseacutees ne correspondant pas agrave leur meacutetier de base

- les grandes entreprises sont manageacuteriales (les dirigeants sont nommeacutes par les actionnaires

en raison de leurs compeacutetences) et moins reacuteactives

22 ndash Le type de production

On distingue industrie (production de biens mateacuteriels ou pour le moins de produits visibles ndash

comme un seacutejour touristique ou un film) et services (fourniture drsquoune prestation immateacuterielle)

Le type de production influence en principe le management de lrsquoentreprise

- lrsquoindustrie neacutecessite (si lrsquoon excepte lrsquoartisanat) un investissement relativement important

une organisation productive stable capable de reacutealiser plusieurs fois des produits identiques

(exemple un modegravele de reacutefrigeacuterateur) ou du moins similaires (exemple un bacirctiment) Le

produit de lrsquoindustrie consomme des matiegraveres et il doit geacuteneacuteralement ecirctre distribueacute jusqursquoau

client

- la production de services peut se satisfaire drsquoun investissement tregraves reacuteduit et neacutecessite un

contact permanent avec le client

Toutefois la standardisation des services et le deacuteveloppement des reacuteseaux informatiques

rapprochent la production de services de celle des biens industriels

- la production drsquoun service reacutepeacutetitif et technique peut imposer une structure lourde et une

organisation tregraves formaliseacutee (voir les grandes socieacuteteacutes drsquoaudit ou de conseil informatique)

Initiation au management copy CRCF ndash J Sornet Page 18 48

- certains services peuvent ecirctre fournis agrave distance sans contact direct avec le client et

distribueacutes par reacuteseau (tenue de comptabiliteacute affacturage gestion clientegravele centre drsquoappel

hellip)

Remarque les services repreacutesentent 75 de lrsquoactiviteacute eacuteconomique franccedilaise

23 ndash La nature de lrsquoorganisation

Les organisations publiques franccedilaises (administrations centrales collectiviteacutes territoriales

hocircpitaux hellip) repreacutesentent une part importante de lrsquoactiviteacute (environ 30 des emplois) La

fonction publique regroupe des organisations aux finaliteacutes diverses et qui ont des problegravemes

de gestion similaires agrave ceux des entreprises auxquelles elles peuvent emprunter des principes

de management Notamment

- pour controcircler les coucircts et assurer la qualiteacute des services

- pour communiquer avec les administreacutes ou les usagers

- pour motiver les personnels et geacuterer les ressources humaines

La transposition directe des techniques de gestion et de management nrsquoest cependant pas

toujours possible car

- la comptabiliteacute publique obeacuteit agrave des regravegles speacutecifiques (proceacutedure budgeacutetaire

notamment)

- le laquo client raquo ne paye pas toujours la prestation du moins directement

- la concurrence est parfois inexistante

- les grandes administrations centraliseacutees sont soumises agrave des choix politiques geacuteneacuteraux

parfois sans connexion eacutevidente avec les besoins opeacuterationnels

- le statut des personnels et les grilles de salaires limitent les possibiliteacutes de gestion des

ressources humaines

Remarque la LOLF (loi organique relative aux lois de finances) est entreacutee en vigueur en

2006 Elle alloue des moyens budgeacutetaires en fonction de programmes et remplace la

reconduction automatique de 90 des budgets Cette reacuteforme se heurte toutefois agrave la

lourdeur des grands ministegraveres ougrave la complexiteacute des activiteacutes est difficile agrave

appreacutehender et ougrave des inerties culturelles peuvent exister agrave tout niveau

Les associations loi de 1901 peuvent avoir une activiteacute comparable agrave celle de grandes

entreprises (voir par exemple les associations de santeacute ou professionnelles) et leur

management est alors similaire malgreacute lrsquoabsence de but lucratif (les beacuteneacutefices ne sont pas

distribuables) Elles ont drsquoailleurs en France un poids eacuteconomique important (elles emploient

environ 1 600 000 salarieacutes)

Cependant lrsquoadheacutesion agrave un systegraveme de valeurs fondateur de lrsquoassociation ou la limite de

lrsquoautoriteacute (quand un volant de beacuteneacutevoles important participe agrave lrsquoactiviteacute) peut introduire des

nuances

- le renforcement des objectifs socieacutetaux

- la faiblesse des relations hieacuterarchiques

- des contraintes de gestion du temps des beacuteneacutevoles

- des modaliteacutes particuliegraveres de recrutement et de motivation des dirigeants

24 ndash Les facteurs contingents

La theacuteorie de la contingence montre qursquoune structure drsquoentreprise nrsquoest efficace que dans

une situation deacutetermineacutee et qursquoil nrsquoexiste que des solutions de management construites dans

un contexte preacutecis

Le management doit ainsi srsquoadapter agrave des facteurs contingents qui ne peuvent ecirctre

controcircleacutes du moins agrave bregraveve eacutecheacuteance Ces facteurs sont par exemple

- lrsquoancienneteacute de lrsquoentreprise (plus elle est ancienne plus lrsquoentreprise a tendance agrave reacutepeacuteter

des comportements eacuteprouveacutes)

Initiation au management copy CRCF ndash J Sornet Page 19 48

- la taille de lrsquoentreprise (la grande entreprise a une composante administrative plus

deacuteveloppeacutee)

- le systegraveme de production (tregraves standardiseacute complexe automatiseacute hellip)

- lrsquoenvironnement

3 ndash Le management et les parties prenantes

Lrsquoentreprise a pour vocation premiegravere de mettre des produits agrave disposition de ses clients en

reacutealisant un profit Pour y arriver elle doit aussi satisfaire ses parties prenantes salarieacutes

actionnaires fournisseurs hellip

Est partie prenante agrave lrsquoentreprise laquo tout groupe ou individu qui peut ecirctre affecteacute ou est

affecteacute par les buts de lrsquoorganisation hellip raquo (Freeman ndash 1984)

Les parties prenantes attendent agrave des degreacutes divers de profiter drsquoune creacuteation de valeur en

provenance de lrsquoentreprise qui doit reacutepondre agrave ces attentes pour assurer sa peacuterenniteacute ou

favoriser son deacuteveloppement

On distingue les parties prenantes primaires ou principales qui sont essentielles agrave lrsquoentreprise

et qui ont geacuteneacuteralement une relation formelle avec elle (clients associeacutes et actionnaires

precircteurs salarieacutes fournisseurs collectiviteacutes) et les parties prenantes secondaires dont

lrsquoinfluence est diffuse (groupes de pression associations meacutedias instances europeacuteennes

agences de notation hellip)

Remarque la consideacuteration de lrsquoensemble des parties prenantes (laquo stakeholders raquo - les

deacutepositaires) fait contrepoids agrave lrsquoimportance accordeacutee aux seuls actionnaires

(laquo shareholders raquo)

Les organisations nrsquoayant pas drsquoobjectif de profit doivent aussi satisfaire leurs parties

prenantes apporter un service aux usagers dans les meilleures conditions eacuteconomiques

limiter un budget assurer la qualiteacute des relations avec les fournisseurs hellip

Dans cette optique le management doit organiser lrsquoaction de faccedilon agrave eacutequilibrer des forces

parfois divergentes

- le contexte fait pression sur lrsquoorganisation contrainte agrave optimiser ses reacutesultats

- lrsquoorganisation cherche par son action agrave assurer sa peacuterenniteacute son deacuteveloppement (en

reacutealisant des profits dans le cas de lrsquoentreprise) et agrave satisfaire ses parties prenantes

- le management agit en pilotant les actions pour contrebalancer la pression du contexte

Actions de

lrsquoorganisation

Management Contexte

Parties

prenantes

Initiation au management copy CRCF ndash J Sornet Page 20 48

APPLICATIONS MP

MP1 Deacutefinir contingent gestion budgeacutetaire

MP2 Deacuteterminer les parties prenantes drsquoun hocircpital public et leurs principales attentes

Mecircme question pour les organisations suivantes

- SNCF (entreprise publique)

- Peugeot

- MAIF (mutuelle drsquoassurance)

MP3 En les situant dans le cycle des activiteacutes du management trouver les actions agrave mener

dans les situations suivantes

- baisse de 10 des ventes dans une entreprise industrielle (produits meacutenagers le reacuteseau de

distribution vient drsquoecirctre reacuteorganiseacute)

- idem dans une entreprise de vente par correspondance soumise agrave la concurrence internet

(les ventes stagnaient depuis six mois malgreacute les efforts promotionnels)

- augmentation des deacutelais drsquoattente des consultations dans une clinique (lrsquohocircpital voisin a

fermeacute son service drsquourgences)

Initiation au management copy CRCF ndash J Sornet Page 21 48

ELEMENTS DE CORRIGE MP

MP1 Deacutefinir (dans le contexte drsquoune entreprise) contingent gestion budgeacutetaire

Contingent = imposeacute par lrsquoexteacuterieur Contingence = effet du hasard de la rencontre de

plusieurs eacuteveacutenements indeacutependants (variables explicatives que lrsquoon ne peut influencer)

Gestion budgeacutetaire = technique drsquoadministration des entreprises srsquoappuyant sur des

preacutevisions dont on deacuteduit apregraves accord des responsables des attributions de moyens sur une

dureacutee limiteacutee Une analyse reacuteguliegravere des eacutecarts entre preacutevisions et reacutealisations permet ensuite

le pilotage des activiteacutes Le budget est un cadre incitatif

La laquo planification budgeacutetaire raquo consiste agrave traduire en budgets une planification strateacutegique

avec systegraveme de reporting

MP2 Deacuteterminer les parties prenantes drsquoun hocircpital public et leurs principales attentes

Mecircme question pour les organisations suivantes

- SNCF (entreprise publique)

- Peugeot

- MAIF (mutuelle drsquoassurance)

Hocircpital

- patients (qualiteacute des soins)

- CNAM (baisse des coucircts)

- collectiviteacute locale (service aux administreacutes)

- eacutetat (ameacutenagement du territoire maicirctrise des budgets optimisation)

- employeacutes (salaire conditions de travail et satisfaction)

- fournisseurs ndash pharmacie autres (CA paiement reacutegulier)

- associations de patients (qualiteacute proximiteacute des soins)

SNCF

- usagers et associations drsquousagers (proximiteacute reacutegulariteacute prix du service)

- reacuteseau ferreacute de France (optimisation des lignes paiement adapteacute)

- fournisseurs (CA paiement reacutegulier)

- employeacutes (salaire conditions de travail seacutecuriteacute de lrsquoemploi)

- eacutetat (ameacutenagement du territoire)

- collectiviteacutes locales (service)

Peugeot

- clients (qualiteacute prix SAV relation commerciale)

- fournisseurs (CA reacutegulariteacute de lrsquoactiviteacute)

- employeacutes (salaire conditions de travail seacutecuriteacute de lrsquoemploi)

- eacutetat (taxes)

- collectiviteacute locale (emploi dynamisation eacuteconomique preacuteservation de lrsquoenvironnement)

- associations de protection de lrsquoenvironnement (activiteacute propre baisse des eacutemissions

nouvelles eacutenergies)

MAIF

- socieacutetaires (protection relation assureur tarif mesureacute)

- professionnels de lrsquoautomobile et autres (agreacutement marge de manœuvre reacuteparations tarifs

eacuteleveacutes)

- fournisseurs (CA paiement reacutegulier)

- eacutetat (taxes engagement pour la seacutecuriteacute)

- employeacutes (salaire conditions de travail seacutecuriteacute de lrsquoemploi)

Initiation au management copy CRCF ndash J Sornet Page 22 48

MP3 En les situant dans le cycle des activiteacutes du management trouver les actions agrave mener

dans les situations suivantes

- baisse de 10 des ventes dans une entreprise industrielle (produits meacutenagers le reacuteseau de

distribution vient drsquoecirctre reacuteorganiseacute)

Adapter le pilotage motiver cadrer si insuffisant retoucher une organisation deacutefectueuse

- idem dans une entreprise de vente par correspondance soumise agrave la concurrence internet

(les ventes stagnaient depuis six mois malgreacute les efforts promotionnels)

Voir pilotage et organisation si une eacutevolution du meacutetier a deacutejagrave eacuteteacute initialiseacutee Sinon re-

conception (adaptation au nouveau contexte) puis planification et reacuteorganisation

- augmentation des deacutelais drsquoattente des consultations dans une clinique (lrsquohocircpital voisin a

fermeacute son service drsquourgences)

Organisation Si insuffisant planification (nouveaux objectifs)

Initiation au management copy CRCF ndash J Sornet Page 23 48

ORGANISATION ET PROCESSUS

La performance de lrsquoentreprise deacutepend de son organisation et de son aptitude agrave produire

aux meilleures conditions Nous allons montrer comment organisation formelle et processus

de production peuvent contribuer agrave cette performance

1 ndash Vers lrsquooptimum

11 ndash Les eacuteconomies occidentales jusqursquoaux anneacutees 70

Jusqursquoen 1945 le principal problegraveme des entreprises eacutetait de produire des biens en quantiteacute

suffisante agrave un prix compatible avec le marcheacute Les grandes entreprises se sont multiplieacutees et

la standardisation a permis de reacuteduire les coucircts (exemple deacuteveloppement de Ford et de la

production agrave la chaicircne de 1908 agrave 1920 qui a permis une baisse du prix des voitures des 23)

On parle de laquo production pousseacutee vers le marcheacute raquo

Cette croissance de la production peu reacuteguleacutee a eacuteteacute marqueacutee par des surproductions en

1910 et 1920 puis par la crise de 1929 qui a prolongeacute ses effets jusqursquoagrave la guerre

De 1945 agrave 1975 environ (les laquo trente glorieuses raquo) la reconstruction la croissance de la

consommation de masse de nouvelles technologies et les eacutechanges internationaux

alimentent lrsquoeacuteconomie La standardisation srsquoeacutetend aux biens de consommation dont les

coucircts baissent fortement et de nouvelles reacutegulations sociales permettent une eacutevolution sans

heurt des revenus La saturation de certains marcheacutes conduit dans les anneacutees 60 agrave la

deacutemarche laquo marketing raquo et agrave la diffeacuterenciation des produits Le produit est laquo dirigeacute par le

marcheacute raquo mais les entreprises conservent une organisation assez classique et les plus grosses

srsquointernationalisent

12 ndash Lrsquoexpeacuterience japonaise et ses prolongements

Tregraves tocirct apregraves la guerre dans un Japon appauvri le constructeur automobile Toyota a ducirc

faire face agrave une restriction du marcheacute des moyens financiers et productifs et des

approvisionnements La firme a donc innoveacute dans un nouveau systegraveme de production

chassant les laquo gaspillages raquo (temps drsquoattente transports stocks deacutefauts hellip) consideacuterant que

seule la fabrication vendable creacutee de la valeur

Toyota srsquoorganise pour fabriquer la quantiteacute et la qualiteacute de produits juste neacutecessaires agrave la

satisfaction des clients la production est laquo tireacutee par le marcheacute raquo La mise en place de ce

systegraveme qui integravegre les fournisseurs ne sera acheveacutee que dans le milieu des anneacutees 70

En 1973 la hausse du peacutetrole inaugure un ralentissement de la croissance des eacuteconomies

occidentales La concurrence accrue provoque alors un inteacuterecirct pour le systegraveme deacuteveloppeacute

au Japon La production au plus juste se deacuteveloppe ainsi dans lrsquoindustrie automobile agrave partir

des anneacutees 80 et elle se reacutepand encore maintenant dans drsquoautres secteurs

Cette approche qui vise un objectif de zeacutero stock et zeacutero deacutefaut impose la maicirctrise de laquo bout

en bout raquo des processus de production et leur ameacutelioration

Initiation au management copy CRCF ndash J Sornet Page 24 48

2 ndash Organiser lrsquoentreprise

21 ndash Direction et organisation

Diriger une entreprise neacutecessite de lrsquoorganiser (de reacutepartir les tacircches) pour qursquoelle puisse

atteindre ses objectifs Lrsquoorganisation permet de satisfaire un marcheacute en tirant parti des

capaciteacutes actuelles de lrsquoentreprise tout en preacuteparant lrsquoavenir

Lrsquoorganisation reacutesulte freacutequemment drsquoun compromis entre des objectifs situeacutes agrave des niveaux

et des eacutecheacuteances diffeacuterents

Exemples

- le leader des chaises roulantes peut tirer profit de sa structure productive et de son

savoir faire pour entrer sur le marcheacute de la bicyclette eacutelectrique

- ecirctre parfaitement structureacute pour alimenter 90 du marcheacute des disquettes ne preacutepare

pas lrsquoavenir

- srsquoorganiser pour conqueacuterir le marcheacute des tire-bouchons eacutelectriques dans les deux ans

perd de son sens si cela altegravere les moyens neacutecessaires agrave la production drsquoappareils

manuels ancienne mais vitale dont la diminution agrave court terme risque de nuire agrave la

solvabiliteacute de lrsquoentreprise et de la conduire agrave la cessation de paiement

22 ndash Lrsquoorganisation fonctionnelle

La majoriteacute des entreprises adopte une laquo organisation fonctionnelle raquo (celle qui est visible

dans les organigrammes) ougrave des regroupements de personnels et drsquoeacutequipements se font

selon un modegravele hieacuterarchique (laquo line raquo) dans des uniteacutes des services ou des deacutepartements

speacutecialiseacutes Cette organisation peut se deacutecliner agrave lrsquointeacuterieur des divisions des grandes

entreprises quand elles scindent leur activiteacute par zone geacuteographique type drsquoactiviteacute

cateacutegorie de clients hellip

Remarque le terme laquo fonction raquo deacutesigne un rocircle particulier dans le fonctionnement de

lrsquoentreprise

Lrsquoorganisation fonctionnelle diffeacuterencie les activiteacutes de lrsquoentreprise en les regroupant par

meacutetier pour utiliser au mieux les compeacutetences et les moyens (meilleur rendement par la

speacutecialisation lrsquoeacutechange de compeacutetences dans une mecircme uniteacute ou gracircce agrave des eacuteconomies

drsquoeacutechelle)

23 ndash La notion de processus de production

Un processus de production se deacutefinit par la succession drsquoactiviteacutes permettant de satisfaire

un client en transformant des ressources (mateacuterielles financiegraveres humaines) en un produit

bien ou service Le processus doit creacuteer une valeur reconnue par le client

Un processus peut servir un client interne agrave lrsquoentreprise (par exemple en produisant un

composant intervenant dans plusieurs produits ou par la maintenance des machines) aussi

bien qursquoun client final On distingue usuellement

- les processus opeacuterationnels (ou maicirctres) aussi appeleacutes processus meacutetier (business process)

qui satisfont directement les clients finaux (conception et fabrication de produits vente hellip)

- les processus de support et de management (geacuterer les ressources humaines geacuterer

lrsquoinformation geacuterer les ressources financiegraveres hellip) qui ont les processus opeacuterationnels comme

clients

Toutes les actions internes agrave une organisation peuvent srsquointeacutegrer dans des processus qui

conditionnent directement ou indirectement la capaciteacute de lrsquoorganisation agrave satisfaire le

client final ou lrsquousager

Initiation au management copy CRCF ndash J Sornet Page 25 48

Aborder le fonctionnement de lrsquoentreprise par ses processus (approche processus) permet

de mettre en eacutevidence les chaicircnes drsquoactiviteacutes qui conduisent aux produits leurs

dysfonctionnements leurs coucircts la formation des deacutelais et la souplesse (la flexibiliteacute)

disponible pour satisfaire la clientegravele finale Lrsquoameacutelioration des processus a un impact visible

et direct sur chaque produit proposeacute aux clients

Lrsquoapproche processus provoque une eacutevolution de la faccedilon de travailler

- en faisant peacuteneacutetrer la laquo voix du client raquo au plus profond de lrsquoentreprise (et plus seulement

dans les services commerciaux et marketing)

- en mettant en eacutevidence des possibiliteacutes de rationalisation (par regroupement ou impartition

de certaines activiteacutes)

Remarque lrsquoapproche par les activiteacutes et les processus est agrave lrsquoorigine de la meacutethode

de deacutetermination des coucircts laquo ABC raquo - activity based costing

24 ndash Processus et fonctions

Le processus est transversal Il enchaicircne des activiteacutes qui traversent lrsquoentreprise en particulier

les services ou les deacutepartements drsquoune organisation fonctionnelle

Exemple

La division du travail par fonctions induit une charge de coordination pour assurer le

deacuteroulement du processus Elle peut geacuteneacuterer des attentes des erreurs ou des conflits drsquointeacuterecirct

(lrsquoobservation montre que des dysfonctionnements sont tregraves souvent constateacutes lors du

passage drsquoun service agrave un autre)

Organisation fonctionnelle et approche processus visent toutes deux un optimum

eacuteconomique mais leurs logiques sont diffeacuterentes

- le processus vise la satisfaction des clients (prix qualiteacute deacutelais service)

- le deacutecoupage fonctionnel cherche agrave optimiser les moyens (maximiser lrsquoeffet drsquoexpeacuterience

partager des infrastructures profiter de pocircles de compeacutetences hellip) Il apporte une ossature

hieacuterarchique stable souvent indispensable

Organisation fonctionnelle et approche processus sont donc compleacutementaires dans la

majoriteacute des cas et doivent ecirctre combineacutees judicieusement

APPLICATIONS OP

OP1 Deacutefinir flexibiliteacute systegraveme impartition

OP2 Citer huit exemples drsquoinformations essentielles pour optimiser un processus de

fabrication

Direction

Deacutepartement

commercial

(C)

Deacutepartement

administratif et

financier (AF)

Deacutepartement

Etudes (E)

Deacutepartement

Production (P)

Activiteacute

C-x Activiteacute

AF-x Activiteacute

E-x

Activiteacute

P-x

Processus x

Clie

nt

Initiation au management copy CRCF ndash J Sornet Page 26 48

OP3 Deacutegager les principes du toyotisme preacutesenteacute ci-dessous En quoi ce systegraveme est-il

initiateur de lrsquoapproche processus

Taiichi Ohno et le Toyotisme

1 - Extrait drsquoun article de Jacques BARRAUX - 1993 - LExpansion

Taiichi Ohno (1912 ndash 1990) hellip ne se prenait pas pour un visionnaire mais en imposant une

nouvelle faccedilon de produire il a reacuteinventeacute le management hellip tout le monde a entendu parler

des mots qui ont populariseacute le toyotisme dont il est le pegravere le juste-agrave-temps hellip Autant

doutils conccedilus pour lrsquoautomobile et qui ont aujourdhui une application universelle

hellip Taiichi Ohno jeune ingeacutenieur entre chez Toyota alors simple constructeur de machines

textiles Degraves 1926 apparaicirct la notion de jidoka hellip cest lart de transfeacuterer de lintelligence aux

machines pour mieux libeacuterer lintelligence des hommes Tout le contraire du taylorisme qui

juge la machine moins impreacutevisible que lhomme En 1933 Toyota se lance dans lautomobile

en sinspirant des meacutethodes ameacutericaines Mais en 1935 agrave loccasion dun voyage aux Etats-

Unis leacutetat-major de lentreprise revient fascineacute de sa visite dans un supermarcheacute La notion

de juste-agrave-temps va naicirctre de lobservation dune grande surface un lieu ougrave les clients ne

prennent que ce dont ils ont besoin et ougrave les rayons sont reacuteapprovisionneacutes pour compenser

les quantiteacutes preacuteleveacutees Ainsi le systegraveme Toyota est-il deacutejagrave dans la tecircte de ses dirigeants avant

mecircme la Seconde Guerre mondiale un demi-siegravecle avant la reacutevolution informatique et la

segmentation intensive des marcheacutes

hellip des esprits curieux comme Franccedilois Dalle en France tombent alors sous le charme des

formules et des paraboles de Taiichi Ohno En voici deux eacutechantillons

Penser agrave lenvers Cela signifie combattre les ideacutees reccedilues En lespegravece il sagit du fordisme et

du taylorisme Ohno ne croit pas agrave la planification aux effets deacutechelle et dexpeacuterience Il

propose un systegraveme industriel agrave lenvers qui permette de diversifier les produits et de les

fabriquer en petites quantiteacutes Nous ne devons plus ecirctre des paysans qui accumulent des

stocks mais des chasseurs On nimpose pas loffre On traque la demande et on la gegravere en

continu

Que les valleacutees soient hautes et les montagnes peu eacuteleveacutees Plutocirct que de concentrer tous

les efforts sur une production agrave un moment donneacute mieux vaut se doter de structures flexibles

permettant de passer agrave tout instant dune seacuterie agrave une autre Il faut eacuteviter les ruptures et les

secousses aplanir les cycles entretenir des flux reacuteguliers dactiviteacutes diversifieacutees Ce qui

implique de ne pas enfermer les hommes et les eacutequipements dans des speacutecialisations trop

eacutetroites

La flexibiliteacute le travail en groupe le refus de la dictature des machines la polyvalence et

surtout lattention constante aux signaux eacutemis par le marcheacute nappartiennent plus au

toyotisme Ces notions sont les fondements du nouvel art dorganiser de vendre et de

produire dans lindustrie comme dans les services hellip

2 - Quelques notions cleacutes

Taiichi Ohno a imagineacute la meacutethode des laquo cinq pourquoi raquo qui consiste agrave se poser cinq fois de

suite la question laquo pourquoi raquo sur le mecircme sujet de faccedilon agrave deacutecouvrir la veacuteritable cause

drsquoun problegraveme Cette meacutethode peut ecirctre appliqueacutee agrave tous les niveaux et permettre

notamment aux agents de fabrication de proposer de veacuteritables ameacuteliorations de la

production

La recherche de la qualiteacute totale (pas de deacutefaut des produits pas de rebuts pas de deacutefaut

des processus) accompagne la deacutemarche de Toyota La qualiteacute a un coucirct compenseacute par

des ventes accrues par lrsquoeacuteconomie des mesures palliatives aux deacutefauts

Initiation au management copy CRCF ndash J Sornet Page 27 48

Fiche OP1 ndash Benchmarking et processus

Le laquo benchmarking raquo consiste agrave comparer le fonctionnement de plusieurs systegravemes pour en

faire notamment ressortir les meilleures pratiques (laquo best practices raquo) Cette technique est

utiliseacutee depuis les anneacutees 80 pour ameacuteliorer la performance des entreprises Elle impose agrave

lrsquoentreprise drsquoeacutevaluer et de remettre en question ses propres modes de fonctionnement afin

de les faire eacutevoluer agrave la lueur de ce qui se fait ailleurs

Le benchmarking permet drsquoameacuteliorer les processus agrave moindre risque en fixant des objectifs

baseacutes sur des faits et donc plus facilement accepteacutes

Une classification des processus en tant que base de reacuteflexion a eacuteteacute eacutetablie aux USA par

lrsquolaquo International Benchmarking Clearinghouse raquo de lrsquoAPQC (american productivity and

quality center) en collaboration avec plusieurs dizaines drsquoentreprises

Elle se reacutesume ainsi

Le terme laquo reengineering raquo (la re-conception ou laquo reacuteingeacutenieacuterie raquo) des processus deacutesigne un

projet drsquoameacutelioration radicale des performances (de 20 agrave 50 ou plus) Il neacutecessite une

parfaite adheacutesion de la direction la constitution drsquoune petite eacutequipe de projet brillante

connaissant parfaitement les activiteacutes de lrsquoentreprise et il peut inclure un benchmarking

Le reengineering provoque geacuteneacuteralement la reacuteduction du nombre de niveaux hieacuterarchiques

(laquo delayering raquo) et lrsquoaccroissement du pouvoir de deacutecision des employeacutes (laquo empowerment raquo

ou laquo empouvoirement raquo) Bien qursquoy conduisant parfois il ne doit pas ecirctre confondu avec la

reacuteduction des activiteacutes (laquo downsizing raquo ou restructuration) et lrsquoexternalisation (laquo outsourcing raquo)

Pro

ce

ssu

s

op

eacutera

tio

nn

els

Pro

ce

ssu

s d

e m

an

ag

em

en

t e

t d

e

sup

po

rt

1 ndash

Comprendre

le marcheacute et

les clients (besoins

satisfaction)

2 ndash

Deacutevelopper

vision et

strateacutegie (contexte

concurrence)

3 ndash

Creacuteer

produits

services

processus

(concevoir

ameacuteliorer)

4 ndash

Marketing et

vente

5 ndash

Produire et

livrer (industrie

dont

ameacutelioration

processus)

6 ndash

Produire et

livrer (services)

7 ndash

Facturer et

servir les

clients (apregraves-

vente

reacuteclamations)

8 ndash Deacutevelopper et geacuterer les ressources humaines

9 ndash Geacuterer les systegravemes drsquoinformation

10 ndash Geacuterer les ressources financiegraveres et les actifs

11 ndash Appliquer un programme environnemental

12 ndash Geacuterer les relations exteacuterieures (actionnaires banques lois relations publiques hellip)

13 ndash Geacuterer lrsquoameacutelioration et le changement (eacutevaluer mesurer motiver qualiteacute totale)

Initiation au management copy CRCF ndash J Sornet Page 28 48

Fiche OP2 ndash Lrsquoorganisation par processus

Lrsquoeacutevolution drsquoune organisation aux activiteacutes reacutepeacutetitives vers lrsquoapproche processus est

geacuteneacuteralement progressive et se met en place par paliers

La mise en œuvre drsquoun veacuteritable management par processus doit ecirctre preacuteceacutedeacutee quand

lrsquoactiviteacute de lrsquoentreprise est complexe drsquoun recensement (une laquo cartographie des

processus raquo) pour mettre en eacutevidence les processus ou les familles de processus cleacutes critiques

pour le succegraves de lrsquoentreprise ougrave les efforts seront prioritaires

Des responsables de processus (laquo process owners raquo) sont ensuite deacutesigneacutes

Le responsable doit concevoir ses processus puis apregraves leur mise en œuvre assurer les

coordinations neacutecessaires les ameacuteliorer et les repreacutesenter aupregraves de la direction

Quand une structure par processus est mise en place des opeacuterateurs exeacutecutants

preacuteceacutedemment regroupeacutes dans les fonctions peuvent ecirctre affecteacutes aux processus et

drsquoanciens responsables de fonctions peuvent devenir des experts au service des processus

Lrsquoorganisation par processus peut imposer un degreacute eacuteleveacute drsquointeacutegration des activiteacutes donc

une polyvalence accrue des personnels et une reacuteduction des niveaux hieacuterarchiques

Elle neacutecessite pour le moins des compeacutetences eacutelargies au niveau des responsables de

processus (organisation administration technique hellip) dont le nombre doit rester limiteacute

(quelques dizaines au plus)

Sauf dans de tregraves petites structures lrsquoorganisation par processus se plaque geacuteneacuteralement sur

une structure plus classique

Initiation au management copy CRCF ndash J Sornet Page 29 48

ELEMENTS DE CORRIGE OP

OP1 Deacutefinir

Flexibiliteacute = adaptation au besoin (horaire variable chaicircnes robotiseacutees)

Systegraveme = ensemble organiseacute dans un but boicircte noire (sanguin nerveux meacutetrique laquo D raquo)

Impartition = sous-traitance ou externalisation (seacuteparation) drsquoactiviteacutes faire appel agrave des

partenaires plutocirct que faire soi-mecircme

OP2 Citer huit exemples drsquoinformations essentielles pour orienter lrsquooptimisation drsquoun processus

Montant des stocks (approvisionnements et produits finis)

Temps drsquoattente

Taux drsquoactiviteacute des ateliers

Rebuts

Deacutelai de production

Taux de reacuteclamations clients (qualiteacute)

Temps passeacutes en retouches finales

Turn over

Nombre drsquoarrecircts maladie

Accidents du travail

Dureacutee des arrecircts machines

OP3 Deacutegager les principes du toyotisme preacutesenteacute dans la fiche 31 En quoi ce systegraveme

repose trsquoil sur lrsquoapproche processus

Produire la quantiteacute juste neacutecessaire (agrave la demande) donc eacuteviter les stocks

Flexibiliteacute intelligence des chaicircnes de production

Qualiteacute (eacuteviter le coucirct de la non-qualiteacute)

La notion de processus est implicite ainsi que la chaicircne de valeur client

Initiation au management copy CRCF ndash J Sornet Page 30 48

DEFIS ET TENDANCES DU MANAGEMENT

Les meacutethodes de management se deacuteveloppent pour affronter le contexte eacuteconomique

Ce chapitre preacutesente les deacutefis auxquels le management contemporain doit faire face

1 ndash Lrsquoeacutevolution eacuteconomique contemporaine

A mesure que lrsquoactiviteacute eacuteconomique mondiale srsquoaccroicirct que la technologie eacutevolue les

changements sont de plus en plus rapides Ils introduisent des situations ineacutedites auxquelles les

entreprises doivent srsquoadapter en cherchant de nouvelles solutions de management Les trois

derniegraveres deacutecennies ont eacuteteacute notamment marqueacutees par les pheacutenomegravenes suivants (que nous

listons sans tenir compte des liens pouvant exister entre eux)

Pheacutenomegravene Traduction Effets

Deacute reacuteglementation

globalisation

financiegravere

titrisation

Libre circulation des capitaux accegraves

facile des particuliers au marcheacute

boursier (directement ou par

lrsquointermeacutediaire des OPCVM et SICAV)

Monteacutee en puissance du financement

des entreprises sur le marcheacute boursier

Fonds de pension

(retraites) et fonds

souverains (eacutetats)

Poids boursier important drsquoinvestisseurs

institutionnels qui cherchent un haut

rendement financier (dividendes ou

valorisation boursiegravere)

Pression sur les grandes entreprises

influence sur les strateacutegies

Mondialisation Liberteacute des eacutechanges internationaux Accroissement de la concurrence

recherche drsquoavantages eacuteconomiques

par la deacutelocalisation (biens et

services) la concentration des efforts

(recentrage) problegravemes drsquoemploi

multiplication des transports perte

drsquoinfluence des politiques

Baisse de lrsquoemploi

occidental

(notamment

industriel)

Moins de fabrications fabrications

automatiseacutees recours aux moyens

informatiques

Activiteacute reporteacutee sur le commerce la

conception et les services chocircmage

charge sociale

Restructurations Optimisation des entreprises

abaissement des coucircts augmentation

des marges recherche drsquoune taille

critique (eacuteconomies drsquoeacutechelle poids

sur le marcheacute)

Recentrages externalisations fusions

deacutelocalisations constitution de grands

groupes

NTIC (nouvelles

technologies de

lrsquoinformation et de

la communication)

Mise en œuvre des reacuteseaux (dont

internet) et drsquoapplications

informatiques communicantes

Nouvelles formes de commerce

marcheacute international deacutelocalisation

du travail intellectuel reacuteorganisation

de la distribution

Rareacutefaction relative

des matiegraveres

premiegraveres

Recherche de substituts exploration

miniegravere coucircts drsquoexploitation des

gisements accrus

Augmentation des coucircts variations

erratiques du cours des matiegraveres

deacutestabilisations politiques

Evolution

geacuteopolitique et

eacuteconomique

mondiale

Chute de lrsquoURSS transformation des

eacuteconomies collectivistes pays

eacutemergents (Chine Inde Breacutesil Russie)

()

Accroissement de la population

mondiale (4 agrave 6 7 milliards de 1970 agrave

2008)

Libeacuteralisme sans frein () nouvelles

puissances eacuteconomiques

opportuniteacutes de deacuteveloppement

nouveau partage des ressources

ineacutegaliteacutes baisse du soutien aux PVD

laquo Terrorisme raquo Actions armeacutees pression de groupes

armeacutes non gouvernementaux

Deacutestabilisations reacutegionales charge

des deacutepenses militaires

Deacuteveloppement

durable

Recherche drsquoune croissance eacutequitable

et respectueuse de lrsquoenvironnement

Pression sur les entreprises (eacutetats

associations de consommateurs

eacutecologistes ONG)

() Reacutecemment quelques affaires (Enron laquo subprimes raquo Vivendi Universal Socieacuteteacute

Geacuteneacuterale Airbus par exemple) et agrave plus grande eacutechelle la crise financiegravere de 2008 ont

montreacute les dangers drsquoune libeacuteralisation sans controcircles suffisants

Initiation au management copy CRCF ndash J Sornet Page 31 48

() Des alliances eacuteconomiques naissent entre pays eacutemergents (notamment en

ameacuterique centrale creacuteation de la Banque du Sud en 2008 par exemple) et lrsquoon

commence agrave imaginer une baisse progressive de lrsquoinfluence eacuteconomique des Etats

Unis

2 ndash Les deacutefis actuels du management

21 ndash Les grandes orientations

Lrsquoeacutevolution eacuteconomique suggegravere quelques pistes parfois concurrentes pour lrsquoaction du

manager contemporain On y retrouve au premier plan la construction drsquoune vision qui est

une composante commune du leadership

Objectif du manager

pour lrsquoentreprise

Justification Facteurs de reacuteussite

Construire une vision Eclairer lrsquoavenir de lrsquoentreprise partager

un but souder motiver

Effort de reacuteelle prospection

volontarisme de la direction

bonne communication

Reacuteactiviteacute et flexibiliteacute

(sous tous les aspects

agrave tous niveaux)

Srsquoadapter rapidement au marcheacute Bonne organisation des processus

personnel compeacutetent autonome

et motiveacute structure hieacuterarchique

alleacutegeacutee robotisation

Deacutegager des profits Reacutemuneacuterer les apporteurs de capitaux

srsquoautofinancer

Ajuster coucircts et structures

Exploiter les nouvelles

technologies

Reacuteactiviteacute ajuster coucircts et deacutelais

reacutepondre au marcheacute suivre les clients

Organiser le SI de faccedilon

pertinente eacuteviter le coucirct excessif

drsquoinvestissements trop en

laquo pointe raquo (laquo essuyer les placirctres raquo)

utiliser judicieusement les services

exteacuterieurs

Bacirctir des alliances

(contrats fusions)

Deacutevelopper une activiteacute limiter les coucircts

de transaction () atteindre la taille

critique et de meilleurs rendements se

recentrer sur une activiteacute profitable

Dominer les processus se donner

une identiteacute lisible externaliser se

doter drsquoune capaciteacute financiegravere

suffisante

Valoriser lrsquoimage Attirer les clients favoriser les alliances

donner confiance (apporteurs de fonds

employeacutes clients partenaires socieacuteteacute

civile)

Instaurer des regravegles de

gouvernance inteacutegrer le

deacuteveloppement durable

respecter lrsquoenvironnement

Geacuterer les risques Faire face aux aleacuteas eacuteconomiques et

technologiques (conjoncture politiques

accidents malveillance)

Creacuteer un systegraveme drsquoalerte geacuterer

la crise (reacuteaction raisonneacutee

sceacutenarios poursuite de

lrsquoexploitation dans un contexte

instable) mise en place de

proceacutedures drsquoapprentissage pour

ameacuteliorer les reacuteactions au fil du

temps

Geacuterer le changement Faire face agrave lrsquoeacutevolution de la demande

la pression sur les prix la variation des

performances financiegraveres la

concurrence la globalisation des

marcheacutes lrsquoeacutevolution technologique aux

fusions ou alliances aux changements

de reacuteglementation de direction hellip ()

Bonne communication pour

donner du sens au changement

et obtenir lrsquoadheacutesion du personnel

Rassembler et geacuterer les

connaissances former le

personnel

Innover Garder un avantage concurrentiel se

diffeacuterencier

Veille technologique et

commerciale investissement

Ouverture

internationale

Elargir le marcheacute saisir les opportuniteacutes Veille commerciale partenariats

() La theacuteorie des coucircts de transaction deacuteveloppeacutee par OE Williamson dans les

anneacutees 70 integravegre les coucircts lieacutes au recours au marcheacute (recherche et choix drsquoun

fournisseur neacutegociation reacutedaction de contrat suivi des eacutechanges risque de rupture

Initiation au management copy CRCF ndash J Sornet Page 32 48

drsquoapprovisionnement hellip) On peut en conclure que lrsquointeacutegration de diffeacuterentes

activiteacutes agrave lrsquoentreprise (la laquo firme raquo) preacutesente des avantages Mais des coucircts de

transaction internes doivent aussi ecirctre consideacutereacutes (preacuteparation organisation

surveillance hellip) et certaines formes de coopeacuteration continue avec les fournisseurs

permettent de reacuteduire le coucirct des transactions externes

() drsquoapregraves laquo Les meilleures pratiques de management raquo - Brilman Heacuterard ndash EO

Une eacutetude du Conference Board (2002) liste les deacutefis du management vus par 700 leaders

mondiaux Soit en reacutesumeacute avec indication du score correspondant

1 ndash Fideacuteliser les clients (42)

2 ndash Reacuteduire les coucircts (38)

3 ndash Accroicirctre flexibiliteacute et reacuteactiviteacute (29)

4 ndash Amener les employeacutes agrave adheacuterer aux valeurs et visions de lrsquoentreprise (26)

5 ndash Deacutevelopper et retenir les leaders (25)

6 ndash Geacuterer acquisitions et alliances (24)

7 ndash Accroicirctre lrsquoinnovation (20)

En fin de classement citoyenneteacute et reacuteputation (4) et ameacutelioration de la diversiteacute (3)

22 ndash Les techniques disponibles

Pour faire face aux deacutefis le manager dispose de nouveaux concepts et de nouvelles

techniques Le tableau ci-dessous en donne un reacutesumeacute et indique les domaines qursquoils

influencent principalement

Initiation au management copy CRCF ndash J Sornet Page 33 48

Principaux concepts techniques outils Incidence principale sur

Internet

- e-commerce (commerce eacutelectronique site

entreprise)

- CRM ou GRC (gestion de la relation client)

- e-procurement (gestion des approvisionnements

par le reacuteseau)

- messagerie eacutelectronique

- e-recrutement

Vente accegraves au marcheacute

Relation client reacuteactiviteacute personnalisation

fideacutelisation

Deacutelais coucircts

Communication transfert de donneacutees (piegraveces

jointes) tous domaines

Communication recrutement

Intranet reacuteseau drsquoentreprise SI

- knowledge management (gestion des

connaissances)

- e-learning (apprentissage en ligne)

- plateforme de travail collaboratif (groupware)

- workflow (circulation eacutelectronique de

documents enchaicircnement de processus)

- e-RH portail RH (libre accegraves aux postes agrave

pourvoir informations candidatures hellip)

- PGI (progiciel de gestion inteacutegreacute) ou ERP

Innovation capaciteacute au changement veille

documentaire

Formation du personnel accompagnement des

changements

Coordination communication interne

Coordination

Communication interne (voire internet en

externe) reacuteduction des coucircts climat drsquoentreprise

recrutement plans de carriegraveres hellip

Coucircts fiabiliteacute du systegraveme drsquoinformation deacutelais

processus (continuiteacute inteacutegration)

Logistique inteacutegreacutee

Supply Chain Management (SCM) gestion de la

logistique (incluant les approvisionnements)

Processus deacutelais coucircts

Externalisation

Valorisation du capital humain

GPEC (gestion preacutevisionnelle des emplois et

compeacutetences)

Coaching

Reacuteactiviteacute de lrsquoentreprise conservation des

compeacutetences rendements individuels turn-over

adaptation des compeacutetences motivation

Efficaciteacute individuelle controcircle reacutegulation

progregraves processus

Approche processus

Optimisation des processus

Deacutemarche qualiteacute totale (TQM ndash total quality

management)

Empowerment (empouvoirement)

Benchmarking reacuteingeacutenieacuterie

Coucircts marges qualiteacute deacutelais flexibiliteacute

externalisation eacutelargissement des compeacutetences

organisation

Ameacutelioration des processus (meacutetiers et supports)

Autonomie compeacutetences des employeacutes

Ameacutelioration des processus restructuration

Management par la valeur

Parties prenantes

Satisfaction des parties prenantes financement

motivation collaborations hellip

Collaboration inter organisations

Reacuteseaux drsquoentreprises alliances

EDI (eacutechange de donneacutees informatiseacutees) extranet

Impartition externalisation (outsourcing)

Coucircts recentrage investissements lancement

drsquoactiviteacute

Coucircts reacuteactiviteacute deacutelais relations avec

lrsquoadministration

Coucircts recentrage limitation des investissements

Ethique drsquoentreprise

Gouvernance drsquoentreprise (mode de direction

encadreacute par des regravegles)

Rocircle socieacutetal deacuteveloppement durable

environnement

Image de lrsquoentreprise reacutegulation du top

management relations actionnaires

Image peacutenaliteacutes et amendes objectifs

strateacutegiques

Initiation au management copy CRCF ndash J Sornet Page 34 48

23 ndash Le rocircle socieacutetal des entreprises

La responsabiliteacute socieacutetale de lrsquoentreprise (RSE) deacutesigne le rocircle qursquoelle prend dans la socieacuteteacute

au-delagrave de son activiteacute purement geacuteneacuteratrice de profit On parle aussi drsquoentreprise citoyenne

La RSE est indissociable du deacuteveloppement durable de porteacutee mondiale et dont les trois

piliers sont

- eacuteconomique (favoriser le deacuteveloppement les eacutechanges internationaux)

- social (accegraves aux soins eacuteducation conditions de travail hellip)

- environnemental (pollution preacuteservation des ressources hellip)

La RSE integravegre notamment une preacuteoccupation sociale de lrsquoentreprise vis-agrave-vis de ses salarieacutes

(seacutecuriteacute et santeacute au travail juste reacutemuneacuteration deacuteveloppement personnel hellip) Elle conduit agrave

tenir compte dans le management drsquoune vision exteacuterieure agrave lrsquoentreprise qui peut avoir des

reacutepercussions possibles sur son activiteacute eacuteconomique

Lrsquoentreprise peut aussi tirer avantage drsquoune deacutemarche responsable par la baisse de certains

coucircts (plus faibles consommations drsquoeacutenergies reacuteduction des transports hellip)

Le rocircle socieacutetal de lrsquoentreprise a eacuteteacute reconnu en France par la loi laquo NRE raquo de 2001 (loi sur les

nouvelles reacutegulations eacuteconomiques) qui oblige les socieacuteteacutes franccedilaise coteacutees sur un marcheacute

reacuteglementeacute agrave rendre compte dans leur rapport annuel de leur gestion sociale et

environnementale au travers de leur activiteacute

Article 116 de la loi Le rapport viseacute agrave larticle L 225-102 rend compte hellip laquo Il comprend

eacutegalement des informations dont la liste est fixeacutee par deacutecret en Conseil dEtat sur la

maniegravere dont la socieacuteteacute prend en compte les conseacutequences sociales et

environnementales de son activiteacute Le preacutesent alineacutea ne sapplique pas aux socieacuteteacutes

dont les titres ne sont pas admis aux neacutegociations sur un marcheacute reacuteglementeacute raquo

Une norme ISO 14000 integravegre ces preacuteoccupations et des taxes eacutecologiques sont

progressivement creacutees

3 ndash Le management par la valeur

31 ndash De lrsquoanalyse au management par la valeur

Lrsquoanalyse de la valeur est neacutee en 1947 aux Etats-Unis (General Electrics) Cette technique

consiste agrave eacutelaborer des produits conformes aux attentes de la clientegravele mais sans excegraves pour

trouver un bon compromis entre valeur pour le client et coucirct Le produit optimal est deacutefini agrave

partir drsquoenquecirctes qui deacuteterminent le besoin client (ou plutocirct drsquoun client laquo type raquo)

Exemple il est inutile de concevoir un petit veacutehicule citadin capable de parcourir

500 000 km sans avarie compte tenu des effets de mode et du faible kilomeacutetrage

annuel Par contre le marcheacute peut exiger un fonctionnement sans faille sur 150 000 km

soit dix ans en moyenne ce qui conditionne les coucircts de production

Cette recherche drsquoun ajustement de valeur au besoin des clients eacutetait un preacutecurseur du

management par la valeur qui recherche plus largement la creacuteation de valeur pour

chacune des parties prenantes de lrsquoentreprise tout en lui meacutenageant un reacutesultat suffisant

Plus geacuteneacuteralement le management par la valeur est deacutefini par une norme europeacuteenne (EN

12973)

Le management par la valeur est un style de management particuliegraverement destineacute agrave

mobiliser les individus agrave deacutevelopper les compeacutetences et agrave promouvoir les synergies et

Initiation au management copy CRCF ndash J Sornet Page 35 48

linnovation avec pour objectif la maximisation de la performance globale dun

organisme Le management par la valeur apporte une nouvelle faccedilon dutiliser nombre

de meacutethodes de management existantes Il est en coheacuterence avec le Management

de la qualiteacute

Cette approche du management pose de nombreuses questions notamment quelles

prioriteacutes et quelles valeurs attribuer aux parties prenantes comment appreacutehender la

perception par les parties prenantes de la valeur qui leur est affecteacutee

32 ndash La valeur client

Le processus drsquoeacutelaboration drsquoun produit qui consomme des ressources coucircteuses doit creacuteer

une valeur suffisante pour provoquer lrsquoachat par le client final La production drsquoune valeur

reconnue par le client est vitale pour lrsquoentreprise mais sa deacutetermination est parfois complexe

La valeur du produit perccedilue par le client integravegre des eacuteleacutements en partie subjectifs

- une valeur drsquousage (le produit reacutepond agrave un besoin)

- une valeur drsquoestime (lrsquoimage apporteacutee par le produit un aspect affectif)

- une valeur drsquoeacutechange (deacuteduite de lrsquoespoir de revente du produit)

Valeurs drsquousage drsquoestime et drsquoeacutechange deacutependent implicitement de la qualiteacute (un bien peu

fiable est impropre agrave lrsquousage attendu de mauvaise qualiteacute notoire il nrsquoapporte pas une

image positive et ses deacutefauts connus nuisent agrave sa revente) Une eacutevaluation de la qualiteacute

intervient donc dans la valeur perccedilue du produit

Par ailleurs le client considegravere le coucirct drsquoobtention du produit (les charges qursquoil doit supporter

pour acqueacuterir le produit lrsquoeffort qursquoil doit faire pour trouver le produit et les frais de mise agrave

disposition)

Le prix perccedilu par le client est geacuteneacuteralement supeacuterieur au prix de vente

Le client achegravete theacuteoriquement le produit qui preacutesente la diffeacuterence valeur perccedilue ndash prix

perccedilu la plus favorable ou le meilleur rapport prix perccedilu qualiteacute perccedilue et dans certains

cas celui qui a le prix produit le plus bas

Remarque les valeurs du scheacutema ci-dessus changent durant le cycle de vie du produit

(un nouveau produit peut avoir une valeur perccedilue plus eacuteleveacutee qursquoen fin de vie) La

valeur client ne peut ecirctre eacutevalueacutee que par enquecirctes et ne peut donc ecirctre deacutefinie avec

certitude

La notion de laquo satisfaction client raquo conseacutecutive agrave une vente influence aussi le prix produit et

le prix perccedilu

- lrsquoentreprise gagne sur les coucircts de recherche de clientegravele

- le client nrsquoa pas agrave rechercher un nouveau fournisseur et beacuteneacuteficie drsquoun coucirct drsquoobtention

plus bas

valeur perccedilue client

prix perccedilu client

coucirct produit Marge (valeur creacuteeacutee pour

lrsquoentreprise)

euros

prix produit

Valeur creacuteeacutee

pour le client

Initiation au management copy CRCF ndash J Sornet Page 36 48

La satisfaction du client deacutepend de facteurs qualitatifs aussi divers que la fiabiliteacute du produit

la vitesse de reacuteaction du fournisseur lrsquoattitude des commerciaux lrsquoefficaciteacute du service

apregraves-vente la netteteacute des contrats ou la justesse de la facture

Valeur perccedilue coucirct marge et satisfaction reacutesultent de processus allant de la conception du

produit jusqursquoagrave sa livraison et son apregraves-vente La deacutemarche laquo processus raquo et lrsquolaquo analyse de la

valeur raquo en forccedilant la recherche de solutions efficientes agrave tout niveau administratif

technique commercial et apregraves-vente sont donc neacutecessaires pour bien positionner

lrsquoentreprise sur son marcheacute

Pour autant le risque commercial ne peut jamais ecirctre annuleacute et lrsquooffre de lrsquoentreprise ne

satisfait geacuteneacuteralement pas en milieu concurrentiel tous ses clients potentiels

33 - La creacuteation de valeur pour les autres parties prenantes

Les salarieacutes

La creacuteation drsquoune valeur suffisante pour les salarieacutes est reconnue comme neacutecessaire car des

observations montrent que la satisfaction des clients en deacutepend Moins souvent eacutevoqueacutee en

peacuteriode de chocircmage elle nrsquoest prioritaire que pour les employeacutes dont lrsquoentreprise souhaite

conserver les compeacutetences

La laquo valeur salarieacute raquo ne comprend pas que le salaire Le sentiment drsquoappartenance agrave un

groupe la reconnaissance lrsquoaccomplissement de soi et la construction professionnelle en

sont des eacuteleacutements importants Comme pour les clients on doit ainsi distinguer la reacutetribution

perccedilue du salaire objectif

Les actionnaires

Lrsquoactionnaire apporte des fonds propres agrave lrsquoentreprise en contrepartie de titres parfois

neacutegociables en bourse et assortis drsquoun droit de vote en assembleacutee geacuteneacuterale La valeur

attribueacutee aux actionnaires est servie en termes moneacutetaires (dividende ou augmentation de la

valeur du titre neacutegociable)

Remarque des facteurs non moneacutetaires comme lrsquoimage de lrsquoentreprise qui deacutepend

en partie de sa communication peuvent influencer la deacutecision drsquoachat de vente ou

de conservation des titres par lrsquoactionnaire

Reacutetribution perccedilue euros

Salaire objectif

Avantage non

moneacutetaire de

lrsquoemploi

Initiation au management copy CRCF ndash J Sornet Page 37 48

Compte tenu de lrsquoimportance croissante de lrsquoactionnariat dans le financement des grandes

entreprises coteacutees en bourse et notamment des investisseurs institutionnels comme les fonds

de pension des indicateurs speacutecifiques ont eacuteteacute introduits pour appreacutecier la performance des

entreprises vue par les actionnaires Par exemple la valeur ajouteacutee eacuteconomique (EVA reg

economic value added marque deacuteposeacutee de Stern Stewart ou VAE ndash valeur ajouteacutee

eacuteconomique parfois deacutenommeacutee VEC ndash valeur eacuteconomique creacuteeacutee) qui prend en compte le

coucirct du capital

LrsquoEVA correspond tregraves scheacutematiquement au calcul suivant

EVA = (PO) profit opeacuterationnel ndash (C) coucirct du capital X (CE) capitaux employeacutes

LrsquoEVA neacutecessite en pratique des retraitements assez complexes Le PO peut se deacuteterminer

selon les principes suivants

- PO = reacutesultat drsquoexploitation (avant inteacuterecircts) ndash impocirct

- PO = beacuteneacutefice courant (tenant compte des inteacuterecircts) + inteacuterecircts ndash eacuteconomie drsquoimpocirct sur les

inteacuterecircts (on exclue les eacuteleacutements financiers et lrsquoimpocirct correspondant) ndash impocirct

- lrsquoimpocirct pris en compte correspond au profit opeacuterationnel consideacutereacute (dans les cas courants agrave

13 du PO)

C = taux moyen de reacutemuneacuteration du capital (reacutesultant par exemple du dividende exigeacute de

certains investisseurs et des taux drsquoemprunts bancaires)

CE = capitaux propres et dettes portant inteacuterecirct

Remarque le profit opeacuterationnel ou reacutesultat opeacuterationnel correspond au NOPAT ndash net

operating profit after tax - anglo-saxon LrsquoEVA est eacutegale au NOPAT diminueacute de la

reacutemuneacuteration des capitaux

Exemple lrsquoentreprise X dispose drsquoun capital de 2 500 000 euro et reacutealise un beacuteneacutefice net

drsquoimpocirct de 450 000 euro (taux 33 13) Un dividende de 6 doit ecirctre verseacute aux

actionnaires et la banque lui a accordeacute un precirct de 1 200 000 euro agrave 4 Les autres

constituants des reacutesultats financier et exceptionnel sont neacutegligeables

Reacutesultat opeacuterationnel = 450 000 + 004 x 1 200 000 x 23 = 482 000 euro

Coucirct du capital = 006 x 2 500 000 + 004 x 1 200 000 x 23 = 182 000 euro

EVA = 300 000 euro

Coucirct moyen pondeacutereacute du capital (C) = (004 x 1 200 000 x23 + 006 x 2 500 000)

3 700 000 Soit 492

Si lrsquoEVA est positive lrsquoentreprise creacuteeacutee de la valeur apregraves reacutemuneacuteration des capitaux et sa

valeur boursiegravere doit augmenter

Lrsquoutilisation de lrsquoEVA comme indicateur influence le management de lrsquoentreprise car il y a

trois moyens pratiques drsquoaugmenter lrsquoEVA

- augmenter le reacutesultat opeacuterationnel

- lancer des investissements ayant une rentabiliteacute supeacuterieure agrave C

- eacuteliminer les activiteacutes ayant une rentabiliteacute infeacuterieure agrave C

Remarque lrsquoutilisation sans nuance de lrsquoEVA comme critegravere de management peut

poser problegraveme Le calcul de lrsquoEVA repose sur des ajustements comptables il est donc

sujet agrave manipulations (provisions capitalisation ou non de la RD hellip) Par ailleurs le

critegravere laquo EVA raquo pris isoleacutement peut conduire agrave chercher la rentabiliteacute agrave court terme agrave

reacuteduire les investissements prospectifs et donc nuire agrave terme au deacuteveloppement de

lrsquoentreprise

Initiation au management copy CRCF ndash J Sornet Page 38 48

Les fournisseurs reccediloivent le paiement de leurs factures plus ou moins rapidement (le deacutelai

de paiement repreacutesente une valeur consentie au fournisseur)

Lrsquoentreprise peut accroicirctre la valeur apporteacutee agrave ses fournisseurs par des actions cibleacutees

comme une contribution agrave la formation de leurs personnels certains transferts de

technologie ou de savoir faire agrave des sous-traitants une coopeacuteration suivie favorisant leur

deacuteveloppement lrsquointeacutegration agrave des campagnes de promotion

A noter que la valeur consentie aux fournisseurs peut avoir une influence sur la qualiteacute et les

deacutelais de livraison des produits

La collectiviteacute reccediloit des taxes et parfois des prestations en nature par deacutefaut ou explicites

(effort de preacuteservation de lrsquoenvironnement ameacutenagement du territoire par les implantations

aide mateacuterielle agrave des projets participation agrave la formation par exemple)

APPLICATIONS DT

DT1 Deacutefinir expliquer deacutereacuteglementation socieacutetal eacuteconomies drsquoeacutechelle coaching EDI

gouvernance

DT2 Deacuteterminer en quoi la deacutemarche TQM srsquoinscrit dans les deacutefis actuels du management

DT3 Apregraves avoir consulteacute les documents ci-dessous extraits du site drsquoAir France

(httpdeveloppement-

durableairfrancecomFRfrlocaldemarcheN4_positionnement_pphtm)

exposer les enjeux et les limites de la RSE et de la gestion des parties prenantes

Initiation au management copy CRCF ndash J Sornet Page 39 48

Dialogue avec les parties prenantes

Initiation au management copy CRCF ndash J Sornet Page 40 48

Attentes des parties prenantes

Initiation au management copy CRCF ndash J Sornet Page 41 48

Creacuteation de valeur pour les parties prenantes

La creacuteation de valeur pour les parties prenantes est au cœur de la strateacutegie du Groupe Le scheacutema de

distribution financiegravere ci-dessous donne un aperccedilu de la distribution des recettes du Groupe aux

diffeacuterentes parties prenantes actionnaires collaborateurs fournisseurs pouvoirs publics

collectiviteacutes locales etc

Initiation au management copy CRCF ndash J Sornet Page 42 48

Fiche DT1 ndash Extrait du sommaire de laquo Problegravemes eacuteconomiques raquo No 2894

La gestion des entreprises bouleverseacutee par les technologies de linternet

Reacutealiteacutes industrielles - Annales des Mines Jean-Michel Yolin

Avec lavegravenement de linternet les processus de conception de production et de vente sont

radicalement remis en cause Quel que soit le secteur dactiviteacute les technologies de linternet

permettent en effet de reacuteduire les deacutelais et de passer dun processus discontinu agrave un processus

continu Lorganisation des entreprises et leur mode de gestion en sont profondeacutement bouleverseacutes

tant au niveau individuel que collectif Linternet rend ainsi possible la reacutealisation dobjectifs que les

entreprises cherchaient agrave atteindre depuis longtemps sans y parvenir meilleure eacutecoute du client

travail sans stocks en flux tendu hieacuterarchies plates autorisant une grande reacuteactiviteacute flexibiliteacute dans

lorganisation et loutil de production acceacuteleacuteration du renouvellement des produits entreprises en

reacuteseau ougrave chacune se recentre sur son cœur de meacutetier etc

Le laquo knowledge management raquo ou comment geacuterer les connaissances

Document de travail du LAMSADE - Michel Grundstein

Peter Drucker lavait preacutedit le capital immateacuteriel eacutetait voueacute agrave devenir un facteur de compeacutetitiviteacute

pour lentreprise La libeacuteralisation des eacutechanges acceacutelegravere les processus de deacutecision de lentreprise

et implique que lassimilation des informations soit agrave la fois de meilleure qualiteacute et plus rapide Ainsi

la fonction qui consiste agrave manager les connaissances au sein de lentreprise savegravere primordiale

Bien que la prise de conscience de limportance du capital immateacuteriel ait eacuteteacute tardive - le concept

de knowledge management est apparu en France aux Etats-Unis et au Japon au milieu des

anneacutees 1990 - agrave lheure actuelle lorganisation de leacutechange dinformations et le partage des

connaissances sont devenus des facteurs cleacutes dune gestion performante de lentreprise Ils

doivent sinscrire dans un projet global destineacute agrave mettre en valeur les savoirs et les savoir-faire

individuels et collectifs

Les leccedilons du laquo coaching raquo pour le management de la qualiteacute

Humanisme et Entreprise - Martine Brasseur

Parmi les nouvelles formes de management en vogue dans les entreprises le coaching figure en

bonne place Appliqueacute au management de la qualiteacute il sagit dune pratique

daccompagnement destineacutee agrave initier et agrave faciliter le processus de deacuteveloppement dun individu

La deacutemarche consiste agrave affirmer que tout individu est en quecircte de qualiteacute agrave condition toutefois

de ne pas lui imposer des contraintes lempecircchant de progresser On considegravere notamment les

erreurs comme potentiellement feacutecondes En deacutefinitive le coach donne au coacheacute la permission

de reacuteussir en lui donnant aussi la permission deacutechouer

Initiation au management copy CRCF ndash J Sornet Page 43 48

Fiche DT2 ndash Management strateacutegique les sept deacutefis agrave relever dici agrave 2016

Extrait drsquoun article du site wwwlentreprisecom -Sabine Blanc - Mis en ligne le 20032007

(httpwwwlentreprisecom325article11977html)

Une eacutetude anglaise publieacutee par lopeacuterateur Orange Grande-Bretagne deacutecrypte la mutation

des formes de travail et les enjeux majeurs pour les entreprises de demain afin decirctre au top

de la compeacutetitiviteacute Voici les challenges-cleacutes pour les managers qui veulent rester dans la

course hellip

1 - Future organisation du travail les quatre laquo mondes raquo possibles

La reacutealiteacute sera probablement un meacutelange de ces quatre sceacutenarios souligne lrsquoeacutetude

Les mondes mutuels Tout se passe dans le cadre des communauteacutes locales vie priveacutee

comme professionnelle Le modegravele coopeacuteratif preacutevaut au lieu du laquo big business raquo Oublieacutes

aussi dans ce systegraveme les trajets pour aller au bureau les gens preacutefegravereront travailler dans de

petites entreprises locales souvent connecteacutees au reacuteseau drsquoautres structures similaires

Les laquo reacutepondants raquo (en anglais laquo replicants raquo) La figure du consultant freelance deviendra

dominante tandis que celle du salarieacute deacuteclinera Il ne sera pas rare de travailler pour plusieurs

entreprises On perdra en seacutecuriteacute de lrsquoemploi en visibiliteacute et en routine ce que lrsquoon gagnera

en liberteacute La majeure partie des tacircches srsquoeffectuera chez soi avec la possibiliteacute de srsquoinstaller

temporairement dans les bureaux de son client du moment Dans un contexte dincertitude

sur lrsquoavenir les travailleurs alterneront peacuteriodes drsquoactiviteacute intense et repos Ce sera agrave eux

drsquoaller vers les entreprises et non lrsquoinverse mecircme si celles-ci devront veiller agrave rester attractives

Les cottages eacutelectroniques Comme ce nom le suggegravere le teacuteleacutetravail deviendrait la norme

univers priveacute et professionnel se confondant Plus besoin de subir une heure de transport les

salarieacutes se logueront de chez eux sur le reacuteseau de lrsquoentreprise Les reacuteunions se tiendront dans

de petits bureaux centraux situeacutes agrave courte distance La flexibiliteacute du temps de travail srsquoimpose

Les salarieacutes disposeront de plus de marge de liberteacute dans leur activiteacute

Les disciples de la nueacutee Cette appellation poeacutetique cache simplement une extension de

lrsquoorganisation actuelle des grandes entreprises avec des salarieacutes se rendant sur un lieu de

travail centraliseacute Le rocircle croissant des technologies de lrsquoinformation multipliera les faccedilons de

collaborer et accroicirctra lrsquoefficaciteacute Le controcircle du travail sera omnipreacutesent La frontiegravere entre

travail et vie priveacutee restera marqueacutee

2 - Sept deacutefis pour les entreprises et leur managers

Quoi qursquoil advienne les entreprises et leurs dirigeants devront concentrer leurs efforts sur sept

points-cleacutes pour srsquoadapter Voici quelques exemples de probleacutematiques souleveacutees par le

rapport et des pistes de solution

Le leadership Les managers devront entre autres savoir persuader et influencer des

travailleurs beaucoup plus indeacutependants Ils auront aussi agrave repenser les niveaux auxquels

prendre les deacutecisions strateacutegiques en haut ou au contraire agrave des degreacutes moins eacuteleveacutes de la

pyramide hieacuterarchique

gt Faire du management une force facilitant les activiteacutes transversales plutocirct que la reacuteduire agrave

la seule fonction de deacutecision

La culture drsquoentreprise Davantage de salarieacutes capables de reacutefleacutechir seront neacutecessaires

tandis que les tacircches qui peuvent ecirctre automatiseacutees ou scripteacutees diminueront Un des

enjeux creacuteer une culture agrave mecircme drsquoattirer et drsquoencourager les personnes preacutesentant ces

qualiteacutes de reacuteflexion requises dans un contexte de compeacutetition accrue et de plus grande

indeacutependance des travailleurs

Initiation au management copy CRCF ndash J Sornet Page 44 48

gt Passer si neacutecessaire drsquoune culture drsquoentreprise forte agrave un mode drsquoengagement plus

consensuel moins rebutant

La marque Conseacutequence du recours croissant agrave lrsquo laquo outsourcing raquo lrsquoimage drsquoune marque

deacutependra plus drsquoagents exteacuterieurs qui ne fonctionnent pas forceacutement selon le mecircme mode

drsquoorganisation Comment garder le controcircle dessus

gt Choisir le mode qui corresponde le plus agrave vos valeurs et preacutevoir un programme de risk

management qui mette en eacutevidence ougrave les conflits sont susceptibles de jaillir

Lrsquoinnovation Plus que jamais il faudra faire face agrave une acceacuteleacuteration du rythme de

lrsquoinnovation en proposant constamment des solutions adapteacutees

gt Tisser des partenariats strateacutegiques avec drsquoautres entreprises pour partager les coucircts et les

fruits de lrsquoinnovation

Le deacutefi opeacuterationnel et technologique De quelle faccedilon controcircler lrsquoinformation crsquoest-agrave-dire

faire en sorte que les bonnes personnes accegravedent facilement agrave une information toujours en

phase tout en maintenant la seacutecuriteacute

gt Recourir agrave des laquo feuilles de route des futurs raquo syntheacutetisant en une page les indicateurs

sociaux et de consommation ainsi que les eacutevolutions technologiques et leacutegislatives qui

influent sur les changements et indiquant comment ils modifient vos marcheacutes vos clients et

votre organisation

La qualiteacute Si de nouveaux proceacutedeacutes ont pu deacutegrader la qualiteacute comme le recours agrave des

centres drsquoappel externaliseacutes drsquoautres ideacutees se sont reacuteveacuteleacutees plus prometteuses comme en

teacutemoigne le succegraves de certaines compagnies aeacuteriennes low cost Elles ont su conjuguer prix

serreacutes et services eacuteleveacutes ce qui devra devenir la norme estime lrsquoeacutetude

gt Continuer de rechercher la qualiteacute Elaborez aussi une bonne prestation service qui inclut

une livraison de qualiteacute voire creacuteez-la en partenariat avec les consommateurs

La leacutegislation La question de la proprieacuteteacute intellectuelle pourrait ecirctre probleacutematique Elle est

deacutejagrave source de conflits comme en teacutemoigne le procegraves pour violation de brevet intenteacute agrave RIM

le fabricant canadien du Blackberry par NTP Que pourra-t-on et que faudra-t-il proteacuteger par

un brevet Il sera eacutegalement neacutecessaire drsquoadapter la leacutegislation aux nouveaux modes

drsquoorganisation

gt Collaborer avec les acteurs du mecircme secteur et les leacutegislateurs pour deacutevelopper les

modegraveles des lieux de travail du futur et bacirctir le droit le plus adeacutequat

Orange a-t-il vu juste dans ses preacutevisions Rendez-vous dans neuf ans pour la reacuteponsehellip

Initiation au management copy CRCF ndash J Sornet Page 45 48

Fiche DT3 ndash Le management par la qualiteacute totale

Extrait drsquoune lettre drsquoinformation du cabinet Baud Accordance Consulting AD2 consultants ndash

2002

1 - Le TQM (Total Quality Management) offre pour lentreprise une vision de la qualiteacute plus

large et transversale

Son principe est simple La finaliteacute de lEntreprise est de deacutevelopper la satisfaction de ses

clients tout en eacutetant beacuteneacuteficiaire cest agrave dire pas agrave nimporte quel prix Elle doit ameacuteliorer sa

rentabiliteacute au travers de la deacutemarche qualiteacute La Qualiteacute Totale vise agrave fournir aux clients

externes et internes une reacuteponse adeacutequate agrave leurs attentes dans le meilleur rapport qualiteacute

prix la meilleure efficience

Elle considegravere pour cela lensemble des processus de lentreprise ayant une incidence sur la

qualiteacute et la satisfaction des clients

Le TQM fait ainsi une large place agrave

la deacutefinition et la planification de la strateacutegie geacuteneacuterale

la coheacuterence de la politique qualiteacute avec la strateacutegie

la deacutemultiplication de la politique qualiteacute dans toutes les directions de lentreprise

la relation client fournisseur interne

la prise en compte de lenvironnement concurrentiel

la consideacuteration de lensemble des risques potentiels financiers sociaux concurrentielshellip

limplication et la motivation du personnel

lanalyse des besoins des clients et le positionnement marketing

la maicirctrise des processus transverses internes

les reacutesultats sous tous ses aspects y compris financiers commerciaux image

De nombreux reacutefeacuterentiels sont relatifs agrave la Qualiteacute Totale hellip Tous ces reacutefeacuterentiels imposent un

questionnement plus profond et indiscret sur le mode de fonctionnement de lentreprise et

son management

helliphellip

2 - LISO 9001 2000 au travers du deacuteploiement des processus (management supports

reacutealisation et ameacutelioration continue) reacutepond quelque peu agrave la mecircme logique

LISO est une ouverture indeacuteniable vers la logique du TQM mais ne se reacutefegravere pas agrave la notion

defficience

Les dirigeants sont cependant sensibles agrave la neacutecessaire reacuteduction des coucircts de non-qualiteacute

et dobtention de la qualiteacute agrave la rentabiliteacute du systegraveme de management de la qualiteacute

mais ne perccediloivent pas toujours la qualiteacute comme une deacutemarche globale

Les deacutemarches qualiteacute commencent bien souvent par la remise en cause de lorganisation

leacutevaluation critique de son efficaciteacute lexamen des processus et la mise en eacutevidence des

lourdeurs administratives

La qualiteacute devient laffaire de tous hellip

Initiation au management copy CRCF ndash J Sornet Page 46 48

Fiche DT4 ndash Le deacuteveloppement durable et la RSE

Extrait du site wwwvigeocom

(httpwwwvigeocomcsr-rating-agencyfrmethodologiecriteres-de-recherche37-

criteres-d-analysehtml)

Deacuteveloppement durable laquo un deacuteveloppement qui reacutepond aux besoins du preacutesent sans compromettre

la capaciteacute des geacuteneacuterations futures de reacutepondre aux leurs raquo (Commission mondiale sur lrsquoenvironnement

et le deacuteveloppement ndash 1987)

Reacutefeacuterentiel drsquoeacutevaluation des entreprises par le groupe Vigeacuteo (le groupe mesure les performances et le

niveau de maicirctrise des risques de responsabiliteacute sociale des entreprises et des organisations - site

wwwvigeocom)

1 Ressources Humaines Ameacutelioration continue des relations professionnelles des relations drsquoemploi et des conditions de travail 2 Droits humains sur les lieux de travail Respect de la liberteacute syndicale et promotion de la neacutegociation collective non discrimination et promotion de lrsquoeacutegaliteacute eacutelimination des formes de travail proscrites (enfants travail forceacute) preacutevention des traitements inhumains ou deacutegradants de type harcegravelements sexuels protection de la vie priveacutee et des donneacutees personnelles 3 Environnement Protection sauvegarde preacutevention des atteintes agrave lenvironnement mise en place drsquoune strateacutegie manageacuteriale approprieacutee eacuteco conception protection de la biodiversiteacute et maicirctrise rationnelle des impacts environnementaux sur lrsquoensemble du cycle de vie des produits ou services

4 Comportements sur les marcheacutes Prise en compte des droits et inteacuterecircts des clients inteacutegration de standards sociaux et environnementaux dans la seacutelection des fournisseurs et sur lrsquoensemble de la chaicircne drsquoapprovisionnement preacutevention effective de la corruption respect des regravegles concurrentielles 5 Gouvernement drsquoentreprise Efficience et probiteacute assurance de lrsquoindeacutependance et de lrsquoefficaciteacute du Conseil drsquoadministration effectiviteacute et efficience des meacutecanismes drsquoaudit et de controcircle et notamment inclusion des risques de responsabiliteacute sociale respect des droits des actionnaires et notamment des minoritaires transparence et rationaliteacute de la reacutemuneacuteration des dirigeants 6 Engagement socieacutetal Effectiviteacute inteacutegration manageacuteriale de lrsquoengagement contribution au deacuteveloppement eacuteconomique et social des territoires drsquoimplantation et de leurs communauteacutes humaines engagements concrets en faveur de la maicirctrise des impacts socieacutetaux des produits et des services contribution transparente et participative agrave des causes drsquointeacuterecirct geacuteneacuteral

Initiation au management copy CRCF ndash J Sornet Page 47 48

ELEMENTS DE CORRIGE DT DT1 Deacutefinir expliquer

Deacutereacuteglementation = suppression des contraintes eacuteconomiques (libre eacutechange des biens et

capitaux)

Socieacutetal = qui se rapporte agrave la structure agrave lrsquoorganisation ou au fonctionnement de la socieacuteteacute

Economies drsquoeacutechelle = reacuteduction des coucircts lieacutee au niveau drsquoactiviteacute (amortissement des

charges fixes)

Coaching = accompagnement de personnes ou deacutequipes pour le deacuteveloppement de leurs

potentiels

EDI = eacutechange de donneacutees informatiseacutees ET standardiseacutees (ex SWIFT bancaire edifact

documents deacuteclaratifs)

Gouvernance = exercice du pouvoir la bonne gouvernance est participative et eacutequitable

conforme agrave lrsquointeacuterecirct commun

DT2 Deacuteterminer en quoi la deacutemarche TQM srsquoinscrit dans les deacutefis actuels du management

Voir notamment fiche 43

Maicirctrise des processus reacuteduction des coucircts reacuteactiviteacute et satisfaction de la clientegravele = faire

face agrave la concurrence

Ameacutelioration de lrsquoimage motivation du personnel

DT3 Apregraves avoir consulteacute les documents ci-dessous extraits du site drsquoAir France

(httpdeveloppement-

durableairfrancecomFRfrlocaldemarcheN4_positionnement_pphtm)

exposer les enjeux et les limites de la RSE et de la gestion des parties prenantes

Trame geacuteneacuterale possible

Introduction

Les deacutefis contemporains (accroissement de la concurrence devenue mondiale recherche

de nouveaux avantages concurrentiels pression de la socieacuteteacute besoin drsquoimage et de projet

lisible pour mener lrsquoentreprise crise et scandales du libeacuteralisme hellip) RSE et PP

Deacuteveloppement (voir cours)

1 ndash Parties prenantes et management par la valeur

PP deacutefinir citer reacutesumer lrsquoavantage rechercheacute (fideacuteliser motiver recherche drsquoalliances

implicites)

PP moyens (dont exemples AF) et meacutethode de management par la valeur (reacutepartie)

2 ndash La responsabiliteacute socieacutetale de lrsquoentreprise

RSE 3 axes

- eacuteconomique (favoriser le deacuteveloppement les eacutechanges internationaux)

- social (accegraves aux soins eacuteducation conditions de travail hellip)

- environnemental (pollution preacuteservation des ressources hellip)

RSE gouvernance drsquoentreprise facteur drsquoimage inteacutegrable dans la deacutemarche PP

Article 116 de la loi Le rapport viseacute agrave larticle L 225-102 rend compte hellip laquo Il comprend

eacutegalement des informations dont la liste est fixeacutee par deacutecret en Conseil dEtat sur la maniegravere

dont la socieacuteteacute prend en compte les conseacutequences sociales et environnementales de son

activiteacute Le preacutesent alineacutea ne sapplique pas aux socieacuteteacutes dont les titres ne sont pas admis aux

neacutegociations sur un marcheacute reacuteglementeacute raquo

Initiation au management copy CRCF ndash J Sornet Page 48 48

RSE exemple AF (ONG fournisseurs)

3 ndash Liens entre PP et RSE

- la RSE introduit de nouvelles PP

- la RSE suppose le respect des PP usuelles (employeacutes clients notamment)

4 - Probleacutematique

- deacutefinir la valeur reacuteellement apporteacutee par une gestion des PP (confusion salaire ndash valeur

idem impocircts hellip ex laquo valeur ajouteacutee raquo)

- communication (neacutecessaire mais aller au-delagrave)

- marginaliteacute des deacutepenses RSE (efficaciteacute sinceacuteriteacute de lrsquoengagement marge de manœuvre)

- charge RSE reporteacutee sur des tiers (ex fournisseurs AF)

- inteacutegration de facteurs non visibles en comptabiliteacute (pertes drsquoemploi nuisances hellip)

Conclusion

Voies incontournables mais pouvant nrsquoavoir qursquoun effet superficiel et temporaire Voir utiliteacute

drsquoaccompagnement leacutegislatif de regravegles de gouvernance

Initiation au management copy CRCF ndash J Sornet Page 8 48

Le management a cependant une viseacutee unique et concregravete la bonne marche des

organisations et les travaux des universitaires rejoignent lrsquoaction des praticiens (les

universitaires reacutealisent drsquoailleurs bien souvent leurs recherches au sein des organisations)

Au plan peacutedagogique les deux visions du management se complegravetent

- la vision acadeacutemique met en perspective des concepts et des theacuteories Elle permet de

srsquoapproprier des raisonnements essentiels et stables notamment concernant la strateacutegie et

les reacuteactions humaines qui sont le moteur des organisations

- la vision pratique relate le veacutecu des praticiens preacutesente des techniques concregravetes et tente

parfois une vulgarisation favorisant la diffusion des concepts Elle integravegre de faccedilon

pragmatique les objectifs eacuteconomiques et les contraintes de fonctionnement des

organisations en se placcedilant parfois dans une vision agrave court terme

Sur le terrain le management reacutealise une synthegravese entre des techniques de psychologie

sociale et des techniques de gestion de diverses origines et de nouvelles meacutethodes

apparaissent aussi sous le label unique laquo management raquo

Le terme manageacuterial (approche manageacuteriale theacuteorie manageacuteriale pratique manageacuteriale

hellip) fait reacutefeacuterence agrave la vision aux preacuteoccupations et aux actions des managers qui doivent

emmener leur organisation vers la reacutealisation de ses objectifs

Exemples

- la deacutetermination des coucircts et des marges reacutesulte de techniques de gestion

indispensable aux deacutecisions des managers

- le laquo CRM raquo (customer relationship management en franccedilais GRC ndash gestion de la

relation client) est un concept reacutecent qui srsquoaccompagne de techniques lieacutee aux plus

reacutecents deacuteveloppements du management et des systegravemes drsquoinformation

- le laquo coaching raquo est une technique de management permettant une eacutevolution

personnelle dans le sens des objectifs de lrsquoentreprise

- le recentrage sur le laquo meacutetier raquo repose sur des techniques de management

Les techniques sont rassurantes mais pas suffisantes et si les theacuteories ne sont pas

indispensables au praticien elles aident agrave comprendre agrave anticiper et agrave bien utiliser les

techniques

La qualiteacute du management reacuteside beaucoup dans la capaciteacute agrave appliquer concepts et

techniques de faccedilon pertinente et agrave innover Cette capaciteacute relegraveve en partie drsquoun laquo art du

management raquo qui srsquoacquiert en grande partie par la pratique

4 ndash Management et expertise comptable

Lrsquoexpert comptable doit manager ses propres eacutequipes Il est par ailleurs supposeacute laquo hellip

conseiller et accompagner le chef drsquoentreprise dans toutes ses deacutecisions hellip raquo (selon lrsquoOEC)

Ce rocircle est particuliegraverement important dans ses relations avec les petites entreprises Il doit

donc avoir une capaciteacute au management

Organisation

Vision acadeacutemique Vision pratique

Initiation au management copy CRCF ndash J Sornet Page 9 48

Lrsquointervention de lrsquoexpert dans le management drsquoune entreprise peut toutefois poser

quelques problegravemes

- Il peut y avoir conflits drsquointeacuterecirct entre activiteacutes de certification des comptes et de conseil en

management (la tendance est agrave la seacuteparation des activiteacutes dans les plus grands cabinets)

- lrsquoactiviteacute de conseil neacutecessite des compeacutetences parfois tregraves speacutecifiques (conseil fiscal

conseil en RH conseil en systegravemes drsquoinformation hellip)

- le conseil est une activiteacute diffeacuterente par sa forme de lrsquoexpertise comptable (interventions

longues peu reacutepeacutetitives peu codifieacutees mises en concurrence) qui neacutecessite une

organisation particuliegravere du cabinet lorsqursquoelle deacutepasse lrsquointervention occasionnelle

APPLICATIONS IM

IM1 Analyser la profession de laquo manager raquo selon Henry Mintzberg (texte extrait de lrsquoouvrage

laquo Le management raquo Eyrolles - Editions drsquoOrganisation) et les principes du management de la

norme ISO

Faire ressortir les eacuteleacutements speacutecifiques agrave chacune de ces approches et mettre en eacutevidence

leurs points communs

Initiation au management copy CRCF ndash J Sornet Page 10 48

Principes du management drsquoapregraves la norme ISO 9001 (2000)

- Orientation vers le client (satisfaire ses attentes)

- Leadership (les dirigeants eacutetablissent les orientations de lrsquoorganisme Ils doivent creacuteer

un environnement interne ougrave les personnes peuvent clairement srsquoimpliquer dans la

reacutealisation des objectifs de lrsquoorganisme)

- Implication du personnel (les personnes sont agrave tout niveau lrsquoessence de lrsquoorganisme et

leur implication permet drsquoutiliser leurs aptitudes au profit de lrsquoorganisme)

- Approche laquo processus raquo (un reacutesultat est mieux atteint quand les ressources et les

activiteacutes neacutecessaires sont geacutereacutees comme un processus)

- Approche systegraveme (assimiler les processus correacuteleacutes agrave un systegraveme contribue agrave

lrsquoefficaciteacute et agrave lrsquoefficience de lrsquoorganisme vis-agrave-vis de ses objectifs)

- Ameacutelioration continue (objectif permanent de lrsquoorganisme)

- Prise de deacutecision efficace (par lrsquoanalyse de donneacutees et drsquoinformations)

- Relations mutuellement beacuteneacutefiques avec les fournisseurs (pour augmenter la capaciteacute

des deux organismes agrave creacuteer de la valeur)

IM2 Distinguer leader et manager

IM3 Compleacuteter le tableau ci-dessous en analysant chaque action preacutesenteacutee Faire ensuite

ressortir les domaines niveaux ou techniques de management pouvant ecirctre mobiliseacutes pour

chaque situation

Initiation au management copy CRCF ndash J Sornet Page 11 48

Caracteacuteristiques

de lrsquoaction

- reacutepeacutetition

- risque

- normes

- ampleur

Prise de

deacutecision

- opeacuterationnelle

strateacutegique

- deacutelai

Informations

neacutecessaires

- nature

- origine

- deacutelai obtention

Cleacutes pour la

reacuteussite

Intervention

exteacuterieure

possible

Assurer la

restauration du

soir

(restaurant

familial)

Construire un

viaduc

(autoroute)

Certifier les

comptes

annuels drsquoun

groupe

national

(cabinet

drsquoaudit)

Lancer une

ligne drsquoavions

(constructeur

aeacuteronautique)

Reacuteduire la

capaciteacute de

production

(groupe

industriel)

Acqueacuterir une

entreprise

concurrente

(teacuteleacutephonie

mobile)

Initiation au management copy CRCF ndash J Sornet Page 12 48

Fiche IM1 - Deacutefinitions du management

Dictionnaire anglais - franccedilais direction administration gestion intrigue manegravege

Wikipeacutedia Le management est lensemble des techniques dorganisation qui sont mises en

oeuvre pour ladministration dune entiteacute

Au point de vue eacutetymologique le verbe manage vient de litalien maneggiare (controcircler)

influenceacute par le mot franccedilais manegravege (faire tourner un cheval dans un manegravege) A cette

notion il faut aussi ajouter la notion de meacutenage (geacuterer les affaires du meacutenage) qui consiste agrave

geacuterer des ressources humaines et des moyens financiers

helliphellip

Fiche IM2 - Etudier le management

Concreacutetiser

Manager neacutecessite de syntheacutetiser des informations parfois complexes incomplegravetes et de

domaines tregraves divers pour en deacuteduire des actions Une approche trop parcellaire peut

conduire agrave lrsquoeacutechec et le savoir-faire est neacutecessaire pour agir vite avec un minimum de risque

Lrsquoeacutetudiant doit se preacuteparer simultaneacutement aux examens et agrave la pratique Il nrsquoa souvent connu

lrsquoentreprise que durant quelques semaines de stage et le manque de laquo recul raquo ne lui permet

pas toujours de concreacutetiser les theacuteories Il doit compenser par la lecture (ouvrages revues

journaux eacuteconomiques et boursiers) et en eacutetant attentif aux informations ambiantes (tout en

relativisant le style journalistique) en mettant en relation le cours les concepts les modegraveles

lrsquoactualiteacute les stages

Savoir traiter un exercice

Pour reacuteussir un examen ou traiter une application peacutedagogique (la conception les points 1 agrave

6 peut repreacutesenter le tiers du temps de travail)

1 ndash Identifier le type de sujet (faut-il trouver une solution pratique ou communiquer une

reacuteflexion geacuteneacuterale )

2 ndash Lire le sujet et relever les mots cleacutes

3 ndash Deacutefinir les mots cleacutes

4 ndash Reacutesumer la probleacutematique du sujet (en quelques lignes)

5 ndash Lister les connaissances reacutefeacuterences et raisonnements reacutepondant au problegraveme (par

recherche spontaneacutee ou raisonneacutee qui quoi ougrave quand comment combien hellip

listage des diffeacuterents points de vue) trouver des exemples (notamment dans les

documents fournis)

6 ndash Organiser la reacuteponse (deacutefinir le plan du deacuteveloppement ougrave des paragraphes bien

identifieacutes sont geacuteneacuteralement neacutecessaires en y liant les parties qui doivent ecirctre en nombre

limiteacute ndash de deux agrave quatre) Preacutevoir drsquoy inteacutegrer la deacutefinition des principales notions

induites par le sujet

7 ndash Reacutediger sous la forme adapteacutee (note technique ou recommandation solution

pratique exposeacute structureacute dissertation)

Introduction et conclusion sont indispensables agrave la dissertation ou agrave lrsquoexposeacute

- lrsquointroduction preacutesente le sujet traiteacute (phrase drsquoaccroche initiale) amorce la

probleacutematique (quelques sous - questions) et annonce le plan

- la conclusion syntheacutetise le deacuteveloppement (arguments) eacutelargit le sujet (prise de recul)

et apporte le point final (une phrase)

Une limite agrave la communication

Il est difficile de faire passer plus de 4 ou 5 ideacutees fortes dans un exposeacute unique

Initiation au management copy CRCF ndash J Sornet Page 13 48

Fiche IM3 - Bref historique

Antiquiteacute

3000 AJC

Peacuteriode greacuteco-

romaine

Transition

feacuteodale

12egraveme siegravecle

europe

15egraveme ndash 17egraveme

siegravecles

19egraveme siegravecle

20egraveme siegravecle

agriculture preacutedominante industrie limiteacutee aux besoins drsquoun individu ou drsquoun clan

pour la confection des outils des vecirctements et de la poterie Force motrice animale

ou humaine pour lrsquoessentiel

Grands travaux drsquoeacutetat en Egypte premiegravere laquo planification ndash organisation ndash controcircle raquo

Deacuteveloppement des communications essor industriel limiteacute peu de progregraves

technique (lrsquoesclavage supplante les innovations)

Deacuteveloppement progressif des eacutechanges commerciaux

La consommation indirecte atteint un bon niveau (surplus agricoles et

deacuteveloppement des villes) Apparition de nouveaux commerccedilants

Etat fort Evolutions technologiques (imprimerie bateaux performants instruments de

navigation) Extension geacuteographique de lrsquoeacuteconomie Apparition des corporations

drsquoartisans

Machine agrave vapeur chemin de fer passage de lrsquoartisanat au capitalisme

entrepreneurial producteur organisation des entreprises

Ecole classique (Taylor Fayol Weber) approche meacutecaniste bureaucratie

hieacuterarchie commandement fonctions et speacutecialisation laquo OS T raquo (organisation

scientifique du travail) organisation source de pouvoir rationaliteacute des individus bases

du management

Deacuteveloppement du capitalisme manageacuterial Electriciteacute peacutetrole puis communications

et information Consommation de masse mondialisation preacuteoccupations

eacutenergeacutetiques et environnementales 3 peacuteriodes

- standardisation grandes entreprises industrielles

- industries de consommation 30 glorieuses marketing multinationales protection

sociale

- deacutereacuteglementation monteacutee des services pays eacutemergents mondialisation et nouvelle

eacuteconomie (internet)

Ecole des relations humaines prise en compte de lrsquoindividu des motivations styles

de direction

Ecole neacuteo-classique et post-classique deacutecentralisation coordonneacutee DPO

management participatif zeacutero deacutefaut flux tendus

Approche systeacutemique partition de lrsquoentreprise eacutetude des interactions feacutedeacuteration

vers lrsquoobjectif controcircle et ajustement

Theacuteories de la deacutecision rationaliteacute limiteacutee contribution reacutetribution coalitions

Ecole socio-technique recherche de compromis technologie organisation

enrichissement des tacircches autonomie des groupes

Approche sociologique effets sociaux du travail jeux de pouvoir dans lrsquoentreprise

reacutegulation sociale

Theacuteories de la contingence facteurs contingents adaptation agrave lrsquoenvironnement

configurations organisationnelles

Theacuteories de la firme controcircle manageacuterial droits de proprieacuteteacute relation drsquoagence

Theacuteories contractualistes firme nœud de contrats coucircts de transaction

opportunisme externalisation internalisation

Approche eacutevolutioniste eacutecologie des organisations modegravele eacutevolutioniste

contraintes de sentier

Approche par les ressources valorisation des ressources compeacutetences cleacutes

apprentissage organisationnel

(Classement simplifieacute)

Initiation au management copy CRCF ndash J Sornet Page 14 48

ELEMENTS DE CORRIGE IM

IM1 Commenter la deacutefinition du management par la norme ISO et le manager de Mintzberg

Efficient = optimum avec les moyens disponibles

ISO (management objectifs) (manager moyens) HM

IM2 Le leader entraicircne naturellement derriegravere lui Le manager nrsquoest pas toujours leader

(mecircme si crsquoest souhaitable) Le leader nrsquoest pas toujours manager (plutocirct notion individuelle)

Leadership = faculteacute de diriger conjugaison drsquoune autoriteacute naturelle ou drsquoun savoir-faire

acquis drsquoune capaciteacute agrave entraicircner des personnes ou des groupes et drsquoune leacutegitimiteacute

statutaire (de position)

IM3 Compleacuteter le tableau ci-dessous en analysant chaque action preacutesenteacutee Faire ensuite

ressortir les domaines niveaux ou techniques de management pouvant ecirctre mobiliseacutes pour

chaque situation

Satisfaction client

Implication du personnel

Processus systegraveme

Ameacutelioration continue

Deacutecision efficace

Recherche de valeur

Image entreprise

Liaisons

Information

Reacutepartition ressources

Reacutegulation

Neacutegociation

Leadership

Initiation au management copy CRCF ndash J Sornet Page 15 48

Caracteacuteristiques

de lrsquoaction

- reacutepeacutetition

- risque

- normes

- ampleur

Prise de

deacutecision

- opeacuteration

- direction

- deacutelai

Informations

neacutecessaires

- nature

- origine

- deacutelai

obtention

Cleacute pour la

reacuteussite

Intervention

exteacuterieure

possible

Assurer la

restauration du

soir

(Restaurant

familial)

Technique

(fabrication)

Vente (terrain)

Appros

Reacutepeacutetitive

(quot)

Risque faible

Normes

drsquohygiegravene

Faible

Opeacuterationnelle

Geacuterant

responsable

Rapide (qq

jours menu et

appros)

Nombre de

couverts

Tarifs usuels

Calendrier

(fecirctes)

Clients docs

divers

expeacuterience

Qq jours

Varieacuteteacute menu

Plats phares

Accueil

Appros

Tarification

Vins

Gestion

congeacutelation

Qualiteacute cuisine

Fournisseurs

Extra

Publiciteacute

Construire un

viaduc

(autoroute)

Technique

Organisation

Appros

Uniteacute (ou peu)

Eleveacute (financier

technique)

Architecture

Eleveacutee

Direction

(aleacuteas)

Opeacuterationnelle

(conduite

chantier)

Immeacutediat agrave qq

semaines

Plans

plannings

Qualifications

Meacuteteacuteo

Disponibiliteacutes

Bureau eacutetudes

Qq sem agrave 24h

Techniciteacute

Appros

Qualifications

Preacutevision

GRH

Contrat juste

SS traitants

Organismes

certificateurs

Controcircle

client

Certifier les

comptes

annuels drsquoun

groupe national

(cabinet

drsquoaudit)

Technique

Relation client

Gestion des

connaissances

Annuelle

Moyen

Regravegles

comptables

fiscales

Moyenne (selon

importance du

cabinet)

Opeacuterationnelle

Qq jours agrave

semaines

Comptable

Juridique

Client

Etat

Qq jours agrave

semaines

Techniciteacute

Expeacuterience

Relation client

Systegraveme info client

Siegravege

Autre cabinet

Lancer une

ligne drsquoavions

(constructeur

aeacuteronautique)

Strateacutegique

RD

Etudes

Uniteacute

Tregraves eacuteleveacute

Aeacuteronautique

Tregraves eacuteleveacutee

Direction

Qq mois agrave

anneacutees

Marcheacute

Etudes

Compagnies

Qq mois agrave

anneacutees

Concept

Outil industriel

Coucirct exploitation

Tarif

Fiabiliteacute

Deacutelaisconcurrence

SI simulation

SS traitants

Bureaux

drsquoeacutetudes

speacutecialiseacutes

Compagnies

Conseils

Reacuteduire la

capaciteacute de

production

(groupe

industriel)

Strateacutegique

RH

Communication

Production

Uniteacute

Moyen

Leacutegislation

(dont RH)

Eleveacutee

Direction

Qq mois agrave

anneacutees

Financiegravere

Industrielle

Marcheacute

Organisation

Organismes

speacutecialiseacutes

DRH

Qq mois

Communication

Connaissance des

compeacutetences

Connaissance outil

industriel

Concurrence

Portefeuille

drsquoactiviteacutes

Cabinet

drsquoorganisation

Conseils

speacutecifiques

Acqueacuterir une

entreprise

concurrente

(teacuteleacutephonie

mobile)

Strateacutegique

Marketing

Production

(reacuteseau)

Financier

Communication

Uniteacute

Tregraves eacuteleveacute

Leacutegislation

telecom

Tregraves eacuteleveacutee

Direction

Qq mois

Financiegravere

Marcheacute

Reacuteseaux

(ampleur

recouvrement

hellip)

Organisations

Interne

Racheteacutee

Sources

speacutecialiseacutees

Qq mois

Communication

Marcheacute

Cours boursiers

Cabinet

drsquoorganisation

Conseils

speacutecifiques

Initiation au management copy CRCF ndash J Sornet Page 16 48

LE MANAGEMENT EN PRATIQUE

Pour assumer sa fonction le management doit couvrir sans discontinuiteacute lrsquoensemble de

lrsquoorganisation et inteacutegrer de nombreux facteurs dont nous allons reacutesumer lrsquoessentiel

1 ndash Les fonctions et activiteacutes du management

Pour Henri Fayol la fonction drsquoadministration de lrsquoentreprise (son management) reposait sur

cinq actions preacutevoir organiser commander coordonner et controcircler (laquo PO3C raquo)

Nous distinguerons cinq activiteacutes de management

- la conception (au plus haut niveau finaliteacute but ou vocation de lrsquoorganisation

meacutetiers dimension politique de croissance hellip)

- la planification (deacutefinition des objectifs eacutecheacuteances)

- lrsquoorganisation (reacutepartition du travail choix des modes de coordination)

- le pilotage de lrsquoaction opeacuterationnelle (motivation animation encadrement

assistance)

- lrsquoeacutevaluation (controcircle des reacutesultats obtenus ajustements)

Dans chacune de ces activiteacutes des deacutecisions et des arbitrages sont neacutecessaires avec des

enjeux plus ou moins importants

Remarques

- Les cinq activiteacutes du management peuvent se retrouver agrave tout niveau de

management si lrsquoentreprise laisse une certaine autonomie de deacutecision agrave ses diffeacuterentes

uniteacutes La conception est naturellement du ressort de la direction geacuteneacuterale et des

conseils drsquoadministration mais elle peut ecirctre preacutesente pregraves du terrain (latitude laisseacutee agrave

une filiale ou agrave un magasin par exemple) De mecircme lrsquoorganisation du travail concerne

un atelier mais aussi la direction qui structure lrsquoentreprise pour assurer ses activiteacutes sa

production

- La planification deacutefinit des objectifs ou des axes strateacutegiques (choix de produits

modaliteacutes de deacuteveloppement des ventes implantations alliances hellip) et les traduit en

donneacutees de gestion preacutevisionnelles syntheacutetiques et eacutechelonneacutees dans le temps afin de

valider les objectifs et de fixer des repegraveres

- Un laquo business plan raquo (plan drsquoaffaires)est notamment lrsquoeacutequivalent de la planification

dans le cas de creacuteation drsquoentreprise ou pour la preacutesentation de tout projet drsquoactiviteacute

Les activiteacutes du management srsquoinscrivent dans des cycles qui peuvent ecirctre scheacutematiseacute

comme suit (lrsquoeacutevaluation peut entraicircner une reacutevision du pilotage de lrsquoorganisation ou des

objectifs sans que lrsquoentreprise ne soit fondamentalement remise en cause)

conception

planification

organisation

pilotage

eacutevaluation

Initiation au management copy CRCF ndash J Sornet Page 17 48

2 ndash Les contextes de management

Le management est influenceacute par son contexte qui justifie des objectifs une organisation

des meacutethodes

Par exemple lrsquoentreprise admet de nombreuses variantes selon sa taille sa forme juridique

son controcircle par lrsquoeacutetat (entreprises publiques) ou par des inteacuterecircts priveacutes Il en va de mecircme des

organismes administratifs qui peuvent deacutependre de directives nationales ou reacutegionales des

associations qui ont des activiteacutes drsquoampleur tregraves variable

21 ndash La dimension de lrsquoentreprise

La dimension drsquoune entreprise se mesure principalement en fonction de son effectif ou de

son chiffre drsquoaffaires Des seuils sont deacutefinis par divers organismes et exploiteacutes agrave des fins

statistiques ou pour la deacutetermination de certaines obligations sociales ou fiscales

(repreacutesentation du personnel cotisations hellip) Il nrsquoy a bien entendu pas de laquo barriegravere de

tailleraquo absolue conditionnant le management drsquoune entreprise

LrsquoUE preacuteconise de distinguer les micro ndash entreprises (jusqursquoagrave 9 salarieacutes) les TPE ndash tregraves petites

entreprises (moins de 20 salarieacutes) les petites entreprises (moins de 50) et les moyennes

entreprises (de 50 agrave 250) Cependant les PME sont parfois situeacutees entre 10 et 500 salarieacutes

Remarques

- en France environ 40 des entreprises emploient de 1 agrave 50 salarieacutes (ce qui repreacutesente

plus de 50 des emplois) et 59 nrsquoen ont aucun

le pays compte environ 2 600 000 entreprises dont moins de 1 ont 250 employeacutes et

plus

- ancienneteacute et taille de lrsquoentreprise sont lieacutees si lrsquoon eacutecarte les restructurations et autres

eacutevolutions drsquoentreprises existantes

La dimension de lrsquoentreprise a une influence sur lrsquoorganisation et le laquo style raquo de son

management

- les PME sont souvent entrepreneuriales (les dirigeants eacutegalement apporteurs de capitaux

sont totalement engageacutes dans la marche de lrsquoentreprise) Elles ont une gestion flexible peu

formaliseacutee plus qualitative que quantitative Les PME sont freacutequemment focaliseacutees sur un seul

type drsquoactiviteacute Pour ne pas alourdir leur structure elles ont tendance agrave sous-traiter les

activiteacutes speacutecialiseacutees ne correspondant pas agrave leur meacutetier de base

- les grandes entreprises sont manageacuteriales (les dirigeants sont nommeacutes par les actionnaires

en raison de leurs compeacutetences) et moins reacuteactives

22 ndash Le type de production

On distingue industrie (production de biens mateacuteriels ou pour le moins de produits visibles ndash

comme un seacutejour touristique ou un film) et services (fourniture drsquoune prestation immateacuterielle)

Le type de production influence en principe le management de lrsquoentreprise

- lrsquoindustrie neacutecessite (si lrsquoon excepte lrsquoartisanat) un investissement relativement important

une organisation productive stable capable de reacutealiser plusieurs fois des produits identiques

(exemple un modegravele de reacutefrigeacuterateur) ou du moins similaires (exemple un bacirctiment) Le

produit de lrsquoindustrie consomme des matiegraveres et il doit geacuteneacuteralement ecirctre distribueacute jusqursquoau

client

- la production de services peut se satisfaire drsquoun investissement tregraves reacuteduit et neacutecessite un

contact permanent avec le client

Toutefois la standardisation des services et le deacuteveloppement des reacuteseaux informatiques

rapprochent la production de services de celle des biens industriels

- la production drsquoun service reacutepeacutetitif et technique peut imposer une structure lourde et une

organisation tregraves formaliseacutee (voir les grandes socieacuteteacutes drsquoaudit ou de conseil informatique)

Initiation au management copy CRCF ndash J Sornet Page 18 48

- certains services peuvent ecirctre fournis agrave distance sans contact direct avec le client et

distribueacutes par reacuteseau (tenue de comptabiliteacute affacturage gestion clientegravele centre drsquoappel

hellip)

Remarque les services repreacutesentent 75 de lrsquoactiviteacute eacuteconomique franccedilaise

23 ndash La nature de lrsquoorganisation

Les organisations publiques franccedilaises (administrations centrales collectiviteacutes territoriales

hocircpitaux hellip) repreacutesentent une part importante de lrsquoactiviteacute (environ 30 des emplois) La

fonction publique regroupe des organisations aux finaliteacutes diverses et qui ont des problegravemes

de gestion similaires agrave ceux des entreprises auxquelles elles peuvent emprunter des principes

de management Notamment

- pour controcircler les coucircts et assurer la qualiteacute des services

- pour communiquer avec les administreacutes ou les usagers

- pour motiver les personnels et geacuterer les ressources humaines

La transposition directe des techniques de gestion et de management nrsquoest cependant pas

toujours possible car

- la comptabiliteacute publique obeacuteit agrave des regravegles speacutecifiques (proceacutedure budgeacutetaire

notamment)

- le laquo client raquo ne paye pas toujours la prestation du moins directement

- la concurrence est parfois inexistante

- les grandes administrations centraliseacutees sont soumises agrave des choix politiques geacuteneacuteraux

parfois sans connexion eacutevidente avec les besoins opeacuterationnels

- le statut des personnels et les grilles de salaires limitent les possibiliteacutes de gestion des

ressources humaines

Remarque la LOLF (loi organique relative aux lois de finances) est entreacutee en vigueur en

2006 Elle alloue des moyens budgeacutetaires en fonction de programmes et remplace la

reconduction automatique de 90 des budgets Cette reacuteforme se heurte toutefois agrave la

lourdeur des grands ministegraveres ougrave la complexiteacute des activiteacutes est difficile agrave

appreacutehender et ougrave des inerties culturelles peuvent exister agrave tout niveau

Les associations loi de 1901 peuvent avoir une activiteacute comparable agrave celle de grandes

entreprises (voir par exemple les associations de santeacute ou professionnelles) et leur

management est alors similaire malgreacute lrsquoabsence de but lucratif (les beacuteneacutefices ne sont pas

distribuables) Elles ont drsquoailleurs en France un poids eacuteconomique important (elles emploient

environ 1 600 000 salarieacutes)

Cependant lrsquoadheacutesion agrave un systegraveme de valeurs fondateur de lrsquoassociation ou la limite de

lrsquoautoriteacute (quand un volant de beacuteneacutevoles important participe agrave lrsquoactiviteacute) peut introduire des

nuances

- le renforcement des objectifs socieacutetaux

- la faiblesse des relations hieacuterarchiques

- des contraintes de gestion du temps des beacuteneacutevoles

- des modaliteacutes particuliegraveres de recrutement et de motivation des dirigeants

24 ndash Les facteurs contingents

La theacuteorie de la contingence montre qursquoune structure drsquoentreprise nrsquoest efficace que dans

une situation deacutetermineacutee et qursquoil nrsquoexiste que des solutions de management construites dans

un contexte preacutecis

Le management doit ainsi srsquoadapter agrave des facteurs contingents qui ne peuvent ecirctre

controcircleacutes du moins agrave bregraveve eacutecheacuteance Ces facteurs sont par exemple

- lrsquoancienneteacute de lrsquoentreprise (plus elle est ancienne plus lrsquoentreprise a tendance agrave reacutepeacuteter

des comportements eacuteprouveacutes)

Initiation au management copy CRCF ndash J Sornet Page 19 48

- la taille de lrsquoentreprise (la grande entreprise a une composante administrative plus

deacuteveloppeacutee)

- le systegraveme de production (tregraves standardiseacute complexe automatiseacute hellip)

- lrsquoenvironnement

3 ndash Le management et les parties prenantes

Lrsquoentreprise a pour vocation premiegravere de mettre des produits agrave disposition de ses clients en

reacutealisant un profit Pour y arriver elle doit aussi satisfaire ses parties prenantes salarieacutes

actionnaires fournisseurs hellip

Est partie prenante agrave lrsquoentreprise laquo tout groupe ou individu qui peut ecirctre affecteacute ou est

affecteacute par les buts de lrsquoorganisation hellip raquo (Freeman ndash 1984)

Les parties prenantes attendent agrave des degreacutes divers de profiter drsquoune creacuteation de valeur en

provenance de lrsquoentreprise qui doit reacutepondre agrave ces attentes pour assurer sa peacuterenniteacute ou

favoriser son deacuteveloppement

On distingue les parties prenantes primaires ou principales qui sont essentielles agrave lrsquoentreprise

et qui ont geacuteneacuteralement une relation formelle avec elle (clients associeacutes et actionnaires

precircteurs salarieacutes fournisseurs collectiviteacutes) et les parties prenantes secondaires dont

lrsquoinfluence est diffuse (groupes de pression associations meacutedias instances europeacuteennes

agences de notation hellip)

Remarque la consideacuteration de lrsquoensemble des parties prenantes (laquo stakeholders raquo - les

deacutepositaires) fait contrepoids agrave lrsquoimportance accordeacutee aux seuls actionnaires

(laquo shareholders raquo)

Les organisations nrsquoayant pas drsquoobjectif de profit doivent aussi satisfaire leurs parties

prenantes apporter un service aux usagers dans les meilleures conditions eacuteconomiques

limiter un budget assurer la qualiteacute des relations avec les fournisseurs hellip

Dans cette optique le management doit organiser lrsquoaction de faccedilon agrave eacutequilibrer des forces

parfois divergentes

- le contexte fait pression sur lrsquoorganisation contrainte agrave optimiser ses reacutesultats

- lrsquoorganisation cherche par son action agrave assurer sa peacuterenniteacute son deacuteveloppement (en

reacutealisant des profits dans le cas de lrsquoentreprise) et agrave satisfaire ses parties prenantes

- le management agit en pilotant les actions pour contrebalancer la pression du contexte

Actions de

lrsquoorganisation

Management Contexte

Parties

prenantes

Initiation au management copy CRCF ndash J Sornet Page 20 48

APPLICATIONS MP

MP1 Deacutefinir contingent gestion budgeacutetaire

MP2 Deacuteterminer les parties prenantes drsquoun hocircpital public et leurs principales attentes

Mecircme question pour les organisations suivantes

- SNCF (entreprise publique)

- Peugeot

- MAIF (mutuelle drsquoassurance)

MP3 En les situant dans le cycle des activiteacutes du management trouver les actions agrave mener

dans les situations suivantes

- baisse de 10 des ventes dans une entreprise industrielle (produits meacutenagers le reacuteseau de

distribution vient drsquoecirctre reacuteorganiseacute)

- idem dans une entreprise de vente par correspondance soumise agrave la concurrence internet

(les ventes stagnaient depuis six mois malgreacute les efforts promotionnels)

- augmentation des deacutelais drsquoattente des consultations dans une clinique (lrsquohocircpital voisin a

fermeacute son service drsquourgences)

Initiation au management copy CRCF ndash J Sornet Page 21 48

ELEMENTS DE CORRIGE MP

MP1 Deacutefinir (dans le contexte drsquoune entreprise) contingent gestion budgeacutetaire

Contingent = imposeacute par lrsquoexteacuterieur Contingence = effet du hasard de la rencontre de

plusieurs eacuteveacutenements indeacutependants (variables explicatives que lrsquoon ne peut influencer)

Gestion budgeacutetaire = technique drsquoadministration des entreprises srsquoappuyant sur des

preacutevisions dont on deacuteduit apregraves accord des responsables des attributions de moyens sur une

dureacutee limiteacutee Une analyse reacuteguliegravere des eacutecarts entre preacutevisions et reacutealisations permet ensuite

le pilotage des activiteacutes Le budget est un cadre incitatif

La laquo planification budgeacutetaire raquo consiste agrave traduire en budgets une planification strateacutegique

avec systegraveme de reporting

MP2 Deacuteterminer les parties prenantes drsquoun hocircpital public et leurs principales attentes

Mecircme question pour les organisations suivantes

- SNCF (entreprise publique)

- Peugeot

- MAIF (mutuelle drsquoassurance)

Hocircpital

- patients (qualiteacute des soins)

- CNAM (baisse des coucircts)

- collectiviteacute locale (service aux administreacutes)

- eacutetat (ameacutenagement du territoire maicirctrise des budgets optimisation)

- employeacutes (salaire conditions de travail et satisfaction)

- fournisseurs ndash pharmacie autres (CA paiement reacutegulier)

- associations de patients (qualiteacute proximiteacute des soins)

SNCF

- usagers et associations drsquousagers (proximiteacute reacutegulariteacute prix du service)

- reacuteseau ferreacute de France (optimisation des lignes paiement adapteacute)

- fournisseurs (CA paiement reacutegulier)

- employeacutes (salaire conditions de travail seacutecuriteacute de lrsquoemploi)

- eacutetat (ameacutenagement du territoire)

- collectiviteacutes locales (service)

Peugeot

- clients (qualiteacute prix SAV relation commerciale)

- fournisseurs (CA reacutegulariteacute de lrsquoactiviteacute)

- employeacutes (salaire conditions de travail seacutecuriteacute de lrsquoemploi)

- eacutetat (taxes)

- collectiviteacute locale (emploi dynamisation eacuteconomique preacuteservation de lrsquoenvironnement)

- associations de protection de lrsquoenvironnement (activiteacute propre baisse des eacutemissions

nouvelles eacutenergies)

MAIF

- socieacutetaires (protection relation assureur tarif mesureacute)

- professionnels de lrsquoautomobile et autres (agreacutement marge de manœuvre reacuteparations tarifs

eacuteleveacutes)

- fournisseurs (CA paiement reacutegulier)

- eacutetat (taxes engagement pour la seacutecuriteacute)

- employeacutes (salaire conditions de travail seacutecuriteacute de lrsquoemploi)

Initiation au management copy CRCF ndash J Sornet Page 22 48

MP3 En les situant dans le cycle des activiteacutes du management trouver les actions agrave mener

dans les situations suivantes

- baisse de 10 des ventes dans une entreprise industrielle (produits meacutenagers le reacuteseau de

distribution vient drsquoecirctre reacuteorganiseacute)

Adapter le pilotage motiver cadrer si insuffisant retoucher une organisation deacutefectueuse

- idem dans une entreprise de vente par correspondance soumise agrave la concurrence internet

(les ventes stagnaient depuis six mois malgreacute les efforts promotionnels)

Voir pilotage et organisation si une eacutevolution du meacutetier a deacutejagrave eacuteteacute initialiseacutee Sinon re-

conception (adaptation au nouveau contexte) puis planification et reacuteorganisation

- augmentation des deacutelais drsquoattente des consultations dans une clinique (lrsquohocircpital voisin a

fermeacute son service drsquourgences)

Organisation Si insuffisant planification (nouveaux objectifs)

Initiation au management copy CRCF ndash J Sornet Page 23 48

ORGANISATION ET PROCESSUS

La performance de lrsquoentreprise deacutepend de son organisation et de son aptitude agrave produire

aux meilleures conditions Nous allons montrer comment organisation formelle et processus

de production peuvent contribuer agrave cette performance

1 ndash Vers lrsquooptimum

11 ndash Les eacuteconomies occidentales jusqursquoaux anneacutees 70

Jusqursquoen 1945 le principal problegraveme des entreprises eacutetait de produire des biens en quantiteacute

suffisante agrave un prix compatible avec le marcheacute Les grandes entreprises se sont multiplieacutees et

la standardisation a permis de reacuteduire les coucircts (exemple deacuteveloppement de Ford et de la

production agrave la chaicircne de 1908 agrave 1920 qui a permis une baisse du prix des voitures des 23)

On parle de laquo production pousseacutee vers le marcheacute raquo

Cette croissance de la production peu reacuteguleacutee a eacuteteacute marqueacutee par des surproductions en

1910 et 1920 puis par la crise de 1929 qui a prolongeacute ses effets jusqursquoagrave la guerre

De 1945 agrave 1975 environ (les laquo trente glorieuses raquo) la reconstruction la croissance de la

consommation de masse de nouvelles technologies et les eacutechanges internationaux

alimentent lrsquoeacuteconomie La standardisation srsquoeacutetend aux biens de consommation dont les

coucircts baissent fortement et de nouvelles reacutegulations sociales permettent une eacutevolution sans

heurt des revenus La saturation de certains marcheacutes conduit dans les anneacutees 60 agrave la

deacutemarche laquo marketing raquo et agrave la diffeacuterenciation des produits Le produit est laquo dirigeacute par le

marcheacute raquo mais les entreprises conservent une organisation assez classique et les plus grosses

srsquointernationalisent

12 ndash Lrsquoexpeacuterience japonaise et ses prolongements

Tregraves tocirct apregraves la guerre dans un Japon appauvri le constructeur automobile Toyota a ducirc

faire face agrave une restriction du marcheacute des moyens financiers et productifs et des

approvisionnements La firme a donc innoveacute dans un nouveau systegraveme de production

chassant les laquo gaspillages raquo (temps drsquoattente transports stocks deacutefauts hellip) consideacuterant que

seule la fabrication vendable creacutee de la valeur

Toyota srsquoorganise pour fabriquer la quantiteacute et la qualiteacute de produits juste neacutecessaires agrave la

satisfaction des clients la production est laquo tireacutee par le marcheacute raquo La mise en place de ce

systegraveme qui integravegre les fournisseurs ne sera acheveacutee que dans le milieu des anneacutees 70

En 1973 la hausse du peacutetrole inaugure un ralentissement de la croissance des eacuteconomies

occidentales La concurrence accrue provoque alors un inteacuterecirct pour le systegraveme deacuteveloppeacute

au Japon La production au plus juste se deacuteveloppe ainsi dans lrsquoindustrie automobile agrave partir

des anneacutees 80 et elle se reacutepand encore maintenant dans drsquoautres secteurs

Cette approche qui vise un objectif de zeacutero stock et zeacutero deacutefaut impose la maicirctrise de laquo bout

en bout raquo des processus de production et leur ameacutelioration

Initiation au management copy CRCF ndash J Sornet Page 24 48

2 ndash Organiser lrsquoentreprise

21 ndash Direction et organisation

Diriger une entreprise neacutecessite de lrsquoorganiser (de reacutepartir les tacircches) pour qursquoelle puisse

atteindre ses objectifs Lrsquoorganisation permet de satisfaire un marcheacute en tirant parti des

capaciteacutes actuelles de lrsquoentreprise tout en preacuteparant lrsquoavenir

Lrsquoorganisation reacutesulte freacutequemment drsquoun compromis entre des objectifs situeacutes agrave des niveaux

et des eacutecheacuteances diffeacuterents

Exemples

- le leader des chaises roulantes peut tirer profit de sa structure productive et de son

savoir faire pour entrer sur le marcheacute de la bicyclette eacutelectrique

- ecirctre parfaitement structureacute pour alimenter 90 du marcheacute des disquettes ne preacutepare

pas lrsquoavenir

- srsquoorganiser pour conqueacuterir le marcheacute des tire-bouchons eacutelectriques dans les deux ans

perd de son sens si cela altegravere les moyens neacutecessaires agrave la production drsquoappareils

manuels ancienne mais vitale dont la diminution agrave court terme risque de nuire agrave la

solvabiliteacute de lrsquoentreprise et de la conduire agrave la cessation de paiement

22 ndash Lrsquoorganisation fonctionnelle

La majoriteacute des entreprises adopte une laquo organisation fonctionnelle raquo (celle qui est visible

dans les organigrammes) ougrave des regroupements de personnels et drsquoeacutequipements se font

selon un modegravele hieacuterarchique (laquo line raquo) dans des uniteacutes des services ou des deacutepartements

speacutecialiseacutes Cette organisation peut se deacutecliner agrave lrsquointeacuterieur des divisions des grandes

entreprises quand elles scindent leur activiteacute par zone geacuteographique type drsquoactiviteacute

cateacutegorie de clients hellip

Remarque le terme laquo fonction raquo deacutesigne un rocircle particulier dans le fonctionnement de

lrsquoentreprise

Lrsquoorganisation fonctionnelle diffeacuterencie les activiteacutes de lrsquoentreprise en les regroupant par

meacutetier pour utiliser au mieux les compeacutetences et les moyens (meilleur rendement par la

speacutecialisation lrsquoeacutechange de compeacutetences dans une mecircme uniteacute ou gracircce agrave des eacuteconomies

drsquoeacutechelle)

23 ndash La notion de processus de production

Un processus de production se deacutefinit par la succession drsquoactiviteacutes permettant de satisfaire

un client en transformant des ressources (mateacuterielles financiegraveres humaines) en un produit

bien ou service Le processus doit creacuteer une valeur reconnue par le client

Un processus peut servir un client interne agrave lrsquoentreprise (par exemple en produisant un

composant intervenant dans plusieurs produits ou par la maintenance des machines) aussi

bien qursquoun client final On distingue usuellement

- les processus opeacuterationnels (ou maicirctres) aussi appeleacutes processus meacutetier (business process)

qui satisfont directement les clients finaux (conception et fabrication de produits vente hellip)

- les processus de support et de management (geacuterer les ressources humaines geacuterer

lrsquoinformation geacuterer les ressources financiegraveres hellip) qui ont les processus opeacuterationnels comme

clients

Toutes les actions internes agrave une organisation peuvent srsquointeacutegrer dans des processus qui

conditionnent directement ou indirectement la capaciteacute de lrsquoorganisation agrave satisfaire le

client final ou lrsquousager

Initiation au management copy CRCF ndash J Sornet Page 25 48

Aborder le fonctionnement de lrsquoentreprise par ses processus (approche processus) permet

de mettre en eacutevidence les chaicircnes drsquoactiviteacutes qui conduisent aux produits leurs

dysfonctionnements leurs coucircts la formation des deacutelais et la souplesse (la flexibiliteacute)

disponible pour satisfaire la clientegravele finale Lrsquoameacutelioration des processus a un impact visible

et direct sur chaque produit proposeacute aux clients

Lrsquoapproche processus provoque une eacutevolution de la faccedilon de travailler

- en faisant peacuteneacutetrer la laquo voix du client raquo au plus profond de lrsquoentreprise (et plus seulement

dans les services commerciaux et marketing)

- en mettant en eacutevidence des possibiliteacutes de rationalisation (par regroupement ou impartition

de certaines activiteacutes)

Remarque lrsquoapproche par les activiteacutes et les processus est agrave lrsquoorigine de la meacutethode

de deacutetermination des coucircts laquo ABC raquo - activity based costing

24 ndash Processus et fonctions

Le processus est transversal Il enchaicircne des activiteacutes qui traversent lrsquoentreprise en particulier

les services ou les deacutepartements drsquoune organisation fonctionnelle

Exemple

La division du travail par fonctions induit une charge de coordination pour assurer le

deacuteroulement du processus Elle peut geacuteneacuterer des attentes des erreurs ou des conflits drsquointeacuterecirct

(lrsquoobservation montre que des dysfonctionnements sont tregraves souvent constateacutes lors du

passage drsquoun service agrave un autre)

Organisation fonctionnelle et approche processus visent toutes deux un optimum

eacuteconomique mais leurs logiques sont diffeacuterentes

- le processus vise la satisfaction des clients (prix qualiteacute deacutelais service)

- le deacutecoupage fonctionnel cherche agrave optimiser les moyens (maximiser lrsquoeffet drsquoexpeacuterience

partager des infrastructures profiter de pocircles de compeacutetences hellip) Il apporte une ossature

hieacuterarchique stable souvent indispensable

Organisation fonctionnelle et approche processus sont donc compleacutementaires dans la

majoriteacute des cas et doivent ecirctre combineacutees judicieusement

APPLICATIONS OP

OP1 Deacutefinir flexibiliteacute systegraveme impartition

OP2 Citer huit exemples drsquoinformations essentielles pour optimiser un processus de

fabrication

Direction

Deacutepartement

commercial

(C)

Deacutepartement

administratif et

financier (AF)

Deacutepartement

Etudes (E)

Deacutepartement

Production (P)

Activiteacute

C-x Activiteacute

AF-x Activiteacute

E-x

Activiteacute

P-x

Processus x

Clie

nt

Initiation au management copy CRCF ndash J Sornet Page 26 48

OP3 Deacutegager les principes du toyotisme preacutesenteacute ci-dessous En quoi ce systegraveme est-il

initiateur de lrsquoapproche processus

Taiichi Ohno et le Toyotisme

1 - Extrait drsquoun article de Jacques BARRAUX - 1993 - LExpansion

Taiichi Ohno (1912 ndash 1990) hellip ne se prenait pas pour un visionnaire mais en imposant une

nouvelle faccedilon de produire il a reacuteinventeacute le management hellip tout le monde a entendu parler

des mots qui ont populariseacute le toyotisme dont il est le pegravere le juste-agrave-temps hellip Autant

doutils conccedilus pour lrsquoautomobile et qui ont aujourdhui une application universelle

hellip Taiichi Ohno jeune ingeacutenieur entre chez Toyota alors simple constructeur de machines

textiles Degraves 1926 apparaicirct la notion de jidoka hellip cest lart de transfeacuterer de lintelligence aux

machines pour mieux libeacuterer lintelligence des hommes Tout le contraire du taylorisme qui

juge la machine moins impreacutevisible que lhomme En 1933 Toyota se lance dans lautomobile

en sinspirant des meacutethodes ameacutericaines Mais en 1935 agrave loccasion dun voyage aux Etats-

Unis leacutetat-major de lentreprise revient fascineacute de sa visite dans un supermarcheacute La notion

de juste-agrave-temps va naicirctre de lobservation dune grande surface un lieu ougrave les clients ne

prennent que ce dont ils ont besoin et ougrave les rayons sont reacuteapprovisionneacutes pour compenser

les quantiteacutes preacuteleveacutees Ainsi le systegraveme Toyota est-il deacutejagrave dans la tecircte de ses dirigeants avant

mecircme la Seconde Guerre mondiale un demi-siegravecle avant la reacutevolution informatique et la

segmentation intensive des marcheacutes

hellip des esprits curieux comme Franccedilois Dalle en France tombent alors sous le charme des

formules et des paraboles de Taiichi Ohno En voici deux eacutechantillons

Penser agrave lenvers Cela signifie combattre les ideacutees reccedilues En lespegravece il sagit du fordisme et

du taylorisme Ohno ne croit pas agrave la planification aux effets deacutechelle et dexpeacuterience Il

propose un systegraveme industriel agrave lenvers qui permette de diversifier les produits et de les

fabriquer en petites quantiteacutes Nous ne devons plus ecirctre des paysans qui accumulent des

stocks mais des chasseurs On nimpose pas loffre On traque la demande et on la gegravere en

continu

Que les valleacutees soient hautes et les montagnes peu eacuteleveacutees Plutocirct que de concentrer tous

les efforts sur une production agrave un moment donneacute mieux vaut se doter de structures flexibles

permettant de passer agrave tout instant dune seacuterie agrave une autre Il faut eacuteviter les ruptures et les

secousses aplanir les cycles entretenir des flux reacuteguliers dactiviteacutes diversifieacutees Ce qui

implique de ne pas enfermer les hommes et les eacutequipements dans des speacutecialisations trop

eacutetroites

La flexibiliteacute le travail en groupe le refus de la dictature des machines la polyvalence et

surtout lattention constante aux signaux eacutemis par le marcheacute nappartiennent plus au

toyotisme Ces notions sont les fondements du nouvel art dorganiser de vendre et de

produire dans lindustrie comme dans les services hellip

2 - Quelques notions cleacutes

Taiichi Ohno a imagineacute la meacutethode des laquo cinq pourquoi raquo qui consiste agrave se poser cinq fois de

suite la question laquo pourquoi raquo sur le mecircme sujet de faccedilon agrave deacutecouvrir la veacuteritable cause

drsquoun problegraveme Cette meacutethode peut ecirctre appliqueacutee agrave tous les niveaux et permettre

notamment aux agents de fabrication de proposer de veacuteritables ameacuteliorations de la

production

La recherche de la qualiteacute totale (pas de deacutefaut des produits pas de rebuts pas de deacutefaut

des processus) accompagne la deacutemarche de Toyota La qualiteacute a un coucirct compenseacute par

des ventes accrues par lrsquoeacuteconomie des mesures palliatives aux deacutefauts

Initiation au management copy CRCF ndash J Sornet Page 27 48

Fiche OP1 ndash Benchmarking et processus

Le laquo benchmarking raquo consiste agrave comparer le fonctionnement de plusieurs systegravemes pour en

faire notamment ressortir les meilleures pratiques (laquo best practices raquo) Cette technique est

utiliseacutee depuis les anneacutees 80 pour ameacuteliorer la performance des entreprises Elle impose agrave

lrsquoentreprise drsquoeacutevaluer et de remettre en question ses propres modes de fonctionnement afin

de les faire eacutevoluer agrave la lueur de ce qui se fait ailleurs

Le benchmarking permet drsquoameacuteliorer les processus agrave moindre risque en fixant des objectifs

baseacutes sur des faits et donc plus facilement accepteacutes

Une classification des processus en tant que base de reacuteflexion a eacuteteacute eacutetablie aux USA par

lrsquolaquo International Benchmarking Clearinghouse raquo de lrsquoAPQC (american productivity and

quality center) en collaboration avec plusieurs dizaines drsquoentreprises

Elle se reacutesume ainsi

Le terme laquo reengineering raquo (la re-conception ou laquo reacuteingeacutenieacuterie raquo) des processus deacutesigne un

projet drsquoameacutelioration radicale des performances (de 20 agrave 50 ou plus) Il neacutecessite une

parfaite adheacutesion de la direction la constitution drsquoune petite eacutequipe de projet brillante

connaissant parfaitement les activiteacutes de lrsquoentreprise et il peut inclure un benchmarking

Le reengineering provoque geacuteneacuteralement la reacuteduction du nombre de niveaux hieacuterarchiques

(laquo delayering raquo) et lrsquoaccroissement du pouvoir de deacutecision des employeacutes (laquo empowerment raquo

ou laquo empouvoirement raquo) Bien qursquoy conduisant parfois il ne doit pas ecirctre confondu avec la

reacuteduction des activiteacutes (laquo downsizing raquo ou restructuration) et lrsquoexternalisation (laquo outsourcing raquo)

Pro

ce

ssu

s

op

eacutera

tio

nn

els

Pro

ce

ssu

s d

e m

an

ag

em

en

t e

t d

e

sup

po

rt

1 ndash

Comprendre

le marcheacute et

les clients (besoins

satisfaction)

2 ndash

Deacutevelopper

vision et

strateacutegie (contexte

concurrence)

3 ndash

Creacuteer

produits

services

processus

(concevoir

ameacuteliorer)

4 ndash

Marketing et

vente

5 ndash

Produire et

livrer (industrie

dont

ameacutelioration

processus)

6 ndash

Produire et

livrer (services)

7 ndash

Facturer et

servir les

clients (apregraves-

vente

reacuteclamations)

8 ndash Deacutevelopper et geacuterer les ressources humaines

9 ndash Geacuterer les systegravemes drsquoinformation

10 ndash Geacuterer les ressources financiegraveres et les actifs

11 ndash Appliquer un programme environnemental

12 ndash Geacuterer les relations exteacuterieures (actionnaires banques lois relations publiques hellip)

13 ndash Geacuterer lrsquoameacutelioration et le changement (eacutevaluer mesurer motiver qualiteacute totale)

Initiation au management copy CRCF ndash J Sornet Page 28 48

Fiche OP2 ndash Lrsquoorganisation par processus

Lrsquoeacutevolution drsquoune organisation aux activiteacutes reacutepeacutetitives vers lrsquoapproche processus est

geacuteneacuteralement progressive et se met en place par paliers

La mise en œuvre drsquoun veacuteritable management par processus doit ecirctre preacuteceacutedeacutee quand

lrsquoactiviteacute de lrsquoentreprise est complexe drsquoun recensement (une laquo cartographie des

processus raquo) pour mettre en eacutevidence les processus ou les familles de processus cleacutes critiques

pour le succegraves de lrsquoentreprise ougrave les efforts seront prioritaires

Des responsables de processus (laquo process owners raquo) sont ensuite deacutesigneacutes

Le responsable doit concevoir ses processus puis apregraves leur mise en œuvre assurer les

coordinations neacutecessaires les ameacuteliorer et les repreacutesenter aupregraves de la direction

Quand une structure par processus est mise en place des opeacuterateurs exeacutecutants

preacuteceacutedemment regroupeacutes dans les fonctions peuvent ecirctre affecteacutes aux processus et

drsquoanciens responsables de fonctions peuvent devenir des experts au service des processus

Lrsquoorganisation par processus peut imposer un degreacute eacuteleveacute drsquointeacutegration des activiteacutes donc

une polyvalence accrue des personnels et une reacuteduction des niveaux hieacuterarchiques

Elle neacutecessite pour le moins des compeacutetences eacutelargies au niveau des responsables de

processus (organisation administration technique hellip) dont le nombre doit rester limiteacute

(quelques dizaines au plus)

Sauf dans de tregraves petites structures lrsquoorganisation par processus se plaque geacuteneacuteralement sur

une structure plus classique

Initiation au management copy CRCF ndash J Sornet Page 29 48

ELEMENTS DE CORRIGE OP

OP1 Deacutefinir

Flexibiliteacute = adaptation au besoin (horaire variable chaicircnes robotiseacutees)

Systegraveme = ensemble organiseacute dans un but boicircte noire (sanguin nerveux meacutetrique laquo D raquo)

Impartition = sous-traitance ou externalisation (seacuteparation) drsquoactiviteacutes faire appel agrave des

partenaires plutocirct que faire soi-mecircme

OP2 Citer huit exemples drsquoinformations essentielles pour orienter lrsquooptimisation drsquoun processus

Montant des stocks (approvisionnements et produits finis)

Temps drsquoattente

Taux drsquoactiviteacute des ateliers

Rebuts

Deacutelai de production

Taux de reacuteclamations clients (qualiteacute)

Temps passeacutes en retouches finales

Turn over

Nombre drsquoarrecircts maladie

Accidents du travail

Dureacutee des arrecircts machines

OP3 Deacutegager les principes du toyotisme preacutesenteacute dans la fiche 31 En quoi ce systegraveme

repose trsquoil sur lrsquoapproche processus

Produire la quantiteacute juste neacutecessaire (agrave la demande) donc eacuteviter les stocks

Flexibiliteacute intelligence des chaicircnes de production

Qualiteacute (eacuteviter le coucirct de la non-qualiteacute)

La notion de processus est implicite ainsi que la chaicircne de valeur client

Initiation au management copy CRCF ndash J Sornet Page 30 48

DEFIS ET TENDANCES DU MANAGEMENT

Les meacutethodes de management se deacuteveloppent pour affronter le contexte eacuteconomique

Ce chapitre preacutesente les deacutefis auxquels le management contemporain doit faire face

1 ndash Lrsquoeacutevolution eacuteconomique contemporaine

A mesure que lrsquoactiviteacute eacuteconomique mondiale srsquoaccroicirct que la technologie eacutevolue les

changements sont de plus en plus rapides Ils introduisent des situations ineacutedites auxquelles les

entreprises doivent srsquoadapter en cherchant de nouvelles solutions de management Les trois

derniegraveres deacutecennies ont eacuteteacute notamment marqueacutees par les pheacutenomegravenes suivants (que nous

listons sans tenir compte des liens pouvant exister entre eux)

Pheacutenomegravene Traduction Effets

Deacute reacuteglementation

globalisation

financiegravere

titrisation

Libre circulation des capitaux accegraves

facile des particuliers au marcheacute

boursier (directement ou par

lrsquointermeacutediaire des OPCVM et SICAV)

Monteacutee en puissance du financement

des entreprises sur le marcheacute boursier

Fonds de pension

(retraites) et fonds

souverains (eacutetats)

Poids boursier important drsquoinvestisseurs

institutionnels qui cherchent un haut

rendement financier (dividendes ou

valorisation boursiegravere)

Pression sur les grandes entreprises

influence sur les strateacutegies

Mondialisation Liberteacute des eacutechanges internationaux Accroissement de la concurrence

recherche drsquoavantages eacuteconomiques

par la deacutelocalisation (biens et

services) la concentration des efforts

(recentrage) problegravemes drsquoemploi

multiplication des transports perte

drsquoinfluence des politiques

Baisse de lrsquoemploi

occidental

(notamment

industriel)

Moins de fabrications fabrications

automatiseacutees recours aux moyens

informatiques

Activiteacute reporteacutee sur le commerce la

conception et les services chocircmage

charge sociale

Restructurations Optimisation des entreprises

abaissement des coucircts augmentation

des marges recherche drsquoune taille

critique (eacuteconomies drsquoeacutechelle poids

sur le marcheacute)

Recentrages externalisations fusions

deacutelocalisations constitution de grands

groupes

NTIC (nouvelles

technologies de

lrsquoinformation et de

la communication)

Mise en œuvre des reacuteseaux (dont

internet) et drsquoapplications

informatiques communicantes

Nouvelles formes de commerce

marcheacute international deacutelocalisation

du travail intellectuel reacuteorganisation

de la distribution

Rareacutefaction relative

des matiegraveres

premiegraveres

Recherche de substituts exploration

miniegravere coucircts drsquoexploitation des

gisements accrus

Augmentation des coucircts variations

erratiques du cours des matiegraveres

deacutestabilisations politiques

Evolution

geacuteopolitique et

eacuteconomique

mondiale

Chute de lrsquoURSS transformation des

eacuteconomies collectivistes pays

eacutemergents (Chine Inde Breacutesil Russie)

()

Accroissement de la population

mondiale (4 agrave 6 7 milliards de 1970 agrave

2008)

Libeacuteralisme sans frein () nouvelles

puissances eacuteconomiques

opportuniteacutes de deacuteveloppement

nouveau partage des ressources

ineacutegaliteacutes baisse du soutien aux PVD

laquo Terrorisme raquo Actions armeacutees pression de groupes

armeacutes non gouvernementaux

Deacutestabilisations reacutegionales charge

des deacutepenses militaires

Deacuteveloppement

durable

Recherche drsquoune croissance eacutequitable

et respectueuse de lrsquoenvironnement

Pression sur les entreprises (eacutetats

associations de consommateurs

eacutecologistes ONG)

() Reacutecemment quelques affaires (Enron laquo subprimes raquo Vivendi Universal Socieacuteteacute

Geacuteneacuterale Airbus par exemple) et agrave plus grande eacutechelle la crise financiegravere de 2008 ont

montreacute les dangers drsquoune libeacuteralisation sans controcircles suffisants

Initiation au management copy CRCF ndash J Sornet Page 31 48

() Des alliances eacuteconomiques naissent entre pays eacutemergents (notamment en

ameacuterique centrale creacuteation de la Banque du Sud en 2008 par exemple) et lrsquoon

commence agrave imaginer une baisse progressive de lrsquoinfluence eacuteconomique des Etats

Unis

2 ndash Les deacutefis actuels du management

21 ndash Les grandes orientations

Lrsquoeacutevolution eacuteconomique suggegravere quelques pistes parfois concurrentes pour lrsquoaction du

manager contemporain On y retrouve au premier plan la construction drsquoune vision qui est

une composante commune du leadership

Objectif du manager

pour lrsquoentreprise

Justification Facteurs de reacuteussite

Construire une vision Eclairer lrsquoavenir de lrsquoentreprise partager

un but souder motiver

Effort de reacuteelle prospection

volontarisme de la direction

bonne communication

Reacuteactiviteacute et flexibiliteacute

(sous tous les aspects

agrave tous niveaux)

Srsquoadapter rapidement au marcheacute Bonne organisation des processus

personnel compeacutetent autonome

et motiveacute structure hieacuterarchique

alleacutegeacutee robotisation

Deacutegager des profits Reacutemuneacuterer les apporteurs de capitaux

srsquoautofinancer

Ajuster coucircts et structures

Exploiter les nouvelles

technologies

Reacuteactiviteacute ajuster coucircts et deacutelais

reacutepondre au marcheacute suivre les clients

Organiser le SI de faccedilon

pertinente eacuteviter le coucirct excessif

drsquoinvestissements trop en

laquo pointe raquo (laquo essuyer les placirctres raquo)

utiliser judicieusement les services

exteacuterieurs

Bacirctir des alliances

(contrats fusions)

Deacutevelopper une activiteacute limiter les coucircts

de transaction () atteindre la taille

critique et de meilleurs rendements se

recentrer sur une activiteacute profitable

Dominer les processus se donner

une identiteacute lisible externaliser se

doter drsquoune capaciteacute financiegravere

suffisante

Valoriser lrsquoimage Attirer les clients favoriser les alliances

donner confiance (apporteurs de fonds

employeacutes clients partenaires socieacuteteacute

civile)

Instaurer des regravegles de

gouvernance inteacutegrer le

deacuteveloppement durable

respecter lrsquoenvironnement

Geacuterer les risques Faire face aux aleacuteas eacuteconomiques et

technologiques (conjoncture politiques

accidents malveillance)

Creacuteer un systegraveme drsquoalerte geacuterer

la crise (reacuteaction raisonneacutee

sceacutenarios poursuite de

lrsquoexploitation dans un contexte

instable) mise en place de

proceacutedures drsquoapprentissage pour

ameacuteliorer les reacuteactions au fil du

temps

Geacuterer le changement Faire face agrave lrsquoeacutevolution de la demande

la pression sur les prix la variation des

performances financiegraveres la

concurrence la globalisation des

marcheacutes lrsquoeacutevolution technologique aux

fusions ou alliances aux changements

de reacuteglementation de direction hellip ()

Bonne communication pour

donner du sens au changement

et obtenir lrsquoadheacutesion du personnel

Rassembler et geacuterer les

connaissances former le

personnel

Innover Garder un avantage concurrentiel se

diffeacuterencier

Veille technologique et

commerciale investissement

Ouverture

internationale

Elargir le marcheacute saisir les opportuniteacutes Veille commerciale partenariats

() La theacuteorie des coucircts de transaction deacuteveloppeacutee par OE Williamson dans les

anneacutees 70 integravegre les coucircts lieacutes au recours au marcheacute (recherche et choix drsquoun

fournisseur neacutegociation reacutedaction de contrat suivi des eacutechanges risque de rupture

Initiation au management copy CRCF ndash J Sornet Page 32 48

drsquoapprovisionnement hellip) On peut en conclure que lrsquointeacutegration de diffeacuterentes

activiteacutes agrave lrsquoentreprise (la laquo firme raquo) preacutesente des avantages Mais des coucircts de

transaction internes doivent aussi ecirctre consideacutereacutes (preacuteparation organisation

surveillance hellip) et certaines formes de coopeacuteration continue avec les fournisseurs

permettent de reacuteduire le coucirct des transactions externes

() drsquoapregraves laquo Les meilleures pratiques de management raquo - Brilman Heacuterard ndash EO

Une eacutetude du Conference Board (2002) liste les deacutefis du management vus par 700 leaders

mondiaux Soit en reacutesumeacute avec indication du score correspondant

1 ndash Fideacuteliser les clients (42)

2 ndash Reacuteduire les coucircts (38)

3 ndash Accroicirctre flexibiliteacute et reacuteactiviteacute (29)

4 ndash Amener les employeacutes agrave adheacuterer aux valeurs et visions de lrsquoentreprise (26)

5 ndash Deacutevelopper et retenir les leaders (25)

6 ndash Geacuterer acquisitions et alliances (24)

7 ndash Accroicirctre lrsquoinnovation (20)

En fin de classement citoyenneteacute et reacuteputation (4) et ameacutelioration de la diversiteacute (3)

22 ndash Les techniques disponibles

Pour faire face aux deacutefis le manager dispose de nouveaux concepts et de nouvelles

techniques Le tableau ci-dessous en donne un reacutesumeacute et indique les domaines qursquoils

influencent principalement

Initiation au management copy CRCF ndash J Sornet Page 33 48

Principaux concepts techniques outils Incidence principale sur

Internet

- e-commerce (commerce eacutelectronique site

entreprise)

- CRM ou GRC (gestion de la relation client)

- e-procurement (gestion des approvisionnements

par le reacuteseau)

- messagerie eacutelectronique

- e-recrutement

Vente accegraves au marcheacute

Relation client reacuteactiviteacute personnalisation

fideacutelisation

Deacutelais coucircts

Communication transfert de donneacutees (piegraveces

jointes) tous domaines

Communication recrutement

Intranet reacuteseau drsquoentreprise SI

- knowledge management (gestion des

connaissances)

- e-learning (apprentissage en ligne)

- plateforme de travail collaboratif (groupware)

- workflow (circulation eacutelectronique de

documents enchaicircnement de processus)

- e-RH portail RH (libre accegraves aux postes agrave

pourvoir informations candidatures hellip)

- PGI (progiciel de gestion inteacutegreacute) ou ERP

Innovation capaciteacute au changement veille

documentaire

Formation du personnel accompagnement des

changements

Coordination communication interne

Coordination

Communication interne (voire internet en

externe) reacuteduction des coucircts climat drsquoentreprise

recrutement plans de carriegraveres hellip

Coucircts fiabiliteacute du systegraveme drsquoinformation deacutelais

processus (continuiteacute inteacutegration)

Logistique inteacutegreacutee

Supply Chain Management (SCM) gestion de la

logistique (incluant les approvisionnements)

Processus deacutelais coucircts

Externalisation

Valorisation du capital humain

GPEC (gestion preacutevisionnelle des emplois et

compeacutetences)

Coaching

Reacuteactiviteacute de lrsquoentreprise conservation des

compeacutetences rendements individuels turn-over

adaptation des compeacutetences motivation

Efficaciteacute individuelle controcircle reacutegulation

progregraves processus

Approche processus

Optimisation des processus

Deacutemarche qualiteacute totale (TQM ndash total quality

management)

Empowerment (empouvoirement)

Benchmarking reacuteingeacutenieacuterie

Coucircts marges qualiteacute deacutelais flexibiliteacute

externalisation eacutelargissement des compeacutetences

organisation

Ameacutelioration des processus (meacutetiers et supports)

Autonomie compeacutetences des employeacutes

Ameacutelioration des processus restructuration

Management par la valeur

Parties prenantes

Satisfaction des parties prenantes financement

motivation collaborations hellip

Collaboration inter organisations

Reacuteseaux drsquoentreprises alliances

EDI (eacutechange de donneacutees informatiseacutees) extranet

Impartition externalisation (outsourcing)

Coucircts recentrage investissements lancement

drsquoactiviteacute

Coucircts reacuteactiviteacute deacutelais relations avec

lrsquoadministration

Coucircts recentrage limitation des investissements

Ethique drsquoentreprise

Gouvernance drsquoentreprise (mode de direction

encadreacute par des regravegles)

Rocircle socieacutetal deacuteveloppement durable

environnement

Image de lrsquoentreprise reacutegulation du top

management relations actionnaires

Image peacutenaliteacutes et amendes objectifs

strateacutegiques

Initiation au management copy CRCF ndash J Sornet Page 34 48

23 ndash Le rocircle socieacutetal des entreprises

La responsabiliteacute socieacutetale de lrsquoentreprise (RSE) deacutesigne le rocircle qursquoelle prend dans la socieacuteteacute

au-delagrave de son activiteacute purement geacuteneacuteratrice de profit On parle aussi drsquoentreprise citoyenne

La RSE est indissociable du deacuteveloppement durable de porteacutee mondiale et dont les trois

piliers sont

- eacuteconomique (favoriser le deacuteveloppement les eacutechanges internationaux)

- social (accegraves aux soins eacuteducation conditions de travail hellip)

- environnemental (pollution preacuteservation des ressources hellip)

La RSE integravegre notamment une preacuteoccupation sociale de lrsquoentreprise vis-agrave-vis de ses salarieacutes

(seacutecuriteacute et santeacute au travail juste reacutemuneacuteration deacuteveloppement personnel hellip) Elle conduit agrave

tenir compte dans le management drsquoune vision exteacuterieure agrave lrsquoentreprise qui peut avoir des

reacutepercussions possibles sur son activiteacute eacuteconomique

Lrsquoentreprise peut aussi tirer avantage drsquoune deacutemarche responsable par la baisse de certains

coucircts (plus faibles consommations drsquoeacutenergies reacuteduction des transports hellip)

Le rocircle socieacutetal de lrsquoentreprise a eacuteteacute reconnu en France par la loi laquo NRE raquo de 2001 (loi sur les

nouvelles reacutegulations eacuteconomiques) qui oblige les socieacuteteacutes franccedilaise coteacutees sur un marcheacute

reacuteglementeacute agrave rendre compte dans leur rapport annuel de leur gestion sociale et

environnementale au travers de leur activiteacute

Article 116 de la loi Le rapport viseacute agrave larticle L 225-102 rend compte hellip laquo Il comprend

eacutegalement des informations dont la liste est fixeacutee par deacutecret en Conseil dEtat sur la

maniegravere dont la socieacuteteacute prend en compte les conseacutequences sociales et

environnementales de son activiteacute Le preacutesent alineacutea ne sapplique pas aux socieacuteteacutes

dont les titres ne sont pas admis aux neacutegociations sur un marcheacute reacuteglementeacute raquo

Une norme ISO 14000 integravegre ces preacuteoccupations et des taxes eacutecologiques sont

progressivement creacutees

3 ndash Le management par la valeur

31 ndash De lrsquoanalyse au management par la valeur

Lrsquoanalyse de la valeur est neacutee en 1947 aux Etats-Unis (General Electrics) Cette technique

consiste agrave eacutelaborer des produits conformes aux attentes de la clientegravele mais sans excegraves pour

trouver un bon compromis entre valeur pour le client et coucirct Le produit optimal est deacutefini agrave

partir drsquoenquecirctes qui deacuteterminent le besoin client (ou plutocirct drsquoun client laquo type raquo)

Exemple il est inutile de concevoir un petit veacutehicule citadin capable de parcourir

500 000 km sans avarie compte tenu des effets de mode et du faible kilomeacutetrage

annuel Par contre le marcheacute peut exiger un fonctionnement sans faille sur 150 000 km

soit dix ans en moyenne ce qui conditionne les coucircts de production

Cette recherche drsquoun ajustement de valeur au besoin des clients eacutetait un preacutecurseur du

management par la valeur qui recherche plus largement la creacuteation de valeur pour

chacune des parties prenantes de lrsquoentreprise tout en lui meacutenageant un reacutesultat suffisant

Plus geacuteneacuteralement le management par la valeur est deacutefini par une norme europeacuteenne (EN

12973)

Le management par la valeur est un style de management particuliegraverement destineacute agrave

mobiliser les individus agrave deacutevelopper les compeacutetences et agrave promouvoir les synergies et

Initiation au management copy CRCF ndash J Sornet Page 35 48

linnovation avec pour objectif la maximisation de la performance globale dun

organisme Le management par la valeur apporte une nouvelle faccedilon dutiliser nombre

de meacutethodes de management existantes Il est en coheacuterence avec le Management

de la qualiteacute

Cette approche du management pose de nombreuses questions notamment quelles

prioriteacutes et quelles valeurs attribuer aux parties prenantes comment appreacutehender la

perception par les parties prenantes de la valeur qui leur est affecteacutee

32 ndash La valeur client

Le processus drsquoeacutelaboration drsquoun produit qui consomme des ressources coucircteuses doit creacuteer

une valeur suffisante pour provoquer lrsquoachat par le client final La production drsquoune valeur

reconnue par le client est vitale pour lrsquoentreprise mais sa deacutetermination est parfois complexe

La valeur du produit perccedilue par le client integravegre des eacuteleacutements en partie subjectifs

- une valeur drsquousage (le produit reacutepond agrave un besoin)

- une valeur drsquoestime (lrsquoimage apporteacutee par le produit un aspect affectif)

- une valeur drsquoeacutechange (deacuteduite de lrsquoespoir de revente du produit)

Valeurs drsquousage drsquoestime et drsquoeacutechange deacutependent implicitement de la qualiteacute (un bien peu

fiable est impropre agrave lrsquousage attendu de mauvaise qualiteacute notoire il nrsquoapporte pas une

image positive et ses deacutefauts connus nuisent agrave sa revente) Une eacutevaluation de la qualiteacute

intervient donc dans la valeur perccedilue du produit

Par ailleurs le client considegravere le coucirct drsquoobtention du produit (les charges qursquoil doit supporter

pour acqueacuterir le produit lrsquoeffort qursquoil doit faire pour trouver le produit et les frais de mise agrave

disposition)

Le prix perccedilu par le client est geacuteneacuteralement supeacuterieur au prix de vente

Le client achegravete theacuteoriquement le produit qui preacutesente la diffeacuterence valeur perccedilue ndash prix

perccedilu la plus favorable ou le meilleur rapport prix perccedilu qualiteacute perccedilue et dans certains

cas celui qui a le prix produit le plus bas

Remarque les valeurs du scheacutema ci-dessus changent durant le cycle de vie du produit

(un nouveau produit peut avoir une valeur perccedilue plus eacuteleveacutee qursquoen fin de vie) La

valeur client ne peut ecirctre eacutevalueacutee que par enquecirctes et ne peut donc ecirctre deacutefinie avec

certitude

La notion de laquo satisfaction client raquo conseacutecutive agrave une vente influence aussi le prix produit et

le prix perccedilu

- lrsquoentreprise gagne sur les coucircts de recherche de clientegravele

- le client nrsquoa pas agrave rechercher un nouveau fournisseur et beacuteneacuteficie drsquoun coucirct drsquoobtention

plus bas

valeur perccedilue client

prix perccedilu client

coucirct produit Marge (valeur creacuteeacutee pour

lrsquoentreprise)

euros

prix produit

Valeur creacuteeacutee

pour le client

Initiation au management copy CRCF ndash J Sornet Page 36 48

La satisfaction du client deacutepend de facteurs qualitatifs aussi divers que la fiabiliteacute du produit

la vitesse de reacuteaction du fournisseur lrsquoattitude des commerciaux lrsquoefficaciteacute du service

apregraves-vente la netteteacute des contrats ou la justesse de la facture

Valeur perccedilue coucirct marge et satisfaction reacutesultent de processus allant de la conception du

produit jusqursquoagrave sa livraison et son apregraves-vente La deacutemarche laquo processus raquo et lrsquolaquo analyse de la

valeur raquo en forccedilant la recherche de solutions efficientes agrave tout niveau administratif

technique commercial et apregraves-vente sont donc neacutecessaires pour bien positionner

lrsquoentreprise sur son marcheacute

Pour autant le risque commercial ne peut jamais ecirctre annuleacute et lrsquooffre de lrsquoentreprise ne

satisfait geacuteneacuteralement pas en milieu concurrentiel tous ses clients potentiels

33 - La creacuteation de valeur pour les autres parties prenantes

Les salarieacutes

La creacuteation drsquoune valeur suffisante pour les salarieacutes est reconnue comme neacutecessaire car des

observations montrent que la satisfaction des clients en deacutepend Moins souvent eacutevoqueacutee en

peacuteriode de chocircmage elle nrsquoest prioritaire que pour les employeacutes dont lrsquoentreprise souhaite

conserver les compeacutetences

La laquo valeur salarieacute raquo ne comprend pas que le salaire Le sentiment drsquoappartenance agrave un

groupe la reconnaissance lrsquoaccomplissement de soi et la construction professionnelle en

sont des eacuteleacutements importants Comme pour les clients on doit ainsi distinguer la reacutetribution

perccedilue du salaire objectif

Les actionnaires

Lrsquoactionnaire apporte des fonds propres agrave lrsquoentreprise en contrepartie de titres parfois

neacutegociables en bourse et assortis drsquoun droit de vote en assembleacutee geacuteneacuterale La valeur

attribueacutee aux actionnaires est servie en termes moneacutetaires (dividende ou augmentation de la

valeur du titre neacutegociable)

Remarque des facteurs non moneacutetaires comme lrsquoimage de lrsquoentreprise qui deacutepend

en partie de sa communication peuvent influencer la deacutecision drsquoachat de vente ou

de conservation des titres par lrsquoactionnaire

Reacutetribution perccedilue euros

Salaire objectif

Avantage non

moneacutetaire de

lrsquoemploi

Initiation au management copy CRCF ndash J Sornet Page 37 48

Compte tenu de lrsquoimportance croissante de lrsquoactionnariat dans le financement des grandes

entreprises coteacutees en bourse et notamment des investisseurs institutionnels comme les fonds

de pension des indicateurs speacutecifiques ont eacuteteacute introduits pour appreacutecier la performance des

entreprises vue par les actionnaires Par exemple la valeur ajouteacutee eacuteconomique (EVA reg

economic value added marque deacuteposeacutee de Stern Stewart ou VAE ndash valeur ajouteacutee

eacuteconomique parfois deacutenommeacutee VEC ndash valeur eacuteconomique creacuteeacutee) qui prend en compte le

coucirct du capital

LrsquoEVA correspond tregraves scheacutematiquement au calcul suivant

EVA = (PO) profit opeacuterationnel ndash (C) coucirct du capital X (CE) capitaux employeacutes

LrsquoEVA neacutecessite en pratique des retraitements assez complexes Le PO peut se deacuteterminer

selon les principes suivants

- PO = reacutesultat drsquoexploitation (avant inteacuterecircts) ndash impocirct

- PO = beacuteneacutefice courant (tenant compte des inteacuterecircts) + inteacuterecircts ndash eacuteconomie drsquoimpocirct sur les

inteacuterecircts (on exclue les eacuteleacutements financiers et lrsquoimpocirct correspondant) ndash impocirct

- lrsquoimpocirct pris en compte correspond au profit opeacuterationnel consideacutereacute (dans les cas courants agrave

13 du PO)

C = taux moyen de reacutemuneacuteration du capital (reacutesultant par exemple du dividende exigeacute de

certains investisseurs et des taux drsquoemprunts bancaires)

CE = capitaux propres et dettes portant inteacuterecirct

Remarque le profit opeacuterationnel ou reacutesultat opeacuterationnel correspond au NOPAT ndash net

operating profit after tax - anglo-saxon LrsquoEVA est eacutegale au NOPAT diminueacute de la

reacutemuneacuteration des capitaux

Exemple lrsquoentreprise X dispose drsquoun capital de 2 500 000 euro et reacutealise un beacuteneacutefice net

drsquoimpocirct de 450 000 euro (taux 33 13) Un dividende de 6 doit ecirctre verseacute aux

actionnaires et la banque lui a accordeacute un precirct de 1 200 000 euro agrave 4 Les autres

constituants des reacutesultats financier et exceptionnel sont neacutegligeables

Reacutesultat opeacuterationnel = 450 000 + 004 x 1 200 000 x 23 = 482 000 euro

Coucirct du capital = 006 x 2 500 000 + 004 x 1 200 000 x 23 = 182 000 euro

EVA = 300 000 euro

Coucirct moyen pondeacutereacute du capital (C) = (004 x 1 200 000 x23 + 006 x 2 500 000)

3 700 000 Soit 492

Si lrsquoEVA est positive lrsquoentreprise creacuteeacutee de la valeur apregraves reacutemuneacuteration des capitaux et sa

valeur boursiegravere doit augmenter

Lrsquoutilisation de lrsquoEVA comme indicateur influence le management de lrsquoentreprise car il y a

trois moyens pratiques drsquoaugmenter lrsquoEVA

- augmenter le reacutesultat opeacuterationnel

- lancer des investissements ayant une rentabiliteacute supeacuterieure agrave C

- eacuteliminer les activiteacutes ayant une rentabiliteacute infeacuterieure agrave C

Remarque lrsquoutilisation sans nuance de lrsquoEVA comme critegravere de management peut

poser problegraveme Le calcul de lrsquoEVA repose sur des ajustements comptables il est donc

sujet agrave manipulations (provisions capitalisation ou non de la RD hellip) Par ailleurs le

critegravere laquo EVA raquo pris isoleacutement peut conduire agrave chercher la rentabiliteacute agrave court terme agrave

reacuteduire les investissements prospectifs et donc nuire agrave terme au deacuteveloppement de

lrsquoentreprise

Initiation au management copy CRCF ndash J Sornet Page 38 48

Les fournisseurs reccediloivent le paiement de leurs factures plus ou moins rapidement (le deacutelai

de paiement repreacutesente une valeur consentie au fournisseur)

Lrsquoentreprise peut accroicirctre la valeur apporteacutee agrave ses fournisseurs par des actions cibleacutees

comme une contribution agrave la formation de leurs personnels certains transferts de

technologie ou de savoir faire agrave des sous-traitants une coopeacuteration suivie favorisant leur

deacuteveloppement lrsquointeacutegration agrave des campagnes de promotion

A noter que la valeur consentie aux fournisseurs peut avoir une influence sur la qualiteacute et les

deacutelais de livraison des produits

La collectiviteacute reccediloit des taxes et parfois des prestations en nature par deacutefaut ou explicites

(effort de preacuteservation de lrsquoenvironnement ameacutenagement du territoire par les implantations

aide mateacuterielle agrave des projets participation agrave la formation par exemple)

APPLICATIONS DT

DT1 Deacutefinir expliquer deacutereacuteglementation socieacutetal eacuteconomies drsquoeacutechelle coaching EDI

gouvernance

DT2 Deacuteterminer en quoi la deacutemarche TQM srsquoinscrit dans les deacutefis actuels du management

DT3 Apregraves avoir consulteacute les documents ci-dessous extraits du site drsquoAir France

(httpdeveloppement-

durableairfrancecomFRfrlocaldemarcheN4_positionnement_pphtm)

exposer les enjeux et les limites de la RSE et de la gestion des parties prenantes

Initiation au management copy CRCF ndash J Sornet Page 39 48

Dialogue avec les parties prenantes

Initiation au management copy CRCF ndash J Sornet Page 40 48

Attentes des parties prenantes

Initiation au management copy CRCF ndash J Sornet Page 41 48

Creacuteation de valeur pour les parties prenantes

La creacuteation de valeur pour les parties prenantes est au cœur de la strateacutegie du Groupe Le scheacutema de

distribution financiegravere ci-dessous donne un aperccedilu de la distribution des recettes du Groupe aux

diffeacuterentes parties prenantes actionnaires collaborateurs fournisseurs pouvoirs publics

collectiviteacutes locales etc

Initiation au management copy CRCF ndash J Sornet Page 42 48

Fiche DT1 ndash Extrait du sommaire de laquo Problegravemes eacuteconomiques raquo No 2894

La gestion des entreprises bouleverseacutee par les technologies de linternet

Reacutealiteacutes industrielles - Annales des Mines Jean-Michel Yolin

Avec lavegravenement de linternet les processus de conception de production et de vente sont

radicalement remis en cause Quel que soit le secteur dactiviteacute les technologies de linternet

permettent en effet de reacuteduire les deacutelais et de passer dun processus discontinu agrave un processus

continu Lorganisation des entreprises et leur mode de gestion en sont profondeacutement bouleverseacutes

tant au niveau individuel que collectif Linternet rend ainsi possible la reacutealisation dobjectifs que les

entreprises cherchaient agrave atteindre depuis longtemps sans y parvenir meilleure eacutecoute du client

travail sans stocks en flux tendu hieacuterarchies plates autorisant une grande reacuteactiviteacute flexibiliteacute dans

lorganisation et loutil de production acceacuteleacuteration du renouvellement des produits entreprises en

reacuteseau ougrave chacune se recentre sur son cœur de meacutetier etc

Le laquo knowledge management raquo ou comment geacuterer les connaissances

Document de travail du LAMSADE - Michel Grundstein

Peter Drucker lavait preacutedit le capital immateacuteriel eacutetait voueacute agrave devenir un facteur de compeacutetitiviteacute

pour lentreprise La libeacuteralisation des eacutechanges acceacutelegravere les processus de deacutecision de lentreprise

et implique que lassimilation des informations soit agrave la fois de meilleure qualiteacute et plus rapide Ainsi

la fonction qui consiste agrave manager les connaissances au sein de lentreprise savegravere primordiale

Bien que la prise de conscience de limportance du capital immateacuteriel ait eacuteteacute tardive - le concept

de knowledge management est apparu en France aux Etats-Unis et au Japon au milieu des

anneacutees 1990 - agrave lheure actuelle lorganisation de leacutechange dinformations et le partage des

connaissances sont devenus des facteurs cleacutes dune gestion performante de lentreprise Ils

doivent sinscrire dans un projet global destineacute agrave mettre en valeur les savoirs et les savoir-faire

individuels et collectifs

Les leccedilons du laquo coaching raquo pour le management de la qualiteacute

Humanisme et Entreprise - Martine Brasseur

Parmi les nouvelles formes de management en vogue dans les entreprises le coaching figure en

bonne place Appliqueacute au management de la qualiteacute il sagit dune pratique

daccompagnement destineacutee agrave initier et agrave faciliter le processus de deacuteveloppement dun individu

La deacutemarche consiste agrave affirmer que tout individu est en quecircte de qualiteacute agrave condition toutefois

de ne pas lui imposer des contraintes lempecircchant de progresser On considegravere notamment les

erreurs comme potentiellement feacutecondes En deacutefinitive le coach donne au coacheacute la permission

de reacuteussir en lui donnant aussi la permission deacutechouer

Initiation au management copy CRCF ndash J Sornet Page 43 48

Fiche DT2 ndash Management strateacutegique les sept deacutefis agrave relever dici agrave 2016

Extrait drsquoun article du site wwwlentreprisecom -Sabine Blanc - Mis en ligne le 20032007

(httpwwwlentreprisecom325article11977html)

Une eacutetude anglaise publieacutee par lopeacuterateur Orange Grande-Bretagne deacutecrypte la mutation

des formes de travail et les enjeux majeurs pour les entreprises de demain afin decirctre au top

de la compeacutetitiviteacute Voici les challenges-cleacutes pour les managers qui veulent rester dans la

course hellip

1 - Future organisation du travail les quatre laquo mondes raquo possibles

La reacutealiteacute sera probablement un meacutelange de ces quatre sceacutenarios souligne lrsquoeacutetude

Les mondes mutuels Tout se passe dans le cadre des communauteacutes locales vie priveacutee

comme professionnelle Le modegravele coopeacuteratif preacutevaut au lieu du laquo big business raquo Oublieacutes

aussi dans ce systegraveme les trajets pour aller au bureau les gens preacutefegravereront travailler dans de

petites entreprises locales souvent connecteacutees au reacuteseau drsquoautres structures similaires

Les laquo reacutepondants raquo (en anglais laquo replicants raquo) La figure du consultant freelance deviendra

dominante tandis que celle du salarieacute deacuteclinera Il ne sera pas rare de travailler pour plusieurs

entreprises On perdra en seacutecuriteacute de lrsquoemploi en visibiliteacute et en routine ce que lrsquoon gagnera

en liberteacute La majeure partie des tacircches srsquoeffectuera chez soi avec la possibiliteacute de srsquoinstaller

temporairement dans les bureaux de son client du moment Dans un contexte dincertitude

sur lrsquoavenir les travailleurs alterneront peacuteriodes drsquoactiviteacute intense et repos Ce sera agrave eux

drsquoaller vers les entreprises et non lrsquoinverse mecircme si celles-ci devront veiller agrave rester attractives

Les cottages eacutelectroniques Comme ce nom le suggegravere le teacuteleacutetravail deviendrait la norme

univers priveacute et professionnel se confondant Plus besoin de subir une heure de transport les

salarieacutes se logueront de chez eux sur le reacuteseau de lrsquoentreprise Les reacuteunions se tiendront dans

de petits bureaux centraux situeacutes agrave courte distance La flexibiliteacute du temps de travail srsquoimpose

Les salarieacutes disposeront de plus de marge de liberteacute dans leur activiteacute

Les disciples de la nueacutee Cette appellation poeacutetique cache simplement une extension de

lrsquoorganisation actuelle des grandes entreprises avec des salarieacutes se rendant sur un lieu de

travail centraliseacute Le rocircle croissant des technologies de lrsquoinformation multipliera les faccedilons de

collaborer et accroicirctra lrsquoefficaciteacute Le controcircle du travail sera omnipreacutesent La frontiegravere entre

travail et vie priveacutee restera marqueacutee

2 - Sept deacutefis pour les entreprises et leur managers

Quoi qursquoil advienne les entreprises et leurs dirigeants devront concentrer leurs efforts sur sept

points-cleacutes pour srsquoadapter Voici quelques exemples de probleacutematiques souleveacutees par le

rapport et des pistes de solution

Le leadership Les managers devront entre autres savoir persuader et influencer des

travailleurs beaucoup plus indeacutependants Ils auront aussi agrave repenser les niveaux auxquels

prendre les deacutecisions strateacutegiques en haut ou au contraire agrave des degreacutes moins eacuteleveacutes de la

pyramide hieacuterarchique

gt Faire du management une force facilitant les activiteacutes transversales plutocirct que la reacuteduire agrave

la seule fonction de deacutecision

La culture drsquoentreprise Davantage de salarieacutes capables de reacutefleacutechir seront neacutecessaires

tandis que les tacircches qui peuvent ecirctre automatiseacutees ou scripteacutees diminueront Un des

enjeux creacuteer une culture agrave mecircme drsquoattirer et drsquoencourager les personnes preacutesentant ces

qualiteacutes de reacuteflexion requises dans un contexte de compeacutetition accrue et de plus grande

indeacutependance des travailleurs

Initiation au management copy CRCF ndash J Sornet Page 44 48

gt Passer si neacutecessaire drsquoune culture drsquoentreprise forte agrave un mode drsquoengagement plus

consensuel moins rebutant

La marque Conseacutequence du recours croissant agrave lrsquo laquo outsourcing raquo lrsquoimage drsquoune marque

deacutependra plus drsquoagents exteacuterieurs qui ne fonctionnent pas forceacutement selon le mecircme mode

drsquoorganisation Comment garder le controcircle dessus

gt Choisir le mode qui corresponde le plus agrave vos valeurs et preacutevoir un programme de risk

management qui mette en eacutevidence ougrave les conflits sont susceptibles de jaillir

Lrsquoinnovation Plus que jamais il faudra faire face agrave une acceacuteleacuteration du rythme de

lrsquoinnovation en proposant constamment des solutions adapteacutees

gt Tisser des partenariats strateacutegiques avec drsquoautres entreprises pour partager les coucircts et les

fruits de lrsquoinnovation

Le deacutefi opeacuterationnel et technologique De quelle faccedilon controcircler lrsquoinformation crsquoest-agrave-dire

faire en sorte que les bonnes personnes accegravedent facilement agrave une information toujours en

phase tout en maintenant la seacutecuriteacute

gt Recourir agrave des laquo feuilles de route des futurs raquo syntheacutetisant en une page les indicateurs

sociaux et de consommation ainsi que les eacutevolutions technologiques et leacutegislatives qui

influent sur les changements et indiquant comment ils modifient vos marcheacutes vos clients et

votre organisation

La qualiteacute Si de nouveaux proceacutedeacutes ont pu deacutegrader la qualiteacute comme le recours agrave des

centres drsquoappel externaliseacutes drsquoautres ideacutees se sont reacuteveacuteleacutees plus prometteuses comme en

teacutemoigne le succegraves de certaines compagnies aeacuteriennes low cost Elles ont su conjuguer prix

serreacutes et services eacuteleveacutes ce qui devra devenir la norme estime lrsquoeacutetude

gt Continuer de rechercher la qualiteacute Elaborez aussi une bonne prestation service qui inclut

une livraison de qualiteacute voire creacuteez-la en partenariat avec les consommateurs

La leacutegislation La question de la proprieacuteteacute intellectuelle pourrait ecirctre probleacutematique Elle est

deacutejagrave source de conflits comme en teacutemoigne le procegraves pour violation de brevet intenteacute agrave RIM

le fabricant canadien du Blackberry par NTP Que pourra-t-on et que faudra-t-il proteacuteger par

un brevet Il sera eacutegalement neacutecessaire drsquoadapter la leacutegislation aux nouveaux modes

drsquoorganisation

gt Collaborer avec les acteurs du mecircme secteur et les leacutegislateurs pour deacutevelopper les

modegraveles des lieux de travail du futur et bacirctir le droit le plus adeacutequat

Orange a-t-il vu juste dans ses preacutevisions Rendez-vous dans neuf ans pour la reacuteponsehellip

Initiation au management copy CRCF ndash J Sornet Page 45 48

Fiche DT3 ndash Le management par la qualiteacute totale

Extrait drsquoune lettre drsquoinformation du cabinet Baud Accordance Consulting AD2 consultants ndash

2002

1 - Le TQM (Total Quality Management) offre pour lentreprise une vision de la qualiteacute plus

large et transversale

Son principe est simple La finaliteacute de lEntreprise est de deacutevelopper la satisfaction de ses

clients tout en eacutetant beacuteneacuteficiaire cest agrave dire pas agrave nimporte quel prix Elle doit ameacuteliorer sa

rentabiliteacute au travers de la deacutemarche qualiteacute La Qualiteacute Totale vise agrave fournir aux clients

externes et internes une reacuteponse adeacutequate agrave leurs attentes dans le meilleur rapport qualiteacute

prix la meilleure efficience

Elle considegravere pour cela lensemble des processus de lentreprise ayant une incidence sur la

qualiteacute et la satisfaction des clients

Le TQM fait ainsi une large place agrave

la deacutefinition et la planification de la strateacutegie geacuteneacuterale

la coheacuterence de la politique qualiteacute avec la strateacutegie

la deacutemultiplication de la politique qualiteacute dans toutes les directions de lentreprise

la relation client fournisseur interne

la prise en compte de lenvironnement concurrentiel

la consideacuteration de lensemble des risques potentiels financiers sociaux concurrentielshellip

limplication et la motivation du personnel

lanalyse des besoins des clients et le positionnement marketing

la maicirctrise des processus transverses internes

les reacutesultats sous tous ses aspects y compris financiers commerciaux image

De nombreux reacutefeacuterentiels sont relatifs agrave la Qualiteacute Totale hellip Tous ces reacutefeacuterentiels imposent un

questionnement plus profond et indiscret sur le mode de fonctionnement de lentreprise et

son management

helliphellip

2 - LISO 9001 2000 au travers du deacuteploiement des processus (management supports

reacutealisation et ameacutelioration continue) reacutepond quelque peu agrave la mecircme logique

LISO est une ouverture indeacuteniable vers la logique du TQM mais ne se reacutefegravere pas agrave la notion

defficience

Les dirigeants sont cependant sensibles agrave la neacutecessaire reacuteduction des coucircts de non-qualiteacute

et dobtention de la qualiteacute agrave la rentabiliteacute du systegraveme de management de la qualiteacute

mais ne perccediloivent pas toujours la qualiteacute comme une deacutemarche globale

Les deacutemarches qualiteacute commencent bien souvent par la remise en cause de lorganisation

leacutevaluation critique de son efficaciteacute lexamen des processus et la mise en eacutevidence des

lourdeurs administratives

La qualiteacute devient laffaire de tous hellip

Initiation au management copy CRCF ndash J Sornet Page 46 48

Fiche DT4 ndash Le deacuteveloppement durable et la RSE

Extrait du site wwwvigeocom

(httpwwwvigeocomcsr-rating-agencyfrmethodologiecriteres-de-recherche37-

criteres-d-analysehtml)

Deacuteveloppement durable laquo un deacuteveloppement qui reacutepond aux besoins du preacutesent sans compromettre

la capaciteacute des geacuteneacuterations futures de reacutepondre aux leurs raquo (Commission mondiale sur lrsquoenvironnement

et le deacuteveloppement ndash 1987)

Reacutefeacuterentiel drsquoeacutevaluation des entreprises par le groupe Vigeacuteo (le groupe mesure les performances et le

niveau de maicirctrise des risques de responsabiliteacute sociale des entreprises et des organisations - site

wwwvigeocom)

1 Ressources Humaines Ameacutelioration continue des relations professionnelles des relations drsquoemploi et des conditions de travail 2 Droits humains sur les lieux de travail Respect de la liberteacute syndicale et promotion de la neacutegociation collective non discrimination et promotion de lrsquoeacutegaliteacute eacutelimination des formes de travail proscrites (enfants travail forceacute) preacutevention des traitements inhumains ou deacutegradants de type harcegravelements sexuels protection de la vie priveacutee et des donneacutees personnelles 3 Environnement Protection sauvegarde preacutevention des atteintes agrave lenvironnement mise en place drsquoune strateacutegie manageacuteriale approprieacutee eacuteco conception protection de la biodiversiteacute et maicirctrise rationnelle des impacts environnementaux sur lrsquoensemble du cycle de vie des produits ou services

4 Comportements sur les marcheacutes Prise en compte des droits et inteacuterecircts des clients inteacutegration de standards sociaux et environnementaux dans la seacutelection des fournisseurs et sur lrsquoensemble de la chaicircne drsquoapprovisionnement preacutevention effective de la corruption respect des regravegles concurrentielles 5 Gouvernement drsquoentreprise Efficience et probiteacute assurance de lrsquoindeacutependance et de lrsquoefficaciteacute du Conseil drsquoadministration effectiviteacute et efficience des meacutecanismes drsquoaudit et de controcircle et notamment inclusion des risques de responsabiliteacute sociale respect des droits des actionnaires et notamment des minoritaires transparence et rationaliteacute de la reacutemuneacuteration des dirigeants 6 Engagement socieacutetal Effectiviteacute inteacutegration manageacuteriale de lrsquoengagement contribution au deacuteveloppement eacuteconomique et social des territoires drsquoimplantation et de leurs communauteacutes humaines engagements concrets en faveur de la maicirctrise des impacts socieacutetaux des produits et des services contribution transparente et participative agrave des causes drsquointeacuterecirct geacuteneacuteral

Initiation au management copy CRCF ndash J Sornet Page 47 48

ELEMENTS DE CORRIGE DT DT1 Deacutefinir expliquer

Deacutereacuteglementation = suppression des contraintes eacuteconomiques (libre eacutechange des biens et

capitaux)

Socieacutetal = qui se rapporte agrave la structure agrave lrsquoorganisation ou au fonctionnement de la socieacuteteacute

Economies drsquoeacutechelle = reacuteduction des coucircts lieacutee au niveau drsquoactiviteacute (amortissement des

charges fixes)

Coaching = accompagnement de personnes ou deacutequipes pour le deacuteveloppement de leurs

potentiels

EDI = eacutechange de donneacutees informatiseacutees ET standardiseacutees (ex SWIFT bancaire edifact

documents deacuteclaratifs)

Gouvernance = exercice du pouvoir la bonne gouvernance est participative et eacutequitable

conforme agrave lrsquointeacuterecirct commun

DT2 Deacuteterminer en quoi la deacutemarche TQM srsquoinscrit dans les deacutefis actuels du management

Voir notamment fiche 43

Maicirctrise des processus reacuteduction des coucircts reacuteactiviteacute et satisfaction de la clientegravele = faire

face agrave la concurrence

Ameacutelioration de lrsquoimage motivation du personnel

DT3 Apregraves avoir consulteacute les documents ci-dessous extraits du site drsquoAir France

(httpdeveloppement-

durableairfrancecomFRfrlocaldemarcheN4_positionnement_pphtm)

exposer les enjeux et les limites de la RSE et de la gestion des parties prenantes

Trame geacuteneacuterale possible

Introduction

Les deacutefis contemporains (accroissement de la concurrence devenue mondiale recherche

de nouveaux avantages concurrentiels pression de la socieacuteteacute besoin drsquoimage et de projet

lisible pour mener lrsquoentreprise crise et scandales du libeacuteralisme hellip) RSE et PP

Deacuteveloppement (voir cours)

1 ndash Parties prenantes et management par la valeur

PP deacutefinir citer reacutesumer lrsquoavantage rechercheacute (fideacuteliser motiver recherche drsquoalliances

implicites)

PP moyens (dont exemples AF) et meacutethode de management par la valeur (reacutepartie)

2 ndash La responsabiliteacute socieacutetale de lrsquoentreprise

RSE 3 axes

- eacuteconomique (favoriser le deacuteveloppement les eacutechanges internationaux)

- social (accegraves aux soins eacuteducation conditions de travail hellip)

- environnemental (pollution preacuteservation des ressources hellip)

RSE gouvernance drsquoentreprise facteur drsquoimage inteacutegrable dans la deacutemarche PP

Article 116 de la loi Le rapport viseacute agrave larticle L 225-102 rend compte hellip laquo Il comprend

eacutegalement des informations dont la liste est fixeacutee par deacutecret en Conseil dEtat sur la maniegravere

dont la socieacuteteacute prend en compte les conseacutequences sociales et environnementales de son

activiteacute Le preacutesent alineacutea ne sapplique pas aux socieacuteteacutes dont les titres ne sont pas admis aux

neacutegociations sur un marcheacute reacuteglementeacute raquo

Initiation au management copy CRCF ndash J Sornet Page 48 48

RSE exemple AF (ONG fournisseurs)

3 ndash Liens entre PP et RSE

- la RSE introduit de nouvelles PP

- la RSE suppose le respect des PP usuelles (employeacutes clients notamment)

4 - Probleacutematique

- deacutefinir la valeur reacuteellement apporteacutee par une gestion des PP (confusion salaire ndash valeur

idem impocircts hellip ex laquo valeur ajouteacutee raquo)

- communication (neacutecessaire mais aller au-delagrave)

- marginaliteacute des deacutepenses RSE (efficaciteacute sinceacuteriteacute de lrsquoengagement marge de manœuvre)

- charge RSE reporteacutee sur des tiers (ex fournisseurs AF)

- inteacutegration de facteurs non visibles en comptabiliteacute (pertes drsquoemploi nuisances hellip)

Conclusion

Voies incontournables mais pouvant nrsquoavoir qursquoun effet superficiel et temporaire Voir utiliteacute

drsquoaccompagnement leacutegislatif de regravegles de gouvernance

Initiation au management copy CRCF ndash J Sornet Page 9 48

Lrsquointervention de lrsquoexpert dans le management drsquoune entreprise peut toutefois poser

quelques problegravemes

- Il peut y avoir conflits drsquointeacuterecirct entre activiteacutes de certification des comptes et de conseil en

management (la tendance est agrave la seacuteparation des activiteacutes dans les plus grands cabinets)

- lrsquoactiviteacute de conseil neacutecessite des compeacutetences parfois tregraves speacutecifiques (conseil fiscal

conseil en RH conseil en systegravemes drsquoinformation hellip)

- le conseil est une activiteacute diffeacuterente par sa forme de lrsquoexpertise comptable (interventions

longues peu reacutepeacutetitives peu codifieacutees mises en concurrence) qui neacutecessite une

organisation particuliegravere du cabinet lorsqursquoelle deacutepasse lrsquointervention occasionnelle

APPLICATIONS IM

IM1 Analyser la profession de laquo manager raquo selon Henry Mintzberg (texte extrait de lrsquoouvrage

laquo Le management raquo Eyrolles - Editions drsquoOrganisation) et les principes du management de la

norme ISO

Faire ressortir les eacuteleacutements speacutecifiques agrave chacune de ces approches et mettre en eacutevidence

leurs points communs

Initiation au management copy CRCF ndash J Sornet Page 10 48

Principes du management drsquoapregraves la norme ISO 9001 (2000)

- Orientation vers le client (satisfaire ses attentes)

- Leadership (les dirigeants eacutetablissent les orientations de lrsquoorganisme Ils doivent creacuteer

un environnement interne ougrave les personnes peuvent clairement srsquoimpliquer dans la

reacutealisation des objectifs de lrsquoorganisme)

- Implication du personnel (les personnes sont agrave tout niveau lrsquoessence de lrsquoorganisme et

leur implication permet drsquoutiliser leurs aptitudes au profit de lrsquoorganisme)

- Approche laquo processus raquo (un reacutesultat est mieux atteint quand les ressources et les

activiteacutes neacutecessaires sont geacutereacutees comme un processus)

- Approche systegraveme (assimiler les processus correacuteleacutes agrave un systegraveme contribue agrave

lrsquoefficaciteacute et agrave lrsquoefficience de lrsquoorganisme vis-agrave-vis de ses objectifs)

- Ameacutelioration continue (objectif permanent de lrsquoorganisme)

- Prise de deacutecision efficace (par lrsquoanalyse de donneacutees et drsquoinformations)

- Relations mutuellement beacuteneacutefiques avec les fournisseurs (pour augmenter la capaciteacute

des deux organismes agrave creacuteer de la valeur)

IM2 Distinguer leader et manager

IM3 Compleacuteter le tableau ci-dessous en analysant chaque action preacutesenteacutee Faire ensuite

ressortir les domaines niveaux ou techniques de management pouvant ecirctre mobiliseacutes pour

chaque situation

Initiation au management copy CRCF ndash J Sornet Page 11 48

Caracteacuteristiques

de lrsquoaction

- reacutepeacutetition

- risque

- normes

- ampleur

Prise de

deacutecision

- opeacuterationnelle

strateacutegique

- deacutelai

Informations

neacutecessaires

- nature

- origine

- deacutelai obtention

Cleacutes pour la

reacuteussite

Intervention

exteacuterieure

possible

Assurer la

restauration du

soir

(restaurant

familial)

Construire un

viaduc

(autoroute)

Certifier les

comptes

annuels drsquoun

groupe

national

(cabinet

drsquoaudit)

Lancer une

ligne drsquoavions

(constructeur

aeacuteronautique)

Reacuteduire la

capaciteacute de

production

(groupe

industriel)

Acqueacuterir une

entreprise

concurrente

(teacuteleacutephonie

mobile)

Initiation au management copy CRCF ndash J Sornet Page 12 48

Fiche IM1 - Deacutefinitions du management

Dictionnaire anglais - franccedilais direction administration gestion intrigue manegravege

Wikipeacutedia Le management est lensemble des techniques dorganisation qui sont mises en

oeuvre pour ladministration dune entiteacute

Au point de vue eacutetymologique le verbe manage vient de litalien maneggiare (controcircler)

influenceacute par le mot franccedilais manegravege (faire tourner un cheval dans un manegravege) A cette

notion il faut aussi ajouter la notion de meacutenage (geacuterer les affaires du meacutenage) qui consiste agrave

geacuterer des ressources humaines et des moyens financiers

helliphellip

Fiche IM2 - Etudier le management

Concreacutetiser

Manager neacutecessite de syntheacutetiser des informations parfois complexes incomplegravetes et de

domaines tregraves divers pour en deacuteduire des actions Une approche trop parcellaire peut

conduire agrave lrsquoeacutechec et le savoir-faire est neacutecessaire pour agir vite avec un minimum de risque

Lrsquoeacutetudiant doit se preacuteparer simultaneacutement aux examens et agrave la pratique Il nrsquoa souvent connu

lrsquoentreprise que durant quelques semaines de stage et le manque de laquo recul raquo ne lui permet

pas toujours de concreacutetiser les theacuteories Il doit compenser par la lecture (ouvrages revues

journaux eacuteconomiques et boursiers) et en eacutetant attentif aux informations ambiantes (tout en

relativisant le style journalistique) en mettant en relation le cours les concepts les modegraveles

lrsquoactualiteacute les stages

Savoir traiter un exercice

Pour reacuteussir un examen ou traiter une application peacutedagogique (la conception les points 1 agrave

6 peut repreacutesenter le tiers du temps de travail)

1 ndash Identifier le type de sujet (faut-il trouver une solution pratique ou communiquer une

reacuteflexion geacuteneacuterale )

2 ndash Lire le sujet et relever les mots cleacutes

3 ndash Deacutefinir les mots cleacutes

4 ndash Reacutesumer la probleacutematique du sujet (en quelques lignes)

5 ndash Lister les connaissances reacutefeacuterences et raisonnements reacutepondant au problegraveme (par

recherche spontaneacutee ou raisonneacutee qui quoi ougrave quand comment combien hellip

listage des diffeacuterents points de vue) trouver des exemples (notamment dans les

documents fournis)

6 ndash Organiser la reacuteponse (deacutefinir le plan du deacuteveloppement ougrave des paragraphes bien

identifieacutes sont geacuteneacuteralement neacutecessaires en y liant les parties qui doivent ecirctre en nombre

limiteacute ndash de deux agrave quatre) Preacutevoir drsquoy inteacutegrer la deacutefinition des principales notions

induites par le sujet

7 ndash Reacutediger sous la forme adapteacutee (note technique ou recommandation solution

pratique exposeacute structureacute dissertation)

Introduction et conclusion sont indispensables agrave la dissertation ou agrave lrsquoexposeacute

- lrsquointroduction preacutesente le sujet traiteacute (phrase drsquoaccroche initiale) amorce la

probleacutematique (quelques sous - questions) et annonce le plan

- la conclusion syntheacutetise le deacuteveloppement (arguments) eacutelargit le sujet (prise de recul)

et apporte le point final (une phrase)

Une limite agrave la communication

Il est difficile de faire passer plus de 4 ou 5 ideacutees fortes dans un exposeacute unique

Initiation au management copy CRCF ndash J Sornet Page 13 48

Fiche IM3 - Bref historique

Antiquiteacute

3000 AJC

Peacuteriode greacuteco-

romaine

Transition

feacuteodale

12egraveme siegravecle

europe

15egraveme ndash 17egraveme

siegravecles

19egraveme siegravecle

20egraveme siegravecle

agriculture preacutedominante industrie limiteacutee aux besoins drsquoun individu ou drsquoun clan

pour la confection des outils des vecirctements et de la poterie Force motrice animale

ou humaine pour lrsquoessentiel

Grands travaux drsquoeacutetat en Egypte premiegravere laquo planification ndash organisation ndash controcircle raquo

Deacuteveloppement des communications essor industriel limiteacute peu de progregraves

technique (lrsquoesclavage supplante les innovations)

Deacuteveloppement progressif des eacutechanges commerciaux

La consommation indirecte atteint un bon niveau (surplus agricoles et

deacuteveloppement des villes) Apparition de nouveaux commerccedilants

Etat fort Evolutions technologiques (imprimerie bateaux performants instruments de

navigation) Extension geacuteographique de lrsquoeacuteconomie Apparition des corporations

drsquoartisans

Machine agrave vapeur chemin de fer passage de lrsquoartisanat au capitalisme

entrepreneurial producteur organisation des entreprises

Ecole classique (Taylor Fayol Weber) approche meacutecaniste bureaucratie

hieacuterarchie commandement fonctions et speacutecialisation laquo OS T raquo (organisation

scientifique du travail) organisation source de pouvoir rationaliteacute des individus bases

du management

Deacuteveloppement du capitalisme manageacuterial Electriciteacute peacutetrole puis communications

et information Consommation de masse mondialisation preacuteoccupations

eacutenergeacutetiques et environnementales 3 peacuteriodes

- standardisation grandes entreprises industrielles

- industries de consommation 30 glorieuses marketing multinationales protection

sociale

- deacutereacuteglementation monteacutee des services pays eacutemergents mondialisation et nouvelle

eacuteconomie (internet)

Ecole des relations humaines prise en compte de lrsquoindividu des motivations styles

de direction

Ecole neacuteo-classique et post-classique deacutecentralisation coordonneacutee DPO

management participatif zeacutero deacutefaut flux tendus

Approche systeacutemique partition de lrsquoentreprise eacutetude des interactions feacutedeacuteration

vers lrsquoobjectif controcircle et ajustement

Theacuteories de la deacutecision rationaliteacute limiteacutee contribution reacutetribution coalitions

Ecole socio-technique recherche de compromis technologie organisation

enrichissement des tacircches autonomie des groupes

Approche sociologique effets sociaux du travail jeux de pouvoir dans lrsquoentreprise

reacutegulation sociale

Theacuteories de la contingence facteurs contingents adaptation agrave lrsquoenvironnement

configurations organisationnelles

Theacuteories de la firme controcircle manageacuterial droits de proprieacuteteacute relation drsquoagence

Theacuteories contractualistes firme nœud de contrats coucircts de transaction

opportunisme externalisation internalisation

Approche eacutevolutioniste eacutecologie des organisations modegravele eacutevolutioniste

contraintes de sentier

Approche par les ressources valorisation des ressources compeacutetences cleacutes

apprentissage organisationnel

(Classement simplifieacute)

Initiation au management copy CRCF ndash J Sornet Page 14 48

ELEMENTS DE CORRIGE IM

IM1 Commenter la deacutefinition du management par la norme ISO et le manager de Mintzberg

Efficient = optimum avec les moyens disponibles

ISO (management objectifs) (manager moyens) HM

IM2 Le leader entraicircne naturellement derriegravere lui Le manager nrsquoest pas toujours leader

(mecircme si crsquoest souhaitable) Le leader nrsquoest pas toujours manager (plutocirct notion individuelle)

Leadership = faculteacute de diriger conjugaison drsquoune autoriteacute naturelle ou drsquoun savoir-faire

acquis drsquoune capaciteacute agrave entraicircner des personnes ou des groupes et drsquoune leacutegitimiteacute

statutaire (de position)

IM3 Compleacuteter le tableau ci-dessous en analysant chaque action preacutesenteacutee Faire ensuite

ressortir les domaines niveaux ou techniques de management pouvant ecirctre mobiliseacutes pour

chaque situation

Satisfaction client

Implication du personnel

Processus systegraveme

Ameacutelioration continue

Deacutecision efficace

Recherche de valeur

Image entreprise

Liaisons

Information

Reacutepartition ressources

Reacutegulation

Neacutegociation

Leadership

Initiation au management copy CRCF ndash J Sornet Page 15 48

Caracteacuteristiques

de lrsquoaction

- reacutepeacutetition

- risque

- normes

- ampleur

Prise de

deacutecision

- opeacuteration

- direction

- deacutelai

Informations

neacutecessaires

- nature

- origine

- deacutelai

obtention

Cleacute pour la

reacuteussite

Intervention

exteacuterieure

possible

Assurer la

restauration du

soir

(Restaurant

familial)

Technique

(fabrication)

Vente (terrain)

Appros

Reacutepeacutetitive

(quot)

Risque faible

Normes

drsquohygiegravene

Faible

Opeacuterationnelle

Geacuterant

responsable

Rapide (qq

jours menu et

appros)

Nombre de

couverts

Tarifs usuels

Calendrier

(fecirctes)

Clients docs

divers

expeacuterience

Qq jours

Varieacuteteacute menu

Plats phares

Accueil

Appros

Tarification

Vins

Gestion

congeacutelation

Qualiteacute cuisine

Fournisseurs

Extra

Publiciteacute

Construire un

viaduc

(autoroute)

Technique

Organisation

Appros

Uniteacute (ou peu)

Eleveacute (financier

technique)

Architecture

Eleveacutee

Direction

(aleacuteas)

Opeacuterationnelle

(conduite

chantier)

Immeacutediat agrave qq

semaines

Plans

plannings

Qualifications

Meacuteteacuteo

Disponibiliteacutes

Bureau eacutetudes

Qq sem agrave 24h

Techniciteacute

Appros

Qualifications

Preacutevision

GRH

Contrat juste

SS traitants

Organismes

certificateurs

Controcircle

client

Certifier les

comptes

annuels drsquoun

groupe national

(cabinet

drsquoaudit)

Technique

Relation client

Gestion des

connaissances

Annuelle

Moyen

Regravegles

comptables

fiscales

Moyenne (selon

importance du

cabinet)

Opeacuterationnelle

Qq jours agrave

semaines

Comptable

Juridique

Client

Etat

Qq jours agrave

semaines

Techniciteacute

Expeacuterience

Relation client

Systegraveme info client

Siegravege

Autre cabinet

Lancer une

ligne drsquoavions

(constructeur

aeacuteronautique)

Strateacutegique

RD

Etudes

Uniteacute

Tregraves eacuteleveacute

Aeacuteronautique

Tregraves eacuteleveacutee

Direction

Qq mois agrave

anneacutees

Marcheacute

Etudes

Compagnies

Qq mois agrave

anneacutees

Concept

Outil industriel

Coucirct exploitation

Tarif

Fiabiliteacute

Deacutelaisconcurrence

SI simulation

SS traitants

Bureaux

drsquoeacutetudes

speacutecialiseacutes

Compagnies

Conseils

Reacuteduire la

capaciteacute de

production

(groupe

industriel)

Strateacutegique

RH

Communication

Production

Uniteacute

Moyen

Leacutegislation

(dont RH)

Eleveacutee

Direction

Qq mois agrave

anneacutees

Financiegravere

Industrielle

Marcheacute

Organisation

Organismes

speacutecialiseacutes

DRH

Qq mois

Communication

Connaissance des

compeacutetences

Connaissance outil

industriel

Concurrence

Portefeuille

drsquoactiviteacutes

Cabinet

drsquoorganisation

Conseils

speacutecifiques

Acqueacuterir une

entreprise

concurrente

(teacuteleacutephonie

mobile)

Strateacutegique

Marketing

Production

(reacuteseau)

Financier

Communication

Uniteacute

Tregraves eacuteleveacute

Leacutegislation

telecom

Tregraves eacuteleveacutee

Direction

Qq mois

Financiegravere

Marcheacute

Reacuteseaux

(ampleur

recouvrement

hellip)

Organisations

Interne

Racheteacutee

Sources

speacutecialiseacutees

Qq mois

Communication

Marcheacute

Cours boursiers

Cabinet

drsquoorganisation

Conseils

speacutecifiques

Initiation au management copy CRCF ndash J Sornet Page 16 48

LE MANAGEMENT EN PRATIQUE

Pour assumer sa fonction le management doit couvrir sans discontinuiteacute lrsquoensemble de

lrsquoorganisation et inteacutegrer de nombreux facteurs dont nous allons reacutesumer lrsquoessentiel

1 ndash Les fonctions et activiteacutes du management

Pour Henri Fayol la fonction drsquoadministration de lrsquoentreprise (son management) reposait sur

cinq actions preacutevoir organiser commander coordonner et controcircler (laquo PO3C raquo)

Nous distinguerons cinq activiteacutes de management

- la conception (au plus haut niveau finaliteacute but ou vocation de lrsquoorganisation

meacutetiers dimension politique de croissance hellip)

- la planification (deacutefinition des objectifs eacutecheacuteances)

- lrsquoorganisation (reacutepartition du travail choix des modes de coordination)

- le pilotage de lrsquoaction opeacuterationnelle (motivation animation encadrement

assistance)

- lrsquoeacutevaluation (controcircle des reacutesultats obtenus ajustements)

Dans chacune de ces activiteacutes des deacutecisions et des arbitrages sont neacutecessaires avec des

enjeux plus ou moins importants

Remarques

- Les cinq activiteacutes du management peuvent se retrouver agrave tout niveau de

management si lrsquoentreprise laisse une certaine autonomie de deacutecision agrave ses diffeacuterentes

uniteacutes La conception est naturellement du ressort de la direction geacuteneacuterale et des

conseils drsquoadministration mais elle peut ecirctre preacutesente pregraves du terrain (latitude laisseacutee agrave

une filiale ou agrave un magasin par exemple) De mecircme lrsquoorganisation du travail concerne

un atelier mais aussi la direction qui structure lrsquoentreprise pour assurer ses activiteacutes sa

production

- La planification deacutefinit des objectifs ou des axes strateacutegiques (choix de produits

modaliteacutes de deacuteveloppement des ventes implantations alliances hellip) et les traduit en

donneacutees de gestion preacutevisionnelles syntheacutetiques et eacutechelonneacutees dans le temps afin de

valider les objectifs et de fixer des repegraveres

- Un laquo business plan raquo (plan drsquoaffaires)est notamment lrsquoeacutequivalent de la planification

dans le cas de creacuteation drsquoentreprise ou pour la preacutesentation de tout projet drsquoactiviteacute

Les activiteacutes du management srsquoinscrivent dans des cycles qui peuvent ecirctre scheacutematiseacute

comme suit (lrsquoeacutevaluation peut entraicircner une reacutevision du pilotage de lrsquoorganisation ou des

objectifs sans que lrsquoentreprise ne soit fondamentalement remise en cause)

conception

planification

organisation

pilotage

eacutevaluation

Initiation au management copy CRCF ndash J Sornet Page 17 48

2 ndash Les contextes de management

Le management est influenceacute par son contexte qui justifie des objectifs une organisation

des meacutethodes

Par exemple lrsquoentreprise admet de nombreuses variantes selon sa taille sa forme juridique

son controcircle par lrsquoeacutetat (entreprises publiques) ou par des inteacuterecircts priveacutes Il en va de mecircme des

organismes administratifs qui peuvent deacutependre de directives nationales ou reacutegionales des

associations qui ont des activiteacutes drsquoampleur tregraves variable

21 ndash La dimension de lrsquoentreprise

La dimension drsquoune entreprise se mesure principalement en fonction de son effectif ou de

son chiffre drsquoaffaires Des seuils sont deacutefinis par divers organismes et exploiteacutes agrave des fins

statistiques ou pour la deacutetermination de certaines obligations sociales ou fiscales

(repreacutesentation du personnel cotisations hellip) Il nrsquoy a bien entendu pas de laquo barriegravere de

tailleraquo absolue conditionnant le management drsquoune entreprise

LrsquoUE preacuteconise de distinguer les micro ndash entreprises (jusqursquoagrave 9 salarieacutes) les TPE ndash tregraves petites

entreprises (moins de 20 salarieacutes) les petites entreprises (moins de 50) et les moyennes

entreprises (de 50 agrave 250) Cependant les PME sont parfois situeacutees entre 10 et 500 salarieacutes

Remarques

- en France environ 40 des entreprises emploient de 1 agrave 50 salarieacutes (ce qui repreacutesente

plus de 50 des emplois) et 59 nrsquoen ont aucun

le pays compte environ 2 600 000 entreprises dont moins de 1 ont 250 employeacutes et

plus

- ancienneteacute et taille de lrsquoentreprise sont lieacutees si lrsquoon eacutecarte les restructurations et autres

eacutevolutions drsquoentreprises existantes

La dimension de lrsquoentreprise a une influence sur lrsquoorganisation et le laquo style raquo de son

management

- les PME sont souvent entrepreneuriales (les dirigeants eacutegalement apporteurs de capitaux

sont totalement engageacutes dans la marche de lrsquoentreprise) Elles ont une gestion flexible peu

formaliseacutee plus qualitative que quantitative Les PME sont freacutequemment focaliseacutees sur un seul

type drsquoactiviteacute Pour ne pas alourdir leur structure elles ont tendance agrave sous-traiter les

activiteacutes speacutecialiseacutees ne correspondant pas agrave leur meacutetier de base

- les grandes entreprises sont manageacuteriales (les dirigeants sont nommeacutes par les actionnaires

en raison de leurs compeacutetences) et moins reacuteactives

22 ndash Le type de production

On distingue industrie (production de biens mateacuteriels ou pour le moins de produits visibles ndash

comme un seacutejour touristique ou un film) et services (fourniture drsquoune prestation immateacuterielle)

Le type de production influence en principe le management de lrsquoentreprise

- lrsquoindustrie neacutecessite (si lrsquoon excepte lrsquoartisanat) un investissement relativement important

une organisation productive stable capable de reacutealiser plusieurs fois des produits identiques

(exemple un modegravele de reacutefrigeacuterateur) ou du moins similaires (exemple un bacirctiment) Le

produit de lrsquoindustrie consomme des matiegraveres et il doit geacuteneacuteralement ecirctre distribueacute jusqursquoau

client

- la production de services peut se satisfaire drsquoun investissement tregraves reacuteduit et neacutecessite un

contact permanent avec le client

Toutefois la standardisation des services et le deacuteveloppement des reacuteseaux informatiques

rapprochent la production de services de celle des biens industriels

- la production drsquoun service reacutepeacutetitif et technique peut imposer une structure lourde et une

organisation tregraves formaliseacutee (voir les grandes socieacuteteacutes drsquoaudit ou de conseil informatique)

Initiation au management copy CRCF ndash J Sornet Page 18 48

- certains services peuvent ecirctre fournis agrave distance sans contact direct avec le client et

distribueacutes par reacuteseau (tenue de comptabiliteacute affacturage gestion clientegravele centre drsquoappel

hellip)

Remarque les services repreacutesentent 75 de lrsquoactiviteacute eacuteconomique franccedilaise

23 ndash La nature de lrsquoorganisation

Les organisations publiques franccedilaises (administrations centrales collectiviteacutes territoriales

hocircpitaux hellip) repreacutesentent une part importante de lrsquoactiviteacute (environ 30 des emplois) La

fonction publique regroupe des organisations aux finaliteacutes diverses et qui ont des problegravemes

de gestion similaires agrave ceux des entreprises auxquelles elles peuvent emprunter des principes

de management Notamment

- pour controcircler les coucircts et assurer la qualiteacute des services

- pour communiquer avec les administreacutes ou les usagers

- pour motiver les personnels et geacuterer les ressources humaines

La transposition directe des techniques de gestion et de management nrsquoest cependant pas

toujours possible car

- la comptabiliteacute publique obeacuteit agrave des regravegles speacutecifiques (proceacutedure budgeacutetaire

notamment)

- le laquo client raquo ne paye pas toujours la prestation du moins directement

- la concurrence est parfois inexistante

- les grandes administrations centraliseacutees sont soumises agrave des choix politiques geacuteneacuteraux

parfois sans connexion eacutevidente avec les besoins opeacuterationnels

- le statut des personnels et les grilles de salaires limitent les possibiliteacutes de gestion des

ressources humaines

Remarque la LOLF (loi organique relative aux lois de finances) est entreacutee en vigueur en

2006 Elle alloue des moyens budgeacutetaires en fonction de programmes et remplace la

reconduction automatique de 90 des budgets Cette reacuteforme se heurte toutefois agrave la

lourdeur des grands ministegraveres ougrave la complexiteacute des activiteacutes est difficile agrave

appreacutehender et ougrave des inerties culturelles peuvent exister agrave tout niveau

Les associations loi de 1901 peuvent avoir une activiteacute comparable agrave celle de grandes

entreprises (voir par exemple les associations de santeacute ou professionnelles) et leur

management est alors similaire malgreacute lrsquoabsence de but lucratif (les beacuteneacutefices ne sont pas

distribuables) Elles ont drsquoailleurs en France un poids eacuteconomique important (elles emploient

environ 1 600 000 salarieacutes)

Cependant lrsquoadheacutesion agrave un systegraveme de valeurs fondateur de lrsquoassociation ou la limite de

lrsquoautoriteacute (quand un volant de beacuteneacutevoles important participe agrave lrsquoactiviteacute) peut introduire des

nuances

- le renforcement des objectifs socieacutetaux

- la faiblesse des relations hieacuterarchiques

- des contraintes de gestion du temps des beacuteneacutevoles

- des modaliteacutes particuliegraveres de recrutement et de motivation des dirigeants

24 ndash Les facteurs contingents

La theacuteorie de la contingence montre qursquoune structure drsquoentreprise nrsquoest efficace que dans

une situation deacutetermineacutee et qursquoil nrsquoexiste que des solutions de management construites dans

un contexte preacutecis

Le management doit ainsi srsquoadapter agrave des facteurs contingents qui ne peuvent ecirctre

controcircleacutes du moins agrave bregraveve eacutecheacuteance Ces facteurs sont par exemple

- lrsquoancienneteacute de lrsquoentreprise (plus elle est ancienne plus lrsquoentreprise a tendance agrave reacutepeacuteter

des comportements eacuteprouveacutes)

Initiation au management copy CRCF ndash J Sornet Page 19 48

- la taille de lrsquoentreprise (la grande entreprise a une composante administrative plus

deacuteveloppeacutee)

- le systegraveme de production (tregraves standardiseacute complexe automatiseacute hellip)

- lrsquoenvironnement

3 ndash Le management et les parties prenantes

Lrsquoentreprise a pour vocation premiegravere de mettre des produits agrave disposition de ses clients en

reacutealisant un profit Pour y arriver elle doit aussi satisfaire ses parties prenantes salarieacutes

actionnaires fournisseurs hellip

Est partie prenante agrave lrsquoentreprise laquo tout groupe ou individu qui peut ecirctre affecteacute ou est

affecteacute par les buts de lrsquoorganisation hellip raquo (Freeman ndash 1984)

Les parties prenantes attendent agrave des degreacutes divers de profiter drsquoune creacuteation de valeur en

provenance de lrsquoentreprise qui doit reacutepondre agrave ces attentes pour assurer sa peacuterenniteacute ou

favoriser son deacuteveloppement

On distingue les parties prenantes primaires ou principales qui sont essentielles agrave lrsquoentreprise

et qui ont geacuteneacuteralement une relation formelle avec elle (clients associeacutes et actionnaires

precircteurs salarieacutes fournisseurs collectiviteacutes) et les parties prenantes secondaires dont

lrsquoinfluence est diffuse (groupes de pression associations meacutedias instances europeacuteennes

agences de notation hellip)

Remarque la consideacuteration de lrsquoensemble des parties prenantes (laquo stakeholders raquo - les

deacutepositaires) fait contrepoids agrave lrsquoimportance accordeacutee aux seuls actionnaires

(laquo shareholders raquo)

Les organisations nrsquoayant pas drsquoobjectif de profit doivent aussi satisfaire leurs parties

prenantes apporter un service aux usagers dans les meilleures conditions eacuteconomiques

limiter un budget assurer la qualiteacute des relations avec les fournisseurs hellip

Dans cette optique le management doit organiser lrsquoaction de faccedilon agrave eacutequilibrer des forces

parfois divergentes

- le contexte fait pression sur lrsquoorganisation contrainte agrave optimiser ses reacutesultats

- lrsquoorganisation cherche par son action agrave assurer sa peacuterenniteacute son deacuteveloppement (en

reacutealisant des profits dans le cas de lrsquoentreprise) et agrave satisfaire ses parties prenantes

- le management agit en pilotant les actions pour contrebalancer la pression du contexte

Actions de

lrsquoorganisation

Management Contexte

Parties

prenantes

Initiation au management copy CRCF ndash J Sornet Page 20 48

APPLICATIONS MP

MP1 Deacutefinir contingent gestion budgeacutetaire

MP2 Deacuteterminer les parties prenantes drsquoun hocircpital public et leurs principales attentes

Mecircme question pour les organisations suivantes

- SNCF (entreprise publique)

- Peugeot

- MAIF (mutuelle drsquoassurance)

MP3 En les situant dans le cycle des activiteacutes du management trouver les actions agrave mener

dans les situations suivantes

- baisse de 10 des ventes dans une entreprise industrielle (produits meacutenagers le reacuteseau de

distribution vient drsquoecirctre reacuteorganiseacute)

- idem dans une entreprise de vente par correspondance soumise agrave la concurrence internet

(les ventes stagnaient depuis six mois malgreacute les efforts promotionnels)

- augmentation des deacutelais drsquoattente des consultations dans une clinique (lrsquohocircpital voisin a

fermeacute son service drsquourgences)

Initiation au management copy CRCF ndash J Sornet Page 21 48

ELEMENTS DE CORRIGE MP

MP1 Deacutefinir (dans le contexte drsquoune entreprise) contingent gestion budgeacutetaire

Contingent = imposeacute par lrsquoexteacuterieur Contingence = effet du hasard de la rencontre de

plusieurs eacuteveacutenements indeacutependants (variables explicatives que lrsquoon ne peut influencer)

Gestion budgeacutetaire = technique drsquoadministration des entreprises srsquoappuyant sur des

preacutevisions dont on deacuteduit apregraves accord des responsables des attributions de moyens sur une

dureacutee limiteacutee Une analyse reacuteguliegravere des eacutecarts entre preacutevisions et reacutealisations permet ensuite

le pilotage des activiteacutes Le budget est un cadre incitatif

La laquo planification budgeacutetaire raquo consiste agrave traduire en budgets une planification strateacutegique

avec systegraveme de reporting

MP2 Deacuteterminer les parties prenantes drsquoun hocircpital public et leurs principales attentes

Mecircme question pour les organisations suivantes

- SNCF (entreprise publique)

- Peugeot

- MAIF (mutuelle drsquoassurance)

Hocircpital

- patients (qualiteacute des soins)

- CNAM (baisse des coucircts)

- collectiviteacute locale (service aux administreacutes)

- eacutetat (ameacutenagement du territoire maicirctrise des budgets optimisation)

- employeacutes (salaire conditions de travail et satisfaction)

- fournisseurs ndash pharmacie autres (CA paiement reacutegulier)

- associations de patients (qualiteacute proximiteacute des soins)

SNCF

- usagers et associations drsquousagers (proximiteacute reacutegulariteacute prix du service)

- reacuteseau ferreacute de France (optimisation des lignes paiement adapteacute)

- fournisseurs (CA paiement reacutegulier)

- employeacutes (salaire conditions de travail seacutecuriteacute de lrsquoemploi)

- eacutetat (ameacutenagement du territoire)

- collectiviteacutes locales (service)

Peugeot

- clients (qualiteacute prix SAV relation commerciale)

- fournisseurs (CA reacutegulariteacute de lrsquoactiviteacute)

- employeacutes (salaire conditions de travail seacutecuriteacute de lrsquoemploi)

- eacutetat (taxes)

- collectiviteacute locale (emploi dynamisation eacuteconomique preacuteservation de lrsquoenvironnement)

- associations de protection de lrsquoenvironnement (activiteacute propre baisse des eacutemissions

nouvelles eacutenergies)

MAIF

- socieacutetaires (protection relation assureur tarif mesureacute)

- professionnels de lrsquoautomobile et autres (agreacutement marge de manœuvre reacuteparations tarifs

eacuteleveacutes)

- fournisseurs (CA paiement reacutegulier)

- eacutetat (taxes engagement pour la seacutecuriteacute)

- employeacutes (salaire conditions de travail seacutecuriteacute de lrsquoemploi)

Initiation au management copy CRCF ndash J Sornet Page 22 48

MP3 En les situant dans le cycle des activiteacutes du management trouver les actions agrave mener

dans les situations suivantes

- baisse de 10 des ventes dans une entreprise industrielle (produits meacutenagers le reacuteseau de

distribution vient drsquoecirctre reacuteorganiseacute)

Adapter le pilotage motiver cadrer si insuffisant retoucher une organisation deacutefectueuse

- idem dans une entreprise de vente par correspondance soumise agrave la concurrence internet

(les ventes stagnaient depuis six mois malgreacute les efforts promotionnels)

Voir pilotage et organisation si une eacutevolution du meacutetier a deacutejagrave eacuteteacute initialiseacutee Sinon re-

conception (adaptation au nouveau contexte) puis planification et reacuteorganisation

- augmentation des deacutelais drsquoattente des consultations dans une clinique (lrsquohocircpital voisin a

fermeacute son service drsquourgences)

Organisation Si insuffisant planification (nouveaux objectifs)

Initiation au management copy CRCF ndash J Sornet Page 23 48

ORGANISATION ET PROCESSUS

La performance de lrsquoentreprise deacutepend de son organisation et de son aptitude agrave produire

aux meilleures conditions Nous allons montrer comment organisation formelle et processus

de production peuvent contribuer agrave cette performance

1 ndash Vers lrsquooptimum

11 ndash Les eacuteconomies occidentales jusqursquoaux anneacutees 70

Jusqursquoen 1945 le principal problegraveme des entreprises eacutetait de produire des biens en quantiteacute

suffisante agrave un prix compatible avec le marcheacute Les grandes entreprises se sont multiplieacutees et

la standardisation a permis de reacuteduire les coucircts (exemple deacuteveloppement de Ford et de la

production agrave la chaicircne de 1908 agrave 1920 qui a permis une baisse du prix des voitures des 23)

On parle de laquo production pousseacutee vers le marcheacute raquo

Cette croissance de la production peu reacuteguleacutee a eacuteteacute marqueacutee par des surproductions en

1910 et 1920 puis par la crise de 1929 qui a prolongeacute ses effets jusqursquoagrave la guerre

De 1945 agrave 1975 environ (les laquo trente glorieuses raquo) la reconstruction la croissance de la

consommation de masse de nouvelles technologies et les eacutechanges internationaux

alimentent lrsquoeacuteconomie La standardisation srsquoeacutetend aux biens de consommation dont les

coucircts baissent fortement et de nouvelles reacutegulations sociales permettent une eacutevolution sans

heurt des revenus La saturation de certains marcheacutes conduit dans les anneacutees 60 agrave la

deacutemarche laquo marketing raquo et agrave la diffeacuterenciation des produits Le produit est laquo dirigeacute par le

marcheacute raquo mais les entreprises conservent une organisation assez classique et les plus grosses

srsquointernationalisent

12 ndash Lrsquoexpeacuterience japonaise et ses prolongements

Tregraves tocirct apregraves la guerre dans un Japon appauvri le constructeur automobile Toyota a ducirc

faire face agrave une restriction du marcheacute des moyens financiers et productifs et des

approvisionnements La firme a donc innoveacute dans un nouveau systegraveme de production

chassant les laquo gaspillages raquo (temps drsquoattente transports stocks deacutefauts hellip) consideacuterant que

seule la fabrication vendable creacutee de la valeur

Toyota srsquoorganise pour fabriquer la quantiteacute et la qualiteacute de produits juste neacutecessaires agrave la

satisfaction des clients la production est laquo tireacutee par le marcheacute raquo La mise en place de ce

systegraveme qui integravegre les fournisseurs ne sera acheveacutee que dans le milieu des anneacutees 70

En 1973 la hausse du peacutetrole inaugure un ralentissement de la croissance des eacuteconomies

occidentales La concurrence accrue provoque alors un inteacuterecirct pour le systegraveme deacuteveloppeacute

au Japon La production au plus juste se deacuteveloppe ainsi dans lrsquoindustrie automobile agrave partir

des anneacutees 80 et elle se reacutepand encore maintenant dans drsquoautres secteurs

Cette approche qui vise un objectif de zeacutero stock et zeacutero deacutefaut impose la maicirctrise de laquo bout

en bout raquo des processus de production et leur ameacutelioration

Initiation au management copy CRCF ndash J Sornet Page 24 48

2 ndash Organiser lrsquoentreprise

21 ndash Direction et organisation

Diriger une entreprise neacutecessite de lrsquoorganiser (de reacutepartir les tacircches) pour qursquoelle puisse

atteindre ses objectifs Lrsquoorganisation permet de satisfaire un marcheacute en tirant parti des

capaciteacutes actuelles de lrsquoentreprise tout en preacuteparant lrsquoavenir

Lrsquoorganisation reacutesulte freacutequemment drsquoun compromis entre des objectifs situeacutes agrave des niveaux

et des eacutecheacuteances diffeacuterents

Exemples

- le leader des chaises roulantes peut tirer profit de sa structure productive et de son

savoir faire pour entrer sur le marcheacute de la bicyclette eacutelectrique

- ecirctre parfaitement structureacute pour alimenter 90 du marcheacute des disquettes ne preacutepare

pas lrsquoavenir

- srsquoorganiser pour conqueacuterir le marcheacute des tire-bouchons eacutelectriques dans les deux ans

perd de son sens si cela altegravere les moyens neacutecessaires agrave la production drsquoappareils

manuels ancienne mais vitale dont la diminution agrave court terme risque de nuire agrave la

solvabiliteacute de lrsquoentreprise et de la conduire agrave la cessation de paiement

22 ndash Lrsquoorganisation fonctionnelle

La majoriteacute des entreprises adopte une laquo organisation fonctionnelle raquo (celle qui est visible

dans les organigrammes) ougrave des regroupements de personnels et drsquoeacutequipements se font

selon un modegravele hieacuterarchique (laquo line raquo) dans des uniteacutes des services ou des deacutepartements

speacutecialiseacutes Cette organisation peut se deacutecliner agrave lrsquointeacuterieur des divisions des grandes

entreprises quand elles scindent leur activiteacute par zone geacuteographique type drsquoactiviteacute

cateacutegorie de clients hellip

Remarque le terme laquo fonction raquo deacutesigne un rocircle particulier dans le fonctionnement de

lrsquoentreprise

Lrsquoorganisation fonctionnelle diffeacuterencie les activiteacutes de lrsquoentreprise en les regroupant par

meacutetier pour utiliser au mieux les compeacutetences et les moyens (meilleur rendement par la

speacutecialisation lrsquoeacutechange de compeacutetences dans une mecircme uniteacute ou gracircce agrave des eacuteconomies

drsquoeacutechelle)

23 ndash La notion de processus de production

Un processus de production se deacutefinit par la succession drsquoactiviteacutes permettant de satisfaire

un client en transformant des ressources (mateacuterielles financiegraveres humaines) en un produit

bien ou service Le processus doit creacuteer une valeur reconnue par le client

Un processus peut servir un client interne agrave lrsquoentreprise (par exemple en produisant un

composant intervenant dans plusieurs produits ou par la maintenance des machines) aussi

bien qursquoun client final On distingue usuellement

- les processus opeacuterationnels (ou maicirctres) aussi appeleacutes processus meacutetier (business process)

qui satisfont directement les clients finaux (conception et fabrication de produits vente hellip)

- les processus de support et de management (geacuterer les ressources humaines geacuterer

lrsquoinformation geacuterer les ressources financiegraveres hellip) qui ont les processus opeacuterationnels comme

clients

Toutes les actions internes agrave une organisation peuvent srsquointeacutegrer dans des processus qui

conditionnent directement ou indirectement la capaciteacute de lrsquoorganisation agrave satisfaire le

client final ou lrsquousager

Initiation au management copy CRCF ndash J Sornet Page 25 48

Aborder le fonctionnement de lrsquoentreprise par ses processus (approche processus) permet

de mettre en eacutevidence les chaicircnes drsquoactiviteacutes qui conduisent aux produits leurs

dysfonctionnements leurs coucircts la formation des deacutelais et la souplesse (la flexibiliteacute)

disponible pour satisfaire la clientegravele finale Lrsquoameacutelioration des processus a un impact visible

et direct sur chaque produit proposeacute aux clients

Lrsquoapproche processus provoque une eacutevolution de la faccedilon de travailler

- en faisant peacuteneacutetrer la laquo voix du client raquo au plus profond de lrsquoentreprise (et plus seulement

dans les services commerciaux et marketing)

- en mettant en eacutevidence des possibiliteacutes de rationalisation (par regroupement ou impartition

de certaines activiteacutes)

Remarque lrsquoapproche par les activiteacutes et les processus est agrave lrsquoorigine de la meacutethode

de deacutetermination des coucircts laquo ABC raquo - activity based costing

24 ndash Processus et fonctions

Le processus est transversal Il enchaicircne des activiteacutes qui traversent lrsquoentreprise en particulier

les services ou les deacutepartements drsquoune organisation fonctionnelle

Exemple

La division du travail par fonctions induit une charge de coordination pour assurer le

deacuteroulement du processus Elle peut geacuteneacuterer des attentes des erreurs ou des conflits drsquointeacuterecirct

(lrsquoobservation montre que des dysfonctionnements sont tregraves souvent constateacutes lors du

passage drsquoun service agrave un autre)

Organisation fonctionnelle et approche processus visent toutes deux un optimum

eacuteconomique mais leurs logiques sont diffeacuterentes

- le processus vise la satisfaction des clients (prix qualiteacute deacutelais service)

- le deacutecoupage fonctionnel cherche agrave optimiser les moyens (maximiser lrsquoeffet drsquoexpeacuterience

partager des infrastructures profiter de pocircles de compeacutetences hellip) Il apporte une ossature

hieacuterarchique stable souvent indispensable

Organisation fonctionnelle et approche processus sont donc compleacutementaires dans la

majoriteacute des cas et doivent ecirctre combineacutees judicieusement

APPLICATIONS OP

OP1 Deacutefinir flexibiliteacute systegraveme impartition

OP2 Citer huit exemples drsquoinformations essentielles pour optimiser un processus de

fabrication

Direction

Deacutepartement

commercial

(C)

Deacutepartement

administratif et

financier (AF)

Deacutepartement

Etudes (E)

Deacutepartement

Production (P)

Activiteacute

C-x Activiteacute

AF-x Activiteacute

E-x

Activiteacute

P-x

Processus x

Clie

nt

Initiation au management copy CRCF ndash J Sornet Page 26 48

OP3 Deacutegager les principes du toyotisme preacutesenteacute ci-dessous En quoi ce systegraveme est-il

initiateur de lrsquoapproche processus

Taiichi Ohno et le Toyotisme

1 - Extrait drsquoun article de Jacques BARRAUX - 1993 - LExpansion

Taiichi Ohno (1912 ndash 1990) hellip ne se prenait pas pour un visionnaire mais en imposant une

nouvelle faccedilon de produire il a reacuteinventeacute le management hellip tout le monde a entendu parler

des mots qui ont populariseacute le toyotisme dont il est le pegravere le juste-agrave-temps hellip Autant

doutils conccedilus pour lrsquoautomobile et qui ont aujourdhui une application universelle

hellip Taiichi Ohno jeune ingeacutenieur entre chez Toyota alors simple constructeur de machines

textiles Degraves 1926 apparaicirct la notion de jidoka hellip cest lart de transfeacuterer de lintelligence aux

machines pour mieux libeacuterer lintelligence des hommes Tout le contraire du taylorisme qui

juge la machine moins impreacutevisible que lhomme En 1933 Toyota se lance dans lautomobile

en sinspirant des meacutethodes ameacutericaines Mais en 1935 agrave loccasion dun voyage aux Etats-

Unis leacutetat-major de lentreprise revient fascineacute de sa visite dans un supermarcheacute La notion

de juste-agrave-temps va naicirctre de lobservation dune grande surface un lieu ougrave les clients ne

prennent que ce dont ils ont besoin et ougrave les rayons sont reacuteapprovisionneacutes pour compenser

les quantiteacutes preacuteleveacutees Ainsi le systegraveme Toyota est-il deacutejagrave dans la tecircte de ses dirigeants avant

mecircme la Seconde Guerre mondiale un demi-siegravecle avant la reacutevolution informatique et la

segmentation intensive des marcheacutes

hellip des esprits curieux comme Franccedilois Dalle en France tombent alors sous le charme des

formules et des paraboles de Taiichi Ohno En voici deux eacutechantillons

Penser agrave lenvers Cela signifie combattre les ideacutees reccedilues En lespegravece il sagit du fordisme et

du taylorisme Ohno ne croit pas agrave la planification aux effets deacutechelle et dexpeacuterience Il

propose un systegraveme industriel agrave lenvers qui permette de diversifier les produits et de les

fabriquer en petites quantiteacutes Nous ne devons plus ecirctre des paysans qui accumulent des

stocks mais des chasseurs On nimpose pas loffre On traque la demande et on la gegravere en

continu

Que les valleacutees soient hautes et les montagnes peu eacuteleveacutees Plutocirct que de concentrer tous

les efforts sur une production agrave un moment donneacute mieux vaut se doter de structures flexibles

permettant de passer agrave tout instant dune seacuterie agrave une autre Il faut eacuteviter les ruptures et les

secousses aplanir les cycles entretenir des flux reacuteguliers dactiviteacutes diversifieacutees Ce qui

implique de ne pas enfermer les hommes et les eacutequipements dans des speacutecialisations trop

eacutetroites

La flexibiliteacute le travail en groupe le refus de la dictature des machines la polyvalence et

surtout lattention constante aux signaux eacutemis par le marcheacute nappartiennent plus au

toyotisme Ces notions sont les fondements du nouvel art dorganiser de vendre et de

produire dans lindustrie comme dans les services hellip

2 - Quelques notions cleacutes

Taiichi Ohno a imagineacute la meacutethode des laquo cinq pourquoi raquo qui consiste agrave se poser cinq fois de

suite la question laquo pourquoi raquo sur le mecircme sujet de faccedilon agrave deacutecouvrir la veacuteritable cause

drsquoun problegraveme Cette meacutethode peut ecirctre appliqueacutee agrave tous les niveaux et permettre

notamment aux agents de fabrication de proposer de veacuteritables ameacuteliorations de la

production

La recherche de la qualiteacute totale (pas de deacutefaut des produits pas de rebuts pas de deacutefaut

des processus) accompagne la deacutemarche de Toyota La qualiteacute a un coucirct compenseacute par

des ventes accrues par lrsquoeacuteconomie des mesures palliatives aux deacutefauts

Initiation au management copy CRCF ndash J Sornet Page 27 48

Fiche OP1 ndash Benchmarking et processus

Le laquo benchmarking raquo consiste agrave comparer le fonctionnement de plusieurs systegravemes pour en

faire notamment ressortir les meilleures pratiques (laquo best practices raquo) Cette technique est

utiliseacutee depuis les anneacutees 80 pour ameacuteliorer la performance des entreprises Elle impose agrave

lrsquoentreprise drsquoeacutevaluer et de remettre en question ses propres modes de fonctionnement afin

de les faire eacutevoluer agrave la lueur de ce qui se fait ailleurs

Le benchmarking permet drsquoameacuteliorer les processus agrave moindre risque en fixant des objectifs

baseacutes sur des faits et donc plus facilement accepteacutes

Une classification des processus en tant que base de reacuteflexion a eacuteteacute eacutetablie aux USA par

lrsquolaquo International Benchmarking Clearinghouse raquo de lrsquoAPQC (american productivity and

quality center) en collaboration avec plusieurs dizaines drsquoentreprises

Elle se reacutesume ainsi

Le terme laquo reengineering raquo (la re-conception ou laquo reacuteingeacutenieacuterie raquo) des processus deacutesigne un

projet drsquoameacutelioration radicale des performances (de 20 agrave 50 ou plus) Il neacutecessite une

parfaite adheacutesion de la direction la constitution drsquoune petite eacutequipe de projet brillante

connaissant parfaitement les activiteacutes de lrsquoentreprise et il peut inclure un benchmarking

Le reengineering provoque geacuteneacuteralement la reacuteduction du nombre de niveaux hieacuterarchiques

(laquo delayering raquo) et lrsquoaccroissement du pouvoir de deacutecision des employeacutes (laquo empowerment raquo

ou laquo empouvoirement raquo) Bien qursquoy conduisant parfois il ne doit pas ecirctre confondu avec la

reacuteduction des activiteacutes (laquo downsizing raquo ou restructuration) et lrsquoexternalisation (laquo outsourcing raquo)

Pro

ce

ssu

s

op

eacutera

tio

nn

els

Pro

ce

ssu

s d

e m

an

ag

em

en

t e

t d

e

sup

po

rt

1 ndash

Comprendre

le marcheacute et

les clients (besoins

satisfaction)

2 ndash

Deacutevelopper

vision et

strateacutegie (contexte

concurrence)

3 ndash

Creacuteer

produits

services

processus

(concevoir

ameacuteliorer)

4 ndash

Marketing et

vente

5 ndash

Produire et

livrer (industrie

dont

ameacutelioration

processus)

6 ndash

Produire et

livrer (services)

7 ndash

Facturer et

servir les

clients (apregraves-

vente

reacuteclamations)

8 ndash Deacutevelopper et geacuterer les ressources humaines

9 ndash Geacuterer les systegravemes drsquoinformation

10 ndash Geacuterer les ressources financiegraveres et les actifs

11 ndash Appliquer un programme environnemental

12 ndash Geacuterer les relations exteacuterieures (actionnaires banques lois relations publiques hellip)

13 ndash Geacuterer lrsquoameacutelioration et le changement (eacutevaluer mesurer motiver qualiteacute totale)

Initiation au management copy CRCF ndash J Sornet Page 28 48

Fiche OP2 ndash Lrsquoorganisation par processus

Lrsquoeacutevolution drsquoune organisation aux activiteacutes reacutepeacutetitives vers lrsquoapproche processus est

geacuteneacuteralement progressive et se met en place par paliers

La mise en œuvre drsquoun veacuteritable management par processus doit ecirctre preacuteceacutedeacutee quand

lrsquoactiviteacute de lrsquoentreprise est complexe drsquoun recensement (une laquo cartographie des

processus raquo) pour mettre en eacutevidence les processus ou les familles de processus cleacutes critiques

pour le succegraves de lrsquoentreprise ougrave les efforts seront prioritaires

Des responsables de processus (laquo process owners raquo) sont ensuite deacutesigneacutes

Le responsable doit concevoir ses processus puis apregraves leur mise en œuvre assurer les

coordinations neacutecessaires les ameacuteliorer et les repreacutesenter aupregraves de la direction

Quand une structure par processus est mise en place des opeacuterateurs exeacutecutants

preacuteceacutedemment regroupeacutes dans les fonctions peuvent ecirctre affecteacutes aux processus et

drsquoanciens responsables de fonctions peuvent devenir des experts au service des processus

Lrsquoorganisation par processus peut imposer un degreacute eacuteleveacute drsquointeacutegration des activiteacutes donc

une polyvalence accrue des personnels et une reacuteduction des niveaux hieacuterarchiques

Elle neacutecessite pour le moins des compeacutetences eacutelargies au niveau des responsables de

processus (organisation administration technique hellip) dont le nombre doit rester limiteacute

(quelques dizaines au plus)

Sauf dans de tregraves petites structures lrsquoorganisation par processus se plaque geacuteneacuteralement sur

une structure plus classique

Initiation au management copy CRCF ndash J Sornet Page 29 48

ELEMENTS DE CORRIGE OP

OP1 Deacutefinir

Flexibiliteacute = adaptation au besoin (horaire variable chaicircnes robotiseacutees)

Systegraveme = ensemble organiseacute dans un but boicircte noire (sanguin nerveux meacutetrique laquo D raquo)

Impartition = sous-traitance ou externalisation (seacuteparation) drsquoactiviteacutes faire appel agrave des

partenaires plutocirct que faire soi-mecircme

OP2 Citer huit exemples drsquoinformations essentielles pour orienter lrsquooptimisation drsquoun processus

Montant des stocks (approvisionnements et produits finis)

Temps drsquoattente

Taux drsquoactiviteacute des ateliers

Rebuts

Deacutelai de production

Taux de reacuteclamations clients (qualiteacute)

Temps passeacutes en retouches finales

Turn over

Nombre drsquoarrecircts maladie

Accidents du travail

Dureacutee des arrecircts machines

OP3 Deacutegager les principes du toyotisme preacutesenteacute dans la fiche 31 En quoi ce systegraveme

repose trsquoil sur lrsquoapproche processus

Produire la quantiteacute juste neacutecessaire (agrave la demande) donc eacuteviter les stocks

Flexibiliteacute intelligence des chaicircnes de production

Qualiteacute (eacuteviter le coucirct de la non-qualiteacute)

La notion de processus est implicite ainsi que la chaicircne de valeur client

Initiation au management copy CRCF ndash J Sornet Page 30 48

DEFIS ET TENDANCES DU MANAGEMENT

Les meacutethodes de management se deacuteveloppent pour affronter le contexte eacuteconomique

Ce chapitre preacutesente les deacutefis auxquels le management contemporain doit faire face

1 ndash Lrsquoeacutevolution eacuteconomique contemporaine

A mesure que lrsquoactiviteacute eacuteconomique mondiale srsquoaccroicirct que la technologie eacutevolue les

changements sont de plus en plus rapides Ils introduisent des situations ineacutedites auxquelles les

entreprises doivent srsquoadapter en cherchant de nouvelles solutions de management Les trois

derniegraveres deacutecennies ont eacuteteacute notamment marqueacutees par les pheacutenomegravenes suivants (que nous

listons sans tenir compte des liens pouvant exister entre eux)

Pheacutenomegravene Traduction Effets

Deacute reacuteglementation

globalisation

financiegravere

titrisation

Libre circulation des capitaux accegraves

facile des particuliers au marcheacute

boursier (directement ou par

lrsquointermeacutediaire des OPCVM et SICAV)

Monteacutee en puissance du financement

des entreprises sur le marcheacute boursier

Fonds de pension

(retraites) et fonds

souverains (eacutetats)

Poids boursier important drsquoinvestisseurs

institutionnels qui cherchent un haut

rendement financier (dividendes ou

valorisation boursiegravere)

Pression sur les grandes entreprises

influence sur les strateacutegies

Mondialisation Liberteacute des eacutechanges internationaux Accroissement de la concurrence

recherche drsquoavantages eacuteconomiques

par la deacutelocalisation (biens et

services) la concentration des efforts

(recentrage) problegravemes drsquoemploi

multiplication des transports perte

drsquoinfluence des politiques

Baisse de lrsquoemploi

occidental

(notamment

industriel)

Moins de fabrications fabrications

automatiseacutees recours aux moyens

informatiques

Activiteacute reporteacutee sur le commerce la

conception et les services chocircmage

charge sociale

Restructurations Optimisation des entreprises

abaissement des coucircts augmentation

des marges recherche drsquoune taille

critique (eacuteconomies drsquoeacutechelle poids

sur le marcheacute)

Recentrages externalisations fusions

deacutelocalisations constitution de grands

groupes

NTIC (nouvelles

technologies de

lrsquoinformation et de

la communication)

Mise en œuvre des reacuteseaux (dont

internet) et drsquoapplications

informatiques communicantes

Nouvelles formes de commerce

marcheacute international deacutelocalisation

du travail intellectuel reacuteorganisation

de la distribution

Rareacutefaction relative

des matiegraveres

premiegraveres

Recherche de substituts exploration

miniegravere coucircts drsquoexploitation des

gisements accrus

Augmentation des coucircts variations

erratiques du cours des matiegraveres

deacutestabilisations politiques

Evolution

geacuteopolitique et

eacuteconomique

mondiale

Chute de lrsquoURSS transformation des

eacuteconomies collectivistes pays

eacutemergents (Chine Inde Breacutesil Russie)

()

Accroissement de la population

mondiale (4 agrave 6 7 milliards de 1970 agrave

2008)

Libeacuteralisme sans frein () nouvelles

puissances eacuteconomiques

opportuniteacutes de deacuteveloppement

nouveau partage des ressources

ineacutegaliteacutes baisse du soutien aux PVD

laquo Terrorisme raquo Actions armeacutees pression de groupes

armeacutes non gouvernementaux

Deacutestabilisations reacutegionales charge

des deacutepenses militaires

Deacuteveloppement

durable

Recherche drsquoune croissance eacutequitable

et respectueuse de lrsquoenvironnement

Pression sur les entreprises (eacutetats

associations de consommateurs

eacutecologistes ONG)

() Reacutecemment quelques affaires (Enron laquo subprimes raquo Vivendi Universal Socieacuteteacute

Geacuteneacuterale Airbus par exemple) et agrave plus grande eacutechelle la crise financiegravere de 2008 ont

montreacute les dangers drsquoune libeacuteralisation sans controcircles suffisants

Initiation au management copy CRCF ndash J Sornet Page 31 48

() Des alliances eacuteconomiques naissent entre pays eacutemergents (notamment en

ameacuterique centrale creacuteation de la Banque du Sud en 2008 par exemple) et lrsquoon

commence agrave imaginer une baisse progressive de lrsquoinfluence eacuteconomique des Etats

Unis

2 ndash Les deacutefis actuels du management

21 ndash Les grandes orientations

Lrsquoeacutevolution eacuteconomique suggegravere quelques pistes parfois concurrentes pour lrsquoaction du

manager contemporain On y retrouve au premier plan la construction drsquoune vision qui est

une composante commune du leadership

Objectif du manager

pour lrsquoentreprise

Justification Facteurs de reacuteussite

Construire une vision Eclairer lrsquoavenir de lrsquoentreprise partager

un but souder motiver

Effort de reacuteelle prospection

volontarisme de la direction

bonne communication

Reacuteactiviteacute et flexibiliteacute

(sous tous les aspects

agrave tous niveaux)

Srsquoadapter rapidement au marcheacute Bonne organisation des processus

personnel compeacutetent autonome

et motiveacute structure hieacuterarchique

alleacutegeacutee robotisation

Deacutegager des profits Reacutemuneacuterer les apporteurs de capitaux

srsquoautofinancer

Ajuster coucircts et structures

Exploiter les nouvelles

technologies

Reacuteactiviteacute ajuster coucircts et deacutelais

reacutepondre au marcheacute suivre les clients

Organiser le SI de faccedilon

pertinente eacuteviter le coucirct excessif

drsquoinvestissements trop en

laquo pointe raquo (laquo essuyer les placirctres raquo)

utiliser judicieusement les services

exteacuterieurs

Bacirctir des alliances

(contrats fusions)

Deacutevelopper une activiteacute limiter les coucircts

de transaction () atteindre la taille

critique et de meilleurs rendements se

recentrer sur une activiteacute profitable

Dominer les processus se donner

une identiteacute lisible externaliser se

doter drsquoune capaciteacute financiegravere

suffisante

Valoriser lrsquoimage Attirer les clients favoriser les alliances

donner confiance (apporteurs de fonds

employeacutes clients partenaires socieacuteteacute

civile)

Instaurer des regravegles de

gouvernance inteacutegrer le

deacuteveloppement durable

respecter lrsquoenvironnement

Geacuterer les risques Faire face aux aleacuteas eacuteconomiques et

technologiques (conjoncture politiques

accidents malveillance)

Creacuteer un systegraveme drsquoalerte geacuterer

la crise (reacuteaction raisonneacutee

sceacutenarios poursuite de

lrsquoexploitation dans un contexte

instable) mise en place de

proceacutedures drsquoapprentissage pour

ameacuteliorer les reacuteactions au fil du

temps

Geacuterer le changement Faire face agrave lrsquoeacutevolution de la demande

la pression sur les prix la variation des

performances financiegraveres la

concurrence la globalisation des

marcheacutes lrsquoeacutevolution technologique aux

fusions ou alliances aux changements

de reacuteglementation de direction hellip ()

Bonne communication pour

donner du sens au changement

et obtenir lrsquoadheacutesion du personnel

Rassembler et geacuterer les

connaissances former le

personnel

Innover Garder un avantage concurrentiel se

diffeacuterencier

Veille technologique et

commerciale investissement

Ouverture

internationale

Elargir le marcheacute saisir les opportuniteacutes Veille commerciale partenariats

() La theacuteorie des coucircts de transaction deacuteveloppeacutee par OE Williamson dans les

anneacutees 70 integravegre les coucircts lieacutes au recours au marcheacute (recherche et choix drsquoun

fournisseur neacutegociation reacutedaction de contrat suivi des eacutechanges risque de rupture

Initiation au management copy CRCF ndash J Sornet Page 32 48

drsquoapprovisionnement hellip) On peut en conclure que lrsquointeacutegration de diffeacuterentes

activiteacutes agrave lrsquoentreprise (la laquo firme raquo) preacutesente des avantages Mais des coucircts de

transaction internes doivent aussi ecirctre consideacutereacutes (preacuteparation organisation

surveillance hellip) et certaines formes de coopeacuteration continue avec les fournisseurs

permettent de reacuteduire le coucirct des transactions externes

() drsquoapregraves laquo Les meilleures pratiques de management raquo - Brilman Heacuterard ndash EO

Une eacutetude du Conference Board (2002) liste les deacutefis du management vus par 700 leaders

mondiaux Soit en reacutesumeacute avec indication du score correspondant

1 ndash Fideacuteliser les clients (42)

2 ndash Reacuteduire les coucircts (38)

3 ndash Accroicirctre flexibiliteacute et reacuteactiviteacute (29)

4 ndash Amener les employeacutes agrave adheacuterer aux valeurs et visions de lrsquoentreprise (26)

5 ndash Deacutevelopper et retenir les leaders (25)

6 ndash Geacuterer acquisitions et alliances (24)

7 ndash Accroicirctre lrsquoinnovation (20)

En fin de classement citoyenneteacute et reacuteputation (4) et ameacutelioration de la diversiteacute (3)

22 ndash Les techniques disponibles

Pour faire face aux deacutefis le manager dispose de nouveaux concepts et de nouvelles

techniques Le tableau ci-dessous en donne un reacutesumeacute et indique les domaines qursquoils

influencent principalement

Initiation au management copy CRCF ndash J Sornet Page 33 48

Principaux concepts techniques outils Incidence principale sur

Internet

- e-commerce (commerce eacutelectronique site

entreprise)

- CRM ou GRC (gestion de la relation client)

- e-procurement (gestion des approvisionnements

par le reacuteseau)

- messagerie eacutelectronique

- e-recrutement

Vente accegraves au marcheacute

Relation client reacuteactiviteacute personnalisation

fideacutelisation

Deacutelais coucircts

Communication transfert de donneacutees (piegraveces

jointes) tous domaines

Communication recrutement

Intranet reacuteseau drsquoentreprise SI

- knowledge management (gestion des

connaissances)

- e-learning (apprentissage en ligne)

- plateforme de travail collaboratif (groupware)

- workflow (circulation eacutelectronique de

documents enchaicircnement de processus)

- e-RH portail RH (libre accegraves aux postes agrave

pourvoir informations candidatures hellip)

- PGI (progiciel de gestion inteacutegreacute) ou ERP

Innovation capaciteacute au changement veille

documentaire

Formation du personnel accompagnement des

changements

Coordination communication interne

Coordination

Communication interne (voire internet en

externe) reacuteduction des coucircts climat drsquoentreprise

recrutement plans de carriegraveres hellip

Coucircts fiabiliteacute du systegraveme drsquoinformation deacutelais

processus (continuiteacute inteacutegration)

Logistique inteacutegreacutee

Supply Chain Management (SCM) gestion de la

logistique (incluant les approvisionnements)

Processus deacutelais coucircts

Externalisation

Valorisation du capital humain

GPEC (gestion preacutevisionnelle des emplois et

compeacutetences)

Coaching

Reacuteactiviteacute de lrsquoentreprise conservation des

compeacutetences rendements individuels turn-over

adaptation des compeacutetences motivation

Efficaciteacute individuelle controcircle reacutegulation

progregraves processus

Approche processus

Optimisation des processus

Deacutemarche qualiteacute totale (TQM ndash total quality

management)

Empowerment (empouvoirement)

Benchmarking reacuteingeacutenieacuterie

Coucircts marges qualiteacute deacutelais flexibiliteacute

externalisation eacutelargissement des compeacutetences

organisation

Ameacutelioration des processus (meacutetiers et supports)

Autonomie compeacutetences des employeacutes

Ameacutelioration des processus restructuration

Management par la valeur

Parties prenantes

Satisfaction des parties prenantes financement

motivation collaborations hellip

Collaboration inter organisations

Reacuteseaux drsquoentreprises alliances

EDI (eacutechange de donneacutees informatiseacutees) extranet

Impartition externalisation (outsourcing)

Coucircts recentrage investissements lancement

drsquoactiviteacute

Coucircts reacuteactiviteacute deacutelais relations avec

lrsquoadministration

Coucircts recentrage limitation des investissements

Ethique drsquoentreprise

Gouvernance drsquoentreprise (mode de direction

encadreacute par des regravegles)

Rocircle socieacutetal deacuteveloppement durable

environnement

Image de lrsquoentreprise reacutegulation du top

management relations actionnaires

Image peacutenaliteacutes et amendes objectifs

strateacutegiques

Initiation au management copy CRCF ndash J Sornet Page 34 48

23 ndash Le rocircle socieacutetal des entreprises

La responsabiliteacute socieacutetale de lrsquoentreprise (RSE) deacutesigne le rocircle qursquoelle prend dans la socieacuteteacute

au-delagrave de son activiteacute purement geacuteneacuteratrice de profit On parle aussi drsquoentreprise citoyenne

La RSE est indissociable du deacuteveloppement durable de porteacutee mondiale et dont les trois

piliers sont

- eacuteconomique (favoriser le deacuteveloppement les eacutechanges internationaux)

- social (accegraves aux soins eacuteducation conditions de travail hellip)

- environnemental (pollution preacuteservation des ressources hellip)

La RSE integravegre notamment une preacuteoccupation sociale de lrsquoentreprise vis-agrave-vis de ses salarieacutes

(seacutecuriteacute et santeacute au travail juste reacutemuneacuteration deacuteveloppement personnel hellip) Elle conduit agrave

tenir compte dans le management drsquoune vision exteacuterieure agrave lrsquoentreprise qui peut avoir des

reacutepercussions possibles sur son activiteacute eacuteconomique

Lrsquoentreprise peut aussi tirer avantage drsquoune deacutemarche responsable par la baisse de certains

coucircts (plus faibles consommations drsquoeacutenergies reacuteduction des transports hellip)

Le rocircle socieacutetal de lrsquoentreprise a eacuteteacute reconnu en France par la loi laquo NRE raquo de 2001 (loi sur les

nouvelles reacutegulations eacuteconomiques) qui oblige les socieacuteteacutes franccedilaise coteacutees sur un marcheacute

reacuteglementeacute agrave rendre compte dans leur rapport annuel de leur gestion sociale et

environnementale au travers de leur activiteacute

Article 116 de la loi Le rapport viseacute agrave larticle L 225-102 rend compte hellip laquo Il comprend

eacutegalement des informations dont la liste est fixeacutee par deacutecret en Conseil dEtat sur la

maniegravere dont la socieacuteteacute prend en compte les conseacutequences sociales et

environnementales de son activiteacute Le preacutesent alineacutea ne sapplique pas aux socieacuteteacutes

dont les titres ne sont pas admis aux neacutegociations sur un marcheacute reacuteglementeacute raquo

Une norme ISO 14000 integravegre ces preacuteoccupations et des taxes eacutecologiques sont

progressivement creacutees

3 ndash Le management par la valeur

31 ndash De lrsquoanalyse au management par la valeur

Lrsquoanalyse de la valeur est neacutee en 1947 aux Etats-Unis (General Electrics) Cette technique

consiste agrave eacutelaborer des produits conformes aux attentes de la clientegravele mais sans excegraves pour

trouver un bon compromis entre valeur pour le client et coucirct Le produit optimal est deacutefini agrave

partir drsquoenquecirctes qui deacuteterminent le besoin client (ou plutocirct drsquoun client laquo type raquo)

Exemple il est inutile de concevoir un petit veacutehicule citadin capable de parcourir

500 000 km sans avarie compte tenu des effets de mode et du faible kilomeacutetrage

annuel Par contre le marcheacute peut exiger un fonctionnement sans faille sur 150 000 km

soit dix ans en moyenne ce qui conditionne les coucircts de production

Cette recherche drsquoun ajustement de valeur au besoin des clients eacutetait un preacutecurseur du

management par la valeur qui recherche plus largement la creacuteation de valeur pour

chacune des parties prenantes de lrsquoentreprise tout en lui meacutenageant un reacutesultat suffisant

Plus geacuteneacuteralement le management par la valeur est deacutefini par une norme europeacuteenne (EN

12973)

Le management par la valeur est un style de management particuliegraverement destineacute agrave

mobiliser les individus agrave deacutevelopper les compeacutetences et agrave promouvoir les synergies et

Initiation au management copy CRCF ndash J Sornet Page 35 48

linnovation avec pour objectif la maximisation de la performance globale dun

organisme Le management par la valeur apporte une nouvelle faccedilon dutiliser nombre

de meacutethodes de management existantes Il est en coheacuterence avec le Management

de la qualiteacute

Cette approche du management pose de nombreuses questions notamment quelles

prioriteacutes et quelles valeurs attribuer aux parties prenantes comment appreacutehender la

perception par les parties prenantes de la valeur qui leur est affecteacutee

32 ndash La valeur client

Le processus drsquoeacutelaboration drsquoun produit qui consomme des ressources coucircteuses doit creacuteer

une valeur suffisante pour provoquer lrsquoachat par le client final La production drsquoune valeur

reconnue par le client est vitale pour lrsquoentreprise mais sa deacutetermination est parfois complexe

La valeur du produit perccedilue par le client integravegre des eacuteleacutements en partie subjectifs

- une valeur drsquousage (le produit reacutepond agrave un besoin)

- une valeur drsquoestime (lrsquoimage apporteacutee par le produit un aspect affectif)

- une valeur drsquoeacutechange (deacuteduite de lrsquoespoir de revente du produit)

Valeurs drsquousage drsquoestime et drsquoeacutechange deacutependent implicitement de la qualiteacute (un bien peu

fiable est impropre agrave lrsquousage attendu de mauvaise qualiteacute notoire il nrsquoapporte pas une

image positive et ses deacutefauts connus nuisent agrave sa revente) Une eacutevaluation de la qualiteacute

intervient donc dans la valeur perccedilue du produit

Par ailleurs le client considegravere le coucirct drsquoobtention du produit (les charges qursquoil doit supporter

pour acqueacuterir le produit lrsquoeffort qursquoil doit faire pour trouver le produit et les frais de mise agrave

disposition)

Le prix perccedilu par le client est geacuteneacuteralement supeacuterieur au prix de vente

Le client achegravete theacuteoriquement le produit qui preacutesente la diffeacuterence valeur perccedilue ndash prix

perccedilu la plus favorable ou le meilleur rapport prix perccedilu qualiteacute perccedilue et dans certains

cas celui qui a le prix produit le plus bas

Remarque les valeurs du scheacutema ci-dessus changent durant le cycle de vie du produit

(un nouveau produit peut avoir une valeur perccedilue plus eacuteleveacutee qursquoen fin de vie) La

valeur client ne peut ecirctre eacutevalueacutee que par enquecirctes et ne peut donc ecirctre deacutefinie avec

certitude

La notion de laquo satisfaction client raquo conseacutecutive agrave une vente influence aussi le prix produit et

le prix perccedilu

- lrsquoentreprise gagne sur les coucircts de recherche de clientegravele

- le client nrsquoa pas agrave rechercher un nouveau fournisseur et beacuteneacuteficie drsquoun coucirct drsquoobtention

plus bas

valeur perccedilue client

prix perccedilu client

coucirct produit Marge (valeur creacuteeacutee pour

lrsquoentreprise)

euros

prix produit

Valeur creacuteeacutee

pour le client

Initiation au management copy CRCF ndash J Sornet Page 36 48

La satisfaction du client deacutepend de facteurs qualitatifs aussi divers que la fiabiliteacute du produit

la vitesse de reacuteaction du fournisseur lrsquoattitude des commerciaux lrsquoefficaciteacute du service

apregraves-vente la netteteacute des contrats ou la justesse de la facture

Valeur perccedilue coucirct marge et satisfaction reacutesultent de processus allant de la conception du

produit jusqursquoagrave sa livraison et son apregraves-vente La deacutemarche laquo processus raquo et lrsquolaquo analyse de la

valeur raquo en forccedilant la recherche de solutions efficientes agrave tout niveau administratif

technique commercial et apregraves-vente sont donc neacutecessaires pour bien positionner

lrsquoentreprise sur son marcheacute

Pour autant le risque commercial ne peut jamais ecirctre annuleacute et lrsquooffre de lrsquoentreprise ne

satisfait geacuteneacuteralement pas en milieu concurrentiel tous ses clients potentiels

33 - La creacuteation de valeur pour les autres parties prenantes

Les salarieacutes

La creacuteation drsquoune valeur suffisante pour les salarieacutes est reconnue comme neacutecessaire car des

observations montrent que la satisfaction des clients en deacutepend Moins souvent eacutevoqueacutee en

peacuteriode de chocircmage elle nrsquoest prioritaire que pour les employeacutes dont lrsquoentreprise souhaite

conserver les compeacutetences

La laquo valeur salarieacute raquo ne comprend pas que le salaire Le sentiment drsquoappartenance agrave un

groupe la reconnaissance lrsquoaccomplissement de soi et la construction professionnelle en

sont des eacuteleacutements importants Comme pour les clients on doit ainsi distinguer la reacutetribution

perccedilue du salaire objectif

Les actionnaires

Lrsquoactionnaire apporte des fonds propres agrave lrsquoentreprise en contrepartie de titres parfois

neacutegociables en bourse et assortis drsquoun droit de vote en assembleacutee geacuteneacuterale La valeur

attribueacutee aux actionnaires est servie en termes moneacutetaires (dividende ou augmentation de la

valeur du titre neacutegociable)

Remarque des facteurs non moneacutetaires comme lrsquoimage de lrsquoentreprise qui deacutepend

en partie de sa communication peuvent influencer la deacutecision drsquoachat de vente ou

de conservation des titres par lrsquoactionnaire

Reacutetribution perccedilue euros

Salaire objectif

Avantage non

moneacutetaire de

lrsquoemploi

Initiation au management copy CRCF ndash J Sornet Page 37 48

Compte tenu de lrsquoimportance croissante de lrsquoactionnariat dans le financement des grandes

entreprises coteacutees en bourse et notamment des investisseurs institutionnels comme les fonds

de pension des indicateurs speacutecifiques ont eacuteteacute introduits pour appreacutecier la performance des

entreprises vue par les actionnaires Par exemple la valeur ajouteacutee eacuteconomique (EVA reg

economic value added marque deacuteposeacutee de Stern Stewart ou VAE ndash valeur ajouteacutee

eacuteconomique parfois deacutenommeacutee VEC ndash valeur eacuteconomique creacuteeacutee) qui prend en compte le

coucirct du capital

LrsquoEVA correspond tregraves scheacutematiquement au calcul suivant

EVA = (PO) profit opeacuterationnel ndash (C) coucirct du capital X (CE) capitaux employeacutes

LrsquoEVA neacutecessite en pratique des retraitements assez complexes Le PO peut se deacuteterminer

selon les principes suivants

- PO = reacutesultat drsquoexploitation (avant inteacuterecircts) ndash impocirct

- PO = beacuteneacutefice courant (tenant compte des inteacuterecircts) + inteacuterecircts ndash eacuteconomie drsquoimpocirct sur les

inteacuterecircts (on exclue les eacuteleacutements financiers et lrsquoimpocirct correspondant) ndash impocirct

- lrsquoimpocirct pris en compte correspond au profit opeacuterationnel consideacutereacute (dans les cas courants agrave

13 du PO)

C = taux moyen de reacutemuneacuteration du capital (reacutesultant par exemple du dividende exigeacute de

certains investisseurs et des taux drsquoemprunts bancaires)

CE = capitaux propres et dettes portant inteacuterecirct

Remarque le profit opeacuterationnel ou reacutesultat opeacuterationnel correspond au NOPAT ndash net

operating profit after tax - anglo-saxon LrsquoEVA est eacutegale au NOPAT diminueacute de la

reacutemuneacuteration des capitaux

Exemple lrsquoentreprise X dispose drsquoun capital de 2 500 000 euro et reacutealise un beacuteneacutefice net

drsquoimpocirct de 450 000 euro (taux 33 13) Un dividende de 6 doit ecirctre verseacute aux

actionnaires et la banque lui a accordeacute un precirct de 1 200 000 euro agrave 4 Les autres

constituants des reacutesultats financier et exceptionnel sont neacutegligeables

Reacutesultat opeacuterationnel = 450 000 + 004 x 1 200 000 x 23 = 482 000 euro

Coucirct du capital = 006 x 2 500 000 + 004 x 1 200 000 x 23 = 182 000 euro

EVA = 300 000 euro

Coucirct moyen pondeacutereacute du capital (C) = (004 x 1 200 000 x23 + 006 x 2 500 000)

3 700 000 Soit 492

Si lrsquoEVA est positive lrsquoentreprise creacuteeacutee de la valeur apregraves reacutemuneacuteration des capitaux et sa

valeur boursiegravere doit augmenter

Lrsquoutilisation de lrsquoEVA comme indicateur influence le management de lrsquoentreprise car il y a

trois moyens pratiques drsquoaugmenter lrsquoEVA

- augmenter le reacutesultat opeacuterationnel

- lancer des investissements ayant une rentabiliteacute supeacuterieure agrave C

- eacuteliminer les activiteacutes ayant une rentabiliteacute infeacuterieure agrave C

Remarque lrsquoutilisation sans nuance de lrsquoEVA comme critegravere de management peut

poser problegraveme Le calcul de lrsquoEVA repose sur des ajustements comptables il est donc

sujet agrave manipulations (provisions capitalisation ou non de la RD hellip) Par ailleurs le

critegravere laquo EVA raquo pris isoleacutement peut conduire agrave chercher la rentabiliteacute agrave court terme agrave

reacuteduire les investissements prospectifs et donc nuire agrave terme au deacuteveloppement de

lrsquoentreprise

Initiation au management copy CRCF ndash J Sornet Page 38 48

Les fournisseurs reccediloivent le paiement de leurs factures plus ou moins rapidement (le deacutelai

de paiement repreacutesente une valeur consentie au fournisseur)

Lrsquoentreprise peut accroicirctre la valeur apporteacutee agrave ses fournisseurs par des actions cibleacutees

comme une contribution agrave la formation de leurs personnels certains transferts de

technologie ou de savoir faire agrave des sous-traitants une coopeacuteration suivie favorisant leur

deacuteveloppement lrsquointeacutegration agrave des campagnes de promotion

A noter que la valeur consentie aux fournisseurs peut avoir une influence sur la qualiteacute et les

deacutelais de livraison des produits

La collectiviteacute reccediloit des taxes et parfois des prestations en nature par deacutefaut ou explicites

(effort de preacuteservation de lrsquoenvironnement ameacutenagement du territoire par les implantations

aide mateacuterielle agrave des projets participation agrave la formation par exemple)

APPLICATIONS DT

DT1 Deacutefinir expliquer deacutereacuteglementation socieacutetal eacuteconomies drsquoeacutechelle coaching EDI

gouvernance

DT2 Deacuteterminer en quoi la deacutemarche TQM srsquoinscrit dans les deacutefis actuels du management

DT3 Apregraves avoir consulteacute les documents ci-dessous extraits du site drsquoAir France

(httpdeveloppement-

durableairfrancecomFRfrlocaldemarcheN4_positionnement_pphtm)

exposer les enjeux et les limites de la RSE et de la gestion des parties prenantes

Initiation au management copy CRCF ndash J Sornet Page 39 48

Dialogue avec les parties prenantes

Initiation au management copy CRCF ndash J Sornet Page 40 48

Attentes des parties prenantes

Initiation au management copy CRCF ndash J Sornet Page 41 48

Creacuteation de valeur pour les parties prenantes

La creacuteation de valeur pour les parties prenantes est au cœur de la strateacutegie du Groupe Le scheacutema de

distribution financiegravere ci-dessous donne un aperccedilu de la distribution des recettes du Groupe aux

diffeacuterentes parties prenantes actionnaires collaborateurs fournisseurs pouvoirs publics

collectiviteacutes locales etc

Initiation au management copy CRCF ndash J Sornet Page 42 48

Fiche DT1 ndash Extrait du sommaire de laquo Problegravemes eacuteconomiques raquo No 2894

La gestion des entreprises bouleverseacutee par les technologies de linternet

Reacutealiteacutes industrielles - Annales des Mines Jean-Michel Yolin

Avec lavegravenement de linternet les processus de conception de production et de vente sont

radicalement remis en cause Quel que soit le secteur dactiviteacute les technologies de linternet

permettent en effet de reacuteduire les deacutelais et de passer dun processus discontinu agrave un processus

continu Lorganisation des entreprises et leur mode de gestion en sont profondeacutement bouleverseacutes

tant au niveau individuel que collectif Linternet rend ainsi possible la reacutealisation dobjectifs que les

entreprises cherchaient agrave atteindre depuis longtemps sans y parvenir meilleure eacutecoute du client

travail sans stocks en flux tendu hieacuterarchies plates autorisant une grande reacuteactiviteacute flexibiliteacute dans

lorganisation et loutil de production acceacuteleacuteration du renouvellement des produits entreprises en

reacuteseau ougrave chacune se recentre sur son cœur de meacutetier etc

Le laquo knowledge management raquo ou comment geacuterer les connaissances

Document de travail du LAMSADE - Michel Grundstein

Peter Drucker lavait preacutedit le capital immateacuteriel eacutetait voueacute agrave devenir un facteur de compeacutetitiviteacute

pour lentreprise La libeacuteralisation des eacutechanges acceacutelegravere les processus de deacutecision de lentreprise

et implique que lassimilation des informations soit agrave la fois de meilleure qualiteacute et plus rapide Ainsi

la fonction qui consiste agrave manager les connaissances au sein de lentreprise savegravere primordiale

Bien que la prise de conscience de limportance du capital immateacuteriel ait eacuteteacute tardive - le concept

de knowledge management est apparu en France aux Etats-Unis et au Japon au milieu des

anneacutees 1990 - agrave lheure actuelle lorganisation de leacutechange dinformations et le partage des

connaissances sont devenus des facteurs cleacutes dune gestion performante de lentreprise Ils

doivent sinscrire dans un projet global destineacute agrave mettre en valeur les savoirs et les savoir-faire

individuels et collectifs

Les leccedilons du laquo coaching raquo pour le management de la qualiteacute

Humanisme et Entreprise - Martine Brasseur

Parmi les nouvelles formes de management en vogue dans les entreprises le coaching figure en

bonne place Appliqueacute au management de la qualiteacute il sagit dune pratique

daccompagnement destineacutee agrave initier et agrave faciliter le processus de deacuteveloppement dun individu

La deacutemarche consiste agrave affirmer que tout individu est en quecircte de qualiteacute agrave condition toutefois

de ne pas lui imposer des contraintes lempecircchant de progresser On considegravere notamment les

erreurs comme potentiellement feacutecondes En deacutefinitive le coach donne au coacheacute la permission

de reacuteussir en lui donnant aussi la permission deacutechouer

Initiation au management copy CRCF ndash J Sornet Page 43 48

Fiche DT2 ndash Management strateacutegique les sept deacutefis agrave relever dici agrave 2016

Extrait drsquoun article du site wwwlentreprisecom -Sabine Blanc - Mis en ligne le 20032007

(httpwwwlentreprisecom325article11977html)

Une eacutetude anglaise publieacutee par lopeacuterateur Orange Grande-Bretagne deacutecrypte la mutation

des formes de travail et les enjeux majeurs pour les entreprises de demain afin decirctre au top

de la compeacutetitiviteacute Voici les challenges-cleacutes pour les managers qui veulent rester dans la

course hellip

1 - Future organisation du travail les quatre laquo mondes raquo possibles

La reacutealiteacute sera probablement un meacutelange de ces quatre sceacutenarios souligne lrsquoeacutetude

Les mondes mutuels Tout se passe dans le cadre des communauteacutes locales vie priveacutee

comme professionnelle Le modegravele coopeacuteratif preacutevaut au lieu du laquo big business raquo Oublieacutes

aussi dans ce systegraveme les trajets pour aller au bureau les gens preacutefegravereront travailler dans de

petites entreprises locales souvent connecteacutees au reacuteseau drsquoautres structures similaires

Les laquo reacutepondants raquo (en anglais laquo replicants raquo) La figure du consultant freelance deviendra

dominante tandis que celle du salarieacute deacuteclinera Il ne sera pas rare de travailler pour plusieurs

entreprises On perdra en seacutecuriteacute de lrsquoemploi en visibiliteacute et en routine ce que lrsquoon gagnera

en liberteacute La majeure partie des tacircches srsquoeffectuera chez soi avec la possibiliteacute de srsquoinstaller

temporairement dans les bureaux de son client du moment Dans un contexte dincertitude

sur lrsquoavenir les travailleurs alterneront peacuteriodes drsquoactiviteacute intense et repos Ce sera agrave eux

drsquoaller vers les entreprises et non lrsquoinverse mecircme si celles-ci devront veiller agrave rester attractives

Les cottages eacutelectroniques Comme ce nom le suggegravere le teacuteleacutetravail deviendrait la norme

univers priveacute et professionnel se confondant Plus besoin de subir une heure de transport les

salarieacutes se logueront de chez eux sur le reacuteseau de lrsquoentreprise Les reacuteunions se tiendront dans

de petits bureaux centraux situeacutes agrave courte distance La flexibiliteacute du temps de travail srsquoimpose

Les salarieacutes disposeront de plus de marge de liberteacute dans leur activiteacute

Les disciples de la nueacutee Cette appellation poeacutetique cache simplement une extension de

lrsquoorganisation actuelle des grandes entreprises avec des salarieacutes se rendant sur un lieu de

travail centraliseacute Le rocircle croissant des technologies de lrsquoinformation multipliera les faccedilons de

collaborer et accroicirctra lrsquoefficaciteacute Le controcircle du travail sera omnipreacutesent La frontiegravere entre

travail et vie priveacutee restera marqueacutee

2 - Sept deacutefis pour les entreprises et leur managers

Quoi qursquoil advienne les entreprises et leurs dirigeants devront concentrer leurs efforts sur sept

points-cleacutes pour srsquoadapter Voici quelques exemples de probleacutematiques souleveacutees par le

rapport et des pistes de solution

Le leadership Les managers devront entre autres savoir persuader et influencer des

travailleurs beaucoup plus indeacutependants Ils auront aussi agrave repenser les niveaux auxquels

prendre les deacutecisions strateacutegiques en haut ou au contraire agrave des degreacutes moins eacuteleveacutes de la

pyramide hieacuterarchique

gt Faire du management une force facilitant les activiteacutes transversales plutocirct que la reacuteduire agrave

la seule fonction de deacutecision

La culture drsquoentreprise Davantage de salarieacutes capables de reacutefleacutechir seront neacutecessaires

tandis que les tacircches qui peuvent ecirctre automatiseacutees ou scripteacutees diminueront Un des

enjeux creacuteer une culture agrave mecircme drsquoattirer et drsquoencourager les personnes preacutesentant ces

qualiteacutes de reacuteflexion requises dans un contexte de compeacutetition accrue et de plus grande

indeacutependance des travailleurs

Initiation au management copy CRCF ndash J Sornet Page 44 48

gt Passer si neacutecessaire drsquoune culture drsquoentreprise forte agrave un mode drsquoengagement plus

consensuel moins rebutant

La marque Conseacutequence du recours croissant agrave lrsquo laquo outsourcing raquo lrsquoimage drsquoune marque

deacutependra plus drsquoagents exteacuterieurs qui ne fonctionnent pas forceacutement selon le mecircme mode

drsquoorganisation Comment garder le controcircle dessus

gt Choisir le mode qui corresponde le plus agrave vos valeurs et preacutevoir un programme de risk

management qui mette en eacutevidence ougrave les conflits sont susceptibles de jaillir

Lrsquoinnovation Plus que jamais il faudra faire face agrave une acceacuteleacuteration du rythme de

lrsquoinnovation en proposant constamment des solutions adapteacutees

gt Tisser des partenariats strateacutegiques avec drsquoautres entreprises pour partager les coucircts et les

fruits de lrsquoinnovation

Le deacutefi opeacuterationnel et technologique De quelle faccedilon controcircler lrsquoinformation crsquoest-agrave-dire

faire en sorte que les bonnes personnes accegravedent facilement agrave une information toujours en

phase tout en maintenant la seacutecuriteacute

gt Recourir agrave des laquo feuilles de route des futurs raquo syntheacutetisant en une page les indicateurs

sociaux et de consommation ainsi que les eacutevolutions technologiques et leacutegislatives qui

influent sur les changements et indiquant comment ils modifient vos marcheacutes vos clients et

votre organisation

La qualiteacute Si de nouveaux proceacutedeacutes ont pu deacutegrader la qualiteacute comme le recours agrave des

centres drsquoappel externaliseacutes drsquoautres ideacutees se sont reacuteveacuteleacutees plus prometteuses comme en

teacutemoigne le succegraves de certaines compagnies aeacuteriennes low cost Elles ont su conjuguer prix

serreacutes et services eacuteleveacutes ce qui devra devenir la norme estime lrsquoeacutetude

gt Continuer de rechercher la qualiteacute Elaborez aussi une bonne prestation service qui inclut

une livraison de qualiteacute voire creacuteez-la en partenariat avec les consommateurs

La leacutegislation La question de la proprieacuteteacute intellectuelle pourrait ecirctre probleacutematique Elle est

deacutejagrave source de conflits comme en teacutemoigne le procegraves pour violation de brevet intenteacute agrave RIM

le fabricant canadien du Blackberry par NTP Que pourra-t-on et que faudra-t-il proteacuteger par

un brevet Il sera eacutegalement neacutecessaire drsquoadapter la leacutegislation aux nouveaux modes

drsquoorganisation

gt Collaborer avec les acteurs du mecircme secteur et les leacutegislateurs pour deacutevelopper les

modegraveles des lieux de travail du futur et bacirctir le droit le plus adeacutequat

Orange a-t-il vu juste dans ses preacutevisions Rendez-vous dans neuf ans pour la reacuteponsehellip

Initiation au management copy CRCF ndash J Sornet Page 45 48

Fiche DT3 ndash Le management par la qualiteacute totale

Extrait drsquoune lettre drsquoinformation du cabinet Baud Accordance Consulting AD2 consultants ndash

2002

1 - Le TQM (Total Quality Management) offre pour lentreprise une vision de la qualiteacute plus

large et transversale

Son principe est simple La finaliteacute de lEntreprise est de deacutevelopper la satisfaction de ses

clients tout en eacutetant beacuteneacuteficiaire cest agrave dire pas agrave nimporte quel prix Elle doit ameacuteliorer sa

rentabiliteacute au travers de la deacutemarche qualiteacute La Qualiteacute Totale vise agrave fournir aux clients

externes et internes une reacuteponse adeacutequate agrave leurs attentes dans le meilleur rapport qualiteacute

prix la meilleure efficience

Elle considegravere pour cela lensemble des processus de lentreprise ayant une incidence sur la

qualiteacute et la satisfaction des clients

Le TQM fait ainsi une large place agrave

la deacutefinition et la planification de la strateacutegie geacuteneacuterale

la coheacuterence de la politique qualiteacute avec la strateacutegie

la deacutemultiplication de la politique qualiteacute dans toutes les directions de lentreprise

la relation client fournisseur interne

la prise en compte de lenvironnement concurrentiel

la consideacuteration de lensemble des risques potentiels financiers sociaux concurrentielshellip

limplication et la motivation du personnel

lanalyse des besoins des clients et le positionnement marketing

la maicirctrise des processus transverses internes

les reacutesultats sous tous ses aspects y compris financiers commerciaux image

De nombreux reacutefeacuterentiels sont relatifs agrave la Qualiteacute Totale hellip Tous ces reacutefeacuterentiels imposent un

questionnement plus profond et indiscret sur le mode de fonctionnement de lentreprise et

son management

helliphellip

2 - LISO 9001 2000 au travers du deacuteploiement des processus (management supports

reacutealisation et ameacutelioration continue) reacutepond quelque peu agrave la mecircme logique

LISO est une ouverture indeacuteniable vers la logique du TQM mais ne se reacutefegravere pas agrave la notion

defficience

Les dirigeants sont cependant sensibles agrave la neacutecessaire reacuteduction des coucircts de non-qualiteacute

et dobtention de la qualiteacute agrave la rentabiliteacute du systegraveme de management de la qualiteacute

mais ne perccediloivent pas toujours la qualiteacute comme une deacutemarche globale

Les deacutemarches qualiteacute commencent bien souvent par la remise en cause de lorganisation

leacutevaluation critique de son efficaciteacute lexamen des processus et la mise en eacutevidence des

lourdeurs administratives

La qualiteacute devient laffaire de tous hellip

Initiation au management copy CRCF ndash J Sornet Page 46 48

Fiche DT4 ndash Le deacuteveloppement durable et la RSE

Extrait du site wwwvigeocom

(httpwwwvigeocomcsr-rating-agencyfrmethodologiecriteres-de-recherche37-

criteres-d-analysehtml)

Deacuteveloppement durable laquo un deacuteveloppement qui reacutepond aux besoins du preacutesent sans compromettre

la capaciteacute des geacuteneacuterations futures de reacutepondre aux leurs raquo (Commission mondiale sur lrsquoenvironnement

et le deacuteveloppement ndash 1987)

Reacutefeacuterentiel drsquoeacutevaluation des entreprises par le groupe Vigeacuteo (le groupe mesure les performances et le

niveau de maicirctrise des risques de responsabiliteacute sociale des entreprises et des organisations - site

wwwvigeocom)

1 Ressources Humaines Ameacutelioration continue des relations professionnelles des relations drsquoemploi et des conditions de travail 2 Droits humains sur les lieux de travail Respect de la liberteacute syndicale et promotion de la neacutegociation collective non discrimination et promotion de lrsquoeacutegaliteacute eacutelimination des formes de travail proscrites (enfants travail forceacute) preacutevention des traitements inhumains ou deacutegradants de type harcegravelements sexuels protection de la vie priveacutee et des donneacutees personnelles 3 Environnement Protection sauvegarde preacutevention des atteintes agrave lenvironnement mise en place drsquoune strateacutegie manageacuteriale approprieacutee eacuteco conception protection de la biodiversiteacute et maicirctrise rationnelle des impacts environnementaux sur lrsquoensemble du cycle de vie des produits ou services

4 Comportements sur les marcheacutes Prise en compte des droits et inteacuterecircts des clients inteacutegration de standards sociaux et environnementaux dans la seacutelection des fournisseurs et sur lrsquoensemble de la chaicircne drsquoapprovisionnement preacutevention effective de la corruption respect des regravegles concurrentielles 5 Gouvernement drsquoentreprise Efficience et probiteacute assurance de lrsquoindeacutependance et de lrsquoefficaciteacute du Conseil drsquoadministration effectiviteacute et efficience des meacutecanismes drsquoaudit et de controcircle et notamment inclusion des risques de responsabiliteacute sociale respect des droits des actionnaires et notamment des minoritaires transparence et rationaliteacute de la reacutemuneacuteration des dirigeants 6 Engagement socieacutetal Effectiviteacute inteacutegration manageacuteriale de lrsquoengagement contribution au deacuteveloppement eacuteconomique et social des territoires drsquoimplantation et de leurs communauteacutes humaines engagements concrets en faveur de la maicirctrise des impacts socieacutetaux des produits et des services contribution transparente et participative agrave des causes drsquointeacuterecirct geacuteneacuteral

Initiation au management copy CRCF ndash J Sornet Page 47 48

ELEMENTS DE CORRIGE DT DT1 Deacutefinir expliquer

Deacutereacuteglementation = suppression des contraintes eacuteconomiques (libre eacutechange des biens et

capitaux)

Socieacutetal = qui se rapporte agrave la structure agrave lrsquoorganisation ou au fonctionnement de la socieacuteteacute

Economies drsquoeacutechelle = reacuteduction des coucircts lieacutee au niveau drsquoactiviteacute (amortissement des

charges fixes)

Coaching = accompagnement de personnes ou deacutequipes pour le deacuteveloppement de leurs

potentiels

EDI = eacutechange de donneacutees informatiseacutees ET standardiseacutees (ex SWIFT bancaire edifact

documents deacuteclaratifs)

Gouvernance = exercice du pouvoir la bonne gouvernance est participative et eacutequitable

conforme agrave lrsquointeacuterecirct commun

DT2 Deacuteterminer en quoi la deacutemarche TQM srsquoinscrit dans les deacutefis actuels du management

Voir notamment fiche 43

Maicirctrise des processus reacuteduction des coucircts reacuteactiviteacute et satisfaction de la clientegravele = faire

face agrave la concurrence

Ameacutelioration de lrsquoimage motivation du personnel

DT3 Apregraves avoir consulteacute les documents ci-dessous extraits du site drsquoAir France

(httpdeveloppement-

durableairfrancecomFRfrlocaldemarcheN4_positionnement_pphtm)

exposer les enjeux et les limites de la RSE et de la gestion des parties prenantes

Trame geacuteneacuterale possible

Introduction

Les deacutefis contemporains (accroissement de la concurrence devenue mondiale recherche

de nouveaux avantages concurrentiels pression de la socieacuteteacute besoin drsquoimage et de projet

lisible pour mener lrsquoentreprise crise et scandales du libeacuteralisme hellip) RSE et PP

Deacuteveloppement (voir cours)

1 ndash Parties prenantes et management par la valeur

PP deacutefinir citer reacutesumer lrsquoavantage rechercheacute (fideacuteliser motiver recherche drsquoalliances

implicites)

PP moyens (dont exemples AF) et meacutethode de management par la valeur (reacutepartie)

2 ndash La responsabiliteacute socieacutetale de lrsquoentreprise

RSE 3 axes

- eacuteconomique (favoriser le deacuteveloppement les eacutechanges internationaux)

- social (accegraves aux soins eacuteducation conditions de travail hellip)

- environnemental (pollution preacuteservation des ressources hellip)

RSE gouvernance drsquoentreprise facteur drsquoimage inteacutegrable dans la deacutemarche PP

Article 116 de la loi Le rapport viseacute agrave larticle L 225-102 rend compte hellip laquo Il comprend

eacutegalement des informations dont la liste est fixeacutee par deacutecret en Conseil dEtat sur la maniegravere

dont la socieacuteteacute prend en compte les conseacutequences sociales et environnementales de son

activiteacute Le preacutesent alineacutea ne sapplique pas aux socieacuteteacutes dont les titres ne sont pas admis aux

neacutegociations sur un marcheacute reacuteglementeacute raquo

Initiation au management copy CRCF ndash J Sornet Page 48 48

RSE exemple AF (ONG fournisseurs)

3 ndash Liens entre PP et RSE

- la RSE introduit de nouvelles PP

- la RSE suppose le respect des PP usuelles (employeacutes clients notamment)

4 - Probleacutematique

- deacutefinir la valeur reacuteellement apporteacutee par une gestion des PP (confusion salaire ndash valeur

idem impocircts hellip ex laquo valeur ajouteacutee raquo)

- communication (neacutecessaire mais aller au-delagrave)

- marginaliteacute des deacutepenses RSE (efficaciteacute sinceacuteriteacute de lrsquoengagement marge de manœuvre)

- charge RSE reporteacutee sur des tiers (ex fournisseurs AF)

- inteacutegration de facteurs non visibles en comptabiliteacute (pertes drsquoemploi nuisances hellip)

Conclusion

Voies incontournables mais pouvant nrsquoavoir qursquoun effet superficiel et temporaire Voir utiliteacute

drsquoaccompagnement leacutegislatif de regravegles de gouvernance

Initiation au management copy CRCF ndash J Sornet Page 10 48

Principes du management drsquoapregraves la norme ISO 9001 (2000)

- Orientation vers le client (satisfaire ses attentes)

- Leadership (les dirigeants eacutetablissent les orientations de lrsquoorganisme Ils doivent creacuteer

un environnement interne ougrave les personnes peuvent clairement srsquoimpliquer dans la

reacutealisation des objectifs de lrsquoorganisme)

- Implication du personnel (les personnes sont agrave tout niveau lrsquoessence de lrsquoorganisme et

leur implication permet drsquoutiliser leurs aptitudes au profit de lrsquoorganisme)

- Approche laquo processus raquo (un reacutesultat est mieux atteint quand les ressources et les

activiteacutes neacutecessaires sont geacutereacutees comme un processus)

- Approche systegraveme (assimiler les processus correacuteleacutes agrave un systegraveme contribue agrave

lrsquoefficaciteacute et agrave lrsquoefficience de lrsquoorganisme vis-agrave-vis de ses objectifs)

- Ameacutelioration continue (objectif permanent de lrsquoorganisme)

- Prise de deacutecision efficace (par lrsquoanalyse de donneacutees et drsquoinformations)

- Relations mutuellement beacuteneacutefiques avec les fournisseurs (pour augmenter la capaciteacute

des deux organismes agrave creacuteer de la valeur)

IM2 Distinguer leader et manager

IM3 Compleacuteter le tableau ci-dessous en analysant chaque action preacutesenteacutee Faire ensuite

ressortir les domaines niveaux ou techniques de management pouvant ecirctre mobiliseacutes pour

chaque situation

Initiation au management copy CRCF ndash J Sornet Page 11 48

Caracteacuteristiques

de lrsquoaction

- reacutepeacutetition

- risque

- normes

- ampleur

Prise de

deacutecision

- opeacuterationnelle

strateacutegique

- deacutelai

Informations

neacutecessaires

- nature

- origine

- deacutelai obtention

Cleacutes pour la

reacuteussite

Intervention

exteacuterieure

possible

Assurer la

restauration du

soir

(restaurant

familial)

Construire un

viaduc

(autoroute)

Certifier les

comptes

annuels drsquoun

groupe

national

(cabinet

drsquoaudit)

Lancer une

ligne drsquoavions

(constructeur

aeacuteronautique)

Reacuteduire la

capaciteacute de

production

(groupe

industriel)

Acqueacuterir une

entreprise

concurrente

(teacuteleacutephonie

mobile)

Initiation au management copy CRCF ndash J Sornet Page 12 48

Fiche IM1 - Deacutefinitions du management

Dictionnaire anglais - franccedilais direction administration gestion intrigue manegravege

Wikipeacutedia Le management est lensemble des techniques dorganisation qui sont mises en

oeuvre pour ladministration dune entiteacute

Au point de vue eacutetymologique le verbe manage vient de litalien maneggiare (controcircler)

influenceacute par le mot franccedilais manegravege (faire tourner un cheval dans un manegravege) A cette

notion il faut aussi ajouter la notion de meacutenage (geacuterer les affaires du meacutenage) qui consiste agrave

geacuterer des ressources humaines et des moyens financiers

helliphellip

Fiche IM2 - Etudier le management

Concreacutetiser

Manager neacutecessite de syntheacutetiser des informations parfois complexes incomplegravetes et de

domaines tregraves divers pour en deacuteduire des actions Une approche trop parcellaire peut

conduire agrave lrsquoeacutechec et le savoir-faire est neacutecessaire pour agir vite avec un minimum de risque

Lrsquoeacutetudiant doit se preacuteparer simultaneacutement aux examens et agrave la pratique Il nrsquoa souvent connu

lrsquoentreprise que durant quelques semaines de stage et le manque de laquo recul raquo ne lui permet

pas toujours de concreacutetiser les theacuteories Il doit compenser par la lecture (ouvrages revues

journaux eacuteconomiques et boursiers) et en eacutetant attentif aux informations ambiantes (tout en

relativisant le style journalistique) en mettant en relation le cours les concepts les modegraveles

lrsquoactualiteacute les stages

Savoir traiter un exercice

Pour reacuteussir un examen ou traiter une application peacutedagogique (la conception les points 1 agrave

6 peut repreacutesenter le tiers du temps de travail)

1 ndash Identifier le type de sujet (faut-il trouver une solution pratique ou communiquer une

reacuteflexion geacuteneacuterale )

2 ndash Lire le sujet et relever les mots cleacutes

3 ndash Deacutefinir les mots cleacutes

4 ndash Reacutesumer la probleacutematique du sujet (en quelques lignes)

5 ndash Lister les connaissances reacutefeacuterences et raisonnements reacutepondant au problegraveme (par

recherche spontaneacutee ou raisonneacutee qui quoi ougrave quand comment combien hellip

listage des diffeacuterents points de vue) trouver des exemples (notamment dans les

documents fournis)

6 ndash Organiser la reacuteponse (deacutefinir le plan du deacuteveloppement ougrave des paragraphes bien

identifieacutes sont geacuteneacuteralement neacutecessaires en y liant les parties qui doivent ecirctre en nombre

limiteacute ndash de deux agrave quatre) Preacutevoir drsquoy inteacutegrer la deacutefinition des principales notions

induites par le sujet

7 ndash Reacutediger sous la forme adapteacutee (note technique ou recommandation solution

pratique exposeacute structureacute dissertation)

Introduction et conclusion sont indispensables agrave la dissertation ou agrave lrsquoexposeacute

- lrsquointroduction preacutesente le sujet traiteacute (phrase drsquoaccroche initiale) amorce la

probleacutematique (quelques sous - questions) et annonce le plan

- la conclusion syntheacutetise le deacuteveloppement (arguments) eacutelargit le sujet (prise de recul)

et apporte le point final (une phrase)

Une limite agrave la communication

Il est difficile de faire passer plus de 4 ou 5 ideacutees fortes dans un exposeacute unique

Initiation au management copy CRCF ndash J Sornet Page 13 48

Fiche IM3 - Bref historique

Antiquiteacute

3000 AJC

Peacuteriode greacuteco-

romaine

Transition

feacuteodale

12egraveme siegravecle

europe

15egraveme ndash 17egraveme

siegravecles

19egraveme siegravecle

20egraveme siegravecle

agriculture preacutedominante industrie limiteacutee aux besoins drsquoun individu ou drsquoun clan

pour la confection des outils des vecirctements et de la poterie Force motrice animale

ou humaine pour lrsquoessentiel

Grands travaux drsquoeacutetat en Egypte premiegravere laquo planification ndash organisation ndash controcircle raquo

Deacuteveloppement des communications essor industriel limiteacute peu de progregraves

technique (lrsquoesclavage supplante les innovations)

Deacuteveloppement progressif des eacutechanges commerciaux

La consommation indirecte atteint un bon niveau (surplus agricoles et

deacuteveloppement des villes) Apparition de nouveaux commerccedilants

Etat fort Evolutions technologiques (imprimerie bateaux performants instruments de

navigation) Extension geacuteographique de lrsquoeacuteconomie Apparition des corporations

drsquoartisans

Machine agrave vapeur chemin de fer passage de lrsquoartisanat au capitalisme

entrepreneurial producteur organisation des entreprises

Ecole classique (Taylor Fayol Weber) approche meacutecaniste bureaucratie

hieacuterarchie commandement fonctions et speacutecialisation laquo OS T raquo (organisation

scientifique du travail) organisation source de pouvoir rationaliteacute des individus bases

du management

Deacuteveloppement du capitalisme manageacuterial Electriciteacute peacutetrole puis communications

et information Consommation de masse mondialisation preacuteoccupations

eacutenergeacutetiques et environnementales 3 peacuteriodes

- standardisation grandes entreprises industrielles

- industries de consommation 30 glorieuses marketing multinationales protection

sociale

- deacutereacuteglementation monteacutee des services pays eacutemergents mondialisation et nouvelle

eacuteconomie (internet)

Ecole des relations humaines prise en compte de lrsquoindividu des motivations styles

de direction

Ecole neacuteo-classique et post-classique deacutecentralisation coordonneacutee DPO

management participatif zeacutero deacutefaut flux tendus

Approche systeacutemique partition de lrsquoentreprise eacutetude des interactions feacutedeacuteration

vers lrsquoobjectif controcircle et ajustement

Theacuteories de la deacutecision rationaliteacute limiteacutee contribution reacutetribution coalitions

Ecole socio-technique recherche de compromis technologie organisation

enrichissement des tacircches autonomie des groupes

Approche sociologique effets sociaux du travail jeux de pouvoir dans lrsquoentreprise

reacutegulation sociale

Theacuteories de la contingence facteurs contingents adaptation agrave lrsquoenvironnement

configurations organisationnelles

Theacuteories de la firme controcircle manageacuterial droits de proprieacuteteacute relation drsquoagence

Theacuteories contractualistes firme nœud de contrats coucircts de transaction

opportunisme externalisation internalisation

Approche eacutevolutioniste eacutecologie des organisations modegravele eacutevolutioniste

contraintes de sentier

Approche par les ressources valorisation des ressources compeacutetences cleacutes

apprentissage organisationnel

(Classement simplifieacute)

Initiation au management copy CRCF ndash J Sornet Page 14 48

ELEMENTS DE CORRIGE IM

IM1 Commenter la deacutefinition du management par la norme ISO et le manager de Mintzberg

Efficient = optimum avec les moyens disponibles

ISO (management objectifs) (manager moyens) HM

IM2 Le leader entraicircne naturellement derriegravere lui Le manager nrsquoest pas toujours leader

(mecircme si crsquoest souhaitable) Le leader nrsquoest pas toujours manager (plutocirct notion individuelle)

Leadership = faculteacute de diriger conjugaison drsquoune autoriteacute naturelle ou drsquoun savoir-faire

acquis drsquoune capaciteacute agrave entraicircner des personnes ou des groupes et drsquoune leacutegitimiteacute

statutaire (de position)

IM3 Compleacuteter le tableau ci-dessous en analysant chaque action preacutesenteacutee Faire ensuite

ressortir les domaines niveaux ou techniques de management pouvant ecirctre mobiliseacutes pour

chaque situation

Satisfaction client

Implication du personnel

Processus systegraveme

Ameacutelioration continue

Deacutecision efficace

Recherche de valeur

Image entreprise

Liaisons

Information

Reacutepartition ressources

Reacutegulation

Neacutegociation

Leadership

Initiation au management copy CRCF ndash J Sornet Page 15 48

Caracteacuteristiques

de lrsquoaction

- reacutepeacutetition

- risque

- normes

- ampleur

Prise de

deacutecision

- opeacuteration

- direction

- deacutelai

Informations

neacutecessaires

- nature

- origine

- deacutelai

obtention

Cleacute pour la

reacuteussite

Intervention

exteacuterieure

possible

Assurer la

restauration du

soir

(Restaurant

familial)

Technique

(fabrication)

Vente (terrain)

Appros

Reacutepeacutetitive

(quot)

Risque faible

Normes

drsquohygiegravene

Faible

Opeacuterationnelle

Geacuterant

responsable

Rapide (qq

jours menu et

appros)

Nombre de

couverts

Tarifs usuels

Calendrier

(fecirctes)

Clients docs

divers

expeacuterience

Qq jours

Varieacuteteacute menu

Plats phares

Accueil

Appros

Tarification

Vins

Gestion

congeacutelation

Qualiteacute cuisine

Fournisseurs

Extra

Publiciteacute

Construire un

viaduc

(autoroute)

Technique

Organisation

Appros

Uniteacute (ou peu)

Eleveacute (financier

technique)

Architecture

Eleveacutee

Direction

(aleacuteas)

Opeacuterationnelle

(conduite

chantier)

Immeacutediat agrave qq

semaines

Plans

plannings

Qualifications

Meacuteteacuteo

Disponibiliteacutes

Bureau eacutetudes

Qq sem agrave 24h

Techniciteacute

Appros

Qualifications

Preacutevision

GRH

Contrat juste

SS traitants

Organismes

certificateurs

Controcircle

client

Certifier les

comptes

annuels drsquoun

groupe national

(cabinet

drsquoaudit)

Technique

Relation client

Gestion des

connaissances

Annuelle

Moyen

Regravegles

comptables

fiscales

Moyenne (selon

importance du

cabinet)

Opeacuterationnelle

Qq jours agrave

semaines

Comptable

Juridique

Client

Etat

Qq jours agrave

semaines

Techniciteacute

Expeacuterience

Relation client

Systegraveme info client

Siegravege

Autre cabinet

Lancer une

ligne drsquoavions

(constructeur

aeacuteronautique)

Strateacutegique

RD

Etudes

Uniteacute

Tregraves eacuteleveacute

Aeacuteronautique

Tregraves eacuteleveacutee

Direction

Qq mois agrave

anneacutees

Marcheacute

Etudes

Compagnies

Qq mois agrave

anneacutees

Concept

Outil industriel

Coucirct exploitation

Tarif

Fiabiliteacute

Deacutelaisconcurrence

SI simulation

SS traitants

Bureaux

drsquoeacutetudes

speacutecialiseacutes

Compagnies

Conseils

Reacuteduire la

capaciteacute de

production

(groupe

industriel)

Strateacutegique

RH

Communication

Production

Uniteacute

Moyen

Leacutegislation

(dont RH)

Eleveacutee

Direction

Qq mois agrave

anneacutees

Financiegravere

Industrielle

Marcheacute

Organisation

Organismes

speacutecialiseacutes

DRH

Qq mois

Communication

Connaissance des

compeacutetences

Connaissance outil

industriel

Concurrence

Portefeuille

drsquoactiviteacutes

Cabinet

drsquoorganisation

Conseils

speacutecifiques

Acqueacuterir une

entreprise

concurrente

(teacuteleacutephonie

mobile)

Strateacutegique

Marketing

Production

(reacuteseau)

Financier

Communication

Uniteacute

Tregraves eacuteleveacute

Leacutegislation

telecom

Tregraves eacuteleveacutee

Direction

Qq mois

Financiegravere

Marcheacute

Reacuteseaux

(ampleur

recouvrement

hellip)

Organisations

Interne

Racheteacutee

Sources

speacutecialiseacutees

Qq mois

Communication

Marcheacute

Cours boursiers

Cabinet

drsquoorganisation

Conseils

speacutecifiques

Initiation au management copy CRCF ndash J Sornet Page 16 48

LE MANAGEMENT EN PRATIQUE

Pour assumer sa fonction le management doit couvrir sans discontinuiteacute lrsquoensemble de

lrsquoorganisation et inteacutegrer de nombreux facteurs dont nous allons reacutesumer lrsquoessentiel

1 ndash Les fonctions et activiteacutes du management

Pour Henri Fayol la fonction drsquoadministration de lrsquoentreprise (son management) reposait sur

cinq actions preacutevoir organiser commander coordonner et controcircler (laquo PO3C raquo)

Nous distinguerons cinq activiteacutes de management

- la conception (au plus haut niveau finaliteacute but ou vocation de lrsquoorganisation

meacutetiers dimension politique de croissance hellip)

- la planification (deacutefinition des objectifs eacutecheacuteances)

- lrsquoorganisation (reacutepartition du travail choix des modes de coordination)

- le pilotage de lrsquoaction opeacuterationnelle (motivation animation encadrement

assistance)

- lrsquoeacutevaluation (controcircle des reacutesultats obtenus ajustements)

Dans chacune de ces activiteacutes des deacutecisions et des arbitrages sont neacutecessaires avec des

enjeux plus ou moins importants

Remarques

- Les cinq activiteacutes du management peuvent se retrouver agrave tout niveau de

management si lrsquoentreprise laisse une certaine autonomie de deacutecision agrave ses diffeacuterentes

uniteacutes La conception est naturellement du ressort de la direction geacuteneacuterale et des

conseils drsquoadministration mais elle peut ecirctre preacutesente pregraves du terrain (latitude laisseacutee agrave

une filiale ou agrave un magasin par exemple) De mecircme lrsquoorganisation du travail concerne

un atelier mais aussi la direction qui structure lrsquoentreprise pour assurer ses activiteacutes sa

production

- La planification deacutefinit des objectifs ou des axes strateacutegiques (choix de produits

modaliteacutes de deacuteveloppement des ventes implantations alliances hellip) et les traduit en

donneacutees de gestion preacutevisionnelles syntheacutetiques et eacutechelonneacutees dans le temps afin de

valider les objectifs et de fixer des repegraveres

- Un laquo business plan raquo (plan drsquoaffaires)est notamment lrsquoeacutequivalent de la planification

dans le cas de creacuteation drsquoentreprise ou pour la preacutesentation de tout projet drsquoactiviteacute

Les activiteacutes du management srsquoinscrivent dans des cycles qui peuvent ecirctre scheacutematiseacute

comme suit (lrsquoeacutevaluation peut entraicircner une reacutevision du pilotage de lrsquoorganisation ou des

objectifs sans que lrsquoentreprise ne soit fondamentalement remise en cause)

conception

planification

organisation

pilotage

eacutevaluation

Initiation au management copy CRCF ndash J Sornet Page 17 48

2 ndash Les contextes de management

Le management est influenceacute par son contexte qui justifie des objectifs une organisation

des meacutethodes

Par exemple lrsquoentreprise admet de nombreuses variantes selon sa taille sa forme juridique

son controcircle par lrsquoeacutetat (entreprises publiques) ou par des inteacuterecircts priveacutes Il en va de mecircme des

organismes administratifs qui peuvent deacutependre de directives nationales ou reacutegionales des

associations qui ont des activiteacutes drsquoampleur tregraves variable

21 ndash La dimension de lrsquoentreprise

La dimension drsquoune entreprise se mesure principalement en fonction de son effectif ou de

son chiffre drsquoaffaires Des seuils sont deacutefinis par divers organismes et exploiteacutes agrave des fins

statistiques ou pour la deacutetermination de certaines obligations sociales ou fiscales

(repreacutesentation du personnel cotisations hellip) Il nrsquoy a bien entendu pas de laquo barriegravere de

tailleraquo absolue conditionnant le management drsquoune entreprise

LrsquoUE preacuteconise de distinguer les micro ndash entreprises (jusqursquoagrave 9 salarieacutes) les TPE ndash tregraves petites

entreprises (moins de 20 salarieacutes) les petites entreprises (moins de 50) et les moyennes

entreprises (de 50 agrave 250) Cependant les PME sont parfois situeacutees entre 10 et 500 salarieacutes

Remarques

- en France environ 40 des entreprises emploient de 1 agrave 50 salarieacutes (ce qui repreacutesente

plus de 50 des emplois) et 59 nrsquoen ont aucun

le pays compte environ 2 600 000 entreprises dont moins de 1 ont 250 employeacutes et

plus

- ancienneteacute et taille de lrsquoentreprise sont lieacutees si lrsquoon eacutecarte les restructurations et autres

eacutevolutions drsquoentreprises existantes

La dimension de lrsquoentreprise a une influence sur lrsquoorganisation et le laquo style raquo de son

management

- les PME sont souvent entrepreneuriales (les dirigeants eacutegalement apporteurs de capitaux

sont totalement engageacutes dans la marche de lrsquoentreprise) Elles ont une gestion flexible peu

formaliseacutee plus qualitative que quantitative Les PME sont freacutequemment focaliseacutees sur un seul

type drsquoactiviteacute Pour ne pas alourdir leur structure elles ont tendance agrave sous-traiter les

activiteacutes speacutecialiseacutees ne correspondant pas agrave leur meacutetier de base

- les grandes entreprises sont manageacuteriales (les dirigeants sont nommeacutes par les actionnaires

en raison de leurs compeacutetences) et moins reacuteactives

22 ndash Le type de production

On distingue industrie (production de biens mateacuteriels ou pour le moins de produits visibles ndash

comme un seacutejour touristique ou un film) et services (fourniture drsquoune prestation immateacuterielle)

Le type de production influence en principe le management de lrsquoentreprise

- lrsquoindustrie neacutecessite (si lrsquoon excepte lrsquoartisanat) un investissement relativement important

une organisation productive stable capable de reacutealiser plusieurs fois des produits identiques

(exemple un modegravele de reacutefrigeacuterateur) ou du moins similaires (exemple un bacirctiment) Le

produit de lrsquoindustrie consomme des matiegraveres et il doit geacuteneacuteralement ecirctre distribueacute jusqursquoau

client

- la production de services peut se satisfaire drsquoun investissement tregraves reacuteduit et neacutecessite un

contact permanent avec le client

Toutefois la standardisation des services et le deacuteveloppement des reacuteseaux informatiques

rapprochent la production de services de celle des biens industriels

- la production drsquoun service reacutepeacutetitif et technique peut imposer une structure lourde et une

organisation tregraves formaliseacutee (voir les grandes socieacuteteacutes drsquoaudit ou de conseil informatique)

Initiation au management copy CRCF ndash J Sornet Page 18 48

- certains services peuvent ecirctre fournis agrave distance sans contact direct avec le client et

distribueacutes par reacuteseau (tenue de comptabiliteacute affacturage gestion clientegravele centre drsquoappel

hellip)

Remarque les services repreacutesentent 75 de lrsquoactiviteacute eacuteconomique franccedilaise

23 ndash La nature de lrsquoorganisation

Les organisations publiques franccedilaises (administrations centrales collectiviteacutes territoriales

hocircpitaux hellip) repreacutesentent une part importante de lrsquoactiviteacute (environ 30 des emplois) La

fonction publique regroupe des organisations aux finaliteacutes diverses et qui ont des problegravemes

de gestion similaires agrave ceux des entreprises auxquelles elles peuvent emprunter des principes

de management Notamment

- pour controcircler les coucircts et assurer la qualiteacute des services

- pour communiquer avec les administreacutes ou les usagers

- pour motiver les personnels et geacuterer les ressources humaines

La transposition directe des techniques de gestion et de management nrsquoest cependant pas

toujours possible car

- la comptabiliteacute publique obeacuteit agrave des regravegles speacutecifiques (proceacutedure budgeacutetaire

notamment)

- le laquo client raquo ne paye pas toujours la prestation du moins directement

- la concurrence est parfois inexistante

- les grandes administrations centraliseacutees sont soumises agrave des choix politiques geacuteneacuteraux

parfois sans connexion eacutevidente avec les besoins opeacuterationnels

- le statut des personnels et les grilles de salaires limitent les possibiliteacutes de gestion des

ressources humaines

Remarque la LOLF (loi organique relative aux lois de finances) est entreacutee en vigueur en

2006 Elle alloue des moyens budgeacutetaires en fonction de programmes et remplace la

reconduction automatique de 90 des budgets Cette reacuteforme se heurte toutefois agrave la

lourdeur des grands ministegraveres ougrave la complexiteacute des activiteacutes est difficile agrave

appreacutehender et ougrave des inerties culturelles peuvent exister agrave tout niveau

Les associations loi de 1901 peuvent avoir une activiteacute comparable agrave celle de grandes

entreprises (voir par exemple les associations de santeacute ou professionnelles) et leur

management est alors similaire malgreacute lrsquoabsence de but lucratif (les beacuteneacutefices ne sont pas

distribuables) Elles ont drsquoailleurs en France un poids eacuteconomique important (elles emploient

environ 1 600 000 salarieacutes)

Cependant lrsquoadheacutesion agrave un systegraveme de valeurs fondateur de lrsquoassociation ou la limite de

lrsquoautoriteacute (quand un volant de beacuteneacutevoles important participe agrave lrsquoactiviteacute) peut introduire des

nuances

- le renforcement des objectifs socieacutetaux

- la faiblesse des relations hieacuterarchiques

- des contraintes de gestion du temps des beacuteneacutevoles

- des modaliteacutes particuliegraveres de recrutement et de motivation des dirigeants

24 ndash Les facteurs contingents

La theacuteorie de la contingence montre qursquoune structure drsquoentreprise nrsquoest efficace que dans

une situation deacutetermineacutee et qursquoil nrsquoexiste que des solutions de management construites dans

un contexte preacutecis

Le management doit ainsi srsquoadapter agrave des facteurs contingents qui ne peuvent ecirctre

controcircleacutes du moins agrave bregraveve eacutecheacuteance Ces facteurs sont par exemple

- lrsquoancienneteacute de lrsquoentreprise (plus elle est ancienne plus lrsquoentreprise a tendance agrave reacutepeacuteter

des comportements eacuteprouveacutes)

Initiation au management copy CRCF ndash J Sornet Page 19 48

- la taille de lrsquoentreprise (la grande entreprise a une composante administrative plus

deacuteveloppeacutee)

- le systegraveme de production (tregraves standardiseacute complexe automatiseacute hellip)

- lrsquoenvironnement

3 ndash Le management et les parties prenantes

Lrsquoentreprise a pour vocation premiegravere de mettre des produits agrave disposition de ses clients en

reacutealisant un profit Pour y arriver elle doit aussi satisfaire ses parties prenantes salarieacutes

actionnaires fournisseurs hellip

Est partie prenante agrave lrsquoentreprise laquo tout groupe ou individu qui peut ecirctre affecteacute ou est

affecteacute par les buts de lrsquoorganisation hellip raquo (Freeman ndash 1984)

Les parties prenantes attendent agrave des degreacutes divers de profiter drsquoune creacuteation de valeur en

provenance de lrsquoentreprise qui doit reacutepondre agrave ces attentes pour assurer sa peacuterenniteacute ou

favoriser son deacuteveloppement

On distingue les parties prenantes primaires ou principales qui sont essentielles agrave lrsquoentreprise

et qui ont geacuteneacuteralement une relation formelle avec elle (clients associeacutes et actionnaires

precircteurs salarieacutes fournisseurs collectiviteacutes) et les parties prenantes secondaires dont

lrsquoinfluence est diffuse (groupes de pression associations meacutedias instances europeacuteennes

agences de notation hellip)

Remarque la consideacuteration de lrsquoensemble des parties prenantes (laquo stakeholders raquo - les

deacutepositaires) fait contrepoids agrave lrsquoimportance accordeacutee aux seuls actionnaires

(laquo shareholders raquo)

Les organisations nrsquoayant pas drsquoobjectif de profit doivent aussi satisfaire leurs parties

prenantes apporter un service aux usagers dans les meilleures conditions eacuteconomiques

limiter un budget assurer la qualiteacute des relations avec les fournisseurs hellip

Dans cette optique le management doit organiser lrsquoaction de faccedilon agrave eacutequilibrer des forces

parfois divergentes

- le contexte fait pression sur lrsquoorganisation contrainte agrave optimiser ses reacutesultats

- lrsquoorganisation cherche par son action agrave assurer sa peacuterenniteacute son deacuteveloppement (en

reacutealisant des profits dans le cas de lrsquoentreprise) et agrave satisfaire ses parties prenantes

- le management agit en pilotant les actions pour contrebalancer la pression du contexte

Actions de

lrsquoorganisation

Management Contexte

Parties

prenantes

Initiation au management copy CRCF ndash J Sornet Page 20 48

APPLICATIONS MP

MP1 Deacutefinir contingent gestion budgeacutetaire

MP2 Deacuteterminer les parties prenantes drsquoun hocircpital public et leurs principales attentes

Mecircme question pour les organisations suivantes

- SNCF (entreprise publique)

- Peugeot

- MAIF (mutuelle drsquoassurance)

MP3 En les situant dans le cycle des activiteacutes du management trouver les actions agrave mener

dans les situations suivantes

- baisse de 10 des ventes dans une entreprise industrielle (produits meacutenagers le reacuteseau de

distribution vient drsquoecirctre reacuteorganiseacute)

- idem dans une entreprise de vente par correspondance soumise agrave la concurrence internet

(les ventes stagnaient depuis six mois malgreacute les efforts promotionnels)

- augmentation des deacutelais drsquoattente des consultations dans une clinique (lrsquohocircpital voisin a

fermeacute son service drsquourgences)

Initiation au management copy CRCF ndash J Sornet Page 21 48

ELEMENTS DE CORRIGE MP

MP1 Deacutefinir (dans le contexte drsquoune entreprise) contingent gestion budgeacutetaire

Contingent = imposeacute par lrsquoexteacuterieur Contingence = effet du hasard de la rencontre de

plusieurs eacuteveacutenements indeacutependants (variables explicatives que lrsquoon ne peut influencer)

Gestion budgeacutetaire = technique drsquoadministration des entreprises srsquoappuyant sur des

preacutevisions dont on deacuteduit apregraves accord des responsables des attributions de moyens sur une

dureacutee limiteacutee Une analyse reacuteguliegravere des eacutecarts entre preacutevisions et reacutealisations permet ensuite

le pilotage des activiteacutes Le budget est un cadre incitatif

La laquo planification budgeacutetaire raquo consiste agrave traduire en budgets une planification strateacutegique

avec systegraveme de reporting

MP2 Deacuteterminer les parties prenantes drsquoun hocircpital public et leurs principales attentes

Mecircme question pour les organisations suivantes

- SNCF (entreprise publique)

- Peugeot

- MAIF (mutuelle drsquoassurance)

Hocircpital

- patients (qualiteacute des soins)

- CNAM (baisse des coucircts)

- collectiviteacute locale (service aux administreacutes)

- eacutetat (ameacutenagement du territoire maicirctrise des budgets optimisation)

- employeacutes (salaire conditions de travail et satisfaction)

- fournisseurs ndash pharmacie autres (CA paiement reacutegulier)

- associations de patients (qualiteacute proximiteacute des soins)

SNCF

- usagers et associations drsquousagers (proximiteacute reacutegulariteacute prix du service)

- reacuteseau ferreacute de France (optimisation des lignes paiement adapteacute)

- fournisseurs (CA paiement reacutegulier)

- employeacutes (salaire conditions de travail seacutecuriteacute de lrsquoemploi)

- eacutetat (ameacutenagement du territoire)

- collectiviteacutes locales (service)

Peugeot

- clients (qualiteacute prix SAV relation commerciale)

- fournisseurs (CA reacutegulariteacute de lrsquoactiviteacute)

- employeacutes (salaire conditions de travail seacutecuriteacute de lrsquoemploi)

- eacutetat (taxes)

- collectiviteacute locale (emploi dynamisation eacuteconomique preacuteservation de lrsquoenvironnement)

- associations de protection de lrsquoenvironnement (activiteacute propre baisse des eacutemissions

nouvelles eacutenergies)

MAIF

- socieacutetaires (protection relation assureur tarif mesureacute)

- professionnels de lrsquoautomobile et autres (agreacutement marge de manœuvre reacuteparations tarifs

eacuteleveacutes)

- fournisseurs (CA paiement reacutegulier)

- eacutetat (taxes engagement pour la seacutecuriteacute)

- employeacutes (salaire conditions de travail seacutecuriteacute de lrsquoemploi)

Initiation au management copy CRCF ndash J Sornet Page 22 48

MP3 En les situant dans le cycle des activiteacutes du management trouver les actions agrave mener

dans les situations suivantes

- baisse de 10 des ventes dans une entreprise industrielle (produits meacutenagers le reacuteseau de

distribution vient drsquoecirctre reacuteorganiseacute)

Adapter le pilotage motiver cadrer si insuffisant retoucher une organisation deacutefectueuse

- idem dans une entreprise de vente par correspondance soumise agrave la concurrence internet

(les ventes stagnaient depuis six mois malgreacute les efforts promotionnels)

Voir pilotage et organisation si une eacutevolution du meacutetier a deacutejagrave eacuteteacute initialiseacutee Sinon re-

conception (adaptation au nouveau contexte) puis planification et reacuteorganisation

- augmentation des deacutelais drsquoattente des consultations dans une clinique (lrsquohocircpital voisin a

fermeacute son service drsquourgences)

Organisation Si insuffisant planification (nouveaux objectifs)

Initiation au management copy CRCF ndash J Sornet Page 23 48

ORGANISATION ET PROCESSUS

La performance de lrsquoentreprise deacutepend de son organisation et de son aptitude agrave produire

aux meilleures conditions Nous allons montrer comment organisation formelle et processus

de production peuvent contribuer agrave cette performance

1 ndash Vers lrsquooptimum

11 ndash Les eacuteconomies occidentales jusqursquoaux anneacutees 70

Jusqursquoen 1945 le principal problegraveme des entreprises eacutetait de produire des biens en quantiteacute

suffisante agrave un prix compatible avec le marcheacute Les grandes entreprises se sont multiplieacutees et

la standardisation a permis de reacuteduire les coucircts (exemple deacuteveloppement de Ford et de la

production agrave la chaicircne de 1908 agrave 1920 qui a permis une baisse du prix des voitures des 23)

On parle de laquo production pousseacutee vers le marcheacute raquo

Cette croissance de la production peu reacuteguleacutee a eacuteteacute marqueacutee par des surproductions en

1910 et 1920 puis par la crise de 1929 qui a prolongeacute ses effets jusqursquoagrave la guerre

De 1945 agrave 1975 environ (les laquo trente glorieuses raquo) la reconstruction la croissance de la

consommation de masse de nouvelles technologies et les eacutechanges internationaux

alimentent lrsquoeacuteconomie La standardisation srsquoeacutetend aux biens de consommation dont les

coucircts baissent fortement et de nouvelles reacutegulations sociales permettent une eacutevolution sans

heurt des revenus La saturation de certains marcheacutes conduit dans les anneacutees 60 agrave la

deacutemarche laquo marketing raquo et agrave la diffeacuterenciation des produits Le produit est laquo dirigeacute par le

marcheacute raquo mais les entreprises conservent une organisation assez classique et les plus grosses

srsquointernationalisent

12 ndash Lrsquoexpeacuterience japonaise et ses prolongements

Tregraves tocirct apregraves la guerre dans un Japon appauvri le constructeur automobile Toyota a ducirc

faire face agrave une restriction du marcheacute des moyens financiers et productifs et des

approvisionnements La firme a donc innoveacute dans un nouveau systegraveme de production

chassant les laquo gaspillages raquo (temps drsquoattente transports stocks deacutefauts hellip) consideacuterant que

seule la fabrication vendable creacutee de la valeur

Toyota srsquoorganise pour fabriquer la quantiteacute et la qualiteacute de produits juste neacutecessaires agrave la

satisfaction des clients la production est laquo tireacutee par le marcheacute raquo La mise en place de ce

systegraveme qui integravegre les fournisseurs ne sera acheveacutee que dans le milieu des anneacutees 70

En 1973 la hausse du peacutetrole inaugure un ralentissement de la croissance des eacuteconomies

occidentales La concurrence accrue provoque alors un inteacuterecirct pour le systegraveme deacuteveloppeacute

au Japon La production au plus juste se deacuteveloppe ainsi dans lrsquoindustrie automobile agrave partir

des anneacutees 80 et elle se reacutepand encore maintenant dans drsquoautres secteurs

Cette approche qui vise un objectif de zeacutero stock et zeacutero deacutefaut impose la maicirctrise de laquo bout

en bout raquo des processus de production et leur ameacutelioration

Initiation au management copy CRCF ndash J Sornet Page 24 48

2 ndash Organiser lrsquoentreprise

21 ndash Direction et organisation

Diriger une entreprise neacutecessite de lrsquoorganiser (de reacutepartir les tacircches) pour qursquoelle puisse

atteindre ses objectifs Lrsquoorganisation permet de satisfaire un marcheacute en tirant parti des

capaciteacutes actuelles de lrsquoentreprise tout en preacuteparant lrsquoavenir

Lrsquoorganisation reacutesulte freacutequemment drsquoun compromis entre des objectifs situeacutes agrave des niveaux

et des eacutecheacuteances diffeacuterents

Exemples

- le leader des chaises roulantes peut tirer profit de sa structure productive et de son

savoir faire pour entrer sur le marcheacute de la bicyclette eacutelectrique

- ecirctre parfaitement structureacute pour alimenter 90 du marcheacute des disquettes ne preacutepare

pas lrsquoavenir

- srsquoorganiser pour conqueacuterir le marcheacute des tire-bouchons eacutelectriques dans les deux ans

perd de son sens si cela altegravere les moyens neacutecessaires agrave la production drsquoappareils

manuels ancienne mais vitale dont la diminution agrave court terme risque de nuire agrave la

solvabiliteacute de lrsquoentreprise et de la conduire agrave la cessation de paiement

22 ndash Lrsquoorganisation fonctionnelle

La majoriteacute des entreprises adopte une laquo organisation fonctionnelle raquo (celle qui est visible

dans les organigrammes) ougrave des regroupements de personnels et drsquoeacutequipements se font

selon un modegravele hieacuterarchique (laquo line raquo) dans des uniteacutes des services ou des deacutepartements

speacutecialiseacutes Cette organisation peut se deacutecliner agrave lrsquointeacuterieur des divisions des grandes

entreprises quand elles scindent leur activiteacute par zone geacuteographique type drsquoactiviteacute

cateacutegorie de clients hellip

Remarque le terme laquo fonction raquo deacutesigne un rocircle particulier dans le fonctionnement de

lrsquoentreprise

Lrsquoorganisation fonctionnelle diffeacuterencie les activiteacutes de lrsquoentreprise en les regroupant par

meacutetier pour utiliser au mieux les compeacutetences et les moyens (meilleur rendement par la

speacutecialisation lrsquoeacutechange de compeacutetences dans une mecircme uniteacute ou gracircce agrave des eacuteconomies

drsquoeacutechelle)

23 ndash La notion de processus de production

Un processus de production se deacutefinit par la succession drsquoactiviteacutes permettant de satisfaire

un client en transformant des ressources (mateacuterielles financiegraveres humaines) en un produit

bien ou service Le processus doit creacuteer une valeur reconnue par le client

Un processus peut servir un client interne agrave lrsquoentreprise (par exemple en produisant un

composant intervenant dans plusieurs produits ou par la maintenance des machines) aussi

bien qursquoun client final On distingue usuellement

- les processus opeacuterationnels (ou maicirctres) aussi appeleacutes processus meacutetier (business process)

qui satisfont directement les clients finaux (conception et fabrication de produits vente hellip)

- les processus de support et de management (geacuterer les ressources humaines geacuterer

lrsquoinformation geacuterer les ressources financiegraveres hellip) qui ont les processus opeacuterationnels comme

clients

Toutes les actions internes agrave une organisation peuvent srsquointeacutegrer dans des processus qui

conditionnent directement ou indirectement la capaciteacute de lrsquoorganisation agrave satisfaire le

client final ou lrsquousager

Initiation au management copy CRCF ndash J Sornet Page 25 48

Aborder le fonctionnement de lrsquoentreprise par ses processus (approche processus) permet

de mettre en eacutevidence les chaicircnes drsquoactiviteacutes qui conduisent aux produits leurs

dysfonctionnements leurs coucircts la formation des deacutelais et la souplesse (la flexibiliteacute)

disponible pour satisfaire la clientegravele finale Lrsquoameacutelioration des processus a un impact visible

et direct sur chaque produit proposeacute aux clients

Lrsquoapproche processus provoque une eacutevolution de la faccedilon de travailler

- en faisant peacuteneacutetrer la laquo voix du client raquo au plus profond de lrsquoentreprise (et plus seulement

dans les services commerciaux et marketing)

- en mettant en eacutevidence des possibiliteacutes de rationalisation (par regroupement ou impartition

de certaines activiteacutes)

Remarque lrsquoapproche par les activiteacutes et les processus est agrave lrsquoorigine de la meacutethode

de deacutetermination des coucircts laquo ABC raquo - activity based costing

24 ndash Processus et fonctions

Le processus est transversal Il enchaicircne des activiteacutes qui traversent lrsquoentreprise en particulier

les services ou les deacutepartements drsquoune organisation fonctionnelle

Exemple

La division du travail par fonctions induit une charge de coordination pour assurer le

deacuteroulement du processus Elle peut geacuteneacuterer des attentes des erreurs ou des conflits drsquointeacuterecirct

(lrsquoobservation montre que des dysfonctionnements sont tregraves souvent constateacutes lors du

passage drsquoun service agrave un autre)

Organisation fonctionnelle et approche processus visent toutes deux un optimum

eacuteconomique mais leurs logiques sont diffeacuterentes

- le processus vise la satisfaction des clients (prix qualiteacute deacutelais service)

- le deacutecoupage fonctionnel cherche agrave optimiser les moyens (maximiser lrsquoeffet drsquoexpeacuterience

partager des infrastructures profiter de pocircles de compeacutetences hellip) Il apporte une ossature

hieacuterarchique stable souvent indispensable

Organisation fonctionnelle et approche processus sont donc compleacutementaires dans la

majoriteacute des cas et doivent ecirctre combineacutees judicieusement

APPLICATIONS OP

OP1 Deacutefinir flexibiliteacute systegraveme impartition

OP2 Citer huit exemples drsquoinformations essentielles pour optimiser un processus de

fabrication

Direction

Deacutepartement

commercial

(C)

Deacutepartement

administratif et

financier (AF)

Deacutepartement

Etudes (E)

Deacutepartement

Production (P)

Activiteacute

C-x Activiteacute

AF-x Activiteacute

E-x

Activiteacute

P-x

Processus x

Clie

nt

Initiation au management copy CRCF ndash J Sornet Page 26 48

OP3 Deacutegager les principes du toyotisme preacutesenteacute ci-dessous En quoi ce systegraveme est-il

initiateur de lrsquoapproche processus

Taiichi Ohno et le Toyotisme

1 - Extrait drsquoun article de Jacques BARRAUX - 1993 - LExpansion

Taiichi Ohno (1912 ndash 1990) hellip ne se prenait pas pour un visionnaire mais en imposant une

nouvelle faccedilon de produire il a reacuteinventeacute le management hellip tout le monde a entendu parler

des mots qui ont populariseacute le toyotisme dont il est le pegravere le juste-agrave-temps hellip Autant

doutils conccedilus pour lrsquoautomobile et qui ont aujourdhui une application universelle

hellip Taiichi Ohno jeune ingeacutenieur entre chez Toyota alors simple constructeur de machines

textiles Degraves 1926 apparaicirct la notion de jidoka hellip cest lart de transfeacuterer de lintelligence aux

machines pour mieux libeacuterer lintelligence des hommes Tout le contraire du taylorisme qui

juge la machine moins impreacutevisible que lhomme En 1933 Toyota se lance dans lautomobile

en sinspirant des meacutethodes ameacutericaines Mais en 1935 agrave loccasion dun voyage aux Etats-

Unis leacutetat-major de lentreprise revient fascineacute de sa visite dans un supermarcheacute La notion

de juste-agrave-temps va naicirctre de lobservation dune grande surface un lieu ougrave les clients ne

prennent que ce dont ils ont besoin et ougrave les rayons sont reacuteapprovisionneacutes pour compenser

les quantiteacutes preacuteleveacutees Ainsi le systegraveme Toyota est-il deacutejagrave dans la tecircte de ses dirigeants avant

mecircme la Seconde Guerre mondiale un demi-siegravecle avant la reacutevolution informatique et la

segmentation intensive des marcheacutes

hellip des esprits curieux comme Franccedilois Dalle en France tombent alors sous le charme des

formules et des paraboles de Taiichi Ohno En voici deux eacutechantillons

Penser agrave lenvers Cela signifie combattre les ideacutees reccedilues En lespegravece il sagit du fordisme et

du taylorisme Ohno ne croit pas agrave la planification aux effets deacutechelle et dexpeacuterience Il

propose un systegraveme industriel agrave lenvers qui permette de diversifier les produits et de les

fabriquer en petites quantiteacutes Nous ne devons plus ecirctre des paysans qui accumulent des

stocks mais des chasseurs On nimpose pas loffre On traque la demande et on la gegravere en

continu

Que les valleacutees soient hautes et les montagnes peu eacuteleveacutees Plutocirct que de concentrer tous

les efforts sur une production agrave un moment donneacute mieux vaut se doter de structures flexibles

permettant de passer agrave tout instant dune seacuterie agrave une autre Il faut eacuteviter les ruptures et les

secousses aplanir les cycles entretenir des flux reacuteguliers dactiviteacutes diversifieacutees Ce qui

implique de ne pas enfermer les hommes et les eacutequipements dans des speacutecialisations trop

eacutetroites

La flexibiliteacute le travail en groupe le refus de la dictature des machines la polyvalence et

surtout lattention constante aux signaux eacutemis par le marcheacute nappartiennent plus au

toyotisme Ces notions sont les fondements du nouvel art dorganiser de vendre et de

produire dans lindustrie comme dans les services hellip

2 - Quelques notions cleacutes

Taiichi Ohno a imagineacute la meacutethode des laquo cinq pourquoi raquo qui consiste agrave se poser cinq fois de

suite la question laquo pourquoi raquo sur le mecircme sujet de faccedilon agrave deacutecouvrir la veacuteritable cause

drsquoun problegraveme Cette meacutethode peut ecirctre appliqueacutee agrave tous les niveaux et permettre

notamment aux agents de fabrication de proposer de veacuteritables ameacuteliorations de la

production

La recherche de la qualiteacute totale (pas de deacutefaut des produits pas de rebuts pas de deacutefaut

des processus) accompagne la deacutemarche de Toyota La qualiteacute a un coucirct compenseacute par

des ventes accrues par lrsquoeacuteconomie des mesures palliatives aux deacutefauts

Initiation au management copy CRCF ndash J Sornet Page 27 48

Fiche OP1 ndash Benchmarking et processus

Le laquo benchmarking raquo consiste agrave comparer le fonctionnement de plusieurs systegravemes pour en

faire notamment ressortir les meilleures pratiques (laquo best practices raquo) Cette technique est

utiliseacutee depuis les anneacutees 80 pour ameacuteliorer la performance des entreprises Elle impose agrave

lrsquoentreprise drsquoeacutevaluer et de remettre en question ses propres modes de fonctionnement afin

de les faire eacutevoluer agrave la lueur de ce qui se fait ailleurs

Le benchmarking permet drsquoameacuteliorer les processus agrave moindre risque en fixant des objectifs

baseacutes sur des faits et donc plus facilement accepteacutes

Une classification des processus en tant que base de reacuteflexion a eacuteteacute eacutetablie aux USA par

lrsquolaquo International Benchmarking Clearinghouse raquo de lrsquoAPQC (american productivity and

quality center) en collaboration avec plusieurs dizaines drsquoentreprises

Elle se reacutesume ainsi

Le terme laquo reengineering raquo (la re-conception ou laquo reacuteingeacutenieacuterie raquo) des processus deacutesigne un

projet drsquoameacutelioration radicale des performances (de 20 agrave 50 ou plus) Il neacutecessite une

parfaite adheacutesion de la direction la constitution drsquoune petite eacutequipe de projet brillante

connaissant parfaitement les activiteacutes de lrsquoentreprise et il peut inclure un benchmarking

Le reengineering provoque geacuteneacuteralement la reacuteduction du nombre de niveaux hieacuterarchiques

(laquo delayering raquo) et lrsquoaccroissement du pouvoir de deacutecision des employeacutes (laquo empowerment raquo

ou laquo empouvoirement raquo) Bien qursquoy conduisant parfois il ne doit pas ecirctre confondu avec la

reacuteduction des activiteacutes (laquo downsizing raquo ou restructuration) et lrsquoexternalisation (laquo outsourcing raquo)

Pro

ce

ssu

s

op

eacutera

tio

nn

els

Pro

ce

ssu

s d

e m

an

ag

em

en

t e

t d

e

sup

po

rt

1 ndash

Comprendre

le marcheacute et

les clients (besoins

satisfaction)

2 ndash

Deacutevelopper

vision et

strateacutegie (contexte

concurrence)

3 ndash

Creacuteer

produits

services

processus

(concevoir

ameacuteliorer)

4 ndash

Marketing et

vente

5 ndash

Produire et

livrer (industrie

dont

ameacutelioration

processus)

6 ndash

Produire et

livrer (services)

7 ndash

Facturer et

servir les

clients (apregraves-

vente

reacuteclamations)

8 ndash Deacutevelopper et geacuterer les ressources humaines

9 ndash Geacuterer les systegravemes drsquoinformation

10 ndash Geacuterer les ressources financiegraveres et les actifs

11 ndash Appliquer un programme environnemental

12 ndash Geacuterer les relations exteacuterieures (actionnaires banques lois relations publiques hellip)

13 ndash Geacuterer lrsquoameacutelioration et le changement (eacutevaluer mesurer motiver qualiteacute totale)

Initiation au management copy CRCF ndash J Sornet Page 28 48

Fiche OP2 ndash Lrsquoorganisation par processus

Lrsquoeacutevolution drsquoune organisation aux activiteacutes reacutepeacutetitives vers lrsquoapproche processus est

geacuteneacuteralement progressive et se met en place par paliers

La mise en œuvre drsquoun veacuteritable management par processus doit ecirctre preacuteceacutedeacutee quand

lrsquoactiviteacute de lrsquoentreprise est complexe drsquoun recensement (une laquo cartographie des

processus raquo) pour mettre en eacutevidence les processus ou les familles de processus cleacutes critiques

pour le succegraves de lrsquoentreprise ougrave les efforts seront prioritaires

Des responsables de processus (laquo process owners raquo) sont ensuite deacutesigneacutes

Le responsable doit concevoir ses processus puis apregraves leur mise en œuvre assurer les

coordinations neacutecessaires les ameacuteliorer et les repreacutesenter aupregraves de la direction

Quand une structure par processus est mise en place des opeacuterateurs exeacutecutants

preacuteceacutedemment regroupeacutes dans les fonctions peuvent ecirctre affecteacutes aux processus et

drsquoanciens responsables de fonctions peuvent devenir des experts au service des processus

Lrsquoorganisation par processus peut imposer un degreacute eacuteleveacute drsquointeacutegration des activiteacutes donc

une polyvalence accrue des personnels et une reacuteduction des niveaux hieacuterarchiques

Elle neacutecessite pour le moins des compeacutetences eacutelargies au niveau des responsables de

processus (organisation administration technique hellip) dont le nombre doit rester limiteacute

(quelques dizaines au plus)

Sauf dans de tregraves petites structures lrsquoorganisation par processus se plaque geacuteneacuteralement sur

une structure plus classique

Initiation au management copy CRCF ndash J Sornet Page 29 48

ELEMENTS DE CORRIGE OP

OP1 Deacutefinir

Flexibiliteacute = adaptation au besoin (horaire variable chaicircnes robotiseacutees)

Systegraveme = ensemble organiseacute dans un but boicircte noire (sanguin nerveux meacutetrique laquo D raquo)

Impartition = sous-traitance ou externalisation (seacuteparation) drsquoactiviteacutes faire appel agrave des

partenaires plutocirct que faire soi-mecircme

OP2 Citer huit exemples drsquoinformations essentielles pour orienter lrsquooptimisation drsquoun processus

Montant des stocks (approvisionnements et produits finis)

Temps drsquoattente

Taux drsquoactiviteacute des ateliers

Rebuts

Deacutelai de production

Taux de reacuteclamations clients (qualiteacute)

Temps passeacutes en retouches finales

Turn over

Nombre drsquoarrecircts maladie

Accidents du travail

Dureacutee des arrecircts machines

OP3 Deacutegager les principes du toyotisme preacutesenteacute dans la fiche 31 En quoi ce systegraveme

repose trsquoil sur lrsquoapproche processus

Produire la quantiteacute juste neacutecessaire (agrave la demande) donc eacuteviter les stocks

Flexibiliteacute intelligence des chaicircnes de production

Qualiteacute (eacuteviter le coucirct de la non-qualiteacute)

La notion de processus est implicite ainsi que la chaicircne de valeur client

Initiation au management copy CRCF ndash J Sornet Page 30 48

DEFIS ET TENDANCES DU MANAGEMENT

Les meacutethodes de management se deacuteveloppent pour affronter le contexte eacuteconomique

Ce chapitre preacutesente les deacutefis auxquels le management contemporain doit faire face

1 ndash Lrsquoeacutevolution eacuteconomique contemporaine

A mesure que lrsquoactiviteacute eacuteconomique mondiale srsquoaccroicirct que la technologie eacutevolue les

changements sont de plus en plus rapides Ils introduisent des situations ineacutedites auxquelles les

entreprises doivent srsquoadapter en cherchant de nouvelles solutions de management Les trois

derniegraveres deacutecennies ont eacuteteacute notamment marqueacutees par les pheacutenomegravenes suivants (que nous

listons sans tenir compte des liens pouvant exister entre eux)

Pheacutenomegravene Traduction Effets

Deacute reacuteglementation

globalisation

financiegravere

titrisation

Libre circulation des capitaux accegraves

facile des particuliers au marcheacute

boursier (directement ou par

lrsquointermeacutediaire des OPCVM et SICAV)

Monteacutee en puissance du financement

des entreprises sur le marcheacute boursier

Fonds de pension

(retraites) et fonds

souverains (eacutetats)

Poids boursier important drsquoinvestisseurs

institutionnels qui cherchent un haut

rendement financier (dividendes ou

valorisation boursiegravere)

Pression sur les grandes entreprises

influence sur les strateacutegies

Mondialisation Liberteacute des eacutechanges internationaux Accroissement de la concurrence

recherche drsquoavantages eacuteconomiques

par la deacutelocalisation (biens et

services) la concentration des efforts

(recentrage) problegravemes drsquoemploi

multiplication des transports perte

drsquoinfluence des politiques

Baisse de lrsquoemploi

occidental

(notamment

industriel)

Moins de fabrications fabrications

automatiseacutees recours aux moyens

informatiques

Activiteacute reporteacutee sur le commerce la

conception et les services chocircmage

charge sociale

Restructurations Optimisation des entreprises

abaissement des coucircts augmentation

des marges recherche drsquoune taille

critique (eacuteconomies drsquoeacutechelle poids

sur le marcheacute)

Recentrages externalisations fusions

deacutelocalisations constitution de grands

groupes

NTIC (nouvelles

technologies de

lrsquoinformation et de

la communication)

Mise en œuvre des reacuteseaux (dont

internet) et drsquoapplications

informatiques communicantes

Nouvelles formes de commerce

marcheacute international deacutelocalisation

du travail intellectuel reacuteorganisation

de la distribution

Rareacutefaction relative

des matiegraveres

premiegraveres

Recherche de substituts exploration

miniegravere coucircts drsquoexploitation des

gisements accrus

Augmentation des coucircts variations

erratiques du cours des matiegraveres

deacutestabilisations politiques

Evolution

geacuteopolitique et

eacuteconomique

mondiale

Chute de lrsquoURSS transformation des

eacuteconomies collectivistes pays

eacutemergents (Chine Inde Breacutesil Russie)

()

Accroissement de la population

mondiale (4 agrave 6 7 milliards de 1970 agrave

2008)

Libeacuteralisme sans frein () nouvelles

puissances eacuteconomiques

opportuniteacutes de deacuteveloppement

nouveau partage des ressources

ineacutegaliteacutes baisse du soutien aux PVD

laquo Terrorisme raquo Actions armeacutees pression de groupes

armeacutes non gouvernementaux

Deacutestabilisations reacutegionales charge

des deacutepenses militaires

Deacuteveloppement

durable

Recherche drsquoune croissance eacutequitable

et respectueuse de lrsquoenvironnement

Pression sur les entreprises (eacutetats

associations de consommateurs

eacutecologistes ONG)

() Reacutecemment quelques affaires (Enron laquo subprimes raquo Vivendi Universal Socieacuteteacute

Geacuteneacuterale Airbus par exemple) et agrave plus grande eacutechelle la crise financiegravere de 2008 ont

montreacute les dangers drsquoune libeacuteralisation sans controcircles suffisants

Initiation au management copy CRCF ndash J Sornet Page 31 48

() Des alliances eacuteconomiques naissent entre pays eacutemergents (notamment en

ameacuterique centrale creacuteation de la Banque du Sud en 2008 par exemple) et lrsquoon

commence agrave imaginer une baisse progressive de lrsquoinfluence eacuteconomique des Etats

Unis

2 ndash Les deacutefis actuels du management

21 ndash Les grandes orientations

Lrsquoeacutevolution eacuteconomique suggegravere quelques pistes parfois concurrentes pour lrsquoaction du

manager contemporain On y retrouve au premier plan la construction drsquoune vision qui est

une composante commune du leadership

Objectif du manager

pour lrsquoentreprise

Justification Facteurs de reacuteussite

Construire une vision Eclairer lrsquoavenir de lrsquoentreprise partager

un but souder motiver

Effort de reacuteelle prospection

volontarisme de la direction

bonne communication

Reacuteactiviteacute et flexibiliteacute

(sous tous les aspects

agrave tous niveaux)

Srsquoadapter rapidement au marcheacute Bonne organisation des processus

personnel compeacutetent autonome

et motiveacute structure hieacuterarchique

alleacutegeacutee robotisation

Deacutegager des profits Reacutemuneacuterer les apporteurs de capitaux

srsquoautofinancer

Ajuster coucircts et structures

Exploiter les nouvelles

technologies

Reacuteactiviteacute ajuster coucircts et deacutelais

reacutepondre au marcheacute suivre les clients

Organiser le SI de faccedilon

pertinente eacuteviter le coucirct excessif

drsquoinvestissements trop en

laquo pointe raquo (laquo essuyer les placirctres raquo)

utiliser judicieusement les services

exteacuterieurs

Bacirctir des alliances

(contrats fusions)

Deacutevelopper une activiteacute limiter les coucircts

de transaction () atteindre la taille

critique et de meilleurs rendements se

recentrer sur une activiteacute profitable

Dominer les processus se donner

une identiteacute lisible externaliser se

doter drsquoune capaciteacute financiegravere

suffisante

Valoriser lrsquoimage Attirer les clients favoriser les alliances

donner confiance (apporteurs de fonds

employeacutes clients partenaires socieacuteteacute

civile)

Instaurer des regravegles de

gouvernance inteacutegrer le

deacuteveloppement durable

respecter lrsquoenvironnement

Geacuterer les risques Faire face aux aleacuteas eacuteconomiques et

technologiques (conjoncture politiques

accidents malveillance)

Creacuteer un systegraveme drsquoalerte geacuterer

la crise (reacuteaction raisonneacutee

sceacutenarios poursuite de

lrsquoexploitation dans un contexte

instable) mise en place de

proceacutedures drsquoapprentissage pour

ameacuteliorer les reacuteactions au fil du

temps

Geacuterer le changement Faire face agrave lrsquoeacutevolution de la demande

la pression sur les prix la variation des

performances financiegraveres la

concurrence la globalisation des

marcheacutes lrsquoeacutevolution technologique aux

fusions ou alliances aux changements

de reacuteglementation de direction hellip ()

Bonne communication pour

donner du sens au changement

et obtenir lrsquoadheacutesion du personnel

Rassembler et geacuterer les

connaissances former le

personnel

Innover Garder un avantage concurrentiel se

diffeacuterencier

Veille technologique et

commerciale investissement

Ouverture

internationale

Elargir le marcheacute saisir les opportuniteacutes Veille commerciale partenariats

() La theacuteorie des coucircts de transaction deacuteveloppeacutee par OE Williamson dans les

anneacutees 70 integravegre les coucircts lieacutes au recours au marcheacute (recherche et choix drsquoun

fournisseur neacutegociation reacutedaction de contrat suivi des eacutechanges risque de rupture

Initiation au management copy CRCF ndash J Sornet Page 32 48

drsquoapprovisionnement hellip) On peut en conclure que lrsquointeacutegration de diffeacuterentes

activiteacutes agrave lrsquoentreprise (la laquo firme raquo) preacutesente des avantages Mais des coucircts de

transaction internes doivent aussi ecirctre consideacutereacutes (preacuteparation organisation

surveillance hellip) et certaines formes de coopeacuteration continue avec les fournisseurs

permettent de reacuteduire le coucirct des transactions externes

() drsquoapregraves laquo Les meilleures pratiques de management raquo - Brilman Heacuterard ndash EO

Une eacutetude du Conference Board (2002) liste les deacutefis du management vus par 700 leaders

mondiaux Soit en reacutesumeacute avec indication du score correspondant

1 ndash Fideacuteliser les clients (42)

2 ndash Reacuteduire les coucircts (38)

3 ndash Accroicirctre flexibiliteacute et reacuteactiviteacute (29)

4 ndash Amener les employeacutes agrave adheacuterer aux valeurs et visions de lrsquoentreprise (26)

5 ndash Deacutevelopper et retenir les leaders (25)

6 ndash Geacuterer acquisitions et alliances (24)

7 ndash Accroicirctre lrsquoinnovation (20)

En fin de classement citoyenneteacute et reacuteputation (4) et ameacutelioration de la diversiteacute (3)

22 ndash Les techniques disponibles

Pour faire face aux deacutefis le manager dispose de nouveaux concepts et de nouvelles

techniques Le tableau ci-dessous en donne un reacutesumeacute et indique les domaines qursquoils

influencent principalement

Initiation au management copy CRCF ndash J Sornet Page 33 48

Principaux concepts techniques outils Incidence principale sur

Internet

- e-commerce (commerce eacutelectronique site

entreprise)

- CRM ou GRC (gestion de la relation client)

- e-procurement (gestion des approvisionnements

par le reacuteseau)

- messagerie eacutelectronique

- e-recrutement

Vente accegraves au marcheacute

Relation client reacuteactiviteacute personnalisation

fideacutelisation

Deacutelais coucircts

Communication transfert de donneacutees (piegraveces

jointes) tous domaines

Communication recrutement

Intranet reacuteseau drsquoentreprise SI

- knowledge management (gestion des

connaissances)

- e-learning (apprentissage en ligne)

- plateforme de travail collaboratif (groupware)

- workflow (circulation eacutelectronique de

documents enchaicircnement de processus)

- e-RH portail RH (libre accegraves aux postes agrave

pourvoir informations candidatures hellip)

- PGI (progiciel de gestion inteacutegreacute) ou ERP

Innovation capaciteacute au changement veille

documentaire

Formation du personnel accompagnement des

changements

Coordination communication interne

Coordination

Communication interne (voire internet en

externe) reacuteduction des coucircts climat drsquoentreprise

recrutement plans de carriegraveres hellip

Coucircts fiabiliteacute du systegraveme drsquoinformation deacutelais

processus (continuiteacute inteacutegration)

Logistique inteacutegreacutee

Supply Chain Management (SCM) gestion de la

logistique (incluant les approvisionnements)

Processus deacutelais coucircts

Externalisation

Valorisation du capital humain

GPEC (gestion preacutevisionnelle des emplois et

compeacutetences)

Coaching

Reacuteactiviteacute de lrsquoentreprise conservation des

compeacutetences rendements individuels turn-over

adaptation des compeacutetences motivation

Efficaciteacute individuelle controcircle reacutegulation

progregraves processus

Approche processus

Optimisation des processus

Deacutemarche qualiteacute totale (TQM ndash total quality

management)

Empowerment (empouvoirement)

Benchmarking reacuteingeacutenieacuterie

Coucircts marges qualiteacute deacutelais flexibiliteacute

externalisation eacutelargissement des compeacutetences

organisation

Ameacutelioration des processus (meacutetiers et supports)

Autonomie compeacutetences des employeacutes

Ameacutelioration des processus restructuration

Management par la valeur

Parties prenantes

Satisfaction des parties prenantes financement

motivation collaborations hellip

Collaboration inter organisations

Reacuteseaux drsquoentreprises alliances

EDI (eacutechange de donneacutees informatiseacutees) extranet

Impartition externalisation (outsourcing)

Coucircts recentrage investissements lancement

drsquoactiviteacute

Coucircts reacuteactiviteacute deacutelais relations avec

lrsquoadministration

Coucircts recentrage limitation des investissements

Ethique drsquoentreprise

Gouvernance drsquoentreprise (mode de direction

encadreacute par des regravegles)

Rocircle socieacutetal deacuteveloppement durable

environnement

Image de lrsquoentreprise reacutegulation du top

management relations actionnaires

Image peacutenaliteacutes et amendes objectifs

strateacutegiques

Initiation au management copy CRCF ndash J Sornet Page 34 48

23 ndash Le rocircle socieacutetal des entreprises

La responsabiliteacute socieacutetale de lrsquoentreprise (RSE) deacutesigne le rocircle qursquoelle prend dans la socieacuteteacute

au-delagrave de son activiteacute purement geacuteneacuteratrice de profit On parle aussi drsquoentreprise citoyenne

La RSE est indissociable du deacuteveloppement durable de porteacutee mondiale et dont les trois

piliers sont

- eacuteconomique (favoriser le deacuteveloppement les eacutechanges internationaux)

- social (accegraves aux soins eacuteducation conditions de travail hellip)

- environnemental (pollution preacuteservation des ressources hellip)

La RSE integravegre notamment une preacuteoccupation sociale de lrsquoentreprise vis-agrave-vis de ses salarieacutes

(seacutecuriteacute et santeacute au travail juste reacutemuneacuteration deacuteveloppement personnel hellip) Elle conduit agrave

tenir compte dans le management drsquoune vision exteacuterieure agrave lrsquoentreprise qui peut avoir des

reacutepercussions possibles sur son activiteacute eacuteconomique

Lrsquoentreprise peut aussi tirer avantage drsquoune deacutemarche responsable par la baisse de certains

coucircts (plus faibles consommations drsquoeacutenergies reacuteduction des transports hellip)

Le rocircle socieacutetal de lrsquoentreprise a eacuteteacute reconnu en France par la loi laquo NRE raquo de 2001 (loi sur les

nouvelles reacutegulations eacuteconomiques) qui oblige les socieacuteteacutes franccedilaise coteacutees sur un marcheacute

reacuteglementeacute agrave rendre compte dans leur rapport annuel de leur gestion sociale et

environnementale au travers de leur activiteacute

Article 116 de la loi Le rapport viseacute agrave larticle L 225-102 rend compte hellip laquo Il comprend

eacutegalement des informations dont la liste est fixeacutee par deacutecret en Conseil dEtat sur la

maniegravere dont la socieacuteteacute prend en compte les conseacutequences sociales et

environnementales de son activiteacute Le preacutesent alineacutea ne sapplique pas aux socieacuteteacutes

dont les titres ne sont pas admis aux neacutegociations sur un marcheacute reacuteglementeacute raquo

Une norme ISO 14000 integravegre ces preacuteoccupations et des taxes eacutecologiques sont

progressivement creacutees

3 ndash Le management par la valeur

31 ndash De lrsquoanalyse au management par la valeur

Lrsquoanalyse de la valeur est neacutee en 1947 aux Etats-Unis (General Electrics) Cette technique

consiste agrave eacutelaborer des produits conformes aux attentes de la clientegravele mais sans excegraves pour

trouver un bon compromis entre valeur pour le client et coucirct Le produit optimal est deacutefini agrave

partir drsquoenquecirctes qui deacuteterminent le besoin client (ou plutocirct drsquoun client laquo type raquo)

Exemple il est inutile de concevoir un petit veacutehicule citadin capable de parcourir

500 000 km sans avarie compte tenu des effets de mode et du faible kilomeacutetrage

annuel Par contre le marcheacute peut exiger un fonctionnement sans faille sur 150 000 km

soit dix ans en moyenne ce qui conditionne les coucircts de production

Cette recherche drsquoun ajustement de valeur au besoin des clients eacutetait un preacutecurseur du

management par la valeur qui recherche plus largement la creacuteation de valeur pour

chacune des parties prenantes de lrsquoentreprise tout en lui meacutenageant un reacutesultat suffisant

Plus geacuteneacuteralement le management par la valeur est deacutefini par une norme europeacuteenne (EN

12973)

Le management par la valeur est un style de management particuliegraverement destineacute agrave

mobiliser les individus agrave deacutevelopper les compeacutetences et agrave promouvoir les synergies et

Initiation au management copy CRCF ndash J Sornet Page 35 48

linnovation avec pour objectif la maximisation de la performance globale dun

organisme Le management par la valeur apporte une nouvelle faccedilon dutiliser nombre

de meacutethodes de management existantes Il est en coheacuterence avec le Management

de la qualiteacute

Cette approche du management pose de nombreuses questions notamment quelles

prioriteacutes et quelles valeurs attribuer aux parties prenantes comment appreacutehender la

perception par les parties prenantes de la valeur qui leur est affecteacutee

32 ndash La valeur client

Le processus drsquoeacutelaboration drsquoun produit qui consomme des ressources coucircteuses doit creacuteer

une valeur suffisante pour provoquer lrsquoachat par le client final La production drsquoune valeur

reconnue par le client est vitale pour lrsquoentreprise mais sa deacutetermination est parfois complexe

La valeur du produit perccedilue par le client integravegre des eacuteleacutements en partie subjectifs

- une valeur drsquousage (le produit reacutepond agrave un besoin)

- une valeur drsquoestime (lrsquoimage apporteacutee par le produit un aspect affectif)

- une valeur drsquoeacutechange (deacuteduite de lrsquoespoir de revente du produit)

Valeurs drsquousage drsquoestime et drsquoeacutechange deacutependent implicitement de la qualiteacute (un bien peu

fiable est impropre agrave lrsquousage attendu de mauvaise qualiteacute notoire il nrsquoapporte pas une

image positive et ses deacutefauts connus nuisent agrave sa revente) Une eacutevaluation de la qualiteacute

intervient donc dans la valeur perccedilue du produit

Par ailleurs le client considegravere le coucirct drsquoobtention du produit (les charges qursquoil doit supporter

pour acqueacuterir le produit lrsquoeffort qursquoil doit faire pour trouver le produit et les frais de mise agrave

disposition)

Le prix perccedilu par le client est geacuteneacuteralement supeacuterieur au prix de vente

Le client achegravete theacuteoriquement le produit qui preacutesente la diffeacuterence valeur perccedilue ndash prix

perccedilu la plus favorable ou le meilleur rapport prix perccedilu qualiteacute perccedilue et dans certains

cas celui qui a le prix produit le plus bas

Remarque les valeurs du scheacutema ci-dessus changent durant le cycle de vie du produit

(un nouveau produit peut avoir une valeur perccedilue plus eacuteleveacutee qursquoen fin de vie) La

valeur client ne peut ecirctre eacutevalueacutee que par enquecirctes et ne peut donc ecirctre deacutefinie avec

certitude

La notion de laquo satisfaction client raquo conseacutecutive agrave une vente influence aussi le prix produit et

le prix perccedilu

- lrsquoentreprise gagne sur les coucircts de recherche de clientegravele

- le client nrsquoa pas agrave rechercher un nouveau fournisseur et beacuteneacuteficie drsquoun coucirct drsquoobtention

plus bas

valeur perccedilue client

prix perccedilu client

coucirct produit Marge (valeur creacuteeacutee pour

lrsquoentreprise)

euros

prix produit

Valeur creacuteeacutee

pour le client

Initiation au management copy CRCF ndash J Sornet Page 36 48

La satisfaction du client deacutepend de facteurs qualitatifs aussi divers que la fiabiliteacute du produit

la vitesse de reacuteaction du fournisseur lrsquoattitude des commerciaux lrsquoefficaciteacute du service

apregraves-vente la netteteacute des contrats ou la justesse de la facture

Valeur perccedilue coucirct marge et satisfaction reacutesultent de processus allant de la conception du

produit jusqursquoagrave sa livraison et son apregraves-vente La deacutemarche laquo processus raquo et lrsquolaquo analyse de la

valeur raquo en forccedilant la recherche de solutions efficientes agrave tout niveau administratif

technique commercial et apregraves-vente sont donc neacutecessaires pour bien positionner

lrsquoentreprise sur son marcheacute

Pour autant le risque commercial ne peut jamais ecirctre annuleacute et lrsquooffre de lrsquoentreprise ne

satisfait geacuteneacuteralement pas en milieu concurrentiel tous ses clients potentiels

33 - La creacuteation de valeur pour les autres parties prenantes

Les salarieacutes

La creacuteation drsquoune valeur suffisante pour les salarieacutes est reconnue comme neacutecessaire car des

observations montrent que la satisfaction des clients en deacutepend Moins souvent eacutevoqueacutee en

peacuteriode de chocircmage elle nrsquoest prioritaire que pour les employeacutes dont lrsquoentreprise souhaite

conserver les compeacutetences

La laquo valeur salarieacute raquo ne comprend pas que le salaire Le sentiment drsquoappartenance agrave un

groupe la reconnaissance lrsquoaccomplissement de soi et la construction professionnelle en

sont des eacuteleacutements importants Comme pour les clients on doit ainsi distinguer la reacutetribution

perccedilue du salaire objectif

Les actionnaires

Lrsquoactionnaire apporte des fonds propres agrave lrsquoentreprise en contrepartie de titres parfois

neacutegociables en bourse et assortis drsquoun droit de vote en assembleacutee geacuteneacuterale La valeur

attribueacutee aux actionnaires est servie en termes moneacutetaires (dividende ou augmentation de la

valeur du titre neacutegociable)

Remarque des facteurs non moneacutetaires comme lrsquoimage de lrsquoentreprise qui deacutepend

en partie de sa communication peuvent influencer la deacutecision drsquoachat de vente ou

de conservation des titres par lrsquoactionnaire

Reacutetribution perccedilue euros

Salaire objectif

Avantage non

moneacutetaire de

lrsquoemploi

Initiation au management copy CRCF ndash J Sornet Page 37 48

Compte tenu de lrsquoimportance croissante de lrsquoactionnariat dans le financement des grandes

entreprises coteacutees en bourse et notamment des investisseurs institutionnels comme les fonds

de pension des indicateurs speacutecifiques ont eacuteteacute introduits pour appreacutecier la performance des

entreprises vue par les actionnaires Par exemple la valeur ajouteacutee eacuteconomique (EVA reg

economic value added marque deacuteposeacutee de Stern Stewart ou VAE ndash valeur ajouteacutee

eacuteconomique parfois deacutenommeacutee VEC ndash valeur eacuteconomique creacuteeacutee) qui prend en compte le

coucirct du capital

LrsquoEVA correspond tregraves scheacutematiquement au calcul suivant

EVA = (PO) profit opeacuterationnel ndash (C) coucirct du capital X (CE) capitaux employeacutes

LrsquoEVA neacutecessite en pratique des retraitements assez complexes Le PO peut se deacuteterminer

selon les principes suivants

- PO = reacutesultat drsquoexploitation (avant inteacuterecircts) ndash impocirct

- PO = beacuteneacutefice courant (tenant compte des inteacuterecircts) + inteacuterecircts ndash eacuteconomie drsquoimpocirct sur les

inteacuterecircts (on exclue les eacuteleacutements financiers et lrsquoimpocirct correspondant) ndash impocirct

- lrsquoimpocirct pris en compte correspond au profit opeacuterationnel consideacutereacute (dans les cas courants agrave

13 du PO)

C = taux moyen de reacutemuneacuteration du capital (reacutesultant par exemple du dividende exigeacute de

certains investisseurs et des taux drsquoemprunts bancaires)

CE = capitaux propres et dettes portant inteacuterecirct

Remarque le profit opeacuterationnel ou reacutesultat opeacuterationnel correspond au NOPAT ndash net

operating profit after tax - anglo-saxon LrsquoEVA est eacutegale au NOPAT diminueacute de la

reacutemuneacuteration des capitaux

Exemple lrsquoentreprise X dispose drsquoun capital de 2 500 000 euro et reacutealise un beacuteneacutefice net

drsquoimpocirct de 450 000 euro (taux 33 13) Un dividende de 6 doit ecirctre verseacute aux

actionnaires et la banque lui a accordeacute un precirct de 1 200 000 euro agrave 4 Les autres

constituants des reacutesultats financier et exceptionnel sont neacutegligeables

Reacutesultat opeacuterationnel = 450 000 + 004 x 1 200 000 x 23 = 482 000 euro

Coucirct du capital = 006 x 2 500 000 + 004 x 1 200 000 x 23 = 182 000 euro

EVA = 300 000 euro

Coucirct moyen pondeacutereacute du capital (C) = (004 x 1 200 000 x23 + 006 x 2 500 000)

3 700 000 Soit 492

Si lrsquoEVA est positive lrsquoentreprise creacuteeacutee de la valeur apregraves reacutemuneacuteration des capitaux et sa

valeur boursiegravere doit augmenter

Lrsquoutilisation de lrsquoEVA comme indicateur influence le management de lrsquoentreprise car il y a

trois moyens pratiques drsquoaugmenter lrsquoEVA

- augmenter le reacutesultat opeacuterationnel

- lancer des investissements ayant une rentabiliteacute supeacuterieure agrave C

- eacuteliminer les activiteacutes ayant une rentabiliteacute infeacuterieure agrave C

Remarque lrsquoutilisation sans nuance de lrsquoEVA comme critegravere de management peut

poser problegraveme Le calcul de lrsquoEVA repose sur des ajustements comptables il est donc

sujet agrave manipulations (provisions capitalisation ou non de la RD hellip) Par ailleurs le

critegravere laquo EVA raquo pris isoleacutement peut conduire agrave chercher la rentabiliteacute agrave court terme agrave

reacuteduire les investissements prospectifs et donc nuire agrave terme au deacuteveloppement de

lrsquoentreprise

Initiation au management copy CRCF ndash J Sornet Page 38 48

Les fournisseurs reccediloivent le paiement de leurs factures plus ou moins rapidement (le deacutelai

de paiement repreacutesente une valeur consentie au fournisseur)

Lrsquoentreprise peut accroicirctre la valeur apporteacutee agrave ses fournisseurs par des actions cibleacutees

comme une contribution agrave la formation de leurs personnels certains transferts de

technologie ou de savoir faire agrave des sous-traitants une coopeacuteration suivie favorisant leur

deacuteveloppement lrsquointeacutegration agrave des campagnes de promotion

A noter que la valeur consentie aux fournisseurs peut avoir une influence sur la qualiteacute et les

deacutelais de livraison des produits

La collectiviteacute reccediloit des taxes et parfois des prestations en nature par deacutefaut ou explicites

(effort de preacuteservation de lrsquoenvironnement ameacutenagement du territoire par les implantations

aide mateacuterielle agrave des projets participation agrave la formation par exemple)

APPLICATIONS DT

DT1 Deacutefinir expliquer deacutereacuteglementation socieacutetal eacuteconomies drsquoeacutechelle coaching EDI

gouvernance

DT2 Deacuteterminer en quoi la deacutemarche TQM srsquoinscrit dans les deacutefis actuels du management

DT3 Apregraves avoir consulteacute les documents ci-dessous extraits du site drsquoAir France

(httpdeveloppement-

durableairfrancecomFRfrlocaldemarcheN4_positionnement_pphtm)

exposer les enjeux et les limites de la RSE et de la gestion des parties prenantes

Initiation au management copy CRCF ndash J Sornet Page 39 48

Dialogue avec les parties prenantes

Initiation au management copy CRCF ndash J Sornet Page 40 48

Attentes des parties prenantes

Initiation au management copy CRCF ndash J Sornet Page 41 48

Creacuteation de valeur pour les parties prenantes

La creacuteation de valeur pour les parties prenantes est au cœur de la strateacutegie du Groupe Le scheacutema de

distribution financiegravere ci-dessous donne un aperccedilu de la distribution des recettes du Groupe aux

diffeacuterentes parties prenantes actionnaires collaborateurs fournisseurs pouvoirs publics

collectiviteacutes locales etc

Initiation au management copy CRCF ndash J Sornet Page 42 48

Fiche DT1 ndash Extrait du sommaire de laquo Problegravemes eacuteconomiques raquo No 2894

La gestion des entreprises bouleverseacutee par les technologies de linternet

Reacutealiteacutes industrielles - Annales des Mines Jean-Michel Yolin

Avec lavegravenement de linternet les processus de conception de production et de vente sont

radicalement remis en cause Quel que soit le secteur dactiviteacute les technologies de linternet

permettent en effet de reacuteduire les deacutelais et de passer dun processus discontinu agrave un processus

continu Lorganisation des entreprises et leur mode de gestion en sont profondeacutement bouleverseacutes

tant au niveau individuel que collectif Linternet rend ainsi possible la reacutealisation dobjectifs que les

entreprises cherchaient agrave atteindre depuis longtemps sans y parvenir meilleure eacutecoute du client

travail sans stocks en flux tendu hieacuterarchies plates autorisant une grande reacuteactiviteacute flexibiliteacute dans

lorganisation et loutil de production acceacuteleacuteration du renouvellement des produits entreprises en

reacuteseau ougrave chacune se recentre sur son cœur de meacutetier etc

Le laquo knowledge management raquo ou comment geacuterer les connaissances

Document de travail du LAMSADE - Michel Grundstein

Peter Drucker lavait preacutedit le capital immateacuteriel eacutetait voueacute agrave devenir un facteur de compeacutetitiviteacute

pour lentreprise La libeacuteralisation des eacutechanges acceacutelegravere les processus de deacutecision de lentreprise

et implique que lassimilation des informations soit agrave la fois de meilleure qualiteacute et plus rapide Ainsi

la fonction qui consiste agrave manager les connaissances au sein de lentreprise savegravere primordiale

Bien que la prise de conscience de limportance du capital immateacuteriel ait eacuteteacute tardive - le concept

de knowledge management est apparu en France aux Etats-Unis et au Japon au milieu des

anneacutees 1990 - agrave lheure actuelle lorganisation de leacutechange dinformations et le partage des

connaissances sont devenus des facteurs cleacutes dune gestion performante de lentreprise Ils

doivent sinscrire dans un projet global destineacute agrave mettre en valeur les savoirs et les savoir-faire

individuels et collectifs

Les leccedilons du laquo coaching raquo pour le management de la qualiteacute

Humanisme et Entreprise - Martine Brasseur

Parmi les nouvelles formes de management en vogue dans les entreprises le coaching figure en

bonne place Appliqueacute au management de la qualiteacute il sagit dune pratique

daccompagnement destineacutee agrave initier et agrave faciliter le processus de deacuteveloppement dun individu

La deacutemarche consiste agrave affirmer que tout individu est en quecircte de qualiteacute agrave condition toutefois

de ne pas lui imposer des contraintes lempecircchant de progresser On considegravere notamment les

erreurs comme potentiellement feacutecondes En deacutefinitive le coach donne au coacheacute la permission

de reacuteussir en lui donnant aussi la permission deacutechouer

Initiation au management copy CRCF ndash J Sornet Page 43 48

Fiche DT2 ndash Management strateacutegique les sept deacutefis agrave relever dici agrave 2016

Extrait drsquoun article du site wwwlentreprisecom -Sabine Blanc - Mis en ligne le 20032007

(httpwwwlentreprisecom325article11977html)

Une eacutetude anglaise publieacutee par lopeacuterateur Orange Grande-Bretagne deacutecrypte la mutation

des formes de travail et les enjeux majeurs pour les entreprises de demain afin decirctre au top

de la compeacutetitiviteacute Voici les challenges-cleacutes pour les managers qui veulent rester dans la

course hellip

1 - Future organisation du travail les quatre laquo mondes raquo possibles

La reacutealiteacute sera probablement un meacutelange de ces quatre sceacutenarios souligne lrsquoeacutetude

Les mondes mutuels Tout se passe dans le cadre des communauteacutes locales vie priveacutee

comme professionnelle Le modegravele coopeacuteratif preacutevaut au lieu du laquo big business raquo Oublieacutes

aussi dans ce systegraveme les trajets pour aller au bureau les gens preacutefegravereront travailler dans de

petites entreprises locales souvent connecteacutees au reacuteseau drsquoautres structures similaires

Les laquo reacutepondants raquo (en anglais laquo replicants raquo) La figure du consultant freelance deviendra

dominante tandis que celle du salarieacute deacuteclinera Il ne sera pas rare de travailler pour plusieurs

entreprises On perdra en seacutecuriteacute de lrsquoemploi en visibiliteacute et en routine ce que lrsquoon gagnera

en liberteacute La majeure partie des tacircches srsquoeffectuera chez soi avec la possibiliteacute de srsquoinstaller

temporairement dans les bureaux de son client du moment Dans un contexte dincertitude

sur lrsquoavenir les travailleurs alterneront peacuteriodes drsquoactiviteacute intense et repos Ce sera agrave eux

drsquoaller vers les entreprises et non lrsquoinverse mecircme si celles-ci devront veiller agrave rester attractives

Les cottages eacutelectroniques Comme ce nom le suggegravere le teacuteleacutetravail deviendrait la norme

univers priveacute et professionnel se confondant Plus besoin de subir une heure de transport les

salarieacutes se logueront de chez eux sur le reacuteseau de lrsquoentreprise Les reacuteunions se tiendront dans

de petits bureaux centraux situeacutes agrave courte distance La flexibiliteacute du temps de travail srsquoimpose

Les salarieacutes disposeront de plus de marge de liberteacute dans leur activiteacute

Les disciples de la nueacutee Cette appellation poeacutetique cache simplement une extension de

lrsquoorganisation actuelle des grandes entreprises avec des salarieacutes se rendant sur un lieu de

travail centraliseacute Le rocircle croissant des technologies de lrsquoinformation multipliera les faccedilons de

collaborer et accroicirctra lrsquoefficaciteacute Le controcircle du travail sera omnipreacutesent La frontiegravere entre

travail et vie priveacutee restera marqueacutee

2 - Sept deacutefis pour les entreprises et leur managers

Quoi qursquoil advienne les entreprises et leurs dirigeants devront concentrer leurs efforts sur sept

points-cleacutes pour srsquoadapter Voici quelques exemples de probleacutematiques souleveacutees par le

rapport et des pistes de solution

Le leadership Les managers devront entre autres savoir persuader et influencer des

travailleurs beaucoup plus indeacutependants Ils auront aussi agrave repenser les niveaux auxquels

prendre les deacutecisions strateacutegiques en haut ou au contraire agrave des degreacutes moins eacuteleveacutes de la

pyramide hieacuterarchique

gt Faire du management une force facilitant les activiteacutes transversales plutocirct que la reacuteduire agrave

la seule fonction de deacutecision

La culture drsquoentreprise Davantage de salarieacutes capables de reacutefleacutechir seront neacutecessaires

tandis que les tacircches qui peuvent ecirctre automatiseacutees ou scripteacutees diminueront Un des

enjeux creacuteer une culture agrave mecircme drsquoattirer et drsquoencourager les personnes preacutesentant ces

qualiteacutes de reacuteflexion requises dans un contexte de compeacutetition accrue et de plus grande

indeacutependance des travailleurs

Initiation au management copy CRCF ndash J Sornet Page 44 48

gt Passer si neacutecessaire drsquoune culture drsquoentreprise forte agrave un mode drsquoengagement plus

consensuel moins rebutant

La marque Conseacutequence du recours croissant agrave lrsquo laquo outsourcing raquo lrsquoimage drsquoune marque

deacutependra plus drsquoagents exteacuterieurs qui ne fonctionnent pas forceacutement selon le mecircme mode

drsquoorganisation Comment garder le controcircle dessus

gt Choisir le mode qui corresponde le plus agrave vos valeurs et preacutevoir un programme de risk

management qui mette en eacutevidence ougrave les conflits sont susceptibles de jaillir

Lrsquoinnovation Plus que jamais il faudra faire face agrave une acceacuteleacuteration du rythme de

lrsquoinnovation en proposant constamment des solutions adapteacutees

gt Tisser des partenariats strateacutegiques avec drsquoautres entreprises pour partager les coucircts et les

fruits de lrsquoinnovation

Le deacutefi opeacuterationnel et technologique De quelle faccedilon controcircler lrsquoinformation crsquoest-agrave-dire

faire en sorte que les bonnes personnes accegravedent facilement agrave une information toujours en

phase tout en maintenant la seacutecuriteacute

gt Recourir agrave des laquo feuilles de route des futurs raquo syntheacutetisant en une page les indicateurs

sociaux et de consommation ainsi que les eacutevolutions technologiques et leacutegislatives qui

influent sur les changements et indiquant comment ils modifient vos marcheacutes vos clients et

votre organisation

La qualiteacute Si de nouveaux proceacutedeacutes ont pu deacutegrader la qualiteacute comme le recours agrave des

centres drsquoappel externaliseacutes drsquoautres ideacutees se sont reacuteveacuteleacutees plus prometteuses comme en

teacutemoigne le succegraves de certaines compagnies aeacuteriennes low cost Elles ont su conjuguer prix

serreacutes et services eacuteleveacutes ce qui devra devenir la norme estime lrsquoeacutetude

gt Continuer de rechercher la qualiteacute Elaborez aussi une bonne prestation service qui inclut

une livraison de qualiteacute voire creacuteez-la en partenariat avec les consommateurs

La leacutegislation La question de la proprieacuteteacute intellectuelle pourrait ecirctre probleacutematique Elle est

deacutejagrave source de conflits comme en teacutemoigne le procegraves pour violation de brevet intenteacute agrave RIM

le fabricant canadien du Blackberry par NTP Que pourra-t-on et que faudra-t-il proteacuteger par

un brevet Il sera eacutegalement neacutecessaire drsquoadapter la leacutegislation aux nouveaux modes

drsquoorganisation

gt Collaborer avec les acteurs du mecircme secteur et les leacutegislateurs pour deacutevelopper les

modegraveles des lieux de travail du futur et bacirctir le droit le plus adeacutequat

Orange a-t-il vu juste dans ses preacutevisions Rendez-vous dans neuf ans pour la reacuteponsehellip

Initiation au management copy CRCF ndash J Sornet Page 45 48

Fiche DT3 ndash Le management par la qualiteacute totale

Extrait drsquoune lettre drsquoinformation du cabinet Baud Accordance Consulting AD2 consultants ndash

2002

1 - Le TQM (Total Quality Management) offre pour lentreprise une vision de la qualiteacute plus

large et transversale

Son principe est simple La finaliteacute de lEntreprise est de deacutevelopper la satisfaction de ses

clients tout en eacutetant beacuteneacuteficiaire cest agrave dire pas agrave nimporte quel prix Elle doit ameacuteliorer sa

rentabiliteacute au travers de la deacutemarche qualiteacute La Qualiteacute Totale vise agrave fournir aux clients

externes et internes une reacuteponse adeacutequate agrave leurs attentes dans le meilleur rapport qualiteacute

prix la meilleure efficience

Elle considegravere pour cela lensemble des processus de lentreprise ayant une incidence sur la

qualiteacute et la satisfaction des clients

Le TQM fait ainsi une large place agrave

la deacutefinition et la planification de la strateacutegie geacuteneacuterale

la coheacuterence de la politique qualiteacute avec la strateacutegie

la deacutemultiplication de la politique qualiteacute dans toutes les directions de lentreprise

la relation client fournisseur interne

la prise en compte de lenvironnement concurrentiel

la consideacuteration de lensemble des risques potentiels financiers sociaux concurrentielshellip

limplication et la motivation du personnel

lanalyse des besoins des clients et le positionnement marketing

la maicirctrise des processus transverses internes

les reacutesultats sous tous ses aspects y compris financiers commerciaux image

De nombreux reacutefeacuterentiels sont relatifs agrave la Qualiteacute Totale hellip Tous ces reacutefeacuterentiels imposent un

questionnement plus profond et indiscret sur le mode de fonctionnement de lentreprise et

son management

helliphellip

2 - LISO 9001 2000 au travers du deacuteploiement des processus (management supports

reacutealisation et ameacutelioration continue) reacutepond quelque peu agrave la mecircme logique

LISO est une ouverture indeacuteniable vers la logique du TQM mais ne se reacutefegravere pas agrave la notion

defficience

Les dirigeants sont cependant sensibles agrave la neacutecessaire reacuteduction des coucircts de non-qualiteacute

et dobtention de la qualiteacute agrave la rentabiliteacute du systegraveme de management de la qualiteacute

mais ne perccediloivent pas toujours la qualiteacute comme une deacutemarche globale

Les deacutemarches qualiteacute commencent bien souvent par la remise en cause de lorganisation

leacutevaluation critique de son efficaciteacute lexamen des processus et la mise en eacutevidence des

lourdeurs administratives

La qualiteacute devient laffaire de tous hellip

Initiation au management copy CRCF ndash J Sornet Page 46 48

Fiche DT4 ndash Le deacuteveloppement durable et la RSE

Extrait du site wwwvigeocom

(httpwwwvigeocomcsr-rating-agencyfrmethodologiecriteres-de-recherche37-

criteres-d-analysehtml)

Deacuteveloppement durable laquo un deacuteveloppement qui reacutepond aux besoins du preacutesent sans compromettre

la capaciteacute des geacuteneacuterations futures de reacutepondre aux leurs raquo (Commission mondiale sur lrsquoenvironnement

et le deacuteveloppement ndash 1987)

Reacutefeacuterentiel drsquoeacutevaluation des entreprises par le groupe Vigeacuteo (le groupe mesure les performances et le

niveau de maicirctrise des risques de responsabiliteacute sociale des entreprises et des organisations - site

wwwvigeocom)

1 Ressources Humaines Ameacutelioration continue des relations professionnelles des relations drsquoemploi et des conditions de travail 2 Droits humains sur les lieux de travail Respect de la liberteacute syndicale et promotion de la neacutegociation collective non discrimination et promotion de lrsquoeacutegaliteacute eacutelimination des formes de travail proscrites (enfants travail forceacute) preacutevention des traitements inhumains ou deacutegradants de type harcegravelements sexuels protection de la vie priveacutee et des donneacutees personnelles 3 Environnement Protection sauvegarde preacutevention des atteintes agrave lenvironnement mise en place drsquoune strateacutegie manageacuteriale approprieacutee eacuteco conception protection de la biodiversiteacute et maicirctrise rationnelle des impacts environnementaux sur lrsquoensemble du cycle de vie des produits ou services

4 Comportements sur les marcheacutes Prise en compte des droits et inteacuterecircts des clients inteacutegration de standards sociaux et environnementaux dans la seacutelection des fournisseurs et sur lrsquoensemble de la chaicircne drsquoapprovisionnement preacutevention effective de la corruption respect des regravegles concurrentielles 5 Gouvernement drsquoentreprise Efficience et probiteacute assurance de lrsquoindeacutependance et de lrsquoefficaciteacute du Conseil drsquoadministration effectiviteacute et efficience des meacutecanismes drsquoaudit et de controcircle et notamment inclusion des risques de responsabiliteacute sociale respect des droits des actionnaires et notamment des minoritaires transparence et rationaliteacute de la reacutemuneacuteration des dirigeants 6 Engagement socieacutetal Effectiviteacute inteacutegration manageacuteriale de lrsquoengagement contribution au deacuteveloppement eacuteconomique et social des territoires drsquoimplantation et de leurs communauteacutes humaines engagements concrets en faveur de la maicirctrise des impacts socieacutetaux des produits et des services contribution transparente et participative agrave des causes drsquointeacuterecirct geacuteneacuteral

Initiation au management copy CRCF ndash J Sornet Page 47 48

ELEMENTS DE CORRIGE DT DT1 Deacutefinir expliquer

Deacutereacuteglementation = suppression des contraintes eacuteconomiques (libre eacutechange des biens et

capitaux)

Socieacutetal = qui se rapporte agrave la structure agrave lrsquoorganisation ou au fonctionnement de la socieacuteteacute

Economies drsquoeacutechelle = reacuteduction des coucircts lieacutee au niveau drsquoactiviteacute (amortissement des

charges fixes)

Coaching = accompagnement de personnes ou deacutequipes pour le deacuteveloppement de leurs

potentiels

EDI = eacutechange de donneacutees informatiseacutees ET standardiseacutees (ex SWIFT bancaire edifact

documents deacuteclaratifs)

Gouvernance = exercice du pouvoir la bonne gouvernance est participative et eacutequitable

conforme agrave lrsquointeacuterecirct commun

DT2 Deacuteterminer en quoi la deacutemarche TQM srsquoinscrit dans les deacutefis actuels du management

Voir notamment fiche 43

Maicirctrise des processus reacuteduction des coucircts reacuteactiviteacute et satisfaction de la clientegravele = faire

face agrave la concurrence

Ameacutelioration de lrsquoimage motivation du personnel

DT3 Apregraves avoir consulteacute les documents ci-dessous extraits du site drsquoAir France

(httpdeveloppement-

durableairfrancecomFRfrlocaldemarcheN4_positionnement_pphtm)

exposer les enjeux et les limites de la RSE et de la gestion des parties prenantes

Trame geacuteneacuterale possible

Introduction

Les deacutefis contemporains (accroissement de la concurrence devenue mondiale recherche

de nouveaux avantages concurrentiels pression de la socieacuteteacute besoin drsquoimage et de projet

lisible pour mener lrsquoentreprise crise et scandales du libeacuteralisme hellip) RSE et PP

Deacuteveloppement (voir cours)

1 ndash Parties prenantes et management par la valeur

PP deacutefinir citer reacutesumer lrsquoavantage rechercheacute (fideacuteliser motiver recherche drsquoalliances

implicites)

PP moyens (dont exemples AF) et meacutethode de management par la valeur (reacutepartie)

2 ndash La responsabiliteacute socieacutetale de lrsquoentreprise

RSE 3 axes

- eacuteconomique (favoriser le deacuteveloppement les eacutechanges internationaux)

- social (accegraves aux soins eacuteducation conditions de travail hellip)

- environnemental (pollution preacuteservation des ressources hellip)

RSE gouvernance drsquoentreprise facteur drsquoimage inteacutegrable dans la deacutemarche PP

Article 116 de la loi Le rapport viseacute agrave larticle L 225-102 rend compte hellip laquo Il comprend

eacutegalement des informations dont la liste est fixeacutee par deacutecret en Conseil dEtat sur la maniegravere

dont la socieacuteteacute prend en compte les conseacutequences sociales et environnementales de son

activiteacute Le preacutesent alineacutea ne sapplique pas aux socieacuteteacutes dont les titres ne sont pas admis aux

neacutegociations sur un marcheacute reacuteglementeacute raquo

Initiation au management copy CRCF ndash J Sornet Page 48 48

RSE exemple AF (ONG fournisseurs)

3 ndash Liens entre PP et RSE

- la RSE introduit de nouvelles PP

- la RSE suppose le respect des PP usuelles (employeacutes clients notamment)

4 - Probleacutematique

- deacutefinir la valeur reacuteellement apporteacutee par une gestion des PP (confusion salaire ndash valeur

idem impocircts hellip ex laquo valeur ajouteacutee raquo)

- communication (neacutecessaire mais aller au-delagrave)

- marginaliteacute des deacutepenses RSE (efficaciteacute sinceacuteriteacute de lrsquoengagement marge de manœuvre)

- charge RSE reporteacutee sur des tiers (ex fournisseurs AF)

- inteacutegration de facteurs non visibles en comptabiliteacute (pertes drsquoemploi nuisances hellip)

Conclusion

Voies incontournables mais pouvant nrsquoavoir qursquoun effet superficiel et temporaire Voir utiliteacute

drsquoaccompagnement leacutegislatif de regravegles de gouvernance

Initiation au management copy CRCF ndash J Sornet Page 11 48

Caracteacuteristiques

de lrsquoaction

- reacutepeacutetition

- risque

- normes

- ampleur

Prise de

deacutecision

- opeacuterationnelle

strateacutegique

- deacutelai

Informations

neacutecessaires

- nature

- origine

- deacutelai obtention

Cleacutes pour la

reacuteussite

Intervention

exteacuterieure

possible

Assurer la

restauration du

soir

(restaurant

familial)

Construire un

viaduc

(autoroute)

Certifier les

comptes

annuels drsquoun

groupe

national

(cabinet

drsquoaudit)

Lancer une

ligne drsquoavions

(constructeur

aeacuteronautique)

Reacuteduire la

capaciteacute de

production

(groupe

industriel)

Acqueacuterir une

entreprise

concurrente

(teacuteleacutephonie

mobile)

Initiation au management copy CRCF ndash J Sornet Page 12 48

Fiche IM1 - Deacutefinitions du management

Dictionnaire anglais - franccedilais direction administration gestion intrigue manegravege

Wikipeacutedia Le management est lensemble des techniques dorganisation qui sont mises en

oeuvre pour ladministration dune entiteacute

Au point de vue eacutetymologique le verbe manage vient de litalien maneggiare (controcircler)

influenceacute par le mot franccedilais manegravege (faire tourner un cheval dans un manegravege) A cette

notion il faut aussi ajouter la notion de meacutenage (geacuterer les affaires du meacutenage) qui consiste agrave

geacuterer des ressources humaines et des moyens financiers

helliphellip

Fiche IM2 - Etudier le management

Concreacutetiser

Manager neacutecessite de syntheacutetiser des informations parfois complexes incomplegravetes et de

domaines tregraves divers pour en deacuteduire des actions Une approche trop parcellaire peut

conduire agrave lrsquoeacutechec et le savoir-faire est neacutecessaire pour agir vite avec un minimum de risque

Lrsquoeacutetudiant doit se preacuteparer simultaneacutement aux examens et agrave la pratique Il nrsquoa souvent connu

lrsquoentreprise que durant quelques semaines de stage et le manque de laquo recul raquo ne lui permet

pas toujours de concreacutetiser les theacuteories Il doit compenser par la lecture (ouvrages revues

journaux eacuteconomiques et boursiers) et en eacutetant attentif aux informations ambiantes (tout en

relativisant le style journalistique) en mettant en relation le cours les concepts les modegraveles

lrsquoactualiteacute les stages

Savoir traiter un exercice

Pour reacuteussir un examen ou traiter une application peacutedagogique (la conception les points 1 agrave

6 peut repreacutesenter le tiers du temps de travail)

1 ndash Identifier le type de sujet (faut-il trouver une solution pratique ou communiquer une

reacuteflexion geacuteneacuterale )

2 ndash Lire le sujet et relever les mots cleacutes

3 ndash Deacutefinir les mots cleacutes

4 ndash Reacutesumer la probleacutematique du sujet (en quelques lignes)

5 ndash Lister les connaissances reacutefeacuterences et raisonnements reacutepondant au problegraveme (par

recherche spontaneacutee ou raisonneacutee qui quoi ougrave quand comment combien hellip

listage des diffeacuterents points de vue) trouver des exemples (notamment dans les

documents fournis)

6 ndash Organiser la reacuteponse (deacutefinir le plan du deacuteveloppement ougrave des paragraphes bien

identifieacutes sont geacuteneacuteralement neacutecessaires en y liant les parties qui doivent ecirctre en nombre

limiteacute ndash de deux agrave quatre) Preacutevoir drsquoy inteacutegrer la deacutefinition des principales notions

induites par le sujet

7 ndash Reacutediger sous la forme adapteacutee (note technique ou recommandation solution

pratique exposeacute structureacute dissertation)

Introduction et conclusion sont indispensables agrave la dissertation ou agrave lrsquoexposeacute

- lrsquointroduction preacutesente le sujet traiteacute (phrase drsquoaccroche initiale) amorce la

probleacutematique (quelques sous - questions) et annonce le plan

- la conclusion syntheacutetise le deacuteveloppement (arguments) eacutelargit le sujet (prise de recul)

et apporte le point final (une phrase)

Une limite agrave la communication

Il est difficile de faire passer plus de 4 ou 5 ideacutees fortes dans un exposeacute unique

Initiation au management copy CRCF ndash J Sornet Page 13 48

Fiche IM3 - Bref historique

Antiquiteacute

3000 AJC

Peacuteriode greacuteco-

romaine

Transition

feacuteodale

12egraveme siegravecle

europe

15egraveme ndash 17egraveme

siegravecles

19egraveme siegravecle

20egraveme siegravecle

agriculture preacutedominante industrie limiteacutee aux besoins drsquoun individu ou drsquoun clan

pour la confection des outils des vecirctements et de la poterie Force motrice animale

ou humaine pour lrsquoessentiel

Grands travaux drsquoeacutetat en Egypte premiegravere laquo planification ndash organisation ndash controcircle raquo

Deacuteveloppement des communications essor industriel limiteacute peu de progregraves

technique (lrsquoesclavage supplante les innovations)

Deacuteveloppement progressif des eacutechanges commerciaux

La consommation indirecte atteint un bon niveau (surplus agricoles et

deacuteveloppement des villes) Apparition de nouveaux commerccedilants

Etat fort Evolutions technologiques (imprimerie bateaux performants instruments de

navigation) Extension geacuteographique de lrsquoeacuteconomie Apparition des corporations

drsquoartisans

Machine agrave vapeur chemin de fer passage de lrsquoartisanat au capitalisme

entrepreneurial producteur organisation des entreprises

Ecole classique (Taylor Fayol Weber) approche meacutecaniste bureaucratie

hieacuterarchie commandement fonctions et speacutecialisation laquo OS T raquo (organisation

scientifique du travail) organisation source de pouvoir rationaliteacute des individus bases

du management

Deacuteveloppement du capitalisme manageacuterial Electriciteacute peacutetrole puis communications

et information Consommation de masse mondialisation preacuteoccupations

eacutenergeacutetiques et environnementales 3 peacuteriodes

- standardisation grandes entreprises industrielles

- industries de consommation 30 glorieuses marketing multinationales protection

sociale

- deacutereacuteglementation monteacutee des services pays eacutemergents mondialisation et nouvelle

eacuteconomie (internet)

Ecole des relations humaines prise en compte de lrsquoindividu des motivations styles

de direction

Ecole neacuteo-classique et post-classique deacutecentralisation coordonneacutee DPO

management participatif zeacutero deacutefaut flux tendus

Approche systeacutemique partition de lrsquoentreprise eacutetude des interactions feacutedeacuteration

vers lrsquoobjectif controcircle et ajustement

Theacuteories de la deacutecision rationaliteacute limiteacutee contribution reacutetribution coalitions

Ecole socio-technique recherche de compromis technologie organisation

enrichissement des tacircches autonomie des groupes

Approche sociologique effets sociaux du travail jeux de pouvoir dans lrsquoentreprise

reacutegulation sociale

Theacuteories de la contingence facteurs contingents adaptation agrave lrsquoenvironnement

configurations organisationnelles

Theacuteories de la firme controcircle manageacuterial droits de proprieacuteteacute relation drsquoagence

Theacuteories contractualistes firme nœud de contrats coucircts de transaction

opportunisme externalisation internalisation

Approche eacutevolutioniste eacutecologie des organisations modegravele eacutevolutioniste

contraintes de sentier

Approche par les ressources valorisation des ressources compeacutetences cleacutes

apprentissage organisationnel

(Classement simplifieacute)

Initiation au management copy CRCF ndash J Sornet Page 14 48

ELEMENTS DE CORRIGE IM

IM1 Commenter la deacutefinition du management par la norme ISO et le manager de Mintzberg

Efficient = optimum avec les moyens disponibles

ISO (management objectifs) (manager moyens) HM

IM2 Le leader entraicircne naturellement derriegravere lui Le manager nrsquoest pas toujours leader

(mecircme si crsquoest souhaitable) Le leader nrsquoest pas toujours manager (plutocirct notion individuelle)

Leadership = faculteacute de diriger conjugaison drsquoune autoriteacute naturelle ou drsquoun savoir-faire

acquis drsquoune capaciteacute agrave entraicircner des personnes ou des groupes et drsquoune leacutegitimiteacute

statutaire (de position)

IM3 Compleacuteter le tableau ci-dessous en analysant chaque action preacutesenteacutee Faire ensuite

ressortir les domaines niveaux ou techniques de management pouvant ecirctre mobiliseacutes pour

chaque situation

Satisfaction client

Implication du personnel

Processus systegraveme

Ameacutelioration continue

Deacutecision efficace

Recherche de valeur

Image entreprise

Liaisons

Information

Reacutepartition ressources

Reacutegulation

Neacutegociation

Leadership

Initiation au management copy CRCF ndash J Sornet Page 15 48

Caracteacuteristiques

de lrsquoaction

- reacutepeacutetition

- risque

- normes

- ampleur

Prise de

deacutecision

- opeacuteration

- direction

- deacutelai

Informations

neacutecessaires

- nature

- origine

- deacutelai

obtention

Cleacute pour la

reacuteussite

Intervention

exteacuterieure

possible

Assurer la

restauration du

soir

(Restaurant

familial)

Technique

(fabrication)

Vente (terrain)

Appros

Reacutepeacutetitive

(quot)

Risque faible

Normes

drsquohygiegravene

Faible

Opeacuterationnelle

Geacuterant

responsable

Rapide (qq

jours menu et

appros)

Nombre de

couverts

Tarifs usuels

Calendrier

(fecirctes)

Clients docs

divers

expeacuterience

Qq jours

Varieacuteteacute menu

Plats phares

Accueil

Appros

Tarification

Vins

Gestion

congeacutelation

Qualiteacute cuisine

Fournisseurs

Extra

Publiciteacute

Construire un

viaduc

(autoroute)

Technique

Organisation

Appros

Uniteacute (ou peu)

Eleveacute (financier

technique)

Architecture

Eleveacutee

Direction

(aleacuteas)

Opeacuterationnelle

(conduite

chantier)

Immeacutediat agrave qq

semaines

Plans

plannings

Qualifications

Meacuteteacuteo

Disponibiliteacutes

Bureau eacutetudes

Qq sem agrave 24h

Techniciteacute

Appros

Qualifications

Preacutevision

GRH

Contrat juste

SS traitants

Organismes

certificateurs

Controcircle

client

Certifier les

comptes

annuels drsquoun

groupe national

(cabinet

drsquoaudit)

Technique

Relation client

Gestion des

connaissances

Annuelle

Moyen

Regravegles

comptables

fiscales

Moyenne (selon

importance du

cabinet)

Opeacuterationnelle

Qq jours agrave

semaines

Comptable

Juridique

Client

Etat

Qq jours agrave

semaines

Techniciteacute

Expeacuterience

Relation client

Systegraveme info client

Siegravege

Autre cabinet

Lancer une

ligne drsquoavions

(constructeur

aeacuteronautique)

Strateacutegique

RD

Etudes

Uniteacute

Tregraves eacuteleveacute

Aeacuteronautique

Tregraves eacuteleveacutee

Direction

Qq mois agrave

anneacutees

Marcheacute

Etudes

Compagnies

Qq mois agrave

anneacutees

Concept

Outil industriel

Coucirct exploitation

Tarif

Fiabiliteacute

Deacutelaisconcurrence

SI simulation

SS traitants

Bureaux

drsquoeacutetudes

speacutecialiseacutes

Compagnies

Conseils

Reacuteduire la

capaciteacute de

production

(groupe

industriel)

Strateacutegique

RH

Communication

Production

Uniteacute

Moyen

Leacutegislation

(dont RH)

Eleveacutee

Direction

Qq mois agrave

anneacutees

Financiegravere

Industrielle

Marcheacute

Organisation

Organismes

speacutecialiseacutes

DRH

Qq mois

Communication

Connaissance des

compeacutetences

Connaissance outil

industriel

Concurrence

Portefeuille

drsquoactiviteacutes

Cabinet

drsquoorganisation

Conseils

speacutecifiques

Acqueacuterir une

entreprise

concurrente

(teacuteleacutephonie

mobile)

Strateacutegique

Marketing

Production

(reacuteseau)

Financier

Communication

Uniteacute

Tregraves eacuteleveacute

Leacutegislation

telecom

Tregraves eacuteleveacutee

Direction

Qq mois

Financiegravere

Marcheacute

Reacuteseaux

(ampleur

recouvrement

hellip)

Organisations

Interne

Racheteacutee

Sources

speacutecialiseacutees

Qq mois

Communication

Marcheacute

Cours boursiers

Cabinet

drsquoorganisation

Conseils

speacutecifiques

Initiation au management copy CRCF ndash J Sornet Page 16 48

LE MANAGEMENT EN PRATIQUE

Pour assumer sa fonction le management doit couvrir sans discontinuiteacute lrsquoensemble de

lrsquoorganisation et inteacutegrer de nombreux facteurs dont nous allons reacutesumer lrsquoessentiel

1 ndash Les fonctions et activiteacutes du management

Pour Henri Fayol la fonction drsquoadministration de lrsquoentreprise (son management) reposait sur

cinq actions preacutevoir organiser commander coordonner et controcircler (laquo PO3C raquo)

Nous distinguerons cinq activiteacutes de management

- la conception (au plus haut niveau finaliteacute but ou vocation de lrsquoorganisation

meacutetiers dimension politique de croissance hellip)

- la planification (deacutefinition des objectifs eacutecheacuteances)

- lrsquoorganisation (reacutepartition du travail choix des modes de coordination)

- le pilotage de lrsquoaction opeacuterationnelle (motivation animation encadrement

assistance)

- lrsquoeacutevaluation (controcircle des reacutesultats obtenus ajustements)

Dans chacune de ces activiteacutes des deacutecisions et des arbitrages sont neacutecessaires avec des

enjeux plus ou moins importants

Remarques

- Les cinq activiteacutes du management peuvent se retrouver agrave tout niveau de

management si lrsquoentreprise laisse une certaine autonomie de deacutecision agrave ses diffeacuterentes

uniteacutes La conception est naturellement du ressort de la direction geacuteneacuterale et des

conseils drsquoadministration mais elle peut ecirctre preacutesente pregraves du terrain (latitude laisseacutee agrave

une filiale ou agrave un magasin par exemple) De mecircme lrsquoorganisation du travail concerne

un atelier mais aussi la direction qui structure lrsquoentreprise pour assurer ses activiteacutes sa

production

- La planification deacutefinit des objectifs ou des axes strateacutegiques (choix de produits

modaliteacutes de deacuteveloppement des ventes implantations alliances hellip) et les traduit en

donneacutees de gestion preacutevisionnelles syntheacutetiques et eacutechelonneacutees dans le temps afin de

valider les objectifs et de fixer des repegraveres

- Un laquo business plan raquo (plan drsquoaffaires)est notamment lrsquoeacutequivalent de la planification

dans le cas de creacuteation drsquoentreprise ou pour la preacutesentation de tout projet drsquoactiviteacute

Les activiteacutes du management srsquoinscrivent dans des cycles qui peuvent ecirctre scheacutematiseacute

comme suit (lrsquoeacutevaluation peut entraicircner une reacutevision du pilotage de lrsquoorganisation ou des

objectifs sans que lrsquoentreprise ne soit fondamentalement remise en cause)

conception

planification

organisation

pilotage

eacutevaluation

Initiation au management copy CRCF ndash J Sornet Page 17 48

2 ndash Les contextes de management

Le management est influenceacute par son contexte qui justifie des objectifs une organisation

des meacutethodes

Par exemple lrsquoentreprise admet de nombreuses variantes selon sa taille sa forme juridique

son controcircle par lrsquoeacutetat (entreprises publiques) ou par des inteacuterecircts priveacutes Il en va de mecircme des

organismes administratifs qui peuvent deacutependre de directives nationales ou reacutegionales des

associations qui ont des activiteacutes drsquoampleur tregraves variable

21 ndash La dimension de lrsquoentreprise

La dimension drsquoune entreprise se mesure principalement en fonction de son effectif ou de

son chiffre drsquoaffaires Des seuils sont deacutefinis par divers organismes et exploiteacutes agrave des fins

statistiques ou pour la deacutetermination de certaines obligations sociales ou fiscales

(repreacutesentation du personnel cotisations hellip) Il nrsquoy a bien entendu pas de laquo barriegravere de

tailleraquo absolue conditionnant le management drsquoune entreprise

LrsquoUE preacuteconise de distinguer les micro ndash entreprises (jusqursquoagrave 9 salarieacutes) les TPE ndash tregraves petites

entreprises (moins de 20 salarieacutes) les petites entreprises (moins de 50) et les moyennes

entreprises (de 50 agrave 250) Cependant les PME sont parfois situeacutees entre 10 et 500 salarieacutes

Remarques

- en France environ 40 des entreprises emploient de 1 agrave 50 salarieacutes (ce qui repreacutesente

plus de 50 des emplois) et 59 nrsquoen ont aucun

le pays compte environ 2 600 000 entreprises dont moins de 1 ont 250 employeacutes et

plus

- ancienneteacute et taille de lrsquoentreprise sont lieacutees si lrsquoon eacutecarte les restructurations et autres

eacutevolutions drsquoentreprises existantes

La dimension de lrsquoentreprise a une influence sur lrsquoorganisation et le laquo style raquo de son

management

- les PME sont souvent entrepreneuriales (les dirigeants eacutegalement apporteurs de capitaux

sont totalement engageacutes dans la marche de lrsquoentreprise) Elles ont une gestion flexible peu

formaliseacutee plus qualitative que quantitative Les PME sont freacutequemment focaliseacutees sur un seul

type drsquoactiviteacute Pour ne pas alourdir leur structure elles ont tendance agrave sous-traiter les

activiteacutes speacutecialiseacutees ne correspondant pas agrave leur meacutetier de base

- les grandes entreprises sont manageacuteriales (les dirigeants sont nommeacutes par les actionnaires

en raison de leurs compeacutetences) et moins reacuteactives

22 ndash Le type de production

On distingue industrie (production de biens mateacuteriels ou pour le moins de produits visibles ndash

comme un seacutejour touristique ou un film) et services (fourniture drsquoune prestation immateacuterielle)

Le type de production influence en principe le management de lrsquoentreprise

- lrsquoindustrie neacutecessite (si lrsquoon excepte lrsquoartisanat) un investissement relativement important

une organisation productive stable capable de reacutealiser plusieurs fois des produits identiques

(exemple un modegravele de reacutefrigeacuterateur) ou du moins similaires (exemple un bacirctiment) Le

produit de lrsquoindustrie consomme des matiegraveres et il doit geacuteneacuteralement ecirctre distribueacute jusqursquoau

client

- la production de services peut se satisfaire drsquoun investissement tregraves reacuteduit et neacutecessite un

contact permanent avec le client

Toutefois la standardisation des services et le deacuteveloppement des reacuteseaux informatiques

rapprochent la production de services de celle des biens industriels

- la production drsquoun service reacutepeacutetitif et technique peut imposer une structure lourde et une

organisation tregraves formaliseacutee (voir les grandes socieacuteteacutes drsquoaudit ou de conseil informatique)

Initiation au management copy CRCF ndash J Sornet Page 18 48

- certains services peuvent ecirctre fournis agrave distance sans contact direct avec le client et

distribueacutes par reacuteseau (tenue de comptabiliteacute affacturage gestion clientegravele centre drsquoappel

hellip)

Remarque les services repreacutesentent 75 de lrsquoactiviteacute eacuteconomique franccedilaise

23 ndash La nature de lrsquoorganisation

Les organisations publiques franccedilaises (administrations centrales collectiviteacutes territoriales

hocircpitaux hellip) repreacutesentent une part importante de lrsquoactiviteacute (environ 30 des emplois) La

fonction publique regroupe des organisations aux finaliteacutes diverses et qui ont des problegravemes

de gestion similaires agrave ceux des entreprises auxquelles elles peuvent emprunter des principes

de management Notamment

- pour controcircler les coucircts et assurer la qualiteacute des services

- pour communiquer avec les administreacutes ou les usagers

- pour motiver les personnels et geacuterer les ressources humaines

La transposition directe des techniques de gestion et de management nrsquoest cependant pas

toujours possible car

- la comptabiliteacute publique obeacuteit agrave des regravegles speacutecifiques (proceacutedure budgeacutetaire

notamment)

- le laquo client raquo ne paye pas toujours la prestation du moins directement

- la concurrence est parfois inexistante

- les grandes administrations centraliseacutees sont soumises agrave des choix politiques geacuteneacuteraux

parfois sans connexion eacutevidente avec les besoins opeacuterationnels

- le statut des personnels et les grilles de salaires limitent les possibiliteacutes de gestion des

ressources humaines

Remarque la LOLF (loi organique relative aux lois de finances) est entreacutee en vigueur en

2006 Elle alloue des moyens budgeacutetaires en fonction de programmes et remplace la

reconduction automatique de 90 des budgets Cette reacuteforme se heurte toutefois agrave la

lourdeur des grands ministegraveres ougrave la complexiteacute des activiteacutes est difficile agrave

appreacutehender et ougrave des inerties culturelles peuvent exister agrave tout niveau

Les associations loi de 1901 peuvent avoir une activiteacute comparable agrave celle de grandes

entreprises (voir par exemple les associations de santeacute ou professionnelles) et leur

management est alors similaire malgreacute lrsquoabsence de but lucratif (les beacuteneacutefices ne sont pas

distribuables) Elles ont drsquoailleurs en France un poids eacuteconomique important (elles emploient

environ 1 600 000 salarieacutes)

Cependant lrsquoadheacutesion agrave un systegraveme de valeurs fondateur de lrsquoassociation ou la limite de

lrsquoautoriteacute (quand un volant de beacuteneacutevoles important participe agrave lrsquoactiviteacute) peut introduire des

nuances

- le renforcement des objectifs socieacutetaux

- la faiblesse des relations hieacuterarchiques

- des contraintes de gestion du temps des beacuteneacutevoles

- des modaliteacutes particuliegraveres de recrutement et de motivation des dirigeants

24 ndash Les facteurs contingents

La theacuteorie de la contingence montre qursquoune structure drsquoentreprise nrsquoest efficace que dans

une situation deacutetermineacutee et qursquoil nrsquoexiste que des solutions de management construites dans

un contexte preacutecis

Le management doit ainsi srsquoadapter agrave des facteurs contingents qui ne peuvent ecirctre

controcircleacutes du moins agrave bregraveve eacutecheacuteance Ces facteurs sont par exemple

- lrsquoancienneteacute de lrsquoentreprise (plus elle est ancienne plus lrsquoentreprise a tendance agrave reacutepeacuteter

des comportements eacuteprouveacutes)

Initiation au management copy CRCF ndash J Sornet Page 19 48

- la taille de lrsquoentreprise (la grande entreprise a une composante administrative plus

deacuteveloppeacutee)

- le systegraveme de production (tregraves standardiseacute complexe automatiseacute hellip)

- lrsquoenvironnement

3 ndash Le management et les parties prenantes

Lrsquoentreprise a pour vocation premiegravere de mettre des produits agrave disposition de ses clients en

reacutealisant un profit Pour y arriver elle doit aussi satisfaire ses parties prenantes salarieacutes

actionnaires fournisseurs hellip

Est partie prenante agrave lrsquoentreprise laquo tout groupe ou individu qui peut ecirctre affecteacute ou est

affecteacute par les buts de lrsquoorganisation hellip raquo (Freeman ndash 1984)

Les parties prenantes attendent agrave des degreacutes divers de profiter drsquoune creacuteation de valeur en

provenance de lrsquoentreprise qui doit reacutepondre agrave ces attentes pour assurer sa peacuterenniteacute ou

favoriser son deacuteveloppement

On distingue les parties prenantes primaires ou principales qui sont essentielles agrave lrsquoentreprise

et qui ont geacuteneacuteralement une relation formelle avec elle (clients associeacutes et actionnaires

precircteurs salarieacutes fournisseurs collectiviteacutes) et les parties prenantes secondaires dont

lrsquoinfluence est diffuse (groupes de pression associations meacutedias instances europeacuteennes

agences de notation hellip)

Remarque la consideacuteration de lrsquoensemble des parties prenantes (laquo stakeholders raquo - les

deacutepositaires) fait contrepoids agrave lrsquoimportance accordeacutee aux seuls actionnaires

(laquo shareholders raquo)

Les organisations nrsquoayant pas drsquoobjectif de profit doivent aussi satisfaire leurs parties

prenantes apporter un service aux usagers dans les meilleures conditions eacuteconomiques

limiter un budget assurer la qualiteacute des relations avec les fournisseurs hellip

Dans cette optique le management doit organiser lrsquoaction de faccedilon agrave eacutequilibrer des forces

parfois divergentes

- le contexte fait pression sur lrsquoorganisation contrainte agrave optimiser ses reacutesultats

- lrsquoorganisation cherche par son action agrave assurer sa peacuterenniteacute son deacuteveloppement (en

reacutealisant des profits dans le cas de lrsquoentreprise) et agrave satisfaire ses parties prenantes

- le management agit en pilotant les actions pour contrebalancer la pression du contexte

Actions de

lrsquoorganisation

Management Contexte

Parties

prenantes

Initiation au management copy CRCF ndash J Sornet Page 20 48

APPLICATIONS MP

MP1 Deacutefinir contingent gestion budgeacutetaire

MP2 Deacuteterminer les parties prenantes drsquoun hocircpital public et leurs principales attentes

Mecircme question pour les organisations suivantes

- SNCF (entreprise publique)

- Peugeot

- MAIF (mutuelle drsquoassurance)

MP3 En les situant dans le cycle des activiteacutes du management trouver les actions agrave mener

dans les situations suivantes

- baisse de 10 des ventes dans une entreprise industrielle (produits meacutenagers le reacuteseau de

distribution vient drsquoecirctre reacuteorganiseacute)

- idem dans une entreprise de vente par correspondance soumise agrave la concurrence internet

(les ventes stagnaient depuis six mois malgreacute les efforts promotionnels)

- augmentation des deacutelais drsquoattente des consultations dans une clinique (lrsquohocircpital voisin a

fermeacute son service drsquourgences)

Initiation au management copy CRCF ndash J Sornet Page 21 48

ELEMENTS DE CORRIGE MP

MP1 Deacutefinir (dans le contexte drsquoune entreprise) contingent gestion budgeacutetaire

Contingent = imposeacute par lrsquoexteacuterieur Contingence = effet du hasard de la rencontre de

plusieurs eacuteveacutenements indeacutependants (variables explicatives que lrsquoon ne peut influencer)

Gestion budgeacutetaire = technique drsquoadministration des entreprises srsquoappuyant sur des

preacutevisions dont on deacuteduit apregraves accord des responsables des attributions de moyens sur une

dureacutee limiteacutee Une analyse reacuteguliegravere des eacutecarts entre preacutevisions et reacutealisations permet ensuite

le pilotage des activiteacutes Le budget est un cadre incitatif

La laquo planification budgeacutetaire raquo consiste agrave traduire en budgets une planification strateacutegique

avec systegraveme de reporting

MP2 Deacuteterminer les parties prenantes drsquoun hocircpital public et leurs principales attentes

Mecircme question pour les organisations suivantes

- SNCF (entreprise publique)

- Peugeot

- MAIF (mutuelle drsquoassurance)

Hocircpital

- patients (qualiteacute des soins)

- CNAM (baisse des coucircts)

- collectiviteacute locale (service aux administreacutes)

- eacutetat (ameacutenagement du territoire maicirctrise des budgets optimisation)

- employeacutes (salaire conditions de travail et satisfaction)

- fournisseurs ndash pharmacie autres (CA paiement reacutegulier)

- associations de patients (qualiteacute proximiteacute des soins)

SNCF

- usagers et associations drsquousagers (proximiteacute reacutegulariteacute prix du service)

- reacuteseau ferreacute de France (optimisation des lignes paiement adapteacute)

- fournisseurs (CA paiement reacutegulier)

- employeacutes (salaire conditions de travail seacutecuriteacute de lrsquoemploi)

- eacutetat (ameacutenagement du territoire)

- collectiviteacutes locales (service)

Peugeot

- clients (qualiteacute prix SAV relation commerciale)

- fournisseurs (CA reacutegulariteacute de lrsquoactiviteacute)

- employeacutes (salaire conditions de travail seacutecuriteacute de lrsquoemploi)

- eacutetat (taxes)

- collectiviteacute locale (emploi dynamisation eacuteconomique preacuteservation de lrsquoenvironnement)

- associations de protection de lrsquoenvironnement (activiteacute propre baisse des eacutemissions

nouvelles eacutenergies)

MAIF

- socieacutetaires (protection relation assureur tarif mesureacute)

- professionnels de lrsquoautomobile et autres (agreacutement marge de manœuvre reacuteparations tarifs

eacuteleveacutes)

- fournisseurs (CA paiement reacutegulier)

- eacutetat (taxes engagement pour la seacutecuriteacute)

- employeacutes (salaire conditions de travail seacutecuriteacute de lrsquoemploi)

Initiation au management copy CRCF ndash J Sornet Page 22 48

MP3 En les situant dans le cycle des activiteacutes du management trouver les actions agrave mener

dans les situations suivantes

- baisse de 10 des ventes dans une entreprise industrielle (produits meacutenagers le reacuteseau de

distribution vient drsquoecirctre reacuteorganiseacute)

Adapter le pilotage motiver cadrer si insuffisant retoucher une organisation deacutefectueuse

- idem dans une entreprise de vente par correspondance soumise agrave la concurrence internet

(les ventes stagnaient depuis six mois malgreacute les efforts promotionnels)

Voir pilotage et organisation si une eacutevolution du meacutetier a deacutejagrave eacuteteacute initialiseacutee Sinon re-

conception (adaptation au nouveau contexte) puis planification et reacuteorganisation

- augmentation des deacutelais drsquoattente des consultations dans une clinique (lrsquohocircpital voisin a

fermeacute son service drsquourgences)

Organisation Si insuffisant planification (nouveaux objectifs)

Initiation au management copy CRCF ndash J Sornet Page 23 48

ORGANISATION ET PROCESSUS

La performance de lrsquoentreprise deacutepend de son organisation et de son aptitude agrave produire

aux meilleures conditions Nous allons montrer comment organisation formelle et processus

de production peuvent contribuer agrave cette performance

1 ndash Vers lrsquooptimum

11 ndash Les eacuteconomies occidentales jusqursquoaux anneacutees 70

Jusqursquoen 1945 le principal problegraveme des entreprises eacutetait de produire des biens en quantiteacute

suffisante agrave un prix compatible avec le marcheacute Les grandes entreprises se sont multiplieacutees et

la standardisation a permis de reacuteduire les coucircts (exemple deacuteveloppement de Ford et de la

production agrave la chaicircne de 1908 agrave 1920 qui a permis une baisse du prix des voitures des 23)

On parle de laquo production pousseacutee vers le marcheacute raquo

Cette croissance de la production peu reacuteguleacutee a eacuteteacute marqueacutee par des surproductions en

1910 et 1920 puis par la crise de 1929 qui a prolongeacute ses effets jusqursquoagrave la guerre

De 1945 agrave 1975 environ (les laquo trente glorieuses raquo) la reconstruction la croissance de la

consommation de masse de nouvelles technologies et les eacutechanges internationaux

alimentent lrsquoeacuteconomie La standardisation srsquoeacutetend aux biens de consommation dont les

coucircts baissent fortement et de nouvelles reacutegulations sociales permettent une eacutevolution sans

heurt des revenus La saturation de certains marcheacutes conduit dans les anneacutees 60 agrave la

deacutemarche laquo marketing raquo et agrave la diffeacuterenciation des produits Le produit est laquo dirigeacute par le

marcheacute raquo mais les entreprises conservent une organisation assez classique et les plus grosses

srsquointernationalisent

12 ndash Lrsquoexpeacuterience japonaise et ses prolongements

Tregraves tocirct apregraves la guerre dans un Japon appauvri le constructeur automobile Toyota a ducirc

faire face agrave une restriction du marcheacute des moyens financiers et productifs et des

approvisionnements La firme a donc innoveacute dans un nouveau systegraveme de production

chassant les laquo gaspillages raquo (temps drsquoattente transports stocks deacutefauts hellip) consideacuterant que

seule la fabrication vendable creacutee de la valeur

Toyota srsquoorganise pour fabriquer la quantiteacute et la qualiteacute de produits juste neacutecessaires agrave la

satisfaction des clients la production est laquo tireacutee par le marcheacute raquo La mise en place de ce

systegraveme qui integravegre les fournisseurs ne sera acheveacutee que dans le milieu des anneacutees 70

En 1973 la hausse du peacutetrole inaugure un ralentissement de la croissance des eacuteconomies

occidentales La concurrence accrue provoque alors un inteacuterecirct pour le systegraveme deacuteveloppeacute

au Japon La production au plus juste se deacuteveloppe ainsi dans lrsquoindustrie automobile agrave partir

des anneacutees 80 et elle se reacutepand encore maintenant dans drsquoautres secteurs

Cette approche qui vise un objectif de zeacutero stock et zeacutero deacutefaut impose la maicirctrise de laquo bout

en bout raquo des processus de production et leur ameacutelioration

Initiation au management copy CRCF ndash J Sornet Page 24 48

2 ndash Organiser lrsquoentreprise

21 ndash Direction et organisation

Diriger une entreprise neacutecessite de lrsquoorganiser (de reacutepartir les tacircches) pour qursquoelle puisse

atteindre ses objectifs Lrsquoorganisation permet de satisfaire un marcheacute en tirant parti des

capaciteacutes actuelles de lrsquoentreprise tout en preacuteparant lrsquoavenir

Lrsquoorganisation reacutesulte freacutequemment drsquoun compromis entre des objectifs situeacutes agrave des niveaux

et des eacutecheacuteances diffeacuterents

Exemples

- le leader des chaises roulantes peut tirer profit de sa structure productive et de son

savoir faire pour entrer sur le marcheacute de la bicyclette eacutelectrique

- ecirctre parfaitement structureacute pour alimenter 90 du marcheacute des disquettes ne preacutepare

pas lrsquoavenir

- srsquoorganiser pour conqueacuterir le marcheacute des tire-bouchons eacutelectriques dans les deux ans

perd de son sens si cela altegravere les moyens neacutecessaires agrave la production drsquoappareils

manuels ancienne mais vitale dont la diminution agrave court terme risque de nuire agrave la

solvabiliteacute de lrsquoentreprise et de la conduire agrave la cessation de paiement

22 ndash Lrsquoorganisation fonctionnelle

La majoriteacute des entreprises adopte une laquo organisation fonctionnelle raquo (celle qui est visible

dans les organigrammes) ougrave des regroupements de personnels et drsquoeacutequipements se font

selon un modegravele hieacuterarchique (laquo line raquo) dans des uniteacutes des services ou des deacutepartements

speacutecialiseacutes Cette organisation peut se deacutecliner agrave lrsquointeacuterieur des divisions des grandes

entreprises quand elles scindent leur activiteacute par zone geacuteographique type drsquoactiviteacute

cateacutegorie de clients hellip

Remarque le terme laquo fonction raquo deacutesigne un rocircle particulier dans le fonctionnement de

lrsquoentreprise

Lrsquoorganisation fonctionnelle diffeacuterencie les activiteacutes de lrsquoentreprise en les regroupant par

meacutetier pour utiliser au mieux les compeacutetences et les moyens (meilleur rendement par la

speacutecialisation lrsquoeacutechange de compeacutetences dans une mecircme uniteacute ou gracircce agrave des eacuteconomies

drsquoeacutechelle)

23 ndash La notion de processus de production

Un processus de production se deacutefinit par la succession drsquoactiviteacutes permettant de satisfaire

un client en transformant des ressources (mateacuterielles financiegraveres humaines) en un produit

bien ou service Le processus doit creacuteer une valeur reconnue par le client

Un processus peut servir un client interne agrave lrsquoentreprise (par exemple en produisant un

composant intervenant dans plusieurs produits ou par la maintenance des machines) aussi

bien qursquoun client final On distingue usuellement

- les processus opeacuterationnels (ou maicirctres) aussi appeleacutes processus meacutetier (business process)

qui satisfont directement les clients finaux (conception et fabrication de produits vente hellip)

- les processus de support et de management (geacuterer les ressources humaines geacuterer

lrsquoinformation geacuterer les ressources financiegraveres hellip) qui ont les processus opeacuterationnels comme

clients

Toutes les actions internes agrave une organisation peuvent srsquointeacutegrer dans des processus qui

conditionnent directement ou indirectement la capaciteacute de lrsquoorganisation agrave satisfaire le

client final ou lrsquousager

Initiation au management copy CRCF ndash J Sornet Page 25 48

Aborder le fonctionnement de lrsquoentreprise par ses processus (approche processus) permet

de mettre en eacutevidence les chaicircnes drsquoactiviteacutes qui conduisent aux produits leurs

dysfonctionnements leurs coucircts la formation des deacutelais et la souplesse (la flexibiliteacute)

disponible pour satisfaire la clientegravele finale Lrsquoameacutelioration des processus a un impact visible

et direct sur chaque produit proposeacute aux clients

Lrsquoapproche processus provoque une eacutevolution de la faccedilon de travailler

- en faisant peacuteneacutetrer la laquo voix du client raquo au plus profond de lrsquoentreprise (et plus seulement

dans les services commerciaux et marketing)

- en mettant en eacutevidence des possibiliteacutes de rationalisation (par regroupement ou impartition

de certaines activiteacutes)

Remarque lrsquoapproche par les activiteacutes et les processus est agrave lrsquoorigine de la meacutethode

de deacutetermination des coucircts laquo ABC raquo - activity based costing

24 ndash Processus et fonctions

Le processus est transversal Il enchaicircne des activiteacutes qui traversent lrsquoentreprise en particulier

les services ou les deacutepartements drsquoune organisation fonctionnelle

Exemple

La division du travail par fonctions induit une charge de coordination pour assurer le

deacuteroulement du processus Elle peut geacuteneacuterer des attentes des erreurs ou des conflits drsquointeacuterecirct

(lrsquoobservation montre que des dysfonctionnements sont tregraves souvent constateacutes lors du

passage drsquoun service agrave un autre)

Organisation fonctionnelle et approche processus visent toutes deux un optimum

eacuteconomique mais leurs logiques sont diffeacuterentes

- le processus vise la satisfaction des clients (prix qualiteacute deacutelais service)

- le deacutecoupage fonctionnel cherche agrave optimiser les moyens (maximiser lrsquoeffet drsquoexpeacuterience

partager des infrastructures profiter de pocircles de compeacutetences hellip) Il apporte une ossature

hieacuterarchique stable souvent indispensable

Organisation fonctionnelle et approche processus sont donc compleacutementaires dans la

majoriteacute des cas et doivent ecirctre combineacutees judicieusement

APPLICATIONS OP

OP1 Deacutefinir flexibiliteacute systegraveme impartition

OP2 Citer huit exemples drsquoinformations essentielles pour optimiser un processus de

fabrication

Direction

Deacutepartement

commercial

(C)

Deacutepartement

administratif et

financier (AF)

Deacutepartement

Etudes (E)

Deacutepartement

Production (P)

Activiteacute

C-x Activiteacute

AF-x Activiteacute

E-x

Activiteacute

P-x

Processus x

Clie

nt

Initiation au management copy CRCF ndash J Sornet Page 26 48

OP3 Deacutegager les principes du toyotisme preacutesenteacute ci-dessous En quoi ce systegraveme est-il

initiateur de lrsquoapproche processus

Taiichi Ohno et le Toyotisme

1 - Extrait drsquoun article de Jacques BARRAUX - 1993 - LExpansion

Taiichi Ohno (1912 ndash 1990) hellip ne se prenait pas pour un visionnaire mais en imposant une

nouvelle faccedilon de produire il a reacuteinventeacute le management hellip tout le monde a entendu parler

des mots qui ont populariseacute le toyotisme dont il est le pegravere le juste-agrave-temps hellip Autant

doutils conccedilus pour lrsquoautomobile et qui ont aujourdhui une application universelle

hellip Taiichi Ohno jeune ingeacutenieur entre chez Toyota alors simple constructeur de machines

textiles Degraves 1926 apparaicirct la notion de jidoka hellip cest lart de transfeacuterer de lintelligence aux

machines pour mieux libeacuterer lintelligence des hommes Tout le contraire du taylorisme qui

juge la machine moins impreacutevisible que lhomme En 1933 Toyota se lance dans lautomobile

en sinspirant des meacutethodes ameacutericaines Mais en 1935 agrave loccasion dun voyage aux Etats-

Unis leacutetat-major de lentreprise revient fascineacute de sa visite dans un supermarcheacute La notion

de juste-agrave-temps va naicirctre de lobservation dune grande surface un lieu ougrave les clients ne

prennent que ce dont ils ont besoin et ougrave les rayons sont reacuteapprovisionneacutes pour compenser

les quantiteacutes preacuteleveacutees Ainsi le systegraveme Toyota est-il deacutejagrave dans la tecircte de ses dirigeants avant

mecircme la Seconde Guerre mondiale un demi-siegravecle avant la reacutevolution informatique et la

segmentation intensive des marcheacutes

hellip des esprits curieux comme Franccedilois Dalle en France tombent alors sous le charme des

formules et des paraboles de Taiichi Ohno En voici deux eacutechantillons

Penser agrave lenvers Cela signifie combattre les ideacutees reccedilues En lespegravece il sagit du fordisme et

du taylorisme Ohno ne croit pas agrave la planification aux effets deacutechelle et dexpeacuterience Il

propose un systegraveme industriel agrave lenvers qui permette de diversifier les produits et de les

fabriquer en petites quantiteacutes Nous ne devons plus ecirctre des paysans qui accumulent des

stocks mais des chasseurs On nimpose pas loffre On traque la demande et on la gegravere en

continu

Que les valleacutees soient hautes et les montagnes peu eacuteleveacutees Plutocirct que de concentrer tous

les efforts sur une production agrave un moment donneacute mieux vaut se doter de structures flexibles

permettant de passer agrave tout instant dune seacuterie agrave une autre Il faut eacuteviter les ruptures et les

secousses aplanir les cycles entretenir des flux reacuteguliers dactiviteacutes diversifieacutees Ce qui

implique de ne pas enfermer les hommes et les eacutequipements dans des speacutecialisations trop

eacutetroites

La flexibiliteacute le travail en groupe le refus de la dictature des machines la polyvalence et

surtout lattention constante aux signaux eacutemis par le marcheacute nappartiennent plus au

toyotisme Ces notions sont les fondements du nouvel art dorganiser de vendre et de

produire dans lindustrie comme dans les services hellip

2 - Quelques notions cleacutes

Taiichi Ohno a imagineacute la meacutethode des laquo cinq pourquoi raquo qui consiste agrave se poser cinq fois de

suite la question laquo pourquoi raquo sur le mecircme sujet de faccedilon agrave deacutecouvrir la veacuteritable cause

drsquoun problegraveme Cette meacutethode peut ecirctre appliqueacutee agrave tous les niveaux et permettre

notamment aux agents de fabrication de proposer de veacuteritables ameacuteliorations de la

production

La recherche de la qualiteacute totale (pas de deacutefaut des produits pas de rebuts pas de deacutefaut

des processus) accompagne la deacutemarche de Toyota La qualiteacute a un coucirct compenseacute par

des ventes accrues par lrsquoeacuteconomie des mesures palliatives aux deacutefauts

Initiation au management copy CRCF ndash J Sornet Page 27 48

Fiche OP1 ndash Benchmarking et processus

Le laquo benchmarking raquo consiste agrave comparer le fonctionnement de plusieurs systegravemes pour en

faire notamment ressortir les meilleures pratiques (laquo best practices raquo) Cette technique est

utiliseacutee depuis les anneacutees 80 pour ameacuteliorer la performance des entreprises Elle impose agrave

lrsquoentreprise drsquoeacutevaluer et de remettre en question ses propres modes de fonctionnement afin

de les faire eacutevoluer agrave la lueur de ce qui se fait ailleurs

Le benchmarking permet drsquoameacuteliorer les processus agrave moindre risque en fixant des objectifs

baseacutes sur des faits et donc plus facilement accepteacutes

Une classification des processus en tant que base de reacuteflexion a eacuteteacute eacutetablie aux USA par

lrsquolaquo International Benchmarking Clearinghouse raquo de lrsquoAPQC (american productivity and

quality center) en collaboration avec plusieurs dizaines drsquoentreprises

Elle se reacutesume ainsi

Le terme laquo reengineering raquo (la re-conception ou laquo reacuteingeacutenieacuterie raquo) des processus deacutesigne un

projet drsquoameacutelioration radicale des performances (de 20 agrave 50 ou plus) Il neacutecessite une

parfaite adheacutesion de la direction la constitution drsquoune petite eacutequipe de projet brillante

connaissant parfaitement les activiteacutes de lrsquoentreprise et il peut inclure un benchmarking

Le reengineering provoque geacuteneacuteralement la reacuteduction du nombre de niveaux hieacuterarchiques

(laquo delayering raquo) et lrsquoaccroissement du pouvoir de deacutecision des employeacutes (laquo empowerment raquo

ou laquo empouvoirement raquo) Bien qursquoy conduisant parfois il ne doit pas ecirctre confondu avec la

reacuteduction des activiteacutes (laquo downsizing raquo ou restructuration) et lrsquoexternalisation (laquo outsourcing raquo)

Pro

ce

ssu

s

op

eacutera

tio

nn

els

Pro

ce

ssu

s d

e m

an

ag

em

en

t e

t d

e

sup

po

rt

1 ndash

Comprendre

le marcheacute et

les clients (besoins

satisfaction)

2 ndash

Deacutevelopper

vision et

strateacutegie (contexte

concurrence)

3 ndash

Creacuteer

produits

services

processus

(concevoir

ameacuteliorer)

4 ndash

Marketing et

vente

5 ndash

Produire et

livrer (industrie

dont

ameacutelioration

processus)

6 ndash

Produire et

livrer (services)

7 ndash

Facturer et

servir les

clients (apregraves-

vente

reacuteclamations)

8 ndash Deacutevelopper et geacuterer les ressources humaines

9 ndash Geacuterer les systegravemes drsquoinformation

10 ndash Geacuterer les ressources financiegraveres et les actifs

11 ndash Appliquer un programme environnemental

12 ndash Geacuterer les relations exteacuterieures (actionnaires banques lois relations publiques hellip)

13 ndash Geacuterer lrsquoameacutelioration et le changement (eacutevaluer mesurer motiver qualiteacute totale)

Initiation au management copy CRCF ndash J Sornet Page 28 48

Fiche OP2 ndash Lrsquoorganisation par processus

Lrsquoeacutevolution drsquoune organisation aux activiteacutes reacutepeacutetitives vers lrsquoapproche processus est

geacuteneacuteralement progressive et se met en place par paliers

La mise en œuvre drsquoun veacuteritable management par processus doit ecirctre preacuteceacutedeacutee quand

lrsquoactiviteacute de lrsquoentreprise est complexe drsquoun recensement (une laquo cartographie des

processus raquo) pour mettre en eacutevidence les processus ou les familles de processus cleacutes critiques

pour le succegraves de lrsquoentreprise ougrave les efforts seront prioritaires

Des responsables de processus (laquo process owners raquo) sont ensuite deacutesigneacutes

Le responsable doit concevoir ses processus puis apregraves leur mise en œuvre assurer les

coordinations neacutecessaires les ameacuteliorer et les repreacutesenter aupregraves de la direction

Quand une structure par processus est mise en place des opeacuterateurs exeacutecutants

preacuteceacutedemment regroupeacutes dans les fonctions peuvent ecirctre affecteacutes aux processus et

drsquoanciens responsables de fonctions peuvent devenir des experts au service des processus

Lrsquoorganisation par processus peut imposer un degreacute eacuteleveacute drsquointeacutegration des activiteacutes donc

une polyvalence accrue des personnels et une reacuteduction des niveaux hieacuterarchiques

Elle neacutecessite pour le moins des compeacutetences eacutelargies au niveau des responsables de

processus (organisation administration technique hellip) dont le nombre doit rester limiteacute

(quelques dizaines au plus)

Sauf dans de tregraves petites structures lrsquoorganisation par processus se plaque geacuteneacuteralement sur

une structure plus classique

Initiation au management copy CRCF ndash J Sornet Page 29 48

ELEMENTS DE CORRIGE OP

OP1 Deacutefinir

Flexibiliteacute = adaptation au besoin (horaire variable chaicircnes robotiseacutees)

Systegraveme = ensemble organiseacute dans un but boicircte noire (sanguin nerveux meacutetrique laquo D raquo)

Impartition = sous-traitance ou externalisation (seacuteparation) drsquoactiviteacutes faire appel agrave des

partenaires plutocirct que faire soi-mecircme

OP2 Citer huit exemples drsquoinformations essentielles pour orienter lrsquooptimisation drsquoun processus

Montant des stocks (approvisionnements et produits finis)

Temps drsquoattente

Taux drsquoactiviteacute des ateliers

Rebuts

Deacutelai de production

Taux de reacuteclamations clients (qualiteacute)

Temps passeacutes en retouches finales

Turn over

Nombre drsquoarrecircts maladie

Accidents du travail

Dureacutee des arrecircts machines

OP3 Deacutegager les principes du toyotisme preacutesenteacute dans la fiche 31 En quoi ce systegraveme

repose trsquoil sur lrsquoapproche processus

Produire la quantiteacute juste neacutecessaire (agrave la demande) donc eacuteviter les stocks

Flexibiliteacute intelligence des chaicircnes de production

Qualiteacute (eacuteviter le coucirct de la non-qualiteacute)

La notion de processus est implicite ainsi que la chaicircne de valeur client

Initiation au management copy CRCF ndash J Sornet Page 30 48

DEFIS ET TENDANCES DU MANAGEMENT

Les meacutethodes de management se deacuteveloppent pour affronter le contexte eacuteconomique

Ce chapitre preacutesente les deacutefis auxquels le management contemporain doit faire face

1 ndash Lrsquoeacutevolution eacuteconomique contemporaine

A mesure que lrsquoactiviteacute eacuteconomique mondiale srsquoaccroicirct que la technologie eacutevolue les

changements sont de plus en plus rapides Ils introduisent des situations ineacutedites auxquelles les

entreprises doivent srsquoadapter en cherchant de nouvelles solutions de management Les trois

derniegraveres deacutecennies ont eacuteteacute notamment marqueacutees par les pheacutenomegravenes suivants (que nous

listons sans tenir compte des liens pouvant exister entre eux)

Pheacutenomegravene Traduction Effets

Deacute reacuteglementation

globalisation

financiegravere

titrisation

Libre circulation des capitaux accegraves

facile des particuliers au marcheacute

boursier (directement ou par

lrsquointermeacutediaire des OPCVM et SICAV)

Monteacutee en puissance du financement

des entreprises sur le marcheacute boursier

Fonds de pension

(retraites) et fonds

souverains (eacutetats)

Poids boursier important drsquoinvestisseurs

institutionnels qui cherchent un haut

rendement financier (dividendes ou

valorisation boursiegravere)

Pression sur les grandes entreprises

influence sur les strateacutegies

Mondialisation Liberteacute des eacutechanges internationaux Accroissement de la concurrence

recherche drsquoavantages eacuteconomiques

par la deacutelocalisation (biens et

services) la concentration des efforts

(recentrage) problegravemes drsquoemploi

multiplication des transports perte

drsquoinfluence des politiques

Baisse de lrsquoemploi

occidental

(notamment

industriel)

Moins de fabrications fabrications

automatiseacutees recours aux moyens

informatiques

Activiteacute reporteacutee sur le commerce la

conception et les services chocircmage

charge sociale

Restructurations Optimisation des entreprises

abaissement des coucircts augmentation

des marges recherche drsquoune taille

critique (eacuteconomies drsquoeacutechelle poids

sur le marcheacute)

Recentrages externalisations fusions

deacutelocalisations constitution de grands

groupes

NTIC (nouvelles

technologies de

lrsquoinformation et de

la communication)

Mise en œuvre des reacuteseaux (dont

internet) et drsquoapplications

informatiques communicantes

Nouvelles formes de commerce

marcheacute international deacutelocalisation

du travail intellectuel reacuteorganisation

de la distribution

Rareacutefaction relative

des matiegraveres

premiegraveres

Recherche de substituts exploration

miniegravere coucircts drsquoexploitation des

gisements accrus

Augmentation des coucircts variations

erratiques du cours des matiegraveres

deacutestabilisations politiques

Evolution

geacuteopolitique et

eacuteconomique

mondiale

Chute de lrsquoURSS transformation des

eacuteconomies collectivistes pays

eacutemergents (Chine Inde Breacutesil Russie)

()

Accroissement de la population

mondiale (4 agrave 6 7 milliards de 1970 agrave

2008)

Libeacuteralisme sans frein () nouvelles

puissances eacuteconomiques

opportuniteacutes de deacuteveloppement

nouveau partage des ressources

ineacutegaliteacutes baisse du soutien aux PVD

laquo Terrorisme raquo Actions armeacutees pression de groupes

armeacutes non gouvernementaux

Deacutestabilisations reacutegionales charge

des deacutepenses militaires

Deacuteveloppement

durable

Recherche drsquoune croissance eacutequitable

et respectueuse de lrsquoenvironnement

Pression sur les entreprises (eacutetats

associations de consommateurs

eacutecologistes ONG)

() Reacutecemment quelques affaires (Enron laquo subprimes raquo Vivendi Universal Socieacuteteacute

Geacuteneacuterale Airbus par exemple) et agrave plus grande eacutechelle la crise financiegravere de 2008 ont

montreacute les dangers drsquoune libeacuteralisation sans controcircles suffisants

Initiation au management copy CRCF ndash J Sornet Page 31 48

() Des alliances eacuteconomiques naissent entre pays eacutemergents (notamment en

ameacuterique centrale creacuteation de la Banque du Sud en 2008 par exemple) et lrsquoon

commence agrave imaginer une baisse progressive de lrsquoinfluence eacuteconomique des Etats

Unis

2 ndash Les deacutefis actuels du management

21 ndash Les grandes orientations

Lrsquoeacutevolution eacuteconomique suggegravere quelques pistes parfois concurrentes pour lrsquoaction du

manager contemporain On y retrouve au premier plan la construction drsquoune vision qui est

une composante commune du leadership

Objectif du manager

pour lrsquoentreprise

Justification Facteurs de reacuteussite

Construire une vision Eclairer lrsquoavenir de lrsquoentreprise partager

un but souder motiver

Effort de reacuteelle prospection

volontarisme de la direction

bonne communication

Reacuteactiviteacute et flexibiliteacute

(sous tous les aspects

agrave tous niveaux)

Srsquoadapter rapidement au marcheacute Bonne organisation des processus

personnel compeacutetent autonome

et motiveacute structure hieacuterarchique

alleacutegeacutee robotisation

Deacutegager des profits Reacutemuneacuterer les apporteurs de capitaux

srsquoautofinancer

Ajuster coucircts et structures

Exploiter les nouvelles

technologies

Reacuteactiviteacute ajuster coucircts et deacutelais

reacutepondre au marcheacute suivre les clients

Organiser le SI de faccedilon

pertinente eacuteviter le coucirct excessif

drsquoinvestissements trop en

laquo pointe raquo (laquo essuyer les placirctres raquo)

utiliser judicieusement les services

exteacuterieurs

Bacirctir des alliances

(contrats fusions)

Deacutevelopper une activiteacute limiter les coucircts

de transaction () atteindre la taille

critique et de meilleurs rendements se

recentrer sur une activiteacute profitable

Dominer les processus se donner

une identiteacute lisible externaliser se

doter drsquoune capaciteacute financiegravere

suffisante

Valoriser lrsquoimage Attirer les clients favoriser les alliances

donner confiance (apporteurs de fonds

employeacutes clients partenaires socieacuteteacute

civile)

Instaurer des regravegles de

gouvernance inteacutegrer le

deacuteveloppement durable

respecter lrsquoenvironnement

Geacuterer les risques Faire face aux aleacuteas eacuteconomiques et

technologiques (conjoncture politiques

accidents malveillance)

Creacuteer un systegraveme drsquoalerte geacuterer

la crise (reacuteaction raisonneacutee

sceacutenarios poursuite de

lrsquoexploitation dans un contexte

instable) mise en place de

proceacutedures drsquoapprentissage pour

ameacuteliorer les reacuteactions au fil du

temps

Geacuterer le changement Faire face agrave lrsquoeacutevolution de la demande

la pression sur les prix la variation des

performances financiegraveres la

concurrence la globalisation des

marcheacutes lrsquoeacutevolution technologique aux

fusions ou alliances aux changements

de reacuteglementation de direction hellip ()

Bonne communication pour

donner du sens au changement

et obtenir lrsquoadheacutesion du personnel

Rassembler et geacuterer les

connaissances former le

personnel

Innover Garder un avantage concurrentiel se

diffeacuterencier

Veille technologique et

commerciale investissement

Ouverture

internationale

Elargir le marcheacute saisir les opportuniteacutes Veille commerciale partenariats

() La theacuteorie des coucircts de transaction deacuteveloppeacutee par OE Williamson dans les

anneacutees 70 integravegre les coucircts lieacutes au recours au marcheacute (recherche et choix drsquoun

fournisseur neacutegociation reacutedaction de contrat suivi des eacutechanges risque de rupture

Initiation au management copy CRCF ndash J Sornet Page 32 48

drsquoapprovisionnement hellip) On peut en conclure que lrsquointeacutegration de diffeacuterentes

activiteacutes agrave lrsquoentreprise (la laquo firme raquo) preacutesente des avantages Mais des coucircts de

transaction internes doivent aussi ecirctre consideacutereacutes (preacuteparation organisation

surveillance hellip) et certaines formes de coopeacuteration continue avec les fournisseurs

permettent de reacuteduire le coucirct des transactions externes

() drsquoapregraves laquo Les meilleures pratiques de management raquo - Brilman Heacuterard ndash EO

Une eacutetude du Conference Board (2002) liste les deacutefis du management vus par 700 leaders

mondiaux Soit en reacutesumeacute avec indication du score correspondant

1 ndash Fideacuteliser les clients (42)

2 ndash Reacuteduire les coucircts (38)

3 ndash Accroicirctre flexibiliteacute et reacuteactiviteacute (29)

4 ndash Amener les employeacutes agrave adheacuterer aux valeurs et visions de lrsquoentreprise (26)

5 ndash Deacutevelopper et retenir les leaders (25)

6 ndash Geacuterer acquisitions et alliances (24)

7 ndash Accroicirctre lrsquoinnovation (20)

En fin de classement citoyenneteacute et reacuteputation (4) et ameacutelioration de la diversiteacute (3)

22 ndash Les techniques disponibles

Pour faire face aux deacutefis le manager dispose de nouveaux concepts et de nouvelles

techniques Le tableau ci-dessous en donne un reacutesumeacute et indique les domaines qursquoils

influencent principalement

Initiation au management copy CRCF ndash J Sornet Page 33 48

Principaux concepts techniques outils Incidence principale sur

Internet

- e-commerce (commerce eacutelectronique site

entreprise)

- CRM ou GRC (gestion de la relation client)

- e-procurement (gestion des approvisionnements

par le reacuteseau)

- messagerie eacutelectronique

- e-recrutement

Vente accegraves au marcheacute

Relation client reacuteactiviteacute personnalisation

fideacutelisation

Deacutelais coucircts

Communication transfert de donneacutees (piegraveces

jointes) tous domaines

Communication recrutement

Intranet reacuteseau drsquoentreprise SI

- knowledge management (gestion des

connaissances)

- e-learning (apprentissage en ligne)

- plateforme de travail collaboratif (groupware)

- workflow (circulation eacutelectronique de

documents enchaicircnement de processus)

- e-RH portail RH (libre accegraves aux postes agrave

pourvoir informations candidatures hellip)

- PGI (progiciel de gestion inteacutegreacute) ou ERP

Innovation capaciteacute au changement veille

documentaire

Formation du personnel accompagnement des

changements

Coordination communication interne

Coordination

Communication interne (voire internet en

externe) reacuteduction des coucircts climat drsquoentreprise

recrutement plans de carriegraveres hellip

Coucircts fiabiliteacute du systegraveme drsquoinformation deacutelais

processus (continuiteacute inteacutegration)

Logistique inteacutegreacutee

Supply Chain Management (SCM) gestion de la

logistique (incluant les approvisionnements)

Processus deacutelais coucircts

Externalisation

Valorisation du capital humain

GPEC (gestion preacutevisionnelle des emplois et

compeacutetences)

Coaching

Reacuteactiviteacute de lrsquoentreprise conservation des

compeacutetences rendements individuels turn-over

adaptation des compeacutetences motivation

Efficaciteacute individuelle controcircle reacutegulation

progregraves processus

Approche processus

Optimisation des processus

Deacutemarche qualiteacute totale (TQM ndash total quality

management)

Empowerment (empouvoirement)

Benchmarking reacuteingeacutenieacuterie

Coucircts marges qualiteacute deacutelais flexibiliteacute

externalisation eacutelargissement des compeacutetences

organisation

Ameacutelioration des processus (meacutetiers et supports)

Autonomie compeacutetences des employeacutes

Ameacutelioration des processus restructuration

Management par la valeur

Parties prenantes

Satisfaction des parties prenantes financement

motivation collaborations hellip

Collaboration inter organisations

Reacuteseaux drsquoentreprises alliances

EDI (eacutechange de donneacutees informatiseacutees) extranet

Impartition externalisation (outsourcing)

Coucircts recentrage investissements lancement

drsquoactiviteacute

Coucircts reacuteactiviteacute deacutelais relations avec

lrsquoadministration

Coucircts recentrage limitation des investissements

Ethique drsquoentreprise

Gouvernance drsquoentreprise (mode de direction

encadreacute par des regravegles)

Rocircle socieacutetal deacuteveloppement durable

environnement

Image de lrsquoentreprise reacutegulation du top

management relations actionnaires

Image peacutenaliteacutes et amendes objectifs

strateacutegiques

Initiation au management copy CRCF ndash J Sornet Page 34 48

23 ndash Le rocircle socieacutetal des entreprises

La responsabiliteacute socieacutetale de lrsquoentreprise (RSE) deacutesigne le rocircle qursquoelle prend dans la socieacuteteacute

au-delagrave de son activiteacute purement geacuteneacuteratrice de profit On parle aussi drsquoentreprise citoyenne

La RSE est indissociable du deacuteveloppement durable de porteacutee mondiale et dont les trois

piliers sont

- eacuteconomique (favoriser le deacuteveloppement les eacutechanges internationaux)

- social (accegraves aux soins eacuteducation conditions de travail hellip)

- environnemental (pollution preacuteservation des ressources hellip)

La RSE integravegre notamment une preacuteoccupation sociale de lrsquoentreprise vis-agrave-vis de ses salarieacutes

(seacutecuriteacute et santeacute au travail juste reacutemuneacuteration deacuteveloppement personnel hellip) Elle conduit agrave

tenir compte dans le management drsquoune vision exteacuterieure agrave lrsquoentreprise qui peut avoir des

reacutepercussions possibles sur son activiteacute eacuteconomique

Lrsquoentreprise peut aussi tirer avantage drsquoune deacutemarche responsable par la baisse de certains

coucircts (plus faibles consommations drsquoeacutenergies reacuteduction des transports hellip)

Le rocircle socieacutetal de lrsquoentreprise a eacuteteacute reconnu en France par la loi laquo NRE raquo de 2001 (loi sur les

nouvelles reacutegulations eacuteconomiques) qui oblige les socieacuteteacutes franccedilaise coteacutees sur un marcheacute

reacuteglementeacute agrave rendre compte dans leur rapport annuel de leur gestion sociale et

environnementale au travers de leur activiteacute

Article 116 de la loi Le rapport viseacute agrave larticle L 225-102 rend compte hellip laquo Il comprend

eacutegalement des informations dont la liste est fixeacutee par deacutecret en Conseil dEtat sur la

maniegravere dont la socieacuteteacute prend en compte les conseacutequences sociales et

environnementales de son activiteacute Le preacutesent alineacutea ne sapplique pas aux socieacuteteacutes

dont les titres ne sont pas admis aux neacutegociations sur un marcheacute reacuteglementeacute raquo

Une norme ISO 14000 integravegre ces preacuteoccupations et des taxes eacutecologiques sont

progressivement creacutees

3 ndash Le management par la valeur

31 ndash De lrsquoanalyse au management par la valeur

Lrsquoanalyse de la valeur est neacutee en 1947 aux Etats-Unis (General Electrics) Cette technique

consiste agrave eacutelaborer des produits conformes aux attentes de la clientegravele mais sans excegraves pour

trouver un bon compromis entre valeur pour le client et coucirct Le produit optimal est deacutefini agrave

partir drsquoenquecirctes qui deacuteterminent le besoin client (ou plutocirct drsquoun client laquo type raquo)

Exemple il est inutile de concevoir un petit veacutehicule citadin capable de parcourir

500 000 km sans avarie compte tenu des effets de mode et du faible kilomeacutetrage

annuel Par contre le marcheacute peut exiger un fonctionnement sans faille sur 150 000 km

soit dix ans en moyenne ce qui conditionne les coucircts de production

Cette recherche drsquoun ajustement de valeur au besoin des clients eacutetait un preacutecurseur du

management par la valeur qui recherche plus largement la creacuteation de valeur pour

chacune des parties prenantes de lrsquoentreprise tout en lui meacutenageant un reacutesultat suffisant

Plus geacuteneacuteralement le management par la valeur est deacutefini par une norme europeacuteenne (EN

12973)

Le management par la valeur est un style de management particuliegraverement destineacute agrave

mobiliser les individus agrave deacutevelopper les compeacutetences et agrave promouvoir les synergies et

Initiation au management copy CRCF ndash J Sornet Page 35 48

linnovation avec pour objectif la maximisation de la performance globale dun

organisme Le management par la valeur apporte une nouvelle faccedilon dutiliser nombre

de meacutethodes de management existantes Il est en coheacuterence avec le Management

de la qualiteacute

Cette approche du management pose de nombreuses questions notamment quelles

prioriteacutes et quelles valeurs attribuer aux parties prenantes comment appreacutehender la

perception par les parties prenantes de la valeur qui leur est affecteacutee

32 ndash La valeur client

Le processus drsquoeacutelaboration drsquoun produit qui consomme des ressources coucircteuses doit creacuteer

une valeur suffisante pour provoquer lrsquoachat par le client final La production drsquoune valeur

reconnue par le client est vitale pour lrsquoentreprise mais sa deacutetermination est parfois complexe

La valeur du produit perccedilue par le client integravegre des eacuteleacutements en partie subjectifs

- une valeur drsquousage (le produit reacutepond agrave un besoin)

- une valeur drsquoestime (lrsquoimage apporteacutee par le produit un aspect affectif)

- une valeur drsquoeacutechange (deacuteduite de lrsquoespoir de revente du produit)

Valeurs drsquousage drsquoestime et drsquoeacutechange deacutependent implicitement de la qualiteacute (un bien peu

fiable est impropre agrave lrsquousage attendu de mauvaise qualiteacute notoire il nrsquoapporte pas une

image positive et ses deacutefauts connus nuisent agrave sa revente) Une eacutevaluation de la qualiteacute

intervient donc dans la valeur perccedilue du produit

Par ailleurs le client considegravere le coucirct drsquoobtention du produit (les charges qursquoil doit supporter

pour acqueacuterir le produit lrsquoeffort qursquoil doit faire pour trouver le produit et les frais de mise agrave

disposition)

Le prix perccedilu par le client est geacuteneacuteralement supeacuterieur au prix de vente

Le client achegravete theacuteoriquement le produit qui preacutesente la diffeacuterence valeur perccedilue ndash prix

perccedilu la plus favorable ou le meilleur rapport prix perccedilu qualiteacute perccedilue et dans certains

cas celui qui a le prix produit le plus bas

Remarque les valeurs du scheacutema ci-dessus changent durant le cycle de vie du produit

(un nouveau produit peut avoir une valeur perccedilue plus eacuteleveacutee qursquoen fin de vie) La

valeur client ne peut ecirctre eacutevalueacutee que par enquecirctes et ne peut donc ecirctre deacutefinie avec

certitude

La notion de laquo satisfaction client raquo conseacutecutive agrave une vente influence aussi le prix produit et

le prix perccedilu

- lrsquoentreprise gagne sur les coucircts de recherche de clientegravele

- le client nrsquoa pas agrave rechercher un nouveau fournisseur et beacuteneacuteficie drsquoun coucirct drsquoobtention

plus bas

valeur perccedilue client

prix perccedilu client

coucirct produit Marge (valeur creacuteeacutee pour

lrsquoentreprise)

euros

prix produit

Valeur creacuteeacutee

pour le client

Initiation au management copy CRCF ndash J Sornet Page 36 48

La satisfaction du client deacutepend de facteurs qualitatifs aussi divers que la fiabiliteacute du produit

la vitesse de reacuteaction du fournisseur lrsquoattitude des commerciaux lrsquoefficaciteacute du service

apregraves-vente la netteteacute des contrats ou la justesse de la facture

Valeur perccedilue coucirct marge et satisfaction reacutesultent de processus allant de la conception du

produit jusqursquoagrave sa livraison et son apregraves-vente La deacutemarche laquo processus raquo et lrsquolaquo analyse de la

valeur raquo en forccedilant la recherche de solutions efficientes agrave tout niveau administratif

technique commercial et apregraves-vente sont donc neacutecessaires pour bien positionner

lrsquoentreprise sur son marcheacute

Pour autant le risque commercial ne peut jamais ecirctre annuleacute et lrsquooffre de lrsquoentreprise ne

satisfait geacuteneacuteralement pas en milieu concurrentiel tous ses clients potentiels

33 - La creacuteation de valeur pour les autres parties prenantes

Les salarieacutes

La creacuteation drsquoune valeur suffisante pour les salarieacutes est reconnue comme neacutecessaire car des

observations montrent que la satisfaction des clients en deacutepend Moins souvent eacutevoqueacutee en

peacuteriode de chocircmage elle nrsquoest prioritaire que pour les employeacutes dont lrsquoentreprise souhaite

conserver les compeacutetences

La laquo valeur salarieacute raquo ne comprend pas que le salaire Le sentiment drsquoappartenance agrave un

groupe la reconnaissance lrsquoaccomplissement de soi et la construction professionnelle en

sont des eacuteleacutements importants Comme pour les clients on doit ainsi distinguer la reacutetribution

perccedilue du salaire objectif

Les actionnaires

Lrsquoactionnaire apporte des fonds propres agrave lrsquoentreprise en contrepartie de titres parfois

neacutegociables en bourse et assortis drsquoun droit de vote en assembleacutee geacuteneacuterale La valeur

attribueacutee aux actionnaires est servie en termes moneacutetaires (dividende ou augmentation de la

valeur du titre neacutegociable)

Remarque des facteurs non moneacutetaires comme lrsquoimage de lrsquoentreprise qui deacutepend

en partie de sa communication peuvent influencer la deacutecision drsquoachat de vente ou

de conservation des titres par lrsquoactionnaire

Reacutetribution perccedilue euros

Salaire objectif

Avantage non

moneacutetaire de

lrsquoemploi

Initiation au management copy CRCF ndash J Sornet Page 37 48

Compte tenu de lrsquoimportance croissante de lrsquoactionnariat dans le financement des grandes

entreprises coteacutees en bourse et notamment des investisseurs institutionnels comme les fonds

de pension des indicateurs speacutecifiques ont eacuteteacute introduits pour appreacutecier la performance des

entreprises vue par les actionnaires Par exemple la valeur ajouteacutee eacuteconomique (EVA reg

economic value added marque deacuteposeacutee de Stern Stewart ou VAE ndash valeur ajouteacutee

eacuteconomique parfois deacutenommeacutee VEC ndash valeur eacuteconomique creacuteeacutee) qui prend en compte le

coucirct du capital

LrsquoEVA correspond tregraves scheacutematiquement au calcul suivant

EVA = (PO) profit opeacuterationnel ndash (C) coucirct du capital X (CE) capitaux employeacutes

LrsquoEVA neacutecessite en pratique des retraitements assez complexes Le PO peut se deacuteterminer

selon les principes suivants

- PO = reacutesultat drsquoexploitation (avant inteacuterecircts) ndash impocirct

- PO = beacuteneacutefice courant (tenant compte des inteacuterecircts) + inteacuterecircts ndash eacuteconomie drsquoimpocirct sur les

inteacuterecircts (on exclue les eacuteleacutements financiers et lrsquoimpocirct correspondant) ndash impocirct

- lrsquoimpocirct pris en compte correspond au profit opeacuterationnel consideacutereacute (dans les cas courants agrave

13 du PO)

C = taux moyen de reacutemuneacuteration du capital (reacutesultant par exemple du dividende exigeacute de

certains investisseurs et des taux drsquoemprunts bancaires)

CE = capitaux propres et dettes portant inteacuterecirct

Remarque le profit opeacuterationnel ou reacutesultat opeacuterationnel correspond au NOPAT ndash net

operating profit after tax - anglo-saxon LrsquoEVA est eacutegale au NOPAT diminueacute de la

reacutemuneacuteration des capitaux

Exemple lrsquoentreprise X dispose drsquoun capital de 2 500 000 euro et reacutealise un beacuteneacutefice net

drsquoimpocirct de 450 000 euro (taux 33 13) Un dividende de 6 doit ecirctre verseacute aux

actionnaires et la banque lui a accordeacute un precirct de 1 200 000 euro agrave 4 Les autres

constituants des reacutesultats financier et exceptionnel sont neacutegligeables

Reacutesultat opeacuterationnel = 450 000 + 004 x 1 200 000 x 23 = 482 000 euro

Coucirct du capital = 006 x 2 500 000 + 004 x 1 200 000 x 23 = 182 000 euro

EVA = 300 000 euro

Coucirct moyen pondeacutereacute du capital (C) = (004 x 1 200 000 x23 + 006 x 2 500 000)

3 700 000 Soit 492

Si lrsquoEVA est positive lrsquoentreprise creacuteeacutee de la valeur apregraves reacutemuneacuteration des capitaux et sa

valeur boursiegravere doit augmenter

Lrsquoutilisation de lrsquoEVA comme indicateur influence le management de lrsquoentreprise car il y a

trois moyens pratiques drsquoaugmenter lrsquoEVA

- augmenter le reacutesultat opeacuterationnel

- lancer des investissements ayant une rentabiliteacute supeacuterieure agrave C

- eacuteliminer les activiteacutes ayant une rentabiliteacute infeacuterieure agrave C

Remarque lrsquoutilisation sans nuance de lrsquoEVA comme critegravere de management peut

poser problegraveme Le calcul de lrsquoEVA repose sur des ajustements comptables il est donc

sujet agrave manipulations (provisions capitalisation ou non de la RD hellip) Par ailleurs le

critegravere laquo EVA raquo pris isoleacutement peut conduire agrave chercher la rentabiliteacute agrave court terme agrave

reacuteduire les investissements prospectifs et donc nuire agrave terme au deacuteveloppement de

lrsquoentreprise

Initiation au management copy CRCF ndash J Sornet Page 38 48

Les fournisseurs reccediloivent le paiement de leurs factures plus ou moins rapidement (le deacutelai

de paiement repreacutesente une valeur consentie au fournisseur)

Lrsquoentreprise peut accroicirctre la valeur apporteacutee agrave ses fournisseurs par des actions cibleacutees

comme une contribution agrave la formation de leurs personnels certains transferts de

technologie ou de savoir faire agrave des sous-traitants une coopeacuteration suivie favorisant leur

deacuteveloppement lrsquointeacutegration agrave des campagnes de promotion

A noter que la valeur consentie aux fournisseurs peut avoir une influence sur la qualiteacute et les

deacutelais de livraison des produits

La collectiviteacute reccediloit des taxes et parfois des prestations en nature par deacutefaut ou explicites

(effort de preacuteservation de lrsquoenvironnement ameacutenagement du territoire par les implantations

aide mateacuterielle agrave des projets participation agrave la formation par exemple)

APPLICATIONS DT

DT1 Deacutefinir expliquer deacutereacuteglementation socieacutetal eacuteconomies drsquoeacutechelle coaching EDI

gouvernance

DT2 Deacuteterminer en quoi la deacutemarche TQM srsquoinscrit dans les deacutefis actuels du management

DT3 Apregraves avoir consulteacute les documents ci-dessous extraits du site drsquoAir France

(httpdeveloppement-

durableairfrancecomFRfrlocaldemarcheN4_positionnement_pphtm)

exposer les enjeux et les limites de la RSE et de la gestion des parties prenantes

Initiation au management copy CRCF ndash J Sornet Page 39 48

Dialogue avec les parties prenantes

Initiation au management copy CRCF ndash J Sornet Page 40 48

Attentes des parties prenantes

Initiation au management copy CRCF ndash J Sornet Page 41 48

Creacuteation de valeur pour les parties prenantes

La creacuteation de valeur pour les parties prenantes est au cœur de la strateacutegie du Groupe Le scheacutema de

distribution financiegravere ci-dessous donne un aperccedilu de la distribution des recettes du Groupe aux

diffeacuterentes parties prenantes actionnaires collaborateurs fournisseurs pouvoirs publics

collectiviteacutes locales etc

Initiation au management copy CRCF ndash J Sornet Page 42 48

Fiche DT1 ndash Extrait du sommaire de laquo Problegravemes eacuteconomiques raquo No 2894

La gestion des entreprises bouleverseacutee par les technologies de linternet

Reacutealiteacutes industrielles - Annales des Mines Jean-Michel Yolin

Avec lavegravenement de linternet les processus de conception de production et de vente sont

radicalement remis en cause Quel que soit le secteur dactiviteacute les technologies de linternet

permettent en effet de reacuteduire les deacutelais et de passer dun processus discontinu agrave un processus

continu Lorganisation des entreprises et leur mode de gestion en sont profondeacutement bouleverseacutes

tant au niveau individuel que collectif Linternet rend ainsi possible la reacutealisation dobjectifs que les

entreprises cherchaient agrave atteindre depuis longtemps sans y parvenir meilleure eacutecoute du client

travail sans stocks en flux tendu hieacuterarchies plates autorisant une grande reacuteactiviteacute flexibiliteacute dans

lorganisation et loutil de production acceacuteleacuteration du renouvellement des produits entreprises en

reacuteseau ougrave chacune se recentre sur son cœur de meacutetier etc

Le laquo knowledge management raquo ou comment geacuterer les connaissances

Document de travail du LAMSADE - Michel Grundstein

Peter Drucker lavait preacutedit le capital immateacuteriel eacutetait voueacute agrave devenir un facteur de compeacutetitiviteacute

pour lentreprise La libeacuteralisation des eacutechanges acceacutelegravere les processus de deacutecision de lentreprise

et implique que lassimilation des informations soit agrave la fois de meilleure qualiteacute et plus rapide Ainsi

la fonction qui consiste agrave manager les connaissances au sein de lentreprise savegravere primordiale

Bien que la prise de conscience de limportance du capital immateacuteriel ait eacuteteacute tardive - le concept

de knowledge management est apparu en France aux Etats-Unis et au Japon au milieu des

anneacutees 1990 - agrave lheure actuelle lorganisation de leacutechange dinformations et le partage des

connaissances sont devenus des facteurs cleacutes dune gestion performante de lentreprise Ils

doivent sinscrire dans un projet global destineacute agrave mettre en valeur les savoirs et les savoir-faire

individuels et collectifs

Les leccedilons du laquo coaching raquo pour le management de la qualiteacute

Humanisme et Entreprise - Martine Brasseur

Parmi les nouvelles formes de management en vogue dans les entreprises le coaching figure en

bonne place Appliqueacute au management de la qualiteacute il sagit dune pratique

daccompagnement destineacutee agrave initier et agrave faciliter le processus de deacuteveloppement dun individu

La deacutemarche consiste agrave affirmer que tout individu est en quecircte de qualiteacute agrave condition toutefois

de ne pas lui imposer des contraintes lempecircchant de progresser On considegravere notamment les

erreurs comme potentiellement feacutecondes En deacutefinitive le coach donne au coacheacute la permission

de reacuteussir en lui donnant aussi la permission deacutechouer

Initiation au management copy CRCF ndash J Sornet Page 43 48

Fiche DT2 ndash Management strateacutegique les sept deacutefis agrave relever dici agrave 2016

Extrait drsquoun article du site wwwlentreprisecom -Sabine Blanc - Mis en ligne le 20032007

(httpwwwlentreprisecom325article11977html)

Une eacutetude anglaise publieacutee par lopeacuterateur Orange Grande-Bretagne deacutecrypte la mutation

des formes de travail et les enjeux majeurs pour les entreprises de demain afin decirctre au top

de la compeacutetitiviteacute Voici les challenges-cleacutes pour les managers qui veulent rester dans la

course hellip

1 - Future organisation du travail les quatre laquo mondes raquo possibles

La reacutealiteacute sera probablement un meacutelange de ces quatre sceacutenarios souligne lrsquoeacutetude

Les mondes mutuels Tout se passe dans le cadre des communauteacutes locales vie priveacutee

comme professionnelle Le modegravele coopeacuteratif preacutevaut au lieu du laquo big business raquo Oublieacutes

aussi dans ce systegraveme les trajets pour aller au bureau les gens preacutefegravereront travailler dans de

petites entreprises locales souvent connecteacutees au reacuteseau drsquoautres structures similaires

Les laquo reacutepondants raquo (en anglais laquo replicants raquo) La figure du consultant freelance deviendra

dominante tandis que celle du salarieacute deacuteclinera Il ne sera pas rare de travailler pour plusieurs

entreprises On perdra en seacutecuriteacute de lrsquoemploi en visibiliteacute et en routine ce que lrsquoon gagnera

en liberteacute La majeure partie des tacircches srsquoeffectuera chez soi avec la possibiliteacute de srsquoinstaller

temporairement dans les bureaux de son client du moment Dans un contexte dincertitude

sur lrsquoavenir les travailleurs alterneront peacuteriodes drsquoactiviteacute intense et repos Ce sera agrave eux

drsquoaller vers les entreprises et non lrsquoinverse mecircme si celles-ci devront veiller agrave rester attractives

Les cottages eacutelectroniques Comme ce nom le suggegravere le teacuteleacutetravail deviendrait la norme

univers priveacute et professionnel se confondant Plus besoin de subir une heure de transport les

salarieacutes se logueront de chez eux sur le reacuteseau de lrsquoentreprise Les reacuteunions se tiendront dans

de petits bureaux centraux situeacutes agrave courte distance La flexibiliteacute du temps de travail srsquoimpose

Les salarieacutes disposeront de plus de marge de liberteacute dans leur activiteacute

Les disciples de la nueacutee Cette appellation poeacutetique cache simplement une extension de

lrsquoorganisation actuelle des grandes entreprises avec des salarieacutes se rendant sur un lieu de

travail centraliseacute Le rocircle croissant des technologies de lrsquoinformation multipliera les faccedilons de

collaborer et accroicirctra lrsquoefficaciteacute Le controcircle du travail sera omnipreacutesent La frontiegravere entre

travail et vie priveacutee restera marqueacutee

2 - Sept deacutefis pour les entreprises et leur managers

Quoi qursquoil advienne les entreprises et leurs dirigeants devront concentrer leurs efforts sur sept

points-cleacutes pour srsquoadapter Voici quelques exemples de probleacutematiques souleveacutees par le

rapport et des pistes de solution

Le leadership Les managers devront entre autres savoir persuader et influencer des

travailleurs beaucoup plus indeacutependants Ils auront aussi agrave repenser les niveaux auxquels

prendre les deacutecisions strateacutegiques en haut ou au contraire agrave des degreacutes moins eacuteleveacutes de la

pyramide hieacuterarchique

gt Faire du management une force facilitant les activiteacutes transversales plutocirct que la reacuteduire agrave

la seule fonction de deacutecision

La culture drsquoentreprise Davantage de salarieacutes capables de reacutefleacutechir seront neacutecessaires

tandis que les tacircches qui peuvent ecirctre automatiseacutees ou scripteacutees diminueront Un des

enjeux creacuteer une culture agrave mecircme drsquoattirer et drsquoencourager les personnes preacutesentant ces

qualiteacutes de reacuteflexion requises dans un contexte de compeacutetition accrue et de plus grande

indeacutependance des travailleurs

Initiation au management copy CRCF ndash J Sornet Page 44 48

gt Passer si neacutecessaire drsquoune culture drsquoentreprise forte agrave un mode drsquoengagement plus

consensuel moins rebutant

La marque Conseacutequence du recours croissant agrave lrsquo laquo outsourcing raquo lrsquoimage drsquoune marque

deacutependra plus drsquoagents exteacuterieurs qui ne fonctionnent pas forceacutement selon le mecircme mode

drsquoorganisation Comment garder le controcircle dessus

gt Choisir le mode qui corresponde le plus agrave vos valeurs et preacutevoir un programme de risk

management qui mette en eacutevidence ougrave les conflits sont susceptibles de jaillir

Lrsquoinnovation Plus que jamais il faudra faire face agrave une acceacuteleacuteration du rythme de

lrsquoinnovation en proposant constamment des solutions adapteacutees

gt Tisser des partenariats strateacutegiques avec drsquoautres entreprises pour partager les coucircts et les

fruits de lrsquoinnovation

Le deacutefi opeacuterationnel et technologique De quelle faccedilon controcircler lrsquoinformation crsquoest-agrave-dire

faire en sorte que les bonnes personnes accegravedent facilement agrave une information toujours en

phase tout en maintenant la seacutecuriteacute

gt Recourir agrave des laquo feuilles de route des futurs raquo syntheacutetisant en une page les indicateurs

sociaux et de consommation ainsi que les eacutevolutions technologiques et leacutegislatives qui

influent sur les changements et indiquant comment ils modifient vos marcheacutes vos clients et

votre organisation

La qualiteacute Si de nouveaux proceacutedeacutes ont pu deacutegrader la qualiteacute comme le recours agrave des

centres drsquoappel externaliseacutes drsquoautres ideacutees se sont reacuteveacuteleacutees plus prometteuses comme en

teacutemoigne le succegraves de certaines compagnies aeacuteriennes low cost Elles ont su conjuguer prix

serreacutes et services eacuteleveacutes ce qui devra devenir la norme estime lrsquoeacutetude

gt Continuer de rechercher la qualiteacute Elaborez aussi une bonne prestation service qui inclut

une livraison de qualiteacute voire creacuteez-la en partenariat avec les consommateurs

La leacutegislation La question de la proprieacuteteacute intellectuelle pourrait ecirctre probleacutematique Elle est

deacutejagrave source de conflits comme en teacutemoigne le procegraves pour violation de brevet intenteacute agrave RIM

le fabricant canadien du Blackberry par NTP Que pourra-t-on et que faudra-t-il proteacuteger par

un brevet Il sera eacutegalement neacutecessaire drsquoadapter la leacutegislation aux nouveaux modes

drsquoorganisation

gt Collaborer avec les acteurs du mecircme secteur et les leacutegislateurs pour deacutevelopper les

modegraveles des lieux de travail du futur et bacirctir le droit le plus adeacutequat

Orange a-t-il vu juste dans ses preacutevisions Rendez-vous dans neuf ans pour la reacuteponsehellip

Initiation au management copy CRCF ndash J Sornet Page 45 48

Fiche DT3 ndash Le management par la qualiteacute totale

Extrait drsquoune lettre drsquoinformation du cabinet Baud Accordance Consulting AD2 consultants ndash

2002

1 - Le TQM (Total Quality Management) offre pour lentreprise une vision de la qualiteacute plus

large et transversale

Son principe est simple La finaliteacute de lEntreprise est de deacutevelopper la satisfaction de ses

clients tout en eacutetant beacuteneacuteficiaire cest agrave dire pas agrave nimporte quel prix Elle doit ameacuteliorer sa

rentabiliteacute au travers de la deacutemarche qualiteacute La Qualiteacute Totale vise agrave fournir aux clients

externes et internes une reacuteponse adeacutequate agrave leurs attentes dans le meilleur rapport qualiteacute

prix la meilleure efficience

Elle considegravere pour cela lensemble des processus de lentreprise ayant une incidence sur la

qualiteacute et la satisfaction des clients

Le TQM fait ainsi une large place agrave

la deacutefinition et la planification de la strateacutegie geacuteneacuterale

la coheacuterence de la politique qualiteacute avec la strateacutegie

la deacutemultiplication de la politique qualiteacute dans toutes les directions de lentreprise

la relation client fournisseur interne

la prise en compte de lenvironnement concurrentiel

la consideacuteration de lensemble des risques potentiels financiers sociaux concurrentielshellip

limplication et la motivation du personnel

lanalyse des besoins des clients et le positionnement marketing

la maicirctrise des processus transverses internes

les reacutesultats sous tous ses aspects y compris financiers commerciaux image

De nombreux reacutefeacuterentiels sont relatifs agrave la Qualiteacute Totale hellip Tous ces reacutefeacuterentiels imposent un

questionnement plus profond et indiscret sur le mode de fonctionnement de lentreprise et

son management

helliphellip

2 - LISO 9001 2000 au travers du deacuteploiement des processus (management supports

reacutealisation et ameacutelioration continue) reacutepond quelque peu agrave la mecircme logique

LISO est une ouverture indeacuteniable vers la logique du TQM mais ne se reacutefegravere pas agrave la notion

defficience

Les dirigeants sont cependant sensibles agrave la neacutecessaire reacuteduction des coucircts de non-qualiteacute

et dobtention de la qualiteacute agrave la rentabiliteacute du systegraveme de management de la qualiteacute

mais ne perccediloivent pas toujours la qualiteacute comme une deacutemarche globale

Les deacutemarches qualiteacute commencent bien souvent par la remise en cause de lorganisation

leacutevaluation critique de son efficaciteacute lexamen des processus et la mise en eacutevidence des

lourdeurs administratives

La qualiteacute devient laffaire de tous hellip

Initiation au management copy CRCF ndash J Sornet Page 46 48

Fiche DT4 ndash Le deacuteveloppement durable et la RSE

Extrait du site wwwvigeocom

(httpwwwvigeocomcsr-rating-agencyfrmethodologiecriteres-de-recherche37-

criteres-d-analysehtml)

Deacuteveloppement durable laquo un deacuteveloppement qui reacutepond aux besoins du preacutesent sans compromettre

la capaciteacute des geacuteneacuterations futures de reacutepondre aux leurs raquo (Commission mondiale sur lrsquoenvironnement

et le deacuteveloppement ndash 1987)

Reacutefeacuterentiel drsquoeacutevaluation des entreprises par le groupe Vigeacuteo (le groupe mesure les performances et le

niveau de maicirctrise des risques de responsabiliteacute sociale des entreprises et des organisations - site

wwwvigeocom)

1 Ressources Humaines Ameacutelioration continue des relations professionnelles des relations drsquoemploi et des conditions de travail 2 Droits humains sur les lieux de travail Respect de la liberteacute syndicale et promotion de la neacutegociation collective non discrimination et promotion de lrsquoeacutegaliteacute eacutelimination des formes de travail proscrites (enfants travail forceacute) preacutevention des traitements inhumains ou deacutegradants de type harcegravelements sexuels protection de la vie priveacutee et des donneacutees personnelles 3 Environnement Protection sauvegarde preacutevention des atteintes agrave lenvironnement mise en place drsquoune strateacutegie manageacuteriale approprieacutee eacuteco conception protection de la biodiversiteacute et maicirctrise rationnelle des impacts environnementaux sur lrsquoensemble du cycle de vie des produits ou services

4 Comportements sur les marcheacutes Prise en compte des droits et inteacuterecircts des clients inteacutegration de standards sociaux et environnementaux dans la seacutelection des fournisseurs et sur lrsquoensemble de la chaicircne drsquoapprovisionnement preacutevention effective de la corruption respect des regravegles concurrentielles 5 Gouvernement drsquoentreprise Efficience et probiteacute assurance de lrsquoindeacutependance et de lrsquoefficaciteacute du Conseil drsquoadministration effectiviteacute et efficience des meacutecanismes drsquoaudit et de controcircle et notamment inclusion des risques de responsabiliteacute sociale respect des droits des actionnaires et notamment des minoritaires transparence et rationaliteacute de la reacutemuneacuteration des dirigeants 6 Engagement socieacutetal Effectiviteacute inteacutegration manageacuteriale de lrsquoengagement contribution au deacuteveloppement eacuteconomique et social des territoires drsquoimplantation et de leurs communauteacutes humaines engagements concrets en faveur de la maicirctrise des impacts socieacutetaux des produits et des services contribution transparente et participative agrave des causes drsquointeacuterecirct geacuteneacuteral

Initiation au management copy CRCF ndash J Sornet Page 47 48

ELEMENTS DE CORRIGE DT DT1 Deacutefinir expliquer

Deacutereacuteglementation = suppression des contraintes eacuteconomiques (libre eacutechange des biens et

capitaux)

Socieacutetal = qui se rapporte agrave la structure agrave lrsquoorganisation ou au fonctionnement de la socieacuteteacute

Economies drsquoeacutechelle = reacuteduction des coucircts lieacutee au niveau drsquoactiviteacute (amortissement des

charges fixes)

Coaching = accompagnement de personnes ou deacutequipes pour le deacuteveloppement de leurs

potentiels

EDI = eacutechange de donneacutees informatiseacutees ET standardiseacutees (ex SWIFT bancaire edifact

documents deacuteclaratifs)

Gouvernance = exercice du pouvoir la bonne gouvernance est participative et eacutequitable

conforme agrave lrsquointeacuterecirct commun

DT2 Deacuteterminer en quoi la deacutemarche TQM srsquoinscrit dans les deacutefis actuels du management

Voir notamment fiche 43

Maicirctrise des processus reacuteduction des coucircts reacuteactiviteacute et satisfaction de la clientegravele = faire

face agrave la concurrence

Ameacutelioration de lrsquoimage motivation du personnel

DT3 Apregraves avoir consulteacute les documents ci-dessous extraits du site drsquoAir France

(httpdeveloppement-

durableairfrancecomFRfrlocaldemarcheN4_positionnement_pphtm)

exposer les enjeux et les limites de la RSE et de la gestion des parties prenantes

Trame geacuteneacuterale possible

Introduction

Les deacutefis contemporains (accroissement de la concurrence devenue mondiale recherche

de nouveaux avantages concurrentiels pression de la socieacuteteacute besoin drsquoimage et de projet

lisible pour mener lrsquoentreprise crise et scandales du libeacuteralisme hellip) RSE et PP

Deacuteveloppement (voir cours)

1 ndash Parties prenantes et management par la valeur

PP deacutefinir citer reacutesumer lrsquoavantage rechercheacute (fideacuteliser motiver recherche drsquoalliances

implicites)

PP moyens (dont exemples AF) et meacutethode de management par la valeur (reacutepartie)

2 ndash La responsabiliteacute socieacutetale de lrsquoentreprise

RSE 3 axes

- eacuteconomique (favoriser le deacuteveloppement les eacutechanges internationaux)

- social (accegraves aux soins eacuteducation conditions de travail hellip)

- environnemental (pollution preacuteservation des ressources hellip)

RSE gouvernance drsquoentreprise facteur drsquoimage inteacutegrable dans la deacutemarche PP

Article 116 de la loi Le rapport viseacute agrave larticle L 225-102 rend compte hellip laquo Il comprend

eacutegalement des informations dont la liste est fixeacutee par deacutecret en Conseil dEtat sur la maniegravere

dont la socieacuteteacute prend en compte les conseacutequences sociales et environnementales de son

activiteacute Le preacutesent alineacutea ne sapplique pas aux socieacuteteacutes dont les titres ne sont pas admis aux

neacutegociations sur un marcheacute reacuteglementeacute raquo

Initiation au management copy CRCF ndash J Sornet Page 48 48

RSE exemple AF (ONG fournisseurs)

3 ndash Liens entre PP et RSE

- la RSE introduit de nouvelles PP

- la RSE suppose le respect des PP usuelles (employeacutes clients notamment)

4 - Probleacutematique

- deacutefinir la valeur reacuteellement apporteacutee par une gestion des PP (confusion salaire ndash valeur

idem impocircts hellip ex laquo valeur ajouteacutee raquo)

- communication (neacutecessaire mais aller au-delagrave)

- marginaliteacute des deacutepenses RSE (efficaciteacute sinceacuteriteacute de lrsquoengagement marge de manœuvre)

- charge RSE reporteacutee sur des tiers (ex fournisseurs AF)

- inteacutegration de facteurs non visibles en comptabiliteacute (pertes drsquoemploi nuisances hellip)

Conclusion

Voies incontournables mais pouvant nrsquoavoir qursquoun effet superficiel et temporaire Voir utiliteacute

drsquoaccompagnement leacutegislatif de regravegles de gouvernance

Initiation au management copy CRCF ndash J Sornet Page 12 48

Fiche IM1 - Deacutefinitions du management

Dictionnaire anglais - franccedilais direction administration gestion intrigue manegravege

Wikipeacutedia Le management est lensemble des techniques dorganisation qui sont mises en

oeuvre pour ladministration dune entiteacute

Au point de vue eacutetymologique le verbe manage vient de litalien maneggiare (controcircler)

influenceacute par le mot franccedilais manegravege (faire tourner un cheval dans un manegravege) A cette

notion il faut aussi ajouter la notion de meacutenage (geacuterer les affaires du meacutenage) qui consiste agrave

geacuterer des ressources humaines et des moyens financiers

helliphellip

Fiche IM2 - Etudier le management

Concreacutetiser

Manager neacutecessite de syntheacutetiser des informations parfois complexes incomplegravetes et de

domaines tregraves divers pour en deacuteduire des actions Une approche trop parcellaire peut

conduire agrave lrsquoeacutechec et le savoir-faire est neacutecessaire pour agir vite avec un minimum de risque

Lrsquoeacutetudiant doit se preacuteparer simultaneacutement aux examens et agrave la pratique Il nrsquoa souvent connu

lrsquoentreprise que durant quelques semaines de stage et le manque de laquo recul raquo ne lui permet

pas toujours de concreacutetiser les theacuteories Il doit compenser par la lecture (ouvrages revues

journaux eacuteconomiques et boursiers) et en eacutetant attentif aux informations ambiantes (tout en

relativisant le style journalistique) en mettant en relation le cours les concepts les modegraveles

lrsquoactualiteacute les stages

Savoir traiter un exercice

Pour reacuteussir un examen ou traiter une application peacutedagogique (la conception les points 1 agrave

6 peut repreacutesenter le tiers du temps de travail)

1 ndash Identifier le type de sujet (faut-il trouver une solution pratique ou communiquer une

reacuteflexion geacuteneacuterale )

2 ndash Lire le sujet et relever les mots cleacutes

3 ndash Deacutefinir les mots cleacutes

4 ndash Reacutesumer la probleacutematique du sujet (en quelques lignes)

5 ndash Lister les connaissances reacutefeacuterences et raisonnements reacutepondant au problegraveme (par

recherche spontaneacutee ou raisonneacutee qui quoi ougrave quand comment combien hellip

listage des diffeacuterents points de vue) trouver des exemples (notamment dans les

documents fournis)

6 ndash Organiser la reacuteponse (deacutefinir le plan du deacuteveloppement ougrave des paragraphes bien

identifieacutes sont geacuteneacuteralement neacutecessaires en y liant les parties qui doivent ecirctre en nombre

limiteacute ndash de deux agrave quatre) Preacutevoir drsquoy inteacutegrer la deacutefinition des principales notions

induites par le sujet

7 ndash Reacutediger sous la forme adapteacutee (note technique ou recommandation solution

pratique exposeacute structureacute dissertation)

Introduction et conclusion sont indispensables agrave la dissertation ou agrave lrsquoexposeacute

- lrsquointroduction preacutesente le sujet traiteacute (phrase drsquoaccroche initiale) amorce la

probleacutematique (quelques sous - questions) et annonce le plan

- la conclusion syntheacutetise le deacuteveloppement (arguments) eacutelargit le sujet (prise de recul)

et apporte le point final (une phrase)

Une limite agrave la communication

Il est difficile de faire passer plus de 4 ou 5 ideacutees fortes dans un exposeacute unique

Initiation au management copy CRCF ndash J Sornet Page 13 48

Fiche IM3 - Bref historique

Antiquiteacute

3000 AJC

Peacuteriode greacuteco-

romaine

Transition

feacuteodale

12egraveme siegravecle

europe

15egraveme ndash 17egraveme

siegravecles

19egraveme siegravecle

20egraveme siegravecle

agriculture preacutedominante industrie limiteacutee aux besoins drsquoun individu ou drsquoun clan

pour la confection des outils des vecirctements et de la poterie Force motrice animale

ou humaine pour lrsquoessentiel

Grands travaux drsquoeacutetat en Egypte premiegravere laquo planification ndash organisation ndash controcircle raquo

Deacuteveloppement des communications essor industriel limiteacute peu de progregraves

technique (lrsquoesclavage supplante les innovations)

Deacuteveloppement progressif des eacutechanges commerciaux

La consommation indirecte atteint un bon niveau (surplus agricoles et

deacuteveloppement des villes) Apparition de nouveaux commerccedilants

Etat fort Evolutions technologiques (imprimerie bateaux performants instruments de

navigation) Extension geacuteographique de lrsquoeacuteconomie Apparition des corporations

drsquoartisans

Machine agrave vapeur chemin de fer passage de lrsquoartisanat au capitalisme

entrepreneurial producteur organisation des entreprises

Ecole classique (Taylor Fayol Weber) approche meacutecaniste bureaucratie

hieacuterarchie commandement fonctions et speacutecialisation laquo OS T raquo (organisation

scientifique du travail) organisation source de pouvoir rationaliteacute des individus bases

du management

Deacuteveloppement du capitalisme manageacuterial Electriciteacute peacutetrole puis communications

et information Consommation de masse mondialisation preacuteoccupations

eacutenergeacutetiques et environnementales 3 peacuteriodes

- standardisation grandes entreprises industrielles

- industries de consommation 30 glorieuses marketing multinationales protection

sociale

- deacutereacuteglementation monteacutee des services pays eacutemergents mondialisation et nouvelle

eacuteconomie (internet)

Ecole des relations humaines prise en compte de lrsquoindividu des motivations styles

de direction

Ecole neacuteo-classique et post-classique deacutecentralisation coordonneacutee DPO

management participatif zeacutero deacutefaut flux tendus

Approche systeacutemique partition de lrsquoentreprise eacutetude des interactions feacutedeacuteration

vers lrsquoobjectif controcircle et ajustement

Theacuteories de la deacutecision rationaliteacute limiteacutee contribution reacutetribution coalitions

Ecole socio-technique recherche de compromis technologie organisation

enrichissement des tacircches autonomie des groupes

Approche sociologique effets sociaux du travail jeux de pouvoir dans lrsquoentreprise

reacutegulation sociale

Theacuteories de la contingence facteurs contingents adaptation agrave lrsquoenvironnement

configurations organisationnelles

Theacuteories de la firme controcircle manageacuterial droits de proprieacuteteacute relation drsquoagence

Theacuteories contractualistes firme nœud de contrats coucircts de transaction

opportunisme externalisation internalisation

Approche eacutevolutioniste eacutecologie des organisations modegravele eacutevolutioniste

contraintes de sentier

Approche par les ressources valorisation des ressources compeacutetences cleacutes

apprentissage organisationnel

(Classement simplifieacute)

Initiation au management copy CRCF ndash J Sornet Page 14 48

ELEMENTS DE CORRIGE IM

IM1 Commenter la deacutefinition du management par la norme ISO et le manager de Mintzberg

Efficient = optimum avec les moyens disponibles

ISO (management objectifs) (manager moyens) HM

IM2 Le leader entraicircne naturellement derriegravere lui Le manager nrsquoest pas toujours leader

(mecircme si crsquoest souhaitable) Le leader nrsquoest pas toujours manager (plutocirct notion individuelle)

Leadership = faculteacute de diriger conjugaison drsquoune autoriteacute naturelle ou drsquoun savoir-faire

acquis drsquoune capaciteacute agrave entraicircner des personnes ou des groupes et drsquoune leacutegitimiteacute

statutaire (de position)

IM3 Compleacuteter le tableau ci-dessous en analysant chaque action preacutesenteacutee Faire ensuite

ressortir les domaines niveaux ou techniques de management pouvant ecirctre mobiliseacutes pour

chaque situation

Satisfaction client

Implication du personnel

Processus systegraveme

Ameacutelioration continue

Deacutecision efficace

Recherche de valeur

Image entreprise

Liaisons

Information

Reacutepartition ressources

Reacutegulation

Neacutegociation

Leadership

Initiation au management copy CRCF ndash J Sornet Page 15 48

Caracteacuteristiques

de lrsquoaction

- reacutepeacutetition

- risque

- normes

- ampleur

Prise de

deacutecision

- opeacuteration

- direction

- deacutelai

Informations

neacutecessaires

- nature

- origine

- deacutelai

obtention

Cleacute pour la

reacuteussite

Intervention

exteacuterieure

possible

Assurer la

restauration du

soir

(Restaurant

familial)

Technique

(fabrication)

Vente (terrain)

Appros

Reacutepeacutetitive

(quot)

Risque faible

Normes

drsquohygiegravene

Faible

Opeacuterationnelle

Geacuterant

responsable

Rapide (qq

jours menu et

appros)

Nombre de

couverts

Tarifs usuels

Calendrier

(fecirctes)

Clients docs

divers

expeacuterience

Qq jours

Varieacuteteacute menu

Plats phares

Accueil

Appros

Tarification

Vins

Gestion

congeacutelation

Qualiteacute cuisine

Fournisseurs

Extra

Publiciteacute

Construire un

viaduc

(autoroute)

Technique

Organisation

Appros

Uniteacute (ou peu)

Eleveacute (financier

technique)

Architecture

Eleveacutee

Direction

(aleacuteas)

Opeacuterationnelle

(conduite

chantier)

Immeacutediat agrave qq

semaines

Plans

plannings

Qualifications

Meacuteteacuteo

Disponibiliteacutes

Bureau eacutetudes

Qq sem agrave 24h

Techniciteacute

Appros

Qualifications

Preacutevision

GRH

Contrat juste

SS traitants

Organismes

certificateurs

Controcircle

client

Certifier les

comptes

annuels drsquoun

groupe national

(cabinet

drsquoaudit)

Technique

Relation client

Gestion des

connaissances

Annuelle

Moyen

Regravegles

comptables

fiscales

Moyenne (selon

importance du

cabinet)

Opeacuterationnelle

Qq jours agrave

semaines

Comptable

Juridique

Client

Etat

Qq jours agrave

semaines

Techniciteacute

Expeacuterience

Relation client

Systegraveme info client

Siegravege

Autre cabinet

Lancer une

ligne drsquoavions

(constructeur

aeacuteronautique)

Strateacutegique

RD

Etudes

Uniteacute

Tregraves eacuteleveacute

Aeacuteronautique

Tregraves eacuteleveacutee

Direction

Qq mois agrave

anneacutees

Marcheacute

Etudes

Compagnies

Qq mois agrave

anneacutees

Concept

Outil industriel

Coucirct exploitation

Tarif

Fiabiliteacute

Deacutelaisconcurrence

SI simulation

SS traitants

Bureaux

drsquoeacutetudes

speacutecialiseacutes

Compagnies

Conseils

Reacuteduire la

capaciteacute de

production

(groupe

industriel)

Strateacutegique

RH

Communication

Production

Uniteacute

Moyen

Leacutegislation

(dont RH)

Eleveacutee

Direction

Qq mois agrave

anneacutees

Financiegravere

Industrielle

Marcheacute

Organisation

Organismes

speacutecialiseacutes

DRH

Qq mois

Communication

Connaissance des

compeacutetences

Connaissance outil

industriel

Concurrence

Portefeuille

drsquoactiviteacutes

Cabinet

drsquoorganisation

Conseils

speacutecifiques

Acqueacuterir une

entreprise

concurrente

(teacuteleacutephonie

mobile)

Strateacutegique

Marketing

Production

(reacuteseau)

Financier

Communication

Uniteacute

Tregraves eacuteleveacute

Leacutegislation

telecom

Tregraves eacuteleveacutee

Direction

Qq mois

Financiegravere

Marcheacute

Reacuteseaux

(ampleur

recouvrement

hellip)

Organisations

Interne

Racheteacutee

Sources

speacutecialiseacutees

Qq mois

Communication

Marcheacute

Cours boursiers

Cabinet

drsquoorganisation

Conseils

speacutecifiques

Initiation au management copy CRCF ndash J Sornet Page 16 48

LE MANAGEMENT EN PRATIQUE

Pour assumer sa fonction le management doit couvrir sans discontinuiteacute lrsquoensemble de

lrsquoorganisation et inteacutegrer de nombreux facteurs dont nous allons reacutesumer lrsquoessentiel

1 ndash Les fonctions et activiteacutes du management

Pour Henri Fayol la fonction drsquoadministration de lrsquoentreprise (son management) reposait sur

cinq actions preacutevoir organiser commander coordonner et controcircler (laquo PO3C raquo)

Nous distinguerons cinq activiteacutes de management

- la conception (au plus haut niveau finaliteacute but ou vocation de lrsquoorganisation

meacutetiers dimension politique de croissance hellip)

- la planification (deacutefinition des objectifs eacutecheacuteances)

- lrsquoorganisation (reacutepartition du travail choix des modes de coordination)

- le pilotage de lrsquoaction opeacuterationnelle (motivation animation encadrement

assistance)

- lrsquoeacutevaluation (controcircle des reacutesultats obtenus ajustements)

Dans chacune de ces activiteacutes des deacutecisions et des arbitrages sont neacutecessaires avec des

enjeux plus ou moins importants

Remarques

- Les cinq activiteacutes du management peuvent se retrouver agrave tout niveau de

management si lrsquoentreprise laisse une certaine autonomie de deacutecision agrave ses diffeacuterentes

uniteacutes La conception est naturellement du ressort de la direction geacuteneacuterale et des

conseils drsquoadministration mais elle peut ecirctre preacutesente pregraves du terrain (latitude laisseacutee agrave

une filiale ou agrave un magasin par exemple) De mecircme lrsquoorganisation du travail concerne

un atelier mais aussi la direction qui structure lrsquoentreprise pour assurer ses activiteacutes sa

production

- La planification deacutefinit des objectifs ou des axes strateacutegiques (choix de produits

modaliteacutes de deacuteveloppement des ventes implantations alliances hellip) et les traduit en

donneacutees de gestion preacutevisionnelles syntheacutetiques et eacutechelonneacutees dans le temps afin de

valider les objectifs et de fixer des repegraveres

- Un laquo business plan raquo (plan drsquoaffaires)est notamment lrsquoeacutequivalent de la planification

dans le cas de creacuteation drsquoentreprise ou pour la preacutesentation de tout projet drsquoactiviteacute

Les activiteacutes du management srsquoinscrivent dans des cycles qui peuvent ecirctre scheacutematiseacute

comme suit (lrsquoeacutevaluation peut entraicircner une reacutevision du pilotage de lrsquoorganisation ou des

objectifs sans que lrsquoentreprise ne soit fondamentalement remise en cause)

conception

planification

organisation

pilotage

eacutevaluation

Initiation au management copy CRCF ndash J Sornet Page 17 48

2 ndash Les contextes de management

Le management est influenceacute par son contexte qui justifie des objectifs une organisation

des meacutethodes

Par exemple lrsquoentreprise admet de nombreuses variantes selon sa taille sa forme juridique

son controcircle par lrsquoeacutetat (entreprises publiques) ou par des inteacuterecircts priveacutes Il en va de mecircme des

organismes administratifs qui peuvent deacutependre de directives nationales ou reacutegionales des

associations qui ont des activiteacutes drsquoampleur tregraves variable

21 ndash La dimension de lrsquoentreprise

La dimension drsquoune entreprise se mesure principalement en fonction de son effectif ou de

son chiffre drsquoaffaires Des seuils sont deacutefinis par divers organismes et exploiteacutes agrave des fins

statistiques ou pour la deacutetermination de certaines obligations sociales ou fiscales

(repreacutesentation du personnel cotisations hellip) Il nrsquoy a bien entendu pas de laquo barriegravere de

tailleraquo absolue conditionnant le management drsquoune entreprise

LrsquoUE preacuteconise de distinguer les micro ndash entreprises (jusqursquoagrave 9 salarieacutes) les TPE ndash tregraves petites

entreprises (moins de 20 salarieacutes) les petites entreprises (moins de 50) et les moyennes

entreprises (de 50 agrave 250) Cependant les PME sont parfois situeacutees entre 10 et 500 salarieacutes

Remarques

- en France environ 40 des entreprises emploient de 1 agrave 50 salarieacutes (ce qui repreacutesente

plus de 50 des emplois) et 59 nrsquoen ont aucun

le pays compte environ 2 600 000 entreprises dont moins de 1 ont 250 employeacutes et

plus

- ancienneteacute et taille de lrsquoentreprise sont lieacutees si lrsquoon eacutecarte les restructurations et autres

eacutevolutions drsquoentreprises existantes

La dimension de lrsquoentreprise a une influence sur lrsquoorganisation et le laquo style raquo de son

management

- les PME sont souvent entrepreneuriales (les dirigeants eacutegalement apporteurs de capitaux

sont totalement engageacutes dans la marche de lrsquoentreprise) Elles ont une gestion flexible peu

formaliseacutee plus qualitative que quantitative Les PME sont freacutequemment focaliseacutees sur un seul

type drsquoactiviteacute Pour ne pas alourdir leur structure elles ont tendance agrave sous-traiter les

activiteacutes speacutecialiseacutees ne correspondant pas agrave leur meacutetier de base

- les grandes entreprises sont manageacuteriales (les dirigeants sont nommeacutes par les actionnaires

en raison de leurs compeacutetences) et moins reacuteactives

22 ndash Le type de production

On distingue industrie (production de biens mateacuteriels ou pour le moins de produits visibles ndash

comme un seacutejour touristique ou un film) et services (fourniture drsquoune prestation immateacuterielle)

Le type de production influence en principe le management de lrsquoentreprise

- lrsquoindustrie neacutecessite (si lrsquoon excepte lrsquoartisanat) un investissement relativement important

une organisation productive stable capable de reacutealiser plusieurs fois des produits identiques

(exemple un modegravele de reacutefrigeacuterateur) ou du moins similaires (exemple un bacirctiment) Le

produit de lrsquoindustrie consomme des matiegraveres et il doit geacuteneacuteralement ecirctre distribueacute jusqursquoau

client

- la production de services peut se satisfaire drsquoun investissement tregraves reacuteduit et neacutecessite un

contact permanent avec le client

Toutefois la standardisation des services et le deacuteveloppement des reacuteseaux informatiques

rapprochent la production de services de celle des biens industriels

- la production drsquoun service reacutepeacutetitif et technique peut imposer une structure lourde et une

organisation tregraves formaliseacutee (voir les grandes socieacuteteacutes drsquoaudit ou de conseil informatique)

Initiation au management copy CRCF ndash J Sornet Page 18 48

- certains services peuvent ecirctre fournis agrave distance sans contact direct avec le client et

distribueacutes par reacuteseau (tenue de comptabiliteacute affacturage gestion clientegravele centre drsquoappel

hellip)

Remarque les services repreacutesentent 75 de lrsquoactiviteacute eacuteconomique franccedilaise

23 ndash La nature de lrsquoorganisation

Les organisations publiques franccedilaises (administrations centrales collectiviteacutes territoriales

hocircpitaux hellip) repreacutesentent une part importante de lrsquoactiviteacute (environ 30 des emplois) La

fonction publique regroupe des organisations aux finaliteacutes diverses et qui ont des problegravemes

de gestion similaires agrave ceux des entreprises auxquelles elles peuvent emprunter des principes

de management Notamment

- pour controcircler les coucircts et assurer la qualiteacute des services

- pour communiquer avec les administreacutes ou les usagers

- pour motiver les personnels et geacuterer les ressources humaines

La transposition directe des techniques de gestion et de management nrsquoest cependant pas

toujours possible car

- la comptabiliteacute publique obeacuteit agrave des regravegles speacutecifiques (proceacutedure budgeacutetaire

notamment)

- le laquo client raquo ne paye pas toujours la prestation du moins directement

- la concurrence est parfois inexistante

- les grandes administrations centraliseacutees sont soumises agrave des choix politiques geacuteneacuteraux

parfois sans connexion eacutevidente avec les besoins opeacuterationnels

- le statut des personnels et les grilles de salaires limitent les possibiliteacutes de gestion des

ressources humaines

Remarque la LOLF (loi organique relative aux lois de finances) est entreacutee en vigueur en

2006 Elle alloue des moyens budgeacutetaires en fonction de programmes et remplace la

reconduction automatique de 90 des budgets Cette reacuteforme se heurte toutefois agrave la

lourdeur des grands ministegraveres ougrave la complexiteacute des activiteacutes est difficile agrave

appreacutehender et ougrave des inerties culturelles peuvent exister agrave tout niveau

Les associations loi de 1901 peuvent avoir une activiteacute comparable agrave celle de grandes

entreprises (voir par exemple les associations de santeacute ou professionnelles) et leur

management est alors similaire malgreacute lrsquoabsence de but lucratif (les beacuteneacutefices ne sont pas

distribuables) Elles ont drsquoailleurs en France un poids eacuteconomique important (elles emploient

environ 1 600 000 salarieacutes)

Cependant lrsquoadheacutesion agrave un systegraveme de valeurs fondateur de lrsquoassociation ou la limite de

lrsquoautoriteacute (quand un volant de beacuteneacutevoles important participe agrave lrsquoactiviteacute) peut introduire des

nuances

- le renforcement des objectifs socieacutetaux

- la faiblesse des relations hieacuterarchiques

- des contraintes de gestion du temps des beacuteneacutevoles

- des modaliteacutes particuliegraveres de recrutement et de motivation des dirigeants

24 ndash Les facteurs contingents

La theacuteorie de la contingence montre qursquoune structure drsquoentreprise nrsquoest efficace que dans

une situation deacutetermineacutee et qursquoil nrsquoexiste que des solutions de management construites dans

un contexte preacutecis

Le management doit ainsi srsquoadapter agrave des facteurs contingents qui ne peuvent ecirctre

controcircleacutes du moins agrave bregraveve eacutecheacuteance Ces facteurs sont par exemple

- lrsquoancienneteacute de lrsquoentreprise (plus elle est ancienne plus lrsquoentreprise a tendance agrave reacutepeacuteter

des comportements eacuteprouveacutes)

Initiation au management copy CRCF ndash J Sornet Page 19 48

- la taille de lrsquoentreprise (la grande entreprise a une composante administrative plus

deacuteveloppeacutee)

- le systegraveme de production (tregraves standardiseacute complexe automatiseacute hellip)

- lrsquoenvironnement

3 ndash Le management et les parties prenantes

Lrsquoentreprise a pour vocation premiegravere de mettre des produits agrave disposition de ses clients en

reacutealisant un profit Pour y arriver elle doit aussi satisfaire ses parties prenantes salarieacutes

actionnaires fournisseurs hellip

Est partie prenante agrave lrsquoentreprise laquo tout groupe ou individu qui peut ecirctre affecteacute ou est

affecteacute par les buts de lrsquoorganisation hellip raquo (Freeman ndash 1984)

Les parties prenantes attendent agrave des degreacutes divers de profiter drsquoune creacuteation de valeur en

provenance de lrsquoentreprise qui doit reacutepondre agrave ces attentes pour assurer sa peacuterenniteacute ou

favoriser son deacuteveloppement

On distingue les parties prenantes primaires ou principales qui sont essentielles agrave lrsquoentreprise

et qui ont geacuteneacuteralement une relation formelle avec elle (clients associeacutes et actionnaires

precircteurs salarieacutes fournisseurs collectiviteacutes) et les parties prenantes secondaires dont

lrsquoinfluence est diffuse (groupes de pression associations meacutedias instances europeacuteennes

agences de notation hellip)

Remarque la consideacuteration de lrsquoensemble des parties prenantes (laquo stakeholders raquo - les

deacutepositaires) fait contrepoids agrave lrsquoimportance accordeacutee aux seuls actionnaires

(laquo shareholders raquo)

Les organisations nrsquoayant pas drsquoobjectif de profit doivent aussi satisfaire leurs parties

prenantes apporter un service aux usagers dans les meilleures conditions eacuteconomiques

limiter un budget assurer la qualiteacute des relations avec les fournisseurs hellip

Dans cette optique le management doit organiser lrsquoaction de faccedilon agrave eacutequilibrer des forces

parfois divergentes

- le contexte fait pression sur lrsquoorganisation contrainte agrave optimiser ses reacutesultats

- lrsquoorganisation cherche par son action agrave assurer sa peacuterenniteacute son deacuteveloppement (en

reacutealisant des profits dans le cas de lrsquoentreprise) et agrave satisfaire ses parties prenantes

- le management agit en pilotant les actions pour contrebalancer la pression du contexte

Actions de

lrsquoorganisation

Management Contexte

Parties

prenantes

Initiation au management copy CRCF ndash J Sornet Page 20 48

APPLICATIONS MP

MP1 Deacutefinir contingent gestion budgeacutetaire

MP2 Deacuteterminer les parties prenantes drsquoun hocircpital public et leurs principales attentes

Mecircme question pour les organisations suivantes

- SNCF (entreprise publique)

- Peugeot

- MAIF (mutuelle drsquoassurance)

MP3 En les situant dans le cycle des activiteacutes du management trouver les actions agrave mener

dans les situations suivantes

- baisse de 10 des ventes dans une entreprise industrielle (produits meacutenagers le reacuteseau de

distribution vient drsquoecirctre reacuteorganiseacute)

- idem dans une entreprise de vente par correspondance soumise agrave la concurrence internet

(les ventes stagnaient depuis six mois malgreacute les efforts promotionnels)

- augmentation des deacutelais drsquoattente des consultations dans une clinique (lrsquohocircpital voisin a

fermeacute son service drsquourgences)

Initiation au management copy CRCF ndash J Sornet Page 21 48

ELEMENTS DE CORRIGE MP

MP1 Deacutefinir (dans le contexte drsquoune entreprise) contingent gestion budgeacutetaire

Contingent = imposeacute par lrsquoexteacuterieur Contingence = effet du hasard de la rencontre de

plusieurs eacuteveacutenements indeacutependants (variables explicatives que lrsquoon ne peut influencer)

Gestion budgeacutetaire = technique drsquoadministration des entreprises srsquoappuyant sur des

preacutevisions dont on deacuteduit apregraves accord des responsables des attributions de moyens sur une

dureacutee limiteacutee Une analyse reacuteguliegravere des eacutecarts entre preacutevisions et reacutealisations permet ensuite

le pilotage des activiteacutes Le budget est un cadre incitatif

La laquo planification budgeacutetaire raquo consiste agrave traduire en budgets une planification strateacutegique

avec systegraveme de reporting

MP2 Deacuteterminer les parties prenantes drsquoun hocircpital public et leurs principales attentes

Mecircme question pour les organisations suivantes

- SNCF (entreprise publique)

- Peugeot

- MAIF (mutuelle drsquoassurance)

Hocircpital

- patients (qualiteacute des soins)

- CNAM (baisse des coucircts)

- collectiviteacute locale (service aux administreacutes)

- eacutetat (ameacutenagement du territoire maicirctrise des budgets optimisation)

- employeacutes (salaire conditions de travail et satisfaction)

- fournisseurs ndash pharmacie autres (CA paiement reacutegulier)

- associations de patients (qualiteacute proximiteacute des soins)

SNCF

- usagers et associations drsquousagers (proximiteacute reacutegulariteacute prix du service)

- reacuteseau ferreacute de France (optimisation des lignes paiement adapteacute)

- fournisseurs (CA paiement reacutegulier)

- employeacutes (salaire conditions de travail seacutecuriteacute de lrsquoemploi)

- eacutetat (ameacutenagement du territoire)

- collectiviteacutes locales (service)

Peugeot

- clients (qualiteacute prix SAV relation commerciale)

- fournisseurs (CA reacutegulariteacute de lrsquoactiviteacute)

- employeacutes (salaire conditions de travail seacutecuriteacute de lrsquoemploi)

- eacutetat (taxes)

- collectiviteacute locale (emploi dynamisation eacuteconomique preacuteservation de lrsquoenvironnement)

- associations de protection de lrsquoenvironnement (activiteacute propre baisse des eacutemissions

nouvelles eacutenergies)

MAIF

- socieacutetaires (protection relation assureur tarif mesureacute)

- professionnels de lrsquoautomobile et autres (agreacutement marge de manœuvre reacuteparations tarifs

eacuteleveacutes)

- fournisseurs (CA paiement reacutegulier)

- eacutetat (taxes engagement pour la seacutecuriteacute)

- employeacutes (salaire conditions de travail seacutecuriteacute de lrsquoemploi)

Initiation au management copy CRCF ndash J Sornet Page 22 48

MP3 En les situant dans le cycle des activiteacutes du management trouver les actions agrave mener

dans les situations suivantes

- baisse de 10 des ventes dans une entreprise industrielle (produits meacutenagers le reacuteseau de

distribution vient drsquoecirctre reacuteorganiseacute)

Adapter le pilotage motiver cadrer si insuffisant retoucher une organisation deacutefectueuse

- idem dans une entreprise de vente par correspondance soumise agrave la concurrence internet

(les ventes stagnaient depuis six mois malgreacute les efforts promotionnels)

Voir pilotage et organisation si une eacutevolution du meacutetier a deacutejagrave eacuteteacute initialiseacutee Sinon re-

conception (adaptation au nouveau contexte) puis planification et reacuteorganisation

- augmentation des deacutelais drsquoattente des consultations dans une clinique (lrsquohocircpital voisin a

fermeacute son service drsquourgences)

Organisation Si insuffisant planification (nouveaux objectifs)

Initiation au management copy CRCF ndash J Sornet Page 23 48

ORGANISATION ET PROCESSUS

La performance de lrsquoentreprise deacutepend de son organisation et de son aptitude agrave produire

aux meilleures conditions Nous allons montrer comment organisation formelle et processus

de production peuvent contribuer agrave cette performance

1 ndash Vers lrsquooptimum

11 ndash Les eacuteconomies occidentales jusqursquoaux anneacutees 70

Jusqursquoen 1945 le principal problegraveme des entreprises eacutetait de produire des biens en quantiteacute

suffisante agrave un prix compatible avec le marcheacute Les grandes entreprises se sont multiplieacutees et

la standardisation a permis de reacuteduire les coucircts (exemple deacuteveloppement de Ford et de la

production agrave la chaicircne de 1908 agrave 1920 qui a permis une baisse du prix des voitures des 23)

On parle de laquo production pousseacutee vers le marcheacute raquo

Cette croissance de la production peu reacuteguleacutee a eacuteteacute marqueacutee par des surproductions en

1910 et 1920 puis par la crise de 1929 qui a prolongeacute ses effets jusqursquoagrave la guerre

De 1945 agrave 1975 environ (les laquo trente glorieuses raquo) la reconstruction la croissance de la

consommation de masse de nouvelles technologies et les eacutechanges internationaux

alimentent lrsquoeacuteconomie La standardisation srsquoeacutetend aux biens de consommation dont les

coucircts baissent fortement et de nouvelles reacutegulations sociales permettent une eacutevolution sans

heurt des revenus La saturation de certains marcheacutes conduit dans les anneacutees 60 agrave la

deacutemarche laquo marketing raquo et agrave la diffeacuterenciation des produits Le produit est laquo dirigeacute par le

marcheacute raquo mais les entreprises conservent une organisation assez classique et les plus grosses

srsquointernationalisent

12 ndash Lrsquoexpeacuterience japonaise et ses prolongements

Tregraves tocirct apregraves la guerre dans un Japon appauvri le constructeur automobile Toyota a ducirc

faire face agrave une restriction du marcheacute des moyens financiers et productifs et des

approvisionnements La firme a donc innoveacute dans un nouveau systegraveme de production

chassant les laquo gaspillages raquo (temps drsquoattente transports stocks deacutefauts hellip) consideacuterant que

seule la fabrication vendable creacutee de la valeur

Toyota srsquoorganise pour fabriquer la quantiteacute et la qualiteacute de produits juste neacutecessaires agrave la

satisfaction des clients la production est laquo tireacutee par le marcheacute raquo La mise en place de ce

systegraveme qui integravegre les fournisseurs ne sera acheveacutee que dans le milieu des anneacutees 70

En 1973 la hausse du peacutetrole inaugure un ralentissement de la croissance des eacuteconomies

occidentales La concurrence accrue provoque alors un inteacuterecirct pour le systegraveme deacuteveloppeacute

au Japon La production au plus juste se deacuteveloppe ainsi dans lrsquoindustrie automobile agrave partir

des anneacutees 80 et elle se reacutepand encore maintenant dans drsquoautres secteurs

Cette approche qui vise un objectif de zeacutero stock et zeacutero deacutefaut impose la maicirctrise de laquo bout

en bout raquo des processus de production et leur ameacutelioration

Initiation au management copy CRCF ndash J Sornet Page 24 48

2 ndash Organiser lrsquoentreprise

21 ndash Direction et organisation

Diriger une entreprise neacutecessite de lrsquoorganiser (de reacutepartir les tacircches) pour qursquoelle puisse

atteindre ses objectifs Lrsquoorganisation permet de satisfaire un marcheacute en tirant parti des

capaciteacutes actuelles de lrsquoentreprise tout en preacuteparant lrsquoavenir

Lrsquoorganisation reacutesulte freacutequemment drsquoun compromis entre des objectifs situeacutes agrave des niveaux

et des eacutecheacuteances diffeacuterents

Exemples

- le leader des chaises roulantes peut tirer profit de sa structure productive et de son

savoir faire pour entrer sur le marcheacute de la bicyclette eacutelectrique

- ecirctre parfaitement structureacute pour alimenter 90 du marcheacute des disquettes ne preacutepare

pas lrsquoavenir

- srsquoorganiser pour conqueacuterir le marcheacute des tire-bouchons eacutelectriques dans les deux ans

perd de son sens si cela altegravere les moyens neacutecessaires agrave la production drsquoappareils

manuels ancienne mais vitale dont la diminution agrave court terme risque de nuire agrave la

solvabiliteacute de lrsquoentreprise et de la conduire agrave la cessation de paiement

22 ndash Lrsquoorganisation fonctionnelle

La majoriteacute des entreprises adopte une laquo organisation fonctionnelle raquo (celle qui est visible

dans les organigrammes) ougrave des regroupements de personnels et drsquoeacutequipements se font

selon un modegravele hieacuterarchique (laquo line raquo) dans des uniteacutes des services ou des deacutepartements

speacutecialiseacutes Cette organisation peut se deacutecliner agrave lrsquointeacuterieur des divisions des grandes

entreprises quand elles scindent leur activiteacute par zone geacuteographique type drsquoactiviteacute

cateacutegorie de clients hellip

Remarque le terme laquo fonction raquo deacutesigne un rocircle particulier dans le fonctionnement de

lrsquoentreprise

Lrsquoorganisation fonctionnelle diffeacuterencie les activiteacutes de lrsquoentreprise en les regroupant par

meacutetier pour utiliser au mieux les compeacutetences et les moyens (meilleur rendement par la

speacutecialisation lrsquoeacutechange de compeacutetences dans une mecircme uniteacute ou gracircce agrave des eacuteconomies

drsquoeacutechelle)

23 ndash La notion de processus de production

Un processus de production se deacutefinit par la succession drsquoactiviteacutes permettant de satisfaire

un client en transformant des ressources (mateacuterielles financiegraveres humaines) en un produit

bien ou service Le processus doit creacuteer une valeur reconnue par le client

Un processus peut servir un client interne agrave lrsquoentreprise (par exemple en produisant un

composant intervenant dans plusieurs produits ou par la maintenance des machines) aussi

bien qursquoun client final On distingue usuellement

- les processus opeacuterationnels (ou maicirctres) aussi appeleacutes processus meacutetier (business process)

qui satisfont directement les clients finaux (conception et fabrication de produits vente hellip)

- les processus de support et de management (geacuterer les ressources humaines geacuterer

lrsquoinformation geacuterer les ressources financiegraveres hellip) qui ont les processus opeacuterationnels comme

clients

Toutes les actions internes agrave une organisation peuvent srsquointeacutegrer dans des processus qui

conditionnent directement ou indirectement la capaciteacute de lrsquoorganisation agrave satisfaire le

client final ou lrsquousager

Initiation au management copy CRCF ndash J Sornet Page 25 48

Aborder le fonctionnement de lrsquoentreprise par ses processus (approche processus) permet

de mettre en eacutevidence les chaicircnes drsquoactiviteacutes qui conduisent aux produits leurs

dysfonctionnements leurs coucircts la formation des deacutelais et la souplesse (la flexibiliteacute)

disponible pour satisfaire la clientegravele finale Lrsquoameacutelioration des processus a un impact visible

et direct sur chaque produit proposeacute aux clients

Lrsquoapproche processus provoque une eacutevolution de la faccedilon de travailler

- en faisant peacuteneacutetrer la laquo voix du client raquo au plus profond de lrsquoentreprise (et plus seulement

dans les services commerciaux et marketing)

- en mettant en eacutevidence des possibiliteacutes de rationalisation (par regroupement ou impartition

de certaines activiteacutes)

Remarque lrsquoapproche par les activiteacutes et les processus est agrave lrsquoorigine de la meacutethode

de deacutetermination des coucircts laquo ABC raquo - activity based costing

24 ndash Processus et fonctions

Le processus est transversal Il enchaicircne des activiteacutes qui traversent lrsquoentreprise en particulier

les services ou les deacutepartements drsquoune organisation fonctionnelle

Exemple

La division du travail par fonctions induit une charge de coordination pour assurer le

deacuteroulement du processus Elle peut geacuteneacuterer des attentes des erreurs ou des conflits drsquointeacuterecirct

(lrsquoobservation montre que des dysfonctionnements sont tregraves souvent constateacutes lors du

passage drsquoun service agrave un autre)

Organisation fonctionnelle et approche processus visent toutes deux un optimum

eacuteconomique mais leurs logiques sont diffeacuterentes

- le processus vise la satisfaction des clients (prix qualiteacute deacutelais service)

- le deacutecoupage fonctionnel cherche agrave optimiser les moyens (maximiser lrsquoeffet drsquoexpeacuterience

partager des infrastructures profiter de pocircles de compeacutetences hellip) Il apporte une ossature

hieacuterarchique stable souvent indispensable

Organisation fonctionnelle et approche processus sont donc compleacutementaires dans la

majoriteacute des cas et doivent ecirctre combineacutees judicieusement

APPLICATIONS OP

OP1 Deacutefinir flexibiliteacute systegraveme impartition

OP2 Citer huit exemples drsquoinformations essentielles pour optimiser un processus de

fabrication

Direction

Deacutepartement

commercial

(C)

Deacutepartement

administratif et

financier (AF)

Deacutepartement

Etudes (E)

Deacutepartement

Production (P)

Activiteacute

C-x Activiteacute

AF-x Activiteacute

E-x

Activiteacute

P-x

Processus x

Clie

nt

Initiation au management copy CRCF ndash J Sornet Page 26 48

OP3 Deacutegager les principes du toyotisme preacutesenteacute ci-dessous En quoi ce systegraveme est-il

initiateur de lrsquoapproche processus

Taiichi Ohno et le Toyotisme

1 - Extrait drsquoun article de Jacques BARRAUX - 1993 - LExpansion

Taiichi Ohno (1912 ndash 1990) hellip ne se prenait pas pour un visionnaire mais en imposant une

nouvelle faccedilon de produire il a reacuteinventeacute le management hellip tout le monde a entendu parler

des mots qui ont populariseacute le toyotisme dont il est le pegravere le juste-agrave-temps hellip Autant

doutils conccedilus pour lrsquoautomobile et qui ont aujourdhui une application universelle

hellip Taiichi Ohno jeune ingeacutenieur entre chez Toyota alors simple constructeur de machines

textiles Degraves 1926 apparaicirct la notion de jidoka hellip cest lart de transfeacuterer de lintelligence aux

machines pour mieux libeacuterer lintelligence des hommes Tout le contraire du taylorisme qui

juge la machine moins impreacutevisible que lhomme En 1933 Toyota se lance dans lautomobile

en sinspirant des meacutethodes ameacutericaines Mais en 1935 agrave loccasion dun voyage aux Etats-

Unis leacutetat-major de lentreprise revient fascineacute de sa visite dans un supermarcheacute La notion

de juste-agrave-temps va naicirctre de lobservation dune grande surface un lieu ougrave les clients ne

prennent que ce dont ils ont besoin et ougrave les rayons sont reacuteapprovisionneacutes pour compenser

les quantiteacutes preacuteleveacutees Ainsi le systegraveme Toyota est-il deacutejagrave dans la tecircte de ses dirigeants avant

mecircme la Seconde Guerre mondiale un demi-siegravecle avant la reacutevolution informatique et la

segmentation intensive des marcheacutes

hellip des esprits curieux comme Franccedilois Dalle en France tombent alors sous le charme des

formules et des paraboles de Taiichi Ohno En voici deux eacutechantillons

Penser agrave lenvers Cela signifie combattre les ideacutees reccedilues En lespegravece il sagit du fordisme et

du taylorisme Ohno ne croit pas agrave la planification aux effets deacutechelle et dexpeacuterience Il

propose un systegraveme industriel agrave lenvers qui permette de diversifier les produits et de les

fabriquer en petites quantiteacutes Nous ne devons plus ecirctre des paysans qui accumulent des

stocks mais des chasseurs On nimpose pas loffre On traque la demande et on la gegravere en

continu

Que les valleacutees soient hautes et les montagnes peu eacuteleveacutees Plutocirct que de concentrer tous

les efforts sur une production agrave un moment donneacute mieux vaut se doter de structures flexibles

permettant de passer agrave tout instant dune seacuterie agrave une autre Il faut eacuteviter les ruptures et les

secousses aplanir les cycles entretenir des flux reacuteguliers dactiviteacutes diversifieacutees Ce qui

implique de ne pas enfermer les hommes et les eacutequipements dans des speacutecialisations trop

eacutetroites

La flexibiliteacute le travail en groupe le refus de la dictature des machines la polyvalence et

surtout lattention constante aux signaux eacutemis par le marcheacute nappartiennent plus au

toyotisme Ces notions sont les fondements du nouvel art dorganiser de vendre et de

produire dans lindustrie comme dans les services hellip

2 - Quelques notions cleacutes

Taiichi Ohno a imagineacute la meacutethode des laquo cinq pourquoi raquo qui consiste agrave se poser cinq fois de

suite la question laquo pourquoi raquo sur le mecircme sujet de faccedilon agrave deacutecouvrir la veacuteritable cause

drsquoun problegraveme Cette meacutethode peut ecirctre appliqueacutee agrave tous les niveaux et permettre

notamment aux agents de fabrication de proposer de veacuteritables ameacuteliorations de la

production

La recherche de la qualiteacute totale (pas de deacutefaut des produits pas de rebuts pas de deacutefaut

des processus) accompagne la deacutemarche de Toyota La qualiteacute a un coucirct compenseacute par

des ventes accrues par lrsquoeacuteconomie des mesures palliatives aux deacutefauts

Initiation au management copy CRCF ndash J Sornet Page 27 48

Fiche OP1 ndash Benchmarking et processus

Le laquo benchmarking raquo consiste agrave comparer le fonctionnement de plusieurs systegravemes pour en

faire notamment ressortir les meilleures pratiques (laquo best practices raquo) Cette technique est

utiliseacutee depuis les anneacutees 80 pour ameacuteliorer la performance des entreprises Elle impose agrave

lrsquoentreprise drsquoeacutevaluer et de remettre en question ses propres modes de fonctionnement afin

de les faire eacutevoluer agrave la lueur de ce qui se fait ailleurs

Le benchmarking permet drsquoameacuteliorer les processus agrave moindre risque en fixant des objectifs

baseacutes sur des faits et donc plus facilement accepteacutes

Une classification des processus en tant que base de reacuteflexion a eacuteteacute eacutetablie aux USA par

lrsquolaquo International Benchmarking Clearinghouse raquo de lrsquoAPQC (american productivity and

quality center) en collaboration avec plusieurs dizaines drsquoentreprises

Elle se reacutesume ainsi

Le terme laquo reengineering raquo (la re-conception ou laquo reacuteingeacutenieacuterie raquo) des processus deacutesigne un

projet drsquoameacutelioration radicale des performances (de 20 agrave 50 ou plus) Il neacutecessite une

parfaite adheacutesion de la direction la constitution drsquoune petite eacutequipe de projet brillante

connaissant parfaitement les activiteacutes de lrsquoentreprise et il peut inclure un benchmarking

Le reengineering provoque geacuteneacuteralement la reacuteduction du nombre de niveaux hieacuterarchiques

(laquo delayering raquo) et lrsquoaccroissement du pouvoir de deacutecision des employeacutes (laquo empowerment raquo

ou laquo empouvoirement raquo) Bien qursquoy conduisant parfois il ne doit pas ecirctre confondu avec la

reacuteduction des activiteacutes (laquo downsizing raquo ou restructuration) et lrsquoexternalisation (laquo outsourcing raquo)

Pro

ce

ssu

s

op

eacutera

tio

nn

els

Pro

ce

ssu

s d

e m

an

ag

em

en

t e

t d

e

sup

po

rt

1 ndash

Comprendre

le marcheacute et

les clients (besoins

satisfaction)

2 ndash

Deacutevelopper

vision et

strateacutegie (contexte

concurrence)

3 ndash

Creacuteer

produits

services

processus

(concevoir

ameacuteliorer)

4 ndash

Marketing et

vente

5 ndash

Produire et

livrer (industrie

dont

ameacutelioration

processus)

6 ndash

Produire et

livrer (services)

7 ndash

Facturer et

servir les

clients (apregraves-

vente

reacuteclamations)

8 ndash Deacutevelopper et geacuterer les ressources humaines

9 ndash Geacuterer les systegravemes drsquoinformation

10 ndash Geacuterer les ressources financiegraveres et les actifs

11 ndash Appliquer un programme environnemental

12 ndash Geacuterer les relations exteacuterieures (actionnaires banques lois relations publiques hellip)

13 ndash Geacuterer lrsquoameacutelioration et le changement (eacutevaluer mesurer motiver qualiteacute totale)

Initiation au management copy CRCF ndash J Sornet Page 28 48

Fiche OP2 ndash Lrsquoorganisation par processus

Lrsquoeacutevolution drsquoune organisation aux activiteacutes reacutepeacutetitives vers lrsquoapproche processus est

geacuteneacuteralement progressive et se met en place par paliers

La mise en œuvre drsquoun veacuteritable management par processus doit ecirctre preacuteceacutedeacutee quand

lrsquoactiviteacute de lrsquoentreprise est complexe drsquoun recensement (une laquo cartographie des

processus raquo) pour mettre en eacutevidence les processus ou les familles de processus cleacutes critiques

pour le succegraves de lrsquoentreprise ougrave les efforts seront prioritaires

Des responsables de processus (laquo process owners raquo) sont ensuite deacutesigneacutes

Le responsable doit concevoir ses processus puis apregraves leur mise en œuvre assurer les

coordinations neacutecessaires les ameacuteliorer et les repreacutesenter aupregraves de la direction

Quand une structure par processus est mise en place des opeacuterateurs exeacutecutants

preacuteceacutedemment regroupeacutes dans les fonctions peuvent ecirctre affecteacutes aux processus et

drsquoanciens responsables de fonctions peuvent devenir des experts au service des processus

Lrsquoorganisation par processus peut imposer un degreacute eacuteleveacute drsquointeacutegration des activiteacutes donc

une polyvalence accrue des personnels et une reacuteduction des niveaux hieacuterarchiques

Elle neacutecessite pour le moins des compeacutetences eacutelargies au niveau des responsables de

processus (organisation administration technique hellip) dont le nombre doit rester limiteacute

(quelques dizaines au plus)

Sauf dans de tregraves petites structures lrsquoorganisation par processus se plaque geacuteneacuteralement sur

une structure plus classique

Initiation au management copy CRCF ndash J Sornet Page 29 48

ELEMENTS DE CORRIGE OP

OP1 Deacutefinir

Flexibiliteacute = adaptation au besoin (horaire variable chaicircnes robotiseacutees)

Systegraveme = ensemble organiseacute dans un but boicircte noire (sanguin nerveux meacutetrique laquo D raquo)

Impartition = sous-traitance ou externalisation (seacuteparation) drsquoactiviteacutes faire appel agrave des

partenaires plutocirct que faire soi-mecircme

OP2 Citer huit exemples drsquoinformations essentielles pour orienter lrsquooptimisation drsquoun processus

Montant des stocks (approvisionnements et produits finis)

Temps drsquoattente

Taux drsquoactiviteacute des ateliers

Rebuts

Deacutelai de production

Taux de reacuteclamations clients (qualiteacute)

Temps passeacutes en retouches finales

Turn over

Nombre drsquoarrecircts maladie

Accidents du travail

Dureacutee des arrecircts machines

OP3 Deacutegager les principes du toyotisme preacutesenteacute dans la fiche 31 En quoi ce systegraveme

repose trsquoil sur lrsquoapproche processus

Produire la quantiteacute juste neacutecessaire (agrave la demande) donc eacuteviter les stocks

Flexibiliteacute intelligence des chaicircnes de production

Qualiteacute (eacuteviter le coucirct de la non-qualiteacute)

La notion de processus est implicite ainsi que la chaicircne de valeur client

Initiation au management copy CRCF ndash J Sornet Page 30 48

DEFIS ET TENDANCES DU MANAGEMENT

Les meacutethodes de management se deacuteveloppent pour affronter le contexte eacuteconomique

Ce chapitre preacutesente les deacutefis auxquels le management contemporain doit faire face

1 ndash Lrsquoeacutevolution eacuteconomique contemporaine

A mesure que lrsquoactiviteacute eacuteconomique mondiale srsquoaccroicirct que la technologie eacutevolue les

changements sont de plus en plus rapides Ils introduisent des situations ineacutedites auxquelles les

entreprises doivent srsquoadapter en cherchant de nouvelles solutions de management Les trois

derniegraveres deacutecennies ont eacuteteacute notamment marqueacutees par les pheacutenomegravenes suivants (que nous

listons sans tenir compte des liens pouvant exister entre eux)

Pheacutenomegravene Traduction Effets

Deacute reacuteglementation

globalisation

financiegravere

titrisation

Libre circulation des capitaux accegraves

facile des particuliers au marcheacute

boursier (directement ou par

lrsquointermeacutediaire des OPCVM et SICAV)

Monteacutee en puissance du financement

des entreprises sur le marcheacute boursier

Fonds de pension

(retraites) et fonds

souverains (eacutetats)

Poids boursier important drsquoinvestisseurs

institutionnels qui cherchent un haut

rendement financier (dividendes ou

valorisation boursiegravere)

Pression sur les grandes entreprises

influence sur les strateacutegies

Mondialisation Liberteacute des eacutechanges internationaux Accroissement de la concurrence

recherche drsquoavantages eacuteconomiques

par la deacutelocalisation (biens et

services) la concentration des efforts

(recentrage) problegravemes drsquoemploi

multiplication des transports perte

drsquoinfluence des politiques

Baisse de lrsquoemploi

occidental

(notamment

industriel)

Moins de fabrications fabrications

automatiseacutees recours aux moyens

informatiques

Activiteacute reporteacutee sur le commerce la

conception et les services chocircmage

charge sociale

Restructurations Optimisation des entreprises

abaissement des coucircts augmentation

des marges recherche drsquoune taille

critique (eacuteconomies drsquoeacutechelle poids

sur le marcheacute)

Recentrages externalisations fusions

deacutelocalisations constitution de grands

groupes

NTIC (nouvelles

technologies de

lrsquoinformation et de

la communication)

Mise en œuvre des reacuteseaux (dont

internet) et drsquoapplications

informatiques communicantes

Nouvelles formes de commerce

marcheacute international deacutelocalisation

du travail intellectuel reacuteorganisation

de la distribution

Rareacutefaction relative

des matiegraveres

premiegraveres

Recherche de substituts exploration

miniegravere coucircts drsquoexploitation des

gisements accrus

Augmentation des coucircts variations

erratiques du cours des matiegraveres

deacutestabilisations politiques

Evolution

geacuteopolitique et

eacuteconomique

mondiale

Chute de lrsquoURSS transformation des

eacuteconomies collectivistes pays

eacutemergents (Chine Inde Breacutesil Russie)

()

Accroissement de la population

mondiale (4 agrave 6 7 milliards de 1970 agrave

2008)

Libeacuteralisme sans frein () nouvelles

puissances eacuteconomiques

opportuniteacutes de deacuteveloppement

nouveau partage des ressources

ineacutegaliteacutes baisse du soutien aux PVD

laquo Terrorisme raquo Actions armeacutees pression de groupes

armeacutes non gouvernementaux

Deacutestabilisations reacutegionales charge

des deacutepenses militaires

Deacuteveloppement

durable

Recherche drsquoune croissance eacutequitable

et respectueuse de lrsquoenvironnement

Pression sur les entreprises (eacutetats

associations de consommateurs

eacutecologistes ONG)

() Reacutecemment quelques affaires (Enron laquo subprimes raquo Vivendi Universal Socieacuteteacute

Geacuteneacuterale Airbus par exemple) et agrave plus grande eacutechelle la crise financiegravere de 2008 ont

montreacute les dangers drsquoune libeacuteralisation sans controcircles suffisants

Initiation au management copy CRCF ndash J Sornet Page 31 48

() Des alliances eacuteconomiques naissent entre pays eacutemergents (notamment en

ameacuterique centrale creacuteation de la Banque du Sud en 2008 par exemple) et lrsquoon

commence agrave imaginer une baisse progressive de lrsquoinfluence eacuteconomique des Etats

Unis

2 ndash Les deacutefis actuels du management

21 ndash Les grandes orientations

Lrsquoeacutevolution eacuteconomique suggegravere quelques pistes parfois concurrentes pour lrsquoaction du

manager contemporain On y retrouve au premier plan la construction drsquoune vision qui est

une composante commune du leadership

Objectif du manager

pour lrsquoentreprise

Justification Facteurs de reacuteussite

Construire une vision Eclairer lrsquoavenir de lrsquoentreprise partager

un but souder motiver

Effort de reacuteelle prospection

volontarisme de la direction

bonne communication

Reacuteactiviteacute et flexibiliteacute

(sous tous les aspects

agrave tous niveaux)

Srsquoadapter rapidement au marcheacute Bonne organisation des processus

personnel compeacutetent autonome

et motiveacute structure hieacuterarchique

alleacutegeacutee robotisation

Deacutegager des profits Reacutemuneacuterer les apporteurs de capitaux

srsquoautofinancer

Ajuster coucircts et structures

Exploiter les nouvelles

technologies

Reacuteactiviteacute ajuster coucircts et deacutelais

reacutepondre au marcheacute suivre les clients

Organiser le SI de faccedilon

pertinente eacuteviter le coucirct excessif

drsquoinvestissements trop en

laquo pointe raquo (laquo essuyer les placirctres raquo)

utiliser judicieusement les services

exteacuterieurs

Bacirctir des alliances

(contrats fusions)

Deacutevelopper une activiteacute limiter les coucircts

de transaction () atteindre la taille

critique et de meilleurs rendements se

recentrer sur une activiteacute profitable

Dominer les processus se donner

une identiteacute lisible externaliser se

doter drsquoune capaciteacute financiegravere

suffisante

Valoriser lrsquoimage Attirer les clients favoriser les alliances

donner confiance (apporteurs de fonds

employeacutes clients partenaires socieacuteteacute

civile)

Instaurer des regravegles de

gouvernance inteacutegrer le

deacuteveloppement durable

respecter lrsquoenvironnement

Geacuterer les risques Faire face aux aleacuteas eacuteconomiques et

technologiques (conjoncture politiques

accidents malveillance)

Creacuteer un systegraveme drsquoalerte geacuterer

la crise (reacuteaction raisonneacutee

sceacutenarios poursuite de

lrsquoexploitation dans un contexte

instable) mise en place de

proceacutedures drsquoapprentissage pour

ameacuteliorer les reacuteactions au fil du

temps

Geacuterer le changement Faire face agrave lrsquoeacutevolution de la demande

la pression sur les prix la variation des

performances financiegraveres la

concurrence la globalisation des

marcheacutes lrsquoeacutevolution technologique aux

fusions ou alliances aux changements

de reacuteglementation de direction hellip ()

Bonne communication pour

donner du sens au changement

et obtenir lrsquoadheacutesion du personnel

Rassembler et geacuterer les

connaissances former le

personnel

Innover Garder un avantage concurrentiel se

diffeacuterencier

Veille technologique et

commerciale investissement

Ouverture

internationale

Elargir le marcheacute saisir les opportuniteacutes Veille commerciale partenariats

() La theacuteorie des coucircts de transaction deacuteveloppeacutee par OE Williamson dans les

anneacutees 70 integravegre les coucircts lieacutes au recours au marcheacute (recherche et choix drsquoun

fournisseur neacutegociation reacutedaction de contrat suivi des eacutechanges risque de rupture

Initiation au management copy CRCF ndash J Sornet Page 32 48

drsquoapprovisionnement hellip) On peut en conclure que lrsquointeacutegration de diffeacuterentes

activiteacutes agrave lrsquoentreprise (la laquo firme raquo) preacutesente des avantages Mais des coucircts de

transaction internes doivent aussi ecirctre consideacutereacutes (preacuteparation organisation

surveillance hellip) et certaines formes de coopeacuteration continue avec les fournisseurs

permettent de reacuteduire le coucirct des transactions externes

() drsquoapregraves laquo Les meilleures pratiques de management raquo - Brilman Heacuterard ndash EO

Une eacutetude du Conference Board (2002) liste les deacutefis du management vus par 700 leaders

mondiaux Soit en reacutesumeacute avec indication du score correspondant

1 ndash Fideacuteliser les clients (42)

2 ndash Reacuteduire les coucircts (38)

3 ndash Accroicirctre flexibiliteacute et reacuteactiviteacute (29)

4 ndash Amener les employeacutes agrave adheacuterer aux valeurs et visions de lrsquoentreprise (26)

5 ndash Deacutevelopper et retenir les leaders (25)

6 ndash Geacuterer acquisitions et alliances (24)

7 ndash Accroicirctre lrsquoinnovation (20)

En fin de classement citoyenneteacute et reacuteputation (4) et ameacutelioration de la diversiteacute (3)

22 ndash Les techniques disponibles

Pour faire face aux deacutefis le manager dispose de nouveaux concepts et de nouvelles

techniques Le tableau ci-dessous en donne un reacutesumeacute et indique les domaines qursquoils

influencent principalement

Initiation au management copy CRCF ndash J Sornet Page 33 48

Principaux concepts techniques outils Incidence principale sur

Internet

- e-commerce (commerce eacutelectronique site

entreprise)

- CRM ou GRC (gestion de la relation client)

- e-procurement (gestion des approvisionnements

par le reacuteseau)

- messagerie eacutelectronique

- e-recrutement

Vente accegraves au marcheacute

Relation client reacuteactiviteacute personnalisation

fideacutelisation

Deacutelais coucircts

Communication transfert de donneacutees (piegraveces

jointes) tous domaines

Communication recrutement

Intranet reacuteseau drsquoentreprise SI

- knowledge management (gestion des

connaissances)

- e-learning (apprentissage en ligne)

- plateforme de travail collaboratif (groupware)

- workflow (circulation eacutelectronique de

documents enchaicircnement de processus)

- e-RH portail RH (libre accegraves aux postes agrave

pourvoir informations candidatures hellip)

- PGI (progiciel de gestion inteacutegreacute) ou ERP

Innovation capaciteacute au changement veille

documentaire

Formation du personnel accompagnement des

changements

Coordination communication interne

Coordination

Communication interne (voire internet en

externe) reacuteduction des coucircts climat drsquoentreprise

recrutement plans de carriegraveres hellip

Coucircts fiabiliteacute du systegraveme drsquoinformation deacutelais

processus (continuiteacute inteacutegration)

Logistique inteacutegreacutee

Supply Chain Management (SCM) gestion de la

logistique (incluant les approvisionnements)

Processus deacutelais coucircts

Externalisation

Valorisation du capital humain

GPEC (gestion preacutevisionnelle des emplois et

compeacutetences)

Coaching

Reacuteactiviteacute de lrsquoentreprise conservation des

compeacutetences rendements individuels turn-over

adaptation des compeacutetences motivation

Efficaciteacute individuelle controcircle reacutegulation

progregraves processus

Approche processus

Optimisation des processus

Deacutemarche qualiteacute totale (TQM ndash total quality

management)

Empowerment (empouvoirement)

Benchmarking reacuteingeacutenieacuterie

Coucircts marges qualiteacute deacutelais flexibiliteacute

externalisation eacutelargissement des compeacutetences

organisation

Ameacutelioration des processus (meacutetiers et supports)

Autonomie compeacutetences des employeacutes

Ameacutelioration des processus restructuration

Management par la valeur

Parties prenantes

Satisfaction des parties prenantes financement

motivation collaborations hellip

Collaboration inter organisations

Reacuteseaux drsquoentreprises alliances

EDI (eacutechange de donneacutees informatiseacutees) extranet

Impartition externalisation (outsourcing)

Coucircts recentrage investissements lancement

drsquoactiviteacute

Coucircts reacuteactiviteacute deacutelais relations avec

lrsquoadministration

Coucircts recentrage limitation des investissements

Ethique drsquoentreprise

Gouvernance drsquoentreprise (mode de direction

encadreacute par des regravegles)

Rocircle socieacutetal deacuteveloppement durable

environnement

Image de lrsquoentreprise reacutegulation du top

management relations actionnaires

Image peacutenaliteacutes et amendes objectifs

strateacutegiques

Initiation au management copy CRCF ndash J Sornet Page 34 48

23 ndash Le rocircle socieacutetal des entreprises

La responsabiliteacute socieacutetale de lrsquoentreprise (RSE) deacutesigne le rocircle qursquoelle prend dans la socieacuteteacute

au-delagrave de son activiteacute purement geacuteneacuteratrice de profit On parle aussi drsquoentreprise citoyenne

La RSE est indissociable du deacuteveloppement durable de porteacutee mondiale et dont les trois

piliers sont

- eacuteconomique (favoriser le deacuteveloppement les eacutechanges internationaux)

- social (accegraves aux soins eacuteducation conditions de travail hellip)

- environnemental (pollution preacuteservation des ressources hellip)

La RSE integravegre notamment une preacuteoccupation sociale de lrsquoentreprise vis-agrave-vis de ses salarieacutes

(seacutecuriteacute et santeacute au travail juste reacutemuneacuteration deacuteveloppement personnel hellip) Elle conduit agrave

tenir compte dans le management drsquoune vision exteacuterieure agrave lrsquoentreprise qui peut avoir des

reacutepercussions possibles sur son activiteacute eacuteconomique

Lrsquoentreprise peut aussi tirer avantage drsquoune deacutemarche responsable par la baisse de certains

coucircts (plus faibles consommations drsquoeacutenergies reacuteduction des transports hellip)

Le rocircle socieacutetal de lrsquoentreprise a eacuteteacute reconnu en France par la loi laquo NRE raquo de 2001 (loi sur les

nouvelles reacutegulations eacuteconomiques) qui oblige les socieacuteteacutes franccedilaise coteacutees sur un marcheacute

reacuteglementeacute agrave rendre compte dans leur rapport annuel de leur gestion sociale et

environnementale au travers de leur activiteacute

Article 116 de la loi Le rapport viseacute agrave larticle L 225-102 rend compte hellip laquo Il comprend

eacutegalement des informations dont la liste est fixeacutee par deacutecret en Conseil dEtat sur la

maniegravere dont la socieacuteteacute prend en compte les conseacutequences sociales et

environnementales de son activiteacute Le preacutesent alineacutea ne sapplique pas aux socieacuteteacutes

dont les titres ne sont pas admis aux neacutegociations sur un marcheacute reacuteglementeacute raquo

Une norme ISO 14000 integravegre ces preacuteoccupations et des taxes eacutecologiques sont

progressivement creacutees

3 ndash Le management par la valeur

31 ndash De lrsquoanalyse au management par la valeur

Lrsquoanalyse de la valeur est neacutee en 1947 aux Etats-Unis (General Electrics) Cette technique

consiste agrave eacutelaborer des produits conformes aux attentes de la clientegravele mais sans excegraves pour

trouver un bon compromis entre valeur pour le client et coucirct Le produit optimal est deacutefini agrave

partir drsquoenquecirctes qui deacuteterminent le besoin client (ou plutocirct drsquoun client laquo type raquo)

Exemple il est inutile de concevoir un petit veacutehicule citadin capable de parcourir

500 000 km sans avarie compte tenu des effets de mode et du faible kilomeacutetrage

annuel Par contre le marcheacute peut exiger un fonctionnement sans faille sur 150 000 km

soit dix ans en moyenne ce qui conditionne les coucircts de production

Cette recherche drsquoun ajustement de valeur au besoin des clients eacutetait un preacutecurseur du

management par la valeur qui recherche plus largement la creacuteation de valeur pour

chacune des parties prenantes de lrsquoentreprise tout en lui meacutenageant un reacutesultat suffisant

Plus geacuteneacuteralement le management par la valeur est deacutefini par une norme europeacuteenne (EN

12973)

Le management par la valeur est un style de management particuliegraverement destineacute agrave

mobiliser les individus agrave deacutevelopper les compeacutetences et agrave promouvoir les synergies et

Initiation au management copy CRCF ndash J Sornet Page 35 48

linnovation avec pour objectif la maximisation de la performance globale dun

organisme Le management par la valeur apporte une nouvelle faccedilon dutiliser nombre

de meacutethodes de management existantes Il est en coheacuterence avec le Management

de la qualiteacute

Cette approche du management pose de nombreuses questions notamment quelles

prioriteacutes et quelles valeurs attribuer aux parties prenantes comment appreacutehender la

perception par les parties prenantes de la valeur qui leur est affecteacutee

32 ndash La valeur client

Le processus drsquoeacutelaboration drsquoun produit qui consomme des ressources coucircteuses doit creacuteer

une valeur suffisante pour provoquer lrsquoachat par le client final La production drsquoune valeur

reconnue par le client est vitale pour lrsquoentreprise mais sa deacutetermination est parfois complexe

La valeur du produit perccedilue par le client integravegre des eacuteleacutements en partie subjectifs

- une valeur drsquousage (le produit reacutepond agrave un besoin)

- une valeur drsquoestime (lrsquoimage apporteacutee par le produit un aspect affectif)

- une valeur drsquoeacutechange (deacuteduite de lrsquoespoir de revente du produit)

Valeurs drsquousage drsquoestime et drsquoeacutechange deacutependent implicitement de la qualiteacute (un bien peu

fiable est impropre agrave lrsquousage attendu de mauvaise qualiteacute notoire il nrsquoapporte pas une

image positive et ses deacutefauts connus nuisent agrave sa revente) Une eacutevaluation de la qualiteacute

intervient donc dans la valeur perccedilue du produit

Par ailleurs le client considegravere le coucirct drsquoobtention du produit (les charges qursquoil doit supporter

pour acqueacuterir le produit lrsquoeffort qursquoil doit faire pour trouver le produit et les frais de mise agrave

disposition)

Le prix perccedilu par le client est geacuteneacuteralement supeacuterieur au prix de vente

Le client achegravete theacuteoriquement le produit qui preacutesente la diffeacuterence valeur perccedilue ndash prix

perccedilu la plus favorable ou le meilleur rapport prix perccedilu qualiteacute perccedilue et dans certains

cas celui qui a le prix produit le plus bas

Remarque les valeurs du scheacutema ci-dessus changent durant le cycle de vie du produit

(un nouveau produit peut avoir une valeur perccedilue plus eacuteleveacutee qursquoen fin de vie) La

valeur client ne peut ecirctre eacutevalueacutee que par enquecirctes et ne peut donc ecirctre deacutefinie avec

certitude

La notion de laquo satisfaction client raquo conseacutecutive agrave une vente influence aussi le prix produit et

le prix perccedilu

- lrsquoentreprise gagne sur les coucircts de recherche de clientegravele

- le client nrsquoa pas agrave rechercher un nouveau fournisseur et beacuteneacuteficie drsquoun coucirct drsquoobtention

plus bas

valeur perccedilue client

prix perccedilu client

coucirct produit Marge (valeur creacuteeacutee pour

lrsquoentreprise)

euros

prix produit

Valeur creacuteeacutee

pour le client

Initiation au management copy CRCF ndash J Sornet Page 36 48

La satisfaction du client deacutepend de facteurs qualitatifs aussi divers que la fiabiliteacute du produit

la vitesse de reacuteaction du fournisseur lrsquoattitude des commerciaux lrsquoefficaciteacute du service

apregraves-vente la netteteacute des contrats ou la justesse de la facture

Valeur perccedilue coucirct marge et satisfaction reacutesultent de processus allant de la conception du

produit jusqursquoagrave sa livraison et son apregraves-vente La deacutemarche laquo processus raquo et lrsquolaquo analyse de la

valeur raquo en forccedilant la recherche de solutions efficientes agrave tout niveau administratif

technique commercial et apregraves-vente sont donc neacutecessaires pour bien positionner

lrsquoentreprise sur son marcheacute

Pour autant le risque commercial ne peut jamais ecirctre annuleacute et lrsquooffre de lrsquoentreprise ne

satisfait geacuteneacuteralement pas en milieu concurrentiel tous ses clients potentiels

33 - La creacuteation de valeur pour les autres parties prenantes

Les salarieacutes

La creacuteation drsquoune valeur suffisante pour les salarieacutes est reconnue comme neacutecessaire car des

observations montrent que la satisfaction des clients en deacutepend Moins souvent eacutevoqueacutee en

peacuteriode de chocircmage elle nrsquoest prioritaire que pour les employeacutes dont lrsquoentreprise souhaite

conserver les compeacutetences

La laquo valeur salarieacute raquo ne comprend pas que le salaire Le sentiment drsquoappartenance agrave un

groupe la reconnaissance lrsquoaccomplissement de soi et la construction professionnelle en

sont des eacuteleacutements importants Comme pour les clients on doit ainsi distinguer la reacutetribution

perccedilue du salaire objectif

Les actionnaires

Lrsquoactionnaire apporte des fonds propres agrave lrsquoentreprise en contrepartie de titres parfois

neacutegociables en bourse et assortis drsquoun droit de vote en assembleacutee geacuteneacuterale La valeur

attribueacutee aux actionnaires est servie en termes moneacutetaires (dividende ou augmentation de la

valeur du titre neacutegociable)

Remarque des facteurs non moneacutetaires comme lrsquoimage de lrsquoentreprise qui deacutepend

en partie de sa communication peuvent influencer la deacutecision drsquoachat de vente ou

de conservation des titres par lrsquoactionnaire

Reacutetribution perccedilue euros

Salaire objectif

Avantage non

moneacutetaire de

lrsquoemploi

Initiation au management copy CRCF ndash J Sornet Page 37 48

Compte tenu de lrsquoimportance croissante de lrsquoactionnariat dans le financement des grandes

entreprises coteacutees en bourse et notamment des investisseurs institutionnels comme les fonds

de pension des indicateurs speacutecifiques ont eacuteteacute introduits pour appreacutecier la performance des

entreprises vue par les actionnaires Par exemple la valeur ajouteacutee eacuteconomique (EVA reg

economic value added marque deacuteposeacutee de Stern Stewart ou VAE ndash valeur ajouteacutee

eacuteconomique parfois deacutenommeacutee VEC ndash valeur eacuteconomique creacuteeacutee) qui prend en compte le

coucirct du capital

LrsquoEVA correspond tregraves scheacutematiquement au calcul suivant

EVA = (PO) profit opeacuterationnel ndash (C) coucirct du capital X (CE) capitaux employeacutes

LrsquoEVA neacutecessite en pratique des retraitements assez complexes Le PO peut se deacuteterminer

selon les principes suivants

- PO = reacutesultat drsquoexploitation (avant inteacuterecircts) ndash impocirct

- PO = beacuteneacutefice courant (tenant compte des inteacuterecircts) + inteacuterecircts ndash eacuteconomie drsquoimpocirct sur les

inteacuterecircts (on exclue les eacuteleacutements financiers et lrsquoimpocirct correspondant) ndash impocirct

- lrsquoimpocirct pris en compte correspond au profit opeacuterationnel consideacutereacute (dans les cas courants agrave

13 du PO)

C = taux moyen de reacutemuneacuteration du capital (reacutesultant par exemple du dividende exigeacute de

certains investisseurs et des taux drsquoemprunts bancaires)

CE = capitaux propres et dettes portant inteacuterecirct

Remarque le profit opeacuterationnel ou reacutesultat opeacuterationnel correspond au NOPAT ndash net

operating profit after tax - anglo-saxon LrsquoEVA est eacutegale au NOPAT diminueacute de la

reacutemuneacuteration des capitaux

Exemple lrsquoentreprise X dispose drsquoun capital de 2 500 000 euro et reacutealise un beacuteneacutefice net

drsquoimpocirct de 450 000 euro (taux 33 13) Un dividende de 6 doit ecirctre verseacute aux

actionnaires et la banque lui a accordeacute un precirct de 1 200 000 euro agrave 4 Les autres

constituants des reacutesultats financier et exceptionnel sont neacutegligeables

Reacutesultat opeacuterationnel = 450 000 + 004 x 1 200 000 x 23 = 482 000 euro

Coucirct du capital = 006 x 2 500 000 + 004 x 1 200 000 x 23 = 182 000 euro

EVA = 300 000 euro

Coucirct moyen pondeacutereacute du capital (C) = (004 x 1 200 000 x23 + 006 x 2 500 000)

3 700 000 Soit 492

Si lrsquoEVA est positive lrsquoentreprise creacuteeacutee de la valeur apregraves reacutemuneacuteration des capitaux et sa

valeur boursiegravere doit augmenter

Lrsquoutilisation de lrsquoEVA comme indicateur influence le management de lrsquoentreprise car il y a

trois moyens pratiques drsquoaugmenter lrsquoEVA

- augmenter le reacutesultat opeacuterationnel

- lancer des investissements ayant une rentabiliteacute supeacuterieure agrave C

- eacuteliminer les activiteacutes ayant une rentabiliteacute infeacuterieure agrave C

Remarque lrsquoutilisation sans nuance de lrsquoEVA comme critegravere de management peut

poser problegraveme Le calcul de lrsquoEVA repose sur des ajustements comptables il est donc

sujet agrave manipulations (provisions capitalisation ou non de la RD hellip) Par ailleurs le

critegravere laquo EVA raquo pris isoleacutement peut conduire agrave chercher la rentabiliteacute agrave court terme agrave

reacuteduire les investissements prospectifs et donc nuire agrave terme au deacuteveloppement de

lrsquoentreprise

Initiation au management copy CRCF ndash J Sornet Page 38 48

Les fournisseurs reccediloivent le paiement de leurs factures plus ou moins rapidement (le deacutelai

de paiement repreacutesente une valeur consentie au fournisseur)

Lrsquoentreprise peut accroicirctre la valeur apporteacutee agrave ses fournisseurs par des actions cibleacutees

comme une contribution agrave la formation de leurs personnels certains transferts de

technologie ou de savoir faire agrave des sous-traitants une coopeacuteration suivie favorisant leur

deacuteveloppement lrsquointeacutegration agrave des campagnes de promotion

A noter que la valeur consentie aux fournisseurs peut avoir une influence sur la qualiteacute et les

deacutelais de livraison des produits

La collectiviteacute reccediloit des taxes et parfois des prestations en nature par deacutefaut ou explicites

(effort de preacuteservation de lrsquoenvironnement ameacutenagement du territoire par les implantations

aide mateacuterielle agrave des projets participation agrave la formation par exemple)

APPLICATIONS DT

DT1 Deacutefinir expliquer deacutereacuteglementation socieacutetal eacuteconomies drsquoeacutechelle coaching EDI

gouvernance

DT2 Deacuteterminer en quoi la deacutemarche TQM srsquoinscrit dans les deacutefis actuels du management

DT3 Apregraves avoir consulteacute les documents ci-dessous extraits du site drsquoAir France

(httpdeveloppement-

durableairfrancecomFRfrlocaldemarcheN4_positionnement_pphtm)

exposer les enjeux et les limites de la RSE et de la gestion des parties prenantes

Initiation au management copy CRCF ndash J Sornet Page 39 48

Dialogue avec les parties prenantes

Initiation au management copy CRCF ndash J Sornet Page 40 48

Attentes des parties prenantes

Initiation au management copy CRCF ndash J Sornet Page 41 48

Creacuteation de valeur pour les parties prenantes

La creacuteation de valeur pour les parties prenantes est au cœur de la strateacutegie du Groupe Le scheacutema de

distribution financiegravere ci-dessous donne un aperccedilu de la distribution des recettes du Groupe aux

diffeacuterentes parties prenantes actionnaires collaborateurs fournisseurs pouvoirs publics

collectiviteacutes locales etc

Initiation au management copy CRCF ndash J Sornet Page 42 48

Fiche DT1 ndash Extrait du sommaire de laquo Problegravemes eacuteconomiques raquo No 2894

La gestion des entreprises bouleverseacutee par les technologies de linternet

Reacutealiteacutes industrielles - Annales des Mines Jean-Michel Yolin

Avec lavegravenement de linternet les processus de conception de production et de vente sont

radicalement remis en cause Quel que soit le secteur dactiviteacute les technologies de linternet

permettent en effet de reacuteduire les deacutelais et de passer dun processus discontinu agrave un processus

continu Lorganisation des entreprises et leur mode de gestion en sont profondeacutement bouleverseacutes

tant au niveau individuel que collectif Linternet rend ainsi possible la reacutealisation dobjectifs que les

entreprises cherchaient agrave atteindre depuis longtemps sans y parvenir meilleure eacutecoute du client

travail sans stocks en flux tendu hieacuterarchies plates autorisant une grande reacuteactiviteacute flexibiliteacute dans

lorganisation et loutil de production acceacuteleacuteration du renouvellement des produits entreprises en

reacuteseau ougrave chacune se recentre sur son cœur de meacutetier etc

Le laquo knowledge management raquo ou comment geacuterer les connaissances

Document de travail du LAMSADE - Michel Grundstein

Peter Drucker lavait preacutedit le capital immateacuteriel eacutetait voueacute agrave devenir un facteur de compeacutetitiviteacute

pour lentreprise La libeacuteralisation des eacutechanges acceacutelegravere les processus de deacutecision de lentreprise

et implique que lassimilation des informations soit agrave la fois de meilleure qualiteacute et plus rapide Ainsi

la fonction qui consiste agrave manager les connaissances au sein de lentreprise savegravere primordiale

Bien que la prise de conscience de limportance du capital immateacuteriel ait eacuteteacute tardive - le concept

de knowledge management est apparu en France aux Etats-Unis et au Japon au milieu des

anneacutees 1990 - agrave lheure actuelle lorganisation de leacutechange dinformations et le partage des

connaissances sont devenus des facteurs cleacutes dune gestion performante de lentreprise Ils

doivent sinscrire dans un projet global destineacute agrave mettre en valeur les savoirs et les savoir-faire

individuels et collectifs

Les leccedilons du laquo coaching raquo pour le management de la qualiteacute

Humanisme et Entreprise - Martine Brasseur

Parmi les nouvelles formes de management en vogue dans les entreprises le coaching figure en

bonne place Appliqueacute au management de la qualiteacute il sagit dune pratique

daccompagnement destineacutee agrave initier et agrave faciliter le processus de deacuteveloppement dun individu

La deacutemarche consiste agrave affirmer que tout individu est en quecircte de qualiteacute agrave condition toutefois

de ne pas lui imposer des contraintes lempecircchant de progresser On considegravere notamment les

erreurs comme potentiellement feacutecondes En deacutefinitive le coach donne au coacheacute la permission

de reacuteussir en lui donnant aussi la permission deacutechouer

Initiation au management copy CRCF ndash J Sornet Page 43 48

Fiche DT2 ndash Management strateacutegique les sept deacutefis agrave relever dici agrave 2016

Extrait drsquoun article du site wwwlentreprisecom -Sabine Blanc - Mis en ligne le 20032007

(httpwwwlentreprisecom325article11977html)

Une eacutetude anglaise publieacutee par lopeacuterateur Orange Grande-Bretagne deacutecrypte la mutation

des formes de travail et les enjeux majeurs pour les entreprises de demain afin decirctre au top

de la compeacutetitiviteacute Voici les challenges-cleacutes pour les managers qui veulent rester dans la

course hellip

1 - Future organisation du travail les quatre laquo mondes raquo possibles

La reacutealiteacute sera probablement un meacutelange de ces quatre sceacutenarios souligne lrsquoeacutetude

Les mondes mutuels Tout se passe dans le cadre des communauteacutes locales vie priveacutee

comme professionnelle Le modegravele coopeacuteratif preacutevaut au lieu du laquo big business raquo Oublieacutes

aussi dans ce systegraveme les trajets pour aller au bureau les gens preacutefegravereront travailler dans de

petites entreprises locales souvent connecteacutees au reacuteseau drsquoautres structures similaires

Les laquo reacutepondants raquo (en anglais laquo replicants raquo) La figure du consultant freelance deviendra

dominante tandis que celle du salarieacute deacuteclinera Il ne sera pas rare de travailler pour plusieurs

entreprises On perdra en seacutecuriteacute de lrsquoemploi en visibiliteacute et en routine ce que lrsquoon gagnera

en liberteacute La majeure partie des tacircches srsquoeffectuera chez soi avec la possibiliteacute de srsquoinstaller

temporairement dans les bureaux de son client du moment Dans un contexte dincertitude

sur lrsquoavenir les travailleurs alterneront peacuteriodes drsquoactiviteacute intense et repos Ce sera agrave eux

drsquoaller vers les entreprises et non lrsquoinverse mecircme si celles-ci devront veiller agrave rester attractives

Les cottages eacutelectroniques Comme ce nom le suggegravere le teacuteleacutetravail deviendrait la norme

univers priveacute et professionnel se confondant Plus besoin de subir une heure de transport les

salarieacutes se logueront de chez eux sur le reacuteseau de lrsquoentreprise Les reacuteunions se tiendront dans

de petits bureaux centraux situeacutes agrave courte distance La flexibiliteacute du temps de travail srsquoimpose

Les salarieacutes disposeront de plus de marge de liberteacute dans leur activiteacute

Les disciples de la nueacutee Cette appellation poeacutetique cache simplement une extension de

lrsquoorganisation actuelle des grandes entreprises avec des salarieacutes se rendant sur un lieu de

travail centraliseacute Le rocircle croissant des technologies de lrsquoinformation multipliera les faccedilons de

collaborer et accroicirctra lrsquoefficaciteacute Le controcircle du travail sera omnipreacutesent La frontiegravere entre

travail et vie priveacutee restera marqueacutee

2 - Sept deacutefis pour les entreprises et leur managers

Quoi qursquoil advienne les entreprises et leurs dirigeants devront concentrer leurs efforts sur sept

points-cleacutes pour srsquoadapter Voici quelques exemples de probleacutematiques souleveacutees par le

rapport et des pistes de solution

Le leadership Les managers devront entre autres savoir persuader et influencer des

travailleurs beaucoup plus indeacutependants Ils auront aussi agrave repenser les niveaux auxquels

prendre les deacutecisions strateacutegiques en haut ou au contraire agrave des degreacutes moins eacuteleveacutes de la

pyramide hieacuterarchique

gt Faire du management une force facilitant les activiteacutes transversales plutocirct que la reacuteduire agrave

la seule fonction de deacutecision

La culture drsquoentreprise Davantage de salarieacutes capables de reacutefleacutechir seront neacutecessaires

tandis que les tacircches qui peuvent ecirctre automatiseacutees ou scripteacutees diminueront Un des

enjeux creacuteer une culture agrave mecircme drsquoattirer et drsquoencourager les personnes preacutesentant ces

qualiteacutes de reacuteflexion requises dans un contexte de compeacutetition accrue et de plus grande

indeacutependance des travailleurs

Initiation au management copy CRCF ndash J Sornet Page 44 48

gt Passer si neacutecessaire drsquoune culture drsquoentreprise forte agrave un mode drsquoengagement plus

consensuel moins rebutant

La marque Conseacutequence du recours croissant agrave lrsquo laquo outsourcing raquo lrsquoimage drsquoune marque

deacutependra plus drsquoagents exteacuterieurs qui ne fonctionnent pas forceacutement selon le mecircme mode

drsquoorganisation Comment garder le controcircle dessus

gt Choisir le mode qui corresponde le plus agrave vos valeurs et preacutevoir un programme de risk

management qui mette en eacutevidence ougrave les conflits sont susceptibles de jaillir

Lrsquoinnovation Plus que jamais il faudra faire face agrave une acceacuteleacuteration du rythme de

lrsquoinnovation en proposant constamment des solutions adapteacutees

gt Tisser des partenariats strateacutegiques avec drsquoautres entreprises pour partager les coucircts et les

fruits de lrsquoinnovation

Le deacutefi opeacuterationnel et technologique De quelle faccedilon controcircler lrsquoinformation crsquoest-agrave-dire

faire en sorte que les bonnes personnes accegravedent facilement agrave une information toujours en

phase tout en maintenant la seacutecuriteacute

gt Recourir agrave des laquo feuilles de route des futurs raquo syntheacutetisant en une page les indicateurs

sociaux et de consommation ainsi que les eacutevolutions technologiques et leacutegislatives qui

influent sur les changements et indiquant comment ils modifient vos marcheacutes vos clients et

votre organisation

La qualiteacute Si de nouveaux proceacutedeacutes ont pu deacutegrader la qualiteacute comme le recours agrave des

centres drsquoappel externaliseacutes drsquoautres ideacutees se sont reacuteveacuteleacutees plus prometteuses comme en

teacutemoigne le succegraves de certaines compagnies aeacuteriennes low cost Elles ont su conjuguer prix

serreacutes et services eacuteleveacutes ce qui devra devenir la norme estime lrsquoeacutetude

gt Continuer de rechercher la qualiteacute Elaborez aussi une bonne prestation service qui inclut

une livraison de qualiteacute voire creacuteez-la en partenariat avec les consommateurs

La leacutegislation La question de la proprieacuteteacute intellectuelle pourrait ecirctre probleacutematique Elle est

deacutejagrave source de conflits comme en teacutemoigne le procegraves pour violation de brevet intenteacute agrave RIM

le fabricant canadien du Blackberry par NTP Que pourra-t-on et que faudra-t-il proteacuteger par

un brevet Il sera eacutegalement neacutecessaire drsquoadapter la leacutegislation aux nouveaux modes

drsquoorganisation

gt Collaborer avec les acteurs du mecircme secteur et les leacutegislateurs pour deacutevelopper les

modegraveles des lieux de travail du futur et bacirctir le droit le plus adeacutequat

Orange a-t-il vu juste dans ses preacutevisions Rendez-vous dans neuf ans pour la reacuteponsehellip

Initiation au management copy CRCF ndash J Sornet Page 45 48

Fiche DT3 ndash Le management par la qualiteacute totale

Extrait drsquoune lettre drsquoinformation du cabinet Baud Accordance Consulting AD2 consultants ndash

2002

1 - Le TQM (Total Quality Management) offre pour lentreprise une vision de la qualiteacute plus

large et transversale

Son principe est simple La finaliteacute de lEntreprise est de deacutevelopper la satisfaction de ses

clients tout en eacutetant beacuteneacuteficiaire cest agrave dire pas agrave nimporte quel prix Elle doit ameacuteliorer sa

rentabiliteacute au travers de la deacutemarche qualiteacute La Qualiteacute Totale vise agrave fournir aux clients

externes et internes une reacuteponse adeacutequate agrave leurs attentes dans le meilleur rapport qualiteacute

prix la meilleure efficience

Elle considegravere pour cela lensemble des processus de lentreprise ayant une incidence sur la

qualiteacute et la satisfaction des clients

Le TQM fait ainsi une large place agrave

la deacutefinition et la planification de la strateacutegie geacuteneacuterale

la coheacuterence de la politique qualiteacute avec la strateacutegie

la deacutemultiplication de la politique qualiteacute dans toutes les directions de lentreprise

la relation client fournisseur interne

la prise en compte de lenvironnement concurrentiel

la consideacuteration de lensemble des risques potentiels financiers sociaux concurrentielshellip

limplication et la motivation du personnel

lanalyse des besoins des clients et le positionnement marketing

la maicirctrise des processus transverses internes

les reacutesultats sous tous ses aspects y compris financiers commerciaux image

De nombreux reacutefeacuterentiels sont relatifs agrave la Qualiteacute Totale hellip Tous ces reacutefeacuterentiels imposent un

questionnement plus profond et indiscret sur le mode de fonctionnement de lentreprise et

son management

helliphellip

2 - LISO 9001 2000 au travers du deacuteploiement des processus (management supports

reacutealisation et ameacutelioration continue) reacutepond quelque peu agrave la mecircme logique

LISO est une ouverture indeacuteniable vers la logique du TQM mais ne se reacutefegravere pas agrave la notion

defficience

Les dirigeants sont cependant sensibles agrave la neacutecessaire reacuteduction des coucircts de non-qualiteacute

et dobtention de la qualiteacute agrave la rentabiliteacute du systegraveme de management de la qualiteacute

mais ne perccediloivent pas toujours la qualiteacute comme une deacutemarche globale

Les deacutemarches qualiteacute commencent bien souvent par la remise en cause de lorganisation

leacutevaluation critique de son efficaciteacute lexamen des processus et la mise en eacutevidence des

lourdeurs administratives

La qualiteacute devient laffaire de tous hellip

Initiation au management copy CRCF ndash J Sornet Page 46 48

Fiche DT4 ndash Le deacuteveloppement durable et la RSE

Extrait du site wwwvigeocom

(httpwwwvigeocomcsr-rating-agencyfrmethodologiecriteres-de-recherche37-

criteres-d-analysehtml)

Deacuteveloppement durable laquo un deacuteveloppement qui reacutepond aux besoins du preacutesent sans compromettre

la capaciteacute des geacuteneacuterations futures de reacutepondre aux leurs raquo (Commission mondiale sur lrsquoenvironnement

et le deacuteveloppement ndash 1987)

Reacutefeacuterentiel drsquoeacutevaluation des entreprises par le groupe Vigeacuteo (le groupe mesure les performances et le

niveau de maicirctrise des risques de responsabiliteacute sociale des entreprises et des organisations - site

wwwvigeocom)

1 Ressources Humaines Ameacutelioration continue des relations professionnelles des relations drsquoemploi et des conditions de travail 2 Droits humains sur les lieux de travail Respect de la liberteacute syndicale et promotion de la neacutegociation collective non discrimination et promotion de lrsquoeacutegaliteacute eacutelimination des formes de travail proscrites (enfants travail forceacute) preacutevention des traitements inhumains ou deacutegradants de type harcegravelements sexuels protection de la vie priveacutee et des donneacutees personnelles 3 Environnement Protection sauvegarde preacutevention des atteintes agrave lenvironnement mise en place drsquoune strateacutegie manageacuteriale approprieacutee eacuteco conception protection de la biodiversiteacute et maicirctrise rationnelle des impacts environnementaux sur lrsquoensemble du cycle de vie des produits ou services

4 Comportements sur les marcheacutes Prise en compte des droits et inteacuterecircts des clients inteacutegration de standards sociaux et environnementaux dans la seacutelection des fournisseurs et sur lrsquoensemble de la chaicircne drsquoapprovisionnement preacutevention effective de la corruption respect des regravegles concurrentielles 5 Gouvernement drsquoentreprise Efficience et probiteacute assurance de lrsquoindeacutependance et de lrsquoefficaciteacute du Conseil drsquoadministration effectiviteacute et efficience des meacutecanismes drsquoaudit et de controcircle et notamment inclusion des risques de responsabiliteacute sociale respect des droits des actionnaires et notamment des minoritaires transparence et rationaliteacute de la reacutemuneacuteration des dirigeants 6 Engagement socieacutetal Effectiviteacute inteacutegration manageacuteriale de lrsquoengagement contribution au deacuteveloppement eacuteconomique et social des territoires drsquoimplantation et de leurs communauteacutes humaines engagements concrets en faveur de la maicirctrise des impacts socieacutetaux des produits et des services contribution transparente et participative agrave des causes drsquointeacuterecirct geacuteneacuteral

Initiation au management copy CRCF ndash J Sornet Page 47 48

ELEMENTS DE CORRIGE DT DT1 Deacutefinir expliquer

Deacutereacuteglementation = suppression des contraintes eacuteconomiques (libre eacutechange des biens et

capitaux)

Socieacutetal = qui se rapporte agrave la structure agrave lrsquoorganisation ou au fonctionnement de la socieacuteteacute

Economies drsquoeacutechelle = reacuteduction des coucircts lieacutee au niveau drsquoactiviteacute (amortissement des

charges fixes)

Coaching = accompagnement de personnes ou deacutequipes pour le deacuteveloppement de leurs

potentiels

EDI = eacutechange de donneacutees informatiseacutees ET standardiseacutees (ex SWIFT bancaire edifact

documents deacuteclaratifs)

Gouvernance = exercice du pouvoir la bonne gouvernance est participative et eacutequitable

conforme agrave lrsquointeacuterecirct commun

DT2 Deacuteterminer en quoi la deacutemarche TQM srsquoinscrit dans les deacutefis actuels du management

Voir notamment fiche 43

Maicirctrise des processus reacuteduction des coucircts reacuteactiviteacute et satisfaction de la clientegravele = faire

face agrave la concurrence

Ameacutelioration de lrsquoimage motivation du personnel

DT3 Apregraves avoir consulteacute les documents ci-dessous extraits du site drsquoAir France

(httpdeveloppement-

durableairfrancecomFRfrlocaldemarcheN4_positionnement_pphtm)

exposer les enjeux et les limites de la RSE et de la gestion des parties prenantes

Trame geacuteneacuterale possible

Introduction

Les deacutefis contemporains (accroissement de la concurrence devenue mondiale recherche

de nouveaux avantages concurrentiels pression de la socieacuteteacute besoin drsquoimage et de projet

lisible pour mener lrsquoentreprise crise et scandales du libeacuteralisme hellip) RSE et PP

Deacuteveloppement (voir cours)

1 ndash Parties prenantes et management par la valeur

PP deacutefinir citer reacutesumer lrsquoavantage rechercheacute (fideacuteliser motiver recherche drsquoalliances

implicites)

PP moyens (dont exemples AF) et meacutethode de management par la valeur (reacutepartie)

2 ndash La responsabiliteacute socieacutetale de lrsquoentreprise

RSE 3 axes

- eacuteconomique (favoriser le deacuteveloppement les eacutechanges internationaux)

- social (accegraves aux soins eacuteducation conditions de travail hellip)

- environnemental (pollution preacuteservation des ressources hellip)

RSE gouvernance drsquoentreprise facteur drsquoimage inteacutegrable dans la deacutemarche PP

Article 116 de la loi Le rapport viseacute agrave larticle L 225-102 rend compte hellip laquo Il comprend

eacutegalement des informations dont la liste est fixeacutee par deacutecret en Conseil dEtat sur la maniegravere

dont la socieacuteteacute prend en compte les conseacutequences sociales et environnementales de son

activiteacute Le preacutesent alineacutea ne sapplique pas aux socieacuteteacutes dont les titres ne sont pas admis aux

neacutegociations sur un marcheacute reacuteglementeacute raquo

Initiation au management copy CRCF ndash J Sornet Page 48 48

RSE exemple AF (ONG fournisseurs)

3 ndash Liens entre PP et RSE

- la RSE introduit de nouvelles PP

- la RSE suppose le respect des PP usuelles (employeacutes clients notamment)

4 - Probleacutematique

- deacutefinir la valeur reacuteellement apporteacutee par une gestion des PP (confusion salaire ndash valeur

idem impocircts hellip ex laquo valeur ajouteacutee raquo)

- communication (neacutecessaire mais aller au-delagrave)

- marginaliteacute des deacutepenses RSE (efficaciteacute sinceacuteriteacute de lrsquoengagement marge de manœuvre)

- charge RSE reporteacutee sur des tiers (ex fournisseurs AF)

- inteacutegration de facteurs non visibles en comptabiliteacute (pertes drsquoemploi nuisances hellip)

Conclusion

Voies incontournables mais pouvant nrsquoavoir qursquoun effet superficiel et temporaire Voir utiliteacute

drsquoaccompagnement leacutegislatif de regravegles de gouvernance

Initiation au management copy CRCF ndash J Sornet Page 13 48

Fiche IM3 - Bref historique

Antiquiteacute

3000 AJC

Peacuteriode greacuteco-

romaine

Transition

feacuteodale

12egraveme siegravecle

europe

15egraveme ndash 17egraveme

siegravecles

19egraveme siegravecle

20egraveme siegravecle

agriculture preacutedominante industrie limiteacutee aux besoins drsquoun individu ou drsquoun clan

pour la confection des outils des vecirctements et de la poterie Force motrice animale

ou humaine pour lrsquoessentiel

Grands travaux drsquoeacutetat en Egypte premiegravere laquo planification ndash organisation ndash controcircle raquo

Deacuteveloppement des communications essor industriel limiteacute peu de progregraves

technique (lrsquoesclavage supplante les innovations)

Deacuteveloppement progressif des eacutechanges commerciaux

La consommation indirecte atteint un bon niveau (surplus agricoles et

deacuteveloppement des villes) Apparition de nouveaux commerccedilants

Etat fort Evolutions technologiques (imprimerie bateaux performants instruments de

navigation) Extension geacuteographique de lrsquoeacuteconomie Apparition des corporations

drsquoartisans

Machine agrave vapeur chemin de fer passage de lrsquoartisanat au capitalisme

entrepreneurial producteur organisation des entreprises

Ecole classique (Taylor Fayol Weber) approche meacutecaniste bureaucratie

hieacuterarchie commandement fonctions et speacutecialisation laquo OS T raquo (organisation

scientifique du travail) organisation source de pouvoir rationaliteacute des individus bases

du management

Deacuteveloppement du capitalisme manageacuterial Electriciteacute peacutetrole puis communications

et information Consommation de masse mondialisation preacuteoccupations

eacutenergeacutetiques et environnementales 3 peacuteriodes

- standardisation grandes entreprises industrielles

- industries de consommation 30 glorieuses marketing multinationales protection

sociale

- deacutereacuteglementation monteacutee des services pays eacutemergents mondialisation et nouvelle

eacuteconomie (internet)

Ecole des relations humaines prise en compte de lrsquoindividu des motivations styles

de direction

Ecole neacuteo-classique et post-classique deacutecentralisation coordonneacutee DPO

management participatif zeacutero deacutefaut flux tendus

Approche systeacutemique partition de lrsquoentreprise eacutetude des interactions feacutedeacuteration

vers lrsquoobjectif controcircle et ajustement

Theacuteories de la deacutecision rationaliteacute limiteacutee contribution reacutetribution coalitions

Ecole socio-technique recherche de compromis technologie organisation

enrichissement des tacircches autonomie des groupes

Approche sociologique effets sociaux du travail jeux de pouvoir dans lrsquoentreprise

reacutegulation sociale

Theacuteories de la contingence facteurs contingents adaptation agrave lrsquoenvironnement

configurations organisationnelles

Theacuteories de la firme controcircle manageacuterial droits de proprieacuteteacute relation drsquoagence

Theacuteories contractualistes firme nœud de contrats coucircts de transaction

opportunisme externalisation internalisation

Approche eacutevolutioniste eacutecologie des organisations modegravele eacutevolutioniste

contraintes de sentier

Approche par les ressources valorisation des ressources compeacutetences cleacutes

apprentissage organisationnel

(Classement simplifieacute)

Initiation au management copy CRCF ndash J Sornet Page 14 48

ELEMENTS DE CORRIGE IM

IM1 Commenter la deacutefinition du management par la norme ISO et le manager de Mintzberg

Efficient = optimum avec les moyens disponibles

ISO (management objectifs) (manager moyens) HM

IM2 Le leader entraicircne naturellement derriegravere lui Le manager nrsquoest pas toujours leader

(mecircme si crsquoest souhaitable) Le leader nrsquoest pas toujours manager (plutocirct notion individuelle)

Leadership = faculteacute de diriger conjugaison drsquoune autoriteacute naturelle ou drsquoun savoir-faire

acquis drsquoune capaciteacute agrave entraicircner des personnes ou des groupes et drsquoune leacutegitimiteacute

statutaire (de position)

IM3 Compleacuteter le tableau ci-dessous en analysant chaque action preacutesenteacutee Faire ensuite

ressortir les domaines niveaux ou techniques de management pouvant ecirctre mobiliseacutes pour

chaque situation

Satisfaction client

Implication du personnel

Processus systegraveme

Ameacutelioration continue

Deacutecision efficace

Recherche de valeur

Image entreprise

Liaisons

Information

Reacutepartition ressources

Reacutegulation

Neacutegociation

Leadership

Initiation au management copy CRCF ndash J Sornet Page 15 48

Caracteacuteristiques

de lrsquoaction

- reacutepeacutetition

- risque

- normes

- ampleur

Prise de

deacutecision

- opeacuteration

- direction

- deacutelai

Informations

neacutecessaires

- nature

- origine

- deacutelai

obtention

Cleacute pour la

reacuteussite

Intervention

exteacuterieure

possible

Assurer la

restauration du

soir

(Restaurant

familial)

Technique

(fabrication)

Vente (terrain)

Appros

Reacutepeacutetitive

(quot)

Risque faible

Normes

drsquohygiegravene

Faible

Opeacuterationnelle

Geacuterant

responsable

Rapide (qq

jours menu et

appros)

Nombre de

couverts

Tarifs usuels

Calendrier

(fecirctes)

Clients docs

divers

expeacuterience

Qq jours

Varieacuteteacute menu

Plats phares

Accueil

Appros

Tarification

Vins

Gestion

congeacutelation

Qualiteacute cuisine

Fournisseurs

Extra

Publiciteacute

Construire un

viaduc

(autoroute)

Technique

Organisation

Appros

Uniteacute (ou peu)

Eleveacute (financier

technique)

Architecture

Eleveacutee

Direction

(aleacuteas)

Opeacuterationnelle

(conduite

chantier)

Immeacutediat agrave qq

semaines

Plans

plannings

Qualifications

Meacuteteacuteo

Disponibiliteacutes

Bureau eacutetudes

Qq sem agrave 24h

Techniciteacute

Appros

Qualifications

Preacutevision

GRH

Contrat juste

SS traitants

Organismes

certificateurs

Controcircle

client

Certifier les

comptes

annuels drsquoun

groupe national

(cabinet

drsquoaudit)

Technique

Relation client

Gestion des

connaissances

Annuelle

Moyen

Regravegles

comptables

fiscales

Moyenne (selon

importance du

cabinet)

Opeacuterationnelle

Qq jours agrave

semaines

Comptable

Juridique

Client

Etat

Qq jours agrave

semaines

Techniciteacute

Expeacuterience

Relation client

Systegraveme info client

Siegravege

Autre cabinet

Lancer une

ligne drsquoavions

(constructeur

aeacuteronautique)

Strateacutegique

RD

Etudes

Uniteacute

Tregraves eacuteleveacute

Aeacuteronautique

Tregraves eacuteleveacutee

Direction

Qq mois agrave

anneacutees

Marcheacute

Etudes

Compagnies

Qq mois agrave

anneacutees

Concept

Outil industriel

Coucirct exploitation

Tarif

Fiabiliteacute

Deacutelaisconcurrence

SI simulation

SS traitants

Bureaux

drsquoeacutetudes

speacutecialiseacutes

Compagnies

Conseils

Reacuteduire la

capaciteacute de

production

(groupe

industriel)

Strateacutegique

RH

Communication

Production

Uniteacute

Moyen

Leacutegislation

(dont RH)

Eleveacutee

Direction

Qq mois agrave

anneacutees

Financiegravere

Industrielle

Marcheacute

Organisation

Organismes

speacutecialiseacutes

DRH

Qq mois

Communication

Connaissance des

compeacutetences

Connaissance outil

industriel

Concurrence

Portefeuille

drsquoactiviteacutes

Cabinet

drsquoorganisation

Conseils

speacutecifiques

Acqueacuterir une

entreprise

concurrente

(teacuteleacutephonie

mobile)

Strateacutegique

Marketing

Production

(reacuteseau)

Financier

Communication

Uniteacute

Tregraves eacuteleveacute

Leacutegislation

telecom

Tregraves eacuteleveacutee

Direction

Qq mois

Financiegravere

Marcheacute

Reacuteseaux

(ampleur

recouvrement

hellip)

Organisations

Interne

Racheteacutee

Sources

speacutecialiseacutees

Qq mois

Communication

Marcheacute

Cours boursiers

Cabinet

drsquoorganisation

Conseils

speacutecifiques

Initiation au management copy CRCF ndash J Sornet Page 16 48

LE MANAGEMENT EN PRATIQUE

Pour assumer sa fonction le management doit couvrir sans discontinuiteacute lrsquoensemble de

lrsquoorganisation et inteacutegrer de nombreux facteurs dont nous allons reacutesumer lrsquoessentiel

1 ndash Les fonctions et activiteacutes du management

Pour Henri Fayol la fonction drsquoadministration de lrsquoentreprise (son management) reposait sur

cinq actions preacutevoir organiser commander coordonner et controcircler (laquo PO3C raquo)

Nous distinguerons cinq activiteacutes de management

- la conception (au plus haut niveau finaliteacute but ou vocation de lrsquoorganisation

meacutetiers dimension politique de croissance hellip)

- la planification (deacutefinition des objectifs eacutecheacuteances)

- lrsquoorganisation (reacutepartition du travail choix des modes de coordination)

- le pilotage de lrsquoaction opeacuterationnelle (motivation animation encadrement

assistance)

- lrsquoeacutevaluation (controcircle des reacutesultats obtenus ajustements)

Dans chacune de ces activiteacutes des deacutecisions et des arbitrages sont neacutecessaires avec des

enjeux plus ou moins importants

Remarques

- Les cinq activiteacutes du management peuvent se retrouver agrave tout niveau de

management si lrsquoentreprise laisse une certaine autonomie de deacutecision agrave ses diffeacuterentes

uniteacutes La conception est naturellement du ressort de la direction geacuteneacuterale et des

conseils drsquoadministration mais elle peut ecirctre preacutesente pregraves du terrain (latitude laisseacutee agrave

une filiale ou agrave un magasin par exemple) De mecircme lrsquoorganisation du travail concerne

un atelier mais aussi la direction qui structure lrsquoentreprise pour assurer ses activiteacutes sa

production

- La planification deacutefinit des objectifs ou des axes strateacutegiques (choix de produits

modaliteacutes de deacuteveloppement des ventes implantations alliances hellip) et les traduit en

donneacutees de gestion preacutevisionnelles syntheacutetiques et eacutechelonneacutees dans le temps afin de

valider les objectifs et de fixer des repegraveres

- Un laquo business plan raquo (plan drsquoaffaires)est notamment lrsquoeacutequivalent de la planification

dans le cas de creacuteation drsquoentreprise ou pour la preacutesentation de tout projet drsquoactiviteacute

Les activiteacutes du management srsquoinscrivent dans des cycles qui peuvent ecirctre scheacutematiseacute

comme suit (lrsquoeacutevaluation peut entraicircner une reacutevision du pilotage de lrsquoorganisation ou des

objectifs sans que lrsquoentreprise ne soit fondamentalement remise en cause)

conception

planification

organisation

pilotage

eacutevaluation

Initiation au management copy CRCF ndash J Sornet Page 17 48

2 ndash Les contextes de management

Le management est influenceacute par son contexte qui justifie des objectifs une organisation

des meacutethodes

Par exemple lrsquoentreprise admet de nombreuses variantes selon sa taille sa forme juridique

son controcircle par lrsquoeacutetat (entreprises publiques) ou par des inteacuterecircts priveacutes Il en va de mecircme des

organismes administratifs qui peuvent deacutependre de directives nationales ou reacutegionales des

associations qui ont des activiteacutes drsquoampleur tregraves variable

21 ndash La dimension de lrsquoentreprise

La dimension drsquoune entreprise se mesure principalement en fonction de son effectif ou de

son chiffre drsquoaffaires Des seuils sont deacutefinis par divers organismes et exploiteacutes agrave des fins

statistiques ou pour la deacutetermination de certaines obligations sociales ou fiscales

(repreacutesentation du personnel cotisations hellip) Il nrsquoy a bien entendu pas de laquo barriegravere de

tailleraquo absolue conditionnant le management drsquoune entreprise

LrsquoUE preacuteconise de distinguer les micro ndash entreprises (jusqursquoagrave 9 salarieacutes) les TPE ndash tregraves petites

entreprises (moins de 20 salarieacutes) les petites entreprises (moins de 50) et les moyennes

entreprises (de 50 agrave 250) Cependant les PME sont parfois situeacutees entre 10 et 500 salarieacutes

Remarques

- en France environ 40 des entreprises emploient de 1 agrave 50 salarieacutes (ce qui repreacutesente

plus de 50 des emplois) et 59 nrsquoen ont aucun

le pays compte environ 2 600 000 entreprises dont moins de 1 ont 250 employeacutes et

plus

- ancienneteacute et taille de lrsquoentreprise sont lieacutees si lrsquoon eacutecarte les restructurations et autres

eacutevolutions drsquoentreprises existantes

La dimension de lrsquoentreprise a une influence sur lrsquoorganisation et le laquo style raquo de son

management

- les PME sont souvent entrepreneuriales (les dirigeants eacutegalement apporteurs de capitaux

sont totalement engageacutes dans la marche de lrsquoentreprise) Elles ont une gestion flexible peu

formaliseacutee plus qualitative que quantitative Les PME sont freacutequemment focaliseacutees sur un seul

type drsquoactiviteacute Pour ne pas alourdir leur structure elles ont tendance agrave sous-traiter les

activiteacutes speacutecialiseacutees ne correspondant pas agrave leur meacutetier de base

- les grandes entreprises sont manageacuteriales (les dirigeants sont nommeacutes par les actionnaires

en raison de leurs compeacutetences) et moins reacuteactives

22 ndash Le type de production

On distingue industrie (production de biens mateacuteriels ou pour le moins de produits visibles ndash

comme un seacutejour touristique ou un film) et services (fourniture drsquoune prestation immateacuterielle)

Le type de production influence en principe le management de lrsquoentreprise

- lrsquoindustrie neacutecessite (si lrsquoon excepte lrsquoartisanat) un investissement relativement important

une organisation productive stable capable de reacutealiser plusieurs fois des produits identiques

(exemple un modegravele de reacutefrigeacuterateur) ou du moins similaires (exemple un bacirctiment) Le

produit de lrsquoindustrie consomme des matiegraveres et il doit geacuteneacuteralement ecirctre distribueacute jusqursquoau

client

- la production de services peut se satisfaire drsquoun investissement tregraves reacuteduit et neacutecessite un

contact permanent avec le client

Toutefois la standardisation des services et le deacuteveloppement des reacuteseaux informatiques

rapprochent la production de services de celle des biens industriels

- la production drsquoun service reacutepeacutetitif et technique peut imposer une structure lourde et une

organisation tregraves formaliseacutee (voir les grandes socieacuteteacutes drsquoaudit ou de conseil informatique)

Initiation au management copy CRCF ndash J Sornet Page 18 48

- certains services peuvent ecirctre fournis agrave distance sans contact direct avec le client et

distribueacutes par reacuteseau (tenue de comptabiliteacute affacturage gestion clientegravele centre drsquoappel

hellip)

Remarque les services repreacutesentent 75 de lrsquoactiviteacute eacuteconomique franccedilaise

23 ndash La nature de lrsquoorganisation

Les organisations publiques franccedilaises (administrations centrales collectiviteacutes territoriales

hocircpitaux hellip) repreacutesentent une part importante de lrsquoactiviteacute (environ 30 des emplois) La

fonction publique regroupe des organisations aux finaliteacutes diverses et qui ont des problegravemes

de gestion similaires agrave ceux des entreprises auxquelles elles peuvent emprunter des principes

de management Notamment

- pour controcircler les coucircts et assurer la qualiteacute des services

- pour communiquer avec les administreacutes ou les usagers

- pour motiver les personnels et geacuterer les ressources humaines

La transposition directe des techniques de gestion et de management nrsquoest cependant pas

toujours possible car

- la comptabiliteacute publique obeacuteit agrave des regravegles speacutecifiques (proceacutedure budgeacutetaire

notamment)

- le laquo client raquo ne paye pas toujours la prestation du moins directement

- la concurrence est parfois inexistante

- les grandes administrations centraliseacutees sont soumises agrave des choix politiques geacuteneacuteraux

parfois sans connexion eacutevidente avec les besoins opeacuterationnels

- le statut des personnels et les grilles de salaires limitent les possibiliteacutes de gestion des

ressources humaines

Remarque la LOLF (loi organique relative aux lois de finances) est entreacutee en vigueur en

2006 Elle alloue des moyens budgeacutetaires en fonction de programmes et remplace la

reconduction automatique de 90 des budgets Cette reacuteforme se heurte toutefois agrave la

lourdeur des grands ministegraveres ougrave la complexiteacute des activiteacutes est difficile agrave

appreacutehender et ougrave des inerties culturelles peuvent exister agrave tout niveau

Les associations loi de 1901 peuvent avoir une activiteacute comparable agrave celle de grandes

entreprises (voir par exemple les associations de santeacute ou professionnelles) et leur

management est alors similaire malgreacute lrsquoabsence de but lucratif (les beacuteneacutefices ne sont pas

distribuables) Elles ont drsquoailleurs en France un poids eacuteconomique important (elles emploient

environ 1 600 000 salarieacutes)

Cependant lrsquoadheacutesion agrave un systegraveme de valeurs fondateur de lrsquoassociation ou la limite de

lrsquoautoriteacute (quand un volant de beacuteneacutevoles important participe agrave lrsquoactiviteacute) peut introduire des

nuances

- le renforcement des objectifs socieacutetaux

- la faiblesse des relations hieacuterarchiques

- des contraintes de gestion du temps des beacuteneacutevoles

- des modaliteacutes particuliegraveres de recrutement et de motivation des dirigeants

24 ndash Les facteurs contingents

La theacuteorie de la contingence montre qursquoune structure drsquoentreprise nrsquoest efficace que dans

une situation deacutetermineacutee et qursquoil nrsquoexiste que des solutions de management construites dans

un contexte preacutecis

Le management doit ainsi srsquoadapter agrave des facteurs contingents qui ne peuvent ecirctre

controcircleacutes du moins agrave bregraveve eacutecheacuteance Ces facteurs sont par exemple

- lrsquoancienneteacute de lrsquoentreprise (plus elle est ancienne plus lrsquoentreprise a tendance agrave reacutepeacuteter

des comportements eacuteprouveacutes)

Initiation au management copy CRCF ndash J Sornet Page 19 48

- la taille de lrsquoentreprise (la grande entreprise a une composante administrative plus

deacuteveloppeacutee)

- le systegraveme de production (tregraves standardiseacute complexe automatiseacute hellip)

- lrsquoenvironnement

3 ndash Le management et les parties prenantes

Lrsquoentreprise a pour vocation premiegravere de mettre des produits agrave disposition de ses clients en

reacutealisant un profit Pour y arriver elle doit aussi satisfaire ses parties prenantes salarieacutes

actionnaires fournisseurs hellip

Est partie prenante agrave lrsquoentreprise laquo tout groupe ou individu qui peut ecirctre affecteacute ou est

affecteacute par les buts de lrsquoorganisation hellip raquo (Freeman ndash 1984)

Les parties prenantes attendent agrave des degreacutes divers de profiter drsquoune creacuteation de valeur en

provenance de lrsquoentreprise qui doit reacutepondre agrave ces attentes pour assurer sa peacuterenniteacute ou

favoriser son deacuteveloppement

On distingue les parties prenantes primaires ou principales qui sont essentielles agrave lrsquoentreprise

et qui ont geacuteneacuteralement une relation formelle avec elle (clients associeacutes et actionnaires

precircteurs salarieacutes fournisseurs collectiviteacutes) et les parties prenantes secondaires dont

lrsquoinfluence est diffuse (groupes de pression associations meacutedias instances europeacuteennes

agences de notation hellip)

Remarque la consideacuteration de lrsquoensemble des parties prenantes (laquo stakeholders raquo - les

deacutepositaires) fait contrepoids agrave lrsquoimportance accordeacutee aux seuls actionnaires

(laquo shareholders raquo)

Les organisations nrsquoayant pas drsquoobjectif de profit doivent aussi satisfaire leurs parties

prenantes apporter un service aux usagers dans les meilleures conditions eacuteconomiques

limiter un budget assurer la qualiteacute des relations avec les fournisseurs hellip

Dans cette optique le management doit organiser lrsquoaction de faccedilon agrave eacutequilibrer des forces

parfois divergentes

- le contexte fait pression sur lrsquoorganisation contrainte agrave optimiser ses reacutesultats

- lrsquoorganisation cherche par son action agrave assurer sa peacuterenniteacute son deacuteveloppement (en

reacutealisant des profits dans le cas de lrsquoentreprise) et agrave satisfaire ses parties prenantes

- le management agit en pilotant les actions pour contrebalancer la pression du contexte

Actions de

lrsquoorganisation

Management Contexte

Parties

prenantes

Initiation au management copy CRCF ndash J Sornet Page 20 48

APPLICATIONS MP

MP1 Deacutefinir contingent gestion budgeacutetaire

MP2 Deacuteterminer les parties prenantes drsquoun hocircpital public et leurs principales attentes

Mecircme question pour les organisations suivantes

- SNCF (entreprise publique)

- Peugeot

- MAIF (mutuelle drsquoassurance)

MP3 En les situant dans le cycle des activiteacutes du management trouver les actions agrave mener

dans les situations suivantes

- baisse de 10 des ventes dans une entreprise industrielle (produits meacutenagers le reacuteseau de

distribution vient drsquoecirctre reacuteorganiseacute)

- idem dans une entreprise de vente par correspondance soumise agrave la concurrence internet

(les ventes stagnaient depuis six mois malgreacute les efforts promotionnels)

- augmentation des deacutelais drsquoattente des consultations dans une clinique (lrsquohocircpital voisin a

fermeacute son service drsquourgences)

Initiation au management copy CRCF ndash J Sornet Page 21 48

ELEMENTS DE CORRIGE MP

MP1 Deacutefinir (dans le contexte drsquoune entreprise) contingent gestion budgeacutetaire

Contingent = imposeacute par lrsquoexteacuterieur Contingence = effet du hasard de la rencontre de

plusieurs eacuteveacutenements indeacutependants (variables explicatives que lrsquoon ne peut influencer)

Gestion budgeacutetaire = technique drsquoadministration des entreprises srsquoappuyant sur des

preacutevisions dont on deacuteduit apregraves accord des responsables des attributions de moyens sur une

dureacutee limiteacutee Une analyse reacuteguliegravere des eacutecarts entre preacutevisions et reacutealisations permet ensuite

le pilotage des activiteacutes Le budget est un cadre incitatif

La laquo planification budgeacutetaire raquo consiste agrave traduire en budgets une planification strateacutegique

avec systegraveme de reporting

MP2 Deacuteterminer les parties prenantes drsquoun hocircpital public et leurs principales attentes

Mecircme question pour les organisations suivantes

- SNCF (entreprise publique)

- Peugeot

- MAIF (mutuelle drsquoassurance)

Hocircpital

- patients (qualiteacute des soins)

- CNAM (baisse des coucircts)

- collectiviteacute locale (service aux administreacutes)

- eacutetat (ameacutenagement du territoire maicirctrise des budgets optimisation)

- employeacutes (salaire conditions de travail et satisfaction)

- fournisseurs ndash pharmacie autres (CA paiement reacutegulier)

- associations de patients (qualiteacute proximiteacute des soins)

SNCF

- usagers et associations drsquousagers (proximiteacute reacutegulariteacute prix du service)

- reacuteseau ferreacute de France (optimisation des lignes paiement adapteacute)

- fournisseurs (CA paiement reacutegulier)

- employeacutes (salaire conditions de travail seacutecuriteacute de lrsquoemploi)

- eacutetat (ameacutenagement du territoire)

- collectiviteacutes locales (service)

Peugeot

- clients (qualiteacute prix SAV relation commerciale)

- fournisseurs (CA reacutegulariteacute de lrsquoactiviteacute)

- employeacutes (salaire conditions de travail seacutecuriteacute de lrsquoemploi)

- eacutetat (taxes)

- collectiviteacute locale (emploi dynamisation eacuteconomique preacuteservation de lrsquoenvironnement)

- associations de protection de lrsquoenvironnement (activiteacute propre baisse des eacutemissions

nouvelles eacutenergies)

MAIF

- socieacutetaires (protection relation assureur tarif mesureacute)

- professionnels de lrsquoautomobile et autres (agreacutement marge de manœuvre reacuteparations tarifs

eacuteleveacutes)

- fournisseurs (CA paiement reacutegulier)

- eacutetat (taxes engagement pour la seacutecuriteacute)

- employeacutes (salaire conditions de travail seacutecuriteacute de lrsquoemploi)

Initiation au management copy CRCF ndash J Sornet Page 22 48

MP3 En les situant dans le cycle des activiteacutes du management trouver les actions agrave mener

dans les situations suivantes

- baisse de 10 des ventes dans une entreprise industrielle (produits meacutenagers le reacuteseau de

distribution vient drsquoecirctre reacuteorganiseacute)

Adapter le pilotage motiver cadrer si insuffisant retoucher une organisation deacutefectueuse

- idem dans une entreprise de vente par correspondance soumise agrave la concurrence internet

(les ventes stagnaient depuis six mois malgreacute les efforts promotionnels)

Voir pilotage et organisation si une eacutevolution du meacutetier a deacutejagrave eacuteteacute initialiseacutee Sinon re-

conception (adaptation au nouveau contexte) puis planification et reacuteorganisation

- augmentation des deacutelais drsquoattente des consultations dans une clinique (lrsquohocircpital voisin a

fermeacute son service drsquourgences)

Organisation Si insuffisant planification (nouveaux objectifs)

Initiation au management copy CRCF ndash J Sornet Page 23 48

ORGANISATION ET PROCESSUS

La performance de lrsquoentreprise deacutepend de son organisation et de son aptitude agrave produire

aux meilleures conditions Nous allons montrer comment organisation formelle et processus

de production peuvent contribuer agrave cette performance

1 ndash Vers lrsquooptimum

11 ndash Les eacuteconomies occidentales jusqursquoaux anneacutees 70

Jusqursquoen 1945 le principal problegraveme des entreprises eacutetait de produire des biens en quantiteacute

suffisante agrave un prix compatible avec le marcheacute Les grandes entreprises se sont multiplieacutees et

la standardisation a permis de reacuteduire les coucircts (exemple deacuteveloppement de Ford et de la

production agrave la chaicircne de 1908 agrave 1920 qui a permis une baisse du prix des voitures des 23)

On parle de laquo production pousseacutee vers le marcheacute raquo

Cette croissance de la production peu reacuteguleacutee a eacuteteacute marqueacutee par des surproductions en

1910 et 1920 puis par la crise de 1929 qui a prolongeacute ses effets jusqursquoagrave la guerre

De 1945 agrave 1975 environ (les laquo trente glorieuses raquo) la reconstruction la croissance de la

consommation de masse de nouvelles technologies et les eacutechanges internationaux

alimentent lrsquoeacuteconomie La standardisation srsquoeacutetend aux biens de consommation dont les

coucircts baissent fortement et de nouvelles reacutegulations sociales permettent une eacutevolution sans

heurt des revenus La saturation de certains marcheacutes conduit dans les anneacutees 60 agrave la

deacutemarche laquo marketing raquo et agrave la diffeacuterenciation des produits Le produit est laquo dirigeacute par le

marcheacute raquo mais les entreprises conservent une organisation assez classique et les plus grosses

srsquointernationalisent

12 ndash Lrsquoexpeacuterience japonaise et ses prolongements

Tregraves tocirct apregraves la guerre dans un Japon appauvri le constructeur automobile Toyota a ducirc

faire face agrave une restriction du marcheacute des moyens financiers et productifs et des

approvisionnements La firme a donc innoveacute dans un nouveau systegraveme de production

chassant les laquo gaspillages raquo (temps drsquoattente transports stocks deacutefauts hellip) consideacuterant que

seule la fabrication vendable creacutee de la valeur

Toyota srsquoorganise pour fabriquer la quantiteacute et la qualiteacute de produits juste neacutecessaires agrave la

satisfaction des clients la production est laquo tireacutee par le marcheacute raquo La mise en place de ce

systegraveme qui integravegre les fournisseurs ne sera acheveacutee que dans le milieu des anneacutees 70

En 1973 la hausse du peacutetrole inaugure un ralentissement de la croissance des eacuteconomies

occidentales La concurrence accrue provoque alors un inteacuterecirct pour le systegraveme deacuteveloppeacute

au Japon La production au plus juste se deacuteveloppe ainsi dans lrsquoindustrie automobile agrave partir

des anneacutees 80 et elle se reacutepand encore maintenant dans drsquoautres secteurs

Cette approche qui vise un objectif de zeacutero stock et zeacutero deacutefaut impose la maicirctrise de laquo bout

en bout raquo des processus de production et leur ameacutelioration

Initiation au management copy CRCF ndash J Sornet Page 24 48

2 ndash Organiser lrsquoentreprise

21 ndash Direction et organisation

Diriger une entreprise neacutecessite de lrsquoorganiser (de reacutepartir les tacircches) pour qursquoelle puisse

atteindre ses objectifs Lrsquoorganisation permet de satisfaire un marcheacute en tirant parti des

capaciteacutes actuelles de lrsquoentreprise tout en preacuteparant lrsquoavenir

Lrsquoorganisation reacutesulte freacutequemment drsquoun compromis entre des objectifs situeacutes agrave des niveaux

et des eacutecheacuteances diffeacuterents

Exemples

- le leader des chaises roulantes peut tirer profit de sa structure productive et de son

savoir faire pour entrer sur le marcheacute de la bicyclette eacutelectrique

- ecirctre parfaitement structureacute pour alimenter 90 du marcheacute des disquettes ne preacutepare

pas lrsquoavenir

- srsquoorganiser pour conqueacuterir le marcheacute des tire-bouchons eacutelectriques dans les deux ans

perd de son sens si cela altegravere les moyens neacutecessaires agrave la production drsquoappareils

manuels ancienne mais vitale dont la diminution agrave court terme risque de nuire agrave la

solvabiliteacute de lrsquoentreprise et de la conduire agrave la cessation de paiement

22 ndash Lrsquoorganisation fonctionnelle

La majoriteacute des entreprises adopte une laquo organisation fonctionnelle raquo (celle qui est visible

dans les organigrammes) ougrave des regroupements de personnels et drsquoeacutequipements se font

selon un modegravele hieacuterarchique (laquo line raquo) dans des uniteacutes des services ou des deacutepartements

speacutecialiseacutes Cette organisation peut se deacutecliner agrave lrsquointeacuterieur des divisions des grandes

entreprises quand elles scindent leur activiteacute par zone geacuteographique type drsquoactiviteacute

cateacutegorie de clients hellip

Remarque le terme laquo fonction raquo deacutesigne un rocircle particulier dans le fonctionnement de

lrsquoentreprise

Lrsquoorganisation fonctionnelle diffeacuterencie les activiteacutes de lrsquoentreprise en les regroupant par

meacutetier pour utiliser au mieux les compeacutetences et les moyens (meilleur rendement par la

speacutecialisation lrsquoeacutechange de compeacutetences dans une mecircme uniteacute ou gracircce agrave des eacuteconomies

drsquoeacutechelle)

23 ndash La notion de processus de production

Un processus de production se deacutefinit par la succession drsquoactiviteacutes permettant de satisfaire

un client en transformant des ressources (mateacuterielles financiegraveres humaines) en un produit

bien ou service Le processus doit creacuteer une valeur reconnue par le client

Un processus peut servir un client interne agrave lrsquoentreprise (par exemple en produisant un

composant intervenant dans plusieurs produits ou par la maintenance des machines) aussi

bien qursquoun client final On distingue usuellement

- les processus opeacuterationnels (ou maicirctres) aussi appeleacutes processus meacutetier (business process)

qui satisfont directement les clients finaux (conception et fabrication de produits vente hellip)

- les processus de support et de management (geacuterer les ressources humaines geacuterer

lrsquoinformation geacuterer les ressources financiegraveres hellip) qui ont les processus opeacuterationnels comme

clients

Toutes les actions internes agrave une organisation peuvent srsquointeacutegrer dans des processus qui

conditionnent directement ou indirectement la capaciteacute de lrsquoorganisation agrave satisfaire le

client final ou lrsquousager

Initiation au management copy CRCF ndash J Sornet Page 25 48

Aborder le fonctionnement de lrsquoentreprise par ses processus (approche processus) permet

de mettre en eacutevidence les chaicircnes drsquoactiviteacutes qui conduisent aux produits leurs

dysfonctionnements leurs coucircts la formation des deacutelais et la souplesse (la flexibiliteacute)

disponible pour satisfaire la clientegravele finale Lrsquoameacutelioration des processus a un impact visible

et direct sur chaque produit proposeacute aux clients

Lrsquoapproche processus provoque une eacutevolution de la faccedilon de travailler

- en faisant peacuteneacutetrer la laquo voix du client raquo au plus profond de lrsquoentreprise (et plus seulement

dans les services commerciaux et marketing)

- en mettant en eacutevidence des possibiliteacutes de rationalisation (par regroupement ou impartition

de certaines activiteacutes)

Remarque lrsquoapproche par les activiteacutes et les processus est agrave lrsquoorigine de la meacutethode

de deacutetermination des coucircts laquo ABC raquo - activity based costing

24 ndash Processus et fonctions

Le processus est transversal Il enchaicircne des activiteacutes qui traversent lrsquoentreprise en particulier

les services ou les deacutepartements drsquoune organisation fonctionnelle

Exemple

La division du travail par fonctions induit une charge de coordination pour assurer le

deacuteroulement du processus Elle peut geacuteneacuterer des attentes des erreurs ou des conflits drsquointeacuterecirct

(lrsquoobservation montre que des dysfonctionnements sont tregraves souvent constateacutes lors du

passage drsquoun service agrave un autre)

Organisation fonctionnelle et approche processus visent toutes deux un optimum

eacuteconomique mais leurs logiques sont diffeacuterentes

- le processus vise la satisfaction des clients (prix qualiteacute deacutelais service)

- le deacutecoupage fonctionnel cherche agrave optimiser les moyens (maximiser lrsquoeffet drsquoexpeacuterience

partager des infrastructures profiter de pocircles de compeacutetences hellip) Il apporte une ossature

hieacuterarchique stable souvent indispensable

Organisation fonctionnelle et approche processus sont donc compleacutementaires dans la

majoriteacute des cas et doivent ecirctre combineacutees judicieusement

APPLICATIONS OP

OP1 Deacutefinir flexibiliteacute systegraveme impartition

OP2 Citer huit exemples drsquoinformations essentielles pour optimiser un processus de

fabrication

Direction

Deacutepartement

commercial

(C)

Deacutepartement

administratif et

financier (AF)

Deacutepartement

Etudes (E)

Deacutepartement

Production (P)

Activiteacute

C-x Activiteacute

AF-x Activiteacute

E-x

Activiteacute

P-x

Processus x

Clie

nt

Initiation au management copy CRCF ndash J Sornet Page 26 48

OP3 Deacutegager les principes du toyotisme preacutesenteacute ci-dessous En quoi ce systegraveme est-il

initiateur de lrsquoapproche processus

Taiichi Ohno et le Toyotisme

1 - Extrait drsquoun article de Jacques BARRAUX - 1993 - LExpansion

Taiichi Ohno (1912 ndash 1990) hellip ne se prenait pas pour un visionnaire mais en imposant une

nouvelle faccedilon de produire il a reacuteinventeacute le management hellip tout le monde a entendu parler

des mots qui ont populariseacute le toyotisme dont il est le pegravere le juste-agrave-temps hellip Autant

doutils conccedilus pour lrsquoautomobile et qui ont aujourdhui une application universelle

hellip Taiichi Ohno jeune ingeacutenieur entre chez Toyota alors simple constructeur de machines

textiles Degraves 1926 apparaicirct la notion de jidoka hellip cest lart de transfeacuterer de lintelligence aux

machines pour mieux libeacuterer lintelligence des hommes Tout le contraire du taylorisme qui

juge la machine moins impreacutevisible que lhomme En 1933 Toyota se lance dans lautomobile

en sinspirant des meacutethodes ameacutericaines Mais en 1935 agrave loccasion dun voyage aux Etats-

Unis leacutetat-major de lentreprise revient fascineacute de sa visite dans un supermarcheacute La notion

de juste-agrave-temps va naicirctre de lobservation dune grande surface un lieu ougrave les clients ne

prennent que ce dont ils ont besoin et ougrave les rayons sont reacuteapprovisionneacutes pour compenser

les quantiteacutes preacuteleveacutees Ainsi le systegraveme Toyota est-il deacutejagrave dans la tecircte de ses dirigeants avant

mecircme la Seconde Guerre mondiale un demi-siegravecle avant la reacutevolution informatique et la

segmentation intensive des marcheacutes

hellip des esprits curieux comme Franccedilois Dalle en France tombent alors sous le charme des

formules et des paraboles de Taiichi Ohno En voici deux eacutechantillons

Penser agrave lenvers Cela signifie combattre les ideacutees reccedilues En lespegravece il sagit du fordisme et

du taylorisme Ohno ne croit pas agrave la planification aux effets deacutechelle et dexpeacuterience Il

propose un systegraveme industriel agrave lenvers qui permette de diversifier les produits et de les

fabriquer en petites quantiteacutes Nous ne devons plus ecirctre des paysans qui accumulent des

stocks mais des chasseurs On nimpose pas loffre On traque la demande et on la gegravere en

continu

Que les valleacutees soient hautes et les montagnes peu eacuteleveacutees Plutocirct que de concentrer tous

les efforts sur une production agrave un moment donneacute mieux vaut se doter de structures flexibles

permettant de passer agrave tout instant dune seacuterie agrave une autre Il faut eacuteviter les ruptures et les

secousses aplanir les cycles entretenir des flux reacuteguliers dactiviteacutes diversifieacutees Ce qui

implique de ne pas enfermer les hommes et les eacutequipements dans des speacutecialisations trop

eacutetroites

La flexibiliteacute le travail en groupe le refus de la dictature des machines la polyvalence et

surtout lattention constante aux signaux eacutemis par le marcheacute nappartiennent plus au

toyotisme Ces notions sont les fondements du nouvel art dorganiser de vendre et de

produire dans lindustrie comme dans les services hellip

2 - Quelques notions cleacutes

Taiichi Ohno a imagineacute la meacutethode des laquo cinq pourquoi raquo qui consiste agrave se poser cinq fois de

suite la question laquo pourquoi raquo sur le mecircme sujet de faccedilon agrave deacutecouvrir la veacuteritable cause

drsquoun problegraveme Cette meacutethode peut ecirctre appliqueacutee agrave tous les niveaux et permettre

notamment aux agents de fabrication de proposer de veacuteritables ameacuteliorations de la

production

La recherche de la qualiteacute totale (pas de deacutefaut des produits pas de rebuts pas de deacutefaut

des processus) accompagne la deacutemarche de Toyota La qualiteacute a un coucirct compenseacute par

des ventes accrues par lrsquoeacuteconomie des mesures palliatives aux deacutefauts

Initiation au management copy CRCF ndash J Sornet Page 27 48

Fiche OP1 ndash Benchmarking et processus

Le laquo benchmarking raquo consiste agrave comparer le fonctionnement de plusieurs systegravemes pour en

faire notamment ressortir les meilleures pratiques (laquo best practices raquo) Cette technique est

utiliseacutee depuis les anneacutees 80 pour ameacuteliorer la performance des entreprises Elle impose agrave

lrsquoentreprise drsquoeacutevaluer et de remettre en question ses propres modes de fonctionnement afin

de les faire eacutevoluer agrave la lueur de ce qui se fait ailleurs

Le benchmarking permet drsquoameacuteliorer les processus agrave moindre risque en fixant des objectifs

baseacutes sur des faits et donc plus facilement accepteacutes

Une classification des processus en tant que base de reacuteflexion a eacuteteacute eacutetablie aux USA par

lrsquolaquo International Benchmarking Clearinghouse raquo de lrsquoAPQC (american productivity and

quality center) en collaboration avec plusieurs dizaines drsquoentreprises

Elle se reacutesume ainsi

Le terme laquo reengineering raquo (la re-conception ou laquo reacuteingeacutenieacuterie raquo) des processus deacutesigne un

projet drsquoameacutelioration radicale des performances (de 20 agrave 50 ou plus) Il neacutecessite une

parfaite adheacutesion de la direction la constitution drsquoune petite eacutequipe de projet brillante

connaissant parfaitement les activiteacutes de lrsquoentreprise et il peut inclure un benchmarking

Le reengineering provoque geacuteneacuteralement la reacuteduction du nombre de niveaux hieacuterarchiques

(laquo delayering raquo) et lrsquoaccroissement du pouvoir de deacutecision des employeacutes (laquo empowerment raquo

ou laquo empouvoirement raquo) Bien qursquoy conduisant parfois il ne doit pas ecirctre confondu avec la

reacuteduction des activiteacutes (laquo downsizing raquo ou restructuration) et lrsquoexternalisation (laquo outsourcing raquo)

Pro

ce

ssu

s

op

eacutera

tio

nn

els

Pro

ce

ssu

s d

e m

an

ag

em

en

t e

t d

e

sup

po

rt

1 ndash

Comprendre

le marcheacute et

les clients (besoins

satisfaction)

2 ndash

Deacutevelopper

vision et

strateacutegie (contexte

concurrence)

3 ndash

Creacuteer

produits

services

processus

(concevoir

ameacuteliorer)

4 ndash

Marketing et

vente

5 ndash

Produire et

livrer (industrie

dont

ameacutelioration

processus)

6 ndash

Produire et

livrer (services)

7 ndash

Facturer et

servir les

clients (apregraves-

vente

reacuteclamations)

8 ndash Deacutevelopper et geacuterer les ressources humaines

9 ndash Geacuterer les systegravemes drsquoinformation

10 ndash Geacuterer les ressources financiegraveres et les actifs

11 ndash Appliquer un programme environnemental

12 ndash Geacuterer les relations exteacuterieures (actionnaires banques lois relations publiques hellip)

13 ndash Geacuterer lrsquoameacutelioration et le changement (eacutevaluer mesurer motiver qualiteacute totale)

Initiation au management copy CRCF ndash J Sornet Page 28 48

Fiche OP2 ndash Lrsquoorganisation par processus

Lrsquoeacutevolution drsquoune organisation aux activiteacutes reacutepeacutetitives vers lrsquoapproche processus est

geacuteneacuteralement progressive et se met en place par paliers

La mise en œuvre drsquoun veacuteritable management par processus doit ecirctre preacuteceacutedeacutee quand

lrsquoactiviteacute de lrsquoentreprise est complexe drsquoun recensement (une laquo cartographie des

processus raquo) pour mettre en eacutevidence les processus ou les familles de processus cleacutes critiques

pour le succegraves de lrsquoentreprise ougrave les efforts seront prioritaires

Des responsables de processus (laquo process owners raquo) sont ensuite deacutesigneacutes

Le responsable doit concevoir ses processus puis apregraves leur mise en œuvre assurer les

coordinations neacutecessaires les ameacuteliorer et les repreacutesenter aupregraves de la direction

Quand une structure par processus est mise en place des opeacuterateurs exeacutecutants

preacuteceacutedemment regroupeacutes dans les fonctions peuvent ecirctre affecteacutes aux processus et

drsquoanciens responsables de fonctions peuvent devenir des experts au service des processus

Lrsquoorganisation par processus peut imposer un degreacute eacuteleveacute drsquointeacutegration des activiteacutes donc

une polyvalence accrue des personnels et une reacuteduction des niveaux hieacuterarchiques

Elle neacutecessite pour le moins des compeacutetences eacutelargies au niveau des responsables de

processus (organisation administration technique hellip) dont le nombre doit rester limiteacute

(quelques dizaines au plus)

Sauf dans de tregraves petites structures lrsquoorganisation par processus se plaque geacuteneacuteralement sur

une structure plus classique

Initiation au management copy CRCF ndash J Sornet Page 29 48

ELEMENTS DE CORRIGE OP

OP1 Deacutefinir

Flexibiliteacute = adaptation au besoin (horaire variable chaicircnes robotiseacutees)

Systegraveme = ensemble organiseacute dans un but boicircte noire (sanguin nerveux meacutetrique laquo D raquo)

Impartition = sous-traitance ou externalisation (seacuteparation) drsquoactiviteacutes faire appel agrave des

partenaires plutocirct que faire soi-mecircme

OP2 Citer huit exemples drsquoinformations essentielles pour orienter lrsquooptimisation drsquoun processus

Montant des stocks (approvisionnements et produits finis)

Temps drsquoattente

Taux drsquoactiviteacute des ateliers

Rebuts

Deacutelai de production

Taux de reacuteclamations clients (qualiteacute)

Temps passeacutes en retouches finales

Turn over

Nombre drsquoarrecircts maladie

Accidents du travail

Dureacutee des arrecircts machines

OP3 Deacutegager les principes du toyotisme preacutesenteacute dans la fiche 31 En quoi ce systegraveme

repose trsquoil sur lrsquoapproche processus

Produire la quantiteacute juste neacutecessaire (agrave la demande) donc eacuteviter les stocks

Flexibiliteacute intelligence des chaicircnes de production

Qualiteacute (eacuteviter le coucirct de la non-qualiteacute)

La notion de processus est implicite ainsi que la chaicircne de valeur client

Initiation au management copy CRCF ndash J Sornet Page 30 48

DEFIS ET TENDANCES DU MANAGEMENT

Les meacutethodes de management se deacuteveloppent pour affronter le contexte eacuteconomique

Ce chapitre preacutesente les deacutefis auxquels le management contemporain doit faire face

1 ndash Lrsquoeacutevolution eacuteconomique contemporaine

A mesure que lrsquoactiviteacute eacuteconomique mondiale srsquoaccroicirct que la technologie eacutevolue les

changements sont de plus en plus rapides Ils introduisent des situations ineacutedites auxquelles les

entreprises doivent srsquoadapter en cherchant de nouvelles solutions de management Les trois

derniegraveres deacutecennies ont eacuteteacute notamment marqueacutees par les pheacutenomegravenes suivants (que nous

listons sans tenir compte des liens pouvant exister entre eux)

Pheacutenomegravene Traduction Effets

Deacute reacuteglementation

globalisation

financiegravere

titrisation

Libre circulation des capitaux accegraves

facile des particuliers au marcheacute

boursier (directement ou par

lrsquointermeacutediaire des OPCVM et SICAV)

Monteacutee en puissance du financement

des entreprises sur le marcheacute boursier

Fonds de pension

(retraites) et fonds

souverains (eacutetats)

Poids boursier important drsquoinvestisseurs

institutionnels qui cherchent un haut

rendement financier (dividendes ou

valorisation boursiegravere)

Pression sur les grandes entreprises

influence sur les strateacutegies

Mondialisation Liberteacute des eacutechanges internationaux Accroissement de la concurrence

recherche drsquoavantages eacuteconomiques

par la deacutelocalisation (biens et

services) la concentration des efforts

(recentrage) problegravemes drsquoemploi

multiplication des transports perte

drsquoinfluence des politiques

Baisse de lrsquoemploi

occidental

(notamment

industriel)

Moins de fabrications fabrications

automatiseacutees recours aux moyens

informatiques

Activiteacute reporteacutee sur le commerce la

conception et les services chocircmage

charge sociale

Restructurations Optimisation des entreprises

abaissement des coucircts augmentation

des marges recherche drsquoune taille

critique (eacuteconomies drsquoeacutechelle poids

sur le marcheacute)

Recentrages externalisations fusions

deacutelocalisations constitution de grands

groupes

NTIC (nouvelles

technologies de

lrsquoinformation et de

la communication)

Mise en œuvre des reacuteseaux (dont

internet) et drsquoapplications

informatiques communicantes

Nouvelles formes de commerce

marcheacute international deacutelocalisation

du travail intellectuel reacuteorganisation

de la distribution

Rareacutefaction relative

des matiegraveres

premiegraveres

Recherche de substituts exploration

miniegravere coucircts drsquoexploitation des

gisements accrus

Augmentation des coucircts variations

erratiques du cours des matiegraveres

deacutestabilisations politiques

Evolution

geacuteopolitique et

eacuteconomique

mondiale

Chute de lrsquoURSS transformation des

eacuteconomies collectivistes pays

eacutemergents (Chine Inde Breacutesil Russie)

()

Accroissement de la population

mondiale (4 agrave 6 7 milliards de 1970 agrave

2008)

Libeacuteralisme sans frein () nouvelles

puissances eacuteconomiques

opportuniteacutes de deacuteveloppement

nouveau partage des ressources

ineacutegaliteacutes baisse du soutien aux PVD

laquo Terrorisme raquo Actions armeacutees pression de groupes

armeacutes non gouvernementaux

Deacutestabilisations reacutegionales charge

des deacutepenses militaires

Deacuteveloppement

durable

Recherche drsquoune croissance eacutequitable

et respectueuse de lrsquoenvironnement

Pression sur les entreprises (eacutetats

associations de consommateurs

eacutecologistes ONG)

() Reacutecemment quelques affaires (Enron laquo subprimes raquo Vivendi Universal Socieacuteteacute

Geacuteneacuterale Airbus par exemple) et agrave plus grande eacutechelle la crise financiegravere de 2008 ont

montreacute les dangers drsquoune libeacuteralisation sans controcircles suffisants

Initiation au management copy CRCF ndash J Sornet Page 31 48

() Des alliances eacuteconomiques naissent entre pays eacutemergents (notamment en

ameacuterique centrale creacuteation de la Banque du Sud en 2008 par exemple) et lrsquoon

commence agrave imaginer une baisse progressive de lrsquoinfluence eacuteconomique des Etats

Unis

2 ndash Les deacutefis actuels du management

21 ndash Les grandes orientations

Lrsquoeacutevolution eacuteconomique suggegravere quelques pistes parfois concurrentes pour lrsquoaction du

manager contemporain On y retrouve au premier plan la construction drsquoune vision qui est

une composante commune du leadership

Objectif du manager

pour lrsquoentreprise

Justification Facteurs de reacuteussite

Construire une vision Eclairer lrsquoavenir de lrsquoentreprise partager

un but souder motiver

Effort de reacuteelle prospection

volontarisme de la direction

bonne communication

Reacuteactiviteacute et flexibiliteacute

(sous tous les aspects

agrave tous niveaux)

Srsquoadapter rapidement au marcheacute Bonne organisation des processus

personnel compeacutetent autonome

et motiveacute structure hieacuterarchique

alleacutegeacutee robotisation

Deacutegager des profits Reacutemuneacuterer les apporteurs de capitaux

srsquoautofinancer

Ajuster coucircts et structures

Exploiter les nouvelles

technologies

Reacuteactiviteacute ajuster coucircts et deacutelais

reacutepondre au marcheacute suivre les clients

Organiser le SI de faccedilon

pertinente eacuteviter le coucirct excessif

drsquoinvestissements trop en

laquo pointe raquo (laquo essuyer les placirctres raquo)

utiliser judicieusement les services

exteacuterieurs

Bacirctir des alliances

(contrats fusions)

Deacutevelopper une activiteacute limiter les coucircts

de transaction () atteindre la taille

critique et de meilleurs rendements se

recentrer sur une activiteacute profitable

Dominer les processus se donner

une identiteacute lisible externaliser se

doter drsquoune capaciteacute financiegravere

suffisante

Valoriser lrsquoimage Attirer les clients favoriser les alliances

donner confiance (apporteurs de fonds

employeacutes clients partenaires socieacuteteacute

civile)

Instaurer des regravegles de

gouvernance inteacutegrer le

deacuteveloppement durable

respecter lrsquoenvironnement

Geacuterer les risques Faire face aux aleacuteas eacuteconomiques et

technologiques (conjoncture politiques

accidents malveillance)

Creacuteer un systegraveme drsquoalerte geacuterer

la crise (reacuteaction raisonneacutee

sceacutenarios poursuite de

lrsquoexploitation dans un contexte

instable) mise en place de

proceacutedures drsquoapprentissage pour

ameacuteliorer les reacuteactions au fil du

temps

Geacuterer le changement Faire face agrave lrsquoeacutevolution de la demande

la pression sur les prix la variation des

performances financiegraveres la

concurrence la globalisation des

marcheacutes lrsquoeacutevolution technologique aux

fusions ou alliances aux changements

de reacuteglementation de direction hellip ()

Bonne communication pour

donner du sens au changement

et obtenir lrsquoadheacutesion du personnel

Rassembler et geacuterer les

connaissances former le

personnel

Innover Garder un avantage concurrentiel se

diffeacuterencier

Veille technologique et

commerciale investissement

Ouverture

internationale

Elargir le marcheacute saisir les opportuniteacutes Veille commerciale partenariats

() La theacuteorie des coucircts de transaction deacuteveloppeacutee par OE Williamson dans les

anneacutees 70 integravegre les coucircts lieacutes au recours au marcheacute (recherche et choix drsquoun

fournisseur neacutegociation reacutedaction de contrat suivi des eacutechanges risque de rupture

Initiation au management copy CRCF ndash J Sornet Page 32 48

drsquoapprovisionnement hellip) On peut en conclure que lrsquointeacutegration de diffeacuterentes

activiteacutes agrave lrsquoentreprise (la laquo firme raquo) preacutesente des avantages Mais des coucircts de

transaction internes doivent aussi ecirctre consideacutereacutes (preacuteparation organisation

surveillance hellip) et certaines formes de coopeacuteration continue avec les fournisseurs

permettent de reacuteduire le coucirct des transactions externes

() drsquoapregraves laquo Les meilleures pratiques de management raquo - Brilman Heacuterard ndash EO

Une eacutetude du Conference Board (2002) liste les deacutefis du management vus par 700 leaders

mondiaux Soit en reacutesumeacute avec indication du score correspondant

1 ndash Fideacuteliser les clients (42)

2 ndash Reacuteduire les coucircts (38)

3 ndash Accroicirctre flexibiliteacute et reacuteactiviteacute (29)

4 ndash Amener les employeacutes agrave adheacuterer aux valeurs et visions de lrsquoentreprise (26)

5 ndash Deacutevelopper et retenir les leaders (25)

6 ndash Geacuterer acquisitions et alliances (24)

7 ndash Accroicirctre lrsquoinnovation (20)

En fin de classement citoyenneteacute et reacuteputation (4) et ameacutelioration de la diversiteacute (3)

22 ndash Les techniques disponibles

Pour faire face aux deacutefis le manager dispose de nouveaux concepts et de nouvelles

techniques Le tableau ci-dessous en donne un reacutesumeacute et indique les domaines qursquoils

influencent principalement

Initiation au management copy CRCF ndash J Sornet Page 33 48

Principaux concepts techniques outils Incidence principale sur

Internet

- e-commerce (commerce eacutelectronique site

entreprise)

- CRM ou GRC (gestion de la relation client)

- e-procurement (gestion des approvisionnements

par le reacuteseau)

- messagerie eacutelectronique

- e-recrutement

Vente accegraves au marcheacute

Relation client reacuteactiviteacute personnalisation

fideacutelisation

Deacutelais coucircts

Communication transfert de donneacutees (piegraveces

jointes) tous domaines

Communication recrutement

Intranet reacuteseau drsquoentreprise SI

- knowledge management (gestion des

connaissances)

- e-learning (apprentissage en ligne)

- plateforme de travail collaboratif (groupware)

- workflow (circulation eacutelectronique de

documents enchaicircnement de processus)

- e-RH portail RH (libre accegraves aux postes agrave

pourvoir informations candidatures hellip)

- PGI (progiciel de gestion inteacutegreacute) ou ERP

Innovation capaciteacute au changement veille

documentaire

Formation du personnel accompagnement des

changements

Coordination communication interne

Coordination

Communication interne (voire internet en

externe) reacuteduction des coucircts climat drsquoentreprise

recrutement plans de carriegraveres hellip

Coucircts fiabiliteacute du systegraveme drsquoinformation deacutelais

processus (continuiteacute inteacutegration)

Logistique inteacutegreacutee

Supply Chain Management (SCM) gestion de la

logistique (incluant les approvisionnements)

Processus deacutelais coucircts

Externalisation

Valorisation du capital humain

GPEC (gestion preacutevisionnelle des emplois et

compeacutetences)

Coaching

Reacuteactiviteacute de lrsquoentreprise conservation des

compeacutetences rendements individuels turn-over

adaptation des compeacutetences motivation

Efficaciteacute individuelle controcircle reacutegulation

progregraves processus

Approche processus

Optimisation des processus

Deacutemarche qualiteacute totale (TQM ndash total quality

management)

Empowerment (empouvoirement)

Benchmarking reacuteingeacutenieacuterie

Coucircts marges qualiteacute deacutelais flexibiliteacute

externalisation eacutelargissement des compeacutetences

organisation

Ameacutelioration des processus (meacutetiers et supports)

Autonomie compeacutetences des employeacutes

Ameacutelioration des processus restructuration

Management par la valeur

Parties prenantes

Satisfaction des parties prenantes financement

motivation collaborations hellip

Collaboration inter organisations

Reacuteseaux drsquoentreprises alliances

EDI (eacutechange de donneacutees informatiseacutees) extranet

Impartition externalisation (outsourcing)

Coucircts recentrage investissements lancement

drsquoactiviteacute

Coucircts reacuteactiviteacute deacutelais relations avec

lrsquoadministration

Coucircts recentrage limitation des investissements

Ethique drsquoentreprise

Gouvernance drsquoentreprise (mode de direction

encadreacute par des regravegles)

Rocircle socieacutetal deacuteveloppement durable

environnement

Image de lrsquoentreprise reacutegulation du top

management relations actionnaires

Image peacutenaliteacutes et amendes objectifs

strateacutegiques

Initiation au management copy CRCF ndash J Sornet Page 34 48

23 ndash Le rocircle socieacutetal des entreprises

La responsabiliteacute socieacutetale de lrsquoentreprise (RSE) deacutesigne le rocircle qursquoelle prend dans la socieacuteteacute

au-delagrave de son activiteacute purement geacuteneacuteratrice de profit On parle aussi drsquoentreprise citoyenne

La RSE est indissociable du deacuteveloppement durable de porteacutee mondiale et dont les trois

piliers sont

- eacuteconomique (favoriser le deacuteveloppement les eacutechanges internationaux)

- social (accegraves aux soins eacuteducation conditions de travail hellip)

- environnemental (pollution preacuteservation des ressources hellip)

La RSE integravegre notamment une preacuteoccupation sociale de lrsquoentreprise vis-agrave-vis de ses salarieacutes

(seacutecuriteacute et santeacute au travail juste reacutemuneacuteration deacuteveloppement personnel hellip) Elle conduit agrave

tenir compte dans le management drsquoune vision exteacuterieure agrave lrsquoentreprise qui peut avoir des

reacutepercussions possibles sur son activiteacute eacuteconomique

Lrsquoentreprise peut aussi tirer avantage drsquoune deacutemarche responsable par la baisse de certains

coucircts (plus faibles consommations drsquoeacutenergies reacuteduction des transports hellip)

Le rocircle socieacutetal de lrsquoentreprise a eacuteteacute reconnu en France par la loi laquo NRE raquo de 2001 (loi sur les

nouvelles reacutegulations eacuteconomiques) qui oblige les socieacuteteacutes franccedilaise coteacutees sur un marcheacute

reacuteglementeacute agrave rendre compte dans leur rapport annuel de leur gestion sociale et

environnementale au travers de leur activiteacute

Article 116 de la loi Le rapport viseacute agrave larticle L 225-102 rend compte hellip laquo Il comprend

eacutegalement des informations dont la liste est fixeacutee par deacutecret en Conseil dEtat sur la

maniegravere dont la socieacuteteacute prend en compte les conseacutequences sociales et

environnementales de son activiteacute Le preacutesent alineacutea ne sapplique pas aux socieacuteteacutes

dont les titres ne sont pas admis aux neacutegociations sur un marcheacute reacuteglementeacute raquo

Une norme ISO 14000 integravegre ces preacuteoccupations et des taxes eacutecologiques sont

progressivement creacutees

3 ndash Le management par la valeur

31 ndash De lrsquoanalyse au management par la valeur

Lrsquoanalyse de la valeur est neacutee en 1947 aux Etats-Unis (General Electrics) Cette technique

consiste agrave eacutelaborer des produits conformes aux attentes de la clientegravele mais sans excegraves pour

trouver un bon compromis entre valeur pour le client et coucirct Le produit optimal est deacutefini agrave

partir drsquoenquecirctes qui deacuteterminent le besoin client (ou plutocirct drsquoun client laquo type raquo)

Exemple il est inutile de concevoir un petit veacutehicule citadin capable de parcourir

500 000 km sans avarie compte tenu des effets de mode et du faible kilomeacutetrage

annuel Par contre le marcheacute peut exiger un fonctionnement sans faille sur 150 000 km

soit dix ans en moyenne ce qui conditionne les coucircts de production

Cette recherche drsquoun ajustement de valeur au besoin des clients eacutetait un preacutecurseur du

management par la valeur qui recherche plus largement la creacuteation de valeur pour

chacune des parties prenantes de lrsquoentreprise tout en lui meacutenageant un reacutesultat suffisant

Plus geacuteneacuteralement le management par la valeur est deacutefini par une norme europeacuteenne (EN

12973)

Le management par la valeur est un style de management particuliegraverement destineacute agrave

mobiliser les individus agrave deacutevelopper les compeacutetences et agrave promouvoir les synergies et

Initiation au management copy CRCF ndash J Sornet Page 35 48

linnovation avec pour objectif la maximisation de la performance globale dun

organisme Le management par la valeur apporte une nouvelle faccedilon dutiliser nombre

de meacutethodes de management existantes Il est en coheacuterence avec le Management

de la qualiteacute

Cette approche du management pose de nombreuses questions notamment quelles

prioriteacutes et quelles valeurs attribuer aux parties prenantes comment appreacutehender la

perception par les parties prenantes de la valeur qui leur est affecteacutee

32 ndash La valeur client

Le processus drsquoeacutelaboration drsquoun produit qui consomme des ressources coucircteuses doit creacuteer

une valeur suffisante pour provoquer lrsquoachat par le client final La production drsquoune valeur

reconnue par le client est vitale pour lrsquoentreprise mais sa deacutetermination est parfois complexe

La valeur du produit perccedilue par le client integravegre des eacuteleacutements en partie subjectifs

- une valeur drsquousage (le produit reacutepond agrave un besoin)

- une valeur drsquoestime (lrsquoimage apporteacutee par le produit un aspect affectif)

- une valeur drsquoeacutechange (deacuteduite de lrsquoespoir de revente du produit)

Valeurs drsquousage drsquoestime et drsquoeacutechange deacutependent implicitement de la qualiteacute (un bien peu

fiable est impropre agrave lrsquousage attendu de mauvaise qualiteacute notoire il nrsquoapporte pas une

image positive et ses deacutefauts connus nuisent agrave sa revente) Une eacutevaluation de la qualiteacute

intervient donc dans la valeur perccedilue du produit

Par ailleurs le client considegravere le coucirct drsquoobtention du produit (les charges qursquoil doit supporter

pour acqueacuterir le produit lrsquoeffort qursquoil doit faire pour trouver le produit et les frais de mise agrave

disposition)

Le prix perccedilu par le client est geacuteneacuteralement supeacuterieur au prix de vente

Le client achegravete theacuteoriquement le produit qui preacutesente la diffeacuterence valeur perccedilue ndash prix

perccedilu la plus favorable ou le meilleur rapport prix perccedilu qualiteacute perccedilue et dans certains

cas celui qui a le prix produit le plus bas

Remarque les valeurs du scheacutema ci-dessus changent durant le cycle de vie du produit

(un nouveau produit peut avoir une valeur perccedilue plus eacuteleveacutee qursquoen fin de vie) La

valeur client ne peut ecirctre eacutevalueacutee que par enquecirctes et ne peut donc ecirctre deacutefinie avec

certitude

La notion de laquo satisfaction client raquo conseacutecutive agrave une vente influence aussi le prix produit et

le prix perccedilu

- lrsquoentreprise gagne sur les coucircts de recherche de clientegravele

- le client nrsquoa pas agrave rechercher un nouveau fournisseur et beacuteneacuteficie drsquoun coucirct drsquoobtention

plus bas

valeur perccedilue client

prix perccedilu client

coucirct produit Marge (valeur creacuteeacutee pour

lrsquoentreprise)

euros

prix produit

Valeur creacuteeacutee

pour le client

Initiation au management copy CRCF ndash J Sornet Page 36 48

La satisfaction du client deacutepend de facteurs qualitatifs aussi divers que la fiabiliteacute du produit

la vitesse de reacuteaction du fournisseur lrsquoattitude des commerciaux lrsquoefficaciteacute du service

apregraves-vente la netteteacute des contrats ou la justesse de la facture

Valeur perccedilue coucirct marge et satisfaction reacutesultent de processus allant de la conception du

produit jusqursquoagrave sa livraison et son apregraves-vente La deacutemarche laquo processus raquo et lrsquolaquo analyse de la

valeur raquo en forccedilant la recherche de solutions efficientes agrave tout niveau administratif

technique commercial et apregraves-vente sont donc neacutecessaires pour bien positionner

lrsquoentreprise sur son marcheacute

Pour autant le risque commercial ne peut jamais ecirctre annuleacute et lrsquooffre de lrsquoentreprise ne

satisfait geacuteneacuteralement pas en milieu concurrentiel tous ses clients potentiels

33 - La creacuteation de valeur pour les autres parties prenantes

Les salarieacutes

La creacuteation drsquoune valeur suffisante pour les salarieacutes est reconnue comme neacutecessaire car des

observations montrent que la satisfaction des clients en deacutepend Moins souvent eacutevoqueacutee en

peacuteriode de chocircmage elle nrsquoest prioritaire que pour les employeacutes dont lrsquoentreprise souhaite

conserver les compeacutetences

La laquo valeur salarieacute raquo ne comprend pas que le salaire Le sentiment drsquoappartenance agrave un

groupe la reconnaissance lrsquoaccomplissement de soi et la construction professionnelle en

sont des eacuteleacutements importants Comme pour les clients on doit ainsi distinguer la reacutetribution

perccedilue du salaire objectif

Les actionnaires

Lrsquoactionnaire apporte des fonds propres agrave lrsquoentreprise en contrepartie de titres parfois

neacutegociables en bourse et assortis drsquoun droit de vote en assembleacutee geacuteneacuterale La valeur

attribueacutee aux actionnaires est servie en termes moneacutetaires (dividende ou augmentation de la

valeur du titre neacutegociable)

Remarque des facteurs non moneacutetaires comme lrsquoimage de lrsquoentreprise qui deacutepend

en partie de sa communication peuvent influencer la deacutecision drsquoachat de vente ou

de conservation des titres par lrsquoactionnaire

Reacutetribution perccedilue euros

Salaire objectif

Avantage non

moneacutetaire de

lrsquoemploi

Initiation au management copy CRCF ndash J Sornet Page 37 48

Compte tenu de lrsquoimportance croissante de lrsquoactionnariat dans le financement des grandes

entreprises coteacutees en bourse et notamment des investisseurs institutionnels comme les fonds

de pension des indicateurs speacutecifiques ont eacuteteacute introduits pour appreacutecier la performance des

entreprises vue par les actionnaires Par exemple la valeur ajouteacutee eacuteconomique (EVA reg

economic value added marque deacuteposeacutee de Stern Stewart ou VAE ndash valeur ajouteacutee

eacuteconomique parfois deacutenommeacutee VEC ndash valeur eacuteconomique creacuteeacutee) qui prend en compte le

coucirct du capital

LrsquoEVA correspond tregraves scheacutematiquement au calcul suivant

EVA = (PO) profit opeacuterationnel ndash (C) coucirct du capital X (CE) capitaux employeacutes

LrsquoEVA neacutecessite en pratique des retraitements assez complexes Le PO peut se deacuteterminer

selon les principes suivants

- PO = reacutesultat drsquoexploitation (avant inteacuterecircts) ndash impocirct

- PO = beacuteneacutefice courant (tenant compte des inteacuterecircts) + inteacuterecircts ndash eacuteconomie drsquoimpocirct sur les

inteacuterecircts (on exclue les eacuteleacutements financiers et lrsquoimpocirct correspondant) ndash impocirct

- lrsquoimpocirct pris en compte correspond au profit opeacuterationnel consideacutereacute (dans les cas courants agrave

13 du PO)

C = taux moyen de reacutemuneacuteration du capital (reacutesultant par exemple du dividende exigeacute de

certains investisseurs et des taux drsquoemprunts bancaires)

CE = capitaux propres et dettes portant inteacuterecirct

Remarque le profit opeacuterationnel ou reacutesultat opeacuterationnel correspond au NOPAT ndash net

operating profit after tax - anglo-saxon LrsquoEVA est eacutegale au NOPAT diminueacute de la

reacutemuneacuteration des capitaux

Exemple lrsquoentreprise X dispose drsquoun capital de 2 500 000 euro et reacutealise un beacuteneacutefice net

drsquoimpocirct de 450 000 euro (taux 33 13) Un dividende de 6 doit ecirctre verseacute aux

actionnaires et la banque lui a accordeacute un precirct de 1 200 000 euro agrave 4 Les autres

constituants des reacutesultats financier et exceptionnel sont neacutegligeables

Reacutesultat opeacuterationnel = 450 000 + 004 x 1 200 000 x 23 = 482 000 euro

Coucirct du capital = 006 x 2 500 000 + 004 x 1 200 000 x 23 = 182 000 euro

EVA = 300 000 euro

Coucirct moyen pondeacutereacute du capital (C) = (004 x 1 200 000 x23 + 006 x 2 500 000)

3 700 000 Soit 492

Si lrsquoEVA est positive lrsquoentreprise creacuteeacutee de la valeur apregraves reacutemuneacuteration des capitaux et sa

valeur boursiegravere doit augmenter

Lrsquoutilisation de lrsquoEVA comme indicateur influence le management de lrsquoentreprise car il y a

trois moyens pratiques drsquoaugmenter lrsquoEVA

- augmenter le reacutesultat opeacuterationnel

- lancer des investissements ayant une rentabiliteacute supeacuterieure agrave C

- eacuteliminer les activiteacutes ayant une rentabiliteacute infeacuterieure agrave C

Remarque lrsquoutilisation sans nuance de lrsquoEVA comme critegravere de management peut

poser problegraveme Le calcul de lrsquoEVA repose sur des ajustements comptables il est donc

sujet agrave manipulations (provisions capitalisation ou non de la RD hellip) Par ailleurs le

critegravere laquo EVA raquo pris isoleacutement peut conduire agrave chercher la rentabiliteacute agrave court terme agrave

reacuteduire les investissements prospectifs et donc nuire agrave terme au deacuteveloppement de

lrsquoentreprise

Initiation au management copy CRCF ndash J Sornet Page 38 48

Les fournisseurs reccediloivent le paiement de leurs factures plus ou moins rapidement (le deacutelai

de paiement repreacutesente une valeur consentie au fournisseur)

Lrsquoentreprise peut accroicirctre la valeur apporteacutee agrave ses fournisseurs par des actions cibleacutees

comme une contribution agrave la formation de leurs personnels certains transferts de

technologie ou de savoir faire agrave des sous-traitants une coopeacuteration suivie favorisant leur

deacuteveloppement lrsquointeacutegration agrave des campagnes de promotion

A noter que la valeur consentie aux fournisseurs peut avoir une influence sur la qualiteacute et les

deacutelais de livraison des produits

La collectiviteacute reccediloit des taxes et parfois des prestations en nature par deacutefaut ou explicites

(effort de preacuteservation de lrsquoenvironnement ameacutenagement du territoire par les implantations

aide mateacuterielle agrave des projets participation agrave la formation par exemple)

APPLICATIONS DT

DT1 Deacutefinir expliquer deacutereacuteglementation socieacutetal eacuteconomies drsquoeacutechelle coaching EDI

gouvernance

DT2 Deacuteterminer en quoi la deacutemarche TQM srsquoinscrit dans les deacutefis actuels du management

DT3 Apregraves avoir consulteacute les documents ci-dessous extraits du site drsquoAir France

(httpdeveloppement-

durableairfrancecomFRfrlocaldemarcheN4_positionnement_pphtm)

exposer les enjeux et les limites de la RSE et de la gestion des parties prenantes

Initiation au management copy CRCF ndash J Sornet Page 39 48

Dialogue avec les parties prenantes

Initiation au management copy CRCF ndash J Sornet Page 40 48

Attentes des parties prenantes

Initiation au management copy CRCF ndash J Sornet Page 41 48

Creacuteation de valeur pour les parties prenantes

La creacuteation de valeur pour les parties prenantes est au cœur de la strateacutegie du Groupe Le scheacutema de

distribution financiegravere ci-dessous donne un aperccedilu de la distribution des recettes du Groupe aux

diffeacuterentes parties prenantes actionnaires collaborateurs fournisseurs pouvoirs publics

collectiviteacutes locales etc

Initiation au management copy CRCF ndash J Sornet Page 42 48

Fiche DT1 ndash Extrait du sommaire de laquo Problegravemes eacuteconomiques raquo No 2894

La gestion des entreprises bouleverseacutee par les technologies de linternet

Reacutealiteacutes industrielles - Annales des Mines Jean-Michel Yolin

Avec lavegravenement de linternet les processus de conception de production et de vente sont

radicalement remis en cause Quel que soit le secteur dactiviteacute les technologies de linternet

permettent en effet de reacuteduire les deacutelais et de passer dun processus discontinu agrave un processus

continu Lorganisation des entreprises et leur mode de gestion en sont profondeacutement bouleverseacutes

tant au niveau individuel que collectif Linternet rend ainsi possible la reacutealisation dobjectifs que les

entreprises cherchaient agrave atteindre depuis longtemps sans y parvenir meilleure eacutecoute du client

travail sans stocks en flux tendu hieacuterarchies plates autorisant une grande reacuteactiviteacute flexibiliteacute dans

lorganisation et loutil de production acceacuteleacuteration du renouvellement des produits entreprises en

reacuteseau ougrave chacune se recentre sur son cœur de meacutetier etc

Le laquo knowledge management raquo ou comment geacuterer les connaissances

Document de travail du LAMSADE - Michel Grundstein

Peter Drucker lavait preacutedit le capital immateacuteriel eacutetait voueacute agrave devenir un facteur de compeacutetitiviteacute

pour lentreprise La libeacuteralisation des eacutechanges acceacutelegravere les processus de deacutecision de lentreprise

et implique que lassimilation des informations soit agrave la fois de meilleure qualiteacute et plus rapide Ainsi

la fonction qui consiste agrave manager les connaissances au sein de lentreprise savegravere primordiale

Bien que la prise de conscience de limportance du capital immateacuteriel ait eacuteteacute tardive - le concept

de knowledge management est apparu en France aux Etats-Unis et au Japon au milieu des

anneacutees 1990 - agrave lheure actuelle lorganisation de leacutechange dinformations et le partage des

connaissances sont devenus des facteurs cleacutes dune gestion performante de lentreprise Ils

doivent sinscrire dans un projet global destineacute agrave mettre en valeur les savoirs et les savoir-faire

individuels et collectifs

Les leccedilons du laquo coaching raquo pour le management de la qualiteacute

Humanisme et Entreprise - Martine Brasseur

Parmi les nouvelles formes de management en vogue dans les entreprises le coaching figure en

bonne place Appliqueacute au management de la qualiteacute il sagit dune pratique

daccompagnement destineacutee agrave initier et agrave faciliter le processus de deacuteveloppement dun individu

La deacutemarche consiste agrave affirmer que tout individu est en quecircte de qualiteacute agrave condition toutefois

de ne pas lui imposer des contraintes lempecircchant de progresser On considegravere notamment les

erreurs comme potentiellement feacutecondes En deacutefinitive le coach donne au coacheacute la permission

de reacuteussir en lui donnant aussi la permission deacutechouer

Initiation au management copy CRCF ndash J Sornet Page 43 48

Fiche DT2 ndash Management strateacutegique les sept deacutefis agrave relever dici agrave 2016

Extrait drsquoun article du site wwwlentreprisecom -Sabine Blanc - Mis en ligne le 20032007

(httpwwwlentreprisecom325article11977html)

Une eacutetude anglaise publieacutee par lopeacuterateur Orange Grande-Bretagne deacutecrypte la mutation

des formes de travail et les enjeux majeurs pour les entreprises de demain afin decirctre au top

de la compeacutetitiviteacute Voici les challenges-cleacutes pour les managers qui veulent rester dans la

course hellip

1 - Future organisation du travail les quatre laquo mondes raquo possibles

La reacutealiteacute sera probablement un meacutelange de ces quatre sceacutenarios souligne lrsquoeacutetude

Les mondes mutuels Tout se passe dans le cadre des communauteacutes locales vie priveacutee

comme professionnelle Le modegravele coopeacuteratif preacutevaut au lieu du laquo big business raquo Oublieacutes

aussi dans ce systegraveme les trajets pour aller au bureau les gens preacutefegravereront travailler dans de

petites entreprises locales souvent connecteacutees au reacuteseau drsquoautres structures similaires

Les laquo reacutepondants raquo (en anglais laquo replicants raquo) La figure du consultant freelance deviendra

dominante tandis que celle du salarieacute deacuteclinera Il ne sera pas rare de travailler pour plusieurs

entreprises On perdra en seacutecuriteacute de lrsquoemploi en visibiliteacute et en routine ce que lrsquoon gagnera

en liberteacute La majeure partie des tacircches srsquoeffectuera chez soi avec la possibiliteacute de srsquoinstaller

temporairement dans les bureaux de son client du moment Dans un contexte dincertitude

sur lrsquoavenir les travailleurs alterneront peacuteriodes drsquoactiviteacute intense et repos Ce sera agrave eux

drsquoaller vers les entreprises et non lrsquoinverse mecircme si celles-ci devront veiller agrave rester attractives

Les cottages eacutelectroniques Comme ce nom le suggegravere le teacuteleacutetravail deviendrait la norme

univers priveacute et professionnel se confondant Plus besoin de subir une heure de transport les

salarieacutes se logueront de chez eux sur le reacuteseau de lrsquoentreprise Les reacuteunions se tiendront dans

de petits bureaux centraux situeacutes agrave courte distance La flexibiliteacute du temps de travail srsquoimpose

Les salarieacutes disposeront de plus de marge de liberteacute dans leur activiteacute

Les disciples de la nueacutee Cette appellation poeacutetique cache simplement une extension de

lrsquoorganisation actuelle des grandes entreprises avec des salarieacutes se rendant sur un lieu de

travail centraliseacute Le rocircle croissant des technologies de lrsquoinformation multipliera les faccedilons de

collaborer et accroicirctra lrsquoefficaciteacute Le controcircle du travail sera omnipreacutesent La frontiegravere entre

travail et vie priveacutee restera marqueacutee

2 - Sept deacutefis pour les entreprises et leur managers

Quoi qursquoil advienne les entreprises et leurs dirigeants devront concentrer leurs efforts sur sept

points-cleacutes pour srsquoadapter Voici quelques exemples de probleacutematiques souleveacutees par le

rapport et des pistes de solution

Le leadership Les managers devront entre autres savoir persuader et influencer des

travailleurs beaucoup plus indeacutependants Ils auront aussi agrave repenser les niveaux auxquels

prendre les deacutecisions strateacutegiques en haut ou au contraire agrave des degreacutes moins eacuteleveacutes de la

pyramide hieacuterarchique

gt Faire du management une force facilitant les activiteacutes transversales plutocirct que la reacuteduire agrave

la seule fonction de deacutecision

La culture drsquoentreprise Davantage de salarieacutes capables de reacutefleacutechir seront neacutecessaires

tandis que les tacircches qui peuvent ecirctre automatiseacutees ou scripteacutees diminueront Un des

enjeux creacuteer une culture agrave mecircme drsquoattirer et drsquoencourager les personnes preacutesentant ces

qualiteacutes de reacuteflexion requises dans un contexte de compeacutetition accrue et de plus grande

indeacutependance des travailleurs

Initiation au management copy CRCF ndash J Sornet Page 44 48

gt Passer si neacutecessaire drsquoune culture drsquoentreprise forte agrave un mode drsquoengagement plus

consensuel moins rebutant

La marque Conseacutequence du recours croissant agrave lrsquo laquo outsourcing raquo lrsquoimage drsquoune marque

deacutependra plus drsquoagents exteacuterieurs qui ne fonctionnent pas forceacutement selon le mecircme mode

drsquoorganisation Comment garder le controcircle dessus

gt Choisir le mode qui corresponde le plus agrave vos valeurs et preacutevoir un programme de risk

management qui mette en eacutevidence ougrave les conflits sont susceptibles de jaillir

Lrsquoinnovation Plus que jamais il faudra faire face agrave une acceacuteleacuteration du rythme de

lrsquoinnovation en proposant constamment des solutions adapteacutees

gt Tisser des partenariats strateacutegiques avec drsquoautres entreprises pour partager les coucircts et les

fruits de lrsquoinnovation

Le deacutefi opeacuterationnel et technologique De quelle faccedilon controcircler lrsquoinformation crsquoest-agrave-dire

faire en sorte que les bonnes personnes accegravedent facilement agrave une information toujours en

phase tout en maintenant la seacutecuriteacute

gt Recourir agrave des laquo feuilles de route des futurs raquo syntheacutetisant en une page les indicateurs

sociaux et de consommation ainsi que les eacutevolutions technologiques et leacutegislatives qui

influent sur les changements et indiquant comment ils modifient vos marcheacutes vos clients et

votre organisation

La qualiteacute Si de nouveaux proceacutedeacutes ont pu deacutegrader la qualiteacute comme le recours agrave des

centres drsquoappel externaliseacutes drsquoautres ideacutees se sont reacuteveacuteleacutees plus prometteuses comme en

teacutemoigne le succegraves de certaines compagnies aeacuteriennes low cost Elles ont su conjuguer prix

serreacutes et services eacuteleveacutes ce qui devra devenir la norme estime lrsquoeacutetude

gt Continuer de rechercher la qualiteacute Elaborez aussi une bonne prestation service qui inclut

une livraison de qualiteacute voire creacuteez-la en partenariat avec les consommateurs

La leacutegislation La question de la proprieacuteteacute intellectuelle pourrait ecirctre probleacutematique Elle est

deacutejagrave source de conflits comme en teacutemoigne le procegraves pour violation de brevet intenteacute agrave RIM

le fabricant canadien du Blackberry par NTP Que pourra-t-on et que faudra-t-il proteacuteger par

un brevet Il sera eacutegalement neacutecessaire drsquoadapter la leacutegislation aux nouveaux modes

drsquoorganisation

gt Collaborer avec les acteurs du mecircme secteur et les leacutegislateurs pour deacutevelopper les

modegraveles des lieux de travail du futur et bacirctir le droit le plus adeacutequat

Orange a-t-il vu juste dans ses preacutevisions Rendez-vous dans neuf ans pour la reacuteponsehellip

Initiation au management copy CRCF ndash J Sornet Page 45 48

Fiche DT3 ndash Le management par la qualiteacute totale

Extrait drsquoune lettre drsquoinformation du cabinet Baud Accordance Consulting AD2 consultants ndash

2002

1 - Le TQM (Total Quality Management) offre pour lentreprise une vision de la qualiteacute plus

large et transversale

Son principe est simple La finaliteacute de lEntreprise est de deacutevelopper la satisfaction de ses

clients tout en eacutetant beacuteneacuteficiaire cest agrave dire pas agrave nimporte quel prix Elle doit ameacuteliorer sa

rentabiliteacute au travers de la deacutemarche qualiteacute La Qualiteacute Totale vise agrave fournir aux clients

externes et internes une reacuteponse adeacutequate agrave leurs attentes dans le meilleur rapport qualiteacute

prix la meilleure efficience

Elle considegravere pour cela lensemble des processus de lentreprise ayant une incidence sur la

qualiteacute et la satisfaction des clients

Le TQM fait ainsi une large place agrave

la deacutefinition et la planification de la strateacutegie geacuteneacuterale

la coheacuterence de la politique qualiteacute avec la strateacutegie

la deacutemultiplication de la politique qualiteacute dans toutes les directions de lentreprise

la relation client fournisseur interne

la prise en compte de lenvironnement concurrentiel

la consideacuteration de lensemble des risques potentiels financiers sociaux concurrentielshellip

limplication et la motivation du personnel

lanalyse des besoins des clients et le positionnement marketing

la maicirctrise des processus transverses internes

les reacutesultats sous tous ses aspects y compris financiers commerciaux image

De nombreux reacutefeacuterentiels sont relatifs agrave la Qualiteacute Totale hellip Tous ces reacutefeacuterentiels imposent un

questionnement plus profond et indiscret sur le mode de fonctionnement de lentreprise et

son management

helliphellip

2 - LISO 9001 2000 au travers du deacuteploiement des processus (management supports

reacutealisation et ameacutelioration continue) reacutepond quelque peu agrave la mecircme logique

LISO est une ouverture indeacuteniable vers la logique du TQM mais ne se reacutefegravere pas agrave la notion

defficience

Les dirigeants sont cependant sensibles agrave la neacutecessaire reacuteduction des coucircts de non-qualiteacute

et dobtention de la qualiteacute agrave la rentabiliteacute du systegraveme de management de la qualiteacute

mais ne perccediloivent pas toujours la qualiteacute comme une deacutemarche globale

Les deacutemarches qualiteacute commencent bien souvent par la remise en cause de lorganisation

leacutevaluation critique de son efficaciteacute lexamen des processus et la mise en eacutevidence des

lourdeurs administratives

La qualiteacute devient laffaire de tous hellip

Initiation au management copy CRCF ndash J Sornet Page 46 48

Fiche DT4 ndash Le deacuteveloppement durable et la RSE

Extrait du site wwwvigeocom

(httpwwwvigeocomcsr-rating-agencyfrmethodologiecriteres-de-recherche37-

criteres-d-analysehtml)

Deacuteveloppement durable laquo un deacuteveloppement qui reacutepond aux besoins du preacutesent sans compromettre

la capaciteacute des geacuteneacuterations futures de reacutepondre aux leurs raquo (Commission mondiale sur lrsquoenvironnement

et le deacuteveloppement ndash 1987)

Reacutefeacuterentiel drsquoeacutevaluation des entreprises par le groupe Vigeacuteo (le groupe mesure les performances et le

niveau de maicirctrise des risques de responsabiliteacute sociale des entreprises et des organisations - site

wwwvigeocom)

1 Ressources Humaines Ameacutelioration continue des relations professionnelles des relations drsquoemploi et des conditions de travail 2 Droits humains sur les lieux de travail Respect de la liberteacute syndicale et promotion de la neacutegociation collective non discrimination et promotion de lrsquoeacutegaliteacute eacutelimination des formes de travail proscrites (enfants travail forceacute) preacutevention des traitements inhumains ou deacutegradants de type harcegravelements sexuels protection de la vie priveacutee et des donneacutees personnelles 3 Environnement Protection sauvegarde preacutevention des atteintes agrave lenvironnement mise en place drsquoune strateacutegie manageacuteriale approprieacutee eacuteco conception protection de la biodiversiteacute et maicirctrise rationnelle des impacts environnementaux sur lrsquoensemble du cycle de vie des produits ou services

4 Comportements sur les marcheacutes Prise en compte des droits et inteacuterecircts des clients inteacutegration de standards sociaux et environnementaux dans la seacutelection des fournisseurs et sur lrsquoensemble de la chaicircne drsquoapprovisionnement preacutevention effective de la corruption respect des regravegles concurrentielles 5 Gouvernement drsquoentreprise Efficience et probiteacute assurance de lrsquoindeacutependance et de lrsquoefficaciteacute du Conseil drsquoadministration effectiviteacute et efficience des meacutecanismes drsquoaudit et de controcircle et notamment inclusion des risques de responsabiliteacute sociale respect des droits des actionnaires et notamment des minoritaires transparence et rationaliteacute de la reacutemuneacuteration des dirigeants 6 Engagement socieacutetal Effectiviteacute inteacutegration manageacuteriale de lrsquoengagement contribution au deacuteveloppement eacuteconomique et social des territoires drsquoimplantation et de leurs communauteacutes humaines engagements concrets en faveur de la maicirctrise des impacts socieacutetaux des produits et des services contribution transparente et participative agrave des causes drsquointeacuterecirct geacuteneacuteral

Initiation au management copy CRCF ndash J Sornet Page 47 48

ELEMENTS DE CORRIGE DT DT1 Deacutefinir expliquer

Deacutereacuteglementation = suppression des contraintes eacuteconomiques (libre eacutechange des biens et

capitaux)

Socieacutetal = qui se rapporte agrave la structure agrave lrsquoorganisation ou au fonctionnement de la socieacuteteacute

Economies drsquoeacutechelle = reacuteduction des coucircts lieacutee au niveau drsquoactiviteacute (amortissement des

charges fixes)

Coaching = accompagnement de personnes ou deacutequipes pour le deacuteveloppement de leurs

potentiels

EDI = eacutechange de donneacutees informatiseacutees ET standardiseacutees (ex SWIFT bancaire edifact

documents deacuteclaratifs)

Gouvernance = exercice du pouvoir la bonne gouvernance est participative et eacutequitable

conforme agrave lrsquointeacuterecirct commun

DT2 Deacuteterminer en quoi la deacutemarche TQM srsquoinscrit dans les deacutefis actuels du management

Voir notamment fiche 43

Maicirctrise des processus reacuteduction des coucircts reacuteactiviteacute et satisfaction de la clientegravele = faire

face agrave la concurrence

Ameacutelioration de lrsquoimage motivation du personnel

DT3 Apregraves avoir consulteacute les documents ci-dessous extraits du site drsquoAir France

(httpdeveloppement-

durableairfrancecomFRfrlocaldemarcheN4_positionnement_pphtm)

exposer les enjeux et les limites de la RSE et de la gestion des parties prenantes

Trame geacuteneacuterale possible

Introduction

Les deacutefis contemporains (accroissement de la concurrence devenue mondiale recherche

de nouveaux avantages concurrentiels pression de la socieacuteteacute besoin drsquoimage et de projet

lisible pour mener lrsquoentreprise crise et scandales du libeacuteralisme hellip) RSE et PP

Deacuteveloppement (voir cours)

1 ndash Parties prenantes et management par la valeur

PP deacutefinir citer reacutesumer lrsquoavantage rechercheacute (fideacuteliser motiver recherche drsquoalliances

implicites)

PP moyens (dont exemples AF) et meacutethode de management par la valeur (reacutepartie)

2 ndash La responsabiliteacute socieacutetale de lrsquoentreprise

RSE 3 axes

- eacuteconomique (favoriser le deacuteveloppement les eacutechanges internationaux)

- social (accegraves aux soins eacuteducation conditions de travail hellip)

- environnemental (pollution preacuteservation des ressources hellip)

RSE gouvernance drsquoentreprise facteur drsquoimage inteacutegrable dans la deacutemarche PP

Article 116 de la loi Le rapport viseacute agrave larticle L 225-102 rend compte hellip laquo Il comprend

eacutegalement des informations dont la liste est fixeacutee par deacutecret en Conseil dEtat sur la maniegravere

dont la socieacuteteacute prend en compte les conseacutequences sociales et environnementales de son

activiteacute Le preacutesent alineacutea ne sapplique pas aux socieacuteteacutes dont les titres ne sont pas admis aux

neacutegociations sur un marcheacute reacuteglementeacute raquo

Initiation au management copy CRCF ndash J Sornet Page 48 48

RSE exemple AF (ONG fournisseurs)

3 ndash Liens entre PP et RSE

- la RSE introduit de nouvelles PP

- la RSE suppose le respect des PP usuelles (employeacutes clients notamment)

4 - Probleacutematique

- deacutefinir la valeur reacuteellement apporteacutee par une gestion des PP (confusion salaire ndash valeur

idem impocircts hellip ex laquo valeur ajouteacutee raquo)

- communication (neacutecessaire mais aller au-delagrave)

- marginaliteacute des deacutepenses RSE (efficaciteacute sinceacuteriteacute de lrsquoengagement marge de manœuvre)

- charge RSE reporteacutee sur des tiers (ex fournisseurs AF)

- inteacutegration de facteurs non visibles en comptabiliteacute (pertes drsquoemploi nuisances hellip)

Conclusion

Voies incontournables mais pouvant nrsquoavoir qursquoun effet superficiel et temporaire Voir utiliteacute

drsquoaccompagnement leacutegislatif de regravegles de gouvernance

Initiation au management copy CRCF ndash J Sornet Page 14 48

ELEMENTS DE CORRIGE IM

IM1 Commenter la deacutefinition du management par la norme ISO et le manager de Mintzberg

Efficient = optimum avec les moyens disponibles

ISO (management objectifs) (manager moyens) HM

IM2 Le leader entraicircne naturellement derriegravere lui Le manager nrsquoest pas toujours leader

(mecircme si crsquoest souhaitable) Le leader nrsquoest pas toujours manager (plutocirct notion individuelle)

Leadership = faculteacute de diriger conjugaison drsquoune autoriteacute naturelle ou drsquoun savoir-faire

acquis drsquoune capaciteacute agrave entraicircner des personnes ou des groupes et drsquoune leacutegitimiteacute

statutaire (de position)

IM3 Compleacuteter le tableau ci-dessous en analysant chaque action preacutesenteacutee Faire ensuite

ressortir les domaines niveaux ou techniques de management pouvant ecirctre mobiliseacutes pour

chaque situation

Satisfaction client

Implication du personnel

Processus systegraveme

Ameacutelioration continue

Deacutecision efficace

Recherche de valeur

Image entreprise

Liaisons

Information

Reacutepartition ressources

Reacutegulation

Neacutegociation

Leadership

Initiation au management copy CRCF ndash J Sornet Page 15 48

Caracteacuteristiques

de lrsquoaction

- reacutepeacutetition

- risque

- normes

- ampleur

Prise de

deacutecision

- opeacuteration

- direction

- deacutelai

Informations

neacutecessaires

- nature

- origine

- deacutelai

obtention

Cleacute pour la

reacuteussite

Intervention

exteacuterieure

possible

Assurer la

restauration du

soir

(Restaurant

familial)

Technique

(fabrication)

Vente (terrain)

Appros

Reacutepeacutetitive

(quot)

Risque faible

Normes

drsquohygiegravene

Faible

Opeacuterationnelle

Geacuterant

responsable

Rapide (qq

jours menu et

appros)

Nombre de

couverts

Tarifs usuels

Calendrier

(fecirctes)

Clients docs

divers

expeacuterience

Qq jours

Varieacuteteacute menu

Plats phares

Accueil

Appros

Tarification

Vins

Gestion

congeacutelation

Qualiteacute cuisine

Fournisseurs

Extra

Publiciteacute

Construire un

viaduc

(autoroute)

Technique

Organisation

Appros

Uniteacute (ou peu)

Eleveacute (financier

technique)

Architecture

Eleveacutee

Direction

(aleacuteas)

Opeacuterationnelle

(conduite

chantier)

Immeacutediat agrave qq

semaines

Plans

plannings

Qualifications

Meacuteteacuteo

Disponibiliteacutes

Bureau eacutetudes

Qq sem agrave 24h

Techniciteacute

Appros

Qualifications

Preacutevision

GRH

Contrat juste

SS traitants

Organismes

certificateurs

Controcircle

client

Certifier les

comptes

annuels drsquoun

groupe national

(cabinet

drsquoaudit)

Technique

Relation client

Gestion des

connaissances

Annuelle

Moyen

Regravegles

comptables

fiscales

Moyenne (selon

importance du

cabinet)

Opeacuterationnelle

Qq jours agrave

semaines

Comptable

Juridique

Client

Etat

Qq jours agrave

semaines

Techniciteacute

Expeacuterience

Relation client

Systegraveme info client

Siegravege

Autre cabinet

Lancer une

ligne drsquoavions

(constructeur

aeacuteronautique)

Strateacutegique

RD

Etudes

Uniteacute

Tregraves eacuteleveacute

Aeacuteronautique

Tregraves eacuteleveacutee

Direction

Qq mois agrave

anneacutees

Marcheacute

Etudes

Compagnies

Qq mois agrave

anneacutees

Concept

Outil industriel

Coucirct exploitation

Tarif

Fiabiliteacute

Deacutelaisconcurrence

SI simulation

SS traitants

Bureaux

drsquoeacutetudes

speacutecialiseacutes

Compagnies

Conseils

Reacuteduire la

capaciteacute de

production

(groupe

industriel)

Strateacutegique

RH

Communication

Production

Uniteacute

Moyen

Leacutegislation

(dont RH)

Eleveacutee

Direction

Qq mois agrave

anneacutees

Financiegravere

Industrielle

Marcheacute

Organisation

Organismes

speacutecialiseacutes

DRH

Qq mois

Communication

Connaissance des

compeacutetences

Connaissance outil

industriel

Concurrence

Portefeuille

drsquoactiviteacutes

Cabinet

drsquoorganisation

Conseils

speacutecifiques

Acqueacuterir une

entreprise

concurrente

(teacuteleacutephonie

mobile)

Strateacutegique

Marketing

Production

(reacuteseau)

Financier

Communication

Uniteacute

Tregraves eacuteleveacute

Leacutegislation

telecom

Tregraves eacuteleveacutee

Direction

Qq mois

Financiegravere

Marcheacute

Reacuteseaux

(ampleur

recouvrement

hellip)

Organisations

Interne

Racheteacutee

Sources

speacutecialiseacutees

Qq mois

Communication

Marcheacute

Cours boursiers

Cabinet

drsquoorganisation

Conseils

speacutecifiques

Initiation au management copy CRCF ndash J Sornet Page 16 48

LE MANAGEMENT EN PRATIQUE

Pour assumer sa fonction le management doit couvrir sans discontinuiteacute lrsquoensemble de

lrsquoorganisation et inteacutegrer de nombreux facteurs dont nous allons reacutesumer lrsquoessentiel

1 ndash Les fonctions et activiteacutes du management

Pour Henri Fayol la fonction drsquoadministration de lrsquoentreprise (son management) reposait sur

cinq actions preacutevoir organiser commander coordonner et controcircler (laquo PO3C raquo)

Nous distinguerons cinq activiteacutes de management

- la conception (au plus haut niveau finaliteacute but ou vocation de lrsquoorganisation

meacutetiers dimension politique de croissance hellip)

- la planification (deacutefinition des objectifs eacutecheacuteances)

- lrsquoorganisation (reacutepartition du travail choix des modes de coordination)

- le pilotage de lrsquoaction opeacuterationnelle (motivation animation encadrement

assistance)

- lrsquoeacutevaluation (controcircle des reacutesultats obtenus ajustements)

Dans chacune de ces activiteacutes des deacutecisions et des arbitrages sont neacutecessaires avec des

enjeux plus ou moins importants

Remarques

- Les cinq activiteacutes du management peuvent se retrouver agrave tout niveau de

management si lrsquoentreprise laisse une certaine autonomie de deacutecision agrave ses diffeacuterentes

uniteacutes La conception est naturellement du ressort de la direction geacuteneacuterale et des

conseils drsquoadministration mais elle peut ecirctre preacutesente pregraves du terrain (latitude laisseacutee agrave

une filiale ou agrave un magasin par exemple) De mecircme lrsquoorganisation du travail concerne

un atelier mais aussi la direction qui structure lrsquoentreprise pour assurer ses activiteacutes sa

production

- La planification deacutefinit des objectifs ou des axes strateacutegiques (choix de produits

modaliteacutes de deacuteveloppement des ventes implantations alliances hellip) et les traduit en

donneacutees de gestion preacutevisionnelles syntheacutetiques et eacutechelonneacutees dans le temps afin de

valider les objectifs et de fixer des repegraveres

- Un laquo business plan raquo (plan drsquoaffaires)est notamment lrsquoeacutequivalent de la planification

dans le cas de creacuteation drsquoentreprise ou pour la preacutesentation de tout projet drsquoactiviteacute

Les activiteacutes du management srsquoinscrivent dans des cycles qui peuvent ecirctre scheacutematiseacute

comme suit (lrsquoeacutevaluation peut entraicircner une reacutevision du pilotage de lrsquoorganisation ou des

objectifs sans que lrsquoentreprise ne soit fondamentalement remise en cause)

conception

planification

organisation

pilotage

eacutevaluation

Initiation au management copy CRCF ndash J Sornet Page 17 48

2 ndash Les contextes de management

Le management est influenceacute par son contexte qui justifie des objectifs une organisation

des meacutethodes

Par exemple lrsquoentreprise admet de nombreuses variantes selon sa taille sa forme juridique

son controcircle par lrsquoeacutetat (entreprises publiques) ou par des inteacuterecircts priveacutes Il en va de mecircme des

organismes administratifs qui peuvent deacutependre de directives nationales ou reacutegionales des

associations qui ont des activiteacutes drsquoampleur tregraves variable

21 ndash La dimension de lrsquoentreprise

La dimension drsquoune entreprise se mesure principalement en fonction de son effectif ou de

son chiffre drsquoaffaires Des seuils sont deacutefinis par divers organismes et exploiteacutes agrave des fins

statistiques ou pour la deacutetermination de certaines obligations sociales ou fiscales

(repreacutesentation du personnel cotisations hellip) Il nrsquoy a bien entendu pas de laquo barriegravere de

tailleraquo absolue conditionnant le management drsquoune entreprise

LrsquoUE preacuteconise de distinguer les micro ndash entreprises (jusqursquoagrave 9 salarieacutes) les TPE ndash tregraves petites

entreprises (moins de 20 salarieacutes) les petites entreprises (moins de 50) et les moyennes

entreprises (de 50 agrave 250) Cependant les PME sont parfois situeacutees entre 10 et 500 salarieacutes

Remarques

- en France environ 40 des entreprises emploient de 1 agrave 50 salarieacutes (ce qui repreacutesente

plus de 50 des emplois) et 59 nrsquoen ont aucun

le pays compte environ 2 600 000 entreprises dont moins de 1 ont 250 employeacutes et

plus

- ancienneteacute et taille de lrsquoentreprise sont lieacutees si lrsquoon eacutecarte les restructurations et autres

eacutevolutions drsquoentreprises existantes

La dimension de lrsquoentreprise a une influence sur lrsquoorganisation et le laquo style raquo de son

management

- les PME sont souvent entrepreneuriales (les dirigeants eacutegalement apporteurs de capitaux

sont totalement engageacutes dans la marche de lrsquoentreprise) Elles ont une gestion flexible peu

formaliseacutee plus qualitative que quantitative Les PME sont freacutequemment focaliseacutees sur un seul

type drsquoactiviteacute Pour ne pas alourdir leur structure elles ont tendance agrave sous-traiter les

activiteacutes speacutecialiseacutees ne correspondant pas agrave leur meacutetier de base

- les grandes entreprises sont manageacuteriales (les dirigeants sont nommeacutes par les actionnaires

en raison de leurs compeacutetences) et moins reacuteactives

22 ndash Le type de production

On distingue industrie (production de biens mateacuteriels ou pour le moins de produits visibles ndash

comme un seacutejour touristique ou un film) et services (fourniture drsquoune prestation immateacuterielle)

Le type de production influence en principe le management de lrsquoentreprise

- lrsquoindustrie neacutecessite (si lrsquoon excepte lrsquoartisanat) un investissement relativement important

une organisation productive stable capable de reacutealiser plusieurs fois des produits identiques

(exemple un modegravele de reacutefrigeacuterateur) ou du moins similaires (exemple un bacirctiment) Le

produit de lrsquoindustrie consomme des matiegraveres et il doit geacuteneacuteralement ecirctre distribueacute jusqursquoau

client

- la production de services peut se satisfaire drsquoun investissement tregraves reacuteduit et neacutecessite un

contact permanent avec le client

Toutefois la standardisation des services et le deacuteveloppement des reacuteseaux informatiques

rapprochent la production de services de celle des biens industriels

- la production drsquoun service reacutepeacutetitif et technique peut imposer une structure lourde et une

organisation tregraves formaliseacutee (voir les grandes socieacuteteacutes drsquoaudit ou de conseil informatique)

Initiation au management copy CRCF ndash J Sornet Page 18 48

- certains services peuvent ecirctre fournis agrave distance sans contact direct avec le client et

distribueacutes par reacuteseau (tenue de comptabiliteacute affacturage gestion clientegravele centre drsquoappel

hellip)

Remarque les services repreacutesentent 75 de lrsquoactiviteacute eacuteconomique franccedilaise

23 ndash La nature de lrsquoorganisation

Les organisations publiques franccedilaises (administrations centrales collectiviteacutes territoriales

hocircpitaux hellip) repreacutesentent une part importante de lrsquoactiviteacute (environ 30 des emplois) La

fonction publique regroupe des organisations aux finaliteacutes diverses et qui ont des problegravemes

de gestion similaires agrave ceux des entreprises auxquelles elles peuvent emprunter des principes

de management Notamment

- pour controcircler les coucircts et assurer la qualiteacute des services

- pour communiquer avec les administreacutes ou les usagers

- pour motiver les personnels et geacuterer les ressources humaines

La transposition directe des techniques de gestion et de management nrsquoest cependant pas

toujours possible car

- la comptabiliteacute publique obeacuteit agrave des regravegles speacutecifiques (proceacutedure budgeacutetaire

notamment)

- le laquo client raquo ne paye pas toujours la prestation du moins directement

- la concurrence est parfois inexistante

- les grandes administrations centraliseacutees sont soumises agrave des choix politiques geacuteneacuteraux

parfois sans connexion eacutevidente avec les besoins opeacuterationnels

- le statut des personnels et les grilles de salaires limitent les possibiliteacutes de gestion des

ressources humaines

Remarque la LOLF (loi organique relative aux lois de finances) est entreacutee en vigueur en

2006 Elle alloue des moyens budgeacutetaires en fonction de programmes et remplace la

reconduction automatique de 90 des budgets Cette reacuteforme se heurte toutefois agrave la

lourdeur des grands ministegraveres ougrave la complexiteacute des activiteacutes est difficile agrave

appreacutehender et ougrave des inerties culturelles peuvent exister agrave tout niveau

Les associations loi de 1901 peuvent avoir une activiteacute comparable agrave celle de grandes

entreprises (voir par exemple les associations de santeacute ou professionnelles) et leur

management est alors similaire malgreacute lrsquoabsence de but lucratif (les beacuteneacutefices ne sont pas

distribuables) Elles ont drsquoailleurs en France un poids eacuteconomique important (elles emploient

environ 1 600 000 salarieacutes)

Cependant lrsquoadheacutesion agrave un systegraveme de valeurs fondateur de lrsquoassociation ou la limite de

lrsquoautoriteacute (quand un volant de beacuteneacutevoles important participe agrave lrsquoactiviteacute) peut introduire des

nuances

- le renforcement des objectifs socieacutetaux

- la faiblesse des relations hieacuterarchiques

- des contraintes de gestion du temps des beacuteneacutevoles

- des modaliteacutes particuliegraveres de recrutement et de motivation des dirigeants

24 ndash Les facteurs contingents

La theacuteorie de la contingence montre qursquoune structure drsquoentreprise nrsquoest efficace que dans

une situation deacutetermineacutee et qursquoil nrsquoexiste que des solutions de management construites dans

un contexte preacutecis

Le management doit ainsi srsquoadapter agrave des facteurs contingents qui ne peuvent ecirctre

controcircleacutes du moins agrave bregraveve eacutecheacuteance Ces facteurs sont par exemple

- lrsquoancienneteacute de lrsquoentreprise (plus elle est ancienne plus lrsquoentreprise a tendance agrave reacutepeacuteter

des comportements eacuteprouveacutes)

Initiation au management copy CRCF ndash J Sornet Page 19 48

- la taille de lrsquoentreprise (la grande entreprise a une composante administrative plus

deacuteveloppeacutee)

- le systegraveme de production (tregraves standardiseacute complexe automatiseacute hellip)

- lrsquoenvironnement

3 ndash Le management et les parties prenantes

Lrsquoentreprise a pour vocation premiegravere de mettre des produits agrave disposition de ses clients en

reacutealisant un profit Pour y arriver elle doit aussi satisfaire ses parties prenantes salarieacutes

actionnaires fournisseurs hellip

Est partie prenante agrave lrsquoentreprise laquo tout groupe ou individu qui peut ecirctre affecteacute ou est

affecteacute par les buts de lrsquoorganisation hellip raquo (Freeman ndash 1984)

Les parties prenantes attendent agrave des degreacutes divers de profiter drsquoune creacuteation de valeur en

provenance de lrsquoentreprise qui doit reacutepondre agrave ces attentes pour assurer sa peacuterenniteacute ou

favoriser son deacuteveloppement

On distingue les parties prenantes primaires ou principales qui sont essentielles agrave lrsquoentreprise

et qui ont geacuteneacuteralement une relation formelle avec elle (clients associeacutes et actionnaires

precircteurs salarieacutes fournisseurs collectiviteacutes) et les parties prenantes secondaires dont

lrsquoinfluence est diffuse (groupes de pression associations meacutedias instances europeacuteennes

agences de notation hellip)

Remarque la consideacuteration de lrsquoensemble des parties prenantes (laquo stakeholders raquo - les

deacutepositaires) fait contrepoids agrave lrsquoimportance accordeacutee aux seuls actionnaires

(laquo shareholders raquo)

Les organisations nrsquoayant pas drsquoobjectif de profit doivent aussi satisfaire leurs parties

prenantes apporter un service aux usagers dans les meilleures conditions eacuteconomiques

limiter un budget assurer la qualiteacute des relations avec les fournisseurs hellip

Dans cette optique le management doit organiser lrsquoaction de faccedilon agrave eacutequilibrer des forces

parfois divergentes

- le contexte fait pression sur lrsquoorganisation contrainte agrave optimiser ses reacutesultats

- lrsquoorganisation cherche par son action agrave assurer sa peacuterenniteacute son deacuteveloppement (en

reacutealisant des profits dans le cas de lrsquoentreprise) et agrave satisfaire ses parties prenantes

- le management agit en pilotant les actions pour contrebalancer la pression du contexte

Actions de

lrsquoorganisation

Management Contexte

Parties

prenantes

Initiation au management copy CRCF ndash J Sornet Page 20 48

APPLICATIONS MP

MP1 Deacutefinir contingent gestion budgeacutetaire

MP2 Deacuteterminer les parties prenantes drsquoun hocircpital public et leurs principales attentes

Mecircme question pour les organisations suivantes

- SNCF (entreprise publique)

- Peugeot

- MAIF (mutuelle drsquoassurance)

MP3 En les situant dans le cycle des activiteacutes du management trouver les actions agrave mener

dans les situations suivantes

- baisse de 10 des ventes dans une entreprise industrielle (produits meacutenagers le reacuteseau de

distribution vient drsquoecirctre reacuteorganiseacute)

- idem dans une entreprise de vente par correspondance soumise agrave la concurrence internet

(les ventes stagnaient depuis six mois malgreacute les efforts promotionnels)

- augmentation des deacutelais drsquoattente des consultations dans une clinique (lrsquohocircpital voisin a

fermeacute son service drsquourgences)

Initiation au management copy CRCF ndash J Sornet Page 21 48

ELEMENTS DE CORRIGE MP

MP1 Deacutefinir (dans le contexte drsquoune entreprise) contingent gestion budgeacutetaire

Contingent = imposeacute par lrsquoexteacuterieur Contingence = effet du hasard de la rencontre de

plusieurs eacuteveacutenements indeacutependants (variables explicatives que lrsquoon ne peut influencer)

Gestion budgeacutetaire = technique drsquoadministration des entreprises srsquoappuyant sur des

preacutevisions dont on deacuteduit apregraves accord des responsables des attributions de moyens sur une

dureacutee limiteacutee Une analyse reacuteguliegravere des eacutecarts entre preacutevisions et reacutealisations permet ensuite

le pilotage des activiteacutes Le budget est un cadre incitatif

La laquo planification budgeacutetaire raquo consiste agrave traduire en budgets une planification strateacutegique

avec systegraveme de reporting

MP2 Deacuteterminer les parties prenantes drsquoun hocircpital public et leurs principales attentes

Mecircme question pour les organisations suivantes

- SNCF (entreprise publique)

- Peugeot

- MAIF (mutuelle drsquoassurance)

Hocircpital

- patients (qualiteacute des soins)

- CNAM (baisse des coucircts)

- collectiviteacute locale (service aux administreacutes)

- eacutetat (ameacutenagement du territoire maicirctrise des budgets optimisation)

- employeacutes (salaire conditions de travail et satisfaction)

- fournisseurs ndash pharmacie autres (CA paiement reacutegulier)

- associations de patients (qualiteacute proximiteacute des soins)

SNCF

- usagers et associations drsquousagers (proximiteacute reacutegulariteacute prix du service)

- reacuteseau ferreacute de France (optimisation des lignes paiement adapteacute)

- fournisseurs (CA paiement reacutegulier)

- employeacutes (salaire conditions de travail seacutecuriteacute de lrsquoemploi)

- eacutetat (ameacutenagement du territoire)

- collectiviteacutes locales (service)

Peugeot

- clients (qualiteacute prix SAV relation commerciale)

- fournisseurs (CA reacutegulariteacute de lrsquoactiviteacute)

- employeacutes (salaire conditions de travail seacutecuriteacute de lrsquoemploi)

- eacutetat (taxes)

- collectiviteacute locale (emploi dynamisation eacuteconomique preacuteservation de lrsquoenvironnement)

- associations de protection de lrsquoenvironnement (activiteacute propre baisse des eacutemissions

nouvelles eacutenergies)

MAIF

- socieacutetaires (protection relation assureur tarif mesureacute)

- professionnels de lrsquoautomobile et autres (agreacutement marge de manœuvre reacuteparations tarifs

eacuteleveacutes)

- fournisseurs (CA paiement reacutegulier)

- eacutetat (taxes engagement pour la seacutecuriteacute)

- employeacutes (salaire conditions de travail seacutecuriteacute de lrsquoemploi)

Initiation au management copy CRCF ndash J Sornet Page 22 48

MP3 En les situant dans le cycle des activiteacutes du management trouver les actions agrave mener

dans les situations suivantes

- baisse de 10 des ventes dans une entreprise industrielle (produits meacutenagers le reacuteseau de

distribution vient drsquoecirctre reacuteorganiseacute)

Adapter le pilotage motiver cadrer si insuffisant retoucher une organisation deacutefectueuse

- idem dans une entreprise de vente par correspondance soumise agrave la concurrence internet

(les ventes stagnaient depuis six mois malgreacute les efforts promotionnels)

Voir pilotage et organisation si une eacutevolution du meacutetier a deacutejagrave eacuteteacute initialiseacutee Sinon re-

conception (adaptation au nouveau contexte) puis planification et reacuteorganisation

- augmentation des deacutelais drsquoattente des consultations dans une clinique (lrsquohocircpital voisin a

fermeacute son service drsquourgences)

Organisation Si insuffisant planification (nouveaux objectifs)

Initiation au management copy CRCF ndash J Sornet Page 23 48

ORGANISATION ET PROCESSUS

La performance de lrsquoentreprise deacutepend de son organisation et de son aptitude agrave produire

aux meilleures conditions Nous allons montrer comment organisation formelle et processus

de production peuvent contribuer agrave cette performance

1 ndash Vers lrsquooptimum

11 ndash Les eacuteconomies occidentales jusqursquoaux anneacutees 70

Jusqursquoen 1945 le principal problegraveme des entreprises eacutetait de produire des biens en quantiteacute

suffisante agrave un prix compatible avec le marcheacute Les grandes entreprises se sont multiplieacutees et

la standardisation a permis de reacuteduire les coucircts (exemple deacuteveloppement de Ford et de la

production agrave la chaicircne de 1908 agrave 1920 qui a permis une baisse du prix des voitures des 23)

On parle de laquo production pousseacutee vers le marcheacute raquo

Cette croissance de la production peu reacuteguleacutee a eacuteteacute marqueacutee par des surproductions en

1910 et 1920 puis par la crise de 1929 qui a prolongeacute ses effets jusqursquoagrave la guerre

De 1945 agrave 1975 environ (les laquo trente glorieuses raquo) la reconstruction la croissance de la

consommation de masse de nouvelles technologies et les eacutechanges internationaux

alimentent lrsquoeacuteconomie La standardisation srsquoeacutetend aux biens de consommation dont les

coucircts baissent fortement et de nouvelles reacutegulations sociales permettent une eacutevolution sans

heurt des revenus La saturation de certains marcheacutes conduit dans les anneacutees 60 agrave la

deacutemarche laquo marketing raquo et agrave la diffeacuterenciation des produits Le produit est laquo dirigeacute par le

marcheacute raquo mais les entreprises conservent une organisation assez classique et les plus grosses

srsquointernationalisent

12 ndash Lrsquoexpeacuterience japonaise et ses prolongements

Tregraves tocirct apregraves la guerre dans un Japon appauvri le constructeur automobile Toyota a ducirc

faire face agrave une restriction du marcheacute des moyens financiers et productifs et des

approvisionnements La firme a donc innoveacute dans un nouveau systegraveme de production

chassant les laquo gaspillages raquo (temps drsquoattente transports stocks deacutefauts hellip) consideacuterant que

seule la fabrication vendable creacutee de la valeur

Toyota srsquoorganise pour fabriquer la quantiteacute et la qualiteacute de produits juste neacutecessaires agrave la

satisfaction des clients la production est laquo tireacutee par le marcheacute raquo La mise en place de ce

systegraveme qui integravegre les fournisseurs ne sera acheveacutee que dans le milieu des anneacutees 70

En 1973 la hausse du peacutetrole inaugure un ralentissement de la croissance des eacuteconomies

occidentales La concurrence accrue provoque alors un inteacuterecirct pour le systegraveme deacuteveloppeacute

au Japon La production au plus juste se deacuteveloppe ainsi dans lrsquoindustrie automobile agrave partir

des anneacutees 80 et elle se reacutepand encore maintenant dans drsquoautres secteurs

Cette approche qui vise un objectif de zeacutero stock et zeacutero deacutefaut impose la maicirctrise de laquo bout

en bout raquo des processus de production et leur ameacutelioration

Initiation au management copy CRCF ndash J Sornet Page 24 48

2 ndash Organiser lrsquoentreprise

21 ndash Direction et organisation

Diriger une entreprise neacutecessite de lrsquoorganiser (de reacutepartir les tacircches) pour qursquoelle puisse

atteindre ses objectifs Lrsquoorganisation permet de satisfaire un marcheacute en tirant parti des

capaciteacutes actuelles de lrsquoentreprise tout en preacuteparant lrsquoavenir

Lrsquoorganisation reacutesulte freacutequemment drsquoun compromis entre des objectifs situeacutes agrave des niveaux

et des eacutecheacuteances diffeacuterents

Exemples

- le leader des chaises roulantes peut tirer profit de sa structure productive et de son

savoir faire pour entrer sur le marcheacute de la bicyclette eacutelectrique

- ecirctre parfaitement structureacute pour alimenter 90 du marcheacute des disquettes ne preacutepare

pas lrsquoavenir

- srsquoorganiser pour conqueacuterir le marcheacute des tire-bouchons eacutelectriques dans les deux ans

perd de son sens si cela altegravere les moyens neacutecessaires agrave la production drsquoappareils

manuels ancienne mais vitale dont la diminution agrave court terme risque de nuire agrave la

solvabiliteacute de lrsquoentreprise et de la conduire agrave la cessation de paiement

22 ndash Lrsquoorganisation fonctionnelle

La majoriteacute des entreprises adopte une laquo organisation fonctionnelle raquo (celle qui est visible

dans les organigrammes) ougrave des regroupements de personnels et drsquoeacutequipements se font

selon un modegravele hieacuterarchique (laquo line raquo) dans des uniteacutes des services ou des deacutepartements

speacutecialiseacutes Cette organisation peut se deacutecliner agrave lrsquointeacuterieur des divisions des grandes

entreprises quand elles scindent leur activiteacute par zone geacuteographique type drsquoactiviteacute

cateacutegorie de clients hellip

Remarque le terme laquo fonction raquo deacutesigne un rocircle particulier dans le fonctionnement de

lrsquoentreprise

Lrsquoorganisation fonctionnelle diffeacuterencie les activiteacutes de lrsquoentreprise en les regroupant par

meacutetier pour utiliser au mieux les compeacutetences et les moyens (meilleur rendement par la

speacutecialisation lrsquoeacutechange de compeacutetences dans une mecircme uniteacute ou gracircce agrave des eacuteconomies

drsquoeacutechelle)

23 ndash La notion de processus de production

Un processus de production se deacutefinit par la succession drsquoactiviteacutes permettant de satisfaire

un client en transformant des ressources (mateacuterielles financiegraveres humaines) en un produit

bien ou service Le processus doit creacuteer une valeur reconnue par le client

Un processus peut servir un client interne agrave lrsquoentreprise (par exemple en produisant un

composant intervenant dans plusieurs produits ou par la maintenance des machines) aussi

bien qursquoun client final On distingue usuellement

- les processus opeacuterationnels (ou maicirctres) aussi appeleacutes processus meacutetier (business process)

qui satisfont directement les clients finaux (conception et fabrication de produits vente hellip)

- les processus de support et de management (geacuterer les ressources humaines geacuterer

lrsquoinformation geacuterer les ressources financiegraveres hellip) qui ont les processus opeacuterationnels comme

clients

Toutes les actions internes agrave une organisation peuvent srsquointeacutegrer dans des processus qui

conditionnent directement ou indirectement la capaciteacute de lrsquoorganisation agrave satisfaire le

client final ou lrsquousager

Initiation au management copy CRCF ndash J Sornet Page 25 48

Aborder le fonctionnement de lrsquoentreprise par ses processus (approche processus) permet

de mettre en eacutevidence les chaicircnes drsquoactiviteacutes qui conduisent aux produits leurs

dysfonctionnements leurs coucircts la formation des deacutelais et la souplesse (la flexibiliteacute)

disponible pour satisfaire la clientegravele finale Lrsquoameacutelioration des processus a un impact visible

et direct sur chaque produit proposeacute aux clients

Lrsquoapproche processus provoque une eacutevolution de la faccedilon de travailler

- en faisant peacuteneacutetrer la laquo voix du client raquo au plus profond de lrsquoentreprise (et plus seulement

dans les services commerciaux et marketing)

- en mettant en eacutevidence des possibiliteacutes de rationalisation (par regroupement ou impartition

de certaines activiteacutes)

Remarque lrsquoapproche par les activiteacutes et les processus est agrave lrsquoorigine de la meacutethode

de deacutetermination des coucircts laquo ABC raquo - activity based costing

24 ndash Processus et fonctions

Le processus est transversal Il enchaicircne des activiteacutes qui traversent lrsquoentreprise en particulier

les services ou les deacutepartements drsquoune organisation fonctionnelle

Exemple

La division du travail par fonctions induit une charge de coordination pour assurer le

deacuteroulement du processus Elle peut geacuteneacuterer des attentes des erreurs ou des conflits drsquointeacuterecirct

(lrsquoobservation montre que des dysfonctionnements sont tregraves souvent constateacutes lors du

passage drsquoun service agrave un autre)

Organisation fonctionnelle et approche processus visent toutes deux un optimum

eacuteconomique mais leurs logiques sont diffeacuterentes

- le processus vise la satisfaction des clients (prix qualiteacute deacutelais service)

- le deacutecoupage fonctionnel cherche agrave optimiser les moyens (maximiser lrsquoeffet drsquoexpeacuterience

partager des infrastructures profiter de pocircles de compeacutetences hellip) Il apporte une ossature

hieacuterarchique stable souvent indispensable

Organisation fonctionnelle et approche processus sont donc compleacutementaires dans la

majoriteacute des cas et doivent ecirctre combineacutees judicieusement

APPLICATIONS OP

OP1 Deacutefinir flexibiliteacute systegraveme impartition

OP2 Citer huit exemples drsquoinformations essentielles pour optimiser un processus de

fabrication

Direction

Deacutepartement

commercial

(C)

Deacutepartement

administratif et

financier (AF)

Deacutepartement

Etudes (E)

Deacutepartement

Production (P)

Activiteacute

C-x Activiteacute

AF-x Activiteacute

E-x

Activiteacute

P-x

Processus x

Clie

nt

Initiation au management copy CRCF ndash J Sornet Page 26 48

OP3 Deacutegager les principes du toyotisme preacutesenteacute ci-dessous En quoi ce systegraveme est-il

initiateur de lrsquoapproche processus

Taiichi Ohno et le Toyotisme

1 - Extrait drsquoun article de Jacques BARRAUX - 1993 - LExpansion

Taiichi Ohno (1912 ndash 1990) hellip ne se prenait pas pour un visionnaire mais en imposant une

nouvelle faccedilon de produire il a reacuteinventeacute le management hellip tout le monde a entendu parler

des mots qui ont populariseacute le toyotisme dont il est le pegravere le juste-agrave-temps hellip Autant

doutils conccedilus pour lrsquoautomobile et qui ont aujourdhui une application universelle

hellip Taiichi Ohno jeune ingeacutenieur entre chez Toyota alors simple constructeur de machines

textiles Degraves 1926 apparaicirct la notion de jidoka hellip cest lart de transfeacuterer de lintelligence aux

machines pour mieux libeacuterer lintelligence des hommes Tout le contraire du taylorisme qui

juge la machine moins impreacutevisible que lhomme En 1933 Toyota se lance dans lautomobile

en sinspirant des meacutethodes ameacutericaines Mais en 1935 agrave loccasion dun voyage aux Etats-

Unis leacutetat-major de lentreprise revient fascineacute de sa visite dans un supermarcheacute La notion

de juste-agrave-temps va naicirctre de lobservation dune grande surface un lieu ougrave les clients ne

prennent que ce dont ils ont besoin et ougrave les rayons sont reacuteapprovisionneacutes pour compenser

les quantiteacutes preacuteleveacutees Ainsi le systegraveme Toyota est-il deacutejagrave dans la tecircte de ses dirigeants avant

mecircme la Seconde Guerre mondiale un demi-siegravecle avant la reacutevolution informatique et la

segmentation intensive des marcheacutes

hellip des esprits curieux comme Franccedilois Dalle en France tombent alors sous le charme des

formules et des paraboles de Taiichi Ohno En voici deux eacutechantillons

Penser agrave lenvers Cela signifie combattre les ideacutees reccedilues En lespegravece il sagit du fordisme et

du taylorisme Ohno ne croit pas agrave la planification aux effets deacutechelle et dexpeacuterience Il

propose un systegraveme industriel agrave lenvers qui permette de diversifier les produits et de les

fabriquer en petites quantiteacutes Nous ne devons plus ecirctre des paysans qui accumulent des

stocks mais des chasseurs On nimpose pas loffre On traque la demande et on la gegravere en

continu

Que les valleacutees soient hautes et les montagnes peu eacuteleveacutees Plutocirct que de concentrer tous

les efforts sur une production agrave un moment donneacute mieux vaut se doter de structures flexibles

permettant de passer agrave tout instant dune seacuterie agrave une autre Il faut eacuteviter les ruptures et les

secousses aplanir les cycles entretenir des flux reacuteguliers dactiviteacutes diversifieacutees Ce qui

implique de ne pas enfermer les hommes et les eacutequipements dans des speacutecialisations trop

eacutetroites

La flexibiliteacute le travail en groupe le refus de la dictature des machines la polyvalence et

surtout lattention constante aux signaux eacutemis par le marcheacute nappartiennent plus au

toyotisme Ces notions sont les fondements du nouvel art dorganiser de vendre et de

produire dans lindustrie comme dans les services hellip

2 - Quelques notions cleacutes

Taiichi Ohno a imagineacute la meacutethode des laquo cinq pourquoi raquo qui consiste agrave se poser cinq fois de

suite la question laquo pourquoi raquo sur le mecircme sujet de faccedilon agrave deacutecouvrir la veacuteritable cause

drsquoun problegraveme Cette meacutethode peut ecirctre appliqueacutee agrave tous les niveaux et permettre

notamment aux agents de fabrication de proposer de veacuteritables ameacuteliorations de la

production

La recherche de la qualiteacute totale (pas de deacutefaut des produits pas de rebuts pas de deacutefaut

des processus) accompagne la deacutemarche de Toyota La qualiteacute a un coucirct compenseacute par

des ventes accrues par lrsquoeacuteconomie des mesures palliatives aux deacutefauts

Initiation au management copy CRCF ndash J Sornet Page 27 48

Fiche OP1 ndash Benchmarking et processus

Le laquo benchmarking raquo consiste agrave comparer le fonctionnement de plusieurs systegravemes pour en

faire notamment ressortir les meilleures pratiques (laquo best practices raquo) Cette technique est

utiliseacutee depuis les anneacutees 80 pour ameacuteliorer la performance des entreprises Elle impose agrave

lrsquoentreprise drsquoeacutevaluer et de remettre en question ses propres modes de fonctionnement afin

de les faire eacutevoluer agrave la lueur de ce qui se fait ailleurs

Le benchmarking permet drsquoameacuteliorer les processus agrave moindre risque en fixant des objectifs

baseacutes sur des faits et donc plus facilement accepteacutes

Une classification des processus en tant que base de reacuteflexion a eacuteteacute eacutetablie aux USA par

lrsquolaquo International Benchmarking Clearinghouse raquo de lrsquoAPQC (american productivity and

quality center) en collaboration avec plusieurs dizaines drsquoentreprises

Elle se reacutesume ainsi

Le terme laquo reengineering raquo (la re-conception ou laquo reacuteingeacutenieacuterie raquo) des processus deacutesigne un

projet drsquoameacutelioration radicale des performances (de 20 agrave 50 ou plus) Il neacutecessite une

parfaite adheacutesion de la direction la constitution drsquoune petite eacutequipe de projet brillante

connaissant parfaitement les activiteacutes de lrsquoentreprise et il peut inclure un benchmarking

Le reengineering provoque geacuteneacuteralement la reacuteduction du nombre de niveaux hieacuterarchiques

(laquo delayering raquo) et lrsquoaccroissement du pouvoir de deacutecision des employeacutes (laquo empowerment raquo

ou laquo empouvoirement raquo) Bien qursquoy conduisant parfois il ne doit pas ecirctre confondu avec la

reacuteduction des activiteacutes (laquo downsizing raquo ou restructuration) et lrsquoexternalisation (laquo outsourcing raquo)

Pro

ce

ssu

s

op

eacutera

tio

nn

els

Pro

ce

ssu

s d

e m

an

ag

em

en

t e

t d

e

sup

po

rt

1 ndash

Comprendre

le marcheacute et

les clients (besoins

satisfaction)

2 ndash

Deacutevelopper

vision et

strateacutegie (contexte

concurrence)

3 ndash

Creacuteer

produits

services

processus

(concevoir

ameacuteliorer)

4 ndash

Marketing et

vente

5 ndash

Produire et

livrer (industrie

dont

ameacutelioration

processus)

6 ndash

Produire et

livrer (services)

7 ndash

Facturer et

servir les

clients (apregraves-

vente

reacuteclamations)

8 ndash Deacutevelopper et geacuterer les ressources humaines

9 ndash Geacuterer les systegravemes drsquoinformation

10 ndash Geacuterer les ressources financiegraveres et les actifs

11 ndash Appliquer un programme environnemental

12 ndash Geacuterer les relations exteacuterieures (actionnaires banques lois relations publiques hellip)

13 ndash Geacuterer lrsquoameacutelioration et le changement (eacutevaluer mesurer motiver qualiteacute totale)

Initiation au management copy CRCF ndash J Sornet Page 28 48

Fiche OP2 ndash Lrsquoorganisation par processus

Lrsquoeacutevolution drsquoune organisation aux activiteacutes reacutepeacutetitives vers lrsquoapproche processus est

geacuteneacuteralement progressive et se met en place par paliers

La mise en œuvre drsquoun veacuteritable management par processus doit ecirctre preacuteceacutedeacutee quand

lrsquoactiviteacute de lrsquoentreprise est complexe drsquoun recensement (une laquo cartographie des

processus raquo) pour mettre en eacutevidence les processus ou les familles de processus cleacutes critiques

pour le succegraves de lrsquoentreprise ougrave les efforts seront prioritaires

Des responsables de processus (laquo process owners raquo) sont ensuite deacutesigneacutes

Le responsable doit concevoir ses processus puis apregraves leur mise en œuvre assurer les

coordinations neacutecessaires les ameacuteliorer et les repreacutesenter aupregraves de la direction

Quand une structure par processus est mise en place des opeacuterateurs exeacutecutants

preacuteceacutedemment regroupeacutes dans les fonctions peuvent ecirctre affecteacutes aux processus et

drsquoanciens responsables de fonctions peuvent devenir des experts au service des processus

Lrsquoorganisation par processus peut imposer un degreacute eacuteleveacute drsquointeacutegration des activiteacutes donc

une polyvalence accrue des personnels et une reacuteduction des niveaux hieacuterarchiques

Elle neacutecessite pour le moins des compeacutetences eacutelargies au niveau des responsables de

processus (organisation administration technique hellip) dont le nombre doit rester limiteacute

(quelques dizaines au plus)

Sauf dans de tregraves petites structures lrsquoorganisation par processus se plaque geacuteneacuteralement sur

une structure plus classique

Initiation au management copy CRCF ndash J Sornet Page 29 48

ELEMENTS DE CORRIGE OP

OP1 Deacutefinir

Flexibiliteacute = adaptation au besoin (horaire variable chaicircnes robotiseacutees)

Systegraveme = ensemble organiseacute dans un but boicircte noire (sanguin nerveux meacutetrique laquo D raquo)

Impartition = sous-traitance ou externalisation (seacuteparation) drsquoactiviteacutes faire appel agrave des

partenaires plutocirct que faire soi-mecircme

OP2 Citer huit exemples drsquoinformations essentielles pour orienter lrsquooptimisation drsquoun processus

Montant des stocks (approvisionnements et produits finis)

Temps drsquoattente

Taux drsquoactiviteacute des ateliers

Rebuts

Deacutelai de production

Taux de reacuteclamations clients (qualiteacute)

Temps passeacutes en retouches finales

Turn over

Nombre drsquoarrecircts maladie

Accidents du travail

Dureacutee des arrecircts machines

OP3 Deacutegager les principes du toyotisme preacutesenteacute dans la fiche 31 En quoi ce systegraveme

repose trsquoil sur lrsquoapproche processus

Produire la quantiteacute juste neacutecessaire (agrave la demande) donc eacuteviter les stocks

Flexibiliteacute intelligence des chaicircnes de production

Qualiteacute (eacuteviter le coucirct de la non-qualiteacute)

La notion de processus est implicite ainsi que la chaicircne de valeur client

Initiation au management copy CRCF ndash J Sornet Page 30 48

DEFIS ET TENDANCES DU MANAGEMENT

Les meacutethodes de management se deacuteveloppent pour affronter le contexte eacuteconomique

Ce chapitre preacutesente les deacutefis auxquels le management contemporain doit faire face

1 ndash Lrsquoeacutevolution eacuteconomique contemporaine

A mesure que lrsquoactiviteacute eacuteconomique mondiale srsquoaccroicirct que la technologie eacutevolue les

changements sont de plus en plus rapides Ils introduisent des situations ineacutedites auxquelles les

entreprises doivent srsquoadapter en cherchant de nouvelles solutions de management Les trois

derniegraveres deacutecennies ont eacuteteacute notamment marqueacutees par les pheacutenomegravenes suivants (que nous

listons sans tenir compte des liens pouvant exister entre eux)

Pheacutenomegravene Traduction Effets

Deacute reacuteglementation

globalisation

financiegravere

titrisation

Libre circulation des capitaux accegraves

facile des particuliers au marcheacute

boursier (directement ou par

lrsquointermeacutediaire des OPCVM et SICAV)

Monteacutee en puissance du financement

des entreprises sur le marcheacute boursier

Fonds de pension

(retraites) et fonds

souverains (eacutetats)

Poids boursier important drsquoinvestisseurs

institutionnels qui cherchent un haut

rendement financier (dividendes ou

valorisation boursiegravere)

Pression sur les grandes entreprises

influence sur les strateacutegies

Mondialisation Liberteacute des eacutechanges internationaux Accroissement de la concurrence

recherche drsquoavantages eacuteconomiques

par la deacutelocalisation (biens et

services) la concentration des efforts

(recentrage) problegravemes drsquoemploi

multiplication des transports perte

drsquoinfluence des politiques

Baisse de lrsquoemploi

occidental

(notamment

industriel)

Moins de fabrications fabrications

automatiseacutees recours aux moyens

informatiques

Activiteacute reporteacutee sur le commerce la

conception et les services chocircmage

charge sociale

Restructurations Optimisation des entreprises

abaissement des coucircts augmentation

des marges recherche drsquoune taille

critique (eacuteconomies drsquoeacutechelle poids

sur le marcheacute)

Recentrages externalisations fusions

deacutelocalisations constitution de grands

groupes

NTIC (nouvelles

technologies de

lrsquoinformation et de

la communication)

Mise en œuvre des reacuteseaux (dont

internet) et drsquoapplications

informatiques communicantes

Nouvelles formes de commerce

marcheacute international deacutelocalisation

du travail intellectuel reacuteorganisation

de la distribution

Rareacutefaction relative

des matiegraveres

premiegraveres

Recherche de substituts exploration

miniegravere coucircts drsquoexploitation des

gisements accrus

Augmentation des coucircts variations

erratiques du cours des matiegraveres

deacutestabilisations politiques

Evolution

geacuteopolitique et

eacuteconomique

mondiale

Chute de lrsquoURSS transformation des

eacuteconomies collectivistes pays

eacutemergents (Chine Inde Breacutesil Russie)

()

Accroissement de la population

mondiale (4 agrave 6 7 milliards de 1970 agrave

2008)

Libeacuteralisme sans frein () nouvelles

puissances eacuteconomiques

opportuniteacutes de deacuteveloppement

nouveau partage des ressources

ineacutegaliteacutes baisse du soutien aux PVD

laquo Terrorisme raquo Actions armeacutees pression de groupes

armeacutes non gouvernementaux

Deacutestabilisations reacutegionales charge

des deacutepenses militaires

Deacuteveloppement

durable

Recherche drsquoune croissance eacutequitable

et respectueuse de lrsquoenvironnement

Pression sur les entreprises (eacutetats

associations de consommateurs

eacutecologistes ONG)

() Reacutecemment quelques affaires (Enron laquo subprimes raquo Vivendi Universal Socieacuteteacute

Geacuteneacuterale Airbus par exemple) et agrave plus grande eacutechelle la crise financiegravere de 2008 ont

montreacute les dangers drsquoune libeacuteralisation sans controcircles suffisants

Initiation au management copy CRCF ndash J Sornet Page 31 48

() Des alliances eacuteconomiques naissent entre pays eacutemergents (notamment en

ameacuterique centrale creacuteation de la Banque du Sud en 2008 par exemple) et lrsquoon

commence agrave imaginer une baisse progressive de lrsquoinfluence eacuteconomique des Etats

Unis

2 ndash Les deacutefis actuels du management

21 ndash Les grandes orientations

Lrsquoeacutevolution eacuteconomique suggegravere quelques pistes parfois concurrentes pour lrsquoaction du

manager contemporain On y retrouve au premier plan la construction drsquoune vision qui est

une composante commune du leadership

Objectif du manager

pour lrsquoentreprise

Justification Facteurs de reacuteussite

Construire une vision Eclairer lrsquoavenir de lrsquoentreprise partager

un but souder motiver

Effort de reacuteelle prospection

volontarisme de la direction

bonne communication

Reacuteactiviteacute et flexibiliteacute

(sous tous les aspects

agrave tous niveaux)

Srsquoadapter rapidement au marcheacute Bonne organisation des processus

personnel compeacutetent autonome

et motiveacute structure hieacuterarchique

alleacutegeacutee robotisation

Deacutegager des profits Reacutemuneacuterer les apporteurs de capitaux

srsquoautofinancer

Ajuster coucircts et structures

Exploiter les nouvelles

technologies

Reacuteactiviteacute ajuster coucircts et deacutelais

reacutepondre au marcheacute suivre les clients

Organiser le SI de faccedilon

pertinente eacuteviter le coucirct excessif

drsquoinvestissements trop en

laquo pointe raquo (laquo essuyer les placirctres raquo)

utiliser judicieusement les services

exteacuterieurs

Bacirctir des alliances

(contrats fusions)

Deacutevelopper une activiteacute limiter les coucircts

de transaction () atteindre la taille

critique et de meilleurs rendements se

recentrer sur une activiteacute profitable

Dominer les processus se donner

une identiteacute lisible externaliser se

doter drsquoune capaciteacute financiegravere

suffisante

Valoriser lrsquoimage Attirer les clients favoriser les alliances

donner confiance (apporteurs de fonds

employeacutes clients partenaires socieacuteteacute

civile)

Instaurer des regravegles de

gouvernance inteacutegrer le

deacuteveloppement durable

respecter lrsquoenvironnement

Geacuterer les risques Faire face aux aleacuteas eacuteconomiques et

technologiques (conjoncture politiques

accidents malveillance)

Creacuteer un systegraveme drsquoalerte geacuterer

la crise (reacuteaction raisonneacutee

sceacutenarios poursuite de

lrsquoexploitation dans un contexte

instable) mise en place de

proceacutedures drsquoapprentissage pour

ameacuteliorer les reacuteactions au fil du

temps

Geacuterer le changement Faire face agrave lrsquoeacutevolution de la demande

la pression sur les prix la variation des

performances financiegraveres la

concurrence la globalisation des

marcheacutes lrsquoeacutevolution technologique aux

fusions ou alliances aux changements

de reacuteglementation de direction hellip ()

Bonne communication pour

donner du sens au changement

et obtenir lrsquoadheacutesion du personnel

Rassembler et geacuterer les

connaissances former le

personnel

Innover Garder un avantage concurrentiel se

diffeacuterencier

Veille technologique et

commerciale investissement

Ouverture

internationale

Elargir le marcheacute saisir les opportuniteacutes Veille commerciale partenariats

() La theacuteorie des coucircts de transaction deacuteveloppeacutee par OE Williamson dans les

anneacutees 70 integravegre les coucircts lieacutes au recours au marcheacute (recherche et choix drsquoun

fournisseur neacutegociation reacutedaction de contrat suivi des eacutechanges risque de rupture

Initiation au management copy CRCF ndash J Sornet Page 32 48

drsquoapprovisionnement hellip) On peut en conclure que lrsquointeacutegration de diffeacuterentes

activiteacutes agrave lrsquoentreprise (la laquo firme raquo) preacutesente des avantages Mais des coucircts de

transaction internes doivent aussi ecirctre consideacutereacutes (preacuteparation organisation

surveillance hellip) et certaines formes de coopeacuteration continue avec les fournisseurs

permettent de reacuteduire le coucirct des transactions externes

() drsquoapregraves laquo Les meilleures pratiques de management raquo - Brilman Heacuterard ndash EO

Une eacutetude du Conference Board (2002) liste les deacutefis du management vus par 700 leaders

mondiaux Soit en reacutesumeacute avec indication du score correspondant

1 ndash Fideacuteliser les clients (42)

2 ndash Reacuteduire les coucircts (38)

3 ndash Accroicirctre flexibiliteacute et reacuteactiviteacute (29)

4 ndash Amener les employeacutes agrave adheacuterer aux valeurs et visions de lrsquoentreprise (26)

5 ndash Deacutevelopper et retenir les leaders (25)

6 ndash Geacuterer acquisitions et alliances (24)

7 ndash Accroicirctre lrsquoinnovation (20)

En fin de classement citoyenneteacute et reacuteputation (4) et ameacutelioration de la diversiteacute (3)

22 ndash Les techniques disponibles

Pour faire face aux deacutefis le manager dispose de nouveaux concepts et de nouvelles

techniques Le tableau ci-dessous en donne un reacutesumeacute et indique les domaines qursquoils

influencent principalement

Initiation au management copy CRCF ndash J Sornet Page 33 48

Principaux concepts techniques outils Incidence principale sur

Internet

- e-commerce (commerce eacutelectronique site

entreprise)

- CRM ou GRC (gestion de la relation client)

- e-procurement (gestion des approvisionnements

par le reacuteseau)

- messagerie eacutelectronique

- e-recrutement

Vente accegraves au marcheacute

Relation client reacuteactiviteacute personnalisation

fideacutelisation

Deacutelais coucircts

Communication transfert de donneacutees (piegraveces

jointes) tous domaines

Communication recrutement

Intranet reacuteseau drsquoentreprise SI

- knowledge management (gestion des

connaissances)

- e-learning (apprentissage en ligne)

- plateforme de travail collaboratif (groupware)

- workflow (circulation eacutelectronique de

documents enchaicircnement de processus)

- e-RH portail RH (libre accegraves aux postes agrave

pourvoir informations candidatures hellip)

- PGI (progiciel de gestion inteacutegreacute) ou ERP

Innovation capaciteacute au changement veille

documentaire

Formation du personnel accompagnement des

changements

Coordination communication interne

Coordination

Communication interne (voire internet en

externe) reacuteduction des coucircts climat drsquoentreprise

recrutement plans de carriegraveres hellip

Coucircts fiabiliteacute du systegraveme drsquoinformation deacutelais

processus (continuiteacute inteacutegration)

Logistique inteacutegreacutee

Supply Chain Management (SCM) gestion de la

logistique (incluant les approvisionnements)

Processus deacutelais coucircts

Externalisation

Valorisation du capital humain

GPEC (gestion preacutevisionnelle des emplois et

compeacutetences)

Coaching

Reacuteactiviteacute de lrsquoentreprise conservation des

compeacutetences rendements individuels turn-over

adaptation des compeacutetences motivation

Efficaciteacute individuelle controcircle reacutegulation

progregraves processus

Approche processus

Optimisation des processus

Deacutemarche qualiteacute totale (TQM ndash total quality

management)

Empowerment (empouvoirement)

Benchmarking reacuteingeacutenieacuterie

Coucircts marges qualiteacute deacutelais flexibiliteacute

externalisation eacutelargissement des compeacutetences

organisation

Ameacutelioration des processus (meacutetiers et supports)

Autonomie compeacutetences des employeacutes

Ameacutelioration des processus restructuration

Management par la valeur

Parties prenantes

Satisfaction des parties prenantes financement

motivation collaborations hellip

Collaboration inter organisations

Reacuteseaux drsquoentreprises alliances

EDI (eacutechange de donneacutees informatiseacutees) extranet

Impartition externalisation (outsourcing)

Coucircts recentrage investissements lancement

drsquoactiviteacute

Coucircts reacuteactiviteacute deacutelais relations avec

lrsquoadministration

Coucircts recentrage limitation des investissements

Ethique drsquoentreprise

Gouvernance drsquoentreprise (mode de direction

encadreacute par des regravegles)

Rocircle socieacutetal deacuteveloppement durable

environnement

Image de lrsquoentreprise reacutegulation du top

management relations actionnaires

Image peacutenaliteacutes et amendes objectifs

strateacutegiques

Initiation au management copy CRCF ndash J Sornet Page 34 48

23 ndash Le rocircle socieacutetal des entreprises

La responsabiliteacute socieacutetale de lrsquoentreprise (RSE) deacutesigne le rocircle qursquoelle prend dans la socieacuteteacute

au-delagrave de son activiteacute purement geacuteneacuteratrice de profit On parle aussi drsquoentreprise citoyenne

La RSE est indissociable du deacuteveloppement durable de porteacutee mondiale et dont les trois

piliers sont

- eacuteconomique (favoriser le deacuteveloppement les eacutechanges internationaux)

- social (accegraves aux soins eacuteducation conditions de travail hellip)

- environnemental (pollution preacuteservation des ressources hellip)

La RSE integravegre notamment une preacuteoccupation sociale de lrsquoentreprise vis-agrave-vis de ses salarieacutes

(seacutecuriteacute et santeacute au travail juste reacutemuneacuteration deacuteveloppement personnel hellip) Elle conduit agrave

tenir compte dans le management drsquoune vision exteacuterieure agrave lrsquoentreprise qui peut avoir des

reacutepercussions possibles sur son activiteacute eacuteconomique

Lrsquoentreprise peut aussi tirer avantage drsquoune deacutemarche responsable par la baisse de certains

coucircts (plus faibles consommations drsquoeacutenergies reacuteduction des transports hellip)

Le rocircle socieacutetal de lrsquoentreprise a eacuteteacute reconnu en France par la loi laquo NRE raquo de 2001 (loi sur les

nouvelles reacutegulations eacuteconomiques) qui oblige les socieacuteteacutes franccedilaise coteacutees sur un marcheacute

reacuteglementeacute agrave rendre compte dans leur rapport annuel de leur gestion sociale et

environnementale au travers de leur activiteacute

Article 116 de la loi Le rapport viseacute agrave larticle L 225-102 rend compte hellip laquo Il comprend

eacutegalement des informations dont la liste est fixeacutee par deacutecret en Conseil dEtat sur la

maniegravere dont la socieacuteteacute prend en compte les conseacutequences sociales et

environnementales de son activiteacute Le preacutesent alineacutea ne sapplique pas aux socieacuteteacutes

dont les titres ne sont pas admis aux neacutegociations sur un marcheacute reacuteglementeacute raquo

Une norme ISO 14000 integravegre ces preacuteoccupations et des taxes eacutecologiques sont

progressivement creacutees

3 ndash Le management par la valeur

31 ndash De lrsquoanalyse au management par la valeur

Lrsquoanalyse de la valeur est neacutee en 1947 aux Etats-Unis (General Electrics) Cette technique

consiste agrave eacutelaborer des produits conformes aux attentes de la clientegravele mais sans excegraves pour

trouver un bon compromis entre valeur pour le client et coucirct Le produit optimal est deacutefini agrave

partir drsquoenquecirctes qui deacuteterminent le besoin client (ou plutocirct drsquoun client laquo type raquo)

Exemple il est inutile de concevoir un petit veacutehicule citadin capable de parcourir

500 000 km sans avarie compte tenu des effets de mode et du faible kilomeacutetrage

annuel Par contre le marcheacute peut exiger un fonctionnement sans faille sur 150 000 km

soit dix ans en moyenne ce qui conditionne les coucircts de production

Cette recherche drsquoun ajustement de valeur au besoin des clients eacutetait un preacutecurseur du

management par la valeur qui recherche plus largement la creacuteation de valeur pour

chacune des parties prenantes de lrsquoentreprise tout en lui meacutenageant un reacutesultat suffisant

Plus geacuteneacuteralement le management par la valeur est deacutefini par une norme europeacuteenne (EN

12973)

Le management par la valeur est un style de management particuliegraverement destineacute agrave

mobiliser les individus agrave deacutevelopper les compeacutetences et agrave promouvoir les synergies et

Initiation au management copy CRCF ndash J Sornet Page 35 48

linnovation avec pour objectif la maximisation de la performance globale dun

organisme Le management par la valeur apporte une nouvelle faccedilon dutiliser nombre

de meacutethodes de management existantes Il est en coheacuterence avec le Management

de la qualiteacute

Cette approche du management pose de nombreuses questions notamment quelles

prioriteacutes et quelles valeurs attribuer aux parties prenantes comment appreacutehender la

perception par les parties prenantes de la valeur qui leur est affecteacutee

32 ndash La valeur client

Le processus drsquoeacutelaboration drsquoun produit qui consomme des ressources coucircteuses doit creacuteer

une valeur suffisante pour provoquer lrsquoachat par le client final La production drsquoune valeur

reconnue par le client est vitale pour lrsquoentreprise mais sa deacutetermination est parfois complexe

La valeur du produit perccedilue par le client integravegre des eacuteleacutements en partie subjectifs

- une valeur drsquousage (le produit reacutepond agrave un besoin)

- une valeur drsquoestime (lrsquoimage apporteacutee par le produit un aspect affectif)

- une valeur drsquoeacutechange (deacuteduite de lrsquoespoir de revente du produit)

Valeurs drsquousage drsquoestime et drsquoeacutechange deacutependent implicitement de la qualiteacute (un bien peu

fiable est impropre agrave lrsquousage attendu de mauvaise qualiteacute notoire il nrsquoapporte pas une

image positive et ses deacutefauts connus nuisent agrave sa revente) Une eacutevaluation de la qualiteacute

intervient donc dans la valeur perccedilue du produit

Par ailleurs le client considegravere le coucirct drsquoobtention du produit (les charges qursquoil doit supporter

pour acqueacuterir le produit lrsquoeffort qursquoil doit faire pour trouver le produit et les frais de mise agrave

disposition)

Le prix perccedilu par le client est geacuteneacuteralement supeacuterieur au prix de vente

Le client achegravete theacuteoriquement le produit qui preacutesente la diffeacuterence valeur perccedilue ndash prix

perccedilu la plus favorable ou le meilleur rapport prix perccedilu qualiteacute perccedilue et dans certains

cas celui qui a le prix produit le plus bas

Remarque les valeurs du scheacutema ci-dessus changent durant le cycle de vie du produit

(un nouveau produit peut avoir une valeur perccedilue plus eacuteleveacutee qursquoen fin de vie) La

valeur client ne peut ecirctre eacutevalueacutee que par enquecirctes et ne peut donc ecirctre deacutefinie avec

certitude

La notion de laquo satisfaction client raquo conseacutecutive agrave une vente influence aussi le prix produit et

le prix perccedilu

- lrsquoentreprise gagne sur les coucircts de recherche de clientegravele

- le client nrsquoa pas agrave rechercher un nouveau fournisseur et beacuteneacuteficie drsquoun coucirct drsquoobtention

plus bas

valeur perccedilue client

prix perccedilu client

coucirct produit Marge (valeur creacuteeacutee pour

lrsquoentreprise)

euros

prix produit

Valeur creacuteeacutee

pour le client

Initiation au management copy CRCF ndash J Sornet Page 36 48

La satisfaction du client deacutepend de facteurs qualitatifs aussi divers que la fiabiliteacute du produit

la vitesse de reacuteaction du fournisseur lrsquoattitude des commerciaux lrsquoefficaciteacute du service

apregraves-vente la netteteacute des contrats ou la justesse de la facture

Valeur perccedilue coucirct marge et satisfaction reacutesultent de processus allant de la conception du

produit jusqursquoagrave sa livraison et son apregraves-vente La deacutemarche laquo processus raquo et lrsquolaquo analyse de la

valeur raquo en forccedilant la recherche de solutions efficientes agrave tout niveau administratif

technique commercial et apregraves-vente sont donc neacutecessaires pour bien positionner

lrsquoentreprise sur son marcheacute

Pour autant le risque commercial ne peut jamais ecirctre annuleacute et lrsquooffre de lrsquoentreprise ne

satisfait geacuteneacuteralement pas en milieu concurrentiel tous ses clients potentiels

33 - La creacuteation de valeur pour les autres parties prenantes

Les salarieacutes

La creacuteation drsquoune valeur suffisante pour les salarieacutes est reconnue comme neacutecessaire car des

observations montrent que la satisfaction des clients en deacutepend Moins souvent eacutevoqueacutee en

peacuteriode de chocircmage elle nrsquoest prioritaire que pour les employeacutes dont lrsquoentreprise souhaite

conserver les compeacutetences

La laquo valeur salarieacute raquo ne comprend pas que le salaire Le sentiment drsquoappartenance agrave un

groupe la reconnaissance lrsquoaccomplissement de soi et la construction professionnelle en

sont des eacuteleacutements importants Comme pour les clients on doit ainsi distinguer la reacutetribution

perccedilue du salaire objectif

Les actionnaires

Lrsquoactionnaire apporte des fonds propres agrave lrsquoentreprise en contrepartie de titres parfois

neacutegociables en bourse et assortis drsquoun droit de vote en assembleacutee geacuteneacuterale La valeur

attribueacutee aux actionnaires est servie en termes moneacutetaires (dividende ou augmentation de la

valeur du titre neacutegociable)

Remarque des facteurs non moneacutetaires comme lrsquoimage de lrsquoentreprise qui deacutepend

en partie de sa communication peuvent influencer la deacutecision drsquoachat de vente ou

de conservation des titres par lrsquoactionnaire

Reacutetribution perccedilue euros

Salaire objectif

Avantage non

moneacutetaire de

lrsquoemploi

Initiation au management copy CRCF ndash J Sornet Page 37 48

Compte tenu de lrsquoimportance croissante de lrsquoactionnariat dans le financement des grandes

entreprises coteacutees en bourse et notamment des investisseurs institutionnels comme les fonds

de pension des indicateurs speacutecifiques ont eacuteteacute introduits pour appreacutecier la performance des

entreprises vue par les actionnaires Par exemple la valeur ajouteacutee eacuteconomique (EVA reg

economic value added marque deacuteposeacutee de Stern Stewart ou VAE ndash valeur ajouteacutee

eacuteconomique parfois deacutenommeacutee VEC ndash valeur eacuteconomique creacuteeacutee) qui prend en compte le

coucirct du capital

LrsquoEVA correspond tregraves scheacutematiquement au calcul suivant

EVA = (PO) profit opeacuterationnel ndash (C) coucirct du capital X (CE) capitaux employeacutes

LrsquoEVA neacutecessite en pratique des retraitements assez complexes Le PO peut se deacuteterminer

selon les principes suivants

- PO = reacutesultat drsquoexploitation (avant inteacuterecircts) ndash impocirct

- PO = beacuteneacutefice courant (tenant compte des inteacuterecircts) + inteacuterecircts ndash eacuteconomie drsquoimpocirct sur les

inteacuterecircts (on exclue les eacuteleacutements financiers et lrsquoimpocirct correspondant) ndash impocirct

- lrsquoimpocirct pris en compte correspond au profit opeacuterationnel consideacutereacute (dans les cas courants agrave

13 du PO)

C = taux moyen de reacutemuneacuteration du capital (reacutesultant par exemple du dividende exigeacute de

certains investisseurs et des taux drsquoemprunts bancaires)

CE = capitaux propres et dettes portant inteacuterecirct

Remarque le profit opeacuterationnel ou reacutesultat opeacuterationnel correspond au NOPAT ndash net

operating profit after tax - anglo-saxon LrsquoEVA est eacutegale au NOPAT diminueacute de la

reacutemuneacuteration des capitaux

Exemple lrsquoentreprise X dispose drsquoun capital de 2 500 000 euro et reacutealise un beacuteneacutefice net

drsquoimpocirct de 450 000 euro (taux 33 13) Un dividende de 6 doit ecirctre verseacute aux

actionnaires et la banque lui a accordeacute un precirct de 1 200 000 euro agrave 4 Les autres

constituants des reacutesultats financier et exceptionnel sont neacutegligeables

Reacutesultat opeacuterationnel = 450 000 + 004 x 1 200 000 x 23 = 482 000 euro

Coucirct du capital = 006 x 2 500 000 + 004 x 1 200 000 x 23 = 182 000 euro

EVA = 300 000 euro

Coucirct moyen pondeacutereacute du capital (C) = (004 x 1 200 000 x23 + 006 x 2 500 000)

3 700 000 Soit 492

Si lrsquoEVA est positive lrsquoentreprise creacuteeacutee de la valeur apregraves reacutemuneacuteration des capitaux et sa

valeur boursiegravere doit augmenter

Lrsquoutilisation de lrsquoEVA comme indicateur influence le management de lrsquoentreprise car il y a

trois moyens pratiques drsquoaugmenter lrsquoEVA

- augmenter le reacutesultat opeacuterationnel

- lancer des investissements ayant une rentabiliteacute supeacuterieure agrave C

- eacuteliminer les activiteacutes ayant une rentabiliteacute infeacuterieure agrave C

Remarque lrsquoutilisation sans nuance de lrsquoEVA comme critegravere de management peut

poser problegraveme Le calcul de lrsquoEVA repose sur des ajustements comptables il est donc

sujet agrave manipulations (provisions capitalisation ou non de la RD hellip) Par ailleurs le

critegravere laquo EVA raquo pris isoleacutement peut conduire agrave chercher la rentabiliteacute agrave court terme agrave

reacuteduire les investissements prospectifs et donc nuire agrave terme au deacuteveloppement de

lrsquoentreprise

Initiation au management copy CRCF ndash J Sornet Page 38 48

Les fournisseurs reccediloivent le paiement de leurs factures plus ou moins rapidement (le deacutelai

de paiement repreacutesente une valeur consentie au fournisseur)

Lrsquoentreprise peut accroicirctre la valeur apporteacutee agrave ses fournisseurs par des actions cibleacutees

comme une contribution agrave la formation de leurs personnels certains transferts de

technologie ou de savoir faire agrave des sous-traitants une coopeacuteration suivie favorisant leur

deacuteveloppement lrsquointeacutegration agrave des campagnes de promotion

A noter que la valeur consentie aux fournisseurs peut avoir une influence sur la qualiteacute et les

deacutelais de livraison des produits

La collectiviteacute reccediloit des taxes et parfois des prestations en nature par deacutefaut ou explicites

(effort de preacuteservation de lrsquoenvironnement ameacutenagement du territoire par les implantations

aide mateacuterielle agrave des projets participation agrave la formation par exemple)

APPLICATIONS DT

DT1 Deacutefinir expliquer deacutereacuteglementation socieacutetal eacuteconomies drsquoeacutechelle coaching EDI

gouvernance

DT2 Deacuteterminer en quoi la deacutemarche TQM srsquoinscrit dans les deacutefis actuels du management

DT3 Apregraves avoir consulteacute les documents ci-dessous extraits du site drsquoAir France

(httpdeveloppement-

durableairfrancecomFRfrlocaldemarcheN4_positionnement_pphtm)

exposer les enjeux et les limites de la RSE et de la gestion des parties prenantes

Initiation au management copy CRCF ndash J Sornet Page 39 48

Dialogue avec les parties prenantes

Initiation au management copy CRCF ndash J Sornet Page 40 48

Attentes des parties prenantes

Initiation au management copy CRCF ndash J Sornet Page 41 48

Creacuteation de valeur pour les parties prenantes

La creacuteation de valeur pour les parties prenantes est au cœur de la strateacutegie du Groupe Le scheacutema de

distribution financiegravere ci-dessous donne un aperccedilu de la distribution des recettes du Groupe aux

diffeacuterentes parties prenantes actionnaires collaborateurs fournisseurs pouvoirs publics

collectiviteacutes locales etc

Initiation au management copy CRCF ndash J Sornet Page 42 48

Fiche DT1 ndash Extrait du sommaire de laquo Problegravemes eacuteconomiques raquo No 2894

La gestion des entreprises bouleverseacutee par les technologies de linternet

Reacutealiteacutes industrielles - Annales des Mines Jean-Michel Yolin

Avec lavegravenement de linternet les processus de conception de production et de vente sont

radicalement remis en cause Quel que soit le secteur dactiviteacute les technologies de linternet

permettent en effet de reacuteduire les deacutelais et de passer dun processus discontinu agrave un processus

continu Lorganisation des entreprises et leur mode de gestion en sont profondeacutement bouleverseacutes

tant au niveau individuel que collectif Linternet rend ainsi possible la reacutealisation dobjectifs que les

entreprises cherchaient agrave atteindre depuis longtemps sans y parvenir meilleure eacutecoute du client

travail sans stocks en flux tendu hieacuterarchies plates autorisant une grande reacuteactiviteacute flexibiliteacute dans

lorganisation et loutil de production acceacuteleacuteration du renouvellement des produits entreprises en

reacuteseau ougrave chacune se recentre sur son cœur de meacutetier etc

Le laquo knowledge management raquo ou comment geacuterer les connaissances

Document de travail du LAMSADE - Michel Grundstein

Peter Drucker lavait preacutedit le capital immateacuteriel eacutetait voueacute agrave devenir un facteur de compeacutetitiviteacute

pour lentreprise La libeacuteralisation des eacutechanges acceacutelegravere les processus de deacutecision de lentreprise

et implique que lassimilation des informations soit agrave la fois de meilleure qualiteacute et plus rapide Ainsi

la fonction qui consiste agrave manager les connaissances au sein de lentreprise savegravere primordiale

Bien que la prise de conscience de limportance du capital immateacuteriel ait eacuteteacute tardive - le concept

de knowledge management est apparu en France aux Etats-Unis et au Japon au milieu des

anneacutees 1990 - agrave lheure actuelle lorganisation de leacutechange dinformations et le partage des

connaissances sont devenus des facteurs cleacutes dune gestion performante de lentreprise Ils

doivent sinscrire dans un projet global destineacute agrave mettre en valeur les savoirs et les savoir-faire

individuels et collectifs

Les leccedilons du laquo coaching raquo pour le management de la qualiteacute

Humanisme et Entreprise - Martine Brasseur

Parmi les nouvelles formes de management en vogue dans les entreprises le coaching figure en

bonne place Appliqueacute au management de la qualiteacute il sagit dune pratique

daccompagnement destineacutee agrave initier et agrave faciliter le processus de deacuteveloppement dun individu

La deacutemarche consiste agrave affirmer que tout individu est en quecircte de qualiteacute agrave condition toutefois

de ne pas lui imposer des contraintes lempecircchant de progresser On considegravere notamment les

erreurs comme potentiellement feacutecondes En deacutefinitive le coach donne au coacheacute la permission

de reacuteussir en lui donnant aussi la permission deacutechouer

Initiation au management copy CRCF ndash J Sornet Page 43 48

Fiche DT2 ndash Management strateacutegique les sept deacutefis agrave relever dici agrave 2016

Extrait drsquoun article du site wwwlentreprisecom -Sabine Blanc - Mis en ligne le 20032007

(httpwwwlentreprisecom325article11977html)

Une eacutetude anglaise publieacutee par lopeacuterateur Orange Grande-Bretagne deacutecrypte la mutation

des formes de travail et les enjeux majeurs pour les entreprises de demain afin decirctre au top

de la compeacutetitiviteacute Voici les challenges-cleacutes pour les managers qui veulent rester dans la

course hellip

1 - Future organisation du travail les quatre laquo mondes raquo possibles

La reacutealiteacute sera probablement un meacutelange de ces quatre sceacutenarios souligne lrsquoeacutetude

Les mondes mutuels Tout se passe dans le cadre des communauteacutes locales vie priveacutee

comme professionnelle Le modegravele coopeacuteratif preacutevaut au lieu du laquo big business raquo Oublieacutes

aussi dans ce systegraveme les trajets pour aller au bureau les gens preacutefegravereront travailler dans de

petites entreprises locales souvent connecteacutees au reacuteseau drsquoautres structures similaires

Les laquo reacutepondants raquo (en anglais laquo replicants raquo) La figure du consultant freelance deviendra

dominante tandis que celle du salarieacute deacuteclinera Il ne sera pas rare de travailler pour plusieurs

entreprises On perdra en seacutecuriteacute de lrsquoemploi en visibiliteacute et en routine ce que lrsquoon gagnera

en liberteacute La majeure partie des tacircches srsquoeffectuera chez soi avec la possibiliteacute de srsquoinstaller

temporairement dans les bureaux de son client du moment Dans un contexte dincertitude

sur lrsquoavenir les travailleurs alterneront peacuteriodes drsquoactiviteacute intense et repos Ce sera agrave eux

drsquoaller vers les entreprises et non lrsquoinverse mecircme si celles-ci devront veiller agrave rester attractives

Les cottages eacutelectroniques Comme ce nom le suggegravere le teacuteleacutetravail deviendrait la norme

univers priveacute et professionnel se confondant Plus besoin de subir une heure de transport les

salarieacutes se logueront de chez eux sur le reacuteseau de lrsquoentreprise Les reacuteunions se tiendront dans

de petits bureaux centraux situeacutes agrave courte distance La flexibiliteacute du temps de travail srsquoimpose

Les salarieacutes disposeront de plus de marge de liberteacute dans leur activiteacute

Les disciples de la nueacutee Cette appellation poeacutetique cache simplement une extension de

lrsquoorganisation actuelle des grandes entreprises avec des salarieacutes se rendant sur un lieu de

travail centraliseacute Le rocircle croissant des technologies de lrsquoinformation multipliera les faccedilons de

collaborer et accroicirctra lrsquoefficaciteacute Le controcircle du travail sera omnipreacutesent La frontiegravere entre

travail et vie priveacutee restera marqueacutee

2 - Sept deacutefis pour les entreprises et leur managers

Quoi qursquoil advienne les entreprises et leurs dirigeants devront concentrer leurs efforts sur sept

points-cleacutes pour srsquoadapter Voici quelques exemples de probleacutematiques souleveacutees par le

rapport et des pistes de solution

Le leadership Les managers devront entre autres savoir persuader et influencer des

travailleurs beaucoup plus indeacutependants Ils auront aussi agrave repenser les niveaux auxquels

prendre les deacutecisions strateacutegiques en haut ou au contraire agrave des degreacutes moins eacuteleveacutes de la

pyramide hieacuterarchique

gt Faire du management une force facilitant les activiteacutes transversales plutocirct que la reacuteduire agrave

la seule fonction de deacutecision

La culture drsquoentreprise Davantage de salarieacutes capables de reacutefleacutechir seront neacutecessaires

tandis que les tacircches qui peuvent ecirctre automatiseacutees ou scripteacutees diminueront Un des

enjeux creacuteer une culture agrave mecircme drsquoattirer et drsquoencourager les personnes preacutesentant ces

qualiteacutes de reacuteflexion requises dans un contexte de compeacutetition accrue et de plus grande

indeacutependance des travailleurs

Initiation au management copy CRCF ndash J Sornet Page 44 48

gt Passer si neacutecessaire drsquoune culture drsquoentreprise forte agrave un mode drsquoengagement plus

consensuel moins rebutant

La marque Conseacutequence du recours croissant agrave lrsquo laquo outsourcing raquo lrsquoimage drsquoune marque

deacutependra plus drsquoagents exteacuterieurs qui ne fonctionnent pas forceacutement selon le mecircme mode

drsquoorganisation Comment garder le controcircle dessus

gt Choisir le mode qui corresponde le plus agrave vos valeurs et preacutevoir un programme de risk

management qui mette en eacutevidence ougrave les conflits sont susceptibles de jaillir

Lrsquoinnovation Plus que jamais il faudra faire face agrave une acceacuteleacuteration du rythme de

lrsquoinnovation en proposant constamment des solutions adapteacutees

gt Tisser des partenariats strateacutegiques avec drsquoautres entreprises pour partager les coucircts et les

fruits de lrsquoinnovation

Le deacutefi opeacuterationnel et technologique De quelle faccedilon controcircler lrsquoinformation crsquoest-agrave-dire

faire en sorte que les bonnes personnes accegravedent facilement agrave une information toujours en

phase tout en maintenant la seacutecuriteacute

gt Recourir agrave des laquo feuilles de route des futurs raquo syntheacutetisant en une page les indicateurs

sociaux et de consommation ainsi que les eacutevolutions technologiques et leacutegislatives qui

influent sur les changements et indiquant comment ils modifient vos marcheacutes vos clients et

votre organisation

La qualiteacute Si de nouveaux proceacutedeacutes ont pu deacutegrader la qualiteacute comme le recours agrave des

centres drsquoappel externaliseacutes drsquoautres ideacutees se sont reacuteveacuteleacutees plus prometteuses comme en

teacutemoigne le succegraves de certaines compagnies aeacuteriennes low cost Elles ont su conjuguer prix

serreacutes et services eacuteleveacutes ce qui devra devenir la norme estime lrsquoeacutetude

gt Continuer de rechercher la qualiteacute Elaborez aussi une bonne prestation service qui inclut

une livraison de qualiteacute voire creacuteez-la en partenariat avec les consommateurs

La leacutegislation La question de la proprieacuteteacute intellectuelle pourrait ecirctre probleacutematique Elle est

deacutejagrave source de conflits comme en teacutemoigne le procegraves pour violation de brevet intenteacute agrave RIM

le fabricant canadien du Blackberry par NTP Que pourra-t-on et que faudra-t-il proteacuteger par

un brevet Il sera eacutegalement neacutecessaire drsquoadapter la leacutegislation aux nouveaux modes

drsquoorganisation

gt Collaborer avec les acteurs du mecircme secteur et les leacutegislateurs pour deacutevelopper les

modegraveles des lieux de travail du futur et bacirctir le droit le plus adeacutequat

Orange a-t-il vu juste dans ses preacutevisions Rendez-vous dans neuf ans pour la reacuteponsehellip

Initiation au management copy CRCF ndash J Sornet Page 45 48

Fiche DT3 ndash Le management par la qualiteacute totale

Extrait drsquoune lettre drsquoinformation du cabinet Baud Accordance Consulting AD2 consultants ndash

2002

1 - Le TQM (Total Quality Management) offre pour lentreprise une vision de la qualiteacute plus

large et transversale

Son principe est simple La finaliteacute de lEntreprise est de deacutevelopper la satisfaction de ses

clients tout en eacutetant beacuteneacuteficiaire cest agrave dire pas agrave nimporte quel prix Elle doit ameacuteliorer sa

rentabiliteacute au travers de la deacutemarche qualiteacute La Qualiteacute Totale vise agrave fournir aux clients

externes et internes une reacuteponse adeacutequate agrave leurs attentes dans le meilleur rapport qualiteacute

prix la meilleure efficience

Elle considegravere pour cela lensemble des processus de lentreprise ayant une incidence sur la

qualiteacute et la satisfaction des clients

Le TQM fait ainsi une large place agrave

la deacutefinition et la planification de la strateacutegie geacuteneacuterale

la coheacuterence de la politique qualiteacute avec la strateacutegie

la deacutemultiplication de la politique qualiteacute dans toutes les directions de lentreprise

la relation client fournisseur interne

la prise en compte de lenvironnement concurrentiel

la consideacuteration de lensemble des risques potentiels financiers sociaux concurrentielshellip

limplication et la motivation du personnel

lanalyse des besoins des clients et le positionnement marketing

la maicirctrise des processus transverses internes

les reacutesultats sous tous ses aspects y compris financiers commerciaux image

De nombreux reacutefeacuterentiels sont relatifs agrave la Qualiteacute Totale hellip Tous ces reacutefeacuterentiels imposent un

questionnement plus profond et indiscret sur le mode de fonctionnement de lentreprise et

son management

helliphellip

2 - LISO 9001 2000 au travers du deacuteploiement des processus (management supports

reacutealisation et ameacutelioration continue) reacutepond quelque peu agrave la mecircme logique

LISO est une ouverture indeacuteniable vers la logique du TQM mais ne se reacutefegravere pas agrave la notion

defficience

Les dirigeants sont cependant sensibles agrave la neacutecessaire reacuteduction des coucircts de non-qualiteacute

et dobtention de la qualiteacute agrave la rentabiliteacute du systegraveme de management de la qualiteacute

mais ne perccediloivent pas toujours la qualiteacute comme une deacutemarche globale

Les deacutemarches qualiteacute commencent bien souvent par la remise en cause de lorganisation

leacutevaluation critique de son efficaciteacute lexamen des processus et la mise en eacutevidence des

lourdeurs administratives

La qualiteacute devient laffaire de tous hellip

Initiation au management copy CRCF ndash J Sornet Page 46 48

Fiche DT4 ndash Le deacuteveloppement durable et la RSE

Extrait du site wwwvigeocom

(httpwwwvigeocomcsr-rating-agencyfrmethodologiecriteres-de-recherche37-

criteres-d-analysehtml)

Deacuteveloppement durable laquo un deacuteveloppement qui reacutepond aux besoins du preacutesent sans compromettre

la capaciteacute des geacuteneacuterations futures de reacutepondre aux leurs raquo (Commission mondiale sur lrsquoenvironnement

et le deacuteveloppement ndash 1987)

Reacutefeacuterentiel drsquoeacutevaluation des entreprises par le groupe Vigeacuteo (le groupe mesure les performances et le

niveau de maicirctrise des risques de responsabiliteacute sociale des entreprises et des organisations - site

wwwvigeocom)

1 Ressources Humaines Ameacutelioration continue des relations professionnelles des relations drsquoemploi et des conditions de travail 2 Droits humains sur les lieux de travail Respect de la liberteacute syndicale et promotion de la neacutegociation collective non discrimination et promotion de lrsquoeacutegaliteacute eacutelimination des formes de travail proscrites (enfants travail forceacute) preacutevention des traitements inhumains ou deacutegradants de type harcegravelements sexuels protection de la vie priveacutee et des donneacutees personnelles 3 Environnement Protection sauvegarde preacutevention des atteintes agrave lenvironnement mise en place drsquoune strateacutegie manageacuteriale approprieacutee eacuteco conception protection de la biodiversiteacute et maicirctrise rationnelle des impacts environnementaux sur lrsquoensemble du cycle de vie des produits ou services

4 Comportements sur les marcheacutes Prise en compte des droits et inteacuterecircts des clients inteacutegration de standards sociaux et environnementaux dans la seacutelection des fournisseurs et sur lrsquoensemble de la chaicircne drsquoapprovisionnement preacutevention effective de la corruption respect des regravegles concurrentielles 5 Gouvernement drsquoentreprise Efficience et probiteacute assurance de lrsquoindeacutependance et de lrsquoefficaciteacute du Conseil drsquoadministration effectiviteacute et efficience des meacutecanismes drsquoaudit et de controcircle et notamment inclusion des risques de responsabiliteacute sociale respect des droits des actionnaires et notamment des minoritaires transparence et rationaliteacute de la reacutemuneacuteration des dirigeants 6 Engagement socieacutetal Effectiviteacute inteacutegration manageacuteriale de lrsquoengagement contribution au deacuteveloppement eacuteconomique et social des territoires drsquoimplantation et de leurs communauteacutes humaines engagements concrets en faveur de la maicirctrise des impacts socieacutetaux des produits et des services contribution transparente et participative agrave des causes drsquointeacuterecirct geacuteneacuteral

Initiation au management copy CRCF ndash J Sornet Page 47 48

ELEMENTS DE CORRIGE DT DT1 Deacutefinir expliquer

Deacutereacuteglementation = suppression des contraintes eacuteconomiques (libre eacutechange des biens et

capitaux)

Socieacutetal = qui se rapporte agrave la structure agrave lrsquoorganisation ou au fonctionnement de la socieacuteteacute

Economies drsquoeacutechelle = reacuteduction des coucircts lieacutee au niveau drsquoactiviteacute (amortissement des

charges fixes)

Coaching = accompagnement de personnes ou deacutequipes pour le deacuteveloppement de leurs

potentiels

EDI = eacutechange de donneacutees informatiseacutees ET standardiseacutees (ex SWIFT bancaire edifact

documents deacuteclaratifs)

Gouvernance = exercice du pouvoir la bonne gouvernance est participative et eacutequitable

conforme agrave lrsquointeacuterecirct commun

DT2 Deacuteterminer en quoi la deacutemarche TQM srsquoinscrit dans les deacutefis actuels du management

Voir notamment fiche 43

Maicirctrise des processus reacuteduction des coucircts reacuteactiviteacute et satisfaction de la clientegravele = faire

face agrave la concurrence

Ameacutelioration de lrsquoimage motivation du personnel

DT3 Apregraves avoir consulteacute les documents ci-dessous extraits du site drsquoAir France

(httpdeveloppement-

durableairfrancecomFRfrlocaldemarcheN4_positionnement_pphtm)

exposer les enjeux et les limites de la RSE et de la gestion des parties prenantes

Trame geacuteneacuterale possible

Introduction

Les deacutefis contemporains (accroissement de la concurrence devenue mondiale recherche

de nouveaux avantages concurrentiels pression de la socieacuteteacute besoin drsquoimage et de projet

lisible pour mener lrsquoentreprise crise et scandales du libeacuteralisme hellip) RSE et PP

Deacuteveloppement (voir cours)

1 ndash Parties prenantes et management par la valeur

PP deacutefinir citer reacutesumer lrsquoavantage rechercheacute (fideacuteliser motiver recherche drsquoalliances

implicites)

PP moyens (dont exemples AF) et meacutethode de management par la valeur (reacutepartie)

2 ndash La responsabiliteacute socieacutetale de lrsquoentreprise

RSE 3 axes

- eacuteconomique (favoriser le deacuteveloppement les eacutechanges internationaux)

- social (accegraves aux soins eacuteducation conditions de travail hellip)

- environnemental (pollution preacuteservation des ressources hellip)

RSE gouvernance drsquoentreprise facteur drsquoimage inteacutegrable dans la deacutemarche PP

Article 116 de la loi Le rapport viseacute agrave larticle L 225-102 rend compte hellip laquo Il comprend

eacutegalement des informations dont la liste est fixeacutee par deacutecret en Conseil dEtat sur la maniegravere

dont la socieacuteteacute prend en compte les conseacutequences sociales et environnementales de son

activiteacute Le preacutesent alineacutea ne sapplique pas aux socieacuteteacutes dont les titres ne sont pas admis aux

neacutegociations sur un marcheacute reacuteglementeacute raquo

Initiation au management copy CRCF ndash J Sornet Page 48 48

RSE exemple AF (ONG fournisseurs)

3 ndash Liens entre PP et RSE

- la RSE introduit de nouvelles PP

- la RSE suppose le respect des PP usuelles (employeacutes clients notamment)

4 - Probleacutematique

- deacutefinir la valeur reacuteellement apporteacutee par une gestion des PP (confusion salaire ndash valeur

idem impocircts hellip ex laquo valeur ajouteacutee raquo)

- communication (neacutecessaire mais aller au-delagrave)

- marginaliteacute des deacutepenses RSE (efficaciteacute sinceacuteriteacute de lrsquoengagement marge de manœuvre)

- charge RSE reporteacutee sur des tiers (ex fournisseurs AF)

- inteacutegration de facteurs non visibles en comptabiliteacute (pertes drsquoemploi nuisances hellip)

Conclusion

Voies incontournables mais pouvant nrsquoavoir qursquoun effet superficiel et temporaire Voir utiliteacute

drsquoaccompagnement leacutegislatif de regravegles de gouvernance

Initiation au management copy CRCF ndash J Sornet Page 15 48

Caracteacuteristiques

de lrsquoaction

- reacutepeacutetition

- risque

- normes

- ampleur

Prise de

deacutecision

- opeacuteration

- direction

- deacutelai

Informations

neacutecessaires

- nature

- origine

- deacutelai

obtention

Cleacute pour la

reacuteussite

Intervention

exteacuterieure

possible

Assurer la

restauration du

soir

(Restaurant

familial)

Technique

(fabrication)

Vente (terrain)

Appros

Reacutepeacutetitive

(quot)

Risque faible

Normes

drsquohygiegravene

Faible

Opeacuterationnelle

Geacuterant

responsable

Rapide (qq

jours menu et

appros)

Nombre de

couverts

Tarifs usuels

Calendrier

(fecirctes)

Clients docs

divers

expeacuterience

Qq jours

Varieacuteteacute menu

Plats phares

Accueil

Appros

Tarification

Vins

Gestion

congeacutelation

Qualiteacute cuisine

Fournisseurs

Extra

Publiciteacute

Construire un

viaduc

(autoroute)

Technique

Organisation

Appros

Uniteacute (ou peu)

Eleveacute (financier

technique)

Architecture

Eleveacutee

Direction

(aleacuteas)

Opeacuterationnelle

(conduite

chantier)

Immeacutediat agrave qq

semaines

Plans

plannings

Qualifications

Meacuteteacuteo

Disponibiliteacutes

Bureau eacutetudes

Qq sem agrave 24h

Techniciteacute

Appros

Qualifications

Preacutevision

GRH

Contrat juste

SS traitants

Organismes

certificateurs

Controcircle

client

Certifier les

comptes

annuels drsquoun

groupe national

(cabinet

drsquoaudit)

Technique

Relation client

Gestion des

connaissances

Annuelle

Moyen

Regravegles

comptables

fiscales

Moyenne (selon

importance du

cabinet)

Opeacuterationnelle

Qq jours agrave

semaines

Comptable

Juridique

Client

Etat

Qq jours agrave

semaines

Techniciteacute

Expeacuterience

Relation client

Systegraveme info client

Siegravege

Autre cabinet

Lancer une

ligne drsquoavions

(constructeur

aeacuteronautique)

Strateacutegique

RD

Etudes

Uniteacute

Tregraves eacuteleveacute

Aeacuteronautique

Tregraves eacuteleveacutee

Direction

Qq mois agrave

anneacutees

Marcheacute

Etudes

Compagnies

Qq mois agrave

anneacutees

Concept

Outil industriel

Coucirct exploitation

Tarif

Fiabiliteacute

Deacutelaisconcurrence

SI simulation

SS traitants

Bureaux

drsquoeacutetudes

speacutecialiseacutes

Compagnies

Conseils

Reacuteduire la

capaciteacute de

production

(groupe

industriel)

Strateacutegique

RH

Communication

Production

Uniteacute

Moyen

Leacutegislation

(dont RH)

Eleveacutee

Direction

Qq mois agrave

anneacutees

Financiegravere

Industrielle

Marcheacute

Organisation

Organismes

speacutecialiseacutes

DRH

Qq mois

Communication

Connaissance des

compeacutetences

Connaissance outil

industriel

Concurrence

Portefeuille

drsquoactiviteacutes

Cabinet

drsquoorganisation

Conseils

speacutecifiques

Acqueacuterir une

entreprise

concurrente

(teacuteleacutephonie

mobile)

Strateacutegique

Marketing

Production

(reacuteseau)

Financier

Communication

Uniteacute

Tregraves eacuteleveacute

Leacutegislation

telecom

Tregraves eacuteleveacutee

Direction

Qq mois

Financiegravere

Marcheacute

Reacuteseaux

(ampleur

recouvrement

hellip)

Organisations

Interne

Racheteacutee

Sources

speacutecialiseacutees

Qq mois

Communication

Marcheacute

Cours boursiers

Cabinet

drsquoorganisation

Conseils

speacutecifiques

Initiation au management copy CRCF ndash J Sornet Page 16 48

LE MANAGEMENT EN PRATIQUE

Pour assumer sa fonction le management doit couvrir sans discontinuiteacute lrsquoensemble de

lrsquoorganisation et inteacutegrer de nombreux facteurs dont nous allons reacutesumer lrsquoessentiel

1 ndash Les fonctions et activiteacutes du management

Pour Henri Fayol la fonction drsquoadministration de lrsquoentreprise (son management) reposait sur

cinq actions preacutevoir organiser commander coordonner et controcircler (laquo PO3C raquo)

Nous distinguerons cinq activiteacutes de management

- la conception (au plus haut niveau finaliteacute but ou vocation de lrsquoorganisation

meacutetiers dimension politique de croissance hellip)

- la planification (deacutefinition des objectifs eacutecheacuteances)

- lrsquoorganisation (reacutepartition du travail choix des modes de coordination)

- le pilotage de lrsquoaction opeacuterationnelle (motivation animation encadrement

assistance)

- lrsquoeacutevaluation (controcircle des reacutesultats obtenus ajustements)

Dans chacune de ces activiteacutes des deacutecisions et des arbitrages sont neacutecessaires avec des

enjeux plus ou moins importants

Remarques

- Les cinq activiteacutes du management peuvent se retrouver agrave tout niveau de

management si lrsquoentreprise laisse une certaine autonomie de deacutecision agrave ses diffeacuterentes

uniteacutes La conception est naturellement du ressort de la direction geacuteneacuterale et des

conseils drsquoadministration mais elle peut ecirctre preacutesente pregraves du terrain (latitude laisseacutee agrave

une filiale ou agrave un magasin par exemple) De mecircme lrsquoorganisation du travail concerne

un atelier mais aussi la direction qui structure lrsquoentreprise pour assurer ses activiteacutes sa

production

- La planification deacutefinit des objectifs ou des axes strateacutegiques (choix de produits

modaliteacutes de deacuteveloppement des ventes implantations alliances hellip) et les traduit en

donneacutees de gestion preacutevisionnelles syntheacutetiques et eacutechelonneacutees dans le temps afin de

valider les objectifs et de fixer des repegraveres

- Un laquo business plan raquo (plan drsquoaffaires)est notamment lrsquoeacutequivalent de la planification

dans le cas de creacuteation drsquoentreprise ou pour la preacutesentation de tout projet drsquoactiviteacute

Les activiteacutes du management srsquoinscrivent dans des cycles qui peuvent ecirctre scheacutematiseacute

comme suit (lrsquoeacutevaluation peut entraicircner une reacutevision du pilotage de lrsquoorganisation ou des

objectifs sans que lrsquoentreprise ne soit fondamentalement remise en cause)

conception

planification

organisation

pilotage

eacutevaluation

Initiation au management copy CRCF ndash J Sornet Page 17 48

2 ndash Les contextes de management

Le management est influenceacute par son contexte qui justifie des objectifs une organisation

des meacutethodes

Par exemple lrsquoentreprise admet de nombreuses variantes selon sa taille sa forme juridique

son controcircle par lrsquoeacutetat (entreprises publiques) ou par des inteacuterecircts priveacutes Il en va de mecircme des

organismes administratifs qui peuvent deacutependre de directives nationales ou reacutegionales des

associations qui ont des activiteacutes drsquoampleur tregraves variable

21 ndash La dimension de lrsquoentreprise

La dimension drsquoune entreprise se mesure principalement en fonction de son effectif ou de

son chiffre drsquoaffaires Des seuils sont deacutefinis par divers organismes et exploiteacutes agrave des fins

statistiques ou pour la deacutetermination de certaines obligations sociales ou fiscales

(repreacutesentation du personnel cotisations hellip) Il nrsquoy a bien entendu pas de laquo barriegravere de

tailleraquo absolue conditionnant le management drsquoune entreprise

LrsquoUE preacuteconise de distinguer les micro ndash entreprises (jusqursquoagrave 9 salarieacutes) les TPE ndash tregraves petites

entreprises (moins de 20 salarieacutes) les petites entreprises (moins de 50) et les moyennes

entreprises (de 50 agrave 250) Cependant les PME sont parfois situeacutees entre 10 et 500 salarieacutes

Remarques

- en France environ 40 des entreprises emploient de 1 agrave 50 salarieacutes (ce qui repreacutesente

plus de 50 des emplois) et 59 nrsquoen ont aucun

le pays compte environ 2 600 000 entreprises dont moins de 1 ont 250 employeacutes et

plus

- ancienneteacute et taille de lrsquoentreprise sont lieacutees si lrsquoon eacutecarte les restructurations et autres

eacutevolutions drsquoentreprises existantes

La dimension de lrsquoentreprise a une influence sur lrsquoorganisation et le laquo style raquo de son

management

- les PME sont souvent entrepreneuriales (les dirigeants eacutegalement apporteurs de capitaux

sont totalement engageacutes dans la marche de lrsquoentreprise) Elles ont une gestion flexible peu

formaliseacutee plus qualitative que quantitative Les PME sont freacutequemment focaliseacutees sur un seul

type drsquoactiviteacute Pour ne pas alourdir leur structure elles ont tendance agrave sous-traiter les

activiteacutes speacutecialiseacutees ne correspondant pas agrave leur meacutetier de base

- les grandes entreprises sont manageacuteriales (les dirigeants sont nommeacutes par les actionnaires

en raison de leurs compeacutetences) et moins reacuteactives

22 ndash Le type de production

On distingue industrie (production de biens mateacuteriels ou pour le moins de produits visibles ndash

comme un seacutejour touristique ou un film) et services (fourniture drsquoune prestation immateacuterielle)

Le type de production influence en principe le management de lrsquoentreprise

- lrsquoindustrie neacutecessite (si lrsquoon excepte lrsquoartisanat) un investissement relativement important

une organisation productive stable capable de reacutealiser plusieurs fois des produits identiques

(exemple un modegravele de reacutefrigeacuterateur) ou du moins similaires (exemple un bacirctiment) Le

produit de lrsquoindustrie consomme des matiegraveres et il doit geacuteneacuteralement ecirctre distribueacute jusqursquoau

client

- la production de services peut se satisfaire drsquoun investissement tregraves reacuteduit et neacutecessite un

contact permanent avec le client

Toutefois la standardisation des services et le deacuteveloppement des reacuteseaux informatiques

rapprochent la production de services de celle des biens industriels

- la production drsquoun service reacutepeacutetitif et technique peut imposer une structure lourde et une

organisation tregraves formaliseacutee (voir les grandes socieacuteteacutes drsquoaudit ou de conseil informatique)

Initiation au management copy CRCF ndash J Sornet Page 18 48

- certains services peuvent ecirctre fournis agrave distance sans contact direct avec le client et

distribueacutes par reacuteseau (tenue de comptabiliteacute affacturage gestion clientegravele centre drsquoappel

hellip)

Remarque les services repreacutesentent 75 de lrsquoactiviteacute eacuteconomique franccedilaise

23 ndash La nature de lrsquoorganisation

Les organisations publiques franccedilaises (administrations centrales collectiviteacutes territoriales

hocircpitaux hellip) repreacutesentent une part importante de lrsquoactiviteacute (environ 30 des emplois) La

fonction publique regroupe des organisations aux finaliteacutes diverses et qui ont des problegravemes

de gestion similaires agrave ceux des entreprises auxquelles elles peuvent emprunter des principes

de management Notamment

- pour controcircler les coucircts et assurer la qualiteacute des services

- pour communiquer avec les administreacutes ou les usagers

- pour motiver les personnels et geacuterer les ressources humaines

La transposition directe des techniques de gestion et de management nrsquoest cependant pas

toujours possible car

- la comptabiliteacute publique obeacuteit agrave des regravegles speacutecifiques (proceacutedure budgeacutetaire

notamment)

- le laquo client raquo ne paye pas toujours la prestation du moins directement

- la concurrence est parfois inexistante

- les grandes administrations centraliseacutees sont soumises agrave des choix politiques geacuteneacuteraux

parfois sans connexion eacutevidente avec les besoins opeacuterationnels

- le statut des personnels et les grilles de salaires limitent les possibiliteacutes de gestion des

ressources humaines

Remarque la LOLF (loi organique relative aux lois de finances) est entreacutee en vigueur en

2006 Elle alloue des moyens budgeacutetaires en fonction de programmes et remplace la

reconduction automatique de 90 des budgets Cette reacuteforme se heurte toutefois agrave la

lourdeur des grands ministegraveres ougrave la complexiteacute des activiteacutes est difficile agrave

appreacutehender et ougrave des inerties culturelles peuvent exister agrave tout niveau

Les associations loi de 1901 peuvent avoir une activiteacute comparable agrave celle de grandes

entreprises (voir par exemple les associations de santeacute ou professionnelles) et leur

management est alors similaire malgreacute lrsquoabsence de but lucratif (les beacuteneacutefices ne sont pas

distribuables) Elles ont drsquoailleurs en France un poids eacuteconomique important (elles emploient

environ 1 600 000 salarieacutes)

Cependant lrsquoadheacutesion agrave un systegraveme de valeurs fondateur de lrsquoassociation ou la limite de

lrsquoautoriteacute (quand un volant de beacuteneacutevoles important participe agrave lrsquoactiviteacute) peut introduire des

nuances

- le renforcement des objectifs socieacutetaux

- la faiblesse des relations hieacuterarchiques

- des contraintes de gestion du temps des beacuteneacutevoles

- des modaliteacutes particuliegraveres de recrutement et de motivation des dirigeants

24 ndash Les facteurs contingents

La theacuteorie de la contingence montre qursquoune structure drsquoentreprise nrsquoest efficace que dans

une situation deacutetermineacutee et qursquoil nrsquoexiste que des solutions de management construites dans

un contexte preacutecis

Le management doit ainsi srsquoadapter agrave des facteurs contingents qui ne peuvent ecirctre

controcircleacutes du moins agrave bregraveve eacutecheacuteance Ces facteurs sont par exemple

- lrsquoancienneteacute de lrsquoentreprise (plus elle est ancienne plus lrsquoentreprise a tendance agrave reacutepeacuteter

des comportements eacuteprouveacutes)

Initiation au management copy CRCF ndash J Sornet Page 19 48

- la taille de lrsquoentreprise (la grande entreprise a une composante administrative plus

deacuteveloppeacutee)

- le systegraveme de production (tregraves standardiseacute complexe automatiseacute hellip)

- lrsquoenvironnement

3 ndash Le management et les parties prenantes

Lrsquoentreprise a pour vocation premiegravere de mettre des produits agrave disposition de ses clients en

reacutealisant un profit Pour y arriver elle doit aussi satisfaire ses parties prenantes salarieacutes

actionnaires fournisseurs hellip

Est partie prenante agrave lrsquoentreprise laquo tout groupe ou individu qui peut ecirctre affecteacute ou est

affecteacute par les buts de lrsquoorganisation hellip raquo (Freeman ndash 1984)

Les parties prenantes attendent agrave des degreacutes divers de profiter drsquoune creacuteation de valeur en

provenance de lrsquoentreprise qui doit reacutepondre agrave ces attentes pour assurer sa peacuterenniteacute ou

favoriser son deacuteveloppement

On distingue les parties prenantes primaires ou principales qui sont essentielles agrave lrsquoentreprise

et qui ont geacuteneacuteralement une relation formelle avec elle (clients associeacutes et actionnaires

precircteurs salarieacutes fournisseurs collectiviteacutes) et les parties prenantes secondaires dont

lrsquoinfluence est diffuse (groupes de pression associations meacutedias instances europeacuteennes

agences de notation hellip)

Remarque la consideacuteration de lrsquoensemble des parties prenantes (laquo stakeholders raquo - les

deacutepositaires) fait contrepoids agrave lrsquoimportance accordeacutee aux seuls actionnaires

(laquo shareholders raquo)

Les organisations nrsquoayant pas drsquoobjectif de profit doivent aussi satisfaire leurs parties

prenantes apporter un service aux usagers dans les meilleures conditions eacuteconomiques

limiter un budget assurer la qualiteacute des relations avec les fournisseurs hellip

Dans cette optique le management doit organiser lrsquoaction de faccedilon agrave eacutequilibrer des forces

parfois divergentes

- le contexte fait pression sur lrsquoorganisation contrainte agrave optimiser ses reacutesultats

- lrsquoorganisation cherche par son action agrave assurer sa peacuterenniteacute son deacuteveloppement (en

reacutealisant des profits dans le cas de lrsquoentreprise) et agrave satisfaire ses parties prenantes

- le management agit en pilotant les actions pour contrebalancer la pression du contexte

Actions de

lrsquoorganisation

Management Contexte

Parties

prenantes

Initiation au management copy CRCF ndash J Sornet Page 20 48

APPLICATIONS MP

MP1 Deacutefinir contingent gestion budgeacutetaire

MP2 Deacuteterminer les parties prenantes drsquoun hocircpital public et leurs principales attentes

Mecircme question pour les organisations suivantes

- SNCF (entreprise publique)

- Peugeot

- MAIF (mutuelle drsquoassurance)

MP3 En les situant dans le cycle des activiteacutes du management trouver les actions agrave mener

dans les situations suivantes

- baisse de 10 des ventes dans une entreprise industrielle (produits meacutenagers le reacuteseau de

distribution vient drsquoecirctre reacuteorganiseacute)

- idem dans une entreprise de vente par correspondance soumise agrave la concurrence internet

(les ventes stagnaient depuis six mois malgreacute les efforts promotionnels)

- augmentation des deacutelais drsquoattente des consultations dans une clinique (lrsquohocircpital voisin a

fermeacute son service drsquourgences)

Initiation au management copy CRCF ndash J Sornet Page 21 48

ELEMENTS DE CORRIGE MP

MP1 Deacutefinir (dans le contexte drsquoune entreprise) contingent gestion budgeacutetaire

Contingent = imposeacute par lrsquoexteacuterieur Contingence = effet du hasard de la rencontre de

plusieurs eacuteveacutenements indeacutependants (variables explicatives que lrsquoon ne peut influencer)

Gestion budgeacutetaire = technique drsquoadministration des entreprises srsquoappuyant sur des

preacutevisions dont on deacuteduit apregraves accord des responsables des attributions de moyens sur une

dureacutee limiteacutee Une analyse reacuteguliegravere des eacutecarts entre preacutevisions et reacutealisations permet ensuite

le pilotage des activiteacutes Le budget est un cadre incitatif

La laquo planification budgeacutetaire raquo consiste agrave traduire en budgets une planification strateacutegique

avec systegraveme de reporting

MP2 Deacuteterminer les parties prenantes drsquoun hocircpital public et leurs principales attentes

Mecircme question pour les organisations suivantes

- SNCF (entreprise publique)

- Peugeot

- MAIF (mutuelle drsquoassurance)

Hocircpital

- patients (qualiteacute des soins)

- CNAM (baisse des coucircts)

- collectiviteacute locale (service aux administreacutes)

- eacutetat (ameacutenagement du territoire maicirctrise des budgets optimisation)

- employeacutes (salaire conditions de travail et satisfaction)

- fournisseurs ndash pharmacie autres (CA paiement reacutegulier)

- associations de patients (qualiteacute proximiteacute des soins)

SNCF

- usagers et associations drsquousagers (proximiteacute reacutegulariteacute prix du service)

- reacuteseau ferreacute de France (optimisation des lignes paiement adapteacute)

- fournisseurs (CA paiement reacutegulier)

- employeacutes (salaire conditions de travail seacutecuriteacute de lrsquoemploi)

- eacutetat (ameacutenagement du territoire)

- collectiviteacutes locales (service)

Peugeot

- clients (qualiteacute prix SAV relation commerciale)

- fournisseurs (CA reacutegulariteacute de lrsquoactiviteacute)

- employeacutes (salaire conditions de travail seacutecuriteacute de lrsquoemploi)

- eacutetat (taxes)

- collectiviteacute locale (emploi dynamisation eacuteconomique preacuteservation de lrsquoenvironnement)

- associations de protection de lrsquoenvironnement (activiteacute propre baisse des eacutemissions

nouvelles eacutenergies)

MAIF

- socieacutetaires (protection relation assureur tarif mesureacute)

- professionnels de lrsquoautomobile et autres (agreacutement marge de manœuvre reacuteparations tarifs

eacuteleveacutes)

- fournisseurs (CA paiement reacutegulier)

- eacutetat (taxes engagement pour la seacutecuriteacute)

- employeacutes (salaire conditions de travail seacutecuriteacute de lrsquoemploi)

Initiation au management copy CRCF ndash J Sornet Page 22 48

MP3 En les situant dans le cycle des activiteacutes du management trouver les actions agrave mener

dans les situations suivantes

- baisse de 10 des ventes dans une entreprise industrielle (produits meacutenagers le reacuteseau de

distribution vient drsquoecirctre reacuteorganiseacute)

Adapter le pilotage motiver cadrer si insuffisant retoucher une organisation deacutefectueuse

- idem dans une entreprise de vente par correspondance soumise agrave la concurrence internet

(les ventes stagnaient depuis six mois malgreacute les efforts promotionnels)

Voir pilotage et organisation si une eacutevolution du meacutetier a deacutejagrave eacuteteacute initialiseacutee Sinon re-

conception (adaptation au nouveau contexte) puis planification et reacuteorganisation

- augmentation des deacutelais drsquoattente des consultations dans une clinique (lrsquohocircpital voisin a

fermeacute son service drsquourgences)

Organisation Si insuffisant planification (nouveaux objectifs)

Initiation au management copy CRCF ndash J Sornet Page 23 48

ORGANISATION ET PROCESSUS

La performance de lrsquoentreprise deacutepend de son organisation et de son aptitude agrave produire

aux meilleures conditions Nous allons montrer comment organisation formelle et processus

de production peuvent contribuer agrave cette performance

1 ndash Vers lrsquooptimum

11 ndash Les eacuteconomies occidentales jusqursquoaux anneacutees 70

Jusqursquoen 1945 le principal problegraveme des entreprises eacutetait de produire des biens en quantiteacute

suffisante agrave un prix compatible avec le marcheacute Les grandes entreprises se sont multiplieacutees et

la standardisation a permis de reacuteduire les coucircts (exemple deacuteveloppement de Ford et de la

production agrave la chaicircne de 1908 agrave 1920 qui a permis une baisse du prix des voitures des 23)

On parle de laquo production pousseacutee vers le marcheacute raquo

Cette croissance de la production peu reacuteguleacutee a eacuteteacute marqueacutee par des surproductions en

1910 et 1920 puis par la crise de 1929 qui a prolongeacute ses effets jusqursquoagrave la guerre

De 1945 agrave 1975 environ (les laquo trente glorieuses raquo) la reconstruction la croissance de la

consommation de masse de nouvelles technologies et les eacutechanges internationaux

alimentent lrsquoeacuteconomie La standardisation srsquoeacutetend aux biens de consommation dont les

coucircts baissent fortement et de nouvelles reacutegulations sociales permettent une eacutevolution sans

heurt des revenus La saturation de certains marcheacutes conduit dans les anneacutees 60 agrave la

deacutemarche laquo marketing raquo et agrave la diffeacuterenciation des produits Le produit est laquo dirigeacute par le

marcheacute raquo mais les entreprises conservent une organisation assez classique et les plus grosses

srsquointernationalisent

12 ndash Lrsquoexpeacuterience japonaise et ses prolongements

Tregraves tocirct apregraves la guerre dans un Japon appauvri le constructeur automobile Toyota a ducirc

faire face agrave une restriction du marcheacute des moyens financiers et productifs et des

approvisionnements La firme a donc innoveacute dans un nouveau systegraveme de production

chassant les laquo gaspillages raquo (temps drsquoattente transports stocks deacutefauts hellip) consideacuterant que

seule la fabrication vendable creacutee de la valeur

Toyota srsquoorganise pour fabriquer la quantiteacute et la qualiteacute de produits juste neacutecessaires agrave la

satisfaction des clients la production est laquo tireacutee par le marcheacute raquo La mise en place de ce

systegraveme qui integravegre les fournisseurs ne sera acheveacutee que dans le milieu des anneacutees 70

En 1973 la hausse du peacutetrole inaugure un ralentissement de la croissance des eacuteconomies

occidentales La concurrence accrue provoque alors un inteacuterecirct pour le systegraveme deacuteveloppeacute

au Japon La production au plus juste se deacuteveloppe ainsi dans lrsquoindustrie automobile agrave partir

des anneacutees 80 et elle se reacutepand encore maintenant dans drsquoautres secteurs

Cette approche qui vise un objectif de zeacutero stock et zeacutero deacutefaut impose la maicirctrise de laquo bout

en bout raquo des processus de production et leur ameacutelioration

Initiation au management copy CRCF ndash J Sornet Page 24 48

2 ndash Organiser lrsquoentreprise

21 ndash Direction et organisation

Diriger une entreprise neacutecessite de lrsquoorganiser (de reacutepartir les tacircches) pour qursquoelle puisse

atteindre ses objectifs Lrsquoorganisation permet de satisfaire un marcheacute en tirant parti des

capaciteacutes actuelles de lrsquoentreprise tout en preacuteparant lrsquoavenir

Lrsquoorganisation reacutesulte freacutequemment drsquoun compromis entre des objectifs situeacutes agrave des niveaux

et des eacutecheacuteances diffeacuterents

Exemples

- le leader des chaises roulantes peut tirer profit de sa structure productive et de son

savoir faire pour entrer sur le marcheacute de la bicyclette eacutelectrique

- ecirctre parfaitement structureacute pour alimenter 90 du marcheacute des disquettes ne preacutepare

pas lrsquoavenir

- srsquoorganiser pour conqueacuterir le marcheacute des tire-bouchons eacutelectriques dans les deux ans

perd de son sens si cela altegravere les moyens neacutecessaires agrave la production drsquoappareils

manuels ancienne mais vitale dont la diminution agrave court terme risque de nuire agrave la

solvabiliteacute de lrsquoentreprise et de la conduire agrave la cessation de paiement

22 ndash Lrsquoorganisation fonctionnelle

La majoriteacute des entreprises adopte une laquo organisation fonctionnelle raquo (celle qui est visible

dans les organigrammes) ougrave des regroupements de personnels et drsquoeacutequipements se font

selon un modegravele hieacuterarchique (laquo line raquo) dans des uniteacutes des services ou des deacutepartements

speacutecialiseacutes Cette organisation peut se deacutecliner agrave lrsquointeacuterieur des divisions des grandes

entreprises quand elles scindent leur activiteacute par zone geacuteographique type drsquoactiviteacute

cateacutegorie de clients hellip

Remarque le terme laquo fonction raquo deacutesigne un rocircle particulier dans le fonctionnement de

lrsquoentreprise

Lrsquoorganisation fonctionnelle diffeacuterencie les activiteacutes de lrsquoentreprise en les regroupant par

meacutetier pour utiliser au mieux les compeacutetences et les moyens (meilleur rendement par la

speacutecialisation lrsquoeacutechange de compeacutetences dans une mecircme uniteacute ou gracircce agrave des eacuteconomies

drsquoeacutechelle)

23 ndash La notion de processus de production

Un processus de production se deacutefinit par la succession drsquoactiviteacutes permettant de satisfaire

un client en transformant des ressources (mateacuterielles financiegraveres humaines) en un produit

bien ou service Le processus doit creacuteer une valeur reconnue par le client

Un processus peut servir un client interne agrave lrsquoentreprise (par exemple en produisant un

composant intervenant dans plusieurs produits ou par la maintenance des machines) aussi

bien qursquoun client final On distingue usuellement

- les processus opeacuterationnels (ou maicirctres) aussi appeleacutes processus meacutetier (business process)

qui satisfont directement les clients finaux (conception et fabrication de produits vente hellip)

- les processus de support et de management (geacuterer les ressources humaines geacuterer

lrsquoinformation geacuterer les ressources financiegraveres hellip) qui ont les processus opeacuterationnels comme

clients

Toutes les actions internes agrave une organisation peuvent srsquointeacutegrer dans des processus qui

conditionnent directement ou indirectement la capaciteacute de lrsquoorganisation agrave satisfaire le

client final ou lrsquousager

Initiation au management copy CRCF ndash J Sornet Page 25 48

Aborder le fonctionnement de lrsquoentreprise par ses processus (approche processus) permet

de mettre en eacutevidence les chaicircnes drsquoactiviteacutes qui conduisent aux produits leurs

dysfonctionnements leurs coucircts la formation des deacutelais et la souplesse (la flexibiliteacute)

disponible pour satisfaire la clientegravele finale Lrsquoameacutelioration des processus a un impact visible

et direct sur chaque produit proposeacute aux clients

Lrsquoapproche processus provoque une eacutevolution de la faccedilon de travailler

- en faisant peacuteneacutetrer la laquo voix du client raquo au plus profond de lrsquoentreprise (et plus seulement

dans les services commerciaux et marketing)

- en mettant en eacutevidence des possibiliteacutes de rationalisation (par regroupement ou impartition

de certaines activiteacutes)

Remarque lrsquoapproche par les activiteacutes et les processus est agrave lrsquoorigine de la meacutethode

de deacutetermination des coucircts laquo ABC raquo - activity based costing

24 ndash Processus et fonctions

Le processus est transversal Il enchaicircne des activiteacutes qui traversent lrsquoentreprise en particulier

les services ou les deacutepartements drsquoune organisation fonctionnelle

Exemple

La division du travail par fonctions induit une charge de coordination pour assurer le

deacuteroulement du processus Elle peut geacuteneacuterer des attentes des erreurs ou des conflits drsquointeacuterecirct

(lrsquoobservation montre que des dysfonctionnements sont tregraves souvent constateacutes lors du

passage drsquoun service agrave un autre)

Organisation fonctionnelle et approche processus visent toutes deux un optimum

eacuteconomique mais leurs logiques sont diffeacuterentes

- le processus vise la satisfaction des clients (prix qualiteacute deacutelais service)

- le deacutecoupage fonctionnel cherche agrave optimiser les moyens (maximiser lrsquoeffet drsquoexpeacuterience

partager des infrastructures profiter de pocircles de compeacutetences hellip) Il apporte une ossature

hieacuterarchique stable souvent indispensable

Organisation fonctionnelle et approche processus sont donc compleacutementaires dans la

majoriteacute des cas et doivent ecirctre combineacutees judicieusement

APPLICATIONS OP

OP1 Deacutefinir flexibiliteacute systegraveme impartition

OP2 Citer huit exemples drsquoinformations essentielles pour optimiser un processus de

fabrication

Direction

Deacutepartement

commercial

(C)

Deacutepartement

administratif et

financier (AF)

Deacutepartement

Etudes (E)

Deacutepartement

Production (P)

Activiteacute

C-x Activiteacute

AF-x Activiteacute

E-x

Activiteacute

P-x

Processus x

Clie

nt

Initiation au management copy CRCF ndash J Sornet Page 26 48

OP3 Deacutegager les principes du toyotisme preacutesenteacute ci-dessous En quoi ce systegraveme est-il

initiateur de lrsquoapproche processus

Taiichi Ohno et le Toyotisme

1 - Extrait drsquoun article de Jacques BARRAUX - 1993 - LExpansion

Taiichi Ohno (1912 ndash 1990) hellip ne se prenait pas pour un visionnaire mais en imposant une

nouvelle faccedilon de produire il a reacuteinventeacute le management hellip tout le monde a entendu parler

des mots qui ont populariseacute le toyotisme dont il est le pegravere le juste-agrave-temps hellip Autant

doutils conccedilus pour lrsquoautomobile et qui ont aujourdhui une application universelle

hellip Taiichi Ohno jeune ingeacutenieur entre chez Toyota alors simple constructeur de machines

textiles Degraves 1926 apparaicirct la notion de jidoka hellip cest lart de transfeacuterer de lintelligence aux

machines pour mieux libeacuterer lintelligence des hommes Tout le contraire du taylorisme qui

juge la machine moins impreacutevisible que lhomme En 1933 Toyota se lance dans lautomobile

en sinspirant des meacutethodes ameacutericaines Mais en 1935 agrave loccasion dun voyage aux Etats-

Unis leacutetat-major de lentreprise revient fascineacute de sa visite dans un supermarcheacute La notion

de juste-agrave-temps va naicirctre de lobservation dune grande surface un lieu ougrave les clients ne

prennent que ce dont ils ont besoin et ougrave les rayons sont reacuteapprovisionneacutes pour compenser

les quantiteacutes preacuteleveacutees Ainsi le systegraveme Toyota est-il deacutejagrave dans la tecircte de ses dirigeants avant

mecircme la Seconde Guerre mondiale un demi-siegravecle avant la reacutevolution informatique et la

segmentation intensive des marcheacutes

hellip des esprits curieux comme Franccedilois Dalle en France tombent alors sous le charme des

formules et des paraboles de Taiichi Ohno En voici deux eacutechantillons

Penser agrave lenvers Cela signifie combattre les ideacutees reccedilues En lespegravece il sagit du fordisme et

du taylorisme Ohno ne croit pas agrave la planification aux effets deacutechelle et dexpeacuterience Il

propose un systegraveme industriel agrave lenvers qui permette de diversifier les produits et de les

fabriquer en petites quantiteacutes Nous ne devons plus ecirctre des paysans qui accumulent des

stocks mais des chasseurs On nimpose pas loffre On traque la demande et on la gegravere en

continu

Que les valleacutees soient hautes et les montagnes peu eacuteleveacutees Plutocirct que de concentrer tous

les efforts sur une production agrave un moment donneacute mieux vaut se doter de structures flexibles

permettant de passer agrave tout instant dune seacuterie agrave une autre Il faut eacuteviter les ruptures et les

secousses aplanir les cycles entretenir des flux reacuteguliers dactiviteacutes diversifieacutees Ce qui

implique de ne pas enfermer les hommes et les eacutequipements dans des speacutecialisations trop

eacutetroites

La flexibiliteacute le travail en groupe le refus de la dictature des machines la polyvalence et

surtout lattention constante aux signaux eacutemis par le marcheacute nappartiennent plus au

toyotisme Ces notions sont les fondements du nouvel art dorganiser de vendre et de

produire dans lindustrie comme dans les services hellip

2 - Quelques notions cleacutes

Taiichi Ohno a imagineacute la meacutethode des laquo cinq pourquoi raquo qui consiste agrave se poser cinq fois de

suite la question laquo pourquoi raquo sur le mecircme sujet de faccedilon agrave deacutecouvrir la veacuteritable cause

drsquoun problegraveme Cette meacutethode peut ecirctre appliqueacutee agrave tous les niveaux et permettre

notamment aux agents de fabrication de proposer de veacuteritables ameacuteliorations de la

production

La recherche de la qualiteacute totale (pas de deacutefaut des produits pas de rebuts pas de deacutefaut

des processus) accompagne la deacutemarche de Toyota La qualiteacute a un coucirct compenseacute par

des ventes accrues par lrsquoeacuteconomie des mesures palliatives aux deacutefauts

Initiation au management copy CRCF ndash J Sornet Page 27 48

Fiche OP1 ndash Benchmarking et processus

Le laquo benchmarking raquo consiste agrave comparer le fonctionnement de plusieurs systegravemes pour en

faire notamment ressortir les meilleures pratiques (laquo best practices raquo) Cette technique est

utiliseacutee depuis les anneacutees 80 pour ameacuteliorer la performance des entreprises Elle impose agrave

lrsquoentreprise drsquoeacutevaluer et de remettre en question ses propres modes de fonctionnement afin

de les faire eacutevoluer agrave la lueur de ce qui se fait ailleurs

Le benchmarking permet drsquoameacuteliorer les processus agrave moindre risque en fixant des objectifs

baseacutes sur des faits et donc plus facilement accepteacutes

Une classification des processus en tant que base de reacuteflexion a eacuteteacute eacutetablie aux USA par

lrsquolaquo International Benchmarking Clearinghouse raquo de lrsquoAPQC (american productivity and

quality center) en collaboration avec plusieurs dizaines drsquoentreprises

Elle se reacutesume ainsi

Le terme laquo reengineering raquo (la re-conception ou laquo reacuteingeacutenieacuterie raquo) des processus deacutesigne un

projet drsquoameacutelioration radicale des performances (de 20 agrave 50 ou plus) Il neacutecessite une

parfaite adheacutesion de la direction la constitution drsquoune petite eacutequipe de projet brillante

connaissant parfaitement les activiteacutes de lrsquoentreprise et il peut inclure un benchmarking

Le reengineering provoque geacuteneacuteralement la reacuteduction du nombre de niveaux hieacuterarchiques

(laquo delayering raquo) et lrsquoaccroissement du pouvoir de deacutecision des employeacutes (laquo empowerment raquo

ou laquo empouvoirement raquo) Bien qursquoy conduisant parfois il ne doit pas ecirctre confondu avec la

reacuteduction des activiteacutes (laquo downsizing raquo ou restructuration) et lrsquoexternalisation (laquo outsourcing raquo)

Pro

ce

ssu

s

op

eacutera

tio

nn

els

Pro

ce

ssu

s d

e m

an

ag

em

en

t e

t d

e

sup

po

rt

1 ndash

Comprendre

le marcheacute et

les clients (besoins

satisfaction)

2 ndash

Deacutevelopper

vision et

strateacutegie (contexte

concurrence)

3 ndash

Creacuteer

produits

services

processus

(concevoir

ameacuteliorer)

4 ndash

Marketing et

vente

5 ndash

Produire et

livrer (industrie

dont

ameacutelioration

processus)

6 ndash

Produire et

livrer (services)

7 ndash

Facturer et

servir les

clients (apregraves-

vente

reacuteclamations)

8 ndash Deacutevelopper et geacuterer les ressources humaines

9 ndash Geacuterer les systegravemes drsquoinformation

10 ndash Geacuterer les ressources financiegraveres et les actifs

11 ndash Appliquer un programme environnemental

12 ndash Geacuterer les relations exteacuterieures (actionnaires banques lois relations publiques hellip)

13 ndash Geacuterer lrsquoameacutelioration et le changement (eacutevaluer mesurer motiver qualiteacute totale)

Initiation au management copy CRCF ndash J Sornet Page 28 48

Fiche OP2 ndash Lrsquoorganisation par processus

Lrsquoeacutevolution drsquoune organisation aux activiteacutes reacutepeacutetitives vers lrsquoapproche processus est

geacuteneacuteralement progressive et se met en place par paliers

La mise en œuvre drsquoun veacuteritable management par processus doit ecirctre preacuteceacutedeacutee quand

lrsquoactiviteacute de lrsquoentreprise est complexe drsquoun recensement (une laquo cartographie des

processus raquo) pour mettre en eacutevidence les processus ou les familles de processus cleacutes critiques

pour le succegraves de lrsquoentreprise ougrave les efforts seront prioritaires

Des responsables de processus (laquo process owners raquo) sont ensuite deacutesigneacutes

Le responsable doit concevoir ses processus puis apregraves leur mise en œuvre assurer les

coordinations neacutecessaires les ameacuteliorer et les repreacutesenter aupregraves de la direction

Quand une structure par processus est mise en place des opeacuterateurs exeacutecutants

preacuteceacutedemment regroupeacutes dans les fonctions peuvent ecirctre affecteacutes aux processus et

drsquoanciens responsables de fonctions peuvent devenir des experts au service des processus

Lrsquoorganisation par processus peut imposer un degreacute eacuteleveacute drsquointeacutegration des activiteacutes donc

une polyvalence accrue des personnels et une reacuteduction des niveaux hieacuterarchiques

Elle neacutecessite pour le moins des compeacutetences eacutelargies au niveau des responsables de

processus (organisation administration technique hellip) dont le nombre doit rester limiteacute

(quelques dizaines au plus)

Sauf dans de tregraves petites structures lrsquoorganisation par processus se plaque geacuteneacuteralement sur

une structure plus classique

Initiation au management copy CRCF ndash J Sornet Page 29 48

ELEMENTS DE CORRIGE OP

OP1 Deacutefinir

Flexibiliteacute = adaptation au besoin (horaire variable chaicircnes robotiseacutees)

Systegraveme = ensemble organiseacute dans un but boicircte noire (sanguin nerveux meacutetrique laquo D raquo)

Impartition = sous-traitance ou externalisation (seacuteparation) drsquoactiviteacutes faire appel agrave des

partenaires plutocirct que faire soi-mecircme

OP2 Citer huit exemples drsquoinformations essentielles pour orienter lrsquooptimisation drsquoun processus

Montant des stocks (approvisionnements et produits finis)

Temps drsquoattente

Taux drsquoactiviteacute des ateliers

Rebuts

Deacutelai de production

Taux de reacuteclamations clients (qualiteacute)

Temps passeacutes en retouches finales

Turn over

Nombre drsquoarrecircts maladie

Accidents du travail

Dureacutee des arrecircts machines

OP3 Deacutegager les principes du toyotisme preacutesenteacute dans la fiche 31 En quoi ce systegraveme

repose trsquoil sur lrsquoapproche processus

Produire la quantiteacute juste neacutecessaire (agrave la demande) donc eacuteviter les stocks

Flexibiliteacute intelligence des chaicircnes de production

Qualiteacute (eacuteviter le coucirct de la non-qualiteacute)

La notion de processus est implicite ainsi que la chaicircne de valeur client

Initiation au management copy CRCF ndash J Sornet Page 30 48

DEFIS ET TENDANCES DU MANAGEMENT

Les meacutethodes de management se deacuteveloppent pour affronter le contexte eacuteconomique

Ce chapitre preacutesente les deacutefis auxquels le management contemporain doit faire face

1 ndash Lrsquoeacutevolution eacuteconomique contemporaine

A mesure que lrsquoactiviteacute eacuteconomique mondiale srsquoaccroicirct que la technologie eacutevolue les

changements sont de plus en plus rapides Ils introduisent des situations ineacutedites auxquelles les

entreprises doivent srsquoadapter en cherchant de nouvelles solutions de management Les trois

derniegraveres deacutecennies ont eacuteteacute notamment marqueacutees par les pheacutenomegravenes suivants (que nous

listons sans tenir compte des liens pouvant exister entre eux)

Pheacutenomegravene Traduction Effets

Deacute reacuteglementation

globalisation

financiegravere

titrisation

Libre circulation des capitaux accegraves

facile des particuliers au marcheacute

boursier (directement ou par

lrsquointermeacutediaire des OPCVM et SICAV)

Monteacutee en puissance du financement

des entreprises sur le marcheacute boursier

Fonds de pension

(retraites) et fonds

souverains (eacutetats)

Poids boursier important drsquoinvestisseurs

institutionnels qui cherchent un haut

rendement financier (dividendes ou

valorisation boursiegravere)

Pression sur les grandes entreprises

influence sur les strateacutegies

Mondialisation Liberteacute des eacutechanges internationaux Accroissement de la concurrence

recherche drsquoavantages eacuteconomiques

par la deacutelocalisation (biens et

services) la concentration des efforts

(recentrage) problegravemes drsquoemploi

multiplication des transports perte

drsquoinfluence des politiques

Baisse de lrsquoemploi

occidental

(notamment

industriel)

Moins de fabrications fabrications

automatiseacutees recours aux moyens

informatiques

Activiteacute reporteacutee sur le commerce la

conception et les services chocircmage

charge sociale

Restructurations Optimisation des entreprises

abaissement des coucircts augmentation

des marges recherche drsquoune taille

critique (eacuteconomies drsquoeacutechelle poids

sur le marcheacute)

Recentrages externalisations fusions

deacutelocalisations constitution de grands

groupes

NTIC (nouvelles

technologies de

lrsquoinformation et de

la communication)

Mise en œuvre des reacuteseaux (dont

internet) et drsquoapplications

informatiques communicantes

Nouvelles formes de commerce

marcheacute international deacutelocalisation

du travail intellectuel reacuteorganisation

de la distribution

Rareacutefaction relative

des matiegraveres

premiegraveres

Recherche de substituts exploration

miniegravere coucircts drsquoexploitation des

gisements accrus

Augmentation des coucircts variations

erratiques du cours des matiegraveres

deacutestabilisations politiques

Evolution

geacuteopolitique et

eacuteconomique

mondiale

Chute de lrsquoURSS transformation des

eacuteconomies collectivistes pays

eacutemergents (Chine Inde Breacutesil Russie)

()

Accroissement de la population

mondiale (4 agrave 6 7 milliards de 1970 agrave

2008)

Libeacuteralisme sans frein () nouvelles

puissances eacuteconomiques

opportuniteacutes de deacuteveloppement

nouveau partage des ressources

ineacutegaliteacutes baisse du soutien aux PVD

laquo Terrorisme raquo Actions armeacutees pression de groupes

armeacutes non gouvernementaux

Deacutestabilisations reacutegionales charge

des deacutepenses militaires

Deacuteveloppement

durable

Recherche drsquoune croissance eacutequitable

et respectueuse de lrsquoenvironnement

Pression sur les entreprises (eacutetats

associations de consommateurs

eacutecologistes ONG)

() Reacutecemment quelques affaires (Enron laquo subprimes raquo Vivendi Universal Socieacuteteacute

Geacuteneacuterale Airbus par exemple) et agrave plus grande eacutechelle la crise financiegravere de 2008 ont

montreacute les dangers drsquoune libeacuteralisation sans controcircles suffisants

Initiation au management copy CRCF ndash J Sornet Page 31 48

() Des alliances eacuteconomiques naissent entre pays eacutemergents (notamment en

ameacuterique centrale creacuteation de la Banque du Sud en 2008 par exemple) et lrsquoon

commence agrave imaginer une baisse progressive de lrsquoinfluence eacuteconomique des Etats

Unis

2 ndash Les deacutefis actuels du management

21 ndash Les grandes orientations

Lrsquoeacutevolution eacuteconomique suggegravere quelques pistes parfois concurrentes pour lrsquoaction du

manager contemporain On y retrouve au premier plan la construction drsquoune vision qui est

une composante commune du leadership

Objectif du manager

pour lrsquoentreprise

Justification Facteurs de reacuteussite

Construire une vision Eclairer lrsquoavenir de lrsquoentreprise partager

un but souder motiver

Effort de reacuteelle prospection

volontarisme de la direction

bonne communication

Reacuteactiviteacute et flexibiliteacute

(sous tous les aspects

agrave tous niveaux)

Srsquoadapter rapidement au marcheacute Bonne organisation des processus

personnel compeacutetent autonome

et motiveacute structure hieacuterarchique

alleacutegeacutee robotisation

Deacutegager des profits Reacutemuneacuterer les apporteurs de capitaux

srsquoautofinancer

Ajuster coucircts et structures

Exploiter les nouvelles

technologies

Reacuteactiviteacute ajuster coucircts et deacutelais

reacutepondre au marcheacute suivre les clients

Organiser le SI de faccedilon

pertinente eacuteviter le coucirct excessif

drsquoinvestissements trop en

laquo pointe raquo (laquo essuyer les placirctres raquo)

utiliser judicieusement les services

exteacuterieurs

Bacirctir des alliances

(contrats fusions)

Deacutevelopper une activiteacute limiter les coucircts

de transaction () atteindre la taille

critique et de meilleurs rendements se

recentrer sur une activiteacute profitable

Dominer les processus se donner

une identiteacute lisible externaliser se

doter drsquoune capaciteacute financiegravere

suffisante

Valoriser lrsquoimage Attirer les clients favoriser les alliances

donner confiance (apporteurs de fonds

employeacutes clients partenaires socieacuteteacute

civile)

Instaurer des regravegles de

gouvernance inteacutegrer le

deacuteveloppement durable

respecter lrsquoenvironnement

Geacuterer les risques Faire face aux aleacuteas eacuteconomiques et

technologiques (conjoncture politiques

accidents malveillance)

Creacuteer un systegraveme drsquoalerte geacuterer

la crise (reacuteaction raisonneacutee

sceacutenarios poursuite de

lrsquoexploitation dans un contexte

instable) mise en place de

proceacutedures drsquoapprentissage pour

ameacuteliorer les reacuteactions au fil du

temps

Geacuterer le changement Faire face agrave lrsquoeacutevolution de la demande

la pression sur les prix la variation des

performances financiegraveres la

concurrence la globalisation des

marcheacutes lrsquoeacutevolution technologique aux

fusions ou alliances aux changements

de reacuteglementation de direction hellip ()

Bonne communication pour

donner du sens au changement

et obtenir lrsquoadheacutesion du personnel

Rassembler et geacuterer les

connaissances former le

personnel

Innover Garder un avantage concurrentiel se

diffeacuterencier

Veille technologique et

commerciale investissement

Ouverture

internationale

Elargir le marcheacute saisir les opportuniteacutes Veille commerciale partenariats

() La theacuteorie des coucircts de transaction deacuteveloppeacutee par OE Williamson dans les

anneacutees 70 integravegre les coucircts lieacutes au recours au marcheacute (recherche et choix drsquoun

fournisseur neacutegociation reacutedaction de contrat suivi des eacutechanges risque de rupture

Initiation au management copy CRCF ndash J Sornet Page 32 48

drsquoapprovisionnement hellip) On peut en conclure que lrsquointeacutegration de diffeacuterentes

activiteacutes agrave lrsquoentreprise (la laquo firme raquo) preacutesente des avantages Mais des coucircts de

transaction internes doivent aussi ecirctre consideacutereacutes (preacuteparation organisation

surveillance hellip) et certaines formes de coopeacuteration continue avec les fournisseurs

permettent de reacuteduire le coucirct des transactions externes

() drsquoapregraves laquo Les meilleures pratiques de management raquo - Brilman Heacuterard ndash EO

Une eacutetude du Conference Board (2002) liste les deacutefis du management vus par 700 leaders

mondiaux Soit en reacutesumeacute avec indication du score correspondant

1 ndash Fideacuteliser les clients (42)

2 ndash Reacuteduire les coucircts (38)

3 ndash Accroicirctre flexibiliteacute et reacuteactiviteacute (29)

4 ndash Amener les employeacutes agrave adheacuterer aux valeurs et visions de lrsquoentreprise (26)

5 ndash Deacutevelopper et retenir les leaders (25)

6 ndash Geacuterer acquisitions et alliances (24)

7 ndash Accroicirctre lrsquoinnovation (20)

En fin de classement citoyenneteacute et reacuteputation (4) et ameacutelioration de la diversiteacute (3)

22 ndash Les techniques disponibles

Pour faire face aux deacutefis le manager dispose de nouveaux concepts et de nouvelles

techniques Le tableau ci-dessous en donne un reacutesumeacute et indique les domaines qursquoils

influencent principalement

Initiation au management copy CRCF ndash J Sornet Page 33 48

Principaux concepts techniques outils Incidence principale sur

Internet

- e-commerce (commerce eacutelectronique site

entreprise)

- CRM ou GRC (gestion de la relation client)

- e-procurement (gestion des approvisionnements

par le reacuteseau)

- messagerie eacutelectronique

- e-recrutement

Vente accegraves au marcheacute

Relation client reacuteactiviteacute personnalisation

fideacutelisation

Deacutelais coucircts

Communication transfert de donneacutees (piegraveces

jointes) tous domaines

Communication recrutement

Intranet reacuteseau drsquoentreprise SI

- knowledge management (gestion des

connaissances)

- e-learning (apprentissage en ligne)

- plateforme de travail collaboratif (groupware)

- workflow (circulation eacutelectronique de

documents enchaicircnement de processus)

- e-RH portail RH (libre accegraves aux postes agrave

pourvoir informations candidatures hellip)

- PGI (progiciel de gestion inteacutegreacute) ou ERP

Innovation capaciteacute au changement veille

documentaire

Formation du personnel accompagnement des

changements

Coordination communication interne

Coordination

Communication interne (voire internet en

externe) reacuteduction des coucircts climat drsquoentreprise

recrutement plans de carriegraveres hellip

Coucircts fiabiliteacute du systegraveme drsquoinformation deacutelais

processus (continuiteacute inteacutegration)

Logistique inteacutegreacutee

Supply Chain Management (SCM) gestion de la

logistique (incluant les approvisionnements)

Processus deacutelais coucircts

Externalisation

Valorisation du capital humain

GPEC (gestion preacutevisionnelle des emplois et

compeacutetences)

Coaching

Reacuteactiviteacute de lrsquoentreprise conservation des

compeacutetences rendements individuels turn-over

adaptation des compeacutetences motivation

Efficaciteacute individuelle controcircle reacutegulation

progregraves processus

Approche processus

Optimisation des processus

Deacutemarche qualiteacute totale (TQM ndash total quality

management)

Empowerment (empouvoirement)

Benchmarking reacuteingeacutenieacuterie

Coucircts marges qualiteacute deacutelais flexibiliteacute

externalisation eacutelargissement des compeacutetences

organisation

Ameacutelioration des processus (meacutetiers et supports)

Autonomie compeacutetences des employeacutes

Ameacutelioration des processus restructuration

Management par la valeur

Parties prenantes

Satisfaction des parties prenantes financement

motivation collaborations hellip

Collaboration inter organisations

Reacuteseaux drsquoentreprises alliances

EDI (eacutechange de donneacutees informatiseacutees) extranet

Impartition externalisation (outsourcing)

Coucircts recentrage investissements lancement

drsquoactiviteacute

Coucircts reacuteactiviteacute deacutelais relations avec

lrsquoadministration

Coucircts recentrage limitation des investissements

Ethique drsquoentreprise

Gouvernance drsquoentreprise (mode de direction

encadreacute par des regravegles)

Rocircle socieacutetal deacuteveloppement durable

environnement

Image de lrsquoentreprise reacutegulation du top

management relations actionnaires

Image peacutenaliteacutes et amendes objectifs

strateacutegiques

Initiation au management copy CRCF ndash J Sornet Page 34 48

23 ndash Le rocircle socieacutetal des entreprises

La responsabiliteacute socieacutetale de lrsquoentreprise (RSE) deacutesigne le rocircle qursquoelle prend dans la socieacuteteacute

au-delagrave de son activiteacute purement geacuteneacuteratrice de profit On parle aussi drsquoentreprise citoyenne

La RSE est indissociable du deacuteveloppement durable de porteacutee mondiale et dont les trois

piliers sont

- eacuteconomique (favoriser le deacuteveloppement les eacutechanges internationaux)

- social (accegraves aux soins eacuteducation conditions de travail hellip)

- environnemental (pollution preacuteservation des ressources hellip)

La RSE integravegre notamment une preacuteoccupation sociale de lrsquoentreprise vis-agrave-vis de ses salarieacutes

(seacutecuriteacute et santeacute au travail juste reacutemuneacuteration deacuteveloppement personnel hellip) Elle conduit agrave

tenir compte dans le management drsquoune vision exteacuterieure agrave lrsquoentreprise qui peut avoir des

reacutepercussions possibles sur son activiteacute eacuteconomique

Lrsquoentreprise peut aussi tirer avantage drsquoune deacutemarche responsable par la baisse de certains

coucircts (plus faibles consommations drsquoeacutenergies reacuteduction des transports hellip)

Le rocircle socieacutetal de lrsquoentreprise a eacuteteacute reconnu en France par la loi laquo NRE raquo de 2001 (loi sur les

nouvelles reacutegulations eacuteconomiques) qui oblige les socieacuteteacutes franccedilaise coteacutees sur un marcheacute

reacuteglementeacute agrave rendre compte dans leur rapport annuel de leur gestion sociale et

environnementale au travers de leur activiteacute

Article 116 de la loi Le rapport viseacute agrave larticle L 225-102 rend compte hellip laquo Il comprend

eacutegalement des informations dont la liste est fixeacutee par deacutecret en Conseil dEtat sur la

maniegravere dont la socieacuteteacute prend en compte les conseacutequences sociales et

environnementales de son activiteacute Le preacutesent alineacutea ne sapplique pas aux socieacuteteacutes

dont les titres ne sont pas admis aux neacutegociations sur un marcheacute reacuteglementeacute raquo

Une norme ISO 14000 integravegre ces preacuteoccupations et des taxes eacutecologiques sont

progressivement creacutees

3 ndash Le management par la valeur

31 ndash De lrsquoanalyse au management par la valeur

Lrsquoanalyse de la valeur est neacutee en 1947 aux Etats-Unis (General Electrics) Cette technique

consiste agrave eacutelaborer des produits conformes aux attentes de la clientegravele mais sans excegraves pour

trouver un bon compromis entre valeur pour le client et coucirct Le produit optimal est deacutefini agrave

partir drsquoenquecirctes qui deacuteterminent le besoin client (ou plutocirct drsquoun client laquo type raquo)

Exemple il est inutile de concevoir un petit veacutehicule citadin capable de parcourir

500 000 km sans avarie compte tenu des effets de mode et du faible kilomeacutetrage

annuel Par contre le marcheacute peut exiger un fonctionnement sans faille sur 150 000 km

soit dix ans en moyenne ce qui conditionne les coucircts de production

Cette recherche drsquoun ajustement de valeur au besoin des clients eacutetait un preacutecurseur du

management par la valeur qui recherche plus largement la creacuteation de valeur pour

chacune des parties prenantes de lrsquoentreprise tout en lui meacutenageant un reacutesultat suffisant

Plus geacuteneacuteralement le management par la valeur est deacutefini par une norme europeacuteenne (EN

12973)

Le management par la valeur est un style de management particuliegraverement destineacute agrave

mobiliser les individus agrave deacutevelopper les compeacutetences et agrave promouvoir les synergies et

Initiation au management copy CRCF ndash J Sornet Page 35 48

linnovation avec pour objectif la maximisation de la performance globale dun

organisme Le management par la valeur apporte une nouvelle faccedilon dutiliser nombre

de meacutethodes de management existantes Il est en coheacuterence avec le Management

de la qualiteacute

Cette approche du management pose de nombreuses questions notamment quelles

prioriteacutes et quelles valeurs attribuer aux parties prenantes comment appreacutehender la

perception par les parties prenantes de la valeur qui leur est affecteacutee

32 ndash La valeur client

Le processus drsquoeacutelaboration drsquoun produit qui consomme des ressources coucircteuses doit creacuteer

une valeur suffisante pour provoquer lrsquoachat par le client final La production drsquoune valeur

reconnue par le client est vitale pour lrsquoentreprise mais sa deacutetermination est parfois complexe

La valeur du produit perccedilue par le client integravegre des eacuteleacutements en partie subjectifs

- une valeur drsquousage (le produit reacutepond agrave un besoin)

- une valeur drsquoestime (lrsquoimage apporteacutee par le produit un aspect affectif)

- une valeur drsquoeacutechange (deacuteduite de lrsquoespoir de revente du produit)

Valeurs drsquousage drsquoestime et drsquoeacutechange deacutependent implicitement de la qualiteacute (un bien peu

fiable est impropre agrave lrsquousage attendu de mauvaise qualiteacute notoire il nrsquoapporte pas une

image positive et ses deacutefauts connus nuisent agrave sa revente) Une eacutevaluation de la qualiteacute

intervient donc dans la valeur perccedilue du produit

Par ailleurs le client considegravere le coucirct drsquoobtention du produit (les charges qursquoil doit supporter

pour acqueacuterir le produit lrsquoeffort qursquoil doit faire pour trouver le produit et les frais de mise agrave

disposition)

Le prix perccedilu par le client est geacuteneacuteralement supeacuterieur au prix de vente

Le client achegravete theacuteoriquement le produit qui preacutesente la diffeacuterence valeur perccedilue ndash prix

perccedilu la plus favorable ou le meilleur rapport prix perccedilu qualiteacute perccedilue et dans certains

cas celui qui a le prix produit le plus bas

Remarque les valeurs du scheacutema ci-dessus changent durant le cycle de vie du produit

(un nouveau produit peut avoir une valeur perccedilue plus eacuteleveacutee qursquoen fin de vie) La

valeur client ne peut ecirctre eacutevalueacutee que par enquecirctes et ne peut donc ecirctre deacutefinie avec

certitude

La notion de laquo satisfaction client raquo conseacutecutive agrave une vente influence aussi le prix produit et

le prix perccedilu

- lrsquoentreprise gagne sur les coucircts de recherche de clientegravele

- le client nrsquoa pas agrave rechercher un nouveau fournisseur et beacuteneacuteficie drsquoun coucirct drsquoobtention

plus bas

valeur perccedilue client

prix perccedilu client

coucirct produit Marge (valeur creacuteeacutee pour

lrsquoentreprise)

euros

prix produit

Valeur creacuteeacutee

pour le client

Initiation au management copy CRCF ndash J Sornet Page 36 48

La satisfaction du client deacutepend de facteurs qualitatifs aussi divers que la fiabiliteacute du produit

la vitesse de reacuteaction du fournisseur lrsquoattitude des commerciaux lrsquoefficaciteacute du service

apregraves-vente la netteteacute des contrats ou la justesse de la facture

Valeur perccedilue coucirct marge et satisfaction reacutesultent de processus allant de la conception du

produit jusqursquoagrave sa livraison et son apregraves-vente La deacutemarche laquo processus raquo et lrsquolaquo analyse de la

valeur raquo en forccedilant la recherche de solutions efficientes agrave tout niveau administratif

technique commercial et apregraves-vente sont donc neacutecessaires pour bien positionner

lrsquoentreprise sur son marcheacute

Pour autant le risque commercial ne peut jamais ecirctre annuleacute et lrsquooffre de lrsquoentreprise ne

satisfait geacuteneacuteralement pas en milieu concurrentiel tous ses clients potentiels

33 - La creacuteation de valeur pour les autres parties prenantes

Les salarieacutes

La creacuteation drsquoune valeur suffisante pour les salarieacutes est reconnue comme neacutecessaire car des

observations montrent que la satisfaction des clients en deacutepend Moins souvent eacutevoqueacutee en

peacuteriode de chocircmage elle nrsquoest prioritaire que pour les employeacutes dont lrsquoentreprise souhaite

conserver les compeacutetences

La laquo valeur salarieacute raquo ne comprend pas que le salaire Le sentiment drsquoappartenance agrave un

groupe la reconnaissance lrsquoaccomplissement de soi et la construction professionnelle en

sont des eacuteleacutements importants Comme pour les clients on doit ainsi distinguer la reacutetribution

perccedilue du salaire objectif

Les actionnaires

Lrsquoactionnaire apporte des fonds propres agrave lrsquoentreprise en contrepartie de titres parfois

neacutegociables en bourse et assortis drsquoun droit de vote en assembleacutee geacuteneacuterale La valeur

attribueacutee aux actionnaires est servie en termes moneacutetaires (dividende ou augmentation de la

valeur du titre neacutegociable)

Remarque des facteurs non moneacutetaires comme lrsquoimage de lrsquoentreprise qui deacutepend

en partie de sa communication peuvent influencer la deacutecision drsquoachat de vente ou

de conservation des titres par lrsquoactionnaire

Reacutetribution perccedilue euros

Salaire objectif

Avantage non

moneacutetaire de

lrsquoemploi

Initiation au management copy CRCF ndash J Sornet Page 37 48

Compte tenu de lrsquoimportance croissante de lrsquoactionnariat dans le financement des grandes

entreprises coteacutees en bourse et notamment des investisseurs institutionnels comme les fonds

de pension des indicateurs speacutecifiques ont eacuteteacute introduits pour appreacutecier la performance des

entreprises vue par les actionnaires Par exemple la valeur ajouteacutee eacuteconomique (EVA reg

economic value added marque deacuteposeacutee de Stern Stewart ou VAE ndash valeur ajouteacutee

eacuteconomique parfois deacutenommeacutee VEC ndash valeur eacuteconomique creacuteeacutee) qui prend en compte le

coucirct du capital

LrsquoEVA correspond tregraves scheacutematiquement au calcul suivant

EVA = (PO) profit opeacuterationnel ndash (C) coucirct du capital X (CE) capitaux employeacutes

LrsquoEVA neacutecessite en pratique des retraitements assez complexes Le PO peut se deacuteterminer

selon les principes suivants

- PO = reacutesultat drsquoexploitation (avant inteacuterecircts) ndash impocirct

- PO = beacuteneacutefice courant (tenant compte des inteacuterecircts) + inteacuterecircts ndash eacuteconomie drsquoimpocirct sur les

inteacuterecircts (on exclue les eacuteleacutements financiers et lrsquoimpocirct correspondant) ndash impocirct

- lrsquoimpocirct pris en compte correspond au profit opeacuterationnel consideacutereacute (dans les cas courants agrave

13 du PO)

C = taux moyen de reacutemuneacuteration du capital (reacutesultant par exemple du dividende exigeacute de

certains investisseurs et des taux drsquoemprunts bancaires)

CE = capitaux propres et dettes portant inteacuterecirct

Remarque le profit opeacuterationnel ou reacutesultat opeacuterationnel correspond au NOPAT ndash net

operating profit after tax - anglo-saxon LrsquoEVA est eacutegale au NOPAT diminueacute de la

reacutemuneacuteration des capitaux

Exemple lrsquoentreprise X dispose drsquoun capital de 2 500 000 euro et reacutealise un beacuteneacutefice net

drsquoimpocirct de 450 000 euro (taux 33 13) Un dividende de 6 doit ecirctre verseacute aux

actionnaires et la banque lui a accordeacute un precirct de 1 200 000 euro agrave 4 Les autres

constituants des reacutesultats financier et exceptionnel sont neacutegligeables

Reacutesultat opeacuterationnel = 450 000 + 004 x 1 200 000 x 23 = 482 000 euro

Coucirct du capital = 006 x 2 500 000 + 004 x 1 200 000 x 23 = 182 000 euro

EVA = 300 000 euro

Coucirct moyen pondeacutereacute du capital (C) = (004 x 1 200 000 x23 + 006 x 2 500 000)

3 700 000 Soit 492

Si lrsquoEVA est positive lrsquoentreprise creacuteeacutee de la valeur apregraves reacutemuneacuteration des capitaux et sa

valeur boursiegravere doit augmenter

Lrsquoutilisation de lrsquoEVA comme indicateur influence le management de lrsquoentreprise car il y a

trois moyens pratiques drsquoaugmenter lrsquoEVA

- augmenter le reacutesultat opeacuterationnel

- lancer des investissements ayant une rentabiliteacute supeacuterieure agrave C

- eacuteliminer les activiteacutes ayant une rentabiliteacute infeacuterieure agrave C

Remarque lrsquoutilisation sans nuance de lrsquoEVA comme critegravere de management peut

poser problegraveme Le calcul de lrsquoEVA repose sur des ajustements comptables il est donc

sujet agrave manipulations (provisions capitalisation ou non de la RD hellip) Par ailleurs le

critegravere laquo EVA raquo pris isoleacutement peut conduire agrave chercher la rentabiliteacute agrave court terme agrave

reacuteduire les investissements prospectifs et donc nuire agrave terme au deacuteveloppement de

lrsquoentreprise

Initiation au management copy CRCF ndash J Sornet Page 38 48

Les fournisseurs reccediloivent le paiement de leurs factures plus ou moins rapidement (le deacutelai

de paiement repreacutesente une valeur consentie au fournisseur)

Lrsquoentreprise peut accroicirctre la valeur apporteacutee agrave ses fournisseurs par des actions cibleacutees

comme une contribution agrave la formation de leurs personnels certains transferts de

technologie ou de savoir faire agrave des sous-traitants une coopeacuteration suivie favorisant leur

deacuteveloppement lrsquointeacutegration agrave des campagnes de promotion

A noter que la valeur consentie aux fournisseurs peut avoir une influence sur la qualiteacute et les

deacutelais de livraison des produits

La collectiviteacute reccediloit des taxes et parfois des prestations en nature par deacutefaut ou explicites

(effort de preacuteservation de lrsquoenvironnement ameacutenagement du territoire par les implantations

aide mateacuterielle agrave des projets participation agrave la formation par exemple)

APPLICATIONS DT

DT1 Deacutefinir expliquer deacutereacuteglementation socieacutetal eacuteconomies drsquoeacutechelle coaching EDI

gouvernance

DT2 Deacuteterminer en quoi la deacutemarche TQM srsquoinscrit dans les deacutefis actuels du management

DT3 Apregraves avoir consulteacute les documents ci-dessous extraits du site drsquoAir France

(httpdeveloppement-

durableairfrancecomFRfrlocaldemarcheN4_positionnement_pphtm)

exposer les enjeux et les limites de la RSE et de la gestion des parties prenantes

Initiation au management copy CRCF ndash J Sornet Page 39 48

Dialogue avec les parties prenantes

Initiation au management copy CRCF ndash J Sornet Page 40 48

Attentes des parties prenantes

Initiation au management copy CRCF ndash J Sornet Page 41 48

Creacuteation de valeur pour les parties prenantes

La creacuteation de valeur pour les parties prenantes est au cœur de la strateacutegie du Groupe Le scheacutema de

distribution financiegravere ci-dessous donne un aperccedilu de la distribution des recettes du Groupe aux

diffeacuterentes parties prenantes actionnaires collaborateurs fournisseurs pouvoirs publics

collectiviteacutes locales etc

Initiation au management copy CRCF ndash J Sornet Page 42 48

Fiche DT1 ndash Extrait du sommaire de laquo Problegravemes eacuteconomiques raquo No 2894

La gestion des entreprises bouleverseacutee par les technologies de linternet

Reacutealiteacutes industrielles - Annales des Mines Jean-Michel Yolin

Avec lavegravenement de linternet les processus de conception de production et de vente sont

radicalement remis en cause Quel que soit le secteur dactiviteacute les technologies de linternet

permettent en effet de reacuteduire les deacutelais et de passer dun processus discontinu agrave un processus

continu Lorganisation des entreprises et leur mode de gestion en sont profondeacutement bouleverseacutes

tant au niveau individuel que collectif Linternet rend ainsi possible la reacutealisation dobjectifs que les

entreprises cherchaient agrave atteindre depuis longtemps sans y parvenir meilleure eacutecoute du client

travail sans stocks en flux tendu hieacuterarchies plates autorisant une grande reacuteactiviteacute flexibiliteacute dans

lorganisation et loutil de production acceacuteleacuteration du renouvellement des produits entreprises en

reacuteseau ougrave chacune se recentre sur son cœur de meacutetier etc

Le laquo knowledge management raquo ou comment geacuterer les connaissances

Document de travail du LAMSADE - Michel Grundstein

Peter Drucker lavait preacutedit le capital immateacuteriel eacutetait voueacute agrave devenir un facteur de compeacutetitiviteacute

pour lentreprise La libeacuteralisation des eacutechanges acceacutelegravere les processus de deacutecision de lentreprise

et implique que lassimilation des informations soit agrave la fois de meilleure qualiteacute et plus rapide Ainsi

la fonction qui consiste agrave manager les connaissances au sein de lentreprise savegravere primordiale

Bien que la prise de conscience de limportance du capital immateacuteriel ait eacuteteacute tardive - le concept

de knowledge management est apparu en France aux Etats-Unis et au Japon au milieu des

anneacutees 1990 - agrave lheure actuelle lorganisation de leacutechange dinformations et le partage des

connaissances sont devenus des facteurs cleacutes dune gestion performante de lentreprise Ils

doivent sinscrire dans un projet global destineacute agrave mettre en valeur les savoirs et les savoir-faire

individuels et collectifs

Les leccedilons du laquo coaching raquo pour le management de la qualiteacute

Humanisme et Entreprise - Martine Brasseur

Parmi les nouvelles formes de management en vogue dans les entreprises le coaching figure en

bonne place Appliqueacute au management de la qualiteacute il sagit dune pratique

daccompagnement destineacutee agrave initier et agrave faciliter le processus de deacuteveloppement dun individu

La deacutemarche consiste agrave affirmer que tout individu est en quecircte de qualiteacute agrave condition toutefois

de ne pas lui imposer des contraintes lempecircchant de progresser On considegravere notamment les

erreurs comme potentiellement feacutecondes En deacutefinitive le coach donne au coacheacute la permission

de reacuteussir en lui donnant aussi la permission deacutechouer

Initiation au management copy CRCF ndash J Sornet Page 43 48

Fiche DT2 ndash Management strateacutegique les sept deacutefis agrave relever dici agrave 2016

Extrait drsquoun article du site wwwlentreprisecom -Sabine Blanc - Mis en ligne le 20032007

(httpwwwlentreprisecom325article11977html)

Une eacutetude anglaise publieacutee par lopeacuterateur Orange Grande-Bretagne deacutecrypte la mutation

des formes de travail et les enjeux majeurs pour les entreprises de demain afin decirctre au top

de la compeacutetitiviteacute Voici les challenges-cleacutes pour les managers qui veulent rester dans la

course hellip

1 - Future organisation du travail les quatre laquo mondes raquo possibles

La reacutealiteacute sera probablement un meacutelange de ces quatre sceacutenarios souligne lrsquoeacutetude

Les mondes mutuels Tout se passe dans le cadre des communauteacutes locales vie priveacutee

comme professionnelle Le modegravele coopeacuteratif preacutevaut au lieu du laquo big business raquo Oublieacutes

aussi dans ce systegraveme les trajets pour aller au bureau les gens preacutefegravereront travailler dans de

petites entreprises locales souvent connecteacutees au reacuteseau drsquoautres structures similaires

Les laquo reacutepondants raquo (en anglais laquo replicants raquo) La figure du consultant freelance deviendra

dominante tandis que celle du salarieacute deacuteclinera Il ne sera pas rare de travailler pour plusieurs

entreprises On perdra en seacutecuriteacute de lrsquoemploi en visibiliteacute et en routine ce que lrsquoon gagnera

en liberteacute La majeure partie des tacircches srsquoeffectuera chez soi avec la possibiliteacute de srsquoinstaller

temporairement dans les bureaux de son client du moment Dans un contexte dincertitude

sur lrsquoavenir les travailleurs alterneront peacuteriodes drsquoactiviteacute intense et repos Ce sera agrave eux

drsquoaller vers les entreprises et non lrsquoinverse mecircme si celles-ci devront veiller agrave rester attractives

Les cottages eacutelectroniques Comme ce nom le suggegravere le teacuteleacutetravail deviendrait la norme

univers priveacute et professionnel se confondant Plus besoin de subir une heure de transport les

salarieacutes se logueront de chez eux sur le reacuteseau de lrsquoentreprise Les reacuteunions se tiendront dans

de petits bureaux centraux situeacutes agrave courte distance La flexibiliteacute du temps de travail srsquoimpose

Les salarieacutes disposeront de plus de marge de liberteacute dans leur activiteacute

Les disciples de la nueacutee Cette appellation poeacutetique cache simplement une extension de

lrsquoorganisation actuelle des grandes entreprises avec des salarieacutes se rendant sur un lieu de

travail centraliseacute Le rocircle croissant des technologies de lrsquoinformation multipliera les faccedilons de

collaborer et accroicirctra lrsquoefficaciteacute Le controcircle du travail sera omnipreacutesent La frontiegravere entre

travail et vie priveacutee restera marqueacutee

2 - Sept deacutefis pour les entreprises et leur managers

Quoi qursquoil advienne les entreprises et leurs dirigeants devront concentrer leurs efforts sur sept

points-cleacutes pour srsquoadapter Voici quelques exemples de probleacutematiques souleveacutees par le

rapport et des pistes de solution

Le leadership Les managers devront entre autres savoir persuader et influencer des

travailleurs beaucoup plus indeacutependants Ils auront aussi agrave repenser les niveaux auxquels

prendre les deacutecisions strateacutegiques en haut ou au contraire agrave des degreacutes moins eacuteleveacutes de la

pyramide hieacuterarchique

gt Faire du management une force facilitant les activiteacutes transversales plutocirct que la reacuteduire agrave

la seule fonction de deacutecision

La culture drsquoentreprise Davantage de salarieacutes capables de reacutefleacutechir seront neacutecessaires

tandis que les tacircches qui peuvent ecirctre automatiseacutees ou scripteacutees diminueront Un des

enjeux creacuteer une culture agrave mecircme drsquoattirer et drsquoencourager les personnes preacutesentant ces

qualiteacutes de reacuteflexion requises dans un contexte de compeacutetition accrue et de plus grande

indeacutependance des travailleurs

Initiation au management copy CRCF ndash J Sornet Page 44 48

gt Passer si neacutecessaire drsquoune culture drsquoentreprise forte agrave un mode drsquoengagement plus

consensuel moins rebutant

La marque Conseacutequence du recours croissant agrave lrsquo laquo outsourcing raquo lrsquoimage drsquoune marque

deacutependra plus drsquoagents exteacuterieurs qui ne fonctionnent pas forceacutement selon le mecircme mode

drsquoorganisation Comment garder le controcircle dessus

gt Choisir le mode qui corresponde le plus agrave vos valeurs et preacutevoir un programme de risk

management qui mette en eacutevidence ougrave les conflits sont susceptibles de jaillir

Lrsquoinnovation Plus que jamais il faudra faire face agrave une acceacuteleacuteration du rythme de

lrsquoinnovation en proposant constamment des solutions adapteacutees

gt Tisser des partenariats strateacutegiques avec drsquoautres entreprises pour partager les coucircts et les

fruits de lrsquoinnovation

Le deacutefi opeacuterationnel et technologique De quelle faccedilon controcircler lrsquoinformation crsquoest-agrave-dire

faire en sorte que les bonnes personnes accegravedent facilement agrave une information toujours en

phase tout en maintenant la seacutecuriteacute

gt Recourir agrave des laquo feuilles de route des futurs raquo syntheacutetisant en une page les indicateurs

sociaux et de consommation ainsi que les eacutevolutions technologiques et leacutegislatives qui

influent sur les changements et indiquant comment ils modifient vos marcheacutes vos clients et

votre organisation

La qualiteacute Si de nouveaux proceacutedeacutes ont pu deacutegrader la qualiteacute comme le recours agrave des

centres drsquoappel externaliseacutes drsquoautres ideacutees se sont reacuteveacuteleacutees plus prometteuses comme en

teacutemoigne le succegraves de certaines compagnies aeacuteriennes low cost Elles ont su conjuguer prix

serreacutes et services eacuteleveacutes ce qui devra devenir la norme estime lrsquoeacutetude

gt Continuer de rechercher la qualiteacute Elaborez aussi une bonne prestation service qui inclut

une livraison de qualiteacute voire creacuteez-la en partenariat avec les consommateurs

La leacutegislation La question de la proprieacuteteacute intellectuelle pourrait ecirctre probleacutematique Elle est

deacutejagrave source de conflits comme en teacutemoigne le procegraves pour violation de brevet intenteacute agrave RIM

le fabricant canadien du Blackberry par NTP Que pourra-t-on et que faudra-t-il proteacuteger par

un brevet Il sera eacutegalement neacutecessaire drsquoadapter la leacutegislation aux nouveaux modes

drsquoorganisation

gt Collaborer avec les acteurs du mecircme secteur et les leacutegislateurs pour deacutevelopper les

modegraveles des lieux de travail du futur et bacirctir le droit le plus adeacutequat

Orange a-t-il vu juste dans ses preacutevisions Rendez-vous dans neuf ans pour la reacuteponsehellip

Initiation au management copy CRCF ndash J Sornet Page 45 48

Fiche DT3 ndash Le management par la qualiteacute totale

Extrait drsquoune lettre drsquoinformation du cabinet Baud Accordance Consulting AD2 consultants ndash

2002

1 - Le TQM (Total Quality Management) offre pour lentreprise une vision de la qualiteacute plus

large et transversale

Son principe est simple La finaliteacute de lEntreprise est de deacutevelopper la satisfaction de ses

clients tout en eacutetant beacuteneacuteficiaire cest agrave dire pas agrave nimporte quel prix Elle doit ameacuteliorer sa

rentabiliteacute au travers de la deacutemarche qualiteacute La Qualiteacute Totale vise agrave fournir aux clients

externes et internes une reacuteponse adeacutequate agrave leurs attentes dans le meilleur rapport qualiteacute

prix la meilleure efficience

Elle considegravere pour cela lensemble des processus de lentreprise ayant une incidence sur la

qualiteacute et la satisfaction des clients

Le TQM fait ainsi une large place agrave

la deacutefinition et la planification de la strateacutegie geacuteneacuterale

la coheacuterence de la politique qualiteacute avec la strateacutegie

la deacutemultiplication de la politique qualiteacute dans toutes les directions de lentreprise

la relation client fournisseur interne

la prise en compte de lenvironnement concurrentiel

la consideacuteration de lensemble des risques potentiels financiers sociaux concurrentielshellip

limplication et la motivation du personnel

lanalyse des besoins des clients et le positionnement marketing

la maicirctrise des processus transverses internes

les reacutesultats sous tous ses aspects y compris financiers commerciaux image

De nombreux reacutefeacuterentiels sont relatifs agrave la Qualiteacute Totale hellip Tous ces reacutefeacuterentiels imposent un

questionnement plus profond et indiscret sur le mode de fonctionnement de lentreprise et

son management

helliphellip

2 - LISO 9001 2000 au travers du deacuteploiement des processus (management supports

reacutealisation et ameacutelioration continue) reacutepond quelque peu agrave la mecircme logique

LISO est une ouverture indeacuteniable vers la logique du TQM mais ne se reacutefegravere pas agrave la notion

defficience

Les dirigeants sont cependant sensibles agrave la neacutecessaire reacuteduction des coucircts de non-qualiteacute

et dobtention de la qualiteacute agrave la rentabiliteacute du systegraveme de management de la qualiteacute

mais ne perccediloivent pas toujours la qualiteacute comme une deacutemarche globale

Les deacutemarches qualiteacute commencent bien souvent par la remise en cause de lorganisation

leacutevaluation critique de son efficaciteacute lexamen des processus et la mise en eacutevidence des

lourdeurs administratives

La qualiteacute devient laffaire de tous hellip

Initiation au management copy CRCF ndash J Sornet Page 46 48

Fiche DT4 ndash Le deacuteveloppement durable et la RSE

Extrait du site wwwvigeocom

(httpwwwvigeocomcsr-rating-agencyfrmethodologiecriteres-de-recherche37-

criteres-d-analysehtml)

Deacuteveloppement durable laquo un deacuteveloppement qui reacutepond aux besoins du preacutesent sans compromettre

la capaciteacute des geacuteneacuterations futures de reacutepondre aux leurs raquo (Commission mondiale sur lrsquoenvironnement

et le deacuteveloppement ndash 1987)

Reacutefeacuterentiel drsquoeacutevaluation des entreprises par le groupe Vigeacuteo (le groupe mesure les performances et le

niveau de maicirctrise des risques de responsabiliteacute sociale des entreprises et des organisations - site

wwwvigeocom)

1 Ressources Humaines Ameacutelioration continue des relations professionnelles des relations drsquoemploi et des conditions de travail 2 Droits humains sur les lieux de travail Respect de la liberteacute syndicale et promotion de la neacutegociation collective non discrimination et promotion de lrsquoeacutegaliteacute eacutelimination des formes de travail proscrites (enfants travail forceacute) preacutevention des traitements inhumains ou deacutegradants de type harcegravelements sexuels protection de la vie priveacutee et des donneacutees personnelles 3 Environnement Protection sauvegarde preacutevention des atteintes agrave lenvironnement mise en place drsquoune strateacutegie manageacuteriale approprieacutee eacuteco conception protection de la biodiversiteacute et maicirctrise rationnelle des impacts environnementaux sur lrsquoensemble du cycle de vie des produits ou services

4 Comportements sur les marcheacutes Prise en compte des droits et inteacuterecircts des clients inteacutegration de standards sociaux et environnementaux dans la seacutelection des fournisseurs et sur lrsquoensemble de la chaicircne drsquoapprovisionnement preacutevention effective de la corruption respect des regravegles concurrentielles 5 Gouvernement drsquoentreprise Efficience et probiteacute assurance de lrsquoindeacutependance et de lrsquoefficaciteacute du Conseil drsquoadministration effectiviteacute et efficience des meacutecanismes drsquoaudit et de controcircle et notamment inclusion des risques de responsabiliteacute sociale respect des droits des actionnaires et notamment des minoritaires transparence et rationaliteacute de la reacutemuneacuteration des dirigeants 6 Engagement socieacutetal Effectiviteacute inteacutegration manageacuteriale de lrsquoengagement contribution au deacuteveloppement eacuteconomique et social des territoires drsquoimplantation et de leurs communauteacutes humaines engagements concrets en faveur de la maicirctrise des impacts socieacutetaux des produits et des services contribution transparente et participative agrave des causes drsquointeacuterecirct geacuteneacuteral

Initiation au management copy CRCF ndash J Sornet Page 47 48

ELEMENTS DE CORRIGE DT DT1 Deacutefinir expliquer

Deacutereacuteglementation = suppression des contraintes eacuteconomiques (libre eacutechange des biens et

capitaux)

Socieacutetal = qui se rapporte agrave la structure agrave lrsquoorganisation ou au fonctionnement de la socieacuteteacute

Economies drsquoeacutechelle = reacuteduction des coucircts lieacutee au niveau drsquoactiviteacute (amortissement des

charges fixes)

Coaching = accompagnement de personnes ou deacutequipes pour le deacuteveloppement de leurs

potentiels

EDI = eacutechange de donneacutees informatiseacutees ET standardiseacutees (ex SWIFT bancaire edifact

documents deacuteclaratifs)

Gouvernance = exercice du pouvoir la bonne gouvernance est participative et eacutequitable

conforme agrave lrsquointeacuterecirct commun

DT2 Deacuteterminer en quoi la deacutemarche TQM srsquoinscrit dans les deacutefis actuels du management

Voir notamment fiche 43

Maicirctrise des processus reacuteduction des coucircts reacuteactiviteacute et satisfaction de la clientegravele = faire

face agrave la concurrence

Ameacutelioration de lrsquoimage motivation du personnel

DT3 Apregraves avoir consulteacute les documents ci-dessous extraits du site drsquoAir France

(httpdeveloppement-

durableairfrancecomFRfrlocaldemarcheN4_positionnement_pphtm)

exposer les enjeux et les limites de la RSE et de la gestion des parties prenantes

Trame geacuteneacuterale possible

Introduction

Les deacutefis contemporains (accroissement de la concurrence devenue mondiale recherche

de nouveaux avantages concurrentiels pression de la socieacuteteacute besoin drsquoimage et de projet

lisible pour mener lrsquoentreprise crise et scandales du libeacuteralisme hellip) RSE et PP

Deacuteveloppement (voir cours)

1 ndash Parties prenantes et management par la valeur

PP deacutefinir citer reacutesumer lrsquoavantage rechercheacute (fideacuteliser motiver recherche drsquoalliances

implicites)

PP moyens (dont exemples AF) et meacutethode de management par la valeur (reacutepartie)

2 ndash La responsabiliteacute socieacutetale de lrsquoentreprise

RSE 3 axes

- eacuteconomique (favoriser le deacuteveloppement les eacutechanges internationaux)

- social (accegraves aux soins eacuteducation conditions de travail hellip)

- environnemental (pollution preacuteservation des ressources hellip)

RSE gouvernance drsquoentreprise facteur drsquoimage inteacutegrable dans la deacutemarche PP

Article 116 de la loi Le rapport viseacute agrave larticle L 225-102 rend compte hellip laquo Il comprend

eacutegalement des informations dont la liste est fixeacutee par deacutecret en Conseil dEtat sur la maniegravere

dont la socieacuteteacute prend en compte les conseacutequences sociales et environnementales de son

activiteacute Le preacutesent alineacutea ne sapplique pas aux socieacuteteacutes dont les titres ne sont pas admis aux

neacutegociations sur un marcheacute reacuteglementeacute raquo

Initiation au management copy CRCF ndash J Sornet Page 48 48

RSE exemple AF (ONG fournisseurs)

3 ndash Liens entre PP et RSE

- la RSE introduit de nouvelles PP

- la RSE suppose le respect des PP usuelles (employeacutes clients notamment)

4 - Probleacutematique

- deacutefinir la valeur reacuteellement apporteacutee par une gestion des PP (confusion salaire ndash valeur

idem impocircts hellip ex laquo valeur ajouteacutee raquo)

- communication (neacutecessaire mais aller au-delagrave)

- marginaliteacute des deacutepenses RSE (efficaciteacute sinceacuteriteacute de lrsquoengagement marge de manœuvre)

- charge RSE reporteacutee sur des tiers (ex fournisseurs AF)

- inteacutegration de facteurs non visibles en comptabiliteacute (pertes drsquoemploi nuisances hellip)

Conclusion

Voies incontournables mais pouvant nrsquoavoir qursquoun effet superficiel et temporaire Voir utiliteacute

drsquoaccompagnement leacutegislatif de regravegles de gouvernance

Initiation au management copy CRCF ndash J Sornet Page 16 48

LE MANAGEMENT EN PRATIQUE

Pour assumer sa fonction le management doit couvrir sans discontinuiteacute lrsquoensemble de

lrsquoorganisation et inteacutegrer de nombreux facteurs dont nous allons reacutesumer lrsquoessentiel

1 ndash Les fonctions et activiteacutes du management

Pour Henri Fayol la fonction drsquoadministration de lrsquoentreprise (son management) reposait sur

cinq actions preacutevoir organiser commander coordonner et controcircler (laquo PO3C raquo)

Nous distinguerons cinq activiteacutes de management

- la conception (au plus haut niveau finaliteacute but ou vocation de lrsquoorganisation

meacutetiers dimension politique de croissance hellip)

- la planification (deacutefinition des objectifs eacutecheacuteances)

- lrsquoorganisation (reacutepartition du travail choix des modes de coordination)

- le pilotage de lrsquoaction opeacuterationnelle (motivation animation encadrement

assistance)

- lrsquoeacutevaluation (controcircle des reacutesultats obtenus ajustements)

Dans chacune de ces activiteacutes des deacutecisions et des arbitrages sont neacutecessaires avec des

enjeux plus ou moins importants

Remarques

- Les cinq activiteacutes du management peuvent se retrouver agrave tout niveau de

management si lrsquoentreprise laisse une certaine autonomie de deacutecision agrave ses diffeacuterentes

uniteacutes La conception est naturellement du ressort de la direction geacuteneacuterale et des

conseils drsquoadministration mais elle peut ecirctre preacutesente pregraves du terrain (latitude laisseacutee agrave

une filiale ou agrave un magasin par exemple) De mecircme lrsquoorganisation du travail concerne

un atelier mais aussi la direction qui structure lrsquoentreprise pour assurer ses activiteacutes sa

production

- La planification deacutefinit des objectifs ou des axes strateacutegiques (choix de produits

modaliteacutes de deacuteveloppement des ventes implantations alliances hellip) et les traduit en

donneacutees de gestion preacutevisionnelles syntheacutetiques et eacutechelonneacutees dans le temps afin de

valider les objectifs et de fixer des repegraveres

- Un laquo business plan raquo (plan drsquoaffaires)est notamment lrsquoeacutequivalent de la planification

dans le cas de creacuteation drsquoentreprise ou pour la preacutesentation de tout projet drsquoactiviteacute

Les activiteacutes du management srsquoinscrivent dans des cycles qui peuvent ecirctre scheacutematiseacute

comme suit (lrsquoeacutevaluation peut entraicircner une reacutevision du pilotage de lrsquoorganisation ou des

objectifs sans que lrsquoentreprise ne soit fondamentalement remise en cause)

conception

planification

organisation

pilotage

eacutevaluation

Initiation au management copy CRCF ndash J Sornet Page 17 48

2 ndash Les contextes de management

Le management est influenceacute par son contexte qui justifie des objectifs une organisation

des meacutethodes

Par exemple lrsquoentreprise admet de nombreuses variantes selon sa taille sa forme juridique

son controcircle par lrsquoeacutetat (entreprises publiques) ou par des inteacuterecircts priveacutes Il en va de mecircme des

organismes administratifs qui peuvent deacutependre de directives nationales ou reacutegionales des

associations qui ont des activiteacutes drsquoampleur tregraves variable

21 ndash La dimension de lrsquoentreprise

La dimension drsquoune entreprise se mesure principalement en fonction de son effectif ou de

son chiffre drsquoaffaires Des seuils sont deacutefinis par divers organismes et exploiteacutes agrave des fins

statistiques ou pour la deacutetermination de certaines obligations sociales ou fiscales

(repreacutesentation du personnel cotisations hellip) Il nrsquoy a bien entendu pas de laquo barriegravere de

tailleraquo absolue conditionnant le management drsquoune entreprise

LrsquoUE preacuteconise de distinguer les micro ndash entreprises (jusqursquoagrave 9 salarieacutes) les TPE ndash tregraves petites

entreprises (moins de 20 salarieacutes) les petites entreprises (moins de 50) et les moyennes

entreprises (de 50 agrave 250) Cependant les PME sont parfois situeacutees entre 10 et 500 salarieacutes

Remarques

- en France environ 40 des entreprises emploient de 1 agrave 50 salarieacutes (ce qui repreacutesente

plus de 50 des emplois) et 59 nrsquoen ont aucun

le pays compte environ 2 600 000 entreprises dont moins de 1 ont 250 employeacutes et

plus

- ancienneteacute et taille de lrsquoentreprise sont lieacutees si lrsquoon eacutecarte les restructurations et autres

eacutevolutions drsquoentreprises existantes

La dimension de lrsquoentreprise a une influence sur lrsquoorganisation et le laquo style raquo de son

management

- les PME sont souvent entrepreneuriales (les dirigeants eacutegalement apporteurs de capitaux

sont totalement engageacutes dans la marche de lrsquoentreprise) Elles ont une gestion flexible peu

formaliseacutee plus qualitative que quantitative Les PME sont freacutequemment focaliseacutees sur un seul

type drsquoactiviteacute Pour ne pas alourdir leur structure elles ont tendance agrave sous-traiter les

activiteacutes speacutecialiseacutees ne correspondant pas agrave leur meacutetier de base

- les grandes entreprises sont manageacuteriales (les dirigeants sont nommeacutes par les actionnaires

en raison de leurs compeacutetences) et moins reacuteactives

22 ndash Le type de production

On distingue industrie (production de biens mateacuteriels ou pour le moins de produits visibles ndash

comme un seacutejour touristique ou un film) et services (fourniture drsquoune prestation immateacuterielle)

Le type de production influence en principe le management de lrsquoentreprise

- lrsquoindustrie neacutecessite (si lrsquoon excepte lrsquoartisanat) un investissement relativement important

une organisation productive stable capable de reacutealiser plusieurs fois des produits identiques

(exemple un modegravele de reacutefrigeacuterateur) ou du moins similaires (exemple un bacirctiment) Le

produit de lrsquoindustrie consomme des matiegraveres et il doit geacuteneacuteralement ecirctre distribueacute jusqursquoau

client

- la production de services peut se satisfaire drsquoun investissement tregraves reacuteduit et neacutecessite un

contact permanent avec le client

Toutefois la standardisation des services et le deacuteveloppement des reacuteseaux informatiques

rapprochent la production de services de celle des biens industriels

- la production drsquoun service reacutepeacutetitif et technique peut imposer une structure lourde et une

organisation tregraves formaliseacutee (voir les grandes socieacuteteacutes drsquoaudit ou de conseil informatique)

Initiation au management copy CRCF ndash J Sornet Page 18 48

- certains services peuvent ecirctre fournis agrave distance sans contact direct avec le client et

distribueacutes par reacuteseau (tenue de comptabiliteacute affacturage gestion clientegravele centre drsquoappel

hellip)

Remarque les services repreacutesentent 75 de lrsquoactiviteacute eacuteconomique franccedilaise

23 ndash La nature de lrsquoorganisation

Les organisations publiques franccedilaises (administrations centrales collectiviteacutes territoriales

hocircpitaux hellip) repreacutesentent une part importante de lrsquoactiviteacute (environ 30 des emplois) La

fonction publique regroupe des organisations aux finaliteacutes diverses et qui ont des problegravemes

de gestion similaires agrave ceux des entreprises auxquelles elles peuvent emprunter des principes

de management Notamment

- pour controcircler les coucircts et assurer la qualiteacute des services

- pour communiquer avec les administreacutes ou les usagers

- pour motiver les personnels et geacuterer les ressources humaines

La transposition directe des techniques de gestion et de management nrsquoest cependant pas

toujours possible car

- la comptabiliteacute publique obeacuteit agrave des regravegles speacutecifiques (proceacutedure budgeacutetaire

notamment)

- le laquo client raquo ne paye pas toujours la prestation du moins directement

- la concurrence est parfois inexistante

- les grandes administrations centraliseacutees sont soumises agrave des choix politiques geacuteneacuteraux

parfois sans connexion eacutevidente avec les besoins opeacuterationnels

- le statut des personnels et les grilles de salaires limitent les possibiliteacutes de gestion des

ressources humaines

Remarque la LOLF (loi organique relative aux lois de finances) est entreacutee en vigueur en

2006 Elle alloue des moyens budgeacutetaires en fonction de programmes et remplace la

reconduction automatique de 90 des budgets Cette reacuteforme se heurte toutefois agrave la

lourdeur des grands ministegraveres ougrave la complexiteacute des activiteacutes est difficile agrave

appreacutehender et ougrave des inerties culturelles peuvent exister agrave tout niveau

Les associations loi de 1901 peuvent avoir une activiteacute comparable agrave celle de grandes

entreprises (voir par exemple les associations de santeacute ou professionnelles) et leur

management est alors similaire malgreacute lrsquoabsence de but lucratif (les beacuteneacutefices ne sont pas

distribuables) Elles ont drsquoailleurs en France un poids eacuteconomique important (elles emploient

environ 1 600 000 salarieacutes)

Cependant lrsquoadheacutesion agrave un systegraveme de valeurs fondateur de lrsquoassociation ou la limite de

lrsquoautoriteacute (quand un volant de beacuteneacutevoles important participe agrave lrsquoactiviteacute) peut introduire des

nuances

- le renforcement des objectifs socieacutetaux

- la faiblesse des relations hieacuterarchiques

- des contraintes de gestion du temps des beacuteneacutevoles

- des modaliteacutes particuliegraveres de recrutement et de motivation des dirigeants

24 ndash Les facteurs contingents

La theacuteorie de la contingence montre qursquoune structure drsquoentreprise nrsquoest efficace que dans

une situation deacutetermineacutee et qursquoil nrsquoexiste que des solutions de management construites dans

un contexte preacutecis

Le management doit ainsi srsquoadapter agrave des facteurs contingents qui ne peuvent ecirctre

controcircleacutes du moins agrave bregraveve eacutecheacuteance Ces facteurs sont par exemple

- lrsquoancienneteacute de lrsquoentreprise (plus elle est ancienne plus lrsquoentreprise a tendance agrave reacutepeacuteter

des comportements eacuteprouveacutes)

Initiation au management copy CRCF ndash J Sornet Page 19 48

- la taille de lrsquoentreprise (la grande entreprise a une composante administrative plus

deacuteveloppeacutee)

- le systegraveme de production (tregraves standardiseacute complexe automatiseacute hellip)

- lrsquoenvironnement

3 ndash Le management et les parties prenantes

Lrsquoentreprise a pour vocation premiegravere de mettre des produits agrave disposition de ses clients en

reacutealisant un profit Pour y arriver elle doit aussi satisfaire ses parties prenantes salarieacutes

actionnaires fournisseurs hellip

Est partie prenante agrave lrsquoentreprise laquo tout groupe ou individu qui peut ecirctre affecteacute ou est

affecteacute par les buts de lrsquoorganisation hellip raquo (Freeman ndash 1984)

Les parties prenantes attendent agrave des degreacutes divers de profiter drsquoune creacuteation de valeur en

provenance de lrsquoentreprise qui doit reacutepondre agrave ces attentes pour assurer sa peacuterenniteacute ou

favoriser son deacuteveloppement

On distingue les parties prenantes primaires ou principales qui sont essentielles agrave lrsquoentreprise

et qui ont geacuteneacuteralement une relation formelle avec elle (clients associeacutes et actionnaires

precircteurs salarieacutes fournisseurs collectiviteacutes) et les parties prenantes secondaires dont

lrsquoinfluence est diffuse (groupes de pression associations meacutedias instances europeacuteennes

agences de notation hellip)

Remarque la consideacuteration de lrsquoensemble des parties prenantes (laquo stakeholders raquo - les

deacutepositaires) fait contrepoids agrave lrsquoimportance accordeacutee aux seuls actionnaires

(laquo shareholders raquo)

Les organisations nrsquoayant pas drsquoobjectif de profit doivent aussi satisfaire leurs parties

prenantes apporter un service aux usagers dans les meilleures conditions eacuteconomiques

limiter un budget assurer la qualiteacute des relations avec les fournisseurs hellip

Dans cette optique le management doit organiser lrsquoaction de faccedilon agrave eacutequilibrer des forces

parfois divergentes

- le contexte fait pression sur lrsquoorganisation contrainte agrave optimiser ses reacutesultats

- lrsquoorganisation cherche par son action agrave assurer sa peacuterenniteacute son deacuteveloppement (en

reacutealisant des profits dans le cas de lrsquoentreprise) et agrave satisfaire ses parties prenantes

- le management agit en pilotant les actions pour contrebalancer la pression du contexte

Actions de

lrsquoorganisation

Management Contexte

Parties

prenantes

Initiation au management copy CRCF ndash J Sornet Page 20 48

APPLICATIONS MP

MP1 Deacutefinir contingent gestion budgeacutetaire

MP2 Deacuteterminer les parties prenantes drsquoun hocircpital public et leurs principales attentes

Mecircme question pour les organisations suivantes

- SNCF (entreprise publique)

- Peugeot

- MAIF (mutuelle drsquoassurance)

MP3 En les situant dans le cycle des activiteacutes du management trouver les actions agrave mener

dans les situations suivantes

- baisse de 10 des ventes dans une entreprise industrielle (produits meacutenagers le reacuteseau de

distribution vient drsquoecirctre reacuteorganiseacute)

- idem dans une entreprise de vente par correspondance soumise agrave la concurrence internet

(les ventes stagnaient depuis six mois malgreacute les efforts promotionnels)

- augmentation des deacutelais drsquoattente des consultations dans une clinique (lrsquohocircpital voisin a

fermeacute son service drsquourgences)

Initiation au management copy CRCF ndash J Sornet Page 21 48

ELEMENTS DE CORRIGE MP

MP1 Deacutefinir (dans le contexte drsquoune entreprise) contingent gestion budgeacutetaire

Contingent = imposeacute par lrsquoexteacuterieur Contingence = effet du hasard de la rencontre de

plusieurs eacuteveacutenements indeacutependants (variables explicatives que lrsquoon ne peut influencer)

Gestion budgeacutetaire = technique drsquoadministration des entreprises srsquoappuyant sur des

preacutevisions dont on deacuteduit apregraves accord des responsables des attributions de moyens sur une

dureacutee limiteacutee Une analyse reacuteguliegravere des eacutecarts entre preacutevisions et reacutealisations permet ensuite

le pilotage des activiteacutes Le budget est un cadre incitatif

La laquo planification budgeacutetaire raquo consiste agrave traduire en budgets une planification strateacutegique

avec systegraveme de reporting

MP2 Deacuteterminer les parties prenantes drsquoun hocircpital public et leurs principales attentes

Mecircme question pour les organisations suivantes

- SNCF (entreprise publique)

- Peugeot

- MAIF (mutuelle drsquoassurance)

Hocircpital

- patients (qualiteacute des soins)

- CNAM (baisse des coucircts)

- collectiviteacute locale (service aux administreacutes)

- eacutetat (ameacutenagement du territoire maicirctrise des budgets optimisation)

- employeacutes (salaire conditions de travail et satisfaction)

- fournisseurs ndash pharmacie autres (CA paiement reacutegulier)

- associations de patients (qualiteacute proximiteacute des soins)

SNCF

- usagers et associations drsquousagers (proximiteacute reacutegulariteacute prix du service)

- reacuteseau ferreacute de France (optimisation des lignes paiement adapteacute)

- fournisseurs (CA paiement reacutegulier)

- employeacutes (salaire conditions de travail seacutecuriteacute de lrsquoemploi)

- eacutetat (ameacutenagement du territoire)

- collectiviteacutes locales (service)

Peugeot

- clients (qualiteacute prix SAV relation commerciale)

- fournisseurs (CA reacutegulariteacute de lrsquoactiviteacute)

- employeacutes (salaire conditions de travail seacutecuriteacute de lrsquoemploi)

- eacutetat (taxes)

- collectiviteacute locale (emploi dynamisation eacuteconomique preacuteservation de lrsquoenvironnement)

- associations de protection de lrsquoenvironnement (activiteacute propre baisse des eacutemissions

nouvelles eacutenergies)

MAIF

- socieacutetaires (protection relation assureur tarif mesureacute)

- professionnels de lrsquoautomobile et autres (agreacutement marge de manœuvre reacuteparations tarifs

eacuteleveacutes)

- fournisseurs (CA paiement reacutegulier)

- eacutetat (taxes engagement pour la seacutecuriteacute)

- employeacutes (salaire conditions de travail seacutecuriteacute de lrsquoemploi)

Initiation au management copy CRCF ndash J Sornet Page 22 48

MP3 En les situant dans le cycle des activiteacutes du management trouver les actions agrave mener

dans les situations suivantes

- baisse de 10 des ventes dans une entreprise industrielle (produits meacutenagers le reacuteseau de

distribution vient drsquoecirctre reacuteorganiseacute)

Adapter le pilotage motiver cadrer si insuffisant retoucher une organisation deacutefectueuse

- idem dans une entreprise de vente par correspondance soumise agrave la concurrence internet

(les ventes stagnaient depuis six mois malgreacute les efforts promotionnels)

Voir pilotage et organisation si une eacutevolution du meacutetier a deacutejagrave eacuteteacute initialiseacutee Sinon re-

conception (adaptation au nouveau contexte) puis planification et reacuteorganisation

- augmentation des deacutelais drsquoattente des consultations dans une clinique (lrsquohocircpital voisin a

fermeacute son service drsquourgences)

Organisation Si insuffisant planification (nouveaux objectifs)

Initiation au management copy CRCF ndash J Sornet Page 23 48

ORGANISATION ET PROCESSUS

La performance de lrsquoentreprise deacutepend de son organisation et de son aptitude agrave produire

aux meilleures conditions Nous allons montrer comment organisation formelle et processus

de production peuvent contribuer agrave cette performance

1 ndash Vers lrsquooptimum

11 ndash Les eacuteconomies occidentales jusqursquoaux anneacutees 70

Jusqursquoen 1945 le principal problegraveme des entreprises eacutetait de produire des biens en quantiteacute

suffisante agrave un prix compatible avec le marcheacute Les grandes entreprises se sont multiplieacutees et

la standardisation a permis de reacuteduire les coucircts (exemple deacuteveloppement de Ford et de la

production agrave la chaicircne de 1908 agrave 1920 qui a permis une baisse du prix des voitures des 23)

On parle de laquo production pousseacutee vers le marcheacute raquo

Cette croissance de la production peu reacuteguleacutee a eacuteteacute marqueacutee par des surproductions en

1910 et 1920 puis par la crise de 1929 qui a prolongeacute ses effets jusqursquoagrave la guerre

De 1945 agrave 1975 environ (les laquo trente glorieuses raquo) la reconstruction la croissance de la

consommation de masse de nouvelles technologies et les eacutechanges internationaux

alimentent lrsquoeacuteconomie La standardisation srsquoeacutetend aux biens de consommation dont les

coucircts baissent fortement et de nouvelles reacutegulations sociales permettent une eacutevolution sans

heurt des revenus La saturation de certains marcheacutes conduit dans les anneacutees 60 agrave la

deacutemarche laquo marketing raquo et agrave la diffeacuterenciation des produits Le produit est laquo dirigeacute par le

marcheacute raquo mais les entreprises conservent une organisation assez classique et les plus grosses

srsquointernationalisent

12 ndash Lrsquoexpeacuterience japonaise et ses prolongements

Tregraves tocirct apregraves la guerre dans un Japon appauvri le constructeur automobile Toyota a ducirc

faire face agrave une restriction du marcheacute des moyens financiers et productifs et des

approvisionnements La firme a donc innoveacute dans un nouveau systegraveme de production

chassant les laquo gaspillages raquo (temps drsquoattente transports stocks deacutefauts hellip) consideacuterant que

seule la fabrication vendable creacutee de la valeur

Toyota srsquoorganise pour fabriquer la quantiteacute et la qualiteacute de produits juste neacutecessaires agrave la

satisfaction des clients la production est laquo tireacutee par le marcheacute raquo La mise en place de ce

systegraveme qui integravegre les fournisseurs ne sera acheveacutee que dans le milieu des anneacutees 70

En 1973 la hausse du peacutetrole inaugure un ralentissement de la croissance des eacuteconomies

occidentales La concurrence accrue provoque alors un inteacuterecirct pour le systegraveme deacuteveloppeacute

au Japon La production au plus juste se deacuteveloppe ainsi dans lrsquoindustrie automobile agrave partir

des anneacutees 80 et elle se reacutepand encore maintenant dans drsquoautres secteurs

Cette approche qui vise un objectif de zeacutero stock et zeacutero deacutefaut impose la maicirctrise de laquo bout

en bout raquo des processus de production et leur ameacutelioration

Initiation au management copy CRCF ndash J Sornet Page 24 48

2 ndash Organiser lrsquoentreprise

21 ndash Direction et organisation

Diriger une entreprise neacutecessite de lrsquoorganiser (de reacutepartir les tacircches) pour qursquoelle puisse

atteindre ses objectifs Lrsquoorganisation permet de satisfaire un marcheacute en tirant parti des

capaciteacutes actuelles de lrsquoentreprise tout en preacuteparant lrsquoavenir

Lrsquoorganisation reacutesulte freacutequemment drsquoun compromis entre des objectifs situeacutes agrave des niveaux

et des eacutecheacuteances diffeacuterents

Exemples

- le leader des chaises roulantes peut tirer profit de sa structure productive et de son

savoir faire pour entrer sur le marcheacute de la bicyclette eacutelectrique

- ecirctre parfaitement structureacute pour alimenter 90 du marcheacute des disquettes ne preacutepare

pas lrsquoavenir

- srsquoorganiser pour conqueacuterir le marcheacute des tire-bouchons eacutelectriques dans les deux ans

perd de son sens si cela altegravere les moyens neacutecessaires agrave la production drsquoappareils

manuels ancienne mais vitale dont la diminution agrave court terme risque de nuire agrave la

solvabiliteacute de lrsquoentreprise et de la conduire agrave la cessation de paiement

22 ndash Lrsquoorganisation fonctionnelle

La majoriteacute des entreprises adopte une laquo organisation fonctionnelle raquo (celle qui est visible

dans les organigrammes) ougrave des regroupements de personnels et drsquoeacutequipements se font

selon un modegravele hieacuterarchique (laquo line raquo) dans des uniteacutes des services ou des deacutepartements

speacutecialiseacutes Cette organisation peut se deacutecliner agrave lrsquointeacuterieur des divisions des grandes

entreprises quand elles scindent leur activiteacute par zone geacuteographique type drsquoactiviteacute

cateacutegorie de clients hellip

Remarque le terme laquo fonction raquo deacutesigne un rocircle particulier dans le fonctionnement de

lrsquoentreprise

Lrsquoorganisation fonctionnelle diffeacuterencie les activiteacutes de lrsquoentreprise en les regroupant par

meacutetier pour utiliser au mieux les compeacutetences et les moyens (meilleur rendement par la

speacutecialisation lrsquoeacutechange de compeacutetences dans une mecircme uniteacute ou gracircce agrave des eacuteconomies

drsquoeacutechelle)

23 ndash La notion de processus de production

Un processus de production se deacutefinit par la succession drsquoactiviteacutes permettant de satisfaire

un client en transformant des ressources (mateacuterielles financiegraveres humaines) en un produit

bien ou service Le processus doit creacuteer une valeur reconnue par le client

Un processus peut servir un client interne agrave lrsquoentreprise (par exemple en produisant un

composant intervenant dans plusieurs produits ou par la maintenance des machines) aussi

bien qursquoun client final On distingue usuellement

- les processus opeacuterationnels (ou maicirctres) aussi appeleacutes processus meacutetier (business process)

qui satisfont directement les clients finaux (conception et fabrication de produits vente hellip)

- les processus de support et de management (geacuterer les ressources humaines geacuterer

lrsquoinformation geacuterer les ressources financiegraveres hellip) qui ont les processus opeacuterationnels comme

clients

Toutes les actions internes agrave une organisation peuvent srsquointeacutegrer dans des processus qui

conditionnent directement ou indirectement la capaciteacute de lrsquoorganisation agrave satisfaire le

client final ou lrsquousager

Initiation au management copy CRCF ndash J Sornet Page 25 48

Aborder le fonctionnement de lrsquoentreprise par ses processus (approche processus) permet

de mettre en eacutevidence les chaicircnes drsquoactiviteacutes qui conduisent aux produits leurs

dysfonctionnements leurs coucircts la formation des deacutelais et la souplesse (la flexibiliteacute)

disponible pour satisfaire la clientegravele finale Lrsquoameacutelioration des processus a un impact visible

et direct sur chaque produit proposeacute aux clients

Lrsquoapproche processus provoque une eacutevolution de la faccedilon de travailler

- en faisant peacuteneacutetrer la laquo voix du client raquo au plus profond de lrsquoentreprise (et plus seulement

dans les services commerciaux et marketing)

- en mettant en eacutevidence des possibiliteacutes de rationalisation (par regroupement ou impartition

de certaines activiteacutes)

Remarque lrsquoapproche par les activiteacutes et les processus est agrave lrsquoorigine de la meacutethode

de deacutetermination des coucircts laquo ABC raquo - activity based costing

24 ndash Processus et fonctions

Le processus est transversal Il enchaicircne des activiteacutes qui traversent lrsquoentreprise en particulier

les services ou les deacutepartements drsquoune organisation fonctionnelle

Exemple

La division du travail par fonctions induit une charge de coordination pour assurer le

deacuteroulement du processus Elle peut geacuteneacuterer des attentes des erreurs ou des conflits drsquointeacuterecirct

(lrsquoobservation montre que des dysfonctionnements sont tregraves souvent constateacutes lors du

passage drsquoun service agrave un autre)

Organisation fonctionnelle et approche processus visent toutes deux un optimum

eacuteconomique mais leurs logiques sont diffeacuterentes

- le processus vise la satisfaction des clients (prix qualiteacute deacutelais service)

- le deacutecoupage fonctionnel cherche agrave optimiser les moyens (maximiser lrsquoeffet drsquoexpeacuterience

partager des infrastructures profiter de pocircles de compeacutetences hellip) Il apporte une ossature

hieacuterarchique stable souvent indispensable

Organisation fonctionnelle et approche processus sont donc compleacutementaires dans la

majoriteacute des cas et doivent ecirctre combineacutees judicieusement

APPLICATIONS OP

OP1 Deacutefinir flexibiliteacute systegraveme impartition

OP2 Citer huit exemples drsquoinformations essentielles pour optimiser un processus de

fabrication

Direction

Deacutepartement

commercial

(C)

Deacutepartement

administratif et

financier (AF)

Deacutepartement

Etudes (E)

Deacutepartement

Production (P)

Activiteacute

C-x Activiteacute

AF-x Activiteacute

E-x

Activiteacute

P-x

Processus x

Clie

nt

Initiation au management copy CRCF ndash J Sornet Page 26 48

OP3 Deacutegager les principes du toyotisme preacutesenteacute ci-dessous En quoi ce systegraveme est-il

initiateur de lrsquoapproche processus

Taiichi Ohno et le Toyotisme

1 - Extrait drsquoun article de Jacques BARRAUX - 1993 - LExpansion

Taiichi Ohno (1912 ndash 1990) hellip ne se prenait pas pour un visionnaire mais en imposant une

nouvelle faccedilon de produire il a reacuteinventeacute le management hellip tout le monde a entendu parler

des mots qui ont populariseacute le toyotisme dont il est le pegravere le juste-agrave-temps hellip Autant

doutils conccedilus pour lrsquoautomobile et qui ont aujourdhui une application universelle

hellip Taiichi Ohno jeune ingeacutenieur entre chez Toyota alors simple constructeur de machines

textiles Degraves 1926 apparaicirct la notion de jidoka hellip cest lart de transfeacuterer de lintelligence aux

machines pour mieux libeacuterer lintelligence des hommes Tout le contraire du taylorisme qui

juge la machine moins impreacutevisible que lhomme En 1933 Toyota se lance dans lautomobile

en sinspirant des meacutethodes ameacutericaines Mais en 1935 agrave loccasion dun voyage aux Etats-

Unis leacutetat-major de lentreprise revient fascineacute de sa visite dans un supermarcheacute La notion

de juste-agrave-temps va naicirctre de lobservation dune grande surface un lieu ougrave les clients ne

prennent que ce dont ils ont besoin et ougrave les rayons sont reacuteapprovisionneacutes pour compenser

les quantiteacutes preacuteleveacutees Ainsi le systegraveme Toyota est-il deacutejagrave dans la tecircte de ses dirigeants avant

mecircme la Seconde Guerre mondiale un demi-siegravecle avant la reacutevolution informatique et la

segmentation intensive des marcheacutes

hellip des esprits curieux comme Franccedilois Dalle en France tombent alors sous le charme des

formules et des paraboles de Taiichi Ohno En voici deux eacutechantillons

Penser agrave lenvers Cela signifie combattre les ideacutees reccedilues En lespegravece il sagit du fordisme et

du taylorisme Ohno ne croit pas agrave la planification aux effets deacutechelle et dexpeacuterience Il

propose un systegraveme industriel agrave lenvers qui permette de diversifier les produits et de les

fabriquer en petites quantiteacutes Nous ne devons plus ecirctre des paysans qui accumulent des

stocks mais des chasseurs On nimpose pas loffre On traque la demande et on la gegravere en

continu

Que les valleacutees soient hautes et les montagnes peu eacuteleveacutees Plutocirct que de concentrer tous

les efforts sur une production agrave un moment donneacute mieux vaut se doter de structures flexibles

permettant de passer agrave tout instant dune seacuterie agrave une autre Il faut eacuteviter les ruptures et les

secousses aplanir les cycles entretenir des flux reacuteguliers dactiviteacutes diversifieacutees Ce qui

implique de ne pas enfermer les hommes et les eacutequipements dans des speacutecialisations trop

eacutetroites

La flexibiliteacute le travail en groupe le refus de la dictature des machines la polyvalence et

surtout lattention constante aux signaux eacutemis par le marcheacute nappartiennent plus au

toyotisme Ces notions sont les fondements du nouvel art dorganiser de vendre et de

produire dans lindustrie comme dans les services hellip

2 - Quelques notions cleacutes

Taiichi Ohno a imagineacute la meacutethode des laquo cinq pourquoi raquo qui consiste agrave se poser cinq fois de

suite la question laquo pourquoi raquo sur le mecircme sujet de faccedilon agrave deacutecouvrir la veacuteritable cause

drsquoun problegraveme Cette meacutethode peut ecirctre appliqueacutee agrave tous les niveaux et permettre

notamment aux agents de fabrication de proposer de veacuteritables ameacuteliorations de la

production

La recherche de la qualiteacute totale (pas de deacutefaut des produits pas de rebuts pas de deacutefaut

des processus) accompagne la deacutemarche de Toyota La qualiteacute a un coucirct compenseacute par

des ventes accrues par lrsquoeacuteconomie des mesures palliatives aux deacutefauts

Initiation au management copy CRCF ndash J Sornet Page 27 48

Fiche OP1 ndash Benchmarking et processus

Le laquo benchmarking raquo consiste agrave comparer le fonctionnement de plusieurs systegravemes pour en

faire notamment ressortir les meilleures pratiques (laquo best practices raquo) Cette technique est

utiliseacutee depuis les anneacutees 80 pour ameacuteliorer la performance des entreprises Elle impose agrave

lrsquoentreprise drsquoeacutevaluer et de remettre en question ses propres modes de fonctionnement afin

de les faire eacutevoluer agrave la lueur de ce qui se fait ailleurs

Le benchmarking permet drsquoameacuteliorer les processus agrave moindre risque en fixant des objectifs

baseacutes sur des faits et donc plus facilement accepteacutes

Une classification des processus en tant que base de reacuteflexion a eacuteteacute eacutetablie aux USA par

lrsquolaquo International Benchmarking Clearinghouse raquo de lrsquoAPQC (american productivity and

quality center) en collaboration avec plusieurs dizaines drsquoentreprises

Elle se reacutesume ainsi

Le terme laquo reengineering raquo (la re-conception ou laquo reacuteingeacutenieacuterie raquo) des processus deacutesigne un

projet drsquoameacutelioration radicale des performances (de 20 agrave 50 ou plus) Il neacutecessite une

parfaite adheacutesion de la direction la constitution drsquoune petite eacutequipe de projet brillante

connaissant parfaitement les activiteacutes de lrsquoentreprise et il peut inclure un benchmarking

Le reengineering provoque geacuteneacuteralement la reacuteduction du nombre de niveaux hieacuterarchiques

(laquo delayering raquo) et lrsquoaccroissement du pouvoir de deacutecision des employeacutes (laquo empowerment raquo

ou laquo empouvoirement raquo) Bien qursquoy conduisant parfois il ne doit pas ecirctre confondu avec la

reacuteduction des activiteacutes (laquo downsizing raquo ou restructuration) et lrsquoexternalisation (laquo outsourcing raquo)

Pro

ce

ssu

s

op

eacutera

tio

nn

els

Pro

ce

ssu

s d

e m

an

ag

em

en

t e

t d

e

sup

po

rt

1 ndash

Comprendre

le marcheacute et

les clients (besoins

satisfaction)

2 ndash

Deacutevelopper

vision et

strateacutegie (contexte

concurrence)

3 ndash

Creacuteer

produits

services

processus

(concevoir

ameacuteliorer)

4 ndash

Marketing et

vente

5 ndash

Produire et

livrer (industrie

dont

ameacutelioration

processus)

6 ndash

Produire et

livrer (services)

7 ndash

Facturer et

servir les

clients (apregraves-

vente

reacuteclamations)

8 ndash Deacutevelopper et geacuterer les ressources humaines

9 ndash Geacuterer les systegravemes drsquoinformation

10 ndash Geacuterer les ressources financiegraveres et les actifs

11 ndash Appliquer un programme environnemental

12 ndash Geacuterer les relations exteacuterieures (actionnaires banques lois relations publiques hellip)

13 ndash Geacuterer lrsquoameacutelioration et le changement (eacutevaluer mesurer motiver qualiteacute totale)

Initiation au management copy CRCF ndash J Sornet Page 28 48

Fiche OP2 ndash Lrsquoorganisation par processus

Lrsquoeacutevolution drsquoune organisation aux activiteacutes reacutepeacutetitives vers lrsquoapproche processus est

geacuteneacuteralement progressive et se met en place par paliers

La mise en œuvre drsquoun veacuteritable management par processus doit ecirctre preacuteceacutedeacutee quand

lrsquoactiviteacute de lrsquoentreprise est complexe drsquoun recensement (une laquo cartographie des

processus raquo) pour mettre en eacutevidence les processus ou les familles de processus cleacutes critiques

pour le succegraves de lrsquoentreprise ougrave les efforts seront prioritaires

Des responsables de processus (laquo process owners raquo) sont ensuite deacutesigneacutes

Le responsable doit concevoir ses processus puis apregraves leur mise en œuvre assurer les

coordinations neacutecessaires les ameacuteliorer et les repreacutesenter aupregraves de la direction

Quand une structure par processus est mise en place des opeacuterateurs exeacutecutants

preacuteceacutedemment regroupeacutes dans les fonctions peuvent ecirctre affecteacutes aux processus et

drsquoanciens responsables de fonctions peuvent devenir des experts au service des processus

Lrsquoorganisation par processus peut imposer un degreacute eacuteleveacute drsquointeacutegration des activiteacutes donc

une polyvalence accrue des personnels et une reacuteduction des niveaux hieacuterarchiques

Elle neacutecessite pour le moins des compeacutetences eacutelargies au niveau des responsables de

processus (organisation administration technique hellip) dont le nombre doit rester limiteacute

(quelques dizaines au plus)

Sauf dans de tregraves petites structures lrsquoorganisation par processus se plaque geacuteneacuteralement sur

une structure plus classique

Initiation au management copy CRCF ndash J Sornet Page 29 48

ELEMENTS DE CORRIGE OP

OP1 Deacutefinir

Flexibiliteacute = adaptation au besoin (horaire variable chaicircnes robotiseacutees)

Systegraveme = ensemble organiseacute dans un but boicircte noire (sanguin nerveux meacutetrique laquo D raquo)

Impartition = sous-traitance ou externalisation (seacuteparation) drsquoactiviteacutes faire appel agrave des

partenaires plutocirct que faire soi-mecircme

OP2 Citer huit exemples drsquoinformations essentielles pour orienter lrsquooptimisation drsquoun processus

Montant des stocks (approvisionnements et produits finis)

Temps drsquoattente

Taux drsquoactiviteacute des ateliers

Rebuts

Deacutelai de production

Taux de reacuteclamations clients (qualiteacute)

Temps passeacutes en retouches finales

Turn over

Nombre drsquoarrecircts maladie

Accidents du travail

Dureacutee des arrecircts machines

OP3 Deacutegager les principes du toyotisme preacutesenteacute dans la fiche 31 En quoi ce systegraveme

repose trsquoil sur lrsquoapproche processus

Produire la quantiteacute juste neacutecessaire (agrave la demande) donc eacuteviter les stocks

Flexibiliteacute intelligence des chaicircnes de production

Qualiteacute (eacuteviter le coucirct de la non-qualiteacute)

La notion de processus est implicite ainsi que la chaicircne de valeur client

Initiation au management copy CRCF ndash J Sornet Page 30 48

DEFIS ET TENDANCES DU MANAGEMENT

Les meacutethodes de management se deacuteveloppent pour affronter le contexte eacuteconomique

Ce chapitre preacutesente les deacutefis auxquels le management contemporain doit faire face

1 ndash Lrsquoeacutevolution eacuteconomique contemporaine

A mesure que lrsquoactiviteacute eacuteconomique mondiale srsquoaccroicirct que la technologie eacutevolue les

changements sont de plus en plus rapides Ils introduisent des situations ineacutedites auxquelles les

entreprises doivent srsquoadapter en cherchant de nouvelles solutions de management Les trois

derniegraveres deacutecennies ont eacuteteacute notamment marqueacutees par les pheacutenomegravenes suivants (que nous

listons sans tenir compte des liens pouvant exister entre eux)

Pheacutenomegravene Traduction Effets

Deacute reacuteglementation

globalisation

financiegravere

titrisation

Libre circulation des capitaux accegraves

facile des particuliers au marcheacute

boursier (directement ou par

lrsquointermeacutediaire des OPCVM et SICAV)

Monteacutee en puissance du financement

des entreprises sur le marcheacute boursier

Fonds de pension

(retraites) et fonds

souverains (eacutetats)

Poids boursier important drsquoinvestisseurs

institutionnels qui cherchent un haut

rendement financier (dividendes ou

valorisation boursiegravere)

Pression sur les grandes entreprises

influence sur les strateacutegies

Mondialisation Liberteacute des eacutechanges internationaux Accroissement de la concurrence

recherche drsquoavantages eacuteconomiques

par la deacutelocalisation (biens et

services) la concentration des efforts

(recentrage) problegravemes drsquoemploi

multiplication des transports perte

drsquoinfluence des politiques

Baisse de lrsquoemploi

occidental

(notamment

industriel)

Moins de fabrications fabrications

automatiseacutees recours aux moyens

informatiques

Activiteacute reporteacutee sur le commerce la

conception et les services chocircmage

charge sociale

Restructurations Optimisation des entreprises

abaissement des coucircts augmentation

des marges recherche drsquoune taille

critique (eacuteconomies drsquoeacutechelle poids

sur le marcheacute)

Recentrages externalisations fusions

deacutelocalisations constitution de grands

groupes

NTIC (nouvelles

technologies de

lrsquoinformation et de

la communication)

Mise en œuvre des reacuteseaux (dont

internet) et drsquoapplications

informatiques communicantes

Nouvelles formes de commerce

marcheacute international deacutelocalisation

du travail intellectuel reacuteorganisation

de la distribution

Rareacutefaction relative

des matiegraveres

premiegraveres

Recherche de substituts exploration

miniegravere coucircts drsquoexploitation des

gisements accrus

Augmentation des coucircts variations

erratiques du cours des matiegraveres

deacutestabilisations politiques

Evolution

geacuteopolitique et

eacuteconomique

mondiale

Chute de lrsquoURSS transformation des

eacuteconomies collectivistes pays

eacutemergents (Chine Inde Breacutesil Russie)

()

Accroissement de la population

mondiale (4 agrave 6 7 milliards de 1970 agrave

2008)

Libeacuteralisme sans frein () nouvelles

puissances eacuteconomiques

opportuniteacutes de deacuteveloppement

nouveau partage des ressources

ineacutegaliteacutes baisse du soutien aux PVD

laquo Terrorisme raquo Actions armeacutees pression de groupes

armeacutes non gouvernementaux

Deacutestabilisations reacutegionales charge

des deacutepenses militaires

Deacuteveloppement

durable

Recherche drsquoune croissance eacutequitable

et respectueuse de lrsquoenvironnement

Pression sur les entreprises (eacutetats

associations de consommateurs

eacutecologistes ONG)

() Reacutecemment quelques affaires (Enron laquo subprimes raquo Vivendi Universal Socieacuteteacute

Geacuteneacuterale Airbus par exemple) et agrave plus grande eacutechelle la crise financiegravere de 2008 ont

montreacute les dangers drsquoune libeacuteralisation sans controcircles suffisants

Initiation au management copy CRCF ndash J Sornet Page 31 48

() Des alliances eacuteconomiques naissent entre pays eacutemergents (notamment en

ameacuterique centrale creacuteation de la Banque du Sud en 2008 par exemple) et lrsquoon

commence agrave imaginer une baisse progressive de lrsquoinfluence eacuteconomique des Etats

Unis

2 ndash Les deacutefis actuels du management

21 ndash Les grandes orientations

Lrsquoeacutevolution eacuteconomique suggegravere quelques pistes parfois concurrentes pour lrsquoaction du

manager contemporain On y retrouve au premier plan la construction drsquoune vision qui est

une composante commune du leadership

Objectif du manager

pour lrsquoentreprise

Justification Facteurs de reacuteussite

Construire une vision Eclairer lrsquoavenir de lrsquoentreprise partager

un but souder motiver

Effort de reacuteelle prospection

volontarisme de la direction

bonne communication

Reacuteactiviteacute et flexibiliteacute

(sous tous les aspects

agrave tous niveaux)

Srsquoadapter rapidement au marcheacute Bonne organisation des processus

personnel compeacutetent autonome

et motiveacute structure hieacuterarchique

alleacutegeacutee robotisation

Deacutegager des profits Reacutemuneacuterer les apporteurs de capitaux

srsquoautofinancer

Ajuster coucircts et structures

Exploiter les nouvelles

technologies

Reacuteactiviteacute ajuster coucircts et deacutelais

reacutepondre au marcheacute suivre les clients

Organiser le SI de faccedilon

pertinente eacuteviter le coucirct excessif

drsquoinvestissements trop en

laquo pointe raquo (laquo essuyer les placirctres raquo)

utiliser judicieusement les services

exteacuterieurs

Bacirctir des alliances

(contrats fusions)

Deacutevelopper une activiteacute limiter les coucircts

de transaction () atteindre la taille

critique et de meilleurs rendements se

recentrer sur une activiteacute profitable

Dominer les processus se donner

une identiteacute lisible externaliser se

doter drsquoune capaciteacute financiegravere

suffisante

Valoriser lrsquoimage Attirer les clients favoriser les alliances

donner confiance (apporteurs de fonds

employeacutes clients partenaires socieacuteteacute

civile)

Instaurer des regravegles de

gouvernance inteacutegrer le

deacuteveloppement durable

respecter lrsquoenvironnement

Geacuterer les risques Faire face aux aleacuteas eacuteconomiques et

technologiques (conjoncture politiques

accidents malveillance)

Creacuteer un systegraveme drsquoalerte geacuterer

la crise (reacuteaction raisonneacutee

sceacutenarios poursuite de

lrsquoexploitation dans un contexte

instable) mise en place de

proceacutedures drsquoapprentissage pour

ameacuteliorer les reacuteactions au fil du

temps

Geacuterer le changement Faire face agrave lrsquoeacutevolution de la demande

la pression sur les prix la variation des

performances financiegraveres la

concurrence la globalisation des

marcheacutes lrsquoeacutevolution technologique aux

fusions ou alliances aux changements

de reacuteglementation de direction hellip ()

Bonne communication pour

donner du sens au changement

et obtenir lrsquoadheacutesion du personnel

Rassembler et geacuterer les

connaissances former le

personnel

Innover Garder un avantage concurrentiel se

diffeacuterencier

Veille technologique et

commerciale investissement

Ouverture

internationale

Elargir le marcheacute saisir les opportuniteacutes Veille commerciale partenariats

() La theacuteorie des coucircts de transaction deacuteveloppeacutee par OE Williamson dans les

anneacutees 70 integravegre les coucircts lieacutes au recours au marcheacute (recherche et choix drsquoun

fournisseur neacutegociation reacutedaction de contrat suivi des eacutechanges risque de rupture

Initiation au management copy CRCF ndash J Sornet Page 32 48

drsquoapprovisionnement hellip) On peut en conclure que lrsquointeacutegration de diffeacuterentes

activiteacutes agrave lrsquoentreprise (la laquo firme raquo) preacutesente des avantages Mais des coucircts de

transaction internes doivent aussi ecirctre consideacutereacutes (preacuteparation organisation

surveillance hellip) et certaines formes de coopeacuteration continue avec les fournisseurs

permettent de reacuteduire le coucirct des transactions externes

() drsquoapregraves laquo Les meilleures pratiques de management raquo - Brilman Heacuterard ndash EO

Une eacutetude du Conference Board (2002) liste les deacutefis du management vus par 700 leaders

mondiaux Soit en reacutesumeacute avec indication du score correspondant

1 ndash Fideacuteliser les clients (42)

2 ndash Reacuteduire les coucircts (38)

3 ndash Accroicirctre flexibiliteacute et reacuteactiviteacute (29)

4 ndash Amener les employeacutes agrave adheacuterer aux valeurs et visions de lrsquoentreprise (26)

5 ndash Deacutevelopper et retenir les leaders (25)

6 ndash Geacuterer acquisitions et alliances (24)

7 ndash Accroicirctre lrsquoinnovation (20)

En fin de classement citoyenneteacute et reacuteputation (4) et ameacutelioration de la diversiteacute (3)

22 ndash Les techniques disponibles

Pour faire face aux deacutefis le manager dispose de nouveaux concepts et de nouvelles

techniques Le tableau ci-dessous en donne un reacutesumeacute et indique les domaines qursquoils

influencent principalement

Initiation au management copy CRCF ndash J Sornet Page 33 48

Principaux concepts techniques outils Incidence principale sur

Internet

- e-commerce (commerce eacutelectronique site

entreprise)

- CRM ou GRC (gestion de la relation client)

- e-procurement (gestion des approvisionnements

par le reacuteseau)

- messagerie eacutelectronique

- e-recrutement

Vente accegraves au marcheacute

Relation client reacuteactiviteacute personnalisation

fideacutelisation

Deacutelais coucircts

Communication transfert de donneacutees (piegraveces

jointes) tous domaines

Communication recrutement

Intranet reacuteseau drsquoentreprise SI

- knowledge management (gestion des

connaissances)

- e-learning (apprentissage en ligne)

- plateforme de travail collaboratif (groupware)

- workflow (circulation eacutelectronique de

documents enchaicircnement de processus)

- e-RH portail RH (libre accegraves aux postes agrave

pourvoir informations candidatures hellip)

- PGI (progiciel de gestion inteacutegreacute) ou ERP

Innovation capaciteacute au changement veille

documentaire

Formation du personnel accompagnement des

changements

Coordination communication interne

Coordination

Communication interne (voire internet en

externe) reacuteduction des coucircts climat drsquoentreprise

recrutement plans de carriegraveres hellip

Coucircts fiabiliteacute du systegraveme drsquoinformation deacutelais

processus (continuiteacute inteacutegration)

Logistique inteacutegreacutee

Supply Chain Management (SCM) gestion de la

logistique (incluant les approvisionnements)

Processus deacutelais coucircts

Externalisation

Valorisation du capital humain

GPEC (gestion preacutevisionnelle des emplois et

compeacutetences)

Coaching

Reacuteactiviteacute de lrsquoentreprise conservation des

compeacutetences rendements individuels turn-over

adaptation des compeacutetences motivation

Efficaciteacute individuelle controcircle reacutegulation

progregraves processus

Approche processus

Optimisation des processus

Deacutemarche qualiteacute totale (TQM ndash total quality

management)

Empowerment (empouvoirement)

Benchmarking reacuteingeacutenieacuterie

Coucircts marges qualiteacute deacutelais flexibiliteacute

externalisation eacutelargissement des compeacutetences

organisation

Ameacutelioration des processus (meacutetiers et supports)

Autonomie compeacutetences des employeacutes

Ameacutelioration des processus restructuration

Management par la valeur

Parties prenantes

Satisfaction des parties prenantes financement

motivation collaborations hellip

Collaboration inter organisations

Reacuteseaux drsquoentreprises alliances

EDI (eacutechange de donneacutees informatiseacutees) extranet

Impartition externalisation (outsourcing)

Coucircts recentrage investissements lancement

drsquoactiviteacute

Coucircts reacuteactiviteacute deacutelais relations avec

lrsquoadministration

Coucircts recentrage limitation des investissements

Ethique drsquoentreprise

Gouvernance drsquoentreprise (mode de direction

encadreacute par des regravegles)

Rocircle socieacutetal deacuteveloppement durable

environnement

Image de lrsquoentreprise reacutegulation du top

management relations actionnaires

Image peacutenaliteacutes et amendes objectifs

strateacutegiques

Initiation au management copy CRCF ndash J Sornet Page 34 48

23 ndash Le rocircle socieacutetal des entreprises

La responsabiliteacute socieacutetale de lrsquoentreprise (RSE) deacutesigne le rocircle qursquoelle prend dans la socieacuteteacute

au-delagrave de son activiteacute purement geacuteneacuteratrice de profit On parle aussi drsquoentreprise citoyenne

La RSE est indissociable du deacuteveloppement durable de porteacutee mondiale et dont les trois

piliers sont

- eacuteconomique (favoriser le deacuteveloppement les eacutechanges internationaux)

- social (accegraves aux soins eacuteducation conditions de travail hellip)

- environnemental (pollution preacuteservation des ressources hellip)

La RSE integravegre notamment une preacuteoccupation sociale de lrsquoentreprise vis-agrave-vis de ses salarieacutes

(seacutecuriteacute et santeacute au travail juste reacutemuneacuteration deacuteveloppement personnel hellip) Elle conduit agrave

tenir compte dans le management drsquoune vision exteacuterieure agrave lrsquoentreprise qui peut avoir des

reacutepercussions possibles sur son activiteacute eacuteconomique

Lrsquoentreprise peut aussi tirer avantage drsquoune deacutemarche responsable par la baisse de certains

coucircts (plus faibles consommations drsquoeacutenergies reacuteduction des transports hellip)

Le rocircle socieacutetal de lrsquoentreprise a eacuteteacute reconnu en France par la loi laquo NRE raquo de 2001 (loi sur les

nouvelles reacutegulations eacuteconomiques) qui oblige les socieacuteteacutes franccedilaise coteacutees sur un marcheacute

reacuteglementeacute agrave rendre compte dans leur rapport annuel de leur gestion sociale et

environnementale au travers de leur activiteacute

Article 116 de la loi Le rapport viseacute agrave larticle L 225-102 rend compte hellip laquo Il comprend

eacutegalement des informations dont la liste est fixeacutee par deacutecret en Conseil dEtat sur la

maniegravere dont la socieacuteteacute prend en compte les conseacutequences sociales et

environnementales de son activiteacute Le preacutesent alineacutea ne sapplique pas aux socieacuteteacutes

dont les titres ne sont pas admis aux neacutegociations sur un marcheacute reacuteglementeacute raquo

Une norme ISO 14000 integravegre ces preacuteoccupations et des taxes eacutecologiques sont

progressivement creacutees

3 ndash Le management par la valeur

31 ndash De lrsquoanalyse au management par la valeur

Lrsquoanalyse de la valeur est neacutee en 1947 aux Etats-Unis (General Electrics) Cette technique

consiste agrave eacutelaborer des produits conformes aux attentes de la clientegravele mais sans excegraves pour

trouver un bon compromis entre valeur pour le client et coucirct Le produit optimal est deacutefini agrave

partir drsquoenquecirctes qui deacuteterminent le besoin client (ou plutocirct drsquoun client laquo type raquo)

Exemple il est inutile de concevoir un petit veacutehicule citadin capable de parcourir

500 000 km sans avarie compte tenu des effets de mode et du faible kilomeacutetrage

annuel Par contre le marcheacute peut exiger un fonctionnement sans faille sur 150 000 km

soit dix ans en moyenne ce qui conditionne les coucircts de production

Cette recherche drsquoun ajustement de valeur au besoin des clients eacutetait un preacutecurseur du

management par la valeur qui recherche plus largement la creacuteation de valeur pour

chacune des parties prenantes de lrsquoentreprise tout en lui meacutenageant un reacutesultat suffisant

Plus geacuteneacuteralement le management par la valeur est deacutefini par une norme europeacuteenne (EN

12973)

Le management par la valeur est un style de management particuliegraverement destineacute agrave

mobiliser les individus agrave deacutevelopper les compeacutetences et agrave promouvoir les synergies et

Initiation au management copy CRCF ndash J Sornet Page 35 48

linnovation avec pour objectif la maximisation de la performance globale dun

organisme Le management par la valeur apporte une nouvelle faccedilon dutiliser nombre

de meacutethodes de management existantes Il est en coheacuterence avec le Management

de la qualiteacute

Cette approche du management pose de nombreuses questions notamment quelles

prioriteacutes et quelles valeurs attribuer aux parties prenantes comment appreacutehender la

perception par les parties prenantes de la valeur qui leur est affecteacutee

32 ndash La valeur client

Le processus drsquoeacutelaboration drsquoun produit qui consomme des ressources coucircteuses doit creacuteer

une valeur suffisante pour provoquer lrsquoachat par le client final La production drsquoune valeur

reconnue par le client est vitale pour lrsquoentreprise mais sa deacutetermination est parfois complexe

La valeur du produit perccedilue par le client integravegre des eacuteleacutements en partie subjectifs

- une valeur drsquousage (le produit reacutepond agrave un besoin)

- une valeur drsquoestime (lrsquoimage apporteacutee par le produit un aspect affectif)

- une valeur drsquoeacutechange (deacuteduite de lrsquoespoir de revente du produit)

Valeurs drsquousage drsquoestime et drsquoeacutechange deacutependent implicitement de la qualiteacute (un bien peu

fiable est impropre agrave lrsquousage attendu de mauvaise qualiteacute notoire il nrsquoapporte pas une

image positive et ses deacutefauts connus nuisent agrave sa revente) Une eacutevaluation de la qualiteacute

intervient donc dans la valeur perccedilue du produit

Par ailleurs le client considegravere le coucirct drsquoobtention du produit (les charges qursquoil doit supporter

pour acqueacuterir le produit lrsquoeffort qursquoil doit faire pour trouver le produit et les frais de mise agrave

disposition)

Le prix perccedilu par le client est geacuteneacuteralement supeacuterieur au prix de vente

Le client achegravete theacuteoriquement le produit qui preacutesente la diffeacuterence valeur perccedilue ndash prix

perccedilu la plus favorable ou le meilleur rapport prix perccedilu qualiteacute perccedilue et dans certains

cas celui qui a le prix produit le plus bas

Remarque les valeurs du scheacutema ci-dessus changent durant le cycle de vie du produit

(un nouveau produit peut avoir une valeur perccedilue plus eacuteleveacutee qursquoen fin de vie) La

valeur client ne peut ecirctre eacutevalueacutee que par enquecirctes et ne peut donc ecirctre deacutefinie avec

certitude

La notion de laquo satisfaction client raquo conseacutecutive agrave une vente influence aussi le prix produit et

le prix perccedilu

- lrsquoentreprise gagne sur les coucircts de recherche de clientegravele

- le client nrsquoa pas agrave rechercher un nouveau fournisseur et beacuteneacuteficie drsquoun coucirct drsquoobtention

plus bas

valeur perccedilue client

prix perccedilu client

coucirct produit Marge (valeur creacuteeacutee pour

lrsquoentreprise)

euros

prix produit

Valeur creacuteeacutee

pour le client

Initiation au management copy CRCF ndash J Sornet Page 36 48

La satisfaction du client deacutepend de facteurs qualitatifs aussi divers que la fiabiliteacute du produit

la vitesse de reacuteaction du fournisseur lrsquoattitude des commerciaux lrsquoefficaciteacute du service

apregraves-vente la netteteacute des contrats ou la justesse de la facture

Valeur perccedilue coucirct marge et satisfaction reacutesultent de processus allant de la conception du

produit jusqursquoagrave sa livraison et son apregraves-vente La deacutemarche laquo processus raquo et lrsquolaquo analyse de la

valeur raquo en forccedilant la recherche de solutions efficientes agrave tout niveau administratif

technique commercial et apregraves-vente sont donc neacutecessaires pour bien positionner

lrsquoentreprise sur son marcheacute

Pour autant le risque commercial ne peut jamais ecirctre annuleacute et lrsquooffre de lrsquoentreprise ne

satisfait geacuteneacuteralement pas en milieu concurrentiel tous ses clients potentiels

33 - La creacuteation de valeur pour les autres parties prenantes

Les salarieacutes

La creacuteation drsquoune valeur suffisante pour les salarieacutes est reconnue comme neacutecessaire car des

observations montrent que la satisfaction des clients en deacutepend Moins souvent eacutevoqueacutee en

peacuteriode de chocircmage elle nrsquoest prioritaire que pour les employeacutes dont lrsquoentreprise souhaite

conserver les compeacutetences

La laquo valeur salarieacute raquo ne comprend pas que le salaire Le sentiment drsquoappartenance agrave un

groupe la reconnaissance lrsquoaccomplissement de soi et la construction professionnelle en

sont des eacuteleacutements importants Comme pour les clients on doit ainsi distinguer la reacutetribution

perccedilue du salaire objectif

Les actionnaires

Lrsquoactionnaire apporte des fonds propres agrave lrsquoentreprise en contrepartie de titres parfois

neacutegociables en bourse et assortis drsquoun droit de vote en assembleacutee geacuteneacuterale La valeur

attribueacutee aux actionnaires est servie en termes moneacutetaires (dividende ou augmentation de la

valeur du titre neacutegociable)

Remarque des facteurs non moneacutetaires comme lrsquoimage de lrsquoentreprise qui deacutepend

en partie de sa communication peuvent influencer la deacutecision drsquoachat de vente ou

de conservation des titres par lrsquoactionnaire

Reacutetribution perccedilue euros

Salaire objectif

Avantage non

moneacutetaire de

lrsquoemploi

Initiation au management copy CRCF ndash J Sornet Page 37 48

Compte tenu de lrsquoimportance croissante de lrsquoactionnariat dans le financement des grandes

entreprises coteacutees en bourse et notamment des investisseurs institutionnels comme les fonds

de pension des indicateurs speacutecifiques ont eacuteteacute introduits pour appreacutecier la performance des

entreprises vue par les actionnaires Par exemple la valeur ajouteacutee eacuteconomique (EVA reg

economic value added marque deacuteposeacutee de Stern Stewart ou VAE ndash valeur ajouteacutee

eacuteconomique parfois deacutenommeacutee VEC ndash valeur eacuteconomique creacuteeacutee) qui prend en compte le

coucirct du capital

LrsquoEVA correspond tregraves scheacutematiquement au calcul suivant

EVA = (PO) profit opeacuterationnel ndash (C) coucirct du capital X (CE) capitaux employeacutes

LrsquoEVA neacutecessite en pratique des retraitements assez complexes Le PO peut se deacuteterminer

selon les principes suivants

- PO = reacutesultat drsquoexploitation (avant inteacuterecircts) ndash impocirct

- PO = beacuteneacutefice courant (tenant compte des inteacuterecircts) + inteacuterecircts ndash eacuteconomie drsquoimpocirct sur les

inteacuterecircts (on exclue les eacuteleacutements financiers et lrsquoimpocirct correspondant) ndash impocirct

- lrsquoimpocirct pris en compte correspond au profit opeacuterationnel consideacutereacute (dans les cas courants agrave

13 du PO)

C = taux moyen de reacutemuneacuteration du capital (reacutesultant par exemple du dividende exigeacute de

certains investisseurs et des taux drsquoemprunts bancaires)

CE = capitaux propres et dettes portant inteacuterecirct

Remarque le profit opeacuterationnel ou reacutesultat opeacuterationnel correspond au NOPAT ndash net

operating profit after tax - anglo-saxon LrsquoEVA est eacutegale au NOPAT diminueacute de la

reacutemuneacuteration des capitaux

Exemple lrsquoentreprise X dispose drsquoun capital de 2 500 000 euro et reacutealise un beacuteneacutefice net

drsquoimpocirct de 450 000 euro (taux 33 13) Un dividende de 6 doit ecirctre verseacute aux

actionnaires et la banque lui a accordeacute un precirct de 1 200 000 euro agrave 4 Les autres

constituants des reacutesultats financier et exceptionnel sont neacutegligeables

Reacutesultat opeacuterationnel = 450 000 + 004 x 1 200 000 x 23 = 482 000 euro

Coucirct du capital = 006 x 2 500 000 + 004 x 1 200 000 x 23 = 182 000 euro

EVA = 300 000 euro

Coucirct moyen pondeacutereacute du capital (C) = (004 x 1 200 000 x23 + 006 x 2 500 000)

3 700 000 Soit 492

Si lrsquoEVA est positive lrsquoentreprise creacuteeacutee de la valeur apregraves reacutemuneacuteration des capitaux et sa

valeur boursiegravere doit augmenter

Lrsquoutilisation de lrsquoEVA comme indicateur influence le management de lrsquoentreprise car il y a

trois moyens pratiques drsquoaugmenter lrsquoEVA

- augmenter le reacutesultat opeacuterationnel

- lancer des investissements ayant une rentabiliteacute supeacuterieure agrave C

- eacuteliminer les activiteacutes ayant une rentabiliteacute infeacuterieure agrave C

Remarque lrsquoutilisation sans nuance de lrsquoEVA comme critegravere de management peut

poser problegraveme Le calcul de lrsquoEVA repose sur des ajustements comptables il est donc

sujet agrave manipulations (provisions capitalisation ou non de la RD hellip) Par ailleurs le

critegravere laquo EVA raquo pris isoleacutement peut conduire agrave chercher la rentabiliteacute agrave court terme agrave

reacuteduire les investissements prospectifs et donc nuire agrave terme au deacuteveloppement de

lrsquoentreprise

Initiation au management copy CRCF ndash J Sornet Page 38 48

Les fournisseurs reccediloivent le paiement de leurs factures plus ou moins rapidement (le deacutelai

de paiement repreacutesente une valeur consentie au fournisseur)

Lrsquoentreprise peut accroicirctre la valeur apporteacutee agrave ses fournisseurs par des actions cibleacutees

comme une contribution agrave la formation de leurs personnels certains transferts de

technologie ou de savoir faire agrave des sous-traitants une coopeacuteration suivie favorisant leur

deacuteveloppement lrsquointeacutegration agrave des campagnes de promotion

A noter que la valeur consentie aux fournisseurs peut avoir une influence sur la qualiteacute et les

deacutelais de livraison des produits

La collectiviteacute reccediloit des taxes et parfois des prestations en nature par deacutefaut ou explicites

(effort de preacuteservation de lrsquoenvironnement ameacutenagement du territoire par les implantations

aide mateacuterielle agrave des projets participation agrave la formation par exemple)

APPLICATIONS DT

DT1 Deacutefinir expliquer deacutereacuteglementation socieacutetal eacuteconomies drsquoeacutechelle coaching EDI

gouvernance

DT2 Deacuteterminer en quoi la deacutemarche TQM srsquoinscrit dans les deacutefis actuels du management

DT3 Apregraves avoir consulteacute les documents ci-dessous extraits du site drsquoAir France

(httpdeveloppement-

durableairfrancecomFRfrlocaldemarcheN4_positionnement_pphtm)

exposer les enjeux et les limites de la RSE et de la gestion des parties prenantes

Initiation au management copy CRCF ndash J Sornet Page 39 48

Dialogue avec les parties prenantes

Initiation au management copy CRCF ndash J Sornet Page 40 48

Attentes des parties prenantes

Initiation au management copy CRCF ndash J Sornet Page 41 48

Creacuteation de valeur pour les parties prenantes

La creacuteation de valeur pour les parties prenantes est au cœur de la strateacutegie du Groupe Le scheacutema de

distribution financiegravere ci-dessous donne un aperccedilu de la distribution des recettes du Groupe aux

diffeacuterentes parties prenantes actionnaires collaborateurs fournisseurs pouvoirs publics

collectiviteacutes locales etc

Initiation au management copy CRCF ndash J Sornet Page 42 48

Fiche DT1 ndash Extrait du sommaire de laquo Problegravemes eacuteconomiques raquo No 2894

La gestion des entreprises bouleverseacutee par les technologies de linternet

Reacutealiteacutes industrielles - Annales des Mines Jean-Michel Yolin

Avec lavegravenement de linternet les processus de conception de production et de vente sont

radicalement remis en cause Quel que soit le secteur dactiviteacute les technologies de linternet

permettent en effet de reacuteduire les deacutelais et de passer dun processus discontinu agrave un processus

continu Lorganisation des entreprises et leur mode de gestion en sont profondeacutement bouleverseacutes

tant au niveau individuel que collectif Linternet rend ainsi possible la reacutealisation dobjectifs que les

entreprises cherchaient agrave atteindre depuis longtemps sans y parvenir meilleure eacutecoute du client

travail sans stocks en flux tendu hieacuterarchies plates autorisant une grande reacuteactiviteacute flexibiliteacute dans

lorganisation et loutil de production acceacuteleacuteration du renouvellement des produits entreprises en

reacuteseau ougrave chacune se recentre sur son cœur de meacutetier etc

Le laquo knowledge management raquo ou comment geacuterer les connaissances

Document de travail du LAMSADE - Michel Grundstein

Peter Drucker lavait preacutedit le capital immateacuteriel eacutetait voueacute agrave devenir un facteur de compeacutetitiviteacute

pour lentreprise La libeacuteralisation des eacutechanges acceacutelegravere les processus de deacutecision de lentreprise

et implique que lassimilation des informations soit agrave la fois de meilleure qualiteacute et plus rapide Ainsi

la fonction qui consiste agrave manager les connaissances au sein de lentreprise savegravere primordiale

Bien que la prise de conscience de limportance du capital immateacuteriel ait eacuteteacute tardive - le concept

de knowledge management est apparu en France aux Etats-Unis et au Japon au milieu des

anneacutees 1990 - agrave lheure actuelle lorganisation de leacutechange dinformations et le partage des

connaissances sont devenus des facteurs cleacutes dune gestion performante de lentreprise Ils

doivent sinscrire dans un projet global destineacute agrave mettre en valeur les savoirs et les savoir-faire

individuels et collectifs

Les leccedilons du laquo coaching raquo pour le management de la qualiteacute

Humanisme et Entreprise - Martine Brasseur

Parmi les nouvelles formes de management en vogue dans les entreprises le coaching figure en

bonne place Appliqueacute au management de la qualiteacute il sagit dune pratique

daccompagnement destineacutee agrave initier et agrave faciliter le processus de deacuteveloppement dun individu

La deacutemarche consiste agrave affirmer que tout individu est en quecircte de qualiteacute agrave condition toutefois

de ne pas lui imposer des contraintes lempecircchant de progresser On considegravere notamment les

erreurs comme potentiellement feacutecondes En deacutefinitive le coach donne au coacheacute la permission

de reacuteussir en lui donnant aussi la permission deacutechouer

Initiation au management copy CRCF ndash J Sornet Page 43 48

Fiche DT2 ndash Management strateacutegique les sept deacutefis agrave relever dici agrave 2016

Extrait drsquoun article du site wwwlentreprisecom -Sabine Blanc - Mis en ligne le 20032007

(httpwwwlentreprisecom325article11977html)

Une eacutetude anglaise publieacutee par lopeacuterateur Orange Grande-Bretagne deacutecrypte la mutation

des formes de travail et les enjeux majeurs pour les entreprises de demain afin decirctre au top

de la compeacutetitiviteacute Voici les challenges-cleacutes pour les managers qui veulent rester dans la

course hellip

1 - Future organisation du travail les quatre laquo mondes raquo possibles

La reacutealiteacute sera probablement un meacutelange de ces quatre sceacutenarios souligne lrsquoeacutetude

Les mondes mutuels Tout se passe dans le cadre des communauteacutes locales vie priveacutee

comme professionnelle Le modegravele coopeacuteratif preacutevaut au lieu du laquo big business raquo Oublieacutes

aussi dans ce systegraveme les trajets pour aller au bureau les gens preacutefegravereront travailler dans de

petites entreprises locales souvent connecteacutees au reacuteseau drsquoautres structures similaires

Les laquo reacutepondants raquo (en anglais laquo replicants raquo) La figure du consultant freelance deviendra

dominante tandis que celle du salarieacute deacuteclinera Il ne sera pas rare de travailler pour plusieurs

entreprises On perdra en seacutecuriteacute de lrsquoemploi en visibiliteacute et en routine ce que lrsquoon gagnera

en liberteacute La majeure partie des tacircches srsquoeffectuera chez soi avec la possibiliteacute de srsquoinstaller

temporairement dans les bureaux de son client du moment Dans un contexte dincertitude

sur lrsquoavenir les travailleurs alterneront peacuteriodes drsquoactiviteacute intense et repos Ce sera agrave eux

drsquoaller vers les entreprises et non lrsquoinverse mecircme si celles-ci devront veiller agrave rester attractives

Les cottages eacutelectroniques Comme ce nom le suggegravere le teacuteleacutetravail deviendrait la norme

univers priveacute et professionnel se confondant Plus besoin de subir une heure de transport les

salarieacutes se logueront de chez eux sur le reacuteseau de lrsquoentreprise Les reacuteunions se tiendront dans

de petits bureaux centraux situeacutes agrave courte distance La flexibiliteacute du temps de travail srsquoimpose

Les salarieacutes disposeront de plus de marge de liberteacute dans leur activiteacute

Les disciples de la nueacutee Cette appellation poeacutetique cache simplement une extension de

lrsquoorganisation actuelle des grandes entreprises avec des salarieacutes se rendant sur un lieu de

travail centraliseacute Le rocircle croissant des technologies de lrsquoinformation multipliera les faccedilons de

collaborer et accroicirctra lrsquoefficaciteacute Le controcircle du travail sera omnipreacutesent La frontiegravere entre

travail et vie priveacutee restera marqueacutee

2 - Sept deacutefis pour les entreprises et leur managers

Quoi qursquoil advienne les entreprises et leurs dirigeants devront concentrer leurs efforts sur sept

points-cleacutes pour srsquoadapter Voici quelques exemples de probleacutematiques souleveacutees par le

rapport et des pistes de solution

Le leadership Les managers devront entre autres savoir persuader et influencer des

travailleurs beaucoup plus indeacutependants Ils auront aussi agrave repenser les niveaux auxquels

prendre les deacutecisions strateacutegiques en haut ou au contraire agrave des degreacutes moins eacuteleveacutes de la

pyramide hieacuterarchique

gt Faire du management une force facilitant les activiteacutes transversales plutocirct que la reacuteduire agrave

la seule fonction de deacutecision

La culture drsquoentreprise Davantage de salarieacutes capables de reacutefleacutechir seront neacutecessaires

tandis que les tacircches qui peuvent ecirctre automatiseacutees ou scripteacutees diminueront Un des

enjeux creacuteer une culture agrave mecircme drsquoattirer et drsquoencourager les personnes preacutesentant ces

qualiteacutes de reacuteflexion requises dans un contexte de compeacutetition accrue et de plus grande

indeacutependance des travailleurs

Initiation au management copy CRCF ndash J Sornet Page 44 48

gt Passer si neacutecessaire drsquoune culture drsquoentreprise forte agrave un mode drsquoengagement plus

consensuel moins rebutant

La marque Conseacutequence du recours croissant agrave lrsquo laquo outsourcing raquo lrsquoimage drsquoune marque

deacutependra plus drsquoagents exteacuterieurs qui ne fonctionnent pas forceacutement selon le mecircme mode

drsquoorganisation Comment garder le controcircle dessus

gt Choisir le mode qui corresponde le plus agrave vos valeurs et preacutevoir un programme de risk

management qui mette en eacutevidence ougrave les conflits sont susceptibles de jaillir

Lrsquoinnovation Plus que jamais il faudra faire face agrave une acceacuteleacuteration du rythme de

lrsquoinnovation en proposant constamment des solutions adapteacutees

gt Tisser des partenariats strateacutegiques avec drsquoautres entreprises pour partager les coucircts et les

fruits de lrsquoinnovation

Le deacutefi opeacuterationnel et technologique De quelle faccedilon controcircler lrsquoinformation crsquoest-agrave-dire

faire en sorte que les bonnes personnes accegravedent facilement agrave une information toujours en

phase tout en maintenant la seacutecuriteacute

gt Recourir agrave des laquo feuilles de route des futurs raquo syntheacutetisant en une page les indicateurs

sociaux et de consommation ainsi que les eacutevolutions technologiques et leacutegislatives qui

influent sur les changements et indiquant comment ils modifient vos marcheacutes vos clients et

votre organisation

La qualiteacute Si de nouveaux proceacutedeacutes ont pu deacutegrader la qualiteacute comme le recours agrave des

centres drsquoappel externaliseacutes drsquoautres ideacutees se sont reacuteveacuteleacutees plus prometteuses comme en

teacutemoigne le succegraves de certaines compagnies aeacuteriennes low cost Elles ont su conjuguer prix

serreacutes et services eacuteleveacutes ce qui devra devenir la norme estime lrsquoeacutetude

gt Continuer de rechercher la qualiteacute Elaborez aussi une bonne prestation service qui inclut

une livraison de qualiteacute voire creacuteez-la en partenariat avec les consommateurs

La leacutegislation La question de la proprieacuteteacute intellectuelle pourrait ecirctre probleacutematique Elle est

deacutejagrave source de conflits comme en teacutemoigne le procegraves pour violation de brevet intenteacute agrave RIM

le fabricant canadien du Blackberry par NTP Que pourra-t-on et que faudra-t-il proteacuteger par

un brevet Il sera eacutegalement neacutecessaire drsquoadapter la leacutegislation aux nouveaux modes

drsquoorganisation

gt Collaborer avec les acteurs du mecircme secteur et les leacutegislateurs pour deacutevelopper les

modegraveles des lieux de travail du futur et bacirctir le droit le plus adeacutequat

Orange a-t-il vu juste dans ses preacutevisions Rendez-vous dans neuf ans pour la reacuteponsehellip

Initiation au management copy CRCF ndash J Sornet Page 45 48

Fiche DT3 ndash Le management par la qualiteacute totale

Extrait drsquoune lettre drsquoinformation du cabinet Baud Accordance Consulting AD2 consultants ndash

2002

1 - Le TQM (Total Quality Management) offre pour lentreprise une vision de la qualiteacute plus

large et transversale

Son principe est simple La finaliteacute de lEntreprise est de deacutevelopper la satisfaction de ses

clients tout en eacutetant beacuteneacuteficiaire cest agrave dire pas agrave nimporte quel prix Elle doit ameacuteliorer sa

rentabiliteacute au travers de la deacutemarche qualiteacute La Qualiteacute Totale vise agrave fournir aux clients

externes et internes une reacuteponse adeacutequate agrave leurs attentes dans le meilleur rapport qualiteacute

prix la meilleure efficience

Elle considegravere pour cela lensemble des processus de lentreprise ayant une incidence sur la

qualiteacute et la satisfaction des clients

Le TQM fait ainsi une large place agrave

la deacutefinition et la planification de la strateacutegie geacuteneacuterale

la coheacuterence de la politique qualiteacute avec la strateacutegie

la deacutemultiplication de la politique qualiteacute dans toutes les directions de lentreprise

la relation client fournisseur interne

la prise en compte de lenvironnement concurrentiel

la consideacuteration de lensemble des risques potentiels financiers sociaux concurrentielshellip

limplication et la motivation du personnel

lanalyse des besoins des clients et le positionnement marketing

la maicirctrise des processus transverses internes

les reacutesultats sous tous ses aspects y compris financiers commerciaux image

De nombreux reacutefeacuterentiels sont relatifs agrave la Qualiteacute Totale hellip Tous ces reacutefeacuterentiels imposent un

questionnement plus profond et indiscret sur le mode de fonctionnement de lentreprise et

son management

helliphellip

2 - LISO 9001 2000 au travers du deacuteploiement des processus (management supports

reacutealisation et ameacutelioration continue) reacutepond quelque peu agrave la mecircme logique

LISO est une ouverture indeacuteniable vers la logique du TQM mais ne se reacutefegravere pas agrave la notion

defficience

Les dirigeants sont cependant sensibles agrave la neacutecessaire reacuteduction des coucircts de non-qualiteacute

et dobtention de la qualiteacute agrave la rentabiliteacute du systegraveme de management de la qualiteacute

mais ne perccediloivent pas toujours la qualiteacute comme une deacutemarche globale

Les deacutemarches qualiteacute commencent bien souvent par la remise en cause de lorganisation

leacutevaluation critique de son efficaciteacute lexamen des processus et la mise en eacutevidence des

lourdeurs administratives

La qualiteacute devient laffaire de tous hellip

Initiation au management copy CRCF ndash J Sornet Page 46 48

Fiche DT4 ndash Le deacuteveloppement durable et la RSE

Extrait du site wwwvigeocom

(httpwwwvigeocomcsr-rating-agencyfrmethodologiecriteres-de-recherche37-

criteres-d-analysehtml)

Deacuteveloppement durable laquo un deacuteveloppement qui reacutepond aux besoins du preacutesent sans compromettre

la capaciteacute des geacuteneacuterations futures de reacutepondre aux leurs raquo (Commission mondiale sur lrsquoenvironnement

et le deacuteveloppement ndash 1987)

Reacutefeacuterentiel drsquoeacutevaluation des entreprises par le groupe Vigeacuteo (le groupe mesure les performances et le

niveau de maicirctrise des risques de responsabiliteacute sociale des entreprises et des organisations - site

wwwvigeocom)

1 Ressources Humaines Ameacutelioration continue des relations professionnelles des relations drsquoemploi et des conditions de travail 2 Droits humains sur les lieux de travail Respect de la liberteacute syndicale et promotion de la neacutegociation collective non discrimination et promotion de lrsquoeacutegaliteacute eacutelimination des formes de travail proscrites (enfants travail forceacute) preacutevention des traitements inhumains ou deacutegradants de type harcegravelements sexuels protection de la vie priveacutee et des donneacutees personnelles 3 Environnement Protection sauvegarde preacutevention des atteintes agrave lenvironnement mise en place drsquoune strateacutegie manageacuteriale approprieacutee eacuteco conception protection de la biodiversiteacute et maicirctrise rationnelle des impacts environnementaux sur lrsquoensemble du cycle de vie des produits ou services

4 Comportements sur les marcheacutes Prise en compte des droits et inteacuterecircts des clients inteacutegration de standards sociaux et environnementaux dans la seacutelection des fournisseurs et sur lrsquoensemble de la chaicircne drsquoapprovisionnement preacutevention effective de la corruption respect des regravegles concurrentielles 5 Gouvernement drsquoentreprise Efficience et probiteacute assurance de lrsquoindeacutependance et de lrsquoefficaciteacute du Conseil drsquoadministration effectiviteacute et efficience des meacutecanismes drsquoaudit et de controcircle et notamment inclusion des risques de responsabiliteacute sociale respect des droits des actionnaires et notamment des minoritaires transparence et rationaliteacute de la reacutemuneacuteration des dirigeants 6 Engagement socieacutetal Effectiviteacute inteacutegration manageacuteriale de lrsquoengagement contribution au deacuteveloppement eacuteconomique et social des territoires drsquoimplantation et de leurs communauteacutes humaines engagements concrets en faveur de la maicirctrise des impacts socieacutetaux des produits et des services contribution transparente et participative agrave des causes drsquointeacuterecirct geacuteneacuteral

Initiation au management copy CRCF ndash J Sornet Page 47 48

ELEMENTS DE CORRIGE DT DT1 Deacutefinir expliquer

Deacutereacuteglementation = suppression des contraintes eacuteconomiques (libre eacutechange des biens et

capitaux)

Socieacutetal = qui se rapporte agrave la structure agrave lrsquoorganisation ou au fonctionnement de la socieacuteteacute

Economies drsquoeacutechelle = reacuteduction des coucircts lieacutee au niveau drsquoactiviteacute (amortissement des

charges fixes)

Coaching = accompagnement de personnes ou deacutequipes pour le deacuteveloppement de leurs

potentiels

EDI = eacutechange de donneacutees informatiseacutees ET standardiseacutees (ex SWIFT bancaire edifact

documents deacuteclaratifs)

Gouvernance = exercice du pouvoir la bonne gouvernance est participative et eacutequitable

conforme agrave lrsquointeacuterecirct commun

DT2 Deacuteterminer en quoi la deacutemarche TQM srsquoinscrit dans les deacutefis actuels du management

Voir notamment fiche 43

Maicirctrise des processus reacuteduction des coucircts reacuteactiviteacute et satisfaction de la clientegravele = faire

face agrave la concurrence

Ameacutelioration de lrsquoimage motivation du personnel

DT3 Apregraves avoir consulteacute les documents ci-dessous extraits du site drsquoAir France

(httpdeveloppement-

durableairfrancecomFRfrlocaldemarcheN4_positionnement_pphtm)

exposer les enjeux et les limites de la RSE et de la gestion des parties prenantes

Trame geacuteneacuterale possible

Introduction

Les deacutefis contemporains (accroissement de la concurrence devenue mondiale recherche

de nouveaux avantages concurrentiels pression de la socieacuteteacute besoin drsquoimage et de projet

lisible pour mener lrsquoentreprise crise et scandales du libeacuteralisme hellip) RSE et PP

Deacuteveloppement (voir cours)

1 ndash Parties prenantes et management par la valeur

PP deacutefinir citer reacutesumer lrsquoavantage rechercheacute (fideacuteliser motiver recherche drsquoalliances

implicites)

PP moyens (dont exemples AF) et meacutethode de management par la valeur (reacutepartie)

2 ndash La responsabiliteacute socieacutetale de lrsquoentreprise

RSE 3 axes

- eacuteconomique (favoriser le deacuteveloppement les eacutechanges internationaux)

- social (accegraves aux soins eacuteducation conditions de travail hellip)

- environnemental (pollution preacuteservation des ressources hellip)

RSE gouvernance drsquoentreprise facteur drsquoimage inteacutegrable dans la deacutemarche PP

Article 116 de la loi Le rapport viseacute agrave larticle L 225-102 rend compte hellip laquo Il comprend

eacutegalement des informations dont la liste est fixeacutee par deacutecret en Conseil dEtat sur la maniegravere

dont la socieacuteteacute prend en compte les conseacutequences sociales et environnementales de son

activiteacute Le preacutesent alineacutea ne sapplique pas aux socieacuteteacutes dont les titres ne sont pas admis aux

neacutegociations sur un marcheacute reacuteglementeacute raquo

Initiation au management copy CRCF ndash J Sornet Page 48 48

RSE exemple AF (ONG fournisseurs)

3 ndash Liens entre PP et RSE

- la RSE introduit de nouvelles PP

- la RSE suppose le respect des PP usuelles (employeacutes clients notamment)

4 - Probleacutematique

- deacutefinir la valeur reacuteellement apporteacutee par une gestion des PP (confusion salaire ndash valeur

idem impocircts hellip ex laquo valeur ajouteacutee raquo)

- communication (neacutecessaire mais aller au-delagrave)

- marginaliteacute des deacutepenses RSE (efficaciteacute sinceacuteriteacute de lrsquoengagement marge de manœuvre)

- charge RSE reporteacutee sur des tiers (ex fournisseurs AF)

- inteacutegration de facteurs non visibles en comptabiliteacute (pertes drsquoemploi nuisances hellip)

Conclusion

Voies incontournables mais pouvant nrsquoavoir qursquoun effet superficiel et temporaire Voir utiliteacute

drsquoaccompagnement leacutegislatif de regravegles de gouvernance

Initiation au management copy CRCF ndash J Sornet Page 17 48

2 ndash Les contextes de management

Le management est influenceacute par son contexte qui justifie des objectifs une organisation

des meacutethodes

Par exemple lrsquoentreprise admet de nombreuses variantes selon sa taille sa forme juridique

son controcircle par lrsquoeacutetat (entreprises publiques) ou par des inteacuterecircts priveacutes Il en va de mecircme des

organismes administratifs qui peuvent deacutependre de directives nationales ou reacutegionales des

associations qui ont des activiteacutes drsquoampleur tregraves variable

21 ndash La dimension de lrsquoentreprise

La dimension drsquoune entreprise se mesure principalement en fonction de son effectif ou de

son chiffre drsquoaffaires Des seuils sont deacutefinis par divers organismes et exploiteacutes agrave des fins

statistiques ou pour la deacutetermination de certaines obligations sociales ou fiscales

(repreacutesentation du personnel cotisations hellip) Il nrsquoy a bien entendu pas de laquo barriegravere de

tailleraquo absolue conditionnant le management drsquoune entreprise

LrsquoUE preacuteconise de distinguer les micro ndash entreprises (jusqursquoagrave 9 salarieacutes) les TPE ndash tregraves petites

entreprises (moins de 20 salarieacutes) les petites entreprises (moins de 50) et les moyennes

entreprises (de 50 agrave 250) Cependant les PME sont parfois situeacutees entre 10 et 500 salarieacutes

Remarques

- en France environ 40 des entreprises emploient de 1 agrave 50 salarieacutes (ce qui repreacutesente

plus de 50 des emplois) et 59 nrsquoen ont aucun

le pays compte environ 2 600 000 entreprises dont moins de 1 ont 250 employeacutes et

plus

- ancienneteacute et taille de lrsquoentreprise sont lieacutees si lrsquoon eacutecarte les restructurations et autres

eacutevolutions drsquoentreprises existantes

La dimension de lrsquoentreprise a une influence sur lrsquoorganisation et le laquo style raquo de son

management

- les PME sont souvent entrepreneuriales (les dirigeants eacutegalement apporteurs de capitaux

sont totalement engageacutes dans la marche de lrsquoentreprise) Elles ont une gestion flexible peu

formaliseacutee plus qualitative que quantitative Les PME sont freacutequemment focaliseacutees sur un seul

type drsquoactiviteacute Pour ne pas alourdir leur structure elles ont tendance agrave sous-traiter les

activiteacutes speacutecialiseacutees ne correspondant pas agrave leur meacutetier de base

- les grandes entreprises sont manageacuteriales (les dirigeants sont nommeacutes par les actionnaires

en raison de leurs compeacutetences) et moins reacuteactives

22 ndash Le type de production

On distingue industrie (production de biens mateacuteriels ou pour le moins de produits visibles ndash

comme un seacutejour touristique ou un film) et services (fourniture drsquoune prestation immateacuterielle)

Le type de production influence en principe le management de lrsquoentreprise

- lrsquoindustrie neacutecessite (si lrsquoon excepte lrsquoartisanat) un investissement relativement important

une organisation productive stable capable de reacutealiser plusieurs fois des produits identiques

(exemple un modegravele de reacutefrigeacuterateur) ou du moins similaires (exemple un bacirctiment) Le

produit de lrsquoindustrie consomme des matiegraveres et il doit geacuteneacuteralement ecirctre distribueacute jusqursquoau

client

- la production de services peut se satisfaire drsquoun investissement tregraves reacuteduit et neacutecessite un

contact permanent avec le client

Toutefois la standardisation des services et le deacuteveloppement des reacuteseaux informatiques

rapprochent la production de services de celle des biens industriels

- la production drsquoun service reacutepeacutetitif et technique peut imposer une structure lourde et une

organisation tregraves formaliseacutee (voir les grandes socieacuteteacutes drsquoaudit ou de conseil informatique)

Initiation au management copy CRCF ndash J Sornet Page 18 48

- certains services peuvent ecirctre fournis agrave distance sans contact direct avec le client et

distribueacutes par reacuteseau (tenue de comptabiliteacute affacturage gestion clientegravele centre drsquoappel

hellip)

Remarque les services repreacutesentent 75 de lrsquoactiviteacute eacuteconomique franccedilaise

23 ndash La nature de lrsquoorganisation

Les organisations publiques franccedilaises (administrations centrales collectiviteacutes territoriales

hocircpitaux hellip) repreacutesentent une part importante de lrsquoactiviteacute (environ 30 des emplois) La

fonction publique regroupe des organisations aux finaliteacutes diverses et qui ont des problegravemes

de gestion similaires agrave ceux des entreprises auxquelles elles peuvent emprunter des principes

de management Notamment

- pour controcircler les coucircts et assurer la qualiteacute des services

- pour communiquer avec les administreacutes ou les usagers

- pour motiver les personnels et geacuterer les ressources humaines

La transposition directe des techniques de gestion et de management nrsquoest cependant pas

toujours possible car

- la comptabiliteacute publique obeacuteit agrave des regravegles speacutecifiques (proceacutedure budgeacutetaire

notamment)

- le laquo client raquo ne paye pas toujours la prestation du moins directement

- la concurrence est parfois inexistante

- les grandes administrations centraliseacutees sont soumises agrave des choix politiques geacuteneacuteraux

parfois sans connexion eacutevidente avec les besoins opeacuterationnels

- le statut des personnels et les grilles de salaires limitent les possibiliteacutes de gestion des

ressources humaines

Remarque la LOLF (loi organique relative aux lois de finances) est entreacutee en vigueur en

2006 Elle alloue des moyens budgeacutetaires en fonction de programmes et remplace la

reconduction automatique de 90 des budgets Cette reacuteforme se heurte toutefois agrave la

lourdeur des grands ministegraveres ougrave la complexiteacute des activiteacutes est difficile agrave

appreacutehender et ougrave des inerties culturelles peuvent exister agrave tout niveau

Les associations loi de 1901 peuvent avoir une activiteacute comparable agrave celle de grandes

entreprises (voir par exemple les associations de santeacute ou professionnelles) et leur

management est alors similaire malgreacute lrsquoabsence de but lucratif (les beacuteneacutefices ne sont pas

distribuables) Elles ont drsquoailleurs en France un poids eacuteconomique important (elles emploient

environ 1 600 000 salarieacutes)

Cependant lrsquoadheacutesion agrave un systegraveme de valeurs fondateur de lrsquoassociation ou la limite de

lrsquoautoriteacute (quand un volant de beacuteneacutevoles important participe agrave lrsquoactiviteacute) peut introduire des

nuances

- le renforcement des objectifs socieacutetaux

- la faiblesse des relations hieacuterarchiques

- des contraintes de gestion du temps des beacuteneacutevoles

- des modaliteacutes particuliegraveres de recrutement et de motivation des dirigeants

24 ndash Les facteurs contingents

La theacuteorie de la contingence montre qursquoune structure drsquoentreprise nrsquoest efficace que dans

une situation deacutetermineacutee et qursquoil nrsquoexiste que des solutions de management construites dans

un contexte preacutecis

Le management doit ainsi srsquoadapter agrave des facteurs contingents qui ne peuvent ecirctre

controcircleacutes du moins agrave bregraveve eacutecheacuteance Ces facteurs sont par exemple

- lrsquoancienneteacute de lrsquoentreprise (plus elle est ancienne plus lrsquoentreprise a tendance agrave reacutepeacuteter

des comportements eacuteprouveacutes)

Initiation au management copy CRCF ndash J Sornet Page 19 48

- la taille de lrsquoentreprise (la grande entreprise a une composante administrative plus

deacuteveloppeacutee)

- le systegraveme de production (tregraves standardiseacute complexe automatiseacute hellip)

- lrsquoenvironnement

3 ndash Le management et les parties prenantes

Lrsquoentreprise a pour vocation premiegravere de mettre des produits agrave disposition de ses clients en

reacutealisant un profit Pour y arriver elle doit aussi satisfaire ses parties prenantes salarieacutes

actionnaires fournisseurs hellip

Est partie prenante agrave lrsquoentreprise laquo tout groupe ou individu qui peut ecirctre affecteacute ou est

affecteacute par les buts de lrsquoorganisation hellip raquo (Freeman ndash 1984)

Les parties prenantes attendent agrave des degreacutes divers de profiter drsquoune creacuteation de valeur en

provenance de lrsquoentreprise qui doit reacutepondre agrave ces attentes pour assurer sa peacuterenniteacute ou

favoriser son deacuteveloppement

On distingue les parties prenantes primaires ou principales qui sont essentielles agrave lrsquoentreprise

et qui ont geacuteneacuteralement une relation formelle avec elle (clients associeacutes et actionnaires

precircteurs salarieacutes fournisseurs collectiviteacutes) et les parties prenantes secondaires dont

lrsquoinfluence est diffuse (groupes de pression associations meacutedias instances europeacuteennes

agences de notation hellip)

Remarque la consideacuteration de lrsquoensemble des parties prenantes (laquo stakeholders raquo - les

deacutepositaires) fait contrepoids agrave lrsquoimportance accordeacutee aux seuls actionnaires

(laquo shareholders raquo)

Les organisations nrsquoayant pas drsquoobjectif de profit doivent aussi satisfaire leurs parties

prenantes apporter un service aux usagers dans les meilleures conditions eacuteconomiques

limiter un budget assurer la qualiteacute des relations avec les fournisseurs hellip

Dans cette optique le management doit organiser lrsquoaction de faccedilon agrave eacutequilibrer des forces

parfois divergentes

- le contexte fait pression sur lrsquoorganisation contrainte agrave optimiser ses reacutesultats

- lrsquoorganisation cherche par son action agrave assurer sa peacuterenniteacute son deacuteveloppement (en

reacutealisant des profits dans le cas de lrsquoentreprise) et agrave satisfaire ses parties prenantes

- le management agit en pilotant les actions pour contrebalancer la pression du contexte

Actions de

lrsquoorganisation

Management Contexte

Parties

prenantes

Initiation au management copy CRCF ndash J Sornet Page 20 48

APPLICATIONS MP

MP1 Deacutefinir contingent gestion budgeacutetaire

MP2 Deacuteterminer les parties prenantes drsquoun hocircpital public et leurs principales attentes

Mecircme question pour les organisations suivantes

- SNCF (entreprise publique)

- Peugeot

- MAIF (mutuelle drsquoassurance)

MP3 En les situant dans le cycle des activiteacutes du management trouver les actions agrave mener

dans les situations suivantes

- baisse de 10 des ventes dans une entreprise industrielle (produits meacutenagers le reacuteseau de

distribution vient drsquoecirctre reacuteorganiseacute)

- idem dans une entreprise de vente par correspondance soumise agrave la concurrence internet

(les ventes stagnaient depuis six mois malgreacute les efforts promotionnels)

- augmentation des deacutelais drsquoattente des consultations dans une clinique (lrsquohocircpital voisin a

fermeacute son service drsquourgences)

Initiation au management copy CRCF ndash J Sornet Page 21 48

ELEMENTS DE CORRIGE MP

MP1 Deacutefinir (dans le contexte drsquoune entreprise) contingent gestion budgeacutetaire

Contingent = imposeacute par lrsquoexteacuterieur Contingence = effet du hasard de la rencontre de

plusieurs eacuteveacutenements indeacutependants (variables explicatives que lrsquoon ne peut influencer)

Gestion budgeacutetaire = technique drsquoadministration des entreprises srsquoappuyant sur des

preacutevisions dont on deacuteduit apregraves accord des responsables des attributions de moyens sur une

dureacutee limiteacutee Une analyse reacuteguliegravere des eacutecarts entre preacutevisions et reacutealisations permet ensuite

le pilotage des activiteacutes Le budget est un cadre incitatif

La laquo planification budgeacutetaire raquo consiste agrave traduire en budgets une planification strateacutegique

avec systegraveme de reporting

MP2 Deacuteterminer les parties prenantes drsquoun hocircpital public et leurs principales attentes

Mecircme question pour les organisations suivantes

- SNCF (entreprise publique)

- Peugeot

- MAIF (mutuelle drsquoassurance)

Hocircpital

- patients (qualiteacute des soins)

- CNAM (baisse des coucircts)

- collectiviteacute locale (service aux administreacutes)

- eacutetat (ameacutenagement du territoire maicirctrise des budgets optimisation)

- employeacutes (salaire conditions de travail et satisfaction)

- fournisseurs ndash pharmacie autres (CA paiement reacutegulier)

- associations de patients (qualiteacute proximiteacute des soins)

SNCF

- usagers et associations drsquousagers (proximiteacute reacutegulariteacute prix du service)

- reacuteseau ferreacute de France (optimisation des lignes paiement adapteacute)

- fournisseurs (CA paiement reacutegulier)

- employeacutes (salaire conditions de travail seacutecuriteacute de lrsquoemploi)

- eacutetat (ameacutenagement du territoire)

- collectiviteacutes locales (service)

Peugeot

- clients (qualiteacute prix SAV relation commerciale)

- fournisseurs (CA reacutegulariteacute de lrsquoactiviteacute)

- employeacutes (salaire conditions de travail seacutecuriteacute de lrsquoemploi)

- eacutetat (taxes)

- collectiviteacute locale (emploi dynamisation eacuteconomique preacuteservation de lrsquoenvironnement)

- associations de protection de lrsquoenvironnement (activiteacute propre baisse des eacutemissions

nouvelles eacutenergies)

MAIF

- socieacutetaires (protection relation assureur tarif mesureacute)

- professionnels de lrsquoautomobile et autres (agreacutement marge de manœuvre reacuteparations tarifs

eacuteleveacutes)

- fournisseurs (CA paiement reacutegulier)

- eacutetat (taxes engagement pour la seacutecuriteacute)

- employeacutes (salaire conditions de travail seacutecuriteacute de lrsquoemploi)

Initiation au management copy CRCF ndash J Sornet Page 22 48

MP3 En les situant dans le cycle des activiteacutes du management trouver les actions agrave mener

dans les situations suivantes

- baisse de 10 des ventes dans une entreprise industrielle (produits meacutenagers le reacuteseau de

distribution vient drsquoecirctre reacuteorganiseacute)

Adapter le pilotage motiver cadrer si insuffisant retoucher une organisation deacutefectueuse

- idem dans une entreprise de vente par correspondance soumise agrave la concurrence internet

(les ventes stagnaient depuis six mois malgreacute les efforts promotionnels)

Voir pilotage et organisation si une eacutevolution du meacutetier a deacutejagrave eacuteteacute initialiseacutee Sinon re-

conception (adaptation au nouveau contexte) puis planification et reacuteorganisation

- augmentation des deacutelais drsquoattente des consultations dans une clinique (lrsquohocircpital voisin a

fermeacute son service drsquourgences)

Organisation Si insuffisant planification (nouveaux objectifs)

Initiation au management copy CRCF ndash J Sornet Page 23 48

ORGANISATION ET PROCESSUS

La performance de lrsquoentreprise deacutepend de son organisation et de son aptitude agrave produire

aux meilleures conditions Nous allons montrer comment organisation formelle et processus

de production peuvent contribuer agrave cette performance

1 ndash Vers lrsquooptimum

11 ndash Les eacuteconomies occidentales jusqursquoaux anneacutees 70

Jusqursquoen 1945 le principal problegraveme des entreprises eacutetait de produire des biens en quantiteacute

suffisante agrave un prix compatible avec le marcheacute Les grandes entreprises se sont multiplieacutees et

la standardisation a permis de reacuteduire les coucircts (exemple deacuteveloppement de Ford et de la

production agrave la chaicircne de 1908 agrave 1920 qui a permis une baisse du prix des voitures des 23)

On parle de laquo production pousseacutee vers le marcheacute raquo

Cette croissance de la production peu reacuteguleacutee a eacuteteacute marqueacutee par des surproductions en

1910 et 1920 puis par la crise de 1929 qui a prolongeacute ses effets jusqursquoagrave la guerre

De 1945 agrave 1975 environ (les laquo trente glorieuses raquo) la reconstruction la croissance de la

consommation de masse de nouvelles technologies et les eacutechanges internationaux

alimentent lrsquoeacuteconomie La standardisation srsquoeacutetend aux biens de consommation dont les

coucircts baissent fortement et de nouvelles reacutegulations sociales permettent une eacutevolution sans

heurt des revenus La saturation de certains marcheacutes conduit dans les anneacutees 60 agrave la

deacutemarche laquo marketing raquo et agrave la diffeacuterenciation des produits Le produit est laquo dirigeacute par le

marcheacute raquo mais les entreprises conservent une organisation assez classique et les plus grosses

srsquointernationalisent

12 ndash Lrsquoexpeacuterience japonaise et ses prolongements

Tregraves tocirct apregraves la guerre dans un Japon appauvri le constructeur automobile Toyota a ducirc

faire face agrave une restriction du marcheacute des moyens financiers et productifs et des

approvisionnements La firme a donc innoveacute dans un nouveau systegraveme de production

chassant les laquo gaspillages raquo (temps drsquoattente transports stocks deacutefauts hellip) consideacuterant que

seule la fabrication vendable creacutee de la valeur

Toyota srsquoorganise pour fabriquer la quantiteacute et la qualiteacute de produits juste neacutecessaires agrave la

satisfaction des clients la production est laquo tireacutee par le marcheacute raquo La mise en place de ce

systegraveme qui integravegre les fournisseurs ne sera acheveacutee que dans le milieu des anneacutees 70

En 1973 la hausse du peacutetrole inaugure un ralentissement de la croissance des eacuteconomies

occidentales La concurrence accrue provoque alors un inteacuterecirct pour le systegraveme deacuteveloppeacute

au Japon La production au plus juste se deacuteveloppe ainsi dans lrsquoindustrie automobile agrave partir

des anneacutees 80 et elle se reacutepand encore maintenant dans drsquoautres secteurs

Cette approche qui vise un objectif de zeacutero stock et zeacutero deacutefaut impose la maicirctrise de laquo bout

en bout raquo des processus de production et leur ameacutelioration

Initiation au management copy CRCF ndash J Sornet Page 24 48

2 ndash Organiser lrsquoentreprise

21 ndash Direction et organisation

Diriger une entreprise neacutecessite de lrsquoorganiser (de reacutepartir les tacircches) pour qursquoelle puisse

atteindre ses objectifs Lrsquoorganisation permet de satisfaire un marcheacute en tirant parti des

capaciteacutes actuelles de lrsquoentreprise tout en preacuteparant lrsquoavenir

Lrsquoorganisation reacutesulte freacutequemment drsquoun compromis entre des objectifs situeacutes agrave des niveaux

et des eacutecheacuteances diffeacuterents

Exemples

- le leader des chaises roulantes peut tirer profit de sa structure productive et de son

savoir faire pour entrer sur le marcheacute de la bicyclette eacutelectrique

- ecirctre parfaitement structureacute pour alimenter 90 du marcheacute des disquettes ne preacutepare

pas lrsquoavenir

- srsquoorganiser pour conqueacuterir le marcheacute des tire-bouchons eacutelectriques dans les deux ans

perd de son sens si cela altegravere les moyens neacutecessaires agrave la production drsquoappareils

manuels ancienne mais vitale dont la diminution agrave court terme risque de nuire agrave la

solvabiliteacute de lrsquoentreprise et de la conduire agrave la cessation de paiement

22 ndash Lrsquoorganisation fonctionnelle

La majoriteacute des entreprises adopte une laquo organisation fonctionnelle raquo (celle qui est visible

dans les organigrammes) ougrave des regroupements de personnels et drsquoeacutequipements se font

selon un modegravele hieacuterarchique (laquo line raquo) dans des uniteacutes des services ou des deacutepartements

speacutecialiseacutes Cette organisation peut se deacutecliner agrave lrsquointeacuterieur des divisions des grandes

entreprises quand elles scindent leur activiteacute par zone geacuteographique type drsquoactiviteacute

cateacutegorie de clients hellip

Remarque le terme laquo fonction raquo deacutesigne un rocircle particulier dans le fonctionnement de

lrsquoentreprise

Lrsquoorganisation fonctionnelle diffeacuterencie les activiteacutes de lrsquoentreprise en les regroupant par

meacutetier pour utiliser au mieux les compeacutetences et les moyens (meilleur rendement par la

speacutecialisation lrsquoeacutechange de compeacutetences dans une mecircme uniteacute ou gracircce agrave des eacuteconomies

drsquoeacutechelle)

23 ndash La notion de processus de production

Un processus de production se deacutefinit par la succession drsquoactiviteacutes permettant de satisfaire

un client en transformant des ressources (mateacuterielles financiegraveres humaines) en un produit

bien ou service Le processus doit creacuteer une valeur reconnue par le client

Un processus peut servir un client interne agrave lrsquoentreprise (par exemple en produisant un

composant intervenant dans plusieurs produits ou par la maintenance des machines) aussi

bien qursquoun client final On distingue usuellement

- les processus opeacuterationnels (ou maicirctres) aussi appeleacutes processus meacutetier (business process)

qui satisfont directement les clients finaux (conception et fabrication de produits vente hellip)

- les processus de support et de management (geacuterer les ressources humaines geacuterer

lrsquoinformation geacuterer les ressources financiegraveres hellip) qui ont les processus opeacuterationnels comme

clients

Toutes les actions internes agrave une organisation peuvent srsquointeacutegrer dans des processus qui

conditionnent directement ou indirectement la capaciteacute de lrsquoorganisation agrave satisfaire le

client final ou lrsquousager

Initiation au management copy CRCF ndash J Sornet Page 25 48

Aborder le fonctionnement de lrsquoentreprise par ses processus (approche processus) permet

de mettre en eacutevidence les chaicircnes drsquoactiviteacutes qui conduisent aux produits leurs

dysfonctionnements leurs coucircts la formation des deacutelais et la souplesse (la flexibiliteacute)

disponible pour satisfaire la clientegravele finale Lrsquoameacutelioration des processus a un impact visible

et direct sur chaque produit proposeacute aux clients

Lrsquoapproche processus provoque une eacutevolution de la faccedilon de travailler

- en faisant peacuteneacutetrer la laquo voix du client raquo au plus profond de lrsquoentreprise (et plus seulement

dans les services commerciaux et marketing)

- en mettant en eacutevidence des possibiliteacutes de rationalisation (par regroupement ou impartition

de certaines activiteacutes)

Remarque lrsquoapproche par les activiteacutes et les processus est agrave lrsquoorigine de la meacutethode

de deacutetermination des coucircts laquo ABC raquo - activity based costing

24 ndash Processus et fonctions

Le processus est transversal Il enchaicircne des activiteacutes qui traversent lrsquoentreprise en particulier

les services ou les deacutepartements drsquoune organisation fonctionnelle

Exemple

La division du travail par fonctions induit une charge de coordination pour assurer le

deacuteroulement du processus Elle peut geacuteneacuterer des attentes des erreurs ou des conflits drsquointeacuterecirct

(lrsquoobservation montre que des dysfonctionnements sont tregraves souvent constateacutes lors du

passage drsquoun service agrave un autre)

Organisation fonctionnelle et approche processus visent toutes deux un optimum

eacuteconomique mais leurs logiques sont diffeacuterentes

- le processus vise la satisfaction des clients (prix qualiteacute deacutelais service)

- le deacutecoupage fonctionnel cherche agrave optimiser les moyens (maximiser lrsquoeffet drsquoexpeacuterience

partager des infrastructures profiter de pocircles de compeacutetences hellip) Il apporte une ossature

hieacuterarchique stable souvent indispensable

Organisation fonctionnelle et approche processus sont donc compleacutementaires dans la

majoriteacute des cas et doivent ecirctre combineacutees judicieusement

APPLICATIONS OP

OP1 Deacutefinir flexibiliteacute systegraveme impartition

OP2 Citer huit exemples drsquoinformations essentielles pour optimiser un processus de

fabrication

Direction

Deacutepartement

commercial

(C)

Deacutepartement

administratif et

financier (AF)

Deacutepartement

Etudes (E)

Deacutepartement

Production (P)

Activiteacute

C-x Activiteacute

AF-x Activiteacute

E-x

Activiteacute

P-x

Processus x

Clie

nt

Initiation au management copy CRCF ndash J Sornet Page 26 48

OP3 Deacutegager les principes du toyotisme preacutesenteacute ci-dessous En quoi ce systegraveme est-il

initiateur de lrsquoapproche processus

Taiichi Ohno et le Toyotisme

1 - Extrait drsquoun article de Jacques BARRAUX - 1993 - LExpansion

Taiichi Ohno (1912 ndash 1990) hellip ne se prenait pas pour un visionnaire mais en imposant une

nouvelle faccedilon de produire il a reacuteinventeacute le management hellip tout le monde a entendu parler

des mots qui ont populariseacute le toyotisme dont il est le pegravere le juste-agrave-temps hellip Autant

doutils conccedilus pour lrsquoautomobile et qui ont aujourdhui une application universelle

hellip Taiichi Ohno jeune ingeacutenieur entre chez Toyota alors simple constructeur de machines

textiles Degraves 1926 apparaicirct la notion de jidoka hellip cest lart de transfeacuterer de lintelligence aux

machines pour mieux libeacuterer lintelligence des hommes Tout le contraire du taylorisme qui

juge la machine moins impreacutevisible que lhomme En 1933 Toyota se lance dans lautomobile

en sinspirant des meacutethodes ameacutericaines Mais en 1935 agrave loccasion dun voyage aux Etats-

Unis leacutetat-major de lentreprise revient fascineacute de sa visite dans un supermarcheacute La notion

de juste-agrave-temps va naicirctre de lobservation dune grande surface un lieu ougrave les clients ne

prennent que ce dont ils ont besoin et ougrave les rayons sont reacuteapprovisionneacutes pour compenser

les quantiteacutes preacuteleveacutees Ainsi le systegraveme Toyota est-il deacutejagrave dans la tecircte de ses dirigeants avant

mecircme la Seconde Guerre mondiale un demi-siegravecle avant la reacutevolution informatique et la

segmentation intensive des marcheacutes

hellip des esprits curieux comme Franccedilois Dalle en France tombent alors sous le charme des

formules et des paraboles de Taiichi Ohno En voici deux eacutechantillons

Penser agrave lenvers Cela signifie combattre les ideacutees reccedilues En lespegravece il sagit du fordisme et

du taylorisme Ohno ne croit pas agrave la planification aux effets deacutechelle et dexpeacuterience Il

propose un systegraveme industriel agrave lenvers qui permette de diversifier les produits et de les

fabriquer en petites quantiteacutes Nous ne devons plus ecirctre des paysans qui accumulent des

stocks mais des chasseurs On nimpose pas loffre On traque la demande et on la gegravere en

continu

Que les valleacutees soient hautes et les montagnes peu eacuteleveacutees Plutocirct que de concentrer tous

les efforts sur une production agrave un moment donneacute mieux vaut se doter de structures flexibles

permettant de passer agrave tout instant dune seacuterie agrave une autre Il faut eacuteviter les ruptures et les

secousses aplanir les cycles entretenir des flux reacuteguliers dactiviteacutes diversifieacutees Ce qui

implique de ne pas enfermer les hommes et les eacutequipements dans des speacutecialisations trop

eacutetroites

La flexibiliteacute le travail en groupe le refus de la dictature des machines la polyvalence et

surtout lattention constante aux signaux eacutemis par le marcheacute nappartiennent plus au

toyotisme Ces notions sont les fondements du nouvel art dorganiser de vendre et de

produire dans lindustrie comme dans les services hellip

2 - Quelques notions cleacutes

Taiichi Ohno a imagineacute la meacutethode des laquo cinq pourquoi raquo qui consiste agrave se poser cinq fois de

suite la question laquo pourquoi raquo sur le mecircme sujet de faccedilon agrave deacutecouvrir la veacuteritable cause

drsquoun problegraveme Cette meacutethode peut ecirctre appliqueacutee agrave tous les niveaux et permettre

notamment aux agents de fabrication de proposer de veacuteritables ameacuteliorations de la

production

La recherche de la qualiteacute totale (pas de deacutefaut des produits pas de rebuts pas de deacutefaut

des processus) accompagne la deacutemarche de Toyota La qualiteacute a un coucirct compenseacute par

des ventes accrues par lrsquoeacuteconomie des mesures palliatives aux deacutefauts

Initiation au management copy CRCF ndash J Sornet Page 27 48

Fiche OP1 ndash Benchmarking et processus

Le laquo benchmarking raquo consiste agrave comparer le fonctionnement de plusieurs systegravemes pour en

faire notamment ressortir les meilleures pratiques (laquo best practices raquo) Cette technique est

utiliseacutee depuis les anneacutees 80 pour ameacuteliorer la performance des entreprises Elle impose agrave

lrsquoentreprise drsquoeacutevaluer et de remettre en question ses propres modes de fonctionnement afin

de les faire eacutevoluer agrave la lueur de ce qui se fait ailleurs

Le benchmarking permet drsquoameacuteliorer les processus agrave moindre risque en fixant des objectifs

baseacutes sur des faits et donc plus facilement accepteacutes

Une classification des processus en tant que base de reacuteflexion a eacuteteacute eacutetablie aux USA par

lrsquolaquo International Benchmarking Clearinghouse raquo de lrsquoAPQC (american productivity and

quality center) en collaboration avec plusieurs dizaines drsquoentreprises

Elle se reacutesume ainsi

Le terme laquo reengineering raquo (la re-conception ou laquo reacuteingeacutenieacuterie raquo) des processus deacutesigne un

projet drsquoameacutelioration radicale des performances (de 20 agrave 50 ou plus) Il neacutecessite une

parfaite adheacutesion de la direction la constitution drsquoune petite eacutequipe de projet brillante

connaissant parfaitement les activiteacutes de lrsquoentreprise et il peut inclure un benchmarking

Le reengineering provoque geacuteneacuteralement la reacuteduction du nombre de niveaux hieacuterarchiques

(laquo delayering raquo) et lrsquoaccroissement du pouvoir de deacutecision des employeacutes (laquo empowerment raquo

ou laquo empouvoirement raquo) Bien qursquoy conduisant parfois il ne doit pas ecirctre confondu avec la

reacuteduction des activiteacutes (laquo downsizing raquo ou restructuration) et lrsquoexternalisation (laquo outsourcing raquo)

Pro

ce

ssu

s

op

eacutera

tio

nn

els

Pro

ce

ssu

s d

e m

an

ag

em

en

t e

t d

e

sup

po

rt

1 ndash

Comprendre

le marcheacute et

les clients (besoins

satisfaction)

2 ndash

Deacutevelopper

vision et

strateacutegie (contexte

concurrence)

3 ndash

Creacuteer

produits

services

processus

(concevoir

ameacuteliorer)

4 ndash

Marketing et

vente

5 ndash

Produire et

livrer (industrie

dont

ameacutelioration

processus)

6 ndash

Produire et

livrer (services)

7 ndash

Facturer et

servir les

clients (apregraves-

vente

reacuteclamations)

8 ndash Deacutevelopper et geacuterer les ressources humaines

9 ndash Geacuterer les systegravemes drsquoinformation

10 ndash Geacuterer les ressources financiegraveres et les actifs

11 ndash Appliquer un programme environnemental

12 ndash Geacuterer les relations exteacuterieures (actionnaires banques lois relations publiques hellip)

13 ndash Geacuterer lrsquoameacutelioration et le changement (eacutevaluer mesurer motiver qualiteacute totale)

Initiation au management copy CRCF ndash J Sornet Page 28 48

Fiche OP2 ndash Lrsquoorganisation par processus

Lrsquoeacutevolution drsquoune organisation aux activiteacutes reacutepeacutetitives vers lrsquoapproche processus est

geacuteneacuteralement progressive et se met en place par paliers

La mise en œuvre drsquoun veacuteritable management par processus doit ecirctre preacuteceacutedeacutee quand

lrsquoactiviteacute de lrsquoentreprise est complexe drsquoun recensement (une laquo cartographie des

processus raquo) pour mettre en eacutevidence les processus ou les familles de processus cleacutes critiques

pour le succegraves de lrsquoentreprise ougrave les efforts seront prioritaires

Des responsables de processus (laquo process owners raquo) sont ensuite deacutesigneacutes

Le responsable doit concevoir ses processus puis apregraves leur mise en œuvre assurer les

coordinations neacutecessaires les ameacuteliorer et les repreacutesenter aupregraves de la direction

Quand une structure par processus est mise en place des opeacuterateurs exeacutecutants

preacuteceacutedemment regroupeacutes dans les fonctions peuvent ecirctre affecteacutes aux processus et

drsquoanciens responsables de fonctions peuvent devenir des experts au service des processus

Lrsquoorganisation par processus peut imposer un degreacute eacuteleveacute drsquointeacutegration des activiteacutes donc

une polyvalence accrue des personnels et une reacuteduction des niveaux hieacuterarchiques

Elle neacutecessite pour le moins des compeacutetences eacutelargies au niveau des responsables de

processus (organisation administration technique hellip) dont le nombre doit rester limiteacute

(quelques dizaines au plus)

Sauf dans de tregraves petites structures lrsquoorganisation par processus se plaque geacuteneacuteralement sur

une structure plus classique

Initiation au management copy CRCF ndash J Sornet Page 29 48

ELEMENTS DE CORRIGE OP

OP1 Deacutefinir

Flexibiliteacute = adaptation au besoin (horaire variable chaicircnes robotiseacutees)

Systegraveme = ensemble organiseacute dans un but boicircte noire (sanguin nerveux meacutetrique laquo D raquo)

Impartition = sous-traitance ou externalisation (seacuteparation) drsquoactiviteacutes faire appel agrave des

partenaires plutocirct que faire soi-mecircme

OP2 Citer huit exemples drsquoinformations essentielles pour orienter lrsquooptimisation drsquoun processus

Montant des stocks (approvisionnements et produits finis)

Temps drsquoattente

Taux drsquoactiviteacute des ateliers

Rebuts

Deacutelai de production

Taux de reacuteclamations clients (qualiteacute)

Temps passeacutes en retouches finales

Turn over

Nombre drsquoarrecircts maladie

Accidents du travail

Dureacutee des arrecircts machines

OP3 Deacutegager les principes du toyotisme preacutesenteacute dans la fiche 31 En quoi ce systegraveme

repose trsquoil sur lrsquoapproche processus

Produire la quantiteacute juste neacutecessaire (agrave la demande) donc eacuteviter les stocks

Flexibiliteacute intelligence des chaicircnes de production

Qualiteacute (eacuteviter le coucirct de la non-qualiteacute)

La notion de processus est implicite ainsi que la chaicircne de valeur client

Initiation au management copy CRCF ndash J Sornet Page 30 48

DEFIS ET TENDANCES DU MANAGEMENT

Les meacutethodes de management se deacuteveloppent pour affronter le contexte eacuteconomique

Ce chapitre preacutesente les deacutefis auxquels le management contemporain doit faire face

1 ndash Lrsquoeacutevolution eacuteconomique contemporaine

A mesure que lrsquoactiviteacute eacuteconomique mondiale srsquoaccroicirct que la technologie eacutevolue les

changements sont de plus en plus rapides Ils introduisent des situations ineacutedites auxquelles les

entreprises doivent srsquoadapter en cherchant de nouvelles solutions de management Les trois

derniegraveres deacutecennies ont eacuteteacute notamment marqueacutees par les pheacutenomegravenes suivants (que nous

listons sans tenir compte des liens pouvant exister entre eux)

Pheacutenomegravene Traduction Effets

Deacute reacuteglementation

globalisation

financiegravere

titrisation

Libre circulation des capitaux accegraves

facile des particuliers au marcheacute

boursier (directement ou par

lrsquointermeacutediaire des OPCVM et SICAV)

Monteacutee en puissance du financement

des entreprises sur le marcheacute boursier

Fonds de pension

(retraites) et fonds

souverains (eacutetats)

Poids boursier important drsquoinvestisseurs

institutionnels qui cherchent un haut

rendement financier (dividendes ou

valorisation boursiegravere)

Pression sur les grandes entreprises

influence sur les strateacutegies

Mondialisation Liberteacute des eacutechanges internationaux Accroissement de la concurrence

recherche drsquoavantages eacuteconomiques

par la deacutelocalisation (biens et

services) la concentration des efforts

(recentrage) problegravemes drsquoemploi

multiplication des transports perte

drsquoinfluence des politiques

Baisse de lrsquoemploi

occidental

(notamment

industriel)

Moins de fabrications fabrications

automatiseacutees recours aux moyens

informatiques

Activiteacute reporteacutee sur le commerce la

conception et les services chocircmage

charge sociale

Restructurations Optimisation des entreprises

abaissement des coucircts augmentation

des marges recherche drsquoune taille

critique (eacuteconomies drsquoeacutechelle poids

sur le marcheacute)

Recentrages externalisations fusions

deacutelocalisations constitution de grands

groupes

NTIC (nouvelles

technologies de

lrsquoinformation et de

la communication)

Mise en œuvre des reacuteseaux (dont

internet) et drsquoapplications

informatiques communicantes

Nouvelles formes de commerce

marcheacute international deacutelocalisation

du travail intellectuel reacuteorganisation

de la distribution

Rareacutefaction relative

des matiegraveres

premiegraveres

Recherche de substituts exploration

miniegravere coucircts drsquoexploitation des

gisements accrus

Augmentation des coucircts variations

erratiques du cours des matiegraveres

deacutestabilisations politiques

Evolution

geacuteopolitique et

eacuteconomique

mondiale

Chute de lrsquoURSS transformation des

eacuteconomies collectivistes pays

eacutemergents (Chine Inde Breacutesil Russie)

()

Accroissement de la population

mondiale (4 agrave 6 7 milliards de 1970 agrave

2008)

Libeacuteralisme sans frein () nouvelles

puissances eacuteconomiques

opportuniteacutes de deacuteveloppement

nouveau partage des ressources

ineacutegaliteacutes baisse du soutien aux PVD

laquo Terrorisme raquo Actions armeacutees pression de groupes

armeacutes non gouvernementaux

Deacutestabilisations reacutegionales charge

des deacutepenses militaires

Deacuteveloppement

durable

Recherche drsquoune croissance eacutequitable

et respectueuse de lrsquoenvironnement

Pression sur les entreprises (eacutetats

associations de consommateurs

eacutecologistes ONG)

() Reacutecemment quelques affaires (Enron laquo subprimes raquo Vivendi Universal Socieacuteteacute

Geacuteneacuterale Airbus par exemple) et agrave plus grande eacutechelle la crise financiegravere de 2008 ont

montreacute les dangers drsquoune libeacuteralisation sans controcircles suffisants

Initiation au management copy CRCF ndash J Sornet Page 31 48

() Des alliances eacuteconomiques naissent entre pays eacutemergents (notamment en

ameacuterique centrale creacuteation de la Banque du Sud en 2008 par exemple) et lrsquoon

commence agrave imaginer une baisse progressive de lrsquoinfluence eacuteconomique des Etats

Unis

2 ndash Les deacutefis actuels du management

21 ndash Les grandes orientations

Lrsquoeacutevolution eacuteconomique suggegravere quelques pistes parfois concurrentes pour lrsquoaction du

manager contemporain On y retrouve au premier plan la construction drsquoune vision qui est

une composante commune du leadership

Objectif du manager

pour lrsquoentreprise

Justification Facteurs de reacuteussite

Construire une vision Eclairer lrsquoavenir de lrsquoentreprise partager

un but souder motiver

Effort de reacuteelle prospection

volontarisme de la direction

bonne communication

Reacuteactiviteacute et flexibiliteacute

(sous tous les aspects

agrave tous niveaux)

Srsquoadapter rapidement au marcheacute Bonne organisation des processus

personnel compeacutetent autonome

et motiveacute structure hieacuterarchique

alleacutegeacutee robotisation

Deacutegager des profits Reacutemuneacuterer les apporteurs de capitaux

srsquoautofinancer

Ajuster coucircts et structures

Exploiter les nouvelles

technologies

Reacuteactiviteacute ajuster coucircts et deacutelais

reacutepondre au marcheacute suivre les clients

Organiser le SI de faccedilon

pertinente eacuteviter le coucirct excessif

drsquoinvestissements trop en

laquo pointe raquo (laquo essuyer les placirctres raquo)

utiliser judicieusement les services

exteacuterieurs

Bacirctir des alliances

(contrats fusions)

Deacutevelopper une activiteacute limiter les coucircts

de transaction () atteindre la taille

critique et de meilleurs rendements se

recentrer sur une activiteacute profitable

Dominer les processus se donner

une identiteacute lisible externaliser se

doter drsquoune capaciteacute financiegravere

suffisante

Valoriser lrsquoimage Attirer les clients favoriser les alliances

donner confiance (apporteurs de fonds

employeacutes clients partenaires socieacuteteacute

civile)

Instaurer des regravegles de

gouvernance inteacutegrer le

deacuteveloppement durable

respecter lrsquoenvironnement

Geacuterer les risques Faire face aux aleacuteas eacuteconomiques et

technologiques (conjoncture politiques

accidents malveillance)

Creacuteer un systegraveme drsquoalerte geacuterer

la crise (reacuteaction raisonneacutee

sceacutenarios poursuite de

lrsquoexploitation dans un contexte

instable) mise en place de

proceacutedures drsquoapprentissage pour

ameacuteliorer les reacuteactions au fil du

temps

Geacuterer le changement Faire face agrave lrsquoeacutevolution de la demande

la pression sur les prix la variation des

performances financiegraveres la

concurrence la globalisation des

marcheacutes lrsquoeacutevolution technologique aux

fusions ou alliances aux changements

de reacuteglementation de direction hellip ()

Bonne communication pour

donner du sens au changement

et obtenir lrsquoadheacutesion du personnel

Rassembler et geacuterer les

connaissances former le

personnel

Innover Garder un avantage concurrentiel se

diffeacuterencier

Veille technologique et

commerciale investissement

Ouverture

internationale

Elargir le marcheacute saisir les opportuniteacutes Veille commerciale partenariats

() La theacuteorie des coucircts de transaction deacuteveloppeacutee par OE Williamson dans les

anneacutees 70 integravegre les coucircts lieacutes au recours au marcheacute (recherche et choix drsquoun

fournisseur neacutegociation reacutedaction de contrat suivi des eacutechanges risque de rupture

Initiation au management copy CRCF ndash J Sornet Page 32 48

drsquoapprovisionnement hellip) On peut en conclure que lrsquointeacutegration de diffeacuterentes

activiteacutes agrave lrsquoentreprise (la laquo firme raquo) preacutesente des avantages Mais des coucircts de

transaction internes doivent aussi ecirctre consideacutereacutes (preacuteparation organisation

surveillance hellip) et certaines formes de coopeacuteration continue avec les fournisseurs

permettent de reacuteduire le coucirct des transactions externes

() drsquoapregraves laquo Les meilleures pratiques de management raquo - Brilman Heacuterard ndash EO

Une eacutetude du Conference Board (2002) liste les deacutefis du management vus par 700 leaders

mondiaux Soit en reacutesumeacute avec indication du score correspondant

1 ndash Fideacuteliser les clients (42)

2 ndash Reacuteduire les coucircts (38)

3 ndash Accroicirctre flexibiliteacute et reacuteactiviteacute (29)

4 ndash Amener les employeacutes agrave adheacuterer aux valeurs et visions de lrsquoentreprise (26)

5 ndash Deacutevelopper et retenir les leaders (25)

6 ndash Geacuterer acquisitions et alliances (24)

7 ndash Accroicirctre lrsquoinnovation (20)

En fin de classement citoyenneteacute et reacuteputation (4) et ameacutelioration de la diversiteacute (3)

22 ndash Les techniques disponibles

Pour faire face aux deacutefis le manager dispose de nouveaux concepts et de nouvelles

techniques Le tableau ci-dessous en donne un reacutesumeacute et indique les domaines qursquoils

influencent principalement

Initiation au management copy CRCF ndash J Sornet Page 33 48

Principaux concepts techniques outils Incidence principale sur

Internet

- e-commerce (commerce eacutelectronique site

entreprise)

- CRM ou GRC (gestion de la relation client)

- e-procurement (gestion des approvisionnements

par le reacuteseau)

- messagerie eacutelectronique

- e-recrutement

Vente accegraves au marcheacute

Relation client reacuteactiviteacute personnalisation

fideacutelisation

Deacutelais coucircts

Communication transfert de donneacutees (piegraveces

jointes) tous domaines

Communication recrutement

Intranet reacuteseau drsquoentreprise SI

- knowledge management (gestion des

connaissances)

- e-learning (apprentissage en ligne)

- plateforme de travail collaboratif (groupware)

- workflow (circulation eacutelectronique de

documents enchaicircnement de processus)

- e-RH portail RH (libre accegraves aux postes agrave

pourvoir informations candidatures hellip)

- PGI (progiciel de gestion inteacutegreacute) ou ERP

Innovation capaciteacute au changement veille

documentaire

Formation du personnel accompagnement des

changements

Coordination communication interne

Coordination

Communication interne (voire internet en

externe) reacuteduction des coucircts climat drsquoentreprise

recrutement plans de carriegraveres hellip

Coucircts fiabiliteacute du systegraveme drsquoinformation deacutelais

processus (continuiteacute inteacutegration)

Logistique inteacutegreacutee

Supply Chain Management (SCM) gestion de la

logistique (incluant les approvisionnements)

Processus deacutelais coucircts

Externalisation

Valorisation du capital humain

GPEC (gestion preacutevisionnelle des emplois et

compeacutetences)

Coaching

Reacuteactiviteacute de lrsquoentreprise conservation des

compeacutetences rendements individuels turn-over

adaptation des compeacutetences motivation

Efficaciteacute individuelle controcircle reacutegulation

progregraves processus

Approche processus

Optimisation des processus

Deacutemarche qualiteacute totale (TQM ndash total quality

management)

Empowerment (empouvoirement)

Benchmarking reacuteingeacutenieacuterie

Coucircts marges qualiteacute deacutelais flexibiliteacute

externalisation eacutelargissement des compeacutetences

organisation

Ameacutelioration des processus (meacutetiers et supports)

Autonomie compeacutetences des employeacutes

Ameacutelioration des processus restructuration

Management par la valeur

Parties prenantes

Satisfaction des parties prenantes financement

motivation collaborations hellip

Collaboration inter organisations

Reacuteseaux drsquoentreprises alliances

EDI (eacutechange de donneacutees informatiseacutees) extranet

Impartition externalisation (outsourcing)

Coucircts recentrage investissements lancement

drsquoactiviteacute

Coucircts reacuteactiviteacute deacutelais relations avec

lrsquoadministration

Coucircts recentrage limitation des investissements

Ethique drsquoentreprise

Gouvernance drsquoentreprise (mode de direction

encadreacute par des regravegles)

Rocircle socieacutetal deacuteveloppement durable

environnement

Image de lrsquoentreprise reacutegulation du top

management relations actionnaires

Image peacutenaliteacutes et amendes objectifs

strateacutegiques

Initiation au management copy CRCF ndash J Sornet Page 34 48

23 ndash Le rocircle socieacutetal des entreprises

La responsabiliteacute socieacutetale de lrsquoentreprise (RSE) deacutesigne le rocircle qursquoelle prend dans la socieacuteteacute

au-delagrave de son activiteacute purement geacuteneacuteratrice de profit On parle aussi drsquoentreprise citoyenne

La RSE est indissociable du deacuteveloppement durable de porteacutee mondiale et dont les trois

piliers sont

- eacuteconomique (favoriser le deacuteveloppement les eacutechanges internationaux)

- social (accegraves aux soins eacuteducation conditions de travail hellip)

- environnemental (pollution preacuteservation des ressources hellip)

La RSE integravegre notamment une preacuteoccupation sociale de lrsquoentreprise vis-agrave-vis de ses salarieacutes

(seacutecuriteacute et santeacute au travail juste reacutemuneacuteration deacuteveloppement personnel hellip) Elle conduit agrave

tenir compte dans le management drsquoune vision exteacuterieure agrave lrsquoentreprise qui peut avoir des

reacutepercussions possibles sur son activiteacute eacuteconomique

Lrsquoentreprise peut aussi tirer avantage drsquoune deacutemarche responsable par la baisse de certains

coucircts (plus faibles consommations drsquoeacutenergies reacuteduction des transports hellip)

Le rocircle socieacutetal de lrsquoentreprise a eacuteteacute reconnu en France par la loi laquo NRE raquo de 2001 (loi sur les

nouvelles reacutegulations eacuteconomiques) qui oblige les socieacuteteacutes franccedilaise coteacutees sur un marcheacute

reacuteglementeacute agrave rendre compte dans leur rapport annuel de leur gestion sociale et

environnementale au travers de leur activiteacute

Article 116 de la loi Le rapport viseacute agrave larticle L 225-102 rend compte hellip laquo Il comprend

eacutegalement des informations dont la liste est fixeacutee par deacutecret en Conseil dEtat sur la

maniegravere dont la socieacuteteacute prend en compte les conseacutequences sociales et

environnementales de son activiteacute Le preacutesent alineacutea ne sapplique pas aux socieacuteteacutes

dont les titres ne sont pas admis aux neacutegociations sur un marcheacute reacuteglementeacute raquo

Une norme ISO 14000 integravegre ces preacuteoccupations et des taxes eacutecologiques sont

progressivement creacutees

3 ndash Le management par la valeur

31 ndash De lrsquoanalyse au management par la valeur

Lrsquoanalyse de la valeur est neacutee en 1947 aux Etats-Unis (General Electrics) Cette technique

consiste agrave eacutelaborer des produits conformes aux attentes de la clientegravele mais sans excegraves pour

trouver un bon compromis entre valeur pour le client et coucirct Le produit optimal est deacutefini agrave

partir drsquoenquecirctes qui deacuteterminent le besoin client (ou plutocirct drsquoun client laquo type raquo)

Exemple il est inutile de concevoir un petit veacutehicule citadin capable de parcourir

500 000 km sans avarie compte tenu des effets de mode et du faible kilomeacutetrage

annuel Par contre le marcheacute peut exiger un fonctionnement sans faille sur 150 000 km

soit dix ans en moyenne ce qui conditionne les coucircts de production

Cette recherche drsquoun ajustement de valeur au besoin des clients eacutetait un preacutecurseur du

management par la valeur qui recherche plus largement la creacuteation de valeur pour

chacune des parties prenantes de lrsquoentreprise tout en lui meacutenageant un reacutesultat suffisant

Plus geacuteneacuteralement le management par la valeur est deacutefini par une norme europeacuteenne (EN

12973)

Le management par la valeur est un style de management particuliegraverement destineacute agrave

mobiliser les individus agrave deacutevelopper les compeacutetences et agrave promouvoir les synergies et

Initiation au management copy CRCF ndash J Sornet Page 35 48

linnovation avec pour objectif la maximisation de la performance globale dun

organisme Le management par la valeur apporte une nouvelle faccedilon dutiliser nombre

de meacutethodes de management existantes Il est en coheacuterence avec le Management

de la qualiteacute

Cette approche du management pose de nombreuses questions notamment quelles

prioriteacutes et quelles valeurs attribuer aux parties prenantes comment appreacutehender la

perception par les parties prenantes de la valeur qui leur est affecteacutee

32 ndash La valeur client

Le processus drsquoeacutelaboration drsquoun produit qui consomme des ressources coucircteuses doit creacuteer

une valeur suffisante pour provoquer lrsquoachat par le client final La production drsquoune valeur

reconnue par le client est vitale pour lrsquoentreprise mais sa deacutetermination est parfois complexe

La valeur du produit perccedilue par le client integravegre des eacuteleacutements en partie subjectifs

- une valeur drsquousage (le produit reacutepond agrave un besoin)

- une valeur drsquoestime (lrsquoimage apporteacutee par le produit un aspect affectif)

- une valeur drsquoeacutechange (deacuteduite de lrsquoespoir de revente du produit)

Valeurs drsquousage drsquoestime et drsquoeacutechange deacutependent implicitement de la qualiteacute (un bien peu

fiable est impropre agrave lrsquousage attendu de mauvaise qualiteacute notoire il nrsquoapporte pas une

image positive et ses deacutefauts connus nuisent agrave sa revente) Une eacutevaluation de la qualiteacute

intervient donc dans la valeur perccedilue du produit

Par ailleurs le client considegravere le coucirct drsquoobtention du produit (les charges qursquoil doit supporter

pour acqueacuterir le produit lrsquoeffort qursquoil doit faire pour trouver le produit et les frais de mise agrave

disposition)

Le prix perccedilu par le client est geacuteneacuteralement supeacuterieur au prix de vente

Le client achegravete theacuteoriquement le produit qui preacutesente la diffeacuterence valeur perccedilue ndash prix

perccedilu la plus favorable ou le meilleur rapport prix perccedilu qualiteacute perccedilue et dans certains

cas celui qui a le prix produit le plus bas

Remarque les valeurs du scheacutema ci-dessus changent durant le cycle de vie du produit

(un nouveau produit peut avoir une valeur perccedilue plus eacuteleveacutee qursquoen fin de vie) La

valeur client ne peut ecirctre eacutevalueacutee que par enquecirctes et ne peut donc ecirctre deacutefinie avec

certitude

La notion de laquo satisfaction client raquo conseacutecutive agrave une vente influence aussi le prix produit et

le prix perccedilu

- lrsquoentreprise gagne sur les coucircts de recherche de clientegravele

- le client nrsquoa pas agrave rechercher un nouveau fournisseur et beacuteneacuteficie drsquoun coucirct drsquoobtention

plus bas

valeur perccedilue client

prix perccedilu client

coucirct produit Marge (valeur creacuteeacutee pour

lrsquoentreprise)

euros

prix produit

Valeur creacuteeacutee

pour le client

Initiation au management copy CRCF ndash J Sornet Page 36 48

La satisfaction du client deacutepend de facteurs qualitatifs aussi divers que la fiabiliteacute du produit

la vitesse de reacuteaction du fournisseur lrsquoattitude des commerciaux lrsquoefficaciteacute du service

apregraves-vente la netteteacute des contrats ou la justesse de la facture

Valeur perccedilue coucirct marge et satisfaction reacutesultent de processus allant de la conception du

produit jusqursquoagrave sa livraison et son apregraves-vente La deacutemarche laquo processus raquo et lrsquolaquo analyse de la

valeur raquo en forccedilant la recherche de solutions efficientes agrave tout niveau administratif

technique commercial et apregraves-vente sont donc neacutecessaires pour bien positionner

lrsquoentreprise sur son marcheacute

Pour autant le risque commercial ne peut jamais ecirctre annuleacute et lrsquooffre de lrsquoentreprise ne

satisfait geacuteneacuteralement pas en milieu concurrentiel tous ses clients potentiels

33 - La creacuteation de valeur pour les autres parties prenantes

Les salarieacutes

La creacuteation drsquoune valeur suffisante pour les salarieacutes est reconnue comme neacutecessaire car des

observations montrent que la satisfaction des clients en deacutepend Moins souvent eacutevoqueacutee en

peacuteriode de chocircmage elle nrsquoest prioritaire que pour les employeacutes dont lrsquoentreprise souhaite

conserver les compeacutetences

La laquo valeur salarieacute raquo ne comprend pas que le salaire Le sentiment drsquoappartenance agrave un

groupe la reconnaissance lrsquoaccomplissement de soi et la construction professionnelle en

sont des eacuteleacutements importants Comme pour les clients on doit ainsi distinguer la reacutetribution

perccedilue du salaire objectif

Les actionnaires

Lrsquoactionnaire apporte des fonds propres agrave lrsquoentreprise en contrepartie de titres parfois

neacutegociables en bourse et assortis drsquoun droit de vote en assembleacutee geacuteneacuterale La valeur

attribueacutee aux actionnaires est servie en termes moneacutetaires (dividende ou augmentation de la

valeur du titre neacutegociable)

Remarque des facteurs non moneacutetaires comme lrsquoimage de lrsquoentreprise qui deacutepend

en partie de sa communication peuvent influencer la deacutecision drsquoachat de vente ou

de conservation des titres par lrsquoactionnaire

Reacutetribution perccedilue euros

Salaire objectif

Avantage non

moneacutetaire de

lrsquoemploi

Initiation au management copy CRCF ndash J Sornet Page 37 48

Compte tenu de lrsquoimportance croissante de lrsquoactionnariat dans le financement des grandes

entreprises coteacutees en bourse et notamment des investisseurs institutionnels comme les fonds

de pension des indicateurs speacutecifiques ont eacuteteacute introduits pour appreacutecier la performance des

entreprises vue par les actionnaires Par exemple la valeur ajouteacutee eacuteconomique (EVA reg

economic value added marque deacuteposeacutee de Stern Stewart ou VAE ndash valeur ajouteacutee

eacuteconomique parfois deacutenommeacutee VEC ndash valeur eacuteconomique creacuteeacutee) qui prend en compte le

coucirct du capital

LrsquoEVA correspond tregraves scheacutematiquement au calcul suivant

EVA = (PO) profit opeacuterationnel ndash (C) coucirct du capital X (CE) capitaux employeacutes

LrsquoEVA neacutecessite en pratique des retraitements assez complexes Le PO peut se deacuteterminer

selon les principes suivants

- PO = reacutesultat drsquoexploitation (avant inteacuterecircts) ndash impocirct

- PO = beacuteneacutefice courant (tenant compte des inteacuterecircts) + inteacuterecircts ndash eacuteconomie drsquoimpocirct sur les

inteacuterecircts (on exclue les eacuteleacutements financiers et lrsquoimpocirct correspondant) ndash impocirct

- lrsquoimpocirct pris en compte correspond au profit opeacuterationnel consideacutereacute (dans les cas courants agrave

13 du PO)

C = taux moyen de reacutemuneacuteration du capital (reacutesultant par exemple du dividende exigeacute de

certains investisseurs et des taux drsquoemprunts bancaires)

CE = capitaux propres et dettes portant inteacuterecirct

Remarque le profit opeacuterationnel ou reacutesultat opeacuterationnel correspond au NOPAT ndash net

operating profit after tax - anglo-saxon LrsquoEVA est eacutegale au NOPAT diminueacute de la

reacutemuneacuteration des capitaux

Exemple lrsquoentreprise X dispose drsquoun capital de 2 500 000 euro et reacutealise un beacuteneacutefice net

drsquoimpocirct de 450 000 euro (taux 33 13) Un dividende de 6 doit ecirctre verseacute aux

actionnaires et la banque lui a accordeacute un precirct de 1 200 000 euro agrave 4 Les autres

constituants des reacutesultats financier et exceptionnel sont neacutegligeables

Reacutesultat opeacuterationnel = 450 000 + 004 x 1 200 000 x 23 = 482 000 euro

Coucirct du capital = 006 x 2 500 000 + 004 x 1 200 000 x 23 = 182 000 euro

EVA = 300 000 euro

Coucirct moyen pondeacutereacute du capital (C) = (004 x 1 200 000 x23 + 006 x 2 500 000)

3 700 000 Soit 492

Si lrsquoEVA est positive lrsquoentreprise creacuteeacutee de la valeur apregraves reacutemuneacuteration des capitaux et sa

valeur boursiegravere doit augmenter

Lrsquoutilisation de lrsquoEVA comme indicateur influence le management de lrsquoentreprise car il y a

trois moyens pratiques drsquoaugmenter lrsquoEVA

- augmenter le reacutesultat opeacuterationnel

- lancer des investissements ayant une rentabiliteacute supeacuterieure agrave C

- eacuteliminer les activiteacutes ayant une rentabiliteacute infeacuterieure agrave C

Remarque lrsquoutilisation sans nuance de lrsquoEVA comme critegravere de management peut

poser problegraveme Le calcul de lrsquoEVA repose sur des ajustements comptables il est donc

sujet agrave manipulations (provisions capitalisation ou non de la RD hellip) Par ailleurs le

critegravere laquo EVA raquo pris isoleacutement peut conduire agrave chercher la rentabiliteacute agrave court terme agrave

reacuteduire les investissements prospectifs et donc nuire agrave terme au deacuteveloppement de

lrsquoentreprise

Initiation au management copy CRCF ndash J Sornet Page 38 48

Les fournisseurs reccediloivent le paiement de leurs factures plus ou moins rapidement (le deacutelai

de paiement repreacutesente une valeur consentie au fournisseur)

Lrsquoentreprise peut accroicirctre la valeur apporteacutee agrave ses fournisseurs par des actions cibleacutees

comme une contribution agrave la formation de leurs personnels certains transferts de

technologie ou de savoir faire agrave des sous-traitants une coopeacuteration suivie favorisant leur

deacuteveloppement lrsquointeacutegration agrave des campagnes de promotion

A noter que la valeur consentie aux fournisseurs peut avoir une influence sur la qualiteacute et les

deacutelais de livraison des produits

La collectiviteacute reccediloit des taxes et parfois des prestations en nature par deacutefaut ou explicites

(effort de preacuteservation de lrsquoenvironnement ameacutenagement du territoire par les implantations

aide mateacuterielle agrave des projets participation agrave la formation par exemple)

APPLICATIONS DT

DT1 Deacutefinir expliquer deacutereacuteglementation socieacutetal eacuteconomies drsquoeacutechelle coaching EDI

gouvernance

DT2 Deacuteterminer en quoi la deacutemarche TQM srsquoinscrit dans les deacutefis actuels du management

DT3 Apregraves avoir consulteacute les documents ci-dessous extraits du site drsquoAir France

(httpdeveloppement-

durableairfrancecomFRfrlocaldemarcheN4_positionnement_pphtm)

exposer les enjeux et les limites de la RSE et de la gestion des parties prenantes

Initiation au management copy CRCF ndash J Sornet Page 39 48

Dialogue avec les parties prenantes

Initiation au management copy CRCF ndash J Sornet Page 40 48

Attentes des parties prenantes

Initiation au management copy CRCF ndash J Sornet Page 41 48

Creacuteation de valeur pour les parties prenantes

La creacuteation de valeur pour les parties prenantes est au cœur de la strateacutegie du Groupe Le scheacutema de

distribution financiegravere ci-dessous donne un aperccedilu de la distribution des recettes du Groupe aux

diffeacuterentes parties prenantes actionnaires collaborateurs fournisseurs pouvoirs publics

collectiviteacutes locales etc

Initiation au management copy CRCF ndash J Sornet Page 42 48

Fiche DT1 ndash Extrait du sommaire de laquo Problegravemes eacuteconomiques raquo No 2894

La gestion des entreprises bouleverseacutee par les technologies de linternet

Reacutealiteacutes industrielles - Annales des Mines Jean-Michel Yolin

Avec lavegravenement de linternet les processus de conception de production et de vente sont

radicalement remis en cause Quel que soit le secteur dactiviteacute les technologies de linternet

permettent en effet de reacuteduire les deacutelais et de passer dun processus discontinu agrave un processus

continu Lorganisation des entreprises et leur mode de gestion en sont profondeacutement bouleverseacutes

tant au niveau individuel que collectif Linternet rend ainsi possible la reacutealisation dobjectifs que les

entreprises cherchaient agrave atteindre depuis longtemps sans y parvenir meilleure eacutecoute du client

travail sans stocks en flux tendu hieacuterarchies plates autorisant une grande reacuteactiviteacute flexibiliteacute dans

lorganisation et loutil de production acceacuteleacuteration du renouvellement des produits entreprises en

reacuteseau ougrave chacune se recentre sur son cœur de meacutetier etc

Le laquo knowledge management raquo ou comment geacuterer les connaissances

Document de travail du LAMSADE - Michel Grundstein

Peter Drucker lavait preacutedit le capital immateacuteriel eacutetait voueacute agrave devenir un facteur de compeacutetitiviteacute

pour lentreprise La libeacuteralisation des eacutechanges acceacutelegravere les processus de deacutecision de lentreprise

et implique que lassimilation des informations soit agrave la fois de meilleure qualiteacute et plus rapide Ainsi

la fonction qui consiste agrave manager les connaissances au sein de lentreprise savegravere primordiale

Bien que la prise de conscience de limportance du capital immateacuteriel ait eacuteteacute tardive - le concept

de knowledge management est apparu en France aux Etats-Unis et au Japon au milieu des

anneacutees 1990 - agrave lheure actuelle lorganisation de leacutechange dinformations et le partage des

connaissances sont devenus des facteurs cleacutes dune gestion performante de lentreprise Ils

doivent sinscrire dans un projet global destineacute agrave mettre en valeur les savoirs et les savoir-faire

individuels et collectifs

Les leccedilons du laquo coaching raquo pour le management de la qualiteacute

Humanisme et Entreprise - Martine Brasseur

Parmi les nouvelles formes de management en vogue dans les entreprises le coaching figure en

bonne place Appliqueacute au management de la qualiteacute il sagit dune pratique

daccompagnement destineacutee agrave initier et agrave faciliter le processus de deacuteveloppement dun individu

La deacutemarche consiste agrave affirmer que tout individu est en quecircte de qualiteacute agrave condition toutefois

de ne pas lui imposer des contraintes lempecircchant de progresser On considegravere notamment les

erreurs comme potentiellement feacutecondes En deacutefinitive le coach donne au coacheacute la permission

de reacuteussir en lui donnant aussi la permission deacutechouer

Initiation au management copy CRCF ndash J Sornet Page 43 48

Fiche DT2 ndash Management strateacutegique les sept deacutefis agrave relever dici agrave 2016

Extrait drsquoun article du site wwwlentreprisecom -Sabine Blanc - Mis en ligne le 20032007

(httpwwwlentreprisecom325article11977html)

Une eacutetude anglaise publieacutee par lopeacuterateur Orange Grande-Bretagne deacutecrypte la mutation

des formes de travail et les enjeux majeurs pour les entreprises de demain afin decirctre au top

de la compeacutetitiviteacute Voici les challenges-cleacutes pour les managers qui veulent rester dans la

course hellip

1 - Future organisation du travail les quatre laquo mondes raquo possibles

La reacutealiteacute sera probablement un meacutelange de ces quatre sceacutenarios souligne lrsquoeacutetude

Les mondes mutuels Tout se passe dans le cadre des communauteacutes locales vie priveacutee

comme professionnelle Le modegravele coopeacuteratif preacutevaut au lieu du laquo big business raquo Oublieacutes

aussi dans ce systegraveme les trajets pour aller au bureau les gens preacutefegravereront travailler dans de

petites entreprises locales souvent connecteacutees au reacuteseau drsquoautres structures similaires

Les laquo reacutepondants raquo (en anglais laquo replicants raquo) La figure du consultant freelance deviendra

dominante tandis que celle du salarieacute deacuteclinera Il ne sera pas rare de travailler pour plusieurs

entreprises On perdra en seacutecuriteacute de lrsquoemploi en visibiliteacute et en routine ce que lrsquoon gagnera

en liberteacute La majeure partie des tacircches srsquoeffectuera chez soi avec la possibiliteacute de srsquoinstaller

temporairement dans les bureaux de son client du moment Dans un contexte dincertitude

sur lrsquoavenir les travailleurs alterneront peacuteriodes drsquoactiviteacute intense et repos Ce sera agrave eux

drsquoaller vers les entreprises et non lrsquoinverse mecircme si celles-ci devront veiller agrave rester attractives

Les cottages eacutelectroniques Comme ce nom le suggegravere le teacuteleacutetravail deviendrait la norme

univers priveacute et professionnel se confondant Plus besoin de subir une heure de transport les

salarieacutes se logueront de chez eux sur le reacuteseau de lrsquoentreprise Les reacuteunions se tiendront dans

de petits bureaux centraux situeacutes agrave courte distance La flexibiliteacute du temps de travail srsquoimpose

Les salarieacutes disposeront de plus de marge de liberteacute dans leur activiteacute

Les disciples de la nueacutee Cette appellation poeacutetique cache simplement une extension de

lrsquoorganisation actuelle des grandes entreprises avec des salarieacutes se rendant sur un lieu de

travail centraliseacute Le rocircle croissant des technologies de lrsquoinformation multipliera les faccedilons de

collaborer et accroicirctra lrsquoefficaciteacute Le controcircle du travail sera omnipreacutesent La frontiegravere entre

travail et vie priveacutee restera marqueacutee

2 - Sept deacutefis pour les entreprises et leur managers

Quoi qursquoil advienne les entreprises et leurs dirigeants devront concentrer leurs efforts sur sept

points-cleacutes pour srsquoadapter Voici quelques exemples de probleacutematiques souleveacutees par le

rapport et des pistes de solution

Le leadership Les managers devront entre autres savoir persuader et influencer des

travailleurs beaucoup plus indeacutependants Ils auront aussi agrave repenser les niveaux auxquels

prendre les deacutecisions strateacutegiques en haut ou au contraire agrave des degreacutes moins eacuteleveacutes de la

pyramide hieacuterarchique

gt Faire du management une force facilitant les activiteacutes transversales plutocirct que la reacuteduire agrave

la seule fonction de deacutecision

La culture drsquoentreprise Davantage de salarieacutes capables de reacutefleacutechir seront neacutecessaires

tandis que les tacircches qui peuvent ecirctre automatiseacutees ou scripteacutees diminueront Un des

enjeux creacuteer une culture agrave mecircme drsquoattirer et drsquoencourager les personnes preacutesentant ces

qualiteacutes de reacuteflexion requises dans un contexte de compeacutetition accrue et de plus grande

indeacutependance des travailleurs

Initiation au management copy CRCF ndash J Sornet Page 44 48

gt Passer si neacutecessaire drsquoune culture drsquoentreprise forte agrave un mode drsquoengagement plus

consensuel moins rebutant

La marque Conseacutequence du recours croissant agrave lrsquo laquo outsourcing raquo lrsquoimage drsquoune marque

deacutependra plus drsquoagents exteacuterieurs qui ne fonctionnent pas forceacutement selon le mecircme mode

drsquoorganisation Comment garder le controcircle dessus

gt Choisir le mode qui corresponde le plus agrave vos valeurs et preacutevoir un programme de risk

management qui mette en eacutevidence ougrave les conflits sont susceptibles de jaillir

Lrsquoinnovation Plus que jamais il faudra faire face agrave une acceacuteleacuteration du rythme de

lrsquoinnovation en proposant constamment des solutions adapteacutees

gt Tisser des partenariats strateacutegiques avec drsquoautres entreprises pour partager les coucircts et les

fruits de lrsquoinnovation

Le deacutefi opeacuterationnel et technologique De quelle faccedilon controcircler lrsquoinformation crsquoest-agrave-dire

faire en sorte que les bonnes personnes accegravedent facilement agrave une information toujours en

phase tout en maintenant la seacutecuriteacute

gt Recourir agrave des laquo feuilles de route des futurs raquo syntheacutetisant en une page les indicateurs

sociaux et de consommation ainsi que les eacutevolutions technologiques et leacutegislatives qui

influent sur les changements et indiquant comment ils modifient vos marcheacutes vos clients et

votre organisation

La qualiteacute Si de nouveaux proceacutedeacutes ont pu deacutegrader la qualiteacute comme le recours agrave des

centres drsquoappel externaliseacutes drsquoautres ideacutees se sont reacuteveacuteleacutees plus prometteuses comme en

teacutemoigne le succegraves de certaines compagnies aeacuteriennes low cost Elles ont su conjuguer prix

serreacutes et services eacuteleveacutes ce qui devra devenir la norme estime lrsquoeacutetude

gt Continuer de rechercher la qualiteacute Elaborez aussi une bonne prestation service qui inclut

une livraison de qualiteacute voire creacuteez-la en partenariat avec les consommateurs

La leacutegislation La question de la proprieacuteteacute intellectuelle pourrait ecirctre probleacutematique Elle est

deacutejagrave source de conflits comme en teacutemoigne le procegraves pour violation de brevet intenteacute agrave RIM

le fabricant canadien du Blackberry par NTP Que pourra-t-on et que faudra-t-il proteacuteger par

un brevet Il sera eacutegalement neacutecessaire drsquoadapter la leacutegislation aux nouveaux modes

drsquoorganisation

gt Collaborer avec les acteurs du mecircme secteur et les leacutegislateurs pour deacutevelopper les

modegraveles des lieux de travail du futur et bacirctir le droit le plus adeacutequat

Orange a-t-il vu juste dans ses preacutevisions Rendez-vous dans neuf ans pour la reacuteponsehellip

Initiation au management copy CRCF ndash J Sornet Page 45 48

Fiche DT3 ndash Le management par la qualiteacute totale

Extrait drsquoune lettre drsquoinformation du cabinet Baud Accordance Consulting AD2 consultants ndash

2002

1 - Le TQM (Total Quality Management) offre pour lentreprise une vision de la qualiteacute plus

large et transversale

Son principe est simple La finaliteacute de lEntreprise est de deacutevelopper la satisfaction de ses

clients tout en eacutetant beacuteneacuteficiaire cest agrave dire pas agrave nimporte quel prix Elle doit ameacuteliorer sa

rentabiliteacute au travers de la deacutemarche qualiteacute La Qualiteacute Totale vise agrave fournir aux clients

externes et internes une reacuteponse adeacutequate agrave leurs attentes dans le meilleur rapport qualiteacute

prix la meilleure efficience

Elle considegravere pour cela lensemble des processus de lentreprise ayant une incidence sur la

qualiteacute et la satisfaction des clients

Le TQM fait ainsi une large place agrave

la deacutefinition et la planification de la strateacutegie geacuteneacuterale

la coheacuterence de la politique qualiteacute avec la strateacutegie

la deacutemultiplication de la politique qualiteacute dans toutes les directions de lentreprise

la relation client fournisseur interne

la prise en compte de lenvironnement concurrentiel

la consideacuteration de lensemble des risques potentiels financiers sociaux concurrentielshellip

limplication et la motivation du personnel

lanalyse des besoins des clients et le positionnement marketing

la maicirctrise des processus transverses internes

les reacutesultats sous tous ses aspects y compris financiers commerciaux image

De nombreux reacutefeacuterentiels sont relatifs agrave la Qualiteacute Totale hellip Tous ces reacutefeacuterentiels imposent un

questionnement plus profond et indiscret sur le mode de fonctionnement de lentreprise et

son management

helliphellip

2 - LISO 9001 2000 au travers du deacuteploiement des processus (management supports

reacutealisation et ameacutelioration continue) reacutepond quelque peu agrave la mecircme logique

LISO est une ouverture indeacuteniable vers la logique du TQM mais ne se reacutefegravere pas agrave la notion

defficience

Les dirigeants sont cependant sensibles agrave la neacutecessaire reacuteduction des coucircts de non-qualiteacute

et dobtention de la qualiteacute agrave la rentabiliteacute du systegraveme de management de la qualiteacute

mais ne perccediloivent pas toujours la qualiteacute comme une deacutemarche globale

Les deacutemarches qualiteacute commencent bien souvent par la remise en cause de lorganisation

leacutevaluation critique de son efficaciteacute lexamen des processus et la mise en eacutevidence des

lourdeurs administratives

La qualiteacute devient laffaire de tous hellip

Initiation au management copy CRCF ndash J Sornet Page 46 48

Fiche DT4 ndash Le deacuteveloppement durable et la RSE

Extrait du site wwwvigeocom

(httpwwwvigeocomcsr-rating-agencyfrmethodologiecriteres-de-recherche37-

criteres-d-analysehtml)

Deacuteveloppement durable laquo un deacuteveloppement qui reacutepond aux besoins du preacutesent sans compromettre

la capaciteacute des geacuteneacuterations futures de reacutepondre aux leurs raquo (Commission mondiale sur lrsquoenvironnement

et le deacuteveloppement ndash 1987)

Reacutefeacuterentiel drsquoeacutevaluation des entreprises par le groupe Vigeacuteo (le groupe mesure les performances et le

niveau de maicirctrise des risques de responsabiliteacute sociale des entreprises et des organisations - site

wwwvigeocom)

1 Ressources Humaines Ameacutelioration continue des relations professionnelles des relations drsquoemploi et des conditions de travail 2 Droits humains sur les lieux de travail Respect de la liberteacute syndicale et promotion de la neacutegociation collective non discrimination et promotion de lrsquoeacutegaliteacute eacutelimination des formes de travail proscrites (enfants travail forceacute) preacutevention des traitements inhumains ou deacutegradants de type harcegravelements sexuels protection de la vie priveacutee et des donneacutees personnelles 3 Environnement Protection sauvegarde preacutevention des atteintes agrave lenvironnement mise en place drsquoune strateacutegie manageacuteriale approprieacutee eacuteco conception protection de la biodiversiteacute et maicirctrise rationnelle des impacts environnementaux sur lrsquoensemble du cycle de vie des produits ou services

4 Comportements sur les marcheacutes Prise en compte des droits et inteacuterecircts des clients inteacutegration de standards sociaux et environnementaux dans la seacutelection des fournisseurs et sur lrsquoensemble de la chaicircne drsquoapprovisionnement preacutevention effective de la corruption respect des regravegles concurrentielles 5 Gouvernement drsquoentreprise Efficience et probiteacute assurance de lrsquoindeacutependance et de lrsquoefficaciteacute du Conseil drsquoadministration effectiviteacute et efficience des meacutecanismes drsquoaudit et de controcircle et notamment inclusion des risques de responsabiliteacute sociale respect des droits des actionnaires et notamment des minoritaires transparence et rationaliteacute de la reacutemuneacuteration des dirigeants 6 Engagement socieacutetal Effectiviteacute inteacutegration manageacuteriale de lrsquoengagement contribution au deacuteveloppement eacuteconomique et social des territoires drsquoimplantation et de leurs communauteacutes humaines engagements concrets en faveur de la maicirctrise des impacts socieacutetaux des produits et des services contribution transparente et participative agrave des causes drsquointeacuterecirct geacuteneacuteral

Initiation au management copy CRCF ndash J Sornet Page 47 48

ELEMENTS DE CORRIGE DT DT1 Deacutefinir expliquer

Deacutereacuteglementation = suppression des contraintes eacuteconomiques (libre eacutechange des biens et

capitaux)

Socieacutetal = qui se rapporte agrave la structure agrave lrsquoorganisation ou au fonctionnement de la socieacuteteacute

Economies drsquoeacutechelle = reacuteduction des coucircts lieacutee au niveau drsquoactiviteacute (amortissement des

charges fixes)

Coaching = accompagnement de personnes ou deacutequipes pour le deacuteveloppement de leurs

potentiels

EDI = eacutechange de donneacutees informatiseacutees ET standardiseacutees (ex SWIFT bancaire edifact

documents deacuteclaratifs)

Gouvernance = exercice du pouvoir la bonne gouvernance est participative et eacutequitable

conforme agrave lrsquointeacuterecirct commun

DT2 Deacuteterminer en quoi la deacutemarche TQM srsquoinscrit dans les deacutefis actuels du management

Voir notamment fiche 43

Maicirctrise des processus reacuteduction des coucircts reacuteactiviteacute et satisfaction de la clientegravele = faire

face agrave la concurrence

Ameacutelioration de lrsquoimage motivation du personnel

DT3 Apregraves avoir consulteacute les documents ci-dessous extraits du site drsquoAir France

(httpdeveloppement-

durableairfrancecomFRfrlocaldemarcheN4_positionnement_pphtm)

exposer les enjeux et les limites de la RSE et de la gestion des parties prenantes

Trame geacuteneacuterale possible

Introduction

Les deacutefis contemporains (accroissement de la concurrence devenue mondiale recherche

de nouveaux avantages concurrentiels pression de la socieacuteteacute besoin drsquoimage et de projet

lisible pour mener lrsquoentreprise crise et scandales du libeacuteralisme hellip) RSE et PP

Deacuteveloppement (voir cours)

1 ndash Parties prenantes et management par la valeur

PP deacutefinir citer reacutesumer lrsquoavantage rechercheacute (fideacuteliser motiver recherche drsquoalliances

implicites)

PP moyens (dont exemples AF) et meacutethode de management par la valeur (reacutepartie)

2 ndash La responsabiliteacute socieacutetale de lrsquoentreprise

RSE 3 axes

- eacuteconomique (favoriser le deacuteveloppement les eacutechanges internationaux)

- social (accegraves aux soins eacuteducation conditions de travail hellip)

- environnemental (pollution preacuteservation des ressources hellip)

RSE gouvernance drsquoentreprise facteur drsquoimage inteacutegrable dans la deacutemarche PP

Article 116 de la loi Le rapport viseacute agrave larticle L 225-102 rend compte hellip laquo Il comprend

eacutegalement des informations dont la liste est fixeacutee par deacutecret en Conseil dEtat sur la maniegravere

dont la socieacuteteacute prend en compte les conseacutequences sociales et environnementales de son

activiteacute Le preacutesent alineacutea ne sapplique pas aux socieacuteteacutes dont les titres ne sont pas admis aux

neacutegociations sur un marcheacute reacuteglementeacute raquo

Initiation au management copy CRCF ndash J Sornet Page 48 48

RSE exemple AF (ONG fournisseurs)

3 ndash Liens entre PP et RSE

- la RSE introduit de nouvelles PP

- la RSE suppose le respect des PP usuelles (employeacutes clients notamment)

4 - Probleacutematique

- deacutefinir la valeur reacuteellement apporteacutee par une gestion des PP (confusion salaire ndash valeur

idem impocircts hellip ex laquo valeur ajouteacutee raquo)

- communication (neacutecessaire mais aller au-delagrave)

- marginaliteacute des deacutepenses RSE (efficaciteacute sinceacuteriteacute de lrsquoengagement marge de manœuvre)

- charge RSE reporteacutee sur des tiers (ex fournisseurs AF)

- inteacutegration de facteurs non visibles en comptabiliteacute (pertes drsquoemploi nuisances hellip)

Conclusion

Voies incontournables mais pouvant nrsquoavoir qursquoun effet superficiel et temporaire Voir utiliteacute

drsquoaccompagnement leacutegislatif de regravegles de gouvernance

Initiation au management copy CRCF ndash J Sornet Page 18 48

- certains services peuvent ecirctre fournis agrave distance sans contact direct avec le client et

distribueacutes par reacuteseau (tenue de comptabiliteacute affacturage gestion clientegravele centre drsquoappel

hellip)

Remarque les services repreacutesentent 75 de lrsquoactiviteacute eacuteconomique franccedilaise

23 ndash La nature de lrsquoorganisation

Les organisations publiques franccedilaises (administrations centrales collectiviteacutes territoriales

hocircpitaux hellip) repreacutesentent une part importante de lrsquoactiviteacute (environ 30 des emplois) La

fonction publique regroupe des organisations aux finaliteacutes diverses et qui ont des problegravemes

de gestion similaires agrave ceux des entreprises auxquelles elles peuvent emprunter des principes

de management Notamment

- pour controcircler les coucircts et assurer la qualiteacute des services

- pour communiquer avec les administreacutes ou les usagers

- pour motiver les personnels et geacuterer les ressources humaines

La transposition directe des techniques de gestion et de management nrsquoest cependant pas

toujours possible car

- la comptabiliteacute publique obeacuteit agrave des regravegles speacutecifiques (proceacutedure budgeacutetaire

notamment)

- le laquo client raquo ne paye pas toujours la prestation du moins directement

- la concurrence est parfois inexistante

- les grandes administrations centraliseacutees sont soumises agrave des choix politiques geacuteneacuteraux

parfois sans connexion eacutevidente avec les besoins opeacuterationnels

- le statut des personnels et les grilles de salaires limitent les possibiliteacutes de gestion des

ressources humaines

Remarque la LOLF (loi organique relative aux lois de finances) est entreacutee en vigueur en

2006 Elle alloue des moyens budgeacutetaires en fonction de programmes et remplace la

reconduction automatique de 90 des budgets Cette reacuteforme se heurte toutefois agrave la

lourdeur des grands ministegraveres ougrave la complexiteacute des activiteacutes est difficile agrave

appreacutehender et ougrave des inerties culturelles peuvent exister agrave tout niveau

Les associations loi de 1901 peuvent avoir une activiteacute comparable agrave celle de grandes

entreprises (voir par exemple les associations de santeacute ou professionnelles) et leur

management est alors similaire malgreacute lrsquoabsence de but lucratif (les beacuteneacutefices ne sont pas

distribuables) Elles ont drsquoailleurs en France un poids eacuteconomique important (elles emploient

environ 1 600 000 salarieacutes)

Cependant lrsquoadheacutesion agrave un systegraveme de valeurs fondateur de lrsquoassociation ou la limite de

lrsquoautoriteacute (quand un volant de beacuteneacutevoles important participe agrave lrsquoactiviteacute) peut introduire des

nuances

- le renforcement des objectifs socieacutetaux

- la faiblesse des relations hieacuterarchiques

- des contraintes de gestion du temps des beacuteneacutevoles

- des modaliteacutes particuliegraveres de recrutement et de motivation des dirigeants

24 ndash Les facteurs contingents

La theacuteorie de la contingence montre qursquoune structure drsquoentreprise nrsquoest efficace que dans

une situation deacutetermineacutee et qursquoil nrsquoexiste que des solutions de management construites dans

un contexte preacutecis

Le management doit ainsi srsquoadapter agrave des facteurs contingents qui ne peuvent ecirctre

controcircleacutes du moins agrave bregraveve eacutecheacuteance Ces facteurs sont par exemple

- lrsquoancienneteacute de lrsquoentreprise (plus elle est ancienne plus lrsquoentreprise a tendance agrave reacutepeacuteter

des comportements eacuteprouveacutes)

Initiation au management copy CRCF ndash J Sornet Page 19 48

- la taille de lrsquoentreprise (la grande entreprise a une composante administrative plus

deacuteveloppeacutee)

- le systegraveme de production (tregraves standardiseacute complexe automatiseacute hellip)

- lrsquoenvironnement

3 ndash Le management et les parties prenantes

Lrsquoentreprise a pour vocation premiegravere de mettre des produits agrave disposition de ses clients en

reacutealisant un profit Pour y arriver elle doit aussi satisfaire ses parties prenantes salarieacutes

actionnaires fournisseurs hellip

Est partie prenante agrave lrsquoentreprise laquo tout groupe ou individu qui peut ecirctre affecteacute ou est

affecteacute par les buts de lrsquoorganisation hellip raquo (Freeman ndash 1984)

Les parties prenantes attendent agrave des degreacutes divers de profiter drsquoune creacuteation de valeur en

provenance de lrsquoentreprise qui doit reacutepondre agrave ces attentes pour assurer sa peacuterenniteacute ou

favoriser son deacuteveloppement

On distingue les parties prenantes primaires ou principales qui sont essentielles agrave lrsquoentreprise

et qui ont geacuteneacuteralement une relation formelle avec elle (clients associeacutes et actionnaires

precircteurs salarieacutes fournisseurs collectiviteacutes) et les parties prenantes secondaires dont

lrsquoinfluence est diffuse (groupes de pression associations meacutedias instances europeacuteennes

agences de notation hellip)

Remarque la consideacuteration de lrsquoensemble des parties prenantes (laquo stakeholders raquo - les

deacutepositaires) fait contrepoids agrave lrsquoimportance accordeacutee aux seuls actionnaires

(laquo shareholders raquo)

Les organisations nrsquoayant pas drsquoobjectif de profit doivent aussi satisfaire leurs parties

prenantes apporter un service aux usagers dans les meilleures conditions eacuteconomiques

limiter un budget assurer la qualiteacute des relations avec les fournisseurs hellip

Dans cette optique le management doit organiser lrsquoaction de faccedilon agrave eacutequilibrer des forces

parfois divergentes

- le contexte fait pression sur lrsquoorganisation contrainte agrave optimiser ses reacutesultats

- lrsquoorganisation cherche par son action agrave assurer sa peacuterenniteacute son deacuteveloppement (en

reacutealisant des profits dans le cas de lrsquoentreprise) et agrave satisfaire ses parties prenantes

- le management agit en pilotant les actions pour contrebalancer la pression du contexte

Actions de

lrsquoorganisation

Management Contexte

Parties

prenantes

Initiation au management copy CRCF ndash J Sornet Page 20 48

APPLICATIONS MP

MP1 Deacutefinir contingent gestion budgeacutetaire

MP2 Deacuteterminer les parties prenantes drsquoun hocircpital public et leurs principales attentes

Mecircme question pour les organisations suivantes

- SNCF (entreprise publique)

- Peugeot

- MAIF (mutuelle drsquoassurance)

MP3 En les situant dans le cycle des activiteacutes du management trouver les actions agrave mener

dans les situations suivantes

- baisse de 10 des ventes dans une entreprise industrielle (produits meacutenagers le reacuteseau de

distribution vient drsquoecirctre reacuteorganiseacute)

- idem dans une entreprise de vente par correspondance soumise agrave la concurrence internet

(les ventes stagnaient depuis six mois malgreacute les efforts promotionnels)

- augmentation des deacutelais drsquoattente des consultations dans une clinique (lrsquohocircpital voisin a

fermeacute son service drsquourgences)

Initiation au management copy CRCF ndash J Sornet Page 21 48

ELEMENTS DE CORRIGE MP

MP1 Deacutefinir (dans le contexte drsquoune entreprise) contingent gestion budgeacutetaire

Contingent = imposeacute par lrsquoexteacuterieur Contingence = effet du hasard de la rencontre de

plusieurs eacuteveacutenements indeacutependants (variables explicatives que lrsquoon ne peut influencer)

Gestion budgeacutetaire = technique drsquoadministration des entreprises srsquoappuyant sur des

preacutevisions dont on deacuteduit apregraves accord des responsables des attributions de moyens sur une

dureacutee limiteacutee Une analyse reacuteguliegravere des eacutecarts entre preacutevisions et reacutealisations permet ensuite

le pilotage des activiteacutes Le budget est un cadre incitatif

La laquo planification budgeacutetaire raquo consiste agrave traduire en budgets une planification strateacutegique

avec systegraveme de reporting

MP2 Deacuteterminer les parties prenantes drsquoun hocircpital public et leurs principales attentes

Mecircme question pour les organisations suivantes

- SNCF (entreprise publique)

- Peugeot

- MAIF (mutuelle drsquoassurance)

Hocircpital

- patients (qualiteacute des soins)

- CNAM (baisse des coucircts)

- collectiviteacute locale (service aux administreacutes)

- eacutetat (ameacutenagement du territoire maicirctrise des budgets optimisation)

- employeacutes (salaire conditions de travail et satisfaction)

- fournisseurs ndash pharmacie autres (CA paiement reacutegulier)

- associations de patients (qualiteacute proximiteacute des soins)

SNCF

- usagers et associations drsquousagers (proximiteacute reacutegulariteacute prix du service)

- reacuteseau ferreacute de France (optimisation des lignes paiement adapteacute)

- fournisseurs (CA paiement reacutegulier)

- employeacutes (salaire conditions de travail seacutecuriteacute de lrsquoemploi)

- eacutetat (ameacutenagement du territoire)

- collectiviteacutes locales (service)

Peugeot

- clients (qualiteacute prix SAV relation commerciale)

- fournisseurs (CA reacutegulariteacute de lrsquoactiviteacute)

- employeacutes (salaire conditions de travail seacutecuriteacute de lrsquoemploi)

- eacutetat (taxes)

- collectiviteacute locale (emploi dynamisation eacuteconomique preacuteservation de lrsquoenvironnement)

- associations de protection de lrsquoenvironnement (activiteacute propre baisse des eacutemissions

nouvelles eacutenergies)

MAIF

- socieacutetaires (protection relation assureur tarif mesureacute)

- professionnels de lrsquoautomobile et autres (agreacutement marge de manœuvre reacuteparations tarifs

eacuteleveacutes)

- fournisseurs (CA paiement reacutegulier)

- eacutetat (taxes engagement pour la seacutecuriteacute)

- employeacutes (salaire conditions de travail seacutecuriteacute de lrsquoemploi)

Initiation au management copy CRCF ndash J Sornet Page 22 48

MP3 En les situant dans le cycle des activiteacutes du management trouver les actions agrave mener

dans les situations suivantes

- baisse de 10 des ventes dans une entreprise industrielle (produits meacutenagers le reacuteseau de

distribution vient drsquoecirctre reacuteorganiseacute)

Adapter le pilotage motiver cadrer si insuffisant retoucher une organisation deacutefectueuse

- idem dans une entreprise de vente par correspondance soumise agrave la concurrence internet

(les ventes stagnaient depuis six mois malgreacute les efforts promotionnels)

Voir pilotage et organisation si une eacutevolution du meacutetier a deacutejagrave eacuteteacute initialiseacutee Sinon re-

conception (adaptation au nouveau contexte) puis planification et reacuteorganisation

- augmentation des deacutelais drsquoattente des consultations dans une clinique (lrsquohocircpital voisin a

fermeacute son service drsquourgences)

Organisation Si insuffisant planification (nouveaux objectifs)

Initiation au management copy CRCF ndash J Sornet Page 23 48

ORGANISATION ET PROCESSUS

La performance de lrsquoentreprise deacutepend de son organisation et de son aptitude agrave produire

aux meilleures conditions Nous allons montrer comment organisation formelle et processus

de production peuvent contribuer agrave cette performance

1 ndash Vers lrsquooptimum

11 ndash Les eacuteconomies occidentales jusqursquoaux anneacutees 70

Jusqursquoen 1945 le principal problegraveme des entreprises eacutetait de produire des biens en quantiteacute

suffisante agrave un prix compatible avec le marcheacute Les grandes entreprises se sont multiplieacutees et

la standardisation a permis de reacuteduire les coucircts (exemple deacuteveloppement de Ford et de la

production agrave la chaicircne de 1908 agrave 1920 qui a permis une baisse du prix des voitures des 23)

On parle de laquo production pousseacutee vers le marcheacute raquo

Cette croissance de la production peu reacuteguleacutee a eacuteteacute marqueacutee par des surproductions en

1910 et 1920 puis par la crise de 1929 qui a prolongeacute ses effets jusqursquoagrave la guerre

De 1945 agrave 1975 environ (les laquo trente glorieuses raquo) la reconstruction la croissance de la

consommation de masse de nouvelles technologies et les eacutechanges internationaux

alimentent lrsquoeacuteconomie La standardisation srsquoeacutetend aux biens de consommation dont les

coucircts baissent fortement et de nouvelles reacutegulations sociales permettent une eacutevolution sans

heurt des revenus La saturation de certains marcheacutes conduit dans les anneacutees 60 agrave la

deacutemarche laquo marketing raquo et agrave la diffeacuterenciation des produits Le produit est laquo dirigeacute par le

marcheacute raquo mais les entreprises conservent une organisation assez classique et les plus grosses

srsquointernationalisent

12 ndash Lrsquoexpeacuterience japonaise et ses prolongements

Tregraves tocirct apregraves la guerre dans un Japon appauvri le constructeur automobile Toyota a ducirc

faire face agrave une restriction du marcheacute des moyens financiers et productifs et des

approvisionnements La firme a donc innoveacute dans un nouveau systegraveme de production

chassant les laquo gaspillages raquo (temps drsquoattente transports stocks deacutefauts hellip) consideacuterant que

seule la fabrication vendable creacutee de la valeur

Toyota srsquoorganise pour fabriquer la quantiteacute et la qualiteacute de produits juste neacutecessaires agrave la

satisfaction des clients la production est laquo tireacutee par le marcheacute raquo La mise en place de ce

systegraveme qui integravegre les fournisseurs ne sera acheveacutee que dans le milieu des anneacutees 70

En 1973 la hausse du peacutetrole inaugure un ralentissement de la croissance des eacuteconomies

occidentales La concurrence accrue provoque alors un inteacuterecirct pour le systegraveme deacuteveloppeacute

au Japon La production au plus juste se deacuteveloppe ainsi dans lrsquoindustrie automobile agrave partir

des anneacutees 80 et elle se reacutepand encore maintenant dans drsquoautres secteurs

Cette approche qui vise un objectif de zeacutero stock et zeacutero deacutefaut impose la maicirctrise de laquo bout

en bout raquo des processus de production et leur ameacutelioration

Initiation au management copy CRCF ndash J Sornet Page 24 48

2 ndash Organiser lrsquoentreprise

21 ndash Direction et organisation

Diriger une entreprise neacutecessite de lrsquoorganiser (de reacutepartir les tacircches) pour qursquoelle puisse

atteindre ses objectifs Lrsquoorganisation permet de satisfaire un marcheacute en tirant parti des

capaciteacutes actuelles de lrsquoentreprise tout en preacuteparant lrsquoavenir

Lrsquoorganisation reacutesulte freacutequemment drsquoun compromis entre des objectifs situeacutes agrave des niveaux

et des eacutecheacuteances diffeacuterents

Exemples

- le leader des chaises roulantes peut tirer profit de sa structure productive et de son

savoir faire pour entrer sur le marcheacute de la bicyclette eacutelectrique

- ecirctre parfaitement structureacute pour alimenter 90 du marcheacute des disquettes ne preacutepare

pas lrsquoavenir

- srsquoorganiser pour conqueacuterir le marcheacute des tire-bouchons eacutelectriques dans les deux ans

perd de son sens si cela altegravere les moyens neacutecessaires agrave la production drsquoappareils

manuels ancienne mais vitale dont la diminution agrave court terme risque de nuire agrave la

solvabiliteacute de lrsquoentreprise et de la conduire agrave la cessation de paiement

22 ndash Lrsquoorganisation fonctionnelle

La majoriteacute des entreprises adopte une laquo organisation fonctionnelle raquo (celle qui est visible

dans les organigrammes) ougrave des regroupements de personnels et drsquoeacutequipements se font

selon un modegravele hieacuterarchique (laquo line raquo) dans des uniteacutes des services ou des deacutepartements

speacutecialiseacutes Cette organisation peut se deacutecliner agrave lrsquointeacuterieur des divisions des grandes

entreprises quand elles scindent leur activiteacute par zone geacuteographique type drsquoactiviteacute

cateacutegorie de clients hellip

Remarque le terme laquo fonction raquo deacutesigne un rocircle particulier dans le fonctionnement de

lrsquoentreprise

Lrsquoorganisation fonctionnelle diffeacuterencie les activiteacutes de lrsquoentreprise en les regroupant par

meacutetier pour utiliser au mieux les compeacutetences et les moyens (meilleur rendement par la

speacutecialisation lrsquoeacutechange de compeacutetences dans une mecircme uniteacute ou gracircce agrave des eacuteconomies

drsquoeacutechelle)

23 ndash La notion de processus de production

Un processus de production se deacutefinit par la succession drsquoactiviteacutes permettant de satisfaire

un client en transformant des ressources (mateacuterielles financiegraveres humaines) en un produit

bien ou service Le processus doit creacuteer une valeur reconnue par le client

Un processus peut servir un client interne agrave lrsquoentreprise (par exemple en produisant un

composant intervenant dans plusieurs produits ou par la maintenance des machines) aussi

bien qursquoun client final On distingue usuellement

- les processus opeacuterationnels (ou maicirctres) aussi appeleacutes processus meacutetier (business process)

qui satisfont directement les clients finaux (conception et fabrication de produits vente hellip)

- les processus de support et de management (geacuterer les ressources humaines geacuterer

lrsquoinformation geacuterer les ressources financiegraveres hellip) qui ont les processus opeacuterationnels comme

clients

Toutes les actions internes agrave une organisation peuvent srsquointeacutegrer dans des processus qui

conditionnent directement ou indirectement la capaciteacute de lrsquoorganisation agrave satisfaire le

client final ou lrsquousager

Initiation au management copy CRCF ndash J Sornet Page 25 48

Aborder le fonctionnement de lrsquoentreprise par ses processus (approche processus) permet

de mettre en eacutevidence les chaicircnes drsquoactiviteacutes qui conduisent aux produits leurs

dysfonctionnements leurs coucircts la formation des deacutelais et la souplesse (la flexibiliteacute)

disponible pour satisfaire la clientegravele finale Lrsquoameacutelioration des processus a un impact visible

et direct sur chaque produit proposeacute aux clients

Lrsquoapproche processus provoque une eacutevolution de la faccedilon de travailler

- en faisant peacuteneacutetrer la laquo voix du client raquo au plus profond de lrsquoentreprise (et plus seulement

dans les services commerciaux et marketing)

- en mettant en eacutevidence des possibiliteacutes de rationalisation (par regroupement ou impartition

de certaines activiteacutes)

Remarque lrsquoapproche par les activiteacutes et les processus est agrave lrsquoorigine de la meacutethode

de deacutetermination des coucircts laquo ABC raquo - activity based costing

24 ndash Processus et fonctions

Le processus est transversal Il enchaicircne des activiteacutes qui traversent lrsquoentreprise en particulier

les services ou les deacutepartements drsquoune organisation fonctionnelle

Exemple

La division du travail par fonctions induit une charge de coordination pour assurer le

deacuteroulement du processus Elle peut geacuteneacuterer des attentes des erreurs ou des conflits drsquointeacuterecirct

(lrsquoobservation montre que des dysfonctionnements sont tregraves souvent constateacutes lors du

passage drsquoun service agrave un autre)

Organisation fonctionnelle et approche processus visent toutes deux un optimum

eacuteconomique mais leurs logiques sont diffeacuterentes

- le processus vise la satisfaction des clients (prix qualiteacute deacutelais service)

- le deacutecoupage fonctionnel cherche agrave optimiser les moyens (maximiser lrsquoeffet drsquoexpeacuterience

partager des infrastructures profiter de pocircles de compeacutetences hellip) Il apporte une ossature

hieacuterarchique stable souvent indispensable

Organisation fonctionnelle et approche processus sont donc compleacutementaires dans la

majoriteacute des cas et doivent ecirctre combineacutees judicieusement

APPLICATIONS OP

OP1 Deacutefinir flexibiliteacute systegraveme impartition

OP2 Citer huit exemples drsquoinformations essentielles pour optimiser un processus de

fabrication

Direction

Deacutepartement

commercial

(C)

Deacutepartement

administratif et

financier (AF)

Deacutepartement

Etudes (E)

Deacutepartement

Production (P)

Activiteacute

C-x Activiteacute

AF-x Activiteacute

E-x

Activiteacute

P-x

Processus x

Clie

nt

Initiation au management copy CRCF ndash J Sornet Page 26 48

OP3 Deacutegager les principes du toyotisme preacutesenteacute ci-dessous En quoi ce systegraveme est-il

initiateur de lrsquoapproche processus

Taiichi Ohno et le Toyotisme

1 - Extrait drsquoun article de Jacques BARRAUX - 1993 - LExpansion

Taiichi Ohno (1912 ndash 1990) hellip ne se prenait pas pour un visionnaire mais en imposant une

nouvelle faccedilon de produire il a reacuteinventeacute le management hellip tout le monde a entendu parler

des mots qui ont populariseacute le toyotisme dont il est le pegravere le juste-agrave-temps hellip Autant

doutils conccedilus pour lrsquoautomobile et qui ont aujourdhui une application universelle

hellip Taiichi Ohno jeune ingeacutenieur entre chez Toyota alors simple constructeur de machines

textiles Degraves 1926 apparaicirct la notion de jidoka hellip cest lart de transfeacuterer de lintelligence aux

machines pour mieux libeacuterer lintelligence des hommes Tout le contraire du taylorisme qui

juge la machine moins impreacutevisible que lhomme En 1933 Toyota se lance dans lautomobile

en sinspirant des meacutethodes ameacutericaines Mais en 1935 agrave loccasion dun voyage aux Etats-

Unis leacutetat-major de lentreprise revient fascineacute de sa visite dans un supermarcheacute La notion

de juste-agrave-temps va naicirctre de lobservation dune grande surface un lieu ougrave les clients ne

prennent que ce dont ils ont besoin et ougrave les rayons sont reacuteapprovisionneacutes pour compenser

les quantiteacutes preacuteleveacutees Ainsi le systegraveme Toyota est-il deacutejagrave dans la tecircte de ses dirigeants avant

mecircme la Seconde Guerre mondiale un demi-siegravecle avant la reacutevolution informatique et la

segmentation intensive des marcheacutes

hellip des esprits curieux comme Franccedilois Dalle en France tombent alors sous le charme des

formules et des paraboles de Taiichi Ohno En voici deux eacutechantillons

Penser agrave lenvers Cela signifie combattre les ideacutees reccedilues En lespegravece il sagit du fordisme et

du taylorisme Ohno ne croit pas agrave la planification aux effets deacutechelle et dexpeacuterience Il

propose un systegraveme industriel agrave lenvers qui permette de diversifier les produits et de les

fabriquer en petites quantiteacutes Nous ne devons plus ecirctre des paysans qui accumulent des

stocks mais des chasseurs On nimpose pas loffre On traque la demande et on la gegravere en

continu

Que les valleacutees soient hautes et les montagnes peu eacuteleveacutees Plutocirct que de concentrer tous

les efforts sur une production agrave un moment donneacute mieux vaut se doter de structures flexibles

permettant de passer agrave tout instant dune seacuterie agrave une autre Il faut eacuteviter les ruptures et les

secousses aplanir les cycles entretenir des flux reacuteguliers dactiviteacutes diversifieacutees Ce qui

implique de ne pas enfermer les hommes et les eacutequipements dans des speacutecialisations trop

eacutetroites

La flexibiliteacute le travail en groupe le refus de la dictature des machines la polyvalence et

surtout lattention constante aux signaux eacutemis par le marcheacute nappartiennent plus au

toyotisme Ces notions sont les fondements du nouvel art dorganiser de vendre et de

produire dans lindustrie comme dans les services hellip

2 - Quelques notions cleacutes

Taiichi Ohno a imagineacute la meacutethode des laquo cinq pourquoi raquo qui consiste agrave se poser cinq fois de

suite la question laquo pourquoi raquo sur le mecircme sujet de faccedilon agrave deacutecouvrir la veacuteritable cause

drsquoun problegraveme Cette meacutethode peut ecirctre appliqueacutee agrave tous les niveaux et permettre

notamment aux agents de fabrication de proposer de veacuteritables ameacuteliorations de la

production

La recherche de la qualiteacute totale (pas de deacutefaut des produits pas de rebuts pas de deacutefaut

des processus) accompagne la deacutemarche de Toyota La qualiteacute a un coucirct compenseacute par

des ventes accrues par lrsquoeacuteconomie des mesures palliatives aux deacutefauts

Initiation au management copy CRCF ndash J Sornet Page 27 48

Fiche OP1 ndash Benchmarking et processus

Le laquo benchmarking raquo consiste agrave comparer le fonctionnement de plusieurs systegravemes pour en

faire notamment ressortir les meilleures pratiques (laquo best practices raquo) Cette technique est

utiliseacutee depuis les anneacutees 80 pour ameacuteliorer la performance des entreprises Elle impose agrave

lrsquoentreprise drsquoeacutevaluer et de remettre en question ses propres modes de fonctionnement afin

de les faire eacutevoluer agrave la lueur de ce qui se fait ailleurs

Le benchmarking permet drsquoameacuteliorer les processus agrave moindre risque en fixant des objectifs

baseacutes sur des faits et donc plus facilement accepteacutes

Une classification des processus en tant que base de reacuteflexion a eacuteteacute eacutetablie aux USA par

lrsquolaquo International Benchmarking Clearinghouse raquo de lrsquoAPQC (american productivity and

quality center) en collaboration avec plusieurs dizaines drsquoentreprises

Elle se reacutesume ainsi

Le terme laquo reengineering raquo (la re-conception ou laquo reacuteingeacutenieacuterie raquo) des processus deacutesigne un

projet drsquoameacutelioration radicale des performances (de 20 agrave 50 ou plus) Il neacutecessite une

parfaite adheacutesion de la direction la constitution drsquoune petite eacutequipe de projet brillante

connaissant parfaitement les activiteacutes de lrsquoentreprise et il peut inclure un benchmarking

Le reengineering provoque geacuteneacuteralement la reacuteduction du nombre de niveaux hieacuterarchiques

(laquo delayering raquo) et lrsquoaccroissement du pouvoir de deacutecision des employeacutes (laquo empowerment raquo

ou laquo empouvoirement raquo) Bien qursquoy conduisant parfois il ne doit pas ecirctre confondu avec la

reacuteduction des activiteacutes (laquo downsizing raquo ou restructuration) et lrsquoexternalisation (laquo outsourcing raquo)

Pro

ce

ssu

s

op

eacutera

tio

nn

els

Pro

ce

ssu

s d

e m

an

ag

em

en

t e

t d

e

sup

po

rt

1 ndash

Comprendre

le marcheacute et

les clients (besoins

satisfaction)

2 ndash

Deacutevelopper

vision et

strateacutegie (contexte

concurrence)

3 ndash

Creacuteer

produits

services

processus

(concevoir

ameacuteliorer)

4 ndash

Marketing et

vente

5 ndash

Produire et

livrer (industrie

dont

ameacutelioration

processus)

6 ndash

Produire et

livrer (services)

7 ndash

Facturer et

servir les

clients (apregraves-

vente

reacuteclamations)

8 ndash Deacutevelopper et geacuterer les ressources humaines

9 ndash Geacuterer les systegravemes drsquoinformation

10 ndash Geacuterer les ressources financiegraveres et les actifs

11 ndash Appliquer un programme environnemental

12 ndash Geacuterer les relations exteacuterieures (actionnaires banques lois relations publiques hellip)

13 ndash Geacuterer lrsquoameacutelioration et le changement (eacutevaluer mesurer motiver qualiteacute totale)

Initiation au management copy CRCF ndash J Sornet Page 28 48

Fiche OP2 ndash Lrsquoorganisation par processus

Lrsquoeacutevolution drsquoune organisation aux activiteacutes reacutepeacutetitives vers lrsquoapproche processus est

geacuteneacuteralement progressive et se met en place par paliers

La mise en œuvre drsquoun veacuteritable management par processus doit ecirctre preacuteceacutedeacutee quand

lrsquoactiviteacute de lrsquoentreprise est complexe drsquoun recensement (une laquo cartographie des

processus raquo) pour mettre en eacutevidence les processus ou les familles de processus cleacutes critiques

pour le succegraves de lrsquoentreprise ougrave les efforts seront prioritaires

Des responsables de processus (laquo process owners raquo) sont ensuite deacutesigneacutes

Le responsable doit concevoir ses processus puis apregraves leur mise en œuvre assurer les

coordinations neacutecessaires les ameacuteliorer et les repreacutesenter aupregraves de la direction

Quand une structure par processus est mise en place des opeacuterateurs exeacutecutants

preacuteceacutedemment regroupeacutes dans les fonctions peuvent ecirctre affecteacutes aux processus et

drsquoanciens responsables de fonctions peuvent devenir des experts au service des processus

Lrsquoorganisation par processus peut imposer un degreacute eacuteleveacute drsquointeacutegration des activiteacutes donc

une polyvalence accrue des personnels et une reacuteduction des niveaux hieacuterarchiques

Elle neacutecessite pour le moins des compeacutetences eacutelargies au niveau des responsables de

processus (organisation administration technique hellip) dont le nombre doit rester limiteacute

(quelques dizaines au plus)

Sauf dans de tregraves petites structures lrsquoorganisation par processus se plaque geacuteneacuteralement sur

une structure plus classique

Initiation au management copy CRCF ndash J Sornet Page 29 48

ELEMENTS DE CORRIGE OP

OP1 Deacutefinir

Flexibiliteacute = adaptation au besoin (horaire variable chaicircnes robotiseacutees)

Systegraveme = ensemble organiseacute dans un but boicircte noire (sanguin nerveux meacutetrique laquo D raquo)

Impartition = sous-traitance ou externalisation (seacuteparation) drsquoactiviteacutes faire appel agrave des

partenaires plutocirct que faire soi-mecircme

OP2 Citer huit exemples drsquoinformations essentielles pour orienter lrsquooptimisation drsquoun processus

Montant des stocks (approvisionnements et produits finis)

Temps drsquoattente

Taux drsquoactiviteacute des ateliers

Rebuts

Deacutelai de production

Taux de reacuteclamations clients (qualiteacute)

Temps passeacutes en retouches finales

Turn over

Nombre drsquoarrecircts maladie

Accidents du travail

Dureacutee des arrecircts machines

OP3 Deacutegager les principes du toyotisme preacutesenteacute dans la fiche 31 En quoi ce systegraveme

repose trsquoil sur lrsquoapproche processus

Produire la quantiteacute juste neacutecessaire (agrave la demande) donc eacuteviter les stocks

Flexibiliteacute intelligence des chaicircnes de production

Qualiteacute (eacuteviter le coucirct de la non-qualiteacute)

La notion de processus est implicite ainsi que la chaicircne de valeur client

Initiation au management copy CRCF ndash J Sornet Page 30 48

DEFIS ET TENDANCES DU MANAGEMENT

Les meacutethodes de management se deacuteveloppent pour affronter le contexte eacuteconomique

Ce chapitre preacutesente les deacutefis auxquels le management contemporain doit faire face

1 ndash Lrsquoeacutevolution eacuteconomique contemporaine

A mesure que lrsquoactiviteacute eacuteconomique mondiale srsquoaccroicirct que la technologie eacutevolue les

changements sont de plus en plus rapides Ils introduisent des situations ineacutedites auxquelles les

entreprises doivent srsquoadapter en cherchant de nouvelles solutions de management Les trois

derniegraveres deacutecennies ont eacuteteacute notamment marqueacutees par les pheacutenomegravenes suivants (que nous

listons sans tenir compte des liens pouvant exister entre eux)

Pheacutenomegravene Traduction Effets

Deacute reacuteglementation

globalisation

financiegravere

titrisation

Libre circulation des capitaux accegraves

facile des particuliers au marcheacute

boursier (directement ou par

lrsquointermeacutediaire des OPCVM et SICAV)

Monteacutee en puissance du financement

des entreprises sur le marcheacute boursier

Fonds de pension

(retraites) et fonds

souverains (eacutetats)

Poids boursier important drsquoinvestisseurs

institutionnels qui cherchent un haut

rendement financier (dividendes ou

valorisation boursiegravere)

Pression sur les grandes entreprises

influence sur les strateacutegies

Mondialisation Liberteacute des eacutechanges internationaux Accroissement de la concurrence

recherche drsquoavantages eacuteconomiques

par la deacutelocalisation (biens et

services) la concentration des efforts

(recentrage) problegravemes drsquoemploi

multiplication des transports perte

drsquoinfluence des politiques

Baisse de lrsquoemploi

occidental

(notamment

industriel)

Moins de fabrications fabrications

automatiseacutees recours aux moyens

informatiques

Activiteacute reporteacutee sur le commerce la

conception et les services chocircmage

charge sociale

Restructurations Optimisation des entreprises

abaissement des coucircts augmentation

des marges recherche drsquoune taille

critique (eacuteconomies drsquoeacutechelle poids

sur le marcheacute)

Recentrages externalisations fusions

deacutelocalisations constitution de grands

groupes

NTIC (nouvelles

technologies de

lrsquoinformation et de

la communication)

Mise en œuvre des reacuteseaux (dont

internet) et drsquoapplications

informatiques communicantes

Nouvelles formes de commerce

marcheacute international deacutelocalisation

du travail intellectuel reacuteorganisation

de la distribution

Rareacutefaction relative

des matiegraveres

premiegraveres

Recherche de substituts exploration

miniegravere coucircts drsquoexploitation des

gisements accrus

Augmentation des coucircts variations

erratiques du cours des matiegraveres

deacutestabilisations politiques

Evolution

geacuteopolitique et

eacuteconomique

mondiale

Chute de lrsquoURSS transformation des

eacuteconomies collectivistes pays

eacutemergents (Chine Inde Breacutesil Russie)

()

Accroissement de la population

mondiale (4 agrave 6 7 milliards de 1970 agrave

2008)

Libeacuteralisme sans frein () nouvelles

puissances eacuteconomiques

opportuniteacutes de deacuteveloppement

nouveau partage des ressources

ineacutegaliteacutes baisse du soutien aux PVD

laquo Terrorisme raquo Actions armeacutees pression de groupes

armeacutes non gouvernementaux

Deacutestabilisations reacutegionales charge

des deacutepenses militaires

Deacuteveloppement

durable

Recherche drsquoune croissance eacutequitable

et respectueuse de lrsquoenvironnement

Pression sur les entreprises (eacutetats

associations de consommateurs

eacutecologistes ONG)

() Reacutecemment quelques affaires (Enron laquo subprimes raquo Vivendi Universal Socieacuteteacute

Geacuteneacuterale Airbus par exemple) et agrave plus grande eacutechelle la crise financiegravere de 2008 ont

montreacute les dangers drsquoune libeacuteralisation sans controcircles suffisants

Initiation au management copy CRCF ndash J Sornet Page 31 48

() Des alliances eacuteconomiques naissent entre pays eacutemergents (notamment en

ameacuterique centrale creacuteation de la Banque du Sud en 2008 par exemple) et lrsquoon

commence agrave imaginer une baisse progressive de lrsquoinfluence eacuteconomique des Etats

Unis

2 ndash Les deacutefis actuels du management

21 ndash Les grandes orientations

Lrsquoeacutevolution eacuteconomique suggegravere quelques pistes parfois concurrentes pour lrsquoaction du

manager contemporain On y retrouve au premier plan la construction drsquoune vision qui est

une composante commune du leadership

Objectif du manager

pour lrsquoentreprise

Justification Facteurs de reacuteussite

Construire une vision Eclairer lrsquoavenir de lrsquoentreprise partager

un but souder motiver

Effort de reacuteelle prospection

volontarisme de la direction

bonne communication

Reacuteactiviteacute et flexibiliteacute

(sous tous les aspects

agrave tous niveaux)

Srsquoadapter rapidement au marcheacute Bonne organisation des processus

personnel compeacutetent autonome

et motiveacute structure hieacuterarchique

alleacutegeacutee robotisation

Deacutegager des profits Reacutemuneacuterer les apporteurs de capitaux

srsquoautofinancer

Ajuster coucircts et structures

Exploiter les nouvelles

technologies

Reacuteactiviteacute ajuster coucircts et deacutelais

reacutepondre au marcheacute suivre les clients

Organiser le SI de faccedilon

pertinente eacuteviter le coucirct excessif

drsquoinvestissements trop en

laquo pointe raquo (laquo essuyer les placirctres raquo)

utiliser judicieusement les services

exteacuterieurs

Bacirctir des alliances

(contrats fusions)

Deacutevelopper une activiteacute limiter les coucircts

de transaction () atteindre la taille

critique et de meilleurs rendements se

recentrer sur une activiteacute profitable

Dominer les processus se donner

une identiteacute lisible externaliser se

doter drsquoune capaciteacute financiegravere

suffisante

Valoriser lrsquoimage Attirer les clients favoriser les alliances

donner confiance (apporteurs de fonds

employeacutes clients partenaires socieacuteteacute

civile)

Instaurer des regravegles de

gouvernance inteacutegrer le

deacuteveloppement durable

respecter lrsquoenvironnement

Geacuterer les risques Faire face aux aleacuteas eacuteconomiques et

technologiques (conjoncture politiques

accidents malveillance)

Creacuteer un systegraveme drsquoalerte geacuterer

la crise (reacuteaction raisonneacutee

sceacutenarios poursuite de

lrsquoexploitation dans un contexte

instable) mise en place de

proceacutedures drsquoapprentissage pour

ameacuteliorer les reacuteactions au fil du

temps

Geacuterer le changement Faire face agrave lrsquoeacutevolution de la demande

la pression sur les prix la variation des

performances financiegraveres la

concurrence la globalisation des

marcheacutes lrsquoeacutevolution technologique aux

fusions ou alliances aux changements

de reacuteglementation de direction hellip ()

Bonne communication pour

donner du sens au changement

et obtenir lrsquoadheacutesion du personnel

Rassembler et geacuterer les

connaissances former le

personnel

Innover Garder un avantage concurrentiel se

diffeacuterencier

Veille technologique et

commerciale investissement

Ouverture

internationale

Elargir le marcheacute saisir les opportuniteacutes Veille commerciale partenariats

() La theacuteorie des coucircts de transaction deacuteveloppeacutee par OE Williamson dans les

anneacutees 70 integravegre les coucircts lieacutes au recours au marcheacute (recherche et choix drsquoun

fournisseur neacutegociation reacutedaction de contrat suivi des eacutechanges risque de rupture

Initiation au management copy CRCF ndash J Sornet Page 32 48

drsquoapprovisionnement hellip) On peut en conclure que lrsquointeacutegration de diffeacuterentes

activiteacutes agrave lrsquoentreprise (la laquo firme raquo) preacutesente des avantages Mais des coucircts de

transaction internes doivent aussi ecirctre consideacutereacutes (preacuteparation organisation

surveillance hellip) et certaines formes de coopeacuteration continue avec les fournisseurs

permettent de reacuteduire le coucirct des transactions externes

() drsquoapregraves laquo Les meilleures pratiques de management raquo - Brilman Heacuterard ndash EO

Une eacutetude du Conference Board (2002) liste les deacutefis du management vus par 700 leaders

mondiaux Soit en reacutesumeacute avec indication du score correspondant

1 ndash Fideacuteliser les clients (42)

2 ndash Reacuteduire les coucircts (38)

3 ndash Accroicirctre flexibiliteacute et reacuteactiviteacute (29)

4 ndash Amener les employeacutes agrave adheacuterer aux valeurs et visions de lrsquoentreprise (26)

5 ndash Deacutevelopper et retenir les leaders (25)

6 ndash Geacuterer acquisitions et alliances (24)

7 ndash Accroicirctre lrsquoinnovation (20)

En fin de classement citoyenneteacute et reacuteputation (4) et ameacutelioration de la diversiteacute (3)

22 ndash Les techniques disponibles

Pour faire face aux deacutefis le manager dispose de nouveaux concepts et de nouvelles

techniques Le tableau ci-dessous en donne un reacutesumeacute et indique les domaines qursquoils

influencent principalement

Initiation au management copy CRCF ndash J Sornet Page 33 48

Principaux concepts techniques outils Incidence principale sur

Internet

- e-commerce (commerce eacutelectronique site

entreprise)

- CRM ou GRC (gestion de la relation client)

- e-procurement (gestion des approvisionnements

par le reacuteseau)

- messagerie eacutelectronique

- e-recrutement

Vente accegraves au marcheacute

Relation client reacuteactiviteacute personnalisation

fideacutelisation

Deacutelais coucircts

Communication transfert de donneacutees (piegraveces

jointes) tous domaines

Communication recrutement

Intranet reacuteseau drsquoentreprise SI

- knowledge management (gestion des

connaissances)

- e-learning (apprentissage en ligne)

- plateforme de travail collaboratif (groupware)

- workflow (circulation eacutelectronique de

documents enchaicircnement de processus)

- e-RH portail RH (libre accegraves aux postes agrave

pourvoir informations candidatures hellip)

- PGI (progiciel de gestion inteacutegreacute) ou ERP

Innovation capaciteacute au changement veille

documentaire

Formation du personnel accompagnement des

changements

Coordination communication interne

Coordination

Communication interne (voire internet en

externe) reacuteduction des coucircts climat drsquoentreprise

recrutement plans de carriegraveres hellip

Coucircts fiabiliteacute du systegraveme drsquoinformation deacutelais

processus (continuiteacute inteacutegration)

Logistique inteacutegreacutee

Supply Chain Management (SCM) gestion de la

logistique (incluant les approvisionnements)

Processus deacutelais coucircts

Externalisation

Valorisation du capital humain

GPEC (gestion preacutevisionnelle des emplois et

compeacutetences)

Coaching

Reacuteactiviteacute de lrsquoentreprise conservation des

compeacutetences rendements individuels turn-over

adaptation des compeacutetences motivation

Efficaciteacute individuelle controcircle reacutegulation

progregraves processus

Approche processus

Optimisation des processus

Deacutemarche qualiteacute totale (TQM ndash total quality

management)

Empowerment (empouvoirement)

Benchmarking reacuteingeacutenieacuterie

Coucircts marges qualiteacute deacutelais flexibiliteacute

externalisation eacutelargissement des compeacutetences

organisation

Ameacutelioration des processus (meacutetiers et supports)

Autonomie compeacutetences des employeacutes

Ameacutelioration des processus restructuration

Management par la valeur

Parties prenantes

Satisfaction des parties prenantes financement

motivation collaborations hellip

Collaboration inter organisations

Reacuteseaux drsquoentreprises alliances

EDI (eacutechange de donneacutees informatiseacutees) extranet

Impartition externalisation (outsourcing)

Coucircts recentrage investissements lancement

drsquoactiviteacute

Coucircts reacuteactiviteacute deacutelais relations avec

lrsquoadministration

Coucircts recentrage limitation des investissements

Ethique drsquoentreprise

Gouvernance drsquoentreprise (mode de direction

encadreacute par des regravegles)

Rocircle socieacutetal deacuteveloppement durable

environnement

Image de lrsquoentreprise reacutegulation du top

management relations actionnaires

Image peacutenaliteacutes et amendes objectifs

strateacutegiques

Initiation au management copy CRCF ndash J Sornet Page 34 48

23 ndash Le rocircle socieacutetal des entreprises

La responsabiliteacute socieacutetale de lrsquoentreprise (RSE) deacutesigne le rocircle qursquoelle prend dans la socieacuteteacute

au-delagrave de son activiteacute purement geacuteneacuteratrice de profit On parle aussi drsquoentreprise citoyenne

La RSE est indissociable du deacuteveloppement durable de porteacutee mondiale et dont les trois

piliers sont

- eacuteconomique (favoriser le deacuteveloppement les eacutechanges internationaux)

- social (accegraves aux soins eacuteducation conditions de travail hellip)

- environnemental (pollution preacuteservation des ressources hellip)

La RSE integravegre notamment une preacuteoccupation sociale de lrsquoentreprise vis-agrave-vis de ses salarieacutes

(seacutecuriteacute et santeacute au travail juste reacutemuneacuteration deacuteveloppement personnel hellip) Elle conduit agrave

tenir compte dans le management drsquoune vision exteacuterieure agrave lrsquoentreprise qui peut avoir des

reacutepercussions possibles sur son activiteacute eacuteconomique

Lrsquoentreprise peut aussi tirer avantage drsquoune deacutemarche responsable par la baisse de certains

coucircts (plus faibles consommations drsquoeacutenergies reacuteduction des transports hellip)

Le rocircle socieacutetal de lrsquoentreprise a eacuteteacute reconnu en France par la loi laquo NRE raquo de 2001 (loi sur les

nouvelles reacutegulations eacuteconomiques) qui oblige les socieacuteteacutes franccedilaise coteacutees sur un marcheacute

reacuteglementeacute agrave rendre compte dans leur rapport annuel de leur gestion sociale et

environnementale au travers de leur activiteacute

Article 116 de la loi Le rapport viseacute agrave larticle L 225-102 rend compte hellip laquo Il comprend

eacutegalement des informations dont la liste est fixeacutee par deacutecret en Conseil dEtat sur la

maniegravere dont la socieacuteteacute prend en compte les conseacutequences sociales et

environnementales de son activiteacute Le preacutesent alineacutea ne sapplique pas aux socieacuteteacutes

dont les titres ne sont pas admis aux neacutegociations sur un marcheacute reacuteglementeacute raquo

Une norme ISO 14000 integravegre ces preacuteoccupations et des taxes eacutecologiques sont

progressivement creacutees

3 ndash Le management par la valeur

31 ndash De lrsquoanalyse au management par la valeur

Lrsquoanalyse de la valeur est neacutee en 1947 aux Etats-Unis (General Electrics) Cette technique

consiste agrave eacutelaborer des produits conformes aux attentes de la clientegravele mais sans excegraves pour

trouver un bon compromis entre valeur pour le client et coucirct Le produit optimal est deacutefini agrave

partir drsquoenquecirctes qui deacuteterminent le besoin client (ou plutocirct drsquoun client laquo type raquo)

Exemple il est inutile de concevoir un petit veacutehicule citadin capable de parcourir

500 000 km sans avarie compte tenu des effets de mode et du faible kilomeacutetrage

annuel Par contre le marcheacute peut exiger un fonctionnement sans faille sur 150 000 km

soit dix ans en moyenne ce qui conditionne les coucircts de production

Cette recherche drsquoun ajustement de valeur au besoin des clients eacutetait un preacutecurseur du

management par la valeur qui recherche plus largement la creacuteation de valeur pour

chacune des parties prenantes de lrsquoentreprise tout en lui meacutenageant un reacutesultat suffisant

Plus geacuteneacuteralement le management par la valeur est deacutefini par une norme europeacuteenne (EN

12973)

Le management par la valeur est un style de management particuliegraverement destineacute agrave

mobiliser les individus agrave deacutevelopper les compeacutetences et agrave promouvoir les synergies et

Initiation au management copy CRCF ndash J Sornet Page 35 48

linnovation avec pour objectif la maximisation de la performance globale dun

organisme Le management par la valeur apporte une nouvelle faccedilon dutiliser nombre

de meacutethodes de management existantes Il est en coheacuterence avec le Management

de la qualiteacute

Cette approche du management pose de nombreuses questions notamment quelles

prioriteacutes et quelles valeurs attribuer aux parties prenantes comment appreacutehender la

perception par les parties prenantes de la valeur qui leur est affecteacutee

32 ndash La valeur client

Le processus drsquoeacutelaboration drsquoun produit qui consomme des ressources coucircteuses doit creacuteer

une valeur suffisante pour provoquer lrsquoachat par le client final La production drsquoune valeur

reconnue par le client est vitale pour lrsquoentreprise mais sa deacutetermination est parfois complexe

La valeur du produit perccedilue par le client integravegre des eacuteleacutements en partie subjectifs

- une valeur drsquousage (le produit reacutepond agrave un besoin)

- une valeur drsquoestime (lrsquoimage apporteacutee par le produit un aspect affectif)

- une valeur drsquoeacutechange (deacuteduite de lrsquoespoir de revente du produit)

Valeurs drsquousage drsquoestime et drsquoeacutechange deacutependent implicitement de la qualiteacute (un bien peu

fiable est impropre agrave lrsquousage attendu de mauvaise qualiteacute notoire il nrsquoapporte pas une

image positive et ses deacutefauts connus nuisent agrave sa revente) Une eacutevaluation de la qualiteacute

intervient donc dans la valeur perccedilue du produit

Par ailleurs le client considegravere le coucirct drsquoobtention du produit (les charges qursquoil doit supporter

pour acqueacuterir le produit lrsquoeffort qursquoil doit faire pour trouver le produit et les frais de mise agrave

disposition)

Le prix perccedilu par le client est geacuteneacuteralement supeacuterieur au prix de vente

Le client achegravete theacuteoriquement le produit qui preacutesente la diffeacuterence valeur perccedilue ndash prix

perccedilu la plus favorable ou le meilleur rapport prix perccedilu qualiteacute perccedilue et dans certains

cas celui qui a le prix produit le plus bas

Remarque les valeurs du scheacutema ci-dessus changent durant le cycle de vie du produit

(un nouveau produit peut avoir une valeur perccedilue plus eacuteleveacutee qursquoen fin de vie) La

valeur client ne peut ecirctre eacutevalueacutee que par enquecirctes et ne peut donc ecirctre deacutefinie avec

certitude

La notion de laquo satisfaction client raquo conseacutecutive agrave une vente influence aussi le prix produit et

le prix perccedilu

- lrsquoentreprise gagne sur les coucircts de recherche de clientegravele

- le client nrsquoa pas agrave rechercher un nouveau fournisseur et beacuteneacuteficie drsquoun coucirct drsquoobtention

plus bas

valeur perccedilue client

prix perccedilu client

coucirct produit Marge (valeur creacuteeacutee pour

lrsquoentreprise)

euros

prix produit

Valeur creacuteeacutee

pour le client

Initiation au management copy CRCF ndash J Sornet Page 36 48

La satisfaction du client deacutepend de facteurs qualitatifs aussi divers que la fiabiliteacute du produit

la vitesse de reacuteaction du fournisseur lrsquoattitude des commerciaux lrsquoefficaciteacute du service

apregraves-vente la netteteacute des contrats ou la justesse de la facture

Valeur perccedilue coucirct marge et satisfaction reacutesultent de processus allant de la conception du

produit jusqursquoagrave sa livraison et son apregraves-vente La deacutemarche laquo processus raquo et lrsquolaquo analyse de la

valeur raquo en forccedilant la recherche de solutions efficientes agrave tout niveau administratif

technique commercial et apregraves-vente sont donc neacutecessaires pour bien positionner

lrsquoentreprise sur son marcheacute

Pour autant le risque commercial ne peut jamais ecirctre annuleacute et lrsquooffre de lrsquoentreprise ne

satisfait geacuteneacuteralement pas en milieu concurrentiel tous ses clients potentiels

33 - La creacuteation de valeur pour les autres parties prenantes

Les salarieacutes

La creacuteation drsquoune valeur suffisante pour les salarieacutes est reconnue comme neacutecessaire car des

observations montrent que la satisfaction des clients en deacutepend Moins souvent eacutevoqueacutee en

peacuteriode de chocircmage elle nrsquoest prioritaire que pour les employeacutes dont lrsquoentreprise souhaite

conserver les compeacutetences

La laquo valeur salarieacute raquo ne comprend pas que le salaire Le sentiment drsquoappartenance agrave un

groupe la reconnaissance lrsquoaccomplissement de soi et la construction professionnelle en

sont des eacuteleacutements importants Comme pour les clients on doit ainsi distinguer la reacutetribution

perccedilue du salaire objectif

Les actionnaires

Lrsquoactionnaire apporte des fonds propres agrave lrsquoentreprise en contrepartie de titres parfois

neacutegociables en bourse et assortis drsquoun droit de vote en assembleacutee geacuteneacuterale La valeur

attribueacutee aux actionnaires est servie en termes moneacutetaires (dividende ou augmentation de la

valeur du titre neacutegociable)

Remarque des facteurs non moneacutetaires comme lrsquoimage de lrsquoentreprise qui deacutepend

en partie de sa communication peuvent influencer la deacutecision drsquoachat de vente ou

de conservation des titres par lrsquoactionnaire

Reacutetribution perccedilue euros

Salaire objectif

Avantage non

moneacutetaire de

lrsquoemploi

Initiation au management copy CRCF ndash J Sornet Page 37 48

Compte tenu de lrsquoimportance croissante de lrsquoactionnariat dans le financement des grandes

entreprises coteacutees en bourse et notamment des investisseurs institutionnels comme les fonds

de pension des indicateurs speacutecifiques ont eacuteteacute introduits pour appreacutecier la performance des

entreprises vue par les actionnaires Par exemple la valeur ajouteacutee eacuteconomique (EVA reg

economic value added marque deacuteposeacutee de Stern Stewart ou VAE ndash valeur ajouteacutee

eacuteconomique parfois deacutenommeacutee VEC ndash valeur eacuteconomique creacuteeacutee) qui prend en compte le

coucirct du capital

LrsquoEVA correspond tregraves scheacutematiquement au calcul suivant

EVA = (PO) profit opeacuterationnel ndash (C) coucirct du capital X (CE) capitaux employeacutes

LrsquoEVA neacutecessite en pratique des retraitements assez complexes Le PO peut se deacuteterminer

selon les principes suivants

- PO = reacutesultat drsquoexploitation (avant inteacuterecircts) ndash impocirct

- PO = beacuteneacutefice courant (tenant compte des inteacuterecircts) + inteacuterecircts ndash eacuteconomie drsquoimpocirct sur les

inteacuterecircts (on exclue les eacuteleacutements financiers et lrsquoimpocirct correspondant) ndash impocirct

- lrsquoimpocirct pris en compte correspond au profit opeacuterationnel consideacutereacute (dans les cas courants agrave

13 du PO)

C = taux moyen de reacutemuneacuteration du capital (reacutesultant par exemple du dividende exigeacute de

certains investisseurs et des taux drsquoemprunts bancaires)

CE = capitaux propres et dettes portant inteacuterecirct

Remarque le profit opeacuterationnel ou reacutesultat opeacuterationnel correspond au NOPAT ndash net

operating profit after tax - anglo-saxon LrsquoEVA est eacutegale au NOPAT diminueacute de la

reacutemuneacuteration des capitaux

Exemple lrsquoentreprise X dispose drsquoun capital de 2 500 000 euro et reacutealise un beacuteneacutefice net

drsquoimpocirct de 450 000 euro (taux 33 13) Un dividende de 6 doit ecirctre verseacute aux

actionnaires et la banque lui a accordeacute un precirct de 1 200 000 euro agrave 4 Les autres

constituants des reacutesultats financier et exceptionnel sont neacutegligeables

Reacutesultat opeacuterationnel = 450 000 + 004 x 1 200 000 x 23 = 482 000 euro

Coucirct du capital = 006 x 2 500 000 + 004 x 1 200 000 x 23 = 182 000 euro

EVA = 300 000 euro

Coucirct moyen pondeacutereacute du capital (C) = (004 x 1 200 000 x23 + 006 x 2 500 000)

3 700 000 Soit 492

Si lrsquoEVA est positive lrsquoentreprise creacuteeacutee de la valeur apregraves reacutemuneacuteration des capitaux et sa

valeur boursiegravere doit augmenter

Lrsquoutilisation de lrsquoEVA comme indicateur influence le management de lrsquoentreprise car il y a

trois moyens pratiques drsquoaugmenter lrsquoEVA

- augmenter le reacutesultat opeacuterationnel

- lancer des investissements ayant une rentabiliteacute supeacuterieure agrave C

- eacuteliminer les activiteacutes ayant une rentabiliteacute infeacuterieure agrave C

Remarque lrsquoutilisation sans nuance de lrsquoEVA comme critegravere de management peut

poser problegraveme Le calcul de lrsquoEVA repose sur des ajustements comptables il est donc

sujet agrave manipulations (provisions capitalisation ou non de la RD hellip) Par ailleurs le

critegravere laquo EVA raquo pris isoleacutement peut conduire agrave chercher la rentabiliteacute agrave court terme agrave

reacuteduire les investissements prospectifs et donc nuire agrave terme au deacuteveloppement de

lrsquoentreprise

Initiation au management copy CRCF ndash J Sornet Page 38 48

Les fournisseurs reccediloivent le paiement de leurs factures plus ou moins rapidement (le deacutelai

de paiement repreacutesente une valeur consentie au fournisseur)

Lrsquoentreprise peut accroicirctre la valeur apporteacutee agrave ses fournisseurs par des actions cibleacutees

comme une contribution agrave la formation de leurs personnels certains transferts de

technologie ou de savoir faire agrave des sous-traitants une coopeacuteration suivie favorisant leur

deacuteveloppement lrsquointeacutegration agrave des campagnes de promotion

A noter que la valeur consentie aux fournisseurs peut avoir une influence sur la qualiteacute et les

deacutelais de livraison des produits

La collectiviteacute reccediloit des taxes et parfois des prestations en nature par deacutefaut ou explicites

(effort de preacuteservation de lrsquoenvironnement ameacutenagement du territoire par les implantations

aide mateacuterielle agrave des projets participation agrave la formation par exemple)

APPLICATIONS DT

DT1 Deacutefinir expliquer deacutereacuteglementation socieacutetal eacuteconomies drsquoeacutechelle coaching EDI

gouvernance

DT2 Deacuteterminer en quoi la deacutemarche TQM srsquoinscrit dans les deacutefis actuels du management

DT3 Apregraves avoir consulteacute les documents ci-dessous extraits du site drsquoAir France

(httpdeveloppement-

durableairfrancecomFRfrlocaldemarcheN4_positionnement_pphtm)

exposer les enjeux et les limites de la RSE et de la gestion des parties prenantes

Initiation au management copy CRCF ndash J Sornet Page 39 48

Dialogue avec les parties prenantes

Initiation au management copy CRCF ndash J Sornet Page 40 48

Attentes des parties prenantes

Initiation au management copy CRCF ndash J Sornet Page 41 48

Creacuteation de valeur pour les parties prenantes

La creacuteation de valeur pour les parties prenantes est au cœur de la strateacutegie du Groupe Le scheacutema de

distribution financiegravere ci-dessous donne un aperccedilu de la distribution des recettes du Groupe aux

diffeacuterentes parties prenantes actionnaires collaborateurs fournisseurs pouvoirs publics

collectiviteacutes locales etc

Initiation au management copy CRCF ndash J Sornet Page 42 48

Fiche DT1 ndash Extrait du sommaire de laquo Problegravemes eacuteconomiques raquo No 2894

La gestion des entreprises bouleverseacutee par les technologies de linternet

Reacutealiteacutes industrielles - Annales des Mines Jean-Michel Yolin

Avec lavegravenement de linternet les processus de conception de production et de vente sont

radicalement remis en cause Quel que soit le secteur dactiviteacute les technologies de linternet

permettent en effet de reacuteduire les deacutelais et de passer dun processus discontinu agrave un processus

continu Lorganisation des entreprises et leur mode de gestion en sont profondeacutement bouleverseacutes

tant au niveau individuel que collectif Linternet rend ainsi possible la reacutealisation dobjectifs que les

entreprises cherchaient agrave atteindre depuis longtemps sans y parvenir meilleure eacutecoute du client

travail sans stocks en flux tendu hieacuterarchies plates autorisant une grande reacuteactiviteacute flexibiliteacute dans

lorganisation et loutil de production acceacuteleacuteration du renouvellement des produits entreprises en

reacuteseau ougrave chacune se recentre sur son cœur de meacutetier etc

Le laquo knowledge management raquo ou comment geacuterer les connaissances

Document de travail du LAMSADE - Michel Grundstein

Peter Drucker lavait preacutedit le capital immateacuteriel eacutetait voueacute agrave devenir un facteur de compeacutetitiviteacute

pour lentreprise La libeacuteralisation des eacutechanges acceacutelegravere les processus de deacutecision de lentreprise

et implique que lassimilation des informations soit agrave la fois de meilleure qualiteacute et plus rapide Ainsi

la fonction qui consiste agrave manager les connaissances au sein de lentreprise savegravere primordiale

Bien que la prise de conscience de limportance du capital immateacuteriel ait eacuteteacute tardive - le concept

de knowledge management est apparu en France aux Etats-Unis et au Japon au milieu des

anneacutees 1990 - agrave lheure actuelle lorganisation de leacutechange dinformations et le partage des

connaissances sont devenus des facteurs cleacutes dune gestion performante de lentreprise Ils

doivent sinscrire dans un projet global destineacute agrave mettre en valeur les savoirs et les savoir-faire

individuels et collectifs

Les leccedilons du laquo coaching raquo pour le management de la qualiteacute

Humanisme et Entreprise - Martine Brasseur

Parmi les nouvelles formes de management en vogue dans les entreprises le coaching figure en

bonne place Appliqueacute au management de la qualiteacute il sagit dune pratique

daccompagnement destineacutee agrave initier et agrave faciliter le processus de deacuteveloppement dun individu

La deacutemarche consiste agrave affirmer que tout individu est en quecircte de qualiteacute agrave condition toutefois

de ne pas lui imposer des contraintes lempecircchant de progresser On considegravere notamment les

erreurs comme potentiellement feacutecondes En deacutefinitive le coach donne au coacheacute la permission

de reacuteussir en lui donnant aussi la permission deacutechouer

Initiation au management copy CRCF ndash J Sornet Page 43 48

Fiche DT2 ndash Management strateacutegique les sept deacutefis agrave relever dici agrave 2016

Extrait drsquoun article du site wwwlentreprisecom -Sabine Blanc - Mis en ligne le 20032007

(httpwwwlentreprisecom325article11977html)

Une eacutetude anglaise publieacutee par lopeacuterateur Orange Grande-Bretagne deacutecrypte la mutation

des formes de travail et les enjeux majeurs pour les entreprises de demain afin decirctre au top

de la compeacutetitiviteacute Voici les challenges-cleacutes pour les managers qui veulent rester dans la

course hellip

1 - Future organisation du travail les quatre laquo mondes raquo possibles

La reacutealiteacute sera probablement un meacutelange de ces quatre sceacutenarios souligne lrsquoeacutetude

Les mondes mutuels Tout se passe dans le cadre des communauteacutes locales vie priveacutee

comme professionnelle Le modegravele coopeacuteratif preacutevaut au lieu du laquo big business raquo Oublieacutes

aussi dans ce systegraveme les trajets pour aller au bureau les gens preacutefegravereront travailler dans de

petites entreprises locales souvent connecteacutees au reacuteseau drsquoautres structures similaires

Les laquo reacutepondants raquo (en anglais laquo replicants raquo) La figure du consultant freelance deviendra

dominante tandis que celle du salarieacute deacuteclinera Il ne sera pas rare de travailler pour plusieurs

entreprises On perdra en seacutecuriteacute de lrsquoemploi en visibiliteacute et en routine ce que lrsquoon gagnera

en liberteacute La majeure partie des tacircches srsquoeffectuera chez soi avec la possibiliteacute de srsquoinstaller

temporairement dans les bureaux de son client du moment Dans un contexte dincertitude

sur lrsquoavenir les travailleurs alterneront peacuteriodes drsquoactiviteacute intense et repos Ce sera agrave eux

drsquoaller vers les entreprises et non lrsquoinverse mecircme si celles-ci devront veiller agrave rester attractives

Les cottages eacutelectroniques Comme ce nom le suggegravere le teacuteleacutetravail deviendrait la norme

univers priveacute et professionnel se confondant Plus besoin de subir une heure de transport les

salarieacutes se logueront de chez eux sur le reacuteseau de lrsquoentreprise Les reacuteunions se tiendront dans

de petits bureaux centraux situeacutes agrave courte distance La flexibiliteacute du temps de travail srsquoimpose

Les salarieacutes disposeront de plus de marge de liberteacute dans leur activiteacute

Les disciples de la nueacutee Cette appellation poeacutetique cache simplement une extension de

lrsquoorganisation actuelle des grandes entreprises avec des salarieacutes se rendant sur un lieu de

travail centraliseacute Le rocircle croissant des technologies de lrsquoinformation multipliera les faccedilons de

collaborer et accroicirctra lrsquoefficaciteacute Le controcircle du travail sera omnipreacutesent La frontiegravere entre

travail et vie priveacutee restera marqueacutee

2 - Sept deacutefis pour les entreprises et leur managers

Quoi qursquoil advienne les entreprises et leurs dirigeants devront concentrer leurs efforts sur sept

points-cleacutes pour srsquoadapter Voici quelques exemples de probleacutematiques souleveacutees par le

rapport et des pistes de solution

Le leadership Les managers devront entre autres savoir persuader et influencer des

travailleurs beaucoup plus indeacutependants Ils auront aussi agrave repenser les niveaux auxquels

prendre les deacutecisions strateacutegiques en haut ou au contraire agrave des degreacutes moins eacuteleveacutes de la

pyramide hieacuterarchique

gt Faire du management une force facilitant les activiteacutes transversales plutocirct que la reacuteduire agrave

la seule fonction de deacutecision

La culture drsquoentreprise Davantage de salarieacutes capables de reacutefleacutechir seront neacutecessaires

tandis que les tacircches qui peuvent ecirctre automatiseacutees ou scripteacutees diminueront Un des

enjeux creacuteer une culture agrave mecircme drsquoattirer et drsquoencourager les personnes preacutesentant ces

qualiteacutes de reacuteflexion requises dans un contexte de compeacutetition accrue et de plus grande

indeacutependance des travailleurs

Initiation au management copy CRCF ndash J Sornet Page 44 48

gt Passer si neacutecessaire drsquoune culture drsquoentreprise forte agrave un mode drsquoengagement plus

consensuel moins rebutant

La marque Conseacutequence du recours croissant agrave lrsquo laquo outsourcing raquo lrsquoimage drsquoune marque

deacutependra plus drsquoagents exteacuterieurs qui ne fonctionnent pas forceacutement selon le mecircme mode

drsquoorganisation Comment garder le controcircle dessus

gt Choisir le mode qui corresponde le plus agrave vos valeurs et preacutevoir un programme de risk

management qui mette en eacutevidence ougrave les conflits sont susceptibles de jaillir

Lrsquoinnovation Plus que jamais il faudra faire face agrave une acceacuteleacuteration du rythme de

lrsquoinnovation en proposant constamment des solutions adapteacutees

gt Tisser des partenariats strateacutegiques avec drsquoautres entreprises pour partager les coucircts et les

fruits de lrsquoinnovation

Le deacutefi opeacuterationnel et technologique De quelle faccedilon controcircler lrsquoinformation crsquoest-agrave-dire

faire en sorte que les bonnes personnes accegravedent facilement agrave une information toujours en

phase tout en maintenant la seacutecuriteacute

gt Recourir agrave des laquo feuilles de route des futurs raquo syntheacutetisant en une page les indicateurs

sociaux et de consommation ainsi que les eacutevolutions technologiques et leacutegislatives qui

influent sur les changements et indiquant comment ils modifient vos marcheacutes vos clients et

votre organisation

La qualiteacute Si de nouveaux proceacutedeacutes ont pu deacutegrader la qualiteacute comme le recours agrave des

centres drsquoappel externaliseacutes drsquoautres ideacutees se sont reacuteveacuteleacutees plus prometteuses comme en

teacutemoigne le succegraves de certaines compagnies aeacuteriennes low cost Elles ont su conjuguer prix

serreacutes et services eacuteleveacutes ce qui devra devenir la norme estime lrsquoeacutetude

gt Continuer de rechercher la qualiteacute Elaborez aussi une bonne prestation service qui inclut

une livraison de qualiteacute voire creacuteez-la en partenariat avec les consommateurs

La leacutegislation La question de la proprieacuteteacute intellectuelle pourrait ecirctre probleacutematique Elle est

deacutejagrave source de conflits comme en teacutemoigne le procegraves pour violation de brevet intenteacute agrave RIM

le fabricant canadien du Blackberry par NTP Que pourra-t-on et que faudra-t-il proteacuteger par

un brevet Il sera eacutegalement neacutecessaire drsquoadapter la leacutegislation aux nouveaux modes

drsquoorganisation

gt Collaborer avec les acteurs du mecircme secteur et les leacutegislateurs pour deacutevelopper les

modegraveles des lieux de travail du futur et bacirctir le droit le plus adeacutequat

Orange a-t-il vu juste dans ses preacutevisions Rendez-vous dans neuf ans pour la reacuteponsehellip

Initiation au management copy CRCF ndash J Sornet Page 45 48

Fiche DT3 ndash Le management par la qualiteacute totale

Extrait drsquoune lettre drsquoinformation du cabinet Baud Accordance Consulting AD2 consultants ndash

2002

1 - Le TQM (Total Quality Management) offre pour lentreprise une vision de la qualiteacute plus

large et transversale

Son principe est simple La finaliteacute de lEntreprise est de deacutevelopper la satisfaction de ses

clients tout en eacutetant beacuteneacuteficiaire cest agrave dire pas agrave nimporte quel prix Elle doit ameacuteliorer sa

rentabiliteacute au travers de la deacutemarche qualiteacute La Qualiteacute Totale vise agrave fournir aux clients

externes et internes une reacuteponse adeacutequate agrave leurs attentes dans le meilleur rapport qualiteacute

prix la meilleure efficience

Elle considegravere pour cela lensemble des processus de lentreprise ayant une incidence sur la

qualiteacute et la satisfaction des clients

Le TQM fait ainsi une large place agrave

la deacutefinition et la planification de la strateacutegie geacuteneacuterale

la coheacuterence de la politique qualiteacute avec la strateacutegie

la deacutemultiplication de la politique qualiteacute dans toutes les directions de lentreprise

la relation client fournisseur interne

la prise en compte de lenvironnement concurrentiel

la consideacuteration de lensemble des risques potentiels financiers sociaux concurrentielshellip

limplication et la motivation du personnel

lanalyse des besoins des clients et le positionnement marketing

la maicirctrise des processus transverses internes

les reacutesultats sous tous ses aspects y compris financiers commerciaux image

De nombreux reacutefeacuterentiels sont relatifs agrave la Qualiteacute Totale hellip Tous ces reacutefeacuterentiels imposent un

questionnement plus profond et indiscret sur le mode de fonctionnement de lentreprise et

son management

helliphellip

2 - LISO 9001 2000 au travers du deacuteploiement des processus (management supports

reacutealisation et ameacutelioration continue) reacutepond quelque peu agrave la mecircme logique

LISO est une ouverture indeacuteniable vers la logique du TQM mais ne se reacutefegravere pas agrave la notion

defficience

Les dirigeants sont cependant sensibles agrave la neacutecessaire reacuteduction des coucircts de non-qualiteacute

et dobtention de la qualiteacute agrave la rentabiliteacute du systegraveme de management de la qualiteacute

mais ne perccediloivent pas toujours la qualiteacute comme une deacutemarche globale

Les deacutemarches qualiteacute commencent bien souvent par la remise en cause de lorganisation

leacutevaluation critique de son efficaciteacute lexamen des processus et la mise en eacutevidence des

lourdeurs administratives

La qualiteacute devient laffaire de tous hellip

Initiation au management copy CRCF ndash J Sornet Page 46 48

Fiche DT4 ndash Le deacuteveloppement durable et la RSE

Extrait du site wwwvigeocom

(httpwwwvigeocomcsr-rating-agencyfrmethodologiecriteres-de-recherche37-

criteres-d-analysehtml)

Deacuteveloppement durable laquo un deacuteveloppement qui reacutepond aux besoins du preacutesent sans compromettre

la capaciteacute des geacuteneacuterations futures de reacutepondre aux leurs raquo (Commission mondiale sur lrsquoenvironnement

et le deacuteveloppement ndash 1987)

Reacutefeacuterentiel drsquoeacutevaluation des entreprises par le groupe Vigeacuteo (le groupe mesure les performances et le

niveau de maicirctrise des risques de responsabiliteacute sociale des entreprises et des organisations - site

wwwvigeocom)

1 Ressources Humaines Ameacutelioration continue des relations professionnelles des relations drsquoemploi et des conditions de travail 2 Droits humains sur les lieux de travail Respect de la liberteacute syndicale et promotion de la neacutegociation collective non discrimination et promotion de lrsquoeacutegaliteacute eacutelimination des formes de travail proscrites (enfants travail forceacute) preacutevention des traitements inhumains ou deacutegradants de type harcegravelements sexuels protection de la vie priveacutee et des donneacutees personnelles 3 Environnement Protection sauvegarde preacutevention des atteintes agrave lenvironnement mise en place drsquoune strateacutegie manageacuteriale approprieacutee eacuteco conception protection de la biodiversiteacute et maicirctrise rationnelle des impacts environnementaux sur lrsquoensemble du cycle de vie des produits ou services

4 Comportements sur les marcheacutes Prise en compte des droits et inteacuterecircts des clients inteacutegration de standards sociaux et environnementaux dans la seacutelection des fournisseurs et sur lrsquoensemble de la chaicircne drsquoapprovisionnement preacutevention effective de la corruption respect des regravegles concurrentielles 5 Gouvernement drsquoentreprise Efficience et probiteacute assurance de lrsquoindeacutependance et de lrsquoefficaciteacute du Conseil drsquoadministration effectiviteacute et efficience des meacutecanismes drsquoaudit et de controcircle et notamment inclusion des risques de responsabiliteacute sociale respect des droits des actionnaires et notamment des minoritaires transparence et rationaliteacute de la reacutemuneacuteration des dirigeants 6 Engagement socieacutetal Effectiviteacute inteacutegration manageacuteriale de lrsquoengagement contribution au deacuteveloppement eacuteconomique et social des territoires drsquoimplantation et de leurs communauteacutes humaines engagements concrets en faveur de la maicirctrise des impacts socieacutetaux des produits et des services contribution transparente et participative agrave des causes drsquointeacuterecirct geacuteneacuteral

Initiation au management copy CRCF ndash J Sornet Page 47 48

ELEMENTS DE CORRIGE DT DT1 Deacutefinir expliquer

Deacutereacuteglementation = suppression des contraintes eacuteconomiques (libre eacutechange des biens et

capitaux)

Socieacutetal = qui se rapporte agrave la structure agrave lrsquoorganisation ou au fonctionnement de la socieacuteteacute

Economies drsquoeacutechelle = reacuteduction des coucircts lieacutee au niveau drsquoactiviteacute (amortissement des

charges fixes)

Coaching = accompagnement de personnes ou deacutequipes pour le deacuteveloppement de leurs

potentiels

EDI = eacutechange de donneacutees informatiseacutees ET standardiseacutees (ex SWIFT bancaire edifact

documents deacuteclaratifs)

Gouvernance = exercice du pouvoir la bonne gouvernance est participative et eacutequitable

conforme agrave lrsquointeacuterecirct commun

DT2 Deacuteterminer en quoi la deacutemarche TQM srsquoinscrit dans les deacutefis actuels du management

Voir notamment fiche 43

Maicirctrise des processus reacuteduction des coucircts reacuteactiviteacute et satisfaction de la clientegravele = faire

face agrave la concurrence

Ameacutelioration de lrsquoimage motivation du personnel

DT3 Apregraves avoir consulteacute les documents ci-dessous extraits du site drsquoAir France

(httpdeveloppement-

durableairfrancecomFRfrlocaldemarcheN4_positionnement_pphtm)

exposer les enjeux et les limites de la RSE et de la gestion des parties prenantes

Trame geacuteneacuterale possible

Introduction

Les deacutefis contemporains (accroissement de la concurrence devenue mondiale recherche

de nouveaux avantages concurrentiels pression de la socieacuteteacute besoin drsquoimage et de projet

lisible pour mener lrsquoentreprise crise et scandales du libeacuteralisme hellip) RSE et PP

Deacuteveloppement (voir cours)

1 ndash Parties prenantes et management par la valeur

PP deacutefinir citer reacutesumer lrsquoavantage rechercheacute (fideacuteliser motiver recherche drsquoalliances

implicites)

PP moyens (dont exemples AF) et meacutethode de management par la valeur (reacutepartie)

2 ndash La responsabiliteacute socieacutetale de lrsquoentreprise

RSE 3 axes

- eacuteconomique (favoriser le deacuteveloppement les eacutechanges internationaux)

- social (accegraves aux soins eacuteducation conditions de travail hellip)

- environnemental (pollution preacuteservation des ressources hellip)

RSE gouvernance drsquoentreprise facteur drsquoimage inteacutegrable dans la deacutemarche PP

Article 116 de la loi Le rapport viseacute agrave larticle L 225-102 rend compte hellip laquo Il comprend

eacutegalement des informations dont la liste est fixeacutee par deacutecret en Conseil dEtat sur la maniegravere

dont la socieacuteteacute prend en compte les conseacutequences sociales et environnementales de son

activiteacute Le preacutesent alineacutea ne sapplique pas aux socieacuteteacutes dont les titres ne sont pas admis aux

neacutegociations sur un marcheacute reacuteglementeacute raquo

Initiation au management copy CRCF ndash J Sornet Page 48 48

RSE exemple AF (ONG fournisseurs)

3 ndash Liens entre PP et RSE

- la RSE introduit de nouvelles PP

- la RSE suppose le respect des PP usuelles (employeacutes clients notamment)

4 - Probleacutematique

- deacutefinir la valeur reacuteellement apporteacutee par une gestion des PP (confusion salaire ndash valeur

idem impocircts hellip ex laquo valeur ajouteacutee raquo)

- communication (neacutecessaire mais aller au-delagrave)

- marginaliteacute des deacutepenses RSE (efficaciteacute sinceacuteriteacute de lrsquoengagement marge de manœuvre)

- charge RSE reporteacutee sur des tiers (ex fournisseurs AF)

- inteacutegration de facteurs non visibles en comptabiliteacute (pertes drsquoemploi nuisances hellip)

Conclusion

Voies incontournables mais pouvant nrsquoavoir qursquoun effet superficiel et temporaire Voir utiliteacute

drsquoaccompagnement leacutegislatif de regravegles de gouvernance